You are on page 1of 2356

Statics Worked Examples With Integrated Text and Graphics

Mark P. Rossow
Southern Illinois University Edwardsville
2009 Mark P. Rossow
ii
The traditional way to learn in a problem-solving course such
as statics is to solve a large number of homework problems.
This procedure is often inefficient, time-consuming and
frustrating because so much time is spent searching for the
solution that little time is left for learning the principles that
will enable a student to solve other related problems. Recent
educational research has suggested that a way to learn that is
superior to simply solving problems is to study worked
examples. However, the research has also shown that the
superiority of studying worked examples is maintained only
if the text and graphics appearing in the examples are
"integrated," by which is meant that the text on the page is
positioned immediately adjacent to the figure to which the
text refers. Additionally, liberal use is made of arrows
relating specific words or labels to appropriate features of the
figure, and the sequence of steps in the solution procedure is
identified by circled numbers.
The present book, which is intended as a supplement to a
course in statics, contains 445 worked examples in which
text and graphics are integrated. The range of topics covered
by the examples is sufficiently broad to encompass most
statics courses. Each topic section, except the introductory
section, begins with a Procedures and Strategies discussion
that outlines techniques for problem solving. Next are given
problem statements but not the solutions for all the
examples for that topic.
Why Integrated Text and Graphics?
The worked examples then follow the problem statements.
The solutions, as has already been mentioned, contain
integrated text and graphics, but in addition provide much
more detail and explanation than are found in examples in
most conventional textbooks.
The purpose of providing problem statements separated from
the solutions is to allow students, after they have studied
some of the examples in a particular topic section and wish
to test their understanding, to attempt to work other problems
without inadvertently seeing part of the solution. This
approach to learning is discussed further in the next section,
"How to Study Worked Examples."
A final note: although considerable effort has been expended
in checking and proofreading the examples presented herein,
in a work of this size undoubtedly some errors remain.
Readers who find errors are encouraged to report them to the
author at markrossow@gmail.com.

iii
You might think of the present book as an unusually detailed
and easy-to-read solution manual designed for students, who
are learning the subject matter, rather than for faculty
members, who already know the subject matter and are only
looking for numerical details of particular solutions. The
book does indeed provide numerical details of solutions, but
more importantly it also provides the rationale behind the
solution steps so that you can understand the principles and
solution techniques in the field of statics.
Here is how to make the best use of the book: When you
begin a topic, for example, "equilibrium of a particle," read
the first several examples. Then, when you think that you
understand the solution procedure, go on to the later
examples and try to work them. Look at the book's solution
only when you encounter a difficulty. In this way, your
mind will be focused on a specific question that you want
answered, and you will be motivated to study the example
solution much more closely than if you were simply to read
the example without first having attempted to work it. Once
you understand the solution thoroughly, summarize to
yourself the general principle that the example illustrates.
The way not to study worked examples is simply to read
them straight through without ever challenging yourself to
work an example on your own or without ever pausing to
think and crystallize in your mind the general principles
A Note to Students: How to Study Worked Examples
that the examples embody. If you read but do not study, you
may fall into the trap that educational psychologists refer to
as the "illusion of understanding": you think that you
understand but in fact you do not. Worked examples have
the considerable advantage over the traditional approach of
working homework problems in that they can save you time
and frustration, but working homework problems has the
advantage that you get immediate feedback on whether or
not you are learning. That is, when you cannot solve a
problem, you know immediately that you may have failed to
learn something important (or you may have made a simple
calculation error). It is obvious that you have to study more
or seek assistance. To use worked examples as a substitute
for working homework problems, you must continually
challenge yourself (by, for example, working some problems
without looking at the solutions in advance) so that you get
feedback on whether or not you are learning.
By the way, to save time and avoid frustration when you are
trying to find your own solution to the worked examples, it is
essential that you use a scientific graphing calculator or,
better, a calculation program on a computer. Having such a
computational tool will allow you to concentrate on learning
principles of statics rather than spending time on mere
numerical manipulation. See the next section, Using
Scientific Graphing Calculators Effectively, for ideas on how
best to do your numerical work with such tools.

iv
The solutions given in the present book contain detailed
numerical calculations that make it clear how the final
answers are obtained. However, when you are working an
example by yourself without looking at the solution, as
explained previously in the Section "How to Study Worked
Examples" having to deal with numerical details is a
distraction that may prevent you from seeing and learning the
underlying principle that the example is meant to illustrate.
To avoid this danger, you are strongly encouraged to use a
scientific graphing calculator (or a calculation program such
as Mathcad, Matlab, Maple, Mathematica, TK Solver, etc.).
But you are not only encouraged to use one of these
calculation tools, but also to use it effectively that is, in a
manner specifically designed to minimize errors, maximize
speed, and provide confidence in the final answer. Because
the author has encountered few students even very able
ones who have taught themselves to operate a calculator in
the manner just described, guidelines are given below for
using calculators and calculation programs effectively, so
that you can concentrate on learning concepts rather than
spending time puzzling over how a particular number
appearing in an example is calculated.
Guideline 1. Enter information in the order in which it
appears in the original problem formulation.
A Second Note: Using Scientific Graphing Calculators Effectively
For example, if you have an equation such as the following
to solve,
14.78 + 5X + 6(3.42) + 2.23(X 4) = 0,
do not rearrange this equation on paper and solve for X the
way you learned in algebra class. Instead, enter the equation
directly in your calculator's solver, and let the calculator do
the algebra.
Guideline 2. Check for errors by proofreading rather than
by repeating the calculation.
If Guideline 1 has been followed, then the equation on paper
can be compared character by character with the equation
appearing in the calculator display or on the computer screen.
Error checking by means of proofreading is preferable to
error checking by repeating the calculation because repeating
the calculation may introduce new errors, but proofreading
never does.
Guideline 3. Data that appear in several equations should be
entered only once.
For example, if the value of a force, 3.491 N, appears more
than once in equations or expressions to be entered into the

v
Using Scientific Graphing Calculators Effectively (Continued)
calculator, then the number should first be stored under the
name of a variable, and that name should be used in all terms
subsequently entered into the calculator.
Guideline 4. Store intermediate results in the calculator
rather than writing them on paper.
Just as in Guideline 3, storing intermediate results under the
name of a variable in the calculator decreases the number of
possibilities for errors.
Guideline 5. Use cut-and-paste rather than re-type.
Again, the number of possibilities for errors is reduced.
Guideline 6. Introduce intermediate variables to simplify
writing multilevel mathematical expressions.
Consider this multilevel mathematical expression:
Entering this expression as a single line of characters would
produce
(81*37.2/16)/(( /2)*(20^4+(9.87/12)^4)).
This is a very error-prone operation, since many parentheses
must be inserted that are not present or are not easily
identified with their position in the original expression. It is
much better to introduce intermediate variables. For
example, define X and Y as follows:
X
Y
That is,
X = 81*37.2/16 (1)
and
Y = ( /2)*(20^4+(9.87/12)^4). (2)
Eqs. 1 and 2 can be entered in the calculator, and then the
original multilevel expression can be entered in the form
X/Y,
which is much less prone to error.
(81)(37.2)
16
2
[20
4
+ ( )
4
]
9.87
12
(81)(37.2)
16
2
[20
4
+ ( )
4
]
9.87
12

vi
Using Scientific Graphing Calculators Effectively (Continued)
When should intermediate variables be used? A rule of
thumb is to introduce intermediate variables if parentheses
have to be nested three (or more) deep when the expression is
written on one line
Guideline 7. Know thoroughly how to use the solver.
As was pointed out in the discussion of Guideline 1, the
solver is the key to avoiding errors in algebraic manipulation.
Guideline 8. Use built-in functions whenever possible.
For example, definite integrals should always be evaluated
with the calculator's built-in integration function rather than
by finding the antiderivative and substituting the values of
the limits. Vector operations such as dot product, cross
product, magnitude (norm), and finding a unit vector should
all be done using built-in functions.
Guideline 9. Do not use Reverse Polish Notation.
Reverse Polish Notation (RPN) is an ingenious technique for
reducing the number of keystrokes needed to perform certain
arithmetic operations. As an example, consider the
evaluation of the expression 6(3 + 2) using RPN:
Keystrokes Calculator Display
3 <Enter> 3
2 <Enter> 2
+ <Enter> 5
6 <Enter> 6
* <Enter> 30
Note that the parentheses do not have to be entered, and thus
the user saves two keystrokes, compared to what would have
been required had RPN not been used. RPN is a standard
feature on some types of calculators and has gained wide
acceptance. However, because the complete arithmetic
expression never is displayed on the screen when RPN is
used, the calculator user cannot easily check for errors by
proofreading. Instead the user must check the calculation by
re-entering the data, thus doubling the number of keystrokes
required to obtain a verified answer, and RPN ends up taking
more keystrokes than a conventional left-to-right data-entry
scheme. Furthermore, if the user makes an error in entering
data during the repeat calculation, then yet another repeat
calculation must be performed and the number of keystrokes
is tripled. The more keystrokes, the more opportunity for
making an error.

vii
Using Scientific Graphing Calculators Effectively (Continued)
Guideline 10. Violate any of the other guidelines when
common sense says to do so.
Guidelines 1 to 9 should be followed in the large majority of
cases, but special situations will arise in which the sensible
thing to do is to violate the guidelines. That is why the word
"guideline" has been used rather than "law.

viii
Acknowledgments
The examples and drawings in this text were developed with
the support of Southern Illinois University Edwardsville
Excellence in Undergraduate Education Grants Nos. 05-25
and 09-29 (Mark P. Rossow, Principal Investigator) and
through the hard work and dedication of the following
people:
Jason Anderson
Paul Cayo
Madan Gyanwali
Tyler Hermann
Vishnu Kesaraju
Jennie Moidel
Binod Neupane
Sanjib Neupane
Ramesh Regmi
Laxman Shrestha
Shikhar Shrestha
Sagun Thapa

ix
Contents

1. Preliminaries: Units and Rounding ............................................................................... 1
2. Force and Position Vectors .......................................................................................... 22
2.1 Adding Forces by the Parallelogram Law ......................................................... 23
2.2 Rectangular Components in Two-Dimensional Force Systems ........................ 60
2.3 Rectangular Components in Three-Dimensional Force Systems .................... 113
2.4 Position Vectors. Use in Defining Force Vectors. ........................................ 146
2.5 Applications of Dot Products .......................................................................... 200
3. Equilibrium of a Particle ............................................................................................ 239
3.1 Particles and Two-Dimensional Force Systems ............................................... 240
3.2 Particles and Three-Dimensional Force Systems ............................................. 261
4. Moments and Resultants of Forces Systems .............................................................. 306
4.1 Moments in Two-Dimensional Force Systems ................................................ 307
4.2 Moments in Three-Dimensional Force Systems .............................................. 340
4.3 Moment of a Couple......................................................................................... 401
4.4 Moment of a Force About a Line. .................................................................. 444
4.5 Equivalent Force-Couple Systems ................................................................... 472
4.6 Distributed Loads on Beams ............................................................................ 556
5. Equilibrium of a Rigid Body ...................................................................................... 605
5.1 Constraints and Static Determinacy ................................................................. 606
5.2 Rigid Bodies and Two-Dimensional Force Systems ....................................... 688
5.3 Rigid Bodies and Three-Dimensional Force Systems ..................................... 731
6. Structural Applications .............................................................................................. 790
6.1 Frames and Machines ....................................................................................... 791
6.2 Trusses: Method of Joints and Zero-Force Members ...................................... 903
6.3 Trusses: Method of Sections ............................................................................ 960
6.4 Space Trusses. .............................................................................................. 1000
6.5 Cables: Concentrated Loads ........................................................................... 1035
6.6 Cables: Uniform Loads .................................................................................. 1082
6.7 Cables: Catenaries .......................................................................................... 1133
x
7. Friction ..................................................................................................................... 1191
7.1 Basic Applications ......................................................................................... 1192
7.2 Wedges ........................................................................................................... 1250
7.3 Square-Threaded Screws ................................................................................ 1286
7.4 Flat Belts ........................................................................................................ 1347
7.5 Thrust Bearings and Disks ............................................................................. 1380
7.6 Journal Bearings ............................................................................................. 1402
7.7 Rolling Resistance .......................................................................................... 1450
8. Internal Forces .......................................................................................................... 1481
8.1 Internal Forces in Structural Members ........................................................... 1482
8.2 Shear and Bending-Moment Diagrams: Equation Form ................................ 1536
8.3 Shear and Bending-Moment Diagrams Constructed by Areas ...................... 1597
9. Centroids and Mass Centers ..................................................................................... 1670
9.1 Centroids by Integration ................................................................................. 1671
9.2 Centroids: Method of Composite Parts .......................................................... 1749
9.3 Theorems of Pappus and Guldinus ................................................................ 1845
9.4 Hydrostatic Pressure on Submerged Surfaces. ............................................. 1924
10. Inertia Properties of Plane Areas............................................................................ 1981
10.1 Moments of Inertia by Integration ............................................................... 1982
10.2 Method of Composite Areas ........................................................................ 2027
10.3 Products of Inertia ........................................................................................ 2067
10.4 Moments of Inertia About Inclined Axes; Principal Moments .................... 2115
11. Energy Methods ..................................................................................................... 2165
11.1 Virtual Work ................................................................................................ 2166
11.2 Potential Energy ........................................................................................... 2270
Appendix: Geometric Properties of Lines, Areas, and Solid Shapes ............................ 2338
Index .............................................................................................................................. 2343
1

1. Preliminaries: Units and Rounding
2
1. Preliminaries: Units and Rounding Problem Statement for Example 1
1. Round off the following numbers
to three significant figures.
a) 54.27 m
b) 2 927 124 m
c) 3.6143 10
-3
in.
d) 6.875 km
e) 6.885 km

3
1. Preliminaries: Units and Rounding Problem Statement for Example 2
2. Express the following quantities in proper SI units.
a) 2491 N
b) 0.02491 N
c) 2 491 000 N
d) 0.0987 10
-5
m
e) 0.0987 10
8
km
f) 0.000 987 10
-2
mm

4
1. Preliminaries: Units and Rounding Problem Statement for Example 3
3. Evaluate the arithmetic expressions and express the
results to three significant figures and in proper SI
form.
a)
b)
c) 934.2 mm
2
2 m
d) 99.7 mN
2
3.6 mm
e) 8 kN 12.1 mm
f)
17 mm
13 N
13 N
17 kg
43 ms
(82.1 ms)
2

5
1. Preliminaries: Units and Rounding Problem Statement for Example 4
4. Express the following forces in pounds (lb).
a) 1,230 kip
b) 0.0230 kip
c) 23.0 oz

6
1. Preliminaries: Units and Rounding Problem Statement for Example 5
5. a) A 20-kg box is set upon a scale that displays
weight in SI force units. What weight will the scale
display?
b) A 20-lb box is set upon a scale that displays
weight in U.S. customary units. What weight will
the scale display?

7
1. Preliminaries: Units and Rounding Example 1, page 1 of 5
1. Round off the following numbers
to three significant figures.
a) 54.27 m
b) 2 927 124 m
c) 3.6143 10
-3
in.
d) 6.875 km
e) 6.885 km
1
2
a) 54.27 lies between 54.20 and 54.30.
54.27
54.20
(midpoint
of interval)
54.30
3 significant figures 3 significant figures
Because 54.27 lies closer to 54.3, round to that value:
54.27 m 54.3 m Ans.

8
1. Preliminaries: Units and Rounding Example 1, page 2 of 5
4
b) 2 927 124 lies between 2 920 000 and 2 930 000.
2 927 124
2 920 000
(midpoint
of interval)
2 930 000
3 significant figures 3 significant figures
Because 2 927 124 lies closer to 2 930 000, round to that
value:
2 927 124 m 2 930 000 m
= 2.93 10
6
m Ans.
Use engineering
notation in which the
significant figures are
followed by a factor of
10 raised to a power.
To make it easy to select the
correct SI prefix, use an
exponent that is a multiple of 3.
5
6
3

9
1. Preliminaries: Units and Rounding Example 1, page 3 of 5
7
8
c) 3.6143 10
-3
lies between 3.610 10
-3
and 3.620 10
-3
.
3.610 10
-3
(midpoint of
interval)
3.62 10
-3
3 significant figures 3 significant figures
Round to the closer value:
3.6143 10
-3
in. 3.61 10
-3
in. Ans.
3.6143 10
-3

10
1. Preliminaries: Units and Rounding Example 1, page 4 of 5
9
10
d) 6.875 lies precisely halfway between 6.870 and 6.880.
6.870
6.875
(midpoint
of interval)
6.880
3 significant figures 3 significant figures
The rule is "If the number lies at the midpoint of the interval,
then round off so that the last digit is even":
6.875 km 6.88 km Ans.
Even digit (8)
The rationale for the rule is that over a long chain of
calculations, the rule will lead to rounding down approximately
the same number of times as rounding up, and, on balance, these
rounding errors will cancel.
12
11

11
1. Preliminaries: Units and Rounding Example 1, page 5 of 5
14
e) 6.885 lies precisely halfway between 6.880 and 6.890.
6.880
6.885
(midpoint
of interval)
6.890
3 significant figures 3 significant figures
Round off so that the last digit is even:
6.885 km 6.88 km Ans.
Even digit (8)
In this case (6.885), we rounded down to get an even digit; in
the previous case (6.875), we rounded up to get an even digit.
16
15
6.880 6.870 6.875 6.890 6.885
Even digit (8) Odd digit (7) Odd digit (9)
Round up Round down
13

12
1. Preliminaries: Units and Rounding Example 2, page 1 of 4
1
2. Express the following quantities in proper SI units.
a) 2491 N
b) 0.02491 N
c) 2 491 000 N
d) 0.0987 10
-5
m
e) 0.0987 10
8
km
f) 0.000 987 10
-2
mm
a) 2491 N = 2.491 10
3
N
= 2.491 kN Ans.
Choose the exponent of 10 to be a
multiple of 3, because SI prefixes
are defined for 10
3
, 10
6
, 10
9
, ...,
and 10
-3
, 10
-6
, 10
-9
, ... .
Try to keep the value of the
coefficient between 0.1 and 1000.
Here it is possible. In other cases,
such as 2222 mm
4
, it is not.
Prefix "k" = "kilo" = 10
3
.
2
3
4

13
1. Preliminaries: Units and Rounding Example 2, page 2 of 4
5 b) 0.024 91 N = 24.91 10
-3
N
= 24.91 mN Ans.
Exponent of 10 is a multiple of 3.
Prefix "m" = "milli" = 10
-3
.
7
8
Coefficient lies
between 0.1 and 1000.
6
12 Prefix "M" = "mega" = 10
6
.
c) 2 491 000 N = 2.491 10
6
N
= 2.491 MN Ans.
9
10 Coefficient lies
between 0.1 and 1000.
11 Exponent of 10 is a multiple of 3.

14
1. Preliminaries: Units and Rounding Example 2, page 3 of 4
d) 0.0987 10
-5
m = 0.987 10
-6
m
= 0.987 m Ans.
Prefix " " = "micro" = 10
-6
14
17 Coefficient lies
between 0.1 and 1000.
19
18
Prefix "G" = "giga" = 10
9
.
Exponent of 10 is a multiple of 3.
e) 0.0987 10
8
km = 0.0987 10
8
(10
3
) m
= 0.0987 10
11
m
= 9.87 10
9
m
= 9.87 Gm Ans.
15
Convert to base unit (from km to m). 16
13

15
1. Preliminaries: Units and Rounding Example 2, page 4 of 4
23
22
Prefix "n" = "nano" = 10
-9
.
Exponent of 10 is a multiple of 3.
f) 0.000 987 10
-2
mm = 0.000 987 10
-2
(10
-3
) m
= 9.87 10
-6
(10
-3
) m
= 9.87 10
-9
m
= 9.87 nm Ans.
20
Convert to base unit (from mm to m). 21

16
1. Preliminaries: Units and Rounding Example 3, page 1 of 4
a)
=
= (13/17)(10
3
) N/m
= 0.765 (10
3
) N/m
= 0.765 kN/m Ans.
1
3. Evaluate the arithmetic expressions and express the
results to three significant figures and in proper SI
form.
a)
b)
c) 934.2 mm
2
2 m
d) 99.7 mN
2
3.6 mm
e) 8 kN 12.1 mm
f)
17 mm
13 N
13 N
17 kg
43 ms
(82.1 ms)
2
13 N
17 mm
=
17(10
-3
) m
13 N
13(10
3
) N
17 m
2 Always express the denominator in base units
(In statics, the base units are m, s, and kg).

17
1. Preliminaries: Units and Rounding Example 3, page 2 of 4
b) = (13/17) N/kg
= 0.765 N/kg Ans.
3
13 N
17 kg
4 "kg" is a base unit and so may appear in the
denominator.
The exponent applies to the
prefix "milli" as well as to
the base unit "meters".
6
5 c) 934.2 mm
2
2 m = 934.2 (10
-3
m)
2
2 m
= (934.2 2)(10
-6
m
2
) m
= (1868.4)(10
-6
m
3
)
= (1.87 10
3
)(10
-6
m
3
)
= 1.87 10
-3
m
3
Ans.
In this example, it is not possible to make
the coefficient lie between 0.1 and 1000
by choosing an SI prefix.
7

18
1. Preliminaries: Units and Rounding Example 3, page 3 of 4
The exponent applies to the
prefix "milli" as well as to
the base unit "newtons".
9
8 d) 99.7 mN
2
3.6 mm = 99.7(10
-3
N)
2
3.6(10
-3
m)
= (99.7 3.6)(10
-6
N
2
)(10
-3
m)
= (358.92)(10
-6
N
2
)(10
-3
m)
= (359)(N
2
10
-9
m)
= 359 N
2
nm Ans.
The raised dot indicates
the product of two units.
10
Prefix "n" = "nano" = 10
-9
e) 8 kN 12.1 mm = 8(10
3
N) 12.1(10
-3
m)
= (8 12.1)(10
3-3
Nm)
= 96.8 Nm Ans.
12
11

19
1. Preliminaries: Units and Rounding Example 3, page 4 of 4
13 f)
= 6.38 10
-3
(10
6
) m/s
= 6.38 km/s Ans.
14
43 ms
ms)
2
=
10
-3
s)
2
43 ms
2
43 ms
(10
-6
s
2
=
Raised dot means
"product of meters and
seconds."
No raised dot so "ms"
means "milli-seconds."
15

20
1. Preliminaries: Units and Rounding Example 4, page 1 of 1
4. Express the following forces in pounds (lb).
a) 1,230 kip
b) 0.0230 kip
c) 23.0 oz
1
a) 1,230 kip = 1,230 1,000 lb
= 1.23 10
6
lb Ans.
b) 0.0230 kip = 0.0230 1,000 lb
= 23.0 lb Ans.
c) 23.0 oz = 23.0 oz 1 lb/16 oz)
= 1.4375 lb
= 1.44 lb Ans.
"kip" = "kilo-pound" = 1,000 lb 2
3
4

21
1. Preliminaries: Units and Rounding Example 5, page 1 of 1
5. a) A 20-kg box is set upon a scale that displays
weight in SI force units. What weight will the scale
display?
b) A 20-lb box is set upon a scale that displays
weight in U.S. customary units. What weight will
the scale display?
1 a) Weight = mass acceleration due to gravity
= (20 kg) (9.81 m/s
2
)
= 196.2 kgm/s
2
= 196.2 N Ans.
b) Weight = 20 lb Ans.
("lb" is a force unit; no factor corresponding
to the acceleration of gravity is needed.)
2
3
1 kgm/s
2
1 N

22

2. Force and Position Vectors
23

2.1 Adding Forces by the Parallelogram Law
24
2.1 Adding Forces by the Parallelogram Law: Procedures and Strategies, page 1 of 2
Procedures and Strategies for Solving Problems Involving
Addition of Forces by the Parallelogram Law
To add two force vectors,
1. make a sketch showing the vectors placed tail-to-tail;
2. construct a parallelogram, using the vectors as two of the
sides;
3. draw the diagonal that goes from the tail-tail vertex to the
opposite vertex (This is the resultant the vector sum of
the two forces); and
4. use the sine and cosine laws and geometrical relations
between angles to calculate the magnitude and direction of
the resultant.
A
A
A
A
B
B
B
B
A
B
C
Law of sines
sin a
A
sin b
B
sin c
C
= =
Law of cosines
C
2
= A
2
+ B
2
- 2AB cos c
+
A + B
b
a
c

25
2.1 Adding Forces by the Parallelogram Law: Procedures and Strategies, page 2 of 2
F
u
v
u
v
F
u
v
F
To resolve a given force into components in two given
directions,
1. make a sketch showing the tail of the force vector at the
intersection of two lines in the given directions;
2. construct a parallelogram with the force vector as a
diagonal and the sides parallel to the given directions;
and
3. use the sine and cosine laws and geometrical relations
between angles to calculate the lengths of the sides of the
parallelogram. The sides are the components of the
force vector.
Fu
Fv

26
2.1 Adding Forces by the Parallelogram Law Problem Statement for Example 1
1. Determine the magnitude and direction
of the resultant of the forces shown.
30
150 N
200 N

27
2.1 Adding Forces by the Parallelogram Law Problem Statement for Example 2
y
x
10
20
2. Determine the magnitude and direction
of the resultant of the forces shown.
3 kN
2 kN

28
2.1 Adding Forces by the Parallelogram Law Problem Statement for Example 3
110
100 N
80 N
40
3. Determine the magnitude and direction of the resultant force.

29
2.1 Adding Forces by the Parallelogram Law Problem Statement for Example 4
x
40 lb
60 lb
30
y
4. The resultant of the two forces acting on the screw eye is known to
be vertical. Determine the angle and the magnitude of the resultant.

30
2.1 Adding Forces by the Parallelogram Law Problem Statement for Example 5
5. Determine the magnitude F and the angle , if the
resultant of the two forces acting on the block is to be a
horizontal 80-N force directed to the right.
50 N
F
25

31
2.1 Adding Forces by the Parallelogram Law Problem Statement for Example 6
A
B
30 N
25 N
6. To support the 2-kg flower pot shown, the resultant of the two
wires must point upwards and be equal in magnitude to the
weight of the flower pot. Determine the angles and , if the
forces in the wires are known to be 25 N and 30 N.
C

32
2.1 Adding Forces by the Parallelogram Law Problem Statement for Example 7
25
40
v
u
120 lb
7. Resolve the 120-lb force into components
acting in the u and v directions.

33
2.1 Adding Forces by the Parallelogram Law Problem Statement for Example 8
8. Resolve the 4-kN
horizontal force into
components along truss
members AB and AC.
35
5
12
A
B
C
4 kN

34
2.1 Adding Forces by the Parallelogram Law Problem Statement for Example 9
9. Find two forces, one acting along rod AB and one
along rod CB, which when added, are equivalent to the
200-N vertical force.
200 N
A
B
C
30 40

35
2.1 Adding Forces by the Parallelogram Law Example 1, page 1 of 4
1. Determine the magnitude and direction
of the resultant of the forces shown.
30
150 N
200 N
30
200 N
150 N
30
Construct a parallelogram by drawing two
lines. Each line starts at the tip of one vector
and is parallel to the other vector.
Tip
Parallel
Tip
1

36
2.1 Adding Forces by the Parallelogram Law Example 1, page 2 of 4
30
200 N
150 N
30
Since opposite sides of a parallelogram are
equal in length, the length of each line
represents the magnitude of the vector opposite.
200 N
150 N
2
30
R
The resultant R is drawn from the tails of the
vectors to the opposite vertex of the parallelogram.
Tails
150 N
200 N
150 N
200 N
3

37
2.1 Adding Forces by the Parallelogram Law Example 1, page 3 of 4
200 N
150 N
30
R
150 N
200 N
150 N
200 N
Tails
Head
Heads
4
30
200 N
R
150 N
To calculate the magnitude and direction of R, consider
the triangle formed by one half of the parallelogram.
5
200 N
150 N
This is not the resultant because it is not
drawn from the intersection of the tails.

38
2.1 Adding Forces by the Parallelogram Law Example 1, page 4 of 4
30
200 N
R
150 N
Use trigonometry to calculate the magnitude and direction
of the resultant.
R
2
= (200 N)
2
+ (150 N)
2
2(200 N)(150 N) cos 30
The result is
R = 102.66 N Ans.
=
Solving gives
= 46.9 Ans.
b
a
c
C
A
B
Law of cosines
C
2
= A
2
+ B
2
2AB cos c
Law of sines
= =
6
sin a
A B
sin b
C
sin c
sin
150 N
sin 30
R
= 102.66 N
Trigonometric formulas for a
general triangle are given below.

39
2.1 Adding Forces by the Parallelogram Law Example 2, page 1 of 3
2. Determine the magnitude and direction
of the resultant of the forces shown.
3 kN
2 kN
Construct a parallelogram by drawing
two lines parallel to the forces.
1
y
x
3 kN
2 kN
y
x
3 kN
2 kN
10
20
10
20

40
2.1 Adding Forces by the Parallelogram Law Example 2, page 2 of 3
Draw the resultant R from the tails of the vectors to the
opposite vertex of the parallelogram.
3 kN
2 kN
y
x
3 kN
2 kN
2
Tails
R
2 kN
2 kN
R
3 kN
3 kN
y
x
To calculate the magnitude and direction of R, consider
the triangle formed by one half of the parallelogram.
3
R
3 kN
2 kN
10
20
20
10
10
20
20
10

41
2.1 Adding Forces by the Parallelogram Law Example 2, page 3 of 3
R
3 kN
2 kN
Use trigonometry to calculate the magnitude
and direction of the resultant.
4
total angle
= 10 + 90 + 20 = 120
Law of cosines
R
2
= (3 kN)
2
+ (2 kN)
2
2(3 kN)(2 kN) cos 120
R = 4.359 kN Ans.
sin 120
R
Law of sines
=
Solving gives
= 36.6
sin
3 kN
= 4.359 kN
5
6
R = 4.36 kN
x
y
36.6 + 20 = 56.6 Ans.
= 36.6
7 Angle measured with
respect to the vertical axis
20
10
20
20

42
2.1 Adding Forces by the Parallelogram Law Example 3, page 1 of 3
110
100 N
80 N
Construct a parallelogram 1
100 N
80 N
40
110
y
40
3. Determine the magnitude and direction of the resultant force.

43
2.1 Adding Forces by the Parallelogram Law Example 3, page 2 of 3
Draw the resultant R from the tails of the vectors to
the opposite vertex of the parallelogram.
100 N
80 N
40
110
y
2
100 N
80 N
R
40
110
80 N
100 N
y
R
30
To calculate R, consider the triangle formed
by the lower half of the parallelogram.
3
Calculate angle
180 110 40 = 30
Parallel lines make 30 angle
with vertical direction
4
5
Calculate angle
30 + 40 = 70
6
40
Law of cosines
R
2
= (80 N)
2
+ (100 N)
2
2(80 N)(100 N) cos 70
R = 104.54 N Ans.
7

44
2.1 Adding Forces by the Parallelogram Law Example 3, page 3 of 3
y
100 N
R = 104.54 N
70
Calculate the angle that the
resultant makes with the vertical.
Law of sines
=
Solving gives
= 64.0
R
sin 70
100 N
sin
104.54 N
8
R = 104.54 N
y
Angle measured from the vertical 9
40 + 64.0 = 104.0 Ans.
40
64
40

45
2.1 Adding Forces by the Parallelogram Law Example 4, page 1 of 4
x
40 lb
60 lb
30
y
4. The resultant of the two forces acting on the screw eye is known to
be vertical. Determine the angle and the magnitude of the resultant.

46
2.1 Adding Forces by the Parallelogram Law Example 4, page 2 of 4
40 lb
30
x
y
30
40 lb
x
y
30
40 lb
y
x
To determine what needs to be calculated, make some
sketches of several possible parallelograms.
1
R
R
R
Each parallelogram is based on two facts that are given:
1) One side of the parallelogram is known (40 lb at 30), and
2) The resultant R lies on the y axis.
2

47
2.1 Adding Forces by the Parallelogram Law Example 4, page 3 of 4
30
40 lb
x
y
How do we determine the actual parallelogram? We have
to use the additional fact that one of the forces is 60 lb.
3
The point of the intersection of the arc and the
vertical axis must be the vertex of the
parallelogram since it lies on the vertical axis and
also lies a "distance" of 60 lb from the tip of the
40-lb vector.
4
Now the parallelogram is completely defined.
60 lb
40 lb
30
x
y
5
40 lb
60 lb
Radius of circular
arc = 60 lb

48
2.1 Adding Forces by the Parallelogram Law Example 4, page 4 of 4
To calculate the resultant R and and the
angle (see below), analyze the triangle
formed by the left half of the parallelogram.
Angle = 90 30
= 60
Corresponding angles are equal
y
30
x
40 lb
60 lb
6
8
7
30
Parallel
60 lb
40 lb
60
+ 30
R
sin 60
60 lb
Law of sines
=
Solving gives
= 35.26
sin
40 lb
Law of sines
=
Solving gives
R = 69.0 lb Ans.
sin 60
60 lb
sin( + 30)
R
9
11
54.74
The sum of the angles of the triangle is 180:
+ ( + 30) + 60 = 180
Solving gives
= 54.74 Ans.
10
35.26
R

49
2.1 Adding Forces by the Parallelogram Law Example 5, page 1 of 2
5. Determine the magnitude F and the angle , if the
resultant of the two forces acting on the block is to be a
horizontal 80-N force directed to the right.
50 N
F
Draw the parts of parallelogram that are known:
Two sides are of length 50 N and make
an angle of 25 with the horizontal axis.
The diagonal of the parallelogram (the
resultant) is 80 N long and horizontal.
50 N
50 N
80 N
1
2
3
25
25
25

50
2.1 Adding Forces by the Parallelogram Law Example 5, page 2 of 2
Complete the parallelogram.
25
50 N
50 N
80 N
4
25
25
F
F
Analyze the triangle forming the lower half of the parallelogram.
6
25
F
80 N
50 N Calculate F from the law of cosines.
F
2
= (50 N)
2
+ (80 N)
2
2(50 N)(80 N) cos 25
The result is
F = 40.61 N Ans. sin
50 N
Calculate from the law of sines.
=
Solving gives
= 31.4 Ans.
sin 25
F
40.61 N
7
5

51
2.1 Adding Forces by the Parallelogram Law Example 6, page 1 of 2
A
B
30 N
25 N
6. To support the 2-kg flower pot shown, the resultant of the two
wires must point upwards and be equal in magnitude to the
weight of the flower pot. Determine the angles and , if the
forces in the wires are known to be 25 N and 30 N.
B
Weight of flower pot
mg = (2 kg)(9.81 m/s
2
)
= 19.62 N
1
19.62 N 19.62 N
B
R = 19.62 N
Resultant, R, of forces in wires
balances the weight.
2
C

52
2.1 Adding Forces by the Parallelogram Law Example 6, page 2 of 2
B
25 N
30 N
30 N
25 N
The resultant R = 19.62 N must be the diagonal of a
parallelogram with sides 25 N and 30 N long.
3
Analyze the triangle forming the left-hand
half of the parallelogram.
4
25 N
30 N
19.62 N
19.62 N
Law of cosines to calculate
(25 N)
2
= (30 N)
2
+ (19.62 N)
2
2(30 N)(19.62 N) cos
Solving gives
= 55.90 Ans.
Law of sines to calculate
=
Solving gives
= 83.6 Ans.
30 N
sin
5
sin
25 N
55.90
6

53
2.1 Adding Forces by the Parallelogram Law Example 7, page 1 of 2
25
40
v
u
120 lb
7. Resolve the 120-lb force into components
acting in the u and v directions.
Construct a parallelogram with
the 120-lb force as a diagonal.
Draw another line from the
tip but parallel to u.
Draw a line from the tip of the
force vector parallel to v.
120 lb
v
40
u
40
25
1
2
3
R

u
R

v
Label the components R

u and R

v. 4

54
2.1 Adding Forces by the Parallelogram Law Example 7, page 2 of 2
120 lb
40
25 R

v
R

u
Analyze the triangle forming the left-hand half of the
parallelogram.
180 40 25 = 115
5
Calculate R

u
from the law of sines.
=
Solving gives
R

u = 78.9 lb Ans.
120 lb
sin 40 sin 25
R

u
6
Calculate R

v
from the law of sines.
=
Solving gives
R

v = 169.2 lb Ans.
sin 40 sin 115
R

v 120 lb
7

55
2.1 Adding Forces by the Parallelogram Law Example 8, page 1 of 2
8. Resolve the 4-kN
horizontal force into
components along truss
members AB and AC.
35
5
12
A
B
C
4 kN
Extend line AC.
35
Construct a parallelogram with the
4-kN force as the diagonal.
12
5
4 kN
2
B
C
Draw another line from
the tip but parallel to AC.
4
5
12
A
Draw a line from the tip of the
force vector parallel to AB.
3
1

56
2.1 Adding Forces by the Parallelogram Law Example 8, page 2 of 2
35
12
5
4 kN
B
C
5
12
A
Label the components R

B
and R

C
. 5
R

B
R

C
4 kN
12
5
R

C
R

B
Analyze the triangle forming the upper half of the
parallelogram (The drawing has been enlarged for clarity).
6
35
12
5
Geometry
= tan
-1
= 67.38
= 180 35 67.38

= 77.62
7
77.62
Law of sines to calculate R

C

=
Solving gives
R

C
= 3.78 kN Ans.
sin
sin
R

C
4 kN
67.38
Law of sines to calculate R

B
=
Solving gives
R

B
= 2.35 kN Ans.
R

B

sin 35
4 kN
sin
77.62
8
9
35

57
2.1 Adding Forces by the Parallelogram Law Example 9, page 1 of 3
9. Find two forces, one acting along rod AB and one
along rod CB, which when added, are equivalent to the
200-N vertical force.
200 N
A
B
C
30 40

58
2.1 Adding Forces by the Parallelogram Law Example 9, page 2 of 3
30
40
30
40
30
40 30
200 N
A
B
C
30
1 Construct a parallelogram with
sides parallel to AB and BC and
with the 200-N force as a
diagonal.
Draw a line from the tail of the
force vector parallel to AB.
Draw another line from the
tail but parallel to BC.
Extend AB.
Extend BC.
4
5
3
2
200 N
B
R

C
R

A
Label the sides of the
parallelogram R

A
and R

C
.
6
R

C
R

A

59
2.1 Adding Forces by the Parallelogram Law Example 9, page 3 of 3
200 N
R

C
30
40


180 50 (30 + 40) = 60
200 N
Law of sines to calculate R

A
=
Solving gives
R

A
= 163.0 N Ans.
R

A

sin 50
90 40 = 50
Analyze the triangle
formed by the
left-hand side of the
parallelogram.
Angles are equal
7
11
sin (30 + 40)
Law of sines to calculate R

C
=
Solving gives
R

C
= 184.3 N Ans.
R

C

sin 60
sin (30 + 40)
200 N
12
8
9
10
R

A
40

60

2.2 Rectangular Components in Two-Dimensional Force Systems
61
2.2 Rectangular Components in Two-Dimensional Force Systems Procedures and Strategies, page 1 of 1
F
x
y
F
y
= F sin
Fx = F cos
x
y
5
12
13
F
Fx =
12F
13
F
y
=
5F
13
Procedures and Strategies for Solving Problems Involving
Rectangular Components in Two-Dimensional Force
Systems
1. Two situations commonly arise in which the rectangular
components are to be computed.
a) The force is defined by its magnitude F and the angle
that the force makes with the positive x axis. In this case,
use the equations
Fx = F cos F
y
= F sin
b) The force is defined by a magnitude F and a slope
triangle. The components can be computed by multiplying
the magnitude F by the ratios of the slope triangle the ratio
of the horizontal side to the hypotenuse gives the horizontal
component of the force. The ratio of the vertical side gives
the vertical component of the force.
2. To add forces simply
a) add all x components to obtain Rx, the x component of the
resultant R, and
b) add all y components to obtain R
y
, the y component of the
resultant R.
The magnitude and direction of the resultant can be found
from a right triangle with R, Rx, and R
y
as its sides.
R
x
y
R
y

Rx
R
2
= Rx
2
+ R
y
2
= tan
-1 Rx
R
y

62
2.2 Rectangular Components in Two-Dimensional Force Systems Problem Statement for Example 1
y
x
1. Express the 5-kN force in terms of x and y components.
F = 5 kN
30

63
2.2 Rectangular Components in Two-Dimensional Force Systems Problem Statement for Example 2
2. Resolve the 20-lb force into x and y components.
x
y
F = 20 lb
3
4

64
2.2 Rectangular Components in Two-Dimensional Force Systems Problem Statement for Example 3
3. Express the 260-N force in terms of components
parallel and perpendicular to the inclined plane.
5
12
F = 260 N

65
2.2 Rectangular Components in Two-Dimensional Force Systems Problem Statement for Example 4
4. Determine the components of the 160-N force
perpendicular and parallel to the axis of the nail.
F = 160 N
20
15

66
2.2 Rectangular Components in Two-Dimensional Force Systems Problem Statement for Example 5
5. The connecting rod AB exerts a 2-kN force on the
crankshaft at B. Resolve this force into components
acting perpendicular to BC and along BC.
A
B
C
F = 2 kN
20
30

67
2.2 Rectangular Components in Two-Dimensional Force Systems Problem Statement for Example 6
6. Guy wire AB exerts a horizontal component of force of 0.5 kN
on the utility pole. Determine the total force from the wire acting
on the point of attachment, A. Assume that the force is directed
along the wire from A to B.
10 m
5 m
A
B

68
2.2 Rectangular Components in Two-Dimensional Force Systems Problem Statement for Example 7
7. If the vertical component of the force F
applied to the ring is 10 lb, determine the
magnitude F and also the horizontal component.
F
30

69
2.2 Rectangular Components in Two-Dimensional Force Systems Problem Statement for Example 8
8. The weight W is supported by the boom AB and
cable AC. Knowing that the horizontal and vertical
components of the cable force at A are 5 kN and 3 kN
as shown, determine the distance d.
d
C
B
10 m
F

cable
5 kN
W
3 kN
A

70
2.2 Rectangular Components in Two-Dimensional Force Systems Problem Statement for Example 9
9. Determine the magnitude and direction of
the resultant force acting on the hook.
5
12
20 lb
104 lb
x
y
35

71
2.2 Rectangular Components in Two-Dimensional Force Systems Problem Statement for Example 10
10. Determine the magnitude and direction
of the resultant force acting on the beam.
40
8 kN
15 kN
11 kN
3
4

72
2.2 Rectangular Components in Two-Dimensional Force Systems Problem Statement for Example 11
11. Determine the magnitude and direction
of the resultant force acting on the particle.
x
y
(5 m, 3 m)
25 N
80 N
50 N
(6 m, 2 m)
( m, 6 m)

73
2.2 Rectangular Components in Two-Dimensional Force Systems Problem Statement for Example 12
12. Three forces support the weight W shown. Determine
the value of F, given that the resultant of the three forces
is vertical. Also determine the value of W.
40
W
30
15
120 N
F
y
20 N
x

74
2.2 Rectangular Components in Two-Dimensional Force Systems Problem Statement for Example 13
13. The resultant, R, of the forces A and B acting on the
bracket is known to be a force of magnitude 300 lb
making an angle of 40 with the horizontal direction as
shown. Determine the magnitude of A and B.
x
y
70
40
B
A
300 lb (resultant, R)

75
2.2 Rectangular Components in Two-Dimensional Force Systems Problem Statement for Example 14
14. To support the 100-N block as shown, the resultant of
the 50-N force and the force F must be a 100-N force
directed horizontally to the right. Determine F and .
60
100 N
F
50 N
Pulley

76
2.2 Rectangular Components in Two-Dimensional Force Systems Example 1, page 1 of 3
1
y
x
Construct a parallelogram with the
5-kN force as the diagonal and with
sides in the x and y directions.
1. Express the 5-kN force in terms of x and y components.
F = 5 kN
Because the x and y axes are
perpendicular, the parallelogram
is a special case a rectangle.
2
Analyze the triangle
forming the lower half of
the rectangle.
3
F

x
F

x
F

y
F

y
5 kN
y component, F

y
x component, F

x
F = 5 kN
30
30
30

77
2.2 Rectangular Components in Two-Dimensional Force Systems Example 1, page 2 of 3
4 Calculate F

x
from the definition of the cosine:
cos =
so
A = C cos
In words,
side adjacent (to angle) = hypotenuse
times cosine of angle.
(Memorize this you will use this relation many
times in a course in statics; you don't want to have to
think it out each time)
Applying this equation to the force triangle gives:
F

x
= (5 kN) cos 30
= 4.33 kN (1)
C
A
A
B
C B
C
Similarly calculate F

y
from the definition of the
sine:
sin =
so
B = C sin
In words ,
side opposite (to angle) = hypotenuse
times sine of angle.
(Memorize this.)
Applying this equation to the force triangle gives:
F

y
= (5 kN) sin 30
= 2.50 kN (2)
5

78
2.2 Rectangular Components in Two-Dimensional Force Systems Example 1, page 3 of 3
6
y
x
Thus we have resolved the 5-kN force
into x and y components.
In terms of base vectors, the force is
F = 4.33i + 2.50j kN Ans.
7
The minus sign indicates that the x component
points in the negative x direction.
8
F

x
= 4.33 kN
F

y
= 2.50 kN
F = 5 kN
y
x
i
j
30

79
2.2 Rectangular Components in Two-Dimensional Force Systems Example 2, page 1 of 4
1 Construct a parallelogram (rectangle)
with the 20-lb force as the diagonal.
2. Resolve the 20-lb force into x and y components.
Analyze the triangle forming the
lower half of the rectangle.
2
F

x
F

y
F

y
F

x
F = 20 lb
x
y
4
3
F = 20 lb
3
4
20 lb
F

y
F

x
Equal angles
3
4

80
2.2 Rectangular Components in Two-Dimensional Force Systems Example 2, page 2 of 4
4 From the "slope triangle," we see
cos (3)
sin
Note that we do not have to calculate ;
we already have what we need, sin and cos .
Side adjacent = hypotenuse cos
or
F

x
= (20 lb) cos (1)
Similarly, side opposite = hypotenuse sin
or
F

y
= (20 lb) sin (2)
3
3
4
F

y
F

x
5
4
3
5
3
2
+ 4
2
= 5

81
2.2 Rectangular Components in Two-Dimensional Force Systems Example 2, page 3 of 4
5 Using Eq. 3 in Eq. 1 gives
F

x
= (20 lb) ( )
= 16 lb (5)
In general, then, get the horizontal force
component by multiplying the force by
the horizontal side of the slope triangle
divided by the hypotenuse (Memorize
this result; it is used frequently).
6
3
4
F

x
5
4
5
F = 20 lb
Slope triangle

82
2.2 Rectangular Components in Two-Dimensional Force Systems Example 2, page 4 of 4
7 Similarly, using Eq. 4 in Eq. 2 gives
F

y
= (20 lb) ( )
= 12 lb (6)
In general, get the vertical force
component by multiplying the force by
the vertical side of the slope triangle
divided by the hypotenuse (Memorize
this result).
8
3
4
5
3
5
F = 20 lb
F

y
x
y
16 lb
12 lb
F = 20 lb
j
i
9 Eqs. 5 and 6 now give the components in
terms of base vectors as
F = {16i + 12j} lb Ans.

83
2.2 Rectangular Components in Two-Dimensional Force Systems Example 3, page 1 of 2
1 Introduce an inclined x and y coordinate system.
3. Express the 260-N force in terms of components
parallel and perpendicular to the inclined plane.
5
12
F = 260 N
5
12
F = 260 N
x
y
F

y
F

y
F

x
F

x
12
5
Construct a parallelogram (rectangle) with the
260-N force as a diagonal.
2

84
2.2 Rectangular Components in Two-Dimensional Force Systems Example 3, page 2 of 2
x
y
Analyze the triangle forming the lower half
of the rectangle.
3
F

y
F

x
5
2
+ 12
2
= 13
5 12
13
F = 260 N
F

y
= (260 N)( ) = 240 N
F

x
= (260 N)( ) = 100 N 4
13
5
12
13
5
j
i
Representation in terms of components:
F = { 100i 240j} N Ans.
6
F

x
points in the negative x direction, and 7
F

y
points in the negative y direction.

85
2.2 Rectangular Components in Two-Dimensional Force Systems Example 4, page 1 of 3
1 Introduce an inclined x and y coordinate system.
4. Determine the components of the 160-N force
perpendicular and parallel to the axis of the nail.
F = 160 N
x
y
20
15
20
F = 160 N
15

86
2.2 Rectangular Components in Two-Dimensional Force Systems Example 4, page 2 of 3
2 Geometry
x
y
20
F = 160 N
Equal angles 3
15
Total angle
= 20 + 15
= 35
4
F
=
1
6
0
N
y
x
F

x
F

y
35
5 Draw a parallelogram (rectangle)
with the 160-N force as a diagonal.
15

87
2.2 Rectangular Components in Two-Dimensional Force Systems Example 4, page 3 of 3
6 Analyze the triangle forming the
bottom half of the rectangle.
F
=
1
6
0
N
y
x
35
7 In terms of base vectors,
F = { 131.1i 91.8j} N Ans.
F

y
= (160 N) sin 35
= 91.8 N
F

x
= (160 N) cos 35
= 131.1 N
15
j
i
F

y

88
2.2 Rectangular Components in Two-Dimensional Force Systems Example 5, page 1 of 3
5. The connecting rod AB exerts a 2-kN force on the
crankshaft at B. Resolve this force into components
acting perpendicular to BC and along BC.
A
B
C
F = 2 kN
20
30

89
2.2 Rectangular Components in Two-Dimensional Force Systems Example 5, page 2 of 3
2 kN
Introduce an inclined x and y
coordinate system.
1
Calculate angles 2
x
y
30
Equal 3
4 Equal
Calculate the sum: 20 + 30 = 50 5
B
C
2 kN
x
y
C
B
2 kN
(2 kN) sin 50 = 1.532 kN
(2 kN) cos 50 = 1.286 kN
Calculate components 6
20
30
50
20
30
C
B
A
x
y
20

90
2.2 Rectangular Components in Two-Dimensional Force Systems Example 5, page 3 of 3
In terms of base vectors,
F = 1.532i 1.286j kN Ans.
7
j
y
x
i
30

91
2.2 Rectangular Components in Two-Dimensional Force Systems Example 6, page 1 of 3
6. Guy wire AB exerts a horizontal component of force of 0.5 kN
on the utility pole. Determine the total force from the wire acting
on the point of attachment, A. Assume that the force is directed
along the wire from A to B.
10 m
5 m
A
B

92
2.2 Rectangular Components in Two-Dimensional Force Systems Example 6, page 2 of 3
10 m
5 m
A
B
Express the guy-wire force F in
terms of rectangular components.
1
F
F

y
F

x
= 0.5 kN
x
y
The horizontal component of
force is known to be 0.5 kN.
2

93
2.2 Rectangular Components in Two-Dimensional Force Systems Example 6, page 3 of 3
10 m
5 m
A
B
Relate F

x
to F through geometry. 3
= tan
-1
= 63.43 4
5 m
10 m
A
F

x
= 0.5 kN
F
F

y
0.5 kN = F cos
63.43
Solving gives
F = 1.118 kN Ans.
5

94
2.2 Rectangular Components in Two-Dimensional Force Systems Example 7, page 1 of 1
1 Express F in terms of rectangular components.
7. If the vertical component of the force F
applied to the ring is 10 lb, determine the
magnitude F and also the horizontal component.
x
y
F
30
30
F
F

y
F

x
2 Relate F to F

y
.
30
F

y
= 10 lb
F

x
F
10 lb = F sin 30
Therefore,
F = 20 lb Ans.
Relate F

x
to F.
F

x
= F cos 30
= (20 lb) cos 30
= 17.32 lb Ans.
3

95
2.2 Rectangular Components in Two-Dimensional Force Systems Example 8, page 1 of 2
8. The weight W is supported by the boom AB and
cable AC. Knowing that the horizontal and vertical
components of the cable force at A are 5 kN and 3 kN
as shown, determine the distance d.
d
C
B
10 m
F

cable
5 kN
W
3 kN
A
1 Calculate the angle between F

cable

and its horizontal component.
F

cable
3 kN
5 kN
d
10 m
C
B
A
W
3 kN
5 kN
= tan
-1

= 30.96
3 kN
5 kN
F

cable

96
2.2 Rectangular Components in Two-Dimensional Force Systems Example 8, page 2 of 2
C
A
d
10 m
5 kN
3 kN
The same result could also have been
obtained by using similar triangles.
3
C
A
d
10 m
Use to calculate d. 2
d = (10 m) tan 30.96
= 6.0 m Ans.
=
Therefore,

d = 6.0 m (same as before)
3 kN
5 kN
d
10 m
= 30.96

97
2.2 Rectangular Components in Two-Dimensional Force Systems Example 9, page 1 of 2
1 Express the forces in x and y components.
9. Determine the magnitude and direction of
the resultant force acting on the hook.
2 Calculate the x and y components of the resultant R by summing the
components of the given forces algebraically.
R

x
= F

x
: R

x
= 16.38 lb + 96 lb = 112.38 lb
5
12
20 lb
104 lb
x
y
35
104 lb
12
20 lb
5
35
13
5
2
+ 12
2
= 13
(20 lb) sin 35 = 11.47 lb
(20 lb) cos 35 = 16.38 lb
13
5
(104 lb)( ) = 40 lb
(104 lb)( ) = 96 lb
13
12
R

y
= F

y
: R

y
= 11.47 lb 40 lb = 28.53 lb = 28.53 lb
(arrow indicates negative y direction)
+
+

98
2.2 Rectangular Components in Two-Dimensional Force Systems Example 9, page 2 of 2
3 Calculate the magnitude and direction of the resultant R.
x
y
R = (112.38 lb)
2
+ (28.53 lb)
2
= 115.9 lb
= tan
-1
= 14.2
112.38 lb
28.53 lb
112.38 lb
28.53 lb
R
14.2
115.9 lb
Ans.

99
2.2 Rectangular Components in Two-Dimensional Force Systems Example 10, page 1 of 2
1 Resolve the forces into x and y components.
10. Determine the magnitude and direction
of the resultant force acting on the beam.
2 Calculate the x and y components of the resultant by summing the
components of the given forces algebraically.
R

x
= F

x
: R

x
= 6.128 kN 12 kN = 5.872 kN = 5.872 kN
x
y
3
R

y
= F

y
: R

y
= 5.142 kN 9 kN + 11 kN = 3.142 kN = 3.142 kN +
+
40
8 kN
15 kN
11 kN
3
4
15 kN
4
11 kN
40
8 kN
3
5
(15 kN)( ) = 9 kN
(15 kN)( ) = 12 kN
5
5
4
(8 kN) sin 40 = 5.142 kN
(8 kN) cos 40 = 6.128 kN
Force points left
Force points down

100
2.2 Rectangular Components in Two-Dimensional Force Systems Example 10, page 2 of 2
3 Calculate the magnitude and direction of the resultant.
x
y
R = (5.872 kN)
2
+ (3.142 kN)
2

= 6.66 kN
= tan
-1
= 28.2
5.872 kN
3.142 kN
5.872 kN
3.142 kN
R
28.2
6.66 kN
Ans.

101
2.2 Rectangular Components in Two-Dimensional Force Systems Example 11, page 1 of 5
11. Determine the magnitude and direction
of the resultant force acting on the particle.
x
y
(5 m, 3 m)
25 N
80 N
50 N
(6 m, 2 m)
( m, 6 m)

102
2.2 Rectangular Components in Two-Dimensional Force Systems Example 11, page 2 of 5
1 We want to compute the x and y components of each
force. To do that, we first must compute some angles.
= tan
-1
= 26.57
6 m
3 m
80 N
y
x
3 m
6 m
Angle for
80-N force.
80 N
y
Components
of 80-N force.
x
= 26.57
(80 N) cos 26.57 = 71.55 N
(80 N) sin 26.57 = 35.78 N
( m, 6 m)

103
2.2 Rectangular Components in Two-Dimensional Force Systems Example 11, page 3 of 5
2 Angle and components for 50-N force.
= tan
-1
= 30.96
5 m
3 m
Angle for
50-N force
Components
of 50-N force.
= 30.96
(50 N) sin 30.96 = 25.72 N
(50 N) cos 30.96 = 42.88 N
50 N
x
y
5 m
3 m
x
y
50 N

104
2.2 Rectangular Components in Two-Dimensional Force Systems Example 11, page 4 of 5
3 Angle and components for 25-N force.
= tan
-1
= 18.43
6 m
2 m
y
x
6 m
Angle for
25-N force.
y
Components
of 25-N force.
x
(25 N) cos 18.43 = 23.72 N
(25 N) sin 18.43 = 7.90 N
25 N
(6 m, 2 m)
25 N
2 m
= 18.43

105
2.2 Rectangular Components in Two-Dimensional Force Systems Example 11, page 5 of 5
4 Sum the components algebraically.
5
R

x
= F

x
: R

x
= 35.78 N 42.88 N + 23.72 = 16.62 N
+
+
y
x
R

y
= F

y
: R

y
= 71.55 N 25.72 N 7.90 N = 37.93 N
35.78 N
42.88 N
23.72 N
71.55 N
25.72 N
7.90 N
Calculate the magnitude and direction of the resultant R.
R = (37.93 N)
2
+ (16.62)
2
= 41.4 N
= tan
-1
= 66.3
37.93 N
16.62 N
R
16.62 N
37.93 N
66.3
41.4 N
Ans.

106
2.2 Rectangular Components in Two-Dimensional Force Systems Example 12, page 1 of 3
1 Express the forces in x and y components (For clarity, the
components of the unknown force, F, are shown separately).
12. Three forces support the weight W shown. Determine
the value of F, given that the resultant of the three forces
is vertical. Also determine the value of W.
40
(20 N) sin 15 = 5.176 N
(20 N) cos 15 = 19.32 N
W
30
15
120 N
F
20 N
x
120 N
y
W
F
y
20 N
x
y
x
(120 N) sin 30 = 60 N
(120 N) cos 30 = 103.92 N
F sin 40
F cos 40
W
30
15
40

107
2.2 Rectangular Components in Two-Dimensional Force Systems Example 12, page 2 of 3
3 Use the fact that the resultant is known to be
vertical, so R

x
= 0.
2 Sum the components algebraically.
R

x
= F

x
: R

x
= 103.92 N 19.32 N F sin 40 (1)
R

y
= F

y
: R

y
= 60 N + 5.176 N + F cos 40 (2)
+
+
Eq. 1 becomes
R

x
= 103.92 N 19.32 N F sin 40
0
Solving gives
F = 131.61 N Ans.
x
y
W
R = R

y
R

x
= 0
Substitute this value of F into Eq. 2 and compute R

y
:
R

y
= 60 N + 5.176 N + F cos 40 = 166.0 N
131.61 N
4

108
2.2 Rectangular Components in Two-Dimensional Force Systems Example 12, page 3 of 3
5 The resultant upward force must balance the weight W, so
W = R

y
= 166.0 N Ans.
W
R = R

y
= 166.0 N

109
2.2 Rectangular Components in Two-Dimensional Force Systems Example 13, page 1 of 2
1 Express the forces in x and y components.
70
B
y
40
300 lb
A
x
B sin 70
B cos 70
(300 lb) sin 40 = 192.84 lb R

y
(y component of 300-lb resultant)
(300 lb) cos 40 = 229.81 lb R

x
(x component of 300-lb resultant)
300 lb (resultant, R)
A
B
40
70
y
x
13. The resultant, R, of the forces A and B acting on the
bracket is known to be a force of magnitude 300 lb
making an angle of 40 with the horizontal direction as
shown. Determine the magnitude of A and B.

110
2.2 Rectangular Components in Two-Dimensional Force Systems Example 13, page 2 of 2
4
Solving Eqs. 1 and 2 simultaneously gives:
A = 300 lb Ans.
B = 205 lb Ans.
2 R

x
is the algebraic sum of x components of A and B:
R

x
= F

x
: R

x
= A B cos 70 (1)
229.81 lb
Similarly for R

y
:
R

y
= F

y
: R

y
= B sin 70 (2)
192.84 lb
+
+
3

111
2.2 Rectangular Components in Two-Dimensional Force Systems Example 14, page 1 of 2
1 Express the forces in x and y components.
14. To support the 100-N block as shown, the resultant of
the 50-N force and the force F must be a 100-N force
directed horizontally to the right. Determine F and .
60
100 N
F
50 N
Pulley
F
60
50 N
x
y
F sin
F cos
(50 N) cos 60 = 25 N
(50 N) sin 60 = 43.30 N

112
2.2 Rectangular Components in Two-Dimensional Force Systems Example 14, page 2 of 2
2 Sum the components algebraically.
R

x
= F

x
: R

x
= F cos 25 N (1)
R

y
= F

y
: R

y
= F sin 43.30 N (2)
Because the resultant is known to be horizontal,
R

y
= 0, and the magnitude R is thus equal to the
horizontal component R

x
alone, that is, R = R

x
.
We also know, however, that the magnitude of
the resultant is 100 N, so R = R

x
= 100 N. Thus
Eqs. 1 and 2 become
100 N = F cos + 25 N (3)
0 = F sin 43.30 N (4)
The best way to solve these equations is to use a
calculator that can solve two simultaneous
nonlinear equations. Alternatively, solve Eq. 4
for F:
F = (5)

+
+
sin
43.30 N
Ans.
86.6 N
30
cos
sin
43.30 N
sin tan
1
tan
43.30 N
43.30 N
sin
3 And then substitute for F in Eq. 3:
100 N = F cos + 25 N
Substituting
=
gives
100 N = + 25 N
and solving gives
= 30.0
Using the result in Eq. 5 gives
F = = 86.6 N

113

2.3 Rectangular Components in Three-Dimensional Force Systems
114
2.3 Rectangular Components in Three-Dimensional Force Systems Procedures and Strategies, page 1 of 1
y
x
z
y
z x
F
Fx
y
x
z
F
F cos
(F sin ) cos
F sin
(F sin ) sin
Procedures and Strategies for Solving Problems Involving
Rectangular Components in Three-Dimensional Force Systems
1. If the magnitude F of a force and its direction angles, x
y
, and z
, are
known, then compute the components of the force from the equations
Fx = F cos x F
y
= F cos
y
Fz = F cos z
If only two angles are known, then find the third angle from the equation
cos
2

x + cos
2

y
+ cos
2

z = 1
2. If the magnitude F is known and the direction of the force is defined
through its projection on a horizontal plane, then compute the horizontal
components by projecting the projection onto the horizontal axes.
3. If the rectangular components Fx, F
y
, and Fz are known, then compute
the magnitude F of the force from the equation
F = Fx
2
+ F
y
2
+ Fz
2

and the direction angles from
cos x = Fx/F cos
y
= F
y
/F cos z = Fz/F
4. To compute the resultant of several force, express each force in
rectangular component and add the components:
Rx = Fx R
y
= F
y
Rz = Fz

115
2.3 Rectangular Components in Three-Dimensional Force Systems Problem Statement. for Example 1
F = 200 lb
O
z
y
x
60
45
120
1. Express the force F in terms of x, y, and z components.

116
2.3 Rectangular Components in Three-Dimensional Force Systems Problem Statement for Example 2
2. Express F in terms of x, y, and z components.
x
y
z
A
B
O
F = 50 N
40
35

117
2.3 Rectangular Components in Three-Dimensional Force Systems Problem Statement. for Example 3
3. Express F in terms of x, y, and z components.
x
y
z
A
B
O
F = 8 kN
70
25

118
2.3 Rectangular Components in Three-Dimensional Force Systems Problem Statement. for Example 4
4. Determine the x, y, and z components of the 26-N force shown.
Also determine the coordinate direction angles of the force.
13
5
12
20
F = 26 N
O
B
A
z
y
x

119
2.3 Rectangular Components in Three-Dimensional Force Systems Problem Statement. for Example 5
30
20
80
x
y
z
O
F

1
= 100 N
F

2
= 60 N
F

3
= 40 N
5. Determine the magnitude and coordinate direction angles
of the resultant of the three forces acting on the mast.

120
2.3 Rectangular Components in Three-Dimensional Force Systems Problem Statement. for Example 6
6. Determine the magnitude and coordinate direction angles
of the resultant of the forces acting on the eye-bolt.
x
y
z
O
F

1
= 650 N
F

2
= 800 N
F

3
= 300 N
30
70
50
12
5
13

121
2.3 Rectangular Components in Three-Dimensional Force Systems Problem Statement. for Example 7
7. A 300-lb vertical force is required to pull the pipe out of the ground. Determine
the magnitude and direction angles of the force F

2
which, when applied together
with the 150-lb force F

1
shown, will produce a 300-lb vertical resultant.
F

2
F

1
= 150 lb
O
z
y
x
60
60
45

122
2.3 Rectangular Components in Three-Dimensional Force Systems Problem Statement. for Example 8
8. Two forces, F

1
and F

2
act on the bracket as shown. If the
resultant of F

1
and F

2
lies in the xy plane, determine the
magnitude of F

2
. Also determine the magnitude of the resultant.
50
60
x
y
z
O
F

2
F

1
= 60 N

123
2.3 Rectangular Components in Three-Dimensional Force Systems Example 1, page 1 of 2
View of plane formed by the x axis and F.
1
O
x
F = 200 lb
The component points in the negative direction, so
F

x = 100 lb Ans.
5
d
4
3
O
x
F = 200 lb
Calculate the x component. 2
= 180 120 = 60
d = (200 lb) cos 60 = 100 lb
120
120
F = 200 lb
O
z
y
x
60
45
120
1. Express the force F in terms of x, y, and z components.

124
2.3 Rectangular Components in Three-Dimensional Force Systems Example 1, page 2 of 2
F = 200 lb
O
View of plane formed by the y axis and F 6
7
F

y
View of plane formed by the z axis and F. 8
F = F

xi + F

y
j + F

zk
= { 100i + 141.4j + 100k} lb Ans.
y component
F

y
= (200 lb) cos 45 = 141.4 lb Ans.
z component
F

z = (200 lb) cos 60 = 100 lb Ans.
10
y
z
45
60
F = 200 lb
O
9
Fz

125
2.3 Rectangular Components in Three-Dimensional Force Systems Example 2, page 1 of 2
2. Express F in terms of x, y, and z components.
x
y
z
A
B
O
F = 50 N
View of plane formed by OA, F, and the y axis.
F

y
= (50 N) cos 40 = 38.3 N Ans.
F

OA
= (50 N) sin 40 = 32.14 N
O
y
A
F = 50 N
F

y
1
2
3
40
40
F

OA
35

126
2.3 Rectangular Components in Three-Dimensional Force Systems Example 2, page 2 of 2
View of xz plane from above
O y
x
z
B
F

x
F

BA
32.14 N
F

BA
= (32.14 N) sin 35 = 18.4 N
F

x = (32.14 N) cos 35 = 26.3 N Ans.
4
6
5
F

z = 18.4 N 7
negative direction
F = F

xi + F

y
j + F

zk
= {26.3i + 38.3j 18.4k} N Ans.
8
A
35

127
2.3 Rectangular Components in Three-Dimensional Force Systems Example 3, page 1 of 2
3. Express F in terms of x, y, and z components.
x
y
z
A
B
O
F = 8 kN
70
F = 8 kN
O
y
A
F

OA
F

y
View of the plane formed by OA, F, and the y axis.
F

OA
= (8 kN) sin 70 = 7.518 kN
(8 kN) cos 70 = 2.74 kN
F

y
= 2.74 kN Ans.
Negative direction
1
2
3
4
70
25

128
2.3 Rectangular Components in Three-Dimensional Force Systems Example 3, page 2 of 2
7.518 kN
O
y
B
F

x
View of the xz plane from above.
F

x = (7.518 kN) cos 25 = 6.81 kN
F

z = (7.518 kN) sin 25
= 3.18 kN Ans.
z
x
F

z
5
6
7
F = F

xi + F

y
j + F

zk
= {6.81i 2.74j + 3.18k} kN Ans.
8
25

129
2.3 Rectangular Components in Three-Dimensional Force Systems Example 4, page 1 of 4
4. Determine the x, y, and z components of the 26-N force shown.
Also determine the coordinate direction angles of the force.
View of the plane formed by OA, F, and the y axis.
F

OA
= (26 N)( )

= 24 N
1
2
3
O A
F = 26 N
y
F

OA
F

y
12
13
F

y
= (26 N)( )

= 10 N Ans.
5
13
5
12
13
13
5
12
20
F = 26 N
O
B
A
z
y
x

130
2.3 Rectangular Components in Three-Dimensional Force Systems Example 4, page 2 of 4
x
y, O
z
A
24 N
F

z = (24 N) cos 20 = 22.6 N Ans.
F

x
(24 N) sin 20 = 8.21 N
F

x = 8.21 N Ans.
Negative direction
4
5
6
View of the xz plane as seen from above
O
A
z
B
F = 26 N
y
x
F

x = 8.21 N
Determine the x coordinate direction angle, .
View of the plane formed by the x axis and the force F.
7
8
O
26 N
8.21 N
x
The direction angle is measured
from the the positive part of the
axis. Here it is , not .
= 180
= 180 cos
-1
= 180 71.59
= 108.4 Ans.
9
10
8.21 N
26 N
20
F

z

131
2.3 Rectangular Components in Three-Dimensional Force Systems Example 4, page 3 of 4
O
A
z
B
F = 26 N
y
x
F

y
= 10 N
Determine the y coordinate direction angle, .
View of the plane formed by the y axis and the force F.
O
26 N
= cos
-1
= 67.4 Ans.
10 N
26 N
11
12
10 N
y
13

132
2.3 Rectangular Components in Three-Dimensional Force Systems Example 4, page 4 of 4
O
A
z
B
F = 26 N
y
x
F

z = 22.6 N
Determine the z coordinate direction angle, .
View of the plane formed by the z axis and the force F.
O
26 N
= cos
-1
= 29.6 Ans.
22.6 N
26 N
14
15
16
22.6 N
z
Observation: the calculations for , , and can be
summarized by the general formulas
cos =
cos =
cos =
The algebraic signs of F

x, F

y
, and F

z must be included
when using these formulas.
17
F

x
F
F

y

F
F

z
F

133
2.3 Rectangular Components in Three-Dimensional Force Systems Example 5, page 1 of 3
5. Determine the magnitude and coordinate direction angles
of the resultant of the three forces acting on the mast.
30
20
80
x
y
z
O
F

1
= 100 N
F

2
= 60 N
F

3
= 40 N
x
y
F

1
= 100 N
z
20
O
Express F

1
in rectangular components.
(100 N) sin 80 = 98.48 N
F
1x
= (98.48 N) sin 20 = 33.68 N
F
1z
= (98.48 N) cos 20 = 92.54 N
F
1y
= (100 N) cos 80 = 17.36 N
1
2
3
4
5
6 In vector form,
F

1
= {33.68i 17.36j + 92.54k} N (1)
80

134
2.3 Rectangular Components in Three-Dimensional Force Systems Example 5, page 2 of 3
30
x
y
O
F

2
= 60 N
F

3
= 40 N
Express F

2
and F

3
in rectangular components.
F

2y
= (60 N) sin 30 = 30 N
F

2x
= (60 N) cos 30 = 51.96 N
In vector form,
F

2
= {51.96i + 30j} N (2)
F

3
= {40j} N (3)
7
F

3y
= 40 N
8
9
10
11
Use Eqs. 1, 2, and 3 to compute the resultant, R.
R = F

1
+ F

2
+ F

3
= {33.68i 17.36j + 92.54k} N + {51.96i + 30j} N + {40j} N
= {33.68 + 51.96}i N + { 17.36 + 30 + 40}j N + {92.54k} N
= {85.64i + 52.64j + 92.54k} N
Collect coeffiecients
of i, j, and k.
12
13

135
2.3 Rectangular Components in Three-Dimensional Force Systems Example 5, page 3 of 3
x
y
z
O
R
Coordinate direction angles
cos = =
cos = =
cos = =
Solving for the angles gives
= 51.2 Ans.
= 67.4 Ans.
= 47.4 Ans.
R

z
R
R

y

R
R

x
Magnitude of resultant
R = (85.64 N)
2
+ (52.64 N)
2
+ (92.54 N)
2
= 136.6 N Ans.
14
15
92.54 N
52.64 N
85.64 N
136.6 N
136.6 N
136.6 N

136
2.3 Rectangular Components in Three-Dimensional Force Systems Example 6, page 1 of 4
F

1
= 650 N
F

2
= 800 N
F

3
= 300 N
30
70
50
12
5
13
x
12
13
5
F

1
= 650 N
y
O
z
(650 N)( ) = 600 N
F

1x
= (600 N) cos 30 = 519.6 N
F

1z
= (600 N) sin 30 = 300 N
Express F

1
in rectangular components. 1
3
5
F

1y
= (650 N)( ) = 250 N
4
2
12
13
13
5
In vector form,
F

1
= { 519.6i + 250j 300k} N (1)
6
30
6. Determine the magnitude and coordinate direction angles
of the resultant of the forces acting on the eye-bolt.
x
y
z
O

137
2.3 Rectangular Components in Three-Dimensional Force Systems Example 6, page 2 of 4
F

2
= 800 N
50
F

2x
= (800 N) cos 50 = 514.2 N
F

2z
= (800 N) cos 70 = 273.6 N
Express F

2
in rectangular components.
70

2
7
Since we are not given

2
, we cannot compute F

2y
from the formula
F

2y
= F

2
cos

2
But since we know the magnitude F

2
= 800 N, we can solve for F

2y

from the formula for the magnitude of a vector in terms of its
rectangular components:
F

2
= F

2x
2
+ F

2y
2
+ F

2z
2
Substituting for F

2
, F

2x
, and F

2z
gives
(800 N) = (514.2 N)
2
+ F

2y
2
+ (273.6 N)
2
and solving for F

2y
gives
F

2y
= 548.4 N
Alternatively, we could have solved for

2
by using the identity
satsified by the direction angles:
(cos 50 )
2
+ (cos

2
)
2
+ (cos 70 )
2
= 1
Solving gives

2
= 46.73 and thus
F

2y
= (800 N) cos 46.73 = 548.3 N
This is the same result as before, allowing for round-off error.
8
9
10
x
y
z
O

138
2.3 Rectangular Components in Three-Dimensional Force Systems Example 6, page 3 of 4
x
y
z
O
Also, in vector form, F

3
is
F

3
= {300k} N (3)
F

3
= 300 N
13
Use Eqs. 1, 2, and 3, to compute the resultant R.
R = F

1
+ F

2
+ F

3
= { 519.6i + 250j 300k} N + {514.2i + 548.4j + 273.6k} N + {300k} N
= { 519.6 + 514.2}i N + {250 + 548.4}j N + { 300 + 273.6 + 300}k N
= { 5.4i + 798.4j + 273.6k} N
14
12 In vector form, F

2
can be written as
F

2
= {514.2i + 548.4j + 273.6k} N (2)
11
y
F

2y
x
F

2
z
O
The figure shows that F

2y
is
positive, so choose the plus sign:
F

2y
= +548.4 N

139
2.3 Rectangular Components in Three-Dimensional Force Systems Example 6, page 4 of 4
Coordinate direction angles
cos = =
cos = =
cos = =
Solving gives
= 90.4 Ans.
= 18.9 Ans.
= 71.1 Ans.
R
R

z
R

y

R
844.0 N
844.0 N
798.4 N
273.6 N
R

x
R 844.0 N
5.4 N
16
O
z
y
x
15
Magnitude of resultant
R = ( 5.4 N)
2
+ (798.4 N)
2
+ (273.6 N)
2

= 844.0 N Ans.

140
2.3 Rectangular Components in Three-Dimensional Force Systems Example 7, page 1 of 3
7. A 300-lb vertical force is required to pull the pipe out of the ground. Determine
the magnitude and direction angles of the force F

2
which, when applied together
with the 150-lb force F

1
shown, will produce a 300-lb vertical resultant.
45
60
60
x
y
z
O
F

1
= 150 lb
F

2
F

1
= 150 lb
z

1
= 60
O
x

1
= 60
45
y

1
= 180 45 = 135
(

1
measured from positive z)
Express F

1
in rectangular components
F

1x
= F

1
cos

1
= (150 lb) cos 60 = 75 lb
F

1y
= F

1
cos

1
= (150 lb) cos 60 = 75 lb
F

1z
= F

1
cos

1
= (150 lb) cos 135 = 106.1 lb
1
In vector form,
F

1
= {75i + 75j 106.1k} lb (1)
2

141
2.3 Rectangular Components in Three-Dimensional Force Systems Example 7, page 2 of 3
Express F

2
in terms of components,
F

2
= F

2x
i + F

2y
j + F

2z
k (2)
Use Eqs. 1 and 2 to compute the resultant,
R = F

1
+ F

2
= {75i + 75j 106.1k} lb
+ {F

2x
i + F

2y
j + F

2z
k}
= {75 lb + F

2x
}i + {75 lb + F

2y
}j
+ { 106.1 lb + F

2z
}k (3)
Now we use the fact that R is known to be vertical, directed upwards
with a magnitude of 300 lb:
R = 0i + 300j + 0k
So Eq. 3 becomes
0i + 300j + 0k ={75 lb + F

2x
}i + {75 lb + F

2y
}j
+ { 106.1 lb + F

2z
}k
3
Equating coefficients of i gives
0 = 75 lb + F

2x
Solving gives
F

2x
= 75 lb (4)
Equating coefficients of j, and then k, gives
300 = 75 lb + F

2y
0 = 106.1 lb + F

2z
Solving gives
F

2y
= 225 lb (5)
F

2z
= 106.1 lb (6)
4

142
2.3 Rectangular Components in Three-Dimensional Force Systems Example 7, page 3 of 3
75 lb
F

2x

Coordinate direction angles
cos

2
= =
cos

2
= =
cos

2
= =
Solving gives


2
= 106.8 Ans.


2
= 30.0 Ans.


2
= 65.9 Ans.
F

2

106.1 lb
259.8 lb
259.8 lb
259.8 lb
5
F

2y

F

2

F

2z

F

2

225 lb
z
y
O
F

2
x

2

2

2
Magnitude of F

2
F

2
= (F

2x
)
2
+ (F

2y
)
2
+ (F
2z
)
2

=


= 259.8 lb
( 75 lb)
2
+ (225 lb)
2
+ (106.1 lb)
2
ns.

143
2.3 Rectangular Components in Three-Dimensional Force Systems Example 8, page 1 of 3
8. Two forces, F

1
and F

2
act on the bracket as shown. If the
resultant of F

1
and F

2
lies in the xy plane, determine the
magnitude of F

2
. Also determine the magnitude of the resultant.
50
60
x
y
z
O
F

2
F

1
= 60 N

144
2.3 Rectangular Components in Three-Dimensional Force Systems Example 8, page 2 of 3
Express F

1
in rectangular components:
F

1x
= F

1
cos

1
= (60 N) cos 130 = 38.57 N (1)
F

1y
= F

1
cos

1
= (60 N) cos 60 = 30 N (2)
Since we do not know the value of

1
, we cannot compute F

1z

from
F

1z
= F

1
cos

1
Instead we can use the equation for the magnitude of F

1
:
F

1
= (F

1x
)
2
+ (F

1y
)
2
+ (F

1z
)
2

60 N = ( 38.57 N)
2
+ (30 N)
2
+ (F
1z
)
2
Solving gives
F

1z
= 34.82 N
1

1
= 60
z
O
x
50
F

1
= 60 N
y

1
= 180 50 = 130
F

1x
F

1y
F

1z
F

1z
points in the negative direction so
choose the minus sign
F

1z
= 34.82 N (3)
In vector form, from Eqs. 1, 2, and 3,
F

1
= { 38.57i + 30j 34.82k} N (4)
2
3

145
2.3 Rectangular Components in Three-Dimensional Force Systems Example 8, page 3 of 3
Now, Eq. 4 and the vector form of F

2
,
F

2
= F

2
k
can be used to compute the resultant, R:
R = F

1
+ F

2
= { 38.57i + 30j 34.82k} N + F

2
k (5)
Because the resultant R lies in the x-y plane,
R

z = 0 (6)
Combining Eqs. 5 and 6 gives
R

xi + R

y
j + 0k ={ 38.57i + 30j + (F

2
34.82)k} N
Equating coefficients of i gives
R

x = 38.57 N
Equating coefficients of j gives
R

y
= 30 N
4
Equating coefficients of k gives
0 = F

2
34.82 N
or
F

2
= 34.82 N Ans.
Finally, the magnitude of R is
R = (R

x)
2
+ (R

y
)
2
+ (R

z)
2

= ( 38.57 N)
2
+ (30 N)
2
+ (0)
2

= 48.9 N Ans.
5

146

2.4 Position Vectors. Use in Defining Force Vectors.
147
2.4 Position Vectors. Use in Defining Force Vectors. Procedures and Strategies, page 1 of 2
y
x
z
r
AB
A( 1 m, 5 m, 4 m)
B(5 m, 9 m, 8 m) Procedures and Strategies for Solving Problems
Involving Position Vectors.
1. The components of the position vector from point A to
point B can be found in two ways.
a) Subtract the coordinates of point A from the
coordinates of point B. Note that the tail coordinates
must be subtracted from the head coordinates.
b) Determine the movement required in each coordinate
direction to go from point A to point B.
r
AB
= [5 ( 1) i +(9 5)j + ( 8 4)k

= {6i + 4j 12k} m
1. Move 6 m in the +x direction (x component of +6).
2. Move 4 m in the +y direction (y component of +4).
3. Move 12 m in the z direction (z component of 12).

148
2.4 Position Vectors. Use in Defining Force Vectors. Procedures and Strategies, page 2 of 2
u
2. If the line of action of a force of magnitude F passes
through two points, A and B, then the position
vector from A to B can be used to find the
components of the force vector by using the
following procedure:
a) Express the position vector from A to B in
rectangular component form (If the force points
from B to A, then the position vector should
point from B to A).
b) Find a unit vector parallel to the position vector
by dividing each component of the position
vector by its magnitude.
c) Multiply each component of the unit vector by F
to find the component of the force.
F = 42 N
y
x
z
A( 1 m, 5 m, 4 m)
B(5 m, 9 m, 8 m)
y
x
z
r
AB
= {6i + 4j 12k} m
(Shown above)
F
AB
= 42u
AB
= 42( i + j k)
= {18i + 12j 36k} N
7 7 7
3 2 6
r
AB
= (6)
2
+ (4)
2
+ ( 12)
2
= 14 m
u
AB
= = = i + j k
r
AB
6i + 4j 12k
r
AB 14 7 7 7
3 2 6
A(- 1 m, 5 m, 4 m)
B(5 m, 9 m, 8 m)

149
2.4 Position Vectors. Use in Defining Force Vectors. Problem Statement for Example 1
y
A (3, 4)
B (11, 10)
1. Determine a) the position vector directed from point A to point B and
b) the position vector directed from point B to point A.
x

150
2.4 Position Vectors. Use in Defining Force Vectors. Problem Statement for Example 2
2. Determine the position vector directed from point B to
point A. Also determine the distance between B and A.
2 m
4 m
7 m
1 m
6 m
x
A
B
y
z
3 m

151
2.4 Position Vectors. Use in Defining Force Vectors. Problem Statement for Example 3
4 m
3 m
z
3. Determine a) the position vector r

AB
from A to B, b) unit vector u in
the same direction, and c) the direction cosines of r

AB
.
50
2 m
30
B
8 m
A
y
x

152
2.4 Position Vectors. Use in Defining Force Vectors. Problem Statement for Example 4
30 m
y
x
z
6 m
4 m
A
B
C
D
4. The radio tower is supported by three guy wires. If the tension in
wire CD is 1.6 kN, determine the components of the force from CD
acting on the tower at D. Also determine the coordinate direction
angles of the force vector.

153
2.4 Position Vectors. Use in Defining Force Vectors. Problem Statement for Example 5
5. The awning is supported by cables CD
and AB. If the tension in CD is 80 lb,
determine the components of the force
from the cable acting on the awning at C.
3 ft
2 ft
A
B
C
D
5 ft
y
x
z

154
2.4 Position Vectors. Use in Defining Force Vectors. Problem Statement for Example 6
O
Radius = 1.5 m
z
x
D
C
B
120
120
2 m
A
6. A cylindrical reactor vessel is being lifted by a crane connected to
three 2-m long cables. If the tension in cable AC is 2 kN, determine
the components of the force exerted by the cable on the vessel at C.
y

155
2.4 Position Vectors. Use in Defining Force Vectors. Problem Statement for Example 7
7. Strut EF is employed as a temporary support to help retaining wall ABCD
resist soil pressure. If the compressive force in the strut is F

FE
= 3 kip,
determine the components of the force from the strut acting on the wall at E.
8 ft
3 ft
6 ft
A B
C D
z
y
x
E
F
Soil pressure

156
2.4 Position Vectors. Use in Defining Force Vectors. Problem Statement for Example 8
25 in.
14 in. 14 in.
18 in.
18 in.
A
B
C
D
x
y
z
8. If the axial compression forces in the legs of the tripod cooking-stand
are denoted F

A
, F

B
, and F

C
respectively, determine the resultant force
acting on the connection at D in terms of F

A
, F

B
, and F

C
.

157
2.4 Position Vectors. Use in Defining Force Vectors. Problem Statement for Example 9
45
60
60
A
B
C
D
x
y
z
3 m
8 m
8 m
3 m
20 m
12 m
E
9. The 20-m boom DCE is supported by a ball-and-socket joint at D and two
cables attached at the midpoint C of the boom. If the tension in each cable is
500 N and the coordinates direction angles of the boom are 60, 45, and 60
as shown, determine the resultant of the cable forces acting on the boom.

158
2.4 Position Vectors. Use in Defining Force Vectors. Problem Statement for Example 10
2 ft 2 ft
x
4 ft 4 ft 4 ft
A B
C
D E
F
O
y
z
10. A 12-ft wide movie screen is suspended in a
corner of a room by four wires. The ends C and F of
the screen are located the same distance, L, from the
corner. If the tension in wires DO and DA are 12 lb
and 8 lb respectively, determine the resultant force
from the wires acting on handle D of the screen.
L L

159
2.4 Position Vectors. Use in Defining Force Vectors. Example 1, page 1 of 2
1. Determine a) the position vector directed from point A to point B and
b) the position vector directed from point B to point A.
x
y
x
y
A (3, 4)
B (11, 10)
r

AB
i
j
A (3, 4)
B (11, 10)
Part a)
Draw the vector from A to B.
1
r

AB
= B's coordinates minus A's coordinates
= (x

B
x

A
)i + (y

B
y

A
)j
= (11 3)i + (10 4)j
= 8i + 6j Ans.
2

160
2.4 Position Vectors. Use in Defining Force Vectors. Example 1, page 2 of 2
x
y
r

BA
i
j
A (3, 4)
B (11, 10)
Part b)
Draw the vector from B to A.
r

BA
= A's coordinates minus B's coordinates
= (3 11)i + (4 10)j
= 8i 6j Ans.
3
4
Observation: Since r

AB
equals the negative value of r

BA
, it is easy to
make a mistake and get r

BA
when you intended to calculate r

AB
. The
rule to remember is "Subtract the coordinates of the tail of the vector."
5

161
2.4 Position Vectors. Use in Defining Force Vectors. Example 2, page 1 of 2
2. Determine the position vector directed from point B to
point A. Also determine the distance between B and A.
2 m
4 m
7 m
1 m
6 m
x
A
B
y
z
3 m

162
2.4 Position Vectors. Use in Defining Force Vectors. Example 2, page 2 of 2
2 m
4 m
7 m
1 m
6 m
x
A
B
y
z
Determine the coordinates of A and B. 1
( 4 m, 2 m, 7 m)
(6 m, 3 m, 1 m)
Draw the vector from B to A.
Subtract tail coordinates (point B) from head coordinates (point A).
r

BA
= ( 4 m 6 m)i + [2 m ( 3 m)]j + [7 m ( 1 m)]k
= { 10i + 5j + 8k} m Ans.
r

BA
2
3
Distance between B and A = magnitude of r

BA
= ( 10)
2
+ (5)
2
+ (8)
2
= 13.75 m Ans.
4
Three-dimensional
Pythagorean theorem
3 m

163
2.4 Position Vectors. Use in Defining Force Vectors. Example 3, page 1 of 4
30
B
8 m
A
y
x
4 m
3 m
z
3. Determine a) the position vector r

AB
from A to B, b) unit vector u in
the same direction, and c) the direction cosines of r

AB
.
30
B
8 m
A
y
x
3 m
z
4 m
2 m
(8 m) sin 30 = 4 m
(4 m) sin 50 = 3.064 m
(4 m) cos 50 = 2.571 m
Coordinates of B: ( 3.064 m, 6.928 m, 2.571 m)
Coordinates of A: (4 m, 2 m, 3 m) 7
2
3
4
5
6 1 Determine coordinates of A and B.
50
(8 m) cos 30 = 6.928 m
50
2 m

164
2.4 Position Vectors. Use in Defining Force Vectors. Example 3, page 2 of 4
y
x
z
B ( 3.064 m, 6.928 m, 2.571 m)
A (4 m, 2 m, 3 m)
Draw r

AB
8
Determine the components of r

AB
by subtracting tail coordinates (point A)
from head coordinates (point B):
r

AB
= ( 3.064 m 4 m)i + (6.928 m 2 m)j + [2.571 m ( 3 m)]k
= { 7.064i + 4.928j + 5.571k} m (1) Ans.
9
point A
point B
y
x
z
A
B
r

AB
= { 7.064i + 4.928j + 5.571k} m
Next, determine the unit vector in the same direction as r

AB
.
The best way is to use a calculator with a built-in function for
calculating unit vectors. You simply input r

AB
and the
calculator gives you back the components of the unit vector.
10

165
2.4 Position Vectors. Use in Defining Force Vectors. Example 3, page 3 of 4
y
x
z
A
B
r

AB
If you do not have such a calculator, proceed as follows:
Calculate r

AB
, the length of r

AB
.
r

AB
= ( 7.064)
2
+ (4.928)
2
+ (5.571)
2
= 10.26 m (2)
11
u

AB
r

AB

r

AB

{ 7.064i + 4.928j + 5.571k} m
10.26 m
Divide by r

AB
to get the unit vector in the same direction.
u

AB
=
=
= { i + j + k
= 0.689i + 0.480j + 0.543k (3) Ans.
7.064 m
10.26 m
4.928 m
10.26 m
5.571 m
10.26 m
12
Finally, to determine the directions cosines, recall the
definition of the x direction cosine of r

AB
:
cos =
=
= 0.689
But this is the x component of the unit vector, by Eq. 3:
u

AB
= 0.689i + 0.480j + 0.543k (3)
x component of r

AB

magnitude of r

AB

7.064 m
10.26 m
13
Eq. 1
Eq. 2

166
2.4 Position Vectors. Use in Defining Force Vectors. Example 3, page 4 of 4
Thus in general the components of a unit vector are also the direction
cosines of any vector pointing in the same direction as the unit vector.
For our particular problem, since
u

AB
= 0.689i + 0.480j + 0.543k,
the direction cosines of r

AB
are
cos = 0.689 Ans.
cos = +0.480 Ans.
cos = +0.543 Ans.
Obviously, we could use these equations to solve for the coordinate
direction angles , , and .
14

167
2.4 Position Vectors. Use in Defining Force Vectors. Example 4, page 1 of 5
4. The radio tower is supported by three guy wires. If the tension in
wire CD is 1.6 kN, determine the components of the force from CD
acting on the tower at D. Also determine the coordinate direction
angles of the force vector.
y
x
z
6 m
4 m
A
B
C
D
30 m

168
2.4 Position Vectors. Use in Defining Force Vectors. Example 4, page 2 of 5
6 m
4 m
A
B
C
Since the force in the wire is tension,
the force from the wire acting on the
tower at D is directed from D to C .
Draw the position vector in the
same direction as the force.
1
2
r

DC
F = 1.6 kN
x
z
6 m
4 m
B
C
r

DC
F = 1.6 kN
Determine the coordinates 3
( 4 m, 0, 6 m)
(0, 30 m, 0)
y
D
y
D
30 m
30 m
x
z

169
2.4 Position Vectors. Use in Defining Force Vectors. Example 4, page 3 of 5
x
6 m
4 m
A
B
C
r

DC
F = 1.6 kN
( 4 m, 0, 6 m)
(0, 30 m, 0)
Determine the components of r

DC
by subtracting tail
coordinates (point D) from head coordinates (point C):
r

DC
= ( 4 m 0)i + (0 30 m)j + (6 m 0)k
= { 4i 30j + 6k} m
point D
point C
4
y
D
30 m

170
2.4 Position Vectors. Use in Defining Force Vectors. Example 4, page 4 of 5
x
z
6 m
4 m
A
B
C
r

DC
F = 1.6 kN
( 4 m, 0, 6 m)
(0, 30 m, 0) Determine the unit vector by dividing r

DC
by its length.
r

DC
= { 4i 30j + 6k} m
r

DC
= ( 4)
2
+ ( 30)
2
+ (6)
2


= 30.85 m
u

DC
=
=
= 0.130i 0.972j + 0.194k
r

DC

r

DC

4i 30j + 6k
30.85
5
u
The force vector has the same direction as u and also has a
magnitude of 1.6 kN, so
F = (1.6 kN)u
= (1.6 kN)( 0.130i 0.972j + 0.194k)
= { 0.207i 1.556j + 0.311k} kN Ans.
6
y
D
30 m

171
2.4 Position Vectors. Use in Defining Force Vectors. Example 4, page 5 of 5
y
x
z
C
D
r

DC
u
To find the coordinate direction angles of F, recall that the
components of a unit vector are the direction cosines of all vectors
pointing in the same direction as the unit vector. For our particular
problem, since u (= 0.130i 0.972j + 0.194k) and F are in the
same direction, we have for the direction angles , , and ,
cos = 0.130
cos = 0.972
cos = 0.194
Solving gives
= 97.5 Ans.
= 166.5 Ans.
= Ans.
7
F

172
2.4 Position Vectors. Use in Defining Force Vectors. Example 5, page 1 of 4
3 ft
2 ft
A
B
C
D
5 ft
5. The awning is supported by cables CD
and AB. If the tension in CD is 80 lb,
determine the components of the force
from the cable acting on the awning at C.
y
x
z

173
2.4 Position Vectors. Use in Defining Force Vectors. Example 5, page 2 of 4
y
x
z
A
B
C
D
5 ft
3 ft
r

CD
F = 80 lb
Since the cable is in tension, the cable force acting on
point C of the awning is directed from C to D. The
position vector r

CD
points in the same direction.
1
2 ft

174
2.4 Position Vectors. Use in Defining Force Vectors. Example 5, page 3 of 4
y
x
z
A
B
C
D
5 ft
3 ft
Determine the unit vector in the
direction of r

CD
.
u

CD
=
=
= 0.811i + 0.487j
0.324k
r

CD

r

CD

{ 5i + 3j 2k}
( 5)
2
+ (3)
2
+ ( 2)
2
5
r

CD
F = 80 lb
( 5 ft)i
u

CD
2 ft
We cannot determine the coordinates of C and D because
we are given no distances from the coordinate axes.
2
?
However, we can still determine the
components of r

CD
because we are given the
x, y, and z differences in the coordinates of C
and D.
r

CD
= ( 5 ft)i + (3 ft)j + ( 2 ft)k
3
(3 ft)j
The figure shows that r

CD
points in
the negative x direction, so the x
component is negative. Analogously,
the y component is positive and the z
component negative.
4
( 2 ft)k
Length
of r

CD

175
2.4 Position Vectors. Use in Defining Force Vectors. Example 5, page 4 of 4
The force vector has the same direction as the unit vector and
also has a magnitude of 80 lb, so
F = (80 lb)u

CD
= (80 lb)( 0.811i + 0.487j 0.324k)
= { 64.9i + 38.9j 26.0k} lb Ans.
6

176
2.4 Position Vectors. Use in Defining Force Vectors. Example 6, page 1 of 3
O
Radius = 1.5 m
z
x
D
C
B
120
120
2 m
A
6. A cylindrical reactor vessel is being lifted by a crane connected to
three 2-m long cables. If the tension in cable AC is 2 kN, determine
the components of the force exerted by the cable on the vessel at C.
y

177
2.4 Position Vectors. Use in Defining Force Vectors. Example 6, page 2 of 3
Since the cable is in tension, the force F
CA
of the cable
acting on the vessel at C is directed from C to A. The
position vector r

CA
is also directed from C to A.
1
(0, y

A
, 0)
A
O
2 m
C
Radius = 1.5 m
Determine the coordinates of A 2
y

A
= (2 m)
2
(1.5 m)
2
= 1.323 m
y

A
r

CA
F

CA
= 2 kN
A
2 m
120
120
B
D
x
z
Radius = 1.5 m
O
y
D
3
Top view of vessel
(1.5 m) cos 30 = 1.299 m
x

C
= 1.299 m
(1.5 m) sin 30 = 0.75 m
z

C
= 0.75 m
z
x
B
y, A, O
C
Radius = 1.5 m
120 90 = 30
120
C (x

C
, 0, z

C
)
Determine the coordinates of C.

178
2.4 Position Vectors. Use in Defining Force Vectors. Example 6, page 3 of 3
(0, 1.323 m, 0)
C (1.299 m, 0, 0.75 m)
5
( 1.299)
2
+ (1.323)
2
+ (0.75)
2
1.299i + 1.323j + 0.75k
The force F

CA
is given by its magnitude, 2 kN, multiplied by
the unit vector in the direction of r

CA
:
F

CA
= (2 kN) ( )
= { 1.299i + 1.323j + 0.75k} kN Ans.
4 Determine r

CA
by subtracting coordinates of C from A:
r

CA
= (0 1.299 m)i + (1.323 m 0)j + [0 ( 0.75 m)]k
= { 1.299i + 1.323j + 0.75k} m
y
O
Radius = 1.5 m
z
x
D
B
120
120
2 m
A
F

CA
= 2 kN
r

CA
Divide by the magnitude of r
CA

to obtain the unit vector.
(Actually we could have saved
work here by noting that the
magnitude of r
CA
is already
known: the length of cable CA
was given as 2 m.)
6

179
2.4 Position Vectors. Use in Defining Force Vectors. Example 7, page 1 of 3
7. Strut EF is employed as a temporary support to help retaining wall ABCD
resist soil pressure. If the compressive force in the strut is F

FE
= 3 kip,
determine the components of the force from the strut acting on the wall at E.
8 ft
3 ft
6 ft
A B
C D
z
y
x
E
F
Soil pressure

180
2.4 Position Vectors. Use in Defining Force Vectors. Example 7, page 2 of 3
A B
C D
z
y
x
Soil pressure
Since the force in the strut is compressive, the force from the
strut acting on point E is directed from F to E. The position
vector r

FE
is then chosen in the same direction as the force.
1
r

FE
3 ft
6 ft
8 ft
E
F

FE
= 3 kip
F

181
2.4 Position Vectors. Use in Defining Force Vectors. Example 7, page 3 of 3
( 3)
2
+ (6)
2
+ ( 8)
2
3i + 6j 8k
Since the force has magnitude 3 kip and direction given by the
unit vector in direction of r

FE
, we have:
F

FE
= (3 kip) ( )
= { 0.862i + 1.724j 2.299k} kip Ans.
4
r

FE
F

FE
= 3 kip
A B
C D
z
y
x
Soil pressure
3 ft
F (?, ?, ?)
6 ft
8 ft
E
(6 ft)j
( 8 ft)k
( 3 ft)i
We don't know the coordinates of E and F
so we can't subtract coordinates to get r

FE
.
2
Even though we don't know the coordinates
of points E and F, we do know the differences
in these coordinates, so we can obtain the
components of r

FE
directly from the
dimensions.
r

FE
= { 3i + 6j 8k} ft
3

182
2.4 Position Vectors. Use in Defining Force Vectors. Example 8, page 1 of 5
25 in.
14 in. 14 in.
18 in.
18 in.
A
B
C
D
x
y
z
8. If the axial compression forces in the legs of the tripod cooking-stand
are denoted F

A
, F

B
, and F

C
respectively, determine the resultant force
acting on the connection at D in terms of F

A
, F

B
, and F

C
.

183
2.4 Position Vectors. Use in Defining Force Vectors. Example 8, page 2 of 5
Because the forces in the legs are compressive, the forces from
the legs acting on the connection at D are directed towards D.
1
Draw the position vectors in the same direction as the forces. 2
14 in. 14 in.
18 in.
18 in.
A
B
C
D
x
y
r

AD
r

BD
r

CD
F

A
F

B
F

C
z
25 in.

184
2.4 Position Vectors. Use in Defining Force Vectors. Example 8, page 3 of 5
(0, 25 in., 0)
(14 in., 0, 18 in.)
( 14 in., 0, 0)
(14 in., 0, 18 in.)
r

AD
r

BD
r

CD
F

A
F

B
F

C
25 in.
14 in. 14 in.
18 in.
18 in.
A
B
C
D
x
y
z
Determine the coordinates of the ends of the position vectors. 3

185
2.4 Position Vectors. Use in Defining Force Vectors. Example 8, page 4 of 5
14 in. 14 in.
18 in.
18 in.
A
B
C
D
x
y
z
(0, 25 in., 0)
(14 in., 0, 18 in.)
( 14 in., 0, 0)
(14 in., 0, 18 in.)
Subtract coordinates to get position vectors.
r

CD
= (0 14 in.)i + (25 in. 0)j + [0 ( 18 in.)]k
= { 14i + 25j + 18k} in.
4
5
r

BD
= (0 14 in.)i + (25 in. 0)j + [0 18 in.)]k
= { 14i + 25j 18k} in.
r

AD
= [0 ( 14 in.)]i + (25 in. 0)j + (0 0)k
= {14i + 25j} in.
6
7
r

AD
r

BD
r

CD
F

A
F

B
F

C
25 in.

186
2.4 Position Vectors. Use in Defining Force Vectors. Example 8, page 5 of 5
Force vectors = magnitude unit vector
F

A
= F

A
( )
= 0.489 F

A
i + 0.873 F

A
j
F

B
= F

B
( )
= 0.414 F

B
i + 0.739 F

B
j 0.532 F

B
k
F

C
= F

C
( )
= 0.414 F

C
i + 0.739 F

C
j + 0.532 F

C
k
14i + 25j
(14)
2
+ (25)
2
14i + 25j 18k
( 14)
2
+ (25)
2
+ ( 18)
2
14i + 25j + 18k
( 14)
2
+ (25)
2
+ (18)
2
8
Resultant = sum of forces
R = F

A
+ F

B
+ F

C
= 0.489 F

A
i + 0.873 F

A
j
0.414 F

B
i + 0.739 F

B
j 0.532 F

B
k
0.414 F

C
i + 0.739 F

C
j + 0.532 F

C
k
9
Re-arrange by combining coefficients of i, then of j, and then of k:
R = (0.489 F

A
0.414 F

B
0.414 F

C
)i
+ (0.873 F

A
+ 0.739 F

B
+ 0.739 F

C
)j
+ ( 0.532 F

B
+ 0.532 F

C
)k Ans.
10

187
2.4 Position Vectors. Use in Defining Force Vectors. Example 9, page 1 of 5
45
60
60
A
B
C
D
x
y
z
3 m
8 m
8 m
3 m
20 m
12 m
E
9. The 20-m boom DCE is supported by a ball-and-socket joint at D and two
cables attached at the midpoint C of the boom. If the tension in each cable is
500 N and the coordinates direction angles of the boom are 60, 45, and 60
as shown, determine the resultant of the cable forces acting on the boom.

188
2.4 Position Vectors. Use in Defining Force Vectors. Example 9, page 2 of 5
60
C
D
x
y
z
8 m
8 m
E
1
45
r

D
r

C
r

DC
To obtain the coordinates of C, we
can use the vector sum equation:
r

C
= r

D
+ r

DC
(1)
From the figure, we see
r

D
= {8i + 8k} m (2)
2
3
To obtain r

DC
, recall the definition of
coordinate direction angles, for example, :
cos =
4
x component of r

DC

magnitude of r

DC

60
To compute the resultant of the cable forces, we need to
express them in rectangular components. But to do that,
we first need to obtain the coordinates of point C.

189
2.4 Position Vectors. Use in Defining Force Vectors. Example 9, page 3 of 5
Solving the equation for the x component of r

DC
gives
x component of r

DC
= cos 60 (magnitude of r

DC
)
or,
r

DCx
= 5 m (3)
Similarly
r

DCy
= cos 45 (10 m)
= 7.071 m (4)
r

DCz
= cos 60 (10 m)
= 5 m (5)
5
= = 10 m,
(Divide by 2 because point
C is the midpoint of the
20-m boom)
20 m
2
Eqs. 2-5 can now be used in Eq. 1:
r

C
= r

D
+ r

DC
= {8i + 8k} m + {5i + 7.071j + 5k} m
= {13i + 7.071j + 13k} m (6)
6

190
2.4 Position Vectors. Use in Defining Force Vectors. Example 9, page 4 of 5
A
B
C
D
x
y
z
3 m
3 m
12 m
E
(0, 12 m, 3 m)
(13 m, 7.071 m, 13 m)
r

CA
F

CA
= 500 N
r

CB
F

CB
= 500 N
Since the cables are in tension, the cable forces acting on the boom
are directed away from C. Position vectors r

CA
and r

CB
are
similarly drawn away from C towards the supports at A and B.
7
Determine the coordinates of A.
The coordinates of C are the
components of r

C
, Eq. 6.
8
9
To obtain r

CA
, subtract tail coordinates from head coordinates:
r

CA
= (0 13 m)i + (12 m 7.071 m)j + (3 m 13 m)k
= { 13i + 4.929j 10k} m
10

191
2.4 Position Vectors. Use in Defining Force Vectors. Example 9, page 5 of 5
A
B
C
D
x
y
z
3 m
3 m
12 m
E
(0, 12 m, 3 m)
(13 m, 7.071 m, 13 m)
r

CA
F

CA
= 500 N
r

CB
F

CB
= 500 N
The force in cable CA is then
F

CA
= (500 N) ( )
= { 379.5i + 143.9j 292.0k} N
13i + 4.929j 10k
( 13)
2
+ (4.929)
2
+ ( 10)
2

11
(3 m, 12 m, 0)
To obtain the components of F

CB
, first determine the
coordinates of B. Then subtract coordinates of C from
coordinates of B to get r

CB
:
r

CB
= (3m 13 m)i + (12 m 7.071 m)j + (0 13 m)k
= { 10i + 4.929j 13k} N
and so
F

CB
= (500 N) ( )
= { 292.0i + 143.9j 379.5k} N (8)
12
10i + 4.929j 13k
( 10)
2
+ (4.929)
2
+ ( 13)
2

From Eqs. 7 and 8, the resultant of the cable forces acting
at C is
R = F

CA
+ F

CB
= { 379.5i + 143.9j 292.0k} N
+ { 292.0i + 143.9j 379.5k} N
= { 672i + 288j 672k} N Ans.
13
Equal, as we would expect from symmetry. 14

192
2.4 Position Vectors. Use in Defining Force Vectors. Example 10, page 1 of 8
2 ft 2 ft
x
4 ft 4 ft 4 ft
A B
C
D E
F
O
y
z
10. A 12-ft wide movie screen is suspended in a
corner of a room by four wires. The ends C and F of
the screen are located the same distance, L, from the
corner. If the tension in wires DO and DA are 12 lb
and 8 lb respectively, determine the resultant force
from the wires acting on handle D of the screen.
L L

193
2.4 Position Vectors. Use in Defining Force Vectors. Example 10, page 2 of 8
To compute the resultant of the forces, we
must express them in rectangular components.
But to do that, we first have to find the
coordinates of point D.
1
4 ft 4 ft 4 ft
x
2 ft 2 ft
L L
z
y
O
F
E D
C
B A
To obtain the coordinates of point D,
use the vector sum equation
r

OD
= r

OC
+ r

CD
(1)
2
r

OC
r

CD
r

OD

194
2.4 Position Vectors. Use in Defining Force Vectors. Example 10, page 3 of 8
3 4 ft = 12 ft
3
Since L = (12 ft) cos 45 = 8.485 ft,
r

OC
= { 2j + 8.485k} ft (2)
4
L
L
45
45
O, y
B, F
A, C
x
z
4 ft 4 ft 4 ft
x
2 ft 2 ft
L L
z
y
O
F
E D
C
B A
r

OC
r

CD
r

OD
First, locate point C, using a view
looking down on the xz plane:

195
2.4 Position Vectors. Use in Defining Force Vectors. Example 10, page 4 of 8
4 ft 4 ft 4 ft
x
2 ft 2 ft
L L
z
y
O
F
E D
C
B A
r

OC
r

OD
To find the components of r

CD
, note that it is has magnitude 4 ft,
is parallel to the xz plane (so has no y component) and makes a
45 angle with the z axis. Thus
r

CD
= (4 ft)(cos 45i sin 45k)
= {2.828i 2.828k} ft (3)
5
r
CD
View looking down on the xz plane
45
O, y
B, F
A, C
x
z
r
CD
45
D

196
2.4 Position Vectors. Use in Defining Force Vectors. Example 10, page 5 of 8
Now, determine r

OD
from Eq. 1:
r

OD
= r

OC
+ r

CD
(Eq. 1 repeated)
by Eq. 2 by Eq. 3
= { 2j + 8.485k} ft
+ {2.828i 2.828k} ft
= {2.828i 2j + 5.657k} ft (4)
6
4 ft 4 ft 4 ft
x
2 ft
z
y
O
F
E D
C
B A
r
OD
8.485 ft
8.485 ft
2 ft
r
OC
r
CD

197
2.4 Position Vectors. Use in Defining Force Vectors. Example 10, page 6 of 8
Next express the forces in component form. Begin by
showing the force F
DO
directed away from the
connection D, because the cable DO is in tension.
F
DO
can be expressed in terms of a unit vector parallel
to the position vector r
DO
. The position vector r
DO

has the opposite sense of the position vector r
OD
, so
r

DO
= r

OD
. Thus, taking into account that the
magnitude of F
DO
was given as 12 lb, we have
F
DO
= (12 lb)
= (12 lb)
by Eq. 4
(12 lb)
= { 5.116i + 3.618j 10.234k} lb (5)
7
4 ft 4 ft 4 ft
x
2 ft 2 ft
z
y
O
F
E D
B A
r
DO
F
DO
C
(2.828i 2j + 5.657k)
(2.828)
2
+ ( 2)
2
+ (5.657)
2

r
DO

r
DO

r
OD

r
OD

198
2.4 Position Vectors. Use in Defining Force Vectors. Example 10, page 7 of 8
Next find the components of the force F
DA
of
cable DA acting on handle D. Since F
DA
is in
the same direction as the position vector r
DA
,
we first find the components of r
DA
:
r
DA
= coordinates of point A
coordinates of point D
by Eq. 4
= {8.485k} {2.828i 2j + 5.657k}
= { 2.828i + 2j + 2.828k} ft
F

DA
can be expressed in terms of a unit vector
parallel to the position vector r

DA
. Thus,
taking into account that the magnitude of F

DA

was given as 8 lb, we have
F

DA
= (8 lb)
(8 lb)
= { 5.059i + 3.578j + 5.059k} (6)
8
4 ft 4 ft 4 ft
x
2 ft
z
y
O
F
E D
C
B
A
8.485 ft
r

DA
2.828i + 2j + 2.828k
( 2.828)
2
+ (2)
2
+ (2.828)
2

r
DA
r
DA

F
DA
Coordinates of point
A: (0, 0, 8.485 ft)
2 ft

199
2.4 Position Vectors. Use in Defining Force Vectors. Example 10, page 8 of 8
9 The resultant is the sum of the forces:
R = F

DA
+ F
DO
by Eq. 6 by Eq. 5
= { 5.059i + 3.578j + 5.059k} lb + { 5.116i + 3.618j 10.234k} lb
= { 10.18i + 7.20j 5.17k} lb Ans.

200

2.5 Applications of Dot Products

201
2.5 Applications of Dot Products Procedures and Strategies, page 1 of 1
u
F
F u = (F)(1) cos = Fu
B
A
B
A
Original position Position after B has been
moved
Procedures and Strategies for Solving Problems Involving
Application of Dot Products
1. Compute the angle between two vectors by using the
formula
cos
-1


is the angle between the vectors when they are placed
tail-to-tail. The dot product is calculated by using
rectangular components:
A B = AxBx + A
y
B
y
+ AzBz
2. Compute the projection, say, Fu, of a force F in a
direction specified by a unit vector u by using the formula
Fu = F u
A vector representation of the projection is given by Fuu.
A B
AB

202
2.5 Applications of Dot Products Problem Statement for Example 1
1. Two lines intersect in the plane as shown.
Determine a) the angle , and b) the angle .
B (4 m, 12 m)
A (9 m, 11 m)
C (5 m, 6 m)
x
y

203
2.5 Applications of Dot Products Problem Statement for Example 2
x
y
z
A
B
C
2. A billboard is braced in back by struts AB and AC.
Determine the angle between the struts.
4 m
10 m
3 m
7 m

204
2.5 Applications of Dot Products Problem Statement for Example 3
3. A bin is constructed by cutting off the corner of a box. Top
ABC is glued to the box, and lid ADC pivots about hinge AC. To
make the lid, we need to know the values of the angles at A, D,
and C. Determine these angles.
x
y
z
30 mm
A
B
C
D
hinge
180 mm
200 mm
125 mm
125 mm

205
2.5 Applications of Dot Products Problem Statement for Example 4
4. Determine the angle between the forces.
x
y
z
(7 m, 3 m, 6 m)
F

1
= 800 N
F

2
= 300 N
70
25

206
2.5 Applications of Dot Products Problem Statement for Example 5
5. A crate is suspended from supports at B, C, and D. The tension in cable AB is
known to be F = 800 N. Determine
a) the angle between cables AB and AC;
b) the component F

AC
of the 800-N force that acts in the direction of cable AC;
c) the component F

AD
of the 800-N force that acts in the direction of cable AD; and
d) the component of the 800-N force that acts in the vertical direction.
3.5 m
4 m
2 m
A
B
C
D
x
y
z
F = 800 N
9 m
3 m
7 m
5 m

207
2.5 Applications of Dot Products Problem Statement for Example 6
y
x
z
B
A
F = {4j} kN
2 m
6 m
5 m
6. End B of the beam shown is subjected to an upward force F = 4 kN. Determine the
components of F parallel and perpendicular to the long axis of the beam, AB.

208
2.5 Applications of Dot Products Problem Statement for Example 7
450 mm
200 mm
400 mm
A
B
C
D
F = 60 N
P
z
x
y
30
300 mm
7. When a certain load P is applied to
the bar BCD, a tension of F = 60 N is
produced in cable DA. Determine the
components of F parallel and
perpendicular to segment CD. Neglect
the thickness of the bar.

209
2.5 Applications of Dot Products Example 1, page 1 of 3
Draw position vectors a and b.
1
B (4 m, 12 m)
A (9 m, 11 m)
C (5 m, 6 m)
x
y
B (4 m, 12 m)
C (5 m, 6 m)
y
x
A (9 m, 11 m)
a
b
Calculate components.
a = (9 m 5 m)i + (11 m 6 m)j
= {4i + 5j} m (1)
b = (4 m 5 m)i + (12 m 6 m)j
= { i + 6j} m (2)
2
1. Two lines intersect in the plane as shown. Determine a) the angle , and b) the angle
.

210
2.5 Applications of Dot Products Example 1, page 2 of 3
Writing the dot product in terms of rectangular
components gives
ab = a

xb

x + a

y
b

y
Using this in the numerator of Eq. 4 gives
cos = (5)
Recalling Eqs. 1 and 2,
a = {4i + 5j} m (Eq. 1 repeated)
b = { i + 6j} m (Eq. 2 repeated)
we can evaluate Eq. 5 as
cos =
Solving gives
= 48.1 Ans.
Apply the definition of the vector dot product:
ab = ab cos (3)
Solving gives
cos
ab
ab
3
Angle between tails of vector a and b
a

xb

x + a

y
b

y

ab
4
2
+ 5
2
( 1)
2
+ 6
2
(4)( 1) + (5)(6)
4
can now be calculated:
= 180 48.1
= 131.9 Ans.
5
C (5 m, 6 m)
B (4 m, 12 m)
y
b
x
A (9 m, 11 m) a
= 48.1

211
2.5 Applications of Dot Products Example 1, page 3 of 3
We could have used the dot product to calculate directly, but to do
so, we must define a new vector, a' a. This is because the dot
product formula gives us the angle between the tails of the vectors:
cos = (6)
Using
a' = a,
= { 4i 5j} m
and
b = { i + 6j} m
in Eq. 6 gives
cos =
Solving gives
= 131.9 as before.
(a')b
a b
( 5)
2
+ ( 4)
2
( 1)
2
+ 6
2
( )( 1) + ( )(6)
6
C (5 m, 6 m)
B (4 m, 12 m)
y
b
x
A (9 m, 11 m)
a
a' = a
= 48.1

212
2.5 Applications of Dot Products Example 2, page 1 of 3
x
y
z
A
B
C
2. A billboard is braced in back by struts AB and AC.
Determine the angle between the struts.
4 m
10 m
3 m
7 m

213
2.5 Applications of Dot Products Example 2, page 2 of 3
x
y
z
A
B
C
1 Introduce position vectors r

AC
and r

AB
.
Determine their rectangular components.

r

AB
= (0 7 m)i + (4 m 0)j + (0 3 m)k
= { 7i + 4j 3k} m
r

AC
= (10 m 7 m)i + (4 m 0)j + (0 3 m)k
= {3i + 4j 3k} m
2
r

AB
r

AC
4 m
10 m
3 m
7 m

214
2.5 Applications of Dot Products Example 2, page 3 of 3
Use the vector dot product equation:
cos =
=
=
= 0.0797
3
2
+ 4
2
+ ( 3)
2
( 7)
2
+ 4
2
+ ( 3)
2
Solving cos = 0.0797 gives
= 85.4 Ans.
r

AC
r

AB

r

AC
r

AB
{3i + 4j 3k}{ 7i + 4j 3k}
(3)( 7) + (4)(4) + ( 3)( 3)
3
Multiply x component by x
component, y by y, and z by z.
4
6
5
34 74
This is a tedious and error-prone calculation. A much better
approach is to use a calculator with built-in functions for
evaluating the dot product and magnitude (called the "norm"
function in some calculators). If such a calculator is available, all
you need to do is enter once the components for r

AC
and the
components for r

AB
. The built-in functions perform all the
arithmetic. As a result, typical errors such as missing a minus sign
or mis-copying a number from one line to the next are avoided.

215
2.5 Applications of Dot Products Example 3, page 1 of 3
3. A bin is constructed by cutting off the corner of a box. Top
ABC is glued to the box, and lid ADC pivots about hinge AC. To
make the lid, we need to know the values of the angles at A, D,
and C. Determine these angles.
x
y
z
30 mm
A
B
C
D
Hinge
Introduce position vectors r

DA
and r

DC
Determine their rectangular components.
r

DA
= (0 180 mm)i + (125 mm 30 mm)j
= { 180i + 95j} mm (1)
r

DC
= (125 mm 30 mm)j + (0 200 mm)k
= {95j 200k} mm (2)
1
2
x
y
z
30 mm
A
B
C
D
r

DA
r

DC
180 mm
200 mm
180 mm
200 mm
125 mm

D
125 mm
125 mm
125 mm

216
2.5 Applications of Dot Products Example 3, page 2 of 3
x
y
z
30 mm
A
B
C
D
r

AD
r

AC
Use the vector dot product equation:
cos

D
=
=
=
= 0.2003
( 180)
2
+ 95
2
95
2
+ ( 200)
2
r

DA
r

DC

r

DA
r
DC
{ 180i + 95j}{95j 200k}
( 180)(0) + (95)(95) + (0)( 200)
3
(203.5)(221.4)
Solving gives


D
= 78.45 (3) Ans.
4
5 Next determine the angle at A. Begin by
introducing position vectors r

AC
and r

AD
.
Note that we use vector r

AD
,
not r

DA
, because the dot
product formula gives the
angle between the tails of
the vectors. If we use r

DA

and r

AC
, we would get the
supplement of

A
, not

A
.
6
A
r

AC
r

DA
125 mm
125 mm

A

A
= 180

A
200 mm
180 mm
D
C

217
2.5 Applications of Dot Products Example 3, page 3 of 3
Use the vector dot product equation:
cos

A
=
=
=
= 0.5917
Solving gives


A
= 53.72 (6) Ans.
180
2
+ ( 200)
2
180
2
+ ( 95)
2
r

AC
r

AD

r

AC
r
AD
{180i 200k}{180i 95j}
(180)(180) + (0)( 95) + ( 200)(0)
(269.1)(203.5)
Determine the rectangular components of r

AC
and r

AD
.
r

AC
= (180 mm 0)i + (0 200 mm)k
= {180i 200k} mm (4)
r

AD
= r

DA
= { 180i + 95j} mm
= {180i 95j} mm (5)
7
8
Finally, we could compute

C
, the angle at C, by
using position vectors and the dot product, but it's
easier to use the fact that the angles of a triangle add
to 180:


A
+

D
+

C
= 180
or


C
= 180

A


D
= 47.8 Ans.
78.45 by Eq. 3
53.72 by Eq. 6

218
2.5 Applications of Dot Products Example 4, page 1 of 4
70
25
4. Determine the angle between the forces.
x
y
z
(7 m, 3 m, 6 m)
F

1
= 800 N
F

2
= 300 N

219
2.5 Applications of Dot Products Example 4, page 2 of 4
x
y
z
F

1
= 800 N
Determine the components of F

1
. 1
F

1y
= (800 N) cos 70
= 273.6 N
(800 N) sin 70 = 751.8 N
2
3
F

1x
= (751.8 N) sin 25
= 317.7 N
4
F

1z
= (751.8 N) cos 25
= 681.4 N
5
In component form
F

1
= { 317.7i + 273.6j + 681.4k} N (1)
6
70
25

220
2.5 Applications of Dot Products Example 4, page 3 of 4
Determine the components of F

2
.
A(7 m, 3 m, 6 m)
y
x
7
z
F

2
= 300 N
O
F

2
= (300 N) unit vector pointing from O to A.
= (300 N)
= {216.6i + 92.8j + 185.7k} N (2)
7
2
+ 3
2
+ 6
2
{7i + 3j 6k}
8
Using Eqs. 1 and 2 (repeated here),
F

1
= { 317.7i + 273.6j + 681.4k} N (1)
F

2
= {216.6i + 92.8j + 185.7k} N (2)
we can evaluate from the vector dot product equation
cos = (3)
The formula for the magnitude of F

1
, appearing in the denominator of
Eq. 3 is
F

1
= ( 317.7)
2
+ (273.6)
2
+ (681.4)
2
but we don't have to evaluate this expression because we were initially
given F

1
= 800 N (see steps 2 - 6 above). Similarly, we were initially
given F

2
= 300 N. Using these observations together with Eqs. 1 and 2
in Eq. 3 gives
cos =
=
Solving gives = 69.7 Ans.
9
F

1
F

2

F

1
F

2

F

1
F

2

F

1
F

2

(800)(300)
( 317.7)(216.6) + (273.6)(92.8) + (681.4)(185.7)

221
2.5 Applications of Dot Products Example 4, page 4 of 4
10
F

1

F

1

F

2

F

2

Observation: Eq. 3 can be rearranged to give
cos =
= ( )( )
= (unit vector in F

1
direction)(unit vector in F

2
direction)
That is, the magnitudes of the forces, 800 N and 300 N, divide out.
We could have saved some multiplications by simply replacing "800
N" by "1 N" and "300 N" by "1 N" at the beginning of the problem.
F

1
F

2

F

1
F

2


222
2.5 Applications of Dot Products Example 5, page 1 of 7
3.5 m
4 m
2 m
A
B
C
D
x
y
z
F = 800 N
9 m
3 m
7 m
5 m
5. A crate is suspended from supports at B, C, and D. The tension in cable AB is
known to be F = 800 N. Determine
a) the angle between cables AB and AC;
b) the component F

AC
of the 800-N force that acts in the direction of cable AC;
c) the component F

AD
of the 800-N force that acts in the direction of cable AD; and
d) the component of the 800-N force that acts in the vertical direction.

223
2.5 Applications of Dot Products Example 5, page 2 of 7
3.5 m
4 m
A
B
C
D
x
y
z
F = 800 N
Introduce position vectors r

AB
and r

AC
.
r

AC
r

AB
Determine their rectangular components
r

AB
= ( 3 m 0)i + [0 ( 9 m)]j + (3.5 m 0)k
= { 3i + 9j + 3.5k} m (1)
r

AC
= ( 0)i + [4 m ( 9 m)]j + ( 5 m 0)k
= {13j 5k} m (2)
1
2
3 Use the vector dot product equation:
cos

BC
=
=
=
= 0.7065
( 3)
2
+ 9
2
+ 3.5
2
13
2
+ ( 5)
2
r

AB
r

AC

r

AB
r

AC
{ 3i + 9j + 3.5k}{13j 5k}
( 3)(0) + (9)(13) + (3.5)( 5)
102.25 194
Solving gives


BC
= 45.1 Ans. (3)
4
9 m
3 m
5 m

224
2.5 Applications of Dot Products Example 5, page 3 of 7
3 m
5 m
7 m
F

AC
= (800 N) cos 45.1
= 565 N Ans.
3.5 m
4 m
2 m
A
B
C
D
x
y
z
F = 800 N
To calculate F

AC
, the component of F in the direction
of AC, consider the plane formed by AB and AC:
5
F

AC
F

AC
F = 800 N
C
A
B
9 m

BC
= 45.1

225
2.5 Applications of Dot Products Example 5, page 4 of 7
3 m
7 m
5 m
3.5 m
4 m
A
B
C
D
x
y
z
F = 800 N
To calculate F

AD
, the component of F in the direction
of AD, consider the plane formed by AB and AD:
F

AD
A
B
u

AD
6
F = 800 N
F

AD
u

AD
D
7 From the figure, we see that
F

AD
= F cos

BD
(4)

BD
9 m
2 m

226
2.5 Applications of Dot Products Example 5, page 5 of 7
But we don't know

BD
. We can, however, still use Eq. 4 by
introducing the dot product:
F

AD
= F cos
BD
(4)
= F 1 cos
BD
= F u
AD
cos
BD
= Fu
AD
(5)
= F

xu
ADx
+ F

y
u
ADy
+ F

zu
ADz
(6)
Thus we can calculate F
AD
, if we express F and u
AD
in
rectangular components.
8
magnitude of unit vector in AD direction
insert a factor of 1
definition of dot product

227
2.5 Applications of Dot Products Example 5, page 6 of 7
3 m
7 m
5 m
3.5 m
4 m
2 m
A
B
C
D
y
z
F = 800 N
F

AD
u

AD
r

AD
Since F points from A to B, we can express it as
F = (800 N) unit vector pointing from A to B
= (800 N)
= (800 N)
= { 237.3i + 712.0j + 276.9k} N (6)
( 3)
2
+ 9
2
+ 3.5
2
r

AB
r

AB
3i + 9j + 3.5k
r

AB
9
by Eq. 1
To obtain u

AD
in component form, introduce the
position vector from A to D,
r

AD
= (7 m 0)i + [0 ( 9)]j + ( 2 m 0)k
= {7i 9j 2k} m
Then
u

AD
=
=
= 0.605i + 0.777j 0.173k (7)
10
7
2
+ 9
2
+ ( 2)
2

r

AD
r

AD
7i + 9j 2k
x
9 m

228
2.5 Applications of Dot Products Example 5, page 7 of 7
Substituting for F and u
AD
from Eqs. 6 and 7 in Eq. 5 gives
F

AD
= F

xu
ADx
+ F

y
u
ADy
+ F

zu
ADz
(6)
= ( 237.3)(0.605) + (712.0)(0.777) + (276.9)( 0.173)
= 362 N Ans.
Finally, to find the vertical component of F, we can use the equation
F

vertical
= Fu
vertical
= [{ 237.3i + 712.0j + 276.9k} N]j
= 712 N Ans.
Of course, we could have simply read this value directly from the j
component of F. The dot product equation was not needed.
Nevertheless, it is worthwhile to confirm that the general principle
component = F(unit vector in direction of component)
gives the x, y, and z components of F when the unit vectors are i, j,
and k respectively.
11
The unit vector in the vertical
direction is the base vector, j.

229
2.5 Applications of Dot Products Example 6, page 1 of 4
y
x
z
B
A
F = {4j} kN
2 m
6 m
5 m
6. End B of the beam shown is subjected to an upward force F = 4 kN. Determine the
components of F parallel and perpendicular to the long axis of the beam, AB.

230
2.5 Applications of Dot Products Example 6, page 2 of 4
Resolve F into components parallel (F

| |
) and
perpendicular (F ) to the AB direction.
1
y
x
z
B
F = {4 j} kN
A
F
F

| |
Introduce a unit vector pointing in the direction
from A to B.
Then F

| |
(the component of F in
the direction of u) is given by
F

| |
= Fu (1)
2
3
u

231
2.5 Applications of Dot Products Example 6, page 3 of 4
Express u in rectangular component form by first introducing
the position vector r

AB
.
y
x
z
B
A
u
4
r

AB
2 m
5 m
Now find the components of u
u =
=
= 0.744i + 0.620j + 0.248k (2)
6
2
+ 5
2
+ 2
2
r

AB
r

AB
6i + 5j + 2k
5
Eq. 1 now becomes
F

| |
= Fu (1)
F

| |
= [{4j} kN]{0.744i + 0.620j + 0.248k}
= (0)(0.744) + (4)(0.620) + (0)(0.248)
or
F

| |
= 2.480 kN (3) Ans.
6
6 m

232
2.5 Applications of Dot Products Example 6, page 4 of 4
y
x
z
B
A
F
F
F

| |
F can be computed by the Pythagorean theorem.
F = F
2
F

| |
2
or
F = 3.14 kN Ans.
7
2.480 kN by Eq. 3
4 kN (given)
Finally, we can express F and F

| |
in rectangular component
form if we note first that F

| |
is parallel to u so that
F

| |
= F

| |
u
= (2.480 kN){0.744i + 0.620j + 0.248k}
= {1.845i + 1.538j + 0.615k} kN (4) Ans.
8
We get F from the vector sum
F = F

| |
+ F
so
F = F F

| |
= {4j} kN {1.845i + 1.538j + 0.615k} kN

= { 1.845i + 2.462j 0.615k} kN Ans.
A scalar (number) multiplying a vector
9
F
F
F

| |
u from Eq. 2
B

233
2.5 Applications of Dot Products Example 7, page 1 of 6
450 mm
200 mm
400 mm
A
B
C
D
F = 60 N
P
z
x
y
30
300 mm
7. When a certain load P is applied to
the bar BCD, a tension of F = 60 N is
produced in cable DA. Determine the
components of F parallel and
perpendicular to segment CD. Neglect
the thickness of the bar.

234
2.5 Applications of Dot Products Example 7, page 2 of 6
Resolve F into components parallel (F

| |
) and
perpendicular (F ) to the CD direction.
1
F

| |
F
Introduce a unit vector u pointing
in the direction from C to D.
Then F

| |
(the component in the
direction parallel to u) is given by
F

| |
= Fu (1)
2
3
u
450 mm
200 mm
400 mm
A
B
C
D
F = 60 N
P
z
x
y
30
300 mm

235
2.5 Applications of Dot Products Example 7, page 3 of 6
r

DA
To express F in rectangular components,
first introduce the position vector r

DA
.
4
Determine the x coordinate of D:
400 mm + (300 mm) cos 30 = 659.8 mm
5
Determine the y
coordinate of D
(neglect the thickness
of the member BCD):
6
450 mm
200 mm
A
B
C
D
F = 60 N
P
z
x
y
(300 mm) sin 30 = 150 mm
30
300 mm
(300 mm) cos 30
400 mm

236
2.5 Applications of Dot Products Example 7, page 4 of 6
D (659.8 mm, 150 mm, 0)
Express r

DA
in rectangular components
r

DA
= (0 659.8 mm)i + (450 mm 150 mm)j + (200 mm 0)k
= { 659.8i + 300j + 200k} mm
7
u r

CD
To express the unit vector u in rectangular
components, first introduce the position vector
r

CD
from C to D:
r

CD
= (659.8 mm 400 mm)i
+ (150 mm 0)j + (0 0)k
= {259.8i + 150j} mm
9
The force F has magnitude 60 N and points from D to A so
F = (60 N)
= (60 N)
= { 52.65i + 23.94j + 15.96k} N (2)
( 659.8)
2
+ 300
2
+ 200
2
659.8i + 300j + 200k
r

DA
r

DA
8
450 mm
200 mm
400 mm
A
B
C
F = 60 N
P
z
x
y
r

DA
30
300 mm

237
2.5 Applications of Dot Products Example 7, page 5 of 6
Now find the components of u
u =
=
= 0.866i + 0.500j (3)
259.8
2
+ 150
2
r

CD
r

CD
259.8i + 150j
Eq. 1 now becomes
F

| |
= Fu
F

| |
= { 52.65i + 23.94j + 15.96k}{0.866i + 0.500j}
= 33.63 N (4) Ans.
10
11
by Eq. 2
by Eq. 3
Minus sign means F

| |
is in the
opposite direction to u.
F
The magnitude of F can be computed
by the Pythagorean theorem.
F = 60
2
33.63
2
= 49.7 N Ans.
12
F

| |
= 33.63 N
D
F = 60 N
u

238
2.5 Applications of Dot Products Example 7, page 6 of 6
F
u
F

| |
F
We can express F

| |
and F in rectangular component
form, if we note first that F

| |
is parallel to u so that
F

| |
= F

| |
u
= ( 33.63 N){0.866i + 0.500j}
= { 29.12i 16.82j} N (5) Ans.
by Eq. 3
by Eq. 4
13
Finally we can get F from the vector sum
F = F

| |
+ F
Thus
F = F F

| |
= { 52.65i + 23.94j + 15.96k} N
{ 29.12i 16.82j} N
= { 23.5i + 40.8j + 16.0k} N Ans.
14
A
B
C
D
P
z
x
y
30
300 mm

239

3. Equilibrium of a Particle
240

3.1 Particles and Two-Dimensional Force Systems
241
3.1 Particles and Two-Dimensional Force Systems Procedures and Strategies, page 1 of 1
Procedures and Strategies for Solving Problems Involving Equilibrium
of a Particle in a Two-Dimensional Force System
1. Establish x and y coordinates.
2. Construct a free-body diagram showing and labeling all forces acting on
the particle, including the weight, if the weight or the mass is given.
Assign the sense of forces arbitrarily (If you guess wrong, the magnitude of
the force will be found to be negative, when you solve the equations).
3. Resolve each force into x and y components and set the sum of each
component to zero:
Fx = 0 F
y
= 0
4. Solve the equations. Note that if a force is found to be negative, it has a
sense opposite to that shown on the free-body diagram.
5. If a spring is present, it exerts a force F = kx, where k is the spring
constant and x is the displacement of the end of the spring.

242
3.1 Particles and Two-Dimensional Force Systems Problem Statement for Example 1
1. Determine the tension in cables AB and BC.
90 lb
30
A
B
C

243
3.1 Particles and Two-Dimensional Force Systems Problem Statement for Example 2
1.5 m
2 m
3 m
2 m 4 m
A
B
C
D
Spring
balance
2. The spring balance reads 500 N. Determine the tensions in cords AB and BD.


244
3.1 Particles and Two-Dimensional Force Systems Problem Statement for Example 3
3. The 10-kg block is supported by two identical
springs. The unstretched length of each spring is
500 mm. Calculate the spring constant k.
C
B
A
10 kg
40 40
600 mm

245
3.1 Particles and Two-Dimensional Force Systems Problem Statement for Example 4
60 lb
150 lb
A
B
C
D
4. Determine the value of angle and the
tension in the cables required for equilibrium.
10
20

246
3.1 Particles and Two-Dimensional Force Systems Problem Statement for Example 5
5. The collar at B weighs 5 lb and can slide freely
over the rod. If spring constant k is 2 lb/in.,
determine the unstretched length of the spring.
14 in.
A
B
C
D
20
35

247
3.1 Particles and Two-Dimensional Force Systems Problem Statement for Example 6
F F


6. Two 2-g spheres are suspended by light cords and then given
electrostatic charges of opposite sign. Calculate the attractive
force, F, acting between the spheres.
A
B C
D
30 30

248
3.1 Particles and Two-Dimensional Force Systems Problem Statement for Example 7
4 in.
s
Top view
Side view
k = 5 lb/in.
0.5 lb
0.5 lb
A
A
B
7. The figure shows top and side views of a control knob A that slides
smoothly in a slot cut in a panel and is attached to a spring underneath the
panel. Determine the force in the spring and the distance s, for which the
knob will be in equilibrium when a 0.5-lb force is applied. The
unstretched length of the spring is 4 in., and the weight of the knob is
negligible.

249
3.1 Particles in Two-Dimensional Force Systems Example 1, page 1 of 1
30
30
180 30 90 = 60
F
BC
F
AB
90 lb
y
x
A
B
C
2
1
+
+
30
1. Determine the tension in cables AB and BC.
Free-body diagram of connection B
Equilibrium equations for connection B
F

x = 0: F

AB cos 30 + F

BC cos 60 = 0
F

y = 0: F

AB sin 30 + F

BC sin 60 90 lb = 0
Solving these equations gives
F

AB = 45.0 lb Ans.
F

BC = 77.9 lb Ans.
90 lb

250
3.1 Particles in Two-Dimensional Force Systems Example 2, page 1 of 2
1.5 m
2 m
3 m
2 m 4 m
A
B
C
D
Spring
balance
2. The spring balance reads 500 N. Determine the tensions in cords AB and BD.


251
3.1 Particles in Two-Dimensional Force Systems Example 2, page 2 of 2
2 m
4 m
2 m
3.5 m
( )
( )
2 m +1.5 m
= 3.5 m
F

BD
Force in spring =
tension of 500 N.
F

AB
Equilibrium equations for connection B
F

x = 0 F

AB
cos + (500 N) cos = 0 (1)
F

y
= 0: F

AB
sin + (500 N) sin F

BD
= 0 (2)
4 Substituting 60.26 and = 26.57 in
Eqs. 1 and 2 and solving gives
F

AB
= 901 N Ans.
F
BD
= 1006 N Ans.
x
2
+
C
2 m
= tan
-1
= 60.26
= tan
-1
= 26.57
+
B
y
1 Free body diagram of connection B
3
A
Geometry
B
2 m 4 m

252
3.1 Particles in Two-Dimensional Force Systems Example 3, page 1 of 2
3. The 10-kg block is supported by two identical
springs. The unstretched length of each spring is
500 mm. Calculate the spring constant k.
Weight = mg = (10 kg)(9.81 m/s
2
) = 98.1 N
40 40
Free body diagram of connection C
F
AC
1
C
x
BC
F
y
Equilibrium equations for connection C:
F

x = 0 F

AC
cos 40 + F

BC
cos 40 = 0 (1)
F

y
= 0: F

AC
sin 40 + F

BC
sin 40 98.1 N = 0 (2) +
2
+
C
B
A
10 kg
40 40
40 40
600 mm

253
3.1 Particles in Two-Dimensional Force Systems Example 3, page 2 of 2
4
40
B
C
600 mm
BC =
600 mm
sin 40
= 933.4 mm
5 F

BC
= force in spring BC
= k (extension of spring)
= k (433.4 mm) (3)
6 Solving Eqs. (1), (2), and (3) gives
F

AC
= 76.3 N = F

BC
k = 0.176 N/mm = 0.176 kN/m Ans.
Geometry
Extension of spring = stretched length unstretched length
= 933.4 mm 500 mm
= 433.4 mm
3

254
3.1 Particles in Two-Dimensional Force Systems Example 4, page 1 of 2
60 lb
150 lb
A
B
C
D
4. Determine the value of angle and the
tension in the cables required for equilibrium.
2
+
+
60 lb
Free body diagram of connection B


1
B
x
F
BC
y
Equilibrium equations for connection B:
F

x = 0: F

AB
cos 10 + F

BC
cos 20 = 0 (1)
F

y
= 0: F

AB
sin 10 + F

BC
sin 20 60 lb = 0 (2)
Solving Eqs. 1 and 2 gives
F
AB
= 324.7 lb Ans.
F
BC
= 340.3 lb Ans.
F
AB
10
20
10
10
20

255
3.1 Particles in Two-Dimensional Force Systems Example 4, page 2 of 2
Free body diagram of connection C
C x
y
150 lb
Equilibrium equations for connection C
F

x = 0: (340.3 lb) cos 20 + F

CD
cos = 0 (5)
F

y
= 0: (340.3 lb) sin 20 + F

CD
sin 150 lb = 0 (6)
Eqs. 5 and 6 can be solved with a calculator that can handle two
simultaneous nonlinear equations.
Alternatively re-write Eqs. 5 and 6 as
F

CD
cos = (340.3) cos 20 (7)
F

CD
sin = (340.3) sin 20 +150 (8)
Eliminate F

CD
by dividing Eq. 8 by Eq. 7:

+
+
3
4

F

BC
= 340.3 lb
5
F

CD
cos (340.3) cos 20
tan
F
CD
sin (340.3) sin 20 + 150
=
Solving Eq. 9 gives
= 39.8 Ans.
Using this result in Eq. 7 gives
F

CD
= 416 lb Ans.
F

CD
(9)
20
20

256
3.1 Particles in Two-Dimensional Force Systems Example 5, page 1 of 2
5. The collar at B weighs 5 lb and can slide freely
over the rod. If spring constant k is 2 lb/in.,
determine the unstretched length of the spring.
14 in.
A
B
C
D
20
Equilibrium equations for B:
F

x = 0: N sin + F

spring
cos = 0 (1)
F

y
= 0: N cos + F

spring
sin lb = 0 (2)
B
+
3
+
5 lb

Free body diagram of collar B 1
N

x
y
F

spring
2 Since the collar
can slide freely
over the rod, only
a normal force N
is present; no
friction force
(which would be
parallel to the rod)
is present.
35

257
3.1 Particles in Two-Dimensional Force Systems Example 5, page 2 of 2
A
B
C
D
= 35
Using = 55 and = 35 in Eqs. 1 and 2 and solving
simultaneously gives
N = 3.052 lb and F

spring
= 3.052 lb
F

spring
is related to the extension of the spring:
F

spring
= k (stretched length
unstretched length) (3)
5
= 20 + 35 = 55


6
180 20 35 = 125

stretched length = BC
Law of sines:
Solving gives BC = 9.802 in.
7 Substituting "stretched length" = BC = 9.802 in.,
k = 2 lb/in., and F

spring
= 3.052 lb in Eq. 3 gives
3.052 lb = (2 lb/in.)(9.802 in.
unstretched length).
Solving gives
unstretched length = 8.28 in. Ans.
14 in.
B
C
D
Geometry: and Geometry: stretched length, BC
sin 125 sin 35
14 in. BC
=
20
20
125
35
35
35
55
35
35
4

258
3.1 Particles in Two-Dimensional Force Systems Example 6, page 1 of 1
F F
Weight = mg = (2 g)(1 kg/1000 g)(9.81 m/s
2
)
= 0.01962 kgm/s
2
= 0.01962 N
B
F
x
Free body diagram of sphere B


F
AB
1
y
+
+
2 Equilibrium equations for sphere B
F

x = 0: F

AB
cos 30 + F = 0 (1)
F

y
= 0: F

AB
sin 30 0.01962 N = 0 (2)
30
Solving Eqs. 1 and 2 gives
F

AB
= 0.0392 N = 39.2 mN
F = 0.0340 N = 34.0 mN Ans.
30
6. Two 2-g spheres are suspended by light cords and then given
electrostatic charges of opposite sign. Calculate the attractive
force, F, acting between the spheres.
A
B C
D
30 30

259
3.1 Particles in Two-Dimensional Force Systems Example 7, page 1 of 2
4 in.
s
Top view
Side view
k = 5 lb/in.
0.5 lb
0.5 lb
A
A
spring
F
1
y
A
N
x
3
+
+
Equilibrium Equations for control knob A:
F

x = 0 F

spring
cos 0.5 lb = 0 (1)
F

y
= 0: F

spring
sin + N = 0 (2)
B
0.5 lb
2 Since the knob "slides
smoothly", only a normal
force N is present; no
friction force is present.
Free body diagram of control knob A
(Side view)
7. The figure shows top and side views of a control knob A that slides
smoothly in a slot cut in a panel and is attached to a spring underneath the
panel. Determine the force in the spring and the distance s, for which the
knob will be in equilibrium when a 0.5-lb force is applied. The
unstretched length of the spring is 4 in., and the weight of the knob is
negligible.

260
3.1 Particles in Two-Dimensional Force Systems Example 7, page 2 of 2
Geometry
Substituting the "stretched length" expression given in Eq. 4
into the force-extension relationship for a spring gives
F

spring
= k (stretched length unstretched length)
= (5 lb/in.) ( 4 in.) (5)
Recall that Eq. 1 is
F

spring
cos 0.5 = 0 (Eq. 1 repeated)
Substituting the expression for F

spring
from Eq. 5 into Eq. 1
and multiplying through by sin gives
5(4 4 sin ) cos 0.5 sin = 0 (6)
Eq. 6 is nonlinear and must be solved numerically by, for
example, using the solver on a calculator. The result is =
69.1. Using the value = 69.1 in Eqs. 2, 3, and 4 and then
solving gives
N = 1.31 lb
F

spring
= 1.40 lb Ans.
s = 1.53 in. Ans.
4 5
s
A
B
4 in.
4 in.
sin




sin
4 in.
stretched length of spring = AB

= (4)
tan = (3)
4 in.
s

261

3.2 Particles and Three-Dimensional Force Systems
262
3.2 Particles and Three-Dimensional Force Systems Procedures and Strategies, page 1 of 1
Procedures and Strategies for Solving Problems Involving
Equilibrium of a Particle in a Three-Dimensional Force System
1. Establish an x, y, and z coordinate system.
2. Construct a free-body diagram showing and labeling all forces acting on
the particle, including the weight, if the weight or the mass is given.
Assign the sense of forces arbitrarily (If you guess wrong, the magnitude of
the force will be found to be negative, when you solve the equations).
3. Resolve each force into x, y and z components and set the sum of each
component to zero:
Fx = 0 F
y
= 0 Fz = 0
4. Solve the equations. Note that if a force is found to be negative, it has a
sense opposite to that shown on the free-body diagram.

263
3.2 Particles and Three-Dimensional Force Systems Problem Statement for Example 1
1. The 150 lb block is supported by pole AB and cables AC
and AD. The force in the pole can be considered to act
along the axis of the pole. Determine the forces in the
cables and the pole.
C
B
A
8 ft
5ft
4 ft
x
y
D
150 lb
z

264
3.2 Particles and Three-Dimensional Force Systems Problem Statement for Example 2
Radius = 100 mm
C
B
A
O
y
x
z
2. The chandelier has a mass of 10 kg and is
supported by three wires arranged as shown.
Determine the force in each wire.
50
50
300 mm

265
3.2 Particles and Three-Dimensional Force Systems Problem Statement for Example 3
3. The small 50-lb block rests on the smooth
inclined plane ACDE and is attached to a cord AB.
Determine the value of the applied force P
necessary to keep the block in equilibrium. P acts
parallel to the z axis.
C
P D
E
50 lb
30
15
cord
y
z
x
A
B

266
3.2 Particles and Three-Dimensional Force Systems Problem Statement for Example 4
4. The 400-kg precast concrete section is being
hoisted by a vertical force T from a crane.
Determine the tension in cables AB, AC, and AD.
x
z
A
B
C
D
y
T
3 m
3 m
1 m
1 m
5 m

267
3.2 Particles and Three-Dimensional Force Systems Problem Statement for Example 5
5. Members CA and DA
of the derrick exert
compressive forces along
their respective axes.
Determine the values of
these forces and the
tension in cable AB.
C
A
B
D
300 kg
3 m
3 m
10 m
5 m
3 m
z
x
y

268
3.2 Particles and Three-Dimensional Force Systems Problem Statement for Example 6
6. The 200-lb plate is supported by three cables tied to ring A,
upon which a 200-lb vertical force acts. Determine the tension
in each cable. The edges of the plate are parallel to the
coordinate axes, and the center of the plate at E lies directly
below A.
50
y
z
E
C
D
B
45
x
F = 200 lb
120
A
65
60
40
70

269
3.2 Particles and Three-Dimensional Force Systems Problem Statement for Example 7
65
70
20
60
A
z
x
y
D
B
O
C
7. The compressive force in pole AO acts along the axis
of the pole and has a magnitude of 900 lb. Determine the
tension in the guy wires AC, AB, and AD.
40

270
3.2 Particles in Three-Dimensional Force Systems Example 1, page 1 of 4
1. The 150 lb block is supported by pole AB and cables AC
and AD. The force in the pole can be considered to act
along the axis of the pole. Determine the forces in the
cables and the pole.
C
B
A
8 ft
5ft
4 ft
x
y
D
150 lb
z

271
3.2 Particles in Three-Dimensional Force Systems Example 1, page 2 of 4
A
1 Free body diagram of connection at A
x
y
z
F

AC
F

BA
F

AD
150 lb
2 The forces in cables AC and AD are tensile so are
directed away from A.
Equilibrium equation:
F

AC
+ F

AD
+ F
BA
{150j} lb = 0 (1)
Express the forces in terms of rectangular
components.
Cable AC is parallel to the x axis so
F

AC
= F

AC
i (2)
The minus sign indicates that the force is in the
negative x direction. Similarly, cable AD is parallel
to the z axis so
F

AD
= F

AD
k (3)
3
4
C
B
A
8 ft
5 ft
4 ft
x
y
D
150 lb
z

272
3.2 Particles in Three-Dimensional Force Systems Example 1, page 3 of 4
5 The components of F

BA
can be derived from a position
vector going from B to A.
r

BA
= coordinates of A coordinates of B
= {5i + 4k} ft { 8j} ft
= {5i + 8j + 4k} ft
The magnitude of r

BA
is
r

BA
= 5
2
+ 8
2
+ 4
2
= 10.2470 ft
The unit vector in the direction of BA is
u

BA
= r

BA
/r

BA
=
= 0.4879i + 0.7807j + 0.3904k
5i + 8j + 4k
10.247
z
150 lb
D
y
x
4 ft
5 ft
8 ft
A
B
C
r

BA

273
3.2 Particles in Three-Dimensional Force Systems Example 1, page 4 of 4
7 Since each component must vanish independently,
we have
F

x = 0: F
AC
+ 0.4879F
BA
= 0 (5)
F

y = 0: 150 lb + 0.7807F
BA
= 0 (6)
F

z = 0: F

AD
+ 0.3904F
BA
= 0 (7)
Solving Eqs. 5, 6, and 7 simultaneously gives
F

AC
= 93.7 lb Ans.
F

AD
= 75.0 lb Ans.
F

BA
= 192.1 lb Ans.

Since the force F

BA
is in the same direction as u

BA
, we can write
F

BA
= F

BA
u

BA
= F

BA
(0.4879i + 0.7807j + 0.3904k)
= 0.4879F

BA
i + 0.7807F

BA
j + 0.3904 F

BA
k (4)
Substituting the component forms of the forces given by Eqs. 2, 3,
and 4 into Eq. 1 gives
F

AC
+ F

AD
+ F

BA
{150j} lb = 0 (Eq. 1 repeated)
or
F

AC
i F

AD
k + (0.4879F

BA
i + 0.7807F

BA
j +
0.3904F

BA
k) 150j = 0
Rearranging by collecting coefficients of i, j, and k gives
( F

AC
+ 0.4879F

BA
)i + ( 150 lb + 0.7807F

BA
)j
+ ( F

AD
+ 0.3904F

BA
)k = 0
6

274
3.2 Particles in Three-Dimensional Force Systems Example 2, page 1 of 5
Radius = 100 mm
C
B
A
O
y
x
z
2. The chandelier has a mass of 10 kg and is
supported by three wires arranged as shown.
Determine the force in each wire.
50
50
300 mm

275
3.2 Particles in Three-Dimensional Force Systems Example 2, page 2 of 5
O
B
1 Free body diagram
y
F

DA
F

BA
F

CA
Weight
Weight = mg
= (10 kg)(9.81 m/s
2
)
= 98.1 N
2 The forces in the wires are all tensile, so they are directed
away from the body (the chandelier).
3 Equilibrium equations
F

BA
+ F

CA
+ F

DA
{98.1j} N = 0 (1)
F

BA
300 mm
100 mm
z
y
A
B
O
4 Determine the rectangular components of F

BA
.
F

BA
= F

BA
sin j + F

BA
cos k (2)
5 Geometry
= tan
-1
= 71.5651
D
C
300 mm
100 mm
z
x

276
3.2 Particles in Three-Dimensional Force Systems Example 2, page 3 of 5
A
8 The angle is the same as we computed before.
9 The vertical component is
F

DAy
= F

DA
sin 71.5651
or
F

DAy
= 0.9487F

DA
(4)
10 The component in the horizontal plane (the xz plane) is
F

DA
cos 71.5651 = 0.3162F

DA

O
D
x
z
50
= 71.5651
z
D
F

DA
O
x
y
6 Substituting = 71.5651 in Eq. 2 gives
F

BA
= F

BA
sin 71.5651j + F

BA
cos 71.5651k
or
F

BA
= 0.9487F

BA
j + 0.3162F

BA
k (3)
7 Determine the components of F

DA

277
3.2 Particles in Three-Dimensional Force Systems Example 2, page 4 of 5
13 The easiest way to get F

CA
is to note that it differs from F

DA
only by a
change in sign of the x coordinate.
F

CA
= 0.2422F

CA
i + 0.9487F

CA
j 0.2032F

CA
k (8)
C
z
D
O
x
y
50
F

DA
F

CA
D
z
O
x
11 x and z components of F

DA
0.3162F

DA
F

DAx
= (0.3162F

AD
) sin 50 = 0.2422F

DA
(5)
F

DAz
= (0.3162F

AD
) cos 50 = 0.2032F

DA
(6)
12 Combining Eqs. 4, 5, and 6 gives
F

DA
= 0.2422F

DA
i + 0.9487F

DA
j 0.2032F

DA
k (7)

278
3.2 Particles in Three-Dimensional Force Systems Example 2, page 5 of 5
Substituting the component form of the forces into the equilibrium equation gives
F

BA
+ F

CA
+ F

DA
98.1j = 0 (Eq. 9 repeated)
or
(0.9487F

BA
j + 0.3162F

BA
k) + ( 0.2422F

CA
i + 0.9487F

CA
j 0.2032F

CA
k)
+ (0.2422F

DA
i + 0.9487F

DA
j 0.2032F

DA
k) {98.1j} N = 0 (10)
Rearranging by collecting coefficients of i, j, and k gives
( 0.2422F

CA
+ 0.2422F

DA
)i + (0.9487F

BA
+ 0.9487F

CA
+ 0.9487F

DA

98.1 N)j + (0.3162F

BA
0.2032F

CA
0.2032F

DA
)k = 0 (11)
Since each component must vanish independently, we have
F

x = 0: 0.2422F

CA
+ 0.2422 F

DA
= 0 (12)
F

y
= 0: 0.9487F

BA
+ 0.9487F

CA
+ 0.9487F

DA
98.1 N = 0 (13)
F

z = 0: 0.3162F

BA
0.2032F

CA
0.2032F

DA
= 0 (14)
Solving Eqs. 12, 13, and 14 simultaneously gives
F

BA
= 40.5 N Ans.
F

CA
= 31.5 N Ans.
F

DA
= 31.5 N Ans.
14

279
3.2 Particles in Three-Dimensional Force Systems Example 3, page 1 of 3
1 Free body diagram of block
z
y
B
x
W = weight of block
T
P
N
Normal force from the
inclined plane. Since the
plane is smooth, no friction
forces are present.
Equilibrium equation:
F = 0: W + P + N + T = 0 (1)
Tension in
cord AB
15
30
50 lb
E
D P
C
3. The small 50-lb block rests on the smooth
inclined plane ACDE and is attached to a cord AB.
Determine the value of the applied force P
necessary to keep the block in equilibrium. P acts
parallel to the z axis.
B
A
x
z
y
cord

280
3.2 Particles in Three-Dimensional Force Systems Example 3, page 2 of 3
4 Compute the components of T.
T

z = T cos 30
T sin 30
T

x = (T sin 30) cos 15
T

y
= (T sin 30) sin 15
Thus,
T = (T sin 30) cos 15i + (T sin 30) sin 15j
+ T cos 30k (5)
x
y
A
z
N sin 75
N
N cos 75
3 We must express the forces in terms of
rectangular coordinates. Thus
W = {50j} lb (2)
P = Pk (3)
The components of the force N
perpendicular to the plane can be
determined from the figure below.
N = (N cos 75)i + (N sin 75)j (4)
15
15
75
5
y
z
x
A
B
C
D
E
30
15
T

281
3.2 Particles in Three-Dimensional Force Systems Example 3, page 3 of 3
6 Substituting the rectangular component forms of the forces in Eq. 1 gives
50j Pk + (N cos 75)i + (N sin 75)j (T sin 30 cos 15)i + (T sin 30 sin 15)j + (T cos 30)k = 0 (6)
Rearranging by collecting i, j, and k components gives
(N cos 75 T sin 30 cos 15)i + ( 50 + N sin 75 + T sin 30 sin 15)j + ( P + T cos 30)k = 0 (7)
Since each component must vanish independently we have
F

x = 0: N cos 75 T sin 30 cos 15 = 0 (8)
F

y
= 0: 50 + N sin 75 + T sin 30 sin 15 = 0 (9)
F

z = 0: P + T cos 30 = 0 (10)
Solving Eqs. 8, 9, and 10 simultaneously gives
N = 48.3 lb Ans.
T = 25.9 lb Ans.
P = 22.4 lb Ans.

282
3.2 Particles in Three-Dimensional Force Systems Example 4, page 1 of 7
4. The 400-kg precast concrete section is being
hoisted by a vertical force T from a crane.
Determine the tension in cables AB, AC, and AD.
x
z
A
B
C
D
y
T
3 m
3 m
1 m
1 m
5 m

283
3.2 Particles in Three-Dimensional Force Systems Example 4, page 2 of 7
x
z
A
B
C
D
y
T
3 m
3 m
1 m
1 m
1 Free body diagram of hook, cables, and concrete section.
2 Weight of concrete section = (400 kg)(9.81 m/s
2
)
= 3924 N
3 Equilibrium equation
F
y
= 0: T 3924 N = 0
Thus the vertical force from the crane
equals the weight being supported:
T = 3924 N
+
5 m

284
3.2 Particles in Three-Dimensional Force Systems Example 4, page 3 of 7
5
4
A
Free body diagram of hook A
y
z x
F

AC
F

AB
F

AD
T = 3924 N
Equilibrium equation
{3924j} N + F

AD
+ F

AB
+ F

AC
= 0 (1)

285
3.2 Particles in Three-Dimensional Force Systems Example 4, page 4 of 7
x
z
A
D
6 Express the forces in terms of rectangular
components.
y
B
C
5 m
3 m
r

AD
7 First let
r

AD
= position vector to D from A (the force acting on the hook at A

points to D from A)
= coordinates of D coordinates of A
= { 3i} m {5j} m
= 3i 5j
The length of r

AD
is
r

AD
= 3)
2
+ ( 5)
2
= 5.8310 m
The unit vector in the direction of AD is,
u

AD
=
=
= 0.5145i 0.8575j

3i 5j
r

AD
r

AD
u

AD
5.8310

286
3.2 Particles in Three-Dimensional Force Systems Example 4, page 5 of 7
8
Next let
r

AB
= position vector to B from A
= {3i + k} m {5j} m
= 3i 5j + k
The length of r

AB
is
r

AB
= 3)
2
+ ( 5)
2
+ (1)
2
= 5.9161 m
The unit vector in the direction of AB is
u

AB
=



=

5.9160
= 0.5071i 0.8452j + 0.1690k

x
z
A
D
y
B
C
5 m
3 m
1 m
u

AB
r

AB
3i 5j + k
r

AB
r

AB
Since the force F

AD
is in the same direction as u

AD
, we can write
F

AD
= F

AD
u

AD
= F

AD
( 0.5145i 0.857j)

= 0.5145F

AD
i 0.8575F

AD
j (2)

287
3.2 Particles in Three-Dimensional Force Systems Example 4, page 6 of 7
x
z
A
D
y
B
C
5 m
3 m
r

AB
1 m
9
r

AC
1 m
Thus,
F

AB
= F

AB
u

AB
= 0.5071F

AB
i 0.8452F

AB
j + 0.1690F

AB
k (3)
We can save work if we note that the position vector r

AC
differs
from r

AB
only by the sign of the z component:
r

AB
= 3i 5j + k
r
AC
= 3i 5j k
So the force F

AC
must also differ from F

AB
only by the sign of the z
component:
F

AB
= 0.5071F

AB
i 0.8452F

AB
j + 0.1690F

AB
k
F

AC
= 0.5071F

AC
i 0.8452F

AC
j 0.1690F

AC
k (4)


288
3.2 Particles in Three-Dimensional Force Systems Example 4, page 7 of 7
10 The equilibrium equation, Eq. 1, was
{3924j} N + F

AD
+ F

AB
+ F

AC
= 0 (Eq. 1 repeated)
Substituting the component forms of the forces given by Eqs. 2, 3, and 4 into Eq. 1 gives
3924j 0.5145F

AD
i 0.8575F

AD
j + 0.5071F

AB
i 0.8452F

AB
j + 0.1690F

AB
k + 0.5071F

AC
i 0.8452F

AC
j 0.1690F

AC
k = 0
Rearranging by collecting the i, j, and k components gives
( 0.5145F

AD
+ 0.5071F

AB
+ 0.5071F

AC
)i + (3924 0.8575F

AD
0.8452F

AB
0.8452F

AC
)j + (0.1690F

AB
0.1690F

AC
)k = 0
Since each component must vanish independently, we have
F

x = 0: 0.5145F

AD
+ 0.5071F

AB
+ 0.5071F

AC
= 0 (5)
F

y
= 0: 3924 0.8575F

AD
0.8452F

AB
0.8452F

AC
= 0 (6)
F

z = 0: 0.1690F

AB
0.1690F

AC
= 0 (7)
Solving Eqs. 5, 6, and 7 simultaneously gives
F

AB
= 1.161 kN Ans.
F

AC
= 1.161 kN Ans.
F

AD
= 2.29 kN Ans.

289
3.2 Particles in Three-Dimensional Force Systems Example 5, page 1 of 5
5. Members CA and DA
of the derrick exert
compressive forces along
their respective axes.
Determine the values of
these forces and the
tension in cable AB.
C
A
B
D
300 kg
A
1 Free body diagram of
connection at A.
y
z
x
F

CA

(compression)
F

AB
(tension)
F

DA

(compression)
Weight = (300 kg)(9.81 m/s
2
) = 2943 N
2 Equilibrium equation
{2943j} N + F

AB
+ F

CA
+ F

DA
= 0 (1)
3 m
3 m
10 m
5 m
3 m
z
x
y

290
3.2 Particles in Three-Dimensional Force Systems Example 5, page 2 of 5
A
B
y
z
5 m
3 m
10 m
3 Express the forces in terms of rectangular components.
4
Thus define
r

AB
= coordinates of B coordinates of A
= { 5k} m {10j + 3k} m
= 10j 8k
The length of r

AB
is
r

AB
= ( 10)
2
+ ( 8)
2
= 12.8062 m
The unit vector in the direction of AB is
u

AB
=


=
12.8062
= 0.7809j 0.6247k
Since the force is in the same direction as u

AB
, we can
write
F

AB
= F

AB
u

AB
= F

AB
( 0.7809j 0.6247k)
= 0.7809F

AB
j 0.6247F

AB
k (2)
r

AB
r

AB
10j 8k
The components of F

AB
will be derived from the
position vector to B from A because the force F

AB

points to B from A.
u

AB
r

AB
x
5

291
3.2 Particles in Three-Dimensional Force Systems Example 5, page 3 of 5
3i + 10j + 3k
Next let
r

CA
= position vector to A from C
= coordinates of A coordinates of C
= {10j + 3k} m { 3i} m
= 3i + 10j + 3k
The unit vector in the direction of r

CA
is,
u

CA
=



= 0.2762i 0.9206j + 0.2762k
so the force F

CA
can be expressed as
F

CA
= F

CA
u

CA
= 0.2762F

CA
i + 0.9206 F

CA
j + 0.2762F

CA
k (3)

3)
2
+ ( 10)
2
+ (3)
2
6
u

CA
C
A
z
x
3 m
10 m
y
3 m
r

CA

292
3.2 Particles in Three-Dimensional Force Systems Example 5, page 4 of 5
C
A
D
z
x
3 m
10 m
y
3 m
To determine the components of F

DA
, we can make use of the
observation that the position vectors r

CA
and r

DA
differ only by the
sign of the x component:
r

CA
= +3i + 10j + 3k
r

DA
= 3i + 10j + 3k
So the force F

CA
must also differ from F

DA
only by the sign of the x
component:
F

CA
= +0.2762F

CA
i + 0.9206F

CA
j + 0.2762F

CA
k
F

DA
= 0.2762F

DA
i + 0.9206F

DA
j + 0.2762F

DA
k (4)

7
Substituting the component forms of the forces given by Eqs. 2, 3, and 4 into Eq. 1 gives
2943j + F

AB
+ F
CA
+ F

DA
= 0 (Eq. 1 repeated)
or
2943j + ( 0.7809F

AB
j 0.6247F

AB
k) + (0.2762F

CA
i + 0.9206F

CA
j + 0.2762F

CA
k)
+ ( 0.2762F

DA
i + 0.9206F

DA
j + 0.2762F

DA
k) = 0
r

DA
8
r

CA
3 m

293
3.2 Particles in Three-Dimensional Force Systems Example 5, page 5 of 5
Rearranging by collecting the i, j, and k components gives
(0.2762F

CA
0.2762F

DA
)i + ( 0.7809F

AB
2943 + 0.9206F

CA
+ 0.9206F

DA
)j
+ ( 0.6247F

AB
+ 0.2762F

CA
+ 0.2762F

DA
)k = 0

9 Since each component must vanish independently, we have
Fx = 0: 0.2762F

CA
0.2762F

DA
= 0 (5)
F
y
= 0: 0.7809F

AB
2943 + 0.9206F
CA
+ 0.9206F
DA
= 0 (6)
Fz = 0: 0.6247F
AB
+ 0.2762F
CA
+ 0.2762F
DA
= 0 (7)
Solving Eqs. 5, 6, and 7 simultaneously gives
F
AB
= 2261 N = 2.26 kN Ans
F
CA
= 2558 N = 2.56 kN Ans
F
DA
= 2558 N = 2.56 kN Ans

294
3.2 Particles in Three-Dimensional Force Systems Example 6, page 1 of 7
x
y
z
200 lb
F

AB
F

AC
F

AD
1 Free body diagram of ring A
2 Equilibrium equation
{200j} lb + F

AB
+ F

AC
+ F

AD
= 0 (1)
6. The 200-lb plate is supported by three cables tied to ring A,
upon which a 200-lb vertical force acts. Determine the tension
in each cable. The edges of the plate are parallel to the
coordinate axes, and the center of the plate at E lies directly
below A.
50
y
z
E
C
D
B
45
x
F = 200 lb
120
A
65
60
40
70

295
3.2 Particles in Three-Dimensional Force Systems Example 6, page 2 of 7
x
y
z F

AD
4 The rectangular components of F

AD
are
F

AD
= F

AD
cos i + F

AD
cos j + F

AD
cos k
From the diagram, = 45, = 120, and = 60, thus
F

AD
= F

AD
cos 45 i + F

AD
cos 120 j + F

AD
cos 60 k
= 0.7071F

AD
i 0.5F

AD
j + 0.5F

AD
k (2)
= 45
= 120
= 60
A
We must use the given angles to express the
forces in rectangular components. Begin with
the force cable AD exerts on ring A.
3
40
60
65
A
120
F = 200 lb
x
45
B
D
C
E
z
y
70
50

296
3.2 Particles in Three-Dimensional Force Systems Example 6, page 3 of 7
F

AB
Next determine the rectangular
components of F

AB
.
5
6
7
The component parallel to the
horizontal plane is
F
ABxz
= F

AB
cos 40
= 0.7660F

AB
The component normal to the
horizontal plane is
F
ABy
= F

AB
sin 40
= 0.6428F

AB
40
A
x
B
E
y
50
40

297
3.2 Particles in Three-Dimensional Force Systems Example 6, page 4 of 7
8 The x and z components of F

AB
can be now be found:
F

ABx
= 0.7660F

AB
cos 50 = 0.4924F

AB
F

ABz
= 0.7660F

AB
sin 50 = 0.5868F

AB
9 The x, y, and z components of F

AB
are now known and so we can
write
F

AB
= 0.4924F

AB
i 0.6428F

AB
j 0.5868F

AB
k (3)
40
B
E
z
A
x
y
F
ABxz
= 0.7660F

AB
50
50

298
3.2 Particles in Three-Dimensional Force Systems Example 6, page 5 of 7
The vertical component of F

AC
is
F

ACy
= F

AC
sin 70 = 0.9397F

AC
11
12
Next, determine the rectangular components of F

AC
. 10
The component parallel to the
horizontal plane is
F

ACxz
= F

AC
cos 70 = 0.3420 F

AC
A
x
C
E
z
y
70
F

AC
70

299
3.2 Particles in Three-Dimensional Force Systems Example 6, page 6 of 7
The x and z components of F

AC
are
F

ACx
= (0.3420F

AC
) cos 65 = 0.1445F

AC
F

ACz
= (0.3420F

AC
) sin 65 = 0.3100F

AC
13
14 The x, y, and z components of F

AC
are now known and so we can write
F

AC
= 0.1445F
AC
i 0.9397F

AC
j 0.3100F

AC
k (4)
E
C
65
A
y
x
z
65
F

ACxz
= 0.3420F

AC

300
3.2 Particles in Three-Dimensional Force Systems Example 6, page 7 of 7
Solving Eqs. 5, 6, and 7 simultaneously gives
F

AB
= 35.2 lb Ans.
F

AC
= 158.5 lb Ans.
F

AD
= 56.9 lb Ans.
The equilibrium equation is
200j + F

AB
+ F

AC
+ F

AD
= 0 (Eq. 1 repeated)
or, replacing F

AB
by Eq. 3, F

AC
by Eq. 4, and F

AD
by Eq. 2,
200j + ( 0.4924F

AB
i 0.6428F

AB
j 0.5868F

AB
k)
+ ( 0.1445F
AC
i 0.9397F
AC
j 0.3100F
AC
k)
+ ( 0.7071F

AD
i 0.5F

AD
j + 0.5F

AD
k) = 0
Rearranging by collecting coefficients of i, j, and k gives
( 0.4924F

AB
0.1445F

AC
+ 0.7071F

AD
)i
+ (200 0.6428F

AB
0.9397F

AC
0.50F

AD
)j
+ (0.5868F

AB
0.3100F

AC
+ 0.50F

AD
)k = 0
Since each component must vanish independently, we have
F

x
0: 0.4924F

AB
0.1445F

AC
+ 0.7071F

AD
= 0 (5)
F

y
0: 200 0.6428F

AB
0.9397F

AC
0.5F

AD
= 0 (6)
F

z
0: 0.5868F

AB
0.3100F

AC
+ 0.5F

AD
= 0 (7)
16 15

301
3.2 Particles in Three-Dimensional Force Systems Example 7, page 1 of 5
x
y
z
1 Free body diagram of connection at A
A
900 lb
F

AB
F

AD
F

AC
Compressive force, so
pole pushes up on A
2
3
Equilibrium equation
{900j} lb + F

AC
+ F

AD
+ F

AB
= 0 (1)
We must use the given angles to express the forces
in rectangular components.
65
70
20
60
A
z
x
y
D
B
O
C
7. The compressive force in pole AO acts along the axis
of the pole and has a magnitude of 900 lb. Determine the
tension in the guy wires AC, AB, and AD.
40

302
3.2 Particles in Three-Dimensional Force Systems Example 7, page 2 of 5
y
z
A
B
65
F

AB
4 To determine the rectangular components of F

AB
,
consider a view of the xz plane as seen from the
positive x axis. Thus
F

AB
= F

AB
sin 65j F

AB
cos 65k
= 0.9063F

AB
j 0.4236F

AB
k (2)
x, O
40
C
O
B
D
x
z
A
60
20
70
65
y

303
3.2 Particles in Three-Dimensional Force Systems Example 7, page 3 of 5
A
x
y
O
0.5 F

AC
The x and z components of F
AC
are
F

ACx
= 0.5F

AC
cos 20

= 0.4698F

AC
F

ACz
= 0.5F

AC
sin 20
= 0.1710F

AC
7
F
AC
y
A
x
O
The component parallel to the
horizontal (xz) plane is
F

ACxz
= F

AC
cos 60
= 0.5F

AC
6
Next determine the rectangular components of F

AC
.
The vertical component is
F
ACy
= F
ACy
sin 60
= 0.8660F
AC
5
C
20
z
C
z
20
Thus all components of F
AC
are known:
F

AC
= 0.4698F

AC
i
0.8660F

AC
j
+ 0.1710F

AC
k (3)
8
60
60

304
3.2 Particles in Three-Dimensional Force Systems Example 7, page 4 of 5
9
Finally, we determine the rectangular
components of F
AD
. The vertical component is
F

ADy
= F

AD
sin 70
= 0.9397F

AD
The component parallel to
the horizontal (x-z) plane is
F

ADxz
= F

AD
cos 70
= 0.3420F

AD
O
D
y
A
10
11 Now, the x and z components of F
AD
are
F

ADx
= 0.3420F

AD
sin 40 = 0.2198F

AD
F

ADz
= 0.3420F

AD
cos 40 = 0.2620F

AD
12 Therefore,
F

AD
= 0.2198F

AD
i 0.9397F

AD
j + 0.2620F

AD
k (4)
A
z
x
y
D
O
0.3420F

AD
z
F

AD
70
x
40
40
70

305
3.2 Particles in Three-Dimensional Force Systems Example 7, page 5 of 5
Substituting the component form of the forces (Eqs. 2, 3, and 4) into the equilibrium equation, Eq. 1, gives
900j + F

AB
+ F

AC
+ F
AD
= 0 (Eq. 1 repeated)
900j + ( 0.9063F

AB
j 0.4226F

AB
k)
+ ( 0.4698F

AC
i 0.8660F

AC
j + 0.1710F

AC
k)
+ (0.2198F

AD
i 0.9397F

AD
j + 0.2620F

AD
k) = 0 (5)
Rearranging Eq. 5 by collecting coefficients of i, j, and k gives
( 0.4698F

AC
+ 0.2198F

AD
)i
+ (900 0.9063F

AB
0.8660F

AC
0.9397F

AD
)j
+ ( 0.4226F

AB
+ 0.1710F

AC
+ 0.2620F

AD
)k = 0 (6)
Since each component must vanish independently, we have
F

x = 0: 0.4698F

AC
+ 0.2198F

AD
=0 (7)
F

y
= 0: 900 0.9063F

AB
0.8660F

AC
0.9397F

AD
= 0 (8)
F

z = 0: 0.4226F

AB
+ 0.1710F

AC
+ 0.2620F

AD
= 0 (9)
Solving Eqs. 7, 8, and 9 simultaneously gives
F

AB
= 350 lb, F
AC
= 203 lb, and F
AD
= 433 lb Ans.
13

306

4. Moments and Resultants of Force Systems
307

4.1 Moments in Two-Dimensional Force Systems
308
4.1 Moments in Two-Dimensional Force Systems Procedures and Strategies, page 1 of 1
F
d
O
O
A
F
y
x
F
y
Fx
dx
d
y
F
O
Procedures and Strategies for Solving Problems Involving
Moments in a Two-Dimensional Force System
The moment of a force F about a point O can be computed in
three ways:
1. M
O
= F d,
where d = perpendicular distance from O to the line of
action of the force;
2. M
O
= F d,
where F = component of force perpendicular to the
line connecting point O with the point of application A
of the force, and d = distance OA; and
3. M
O
= F
y
dx Fx d
y
,
where Fx and F
y
are the rectangular components of the
force and dx and d
y
are the x and y distances between O
and A.
Which way is best depends on what information is given in
the problem. For example, if you are given x and y distances
from point O to the force, use the third approach above.
F
A
d

309
4.1 Moments in Two-Dimensional Force Systems Problem Statement for Example 1
200 mm
a)
A
A
b)
40 N
40 N
y
x
H E A V Y D U T Y 2 5 0 M M C H I N A
D
R
O
P
F
O
R
G
E
D
S
T
E
E
L
1
0
"
1. Determine the moment of the 40-N force about the bolt at A when
the wrench is in position a) and in position b).

310
4.1 Moments in Two-Dimensional Force Systems Problem Statement for Example 2
B
A
10 lb
4 ft 8 ft
2. Determine the moment of the 10-lb force
about a) end A and b) end B of the beam.

311
4.1 Moments in Two-Dimensional Force Systems Problem Statement for Example 3
200 mm
a)
A
b)
50 N
DROP FORGED STEEL 10"
3. Determine the moment of the force about
the bolt at A for cases a) and b).
200 mm
30
50 N
DROP FORGED STEEL 10"
A
string tied
to end of
wrench
30

312
4.1 Moments in Two-Dimensional Force Systems Problem Statement for Example 4
4 kN
y
x
4. Determine the moment of the 4-kN force about corner A of the plate a) by
using the definition M

A
= Fd, and b) by using the components of the force.
60
4 m
3 m
A
D
B
C

313
4.1 Moments in Two-Dimensional Force Systems Problem Statement for Example 5
5. Boom AB and cable CB support a crate of weight W.
The force from the cable acting on the boom is known to
be 5 kN and is directed from B to C. Determine the
moment produced by this force about A.
W
5

m
A
B
C
5
12
13
40

314
4.1 Moments in Two-Dimensional Force Systems Problem Statement for Example 6
6. A window is propped open by a stick AB. If a 150-lbin. moment about the
hinge is required to keep the window open, determine the force in the stick.
Assume the force is directed along the axis of the stick.
24 in.
30 in.
12 in.
A
B
C

315
4.1 Moments in Two-Dimensional Force Systems Example 1, page 1 of 3
200 mm
200 mm
200 mm
a)
A
A
b)
40 N
Position a)
40 N
40 N
1
y
x
If the wrench could
rotate freely about A
(the bolt offered no
resistance), then a
counterclockwise
rotation would be
produced.
H E A V Y D U T Y 2 5 0 M M C H I N A
H
E
A
V
Y
D
U
T
Y
2 5 0 M M C
H
I N
A
H E A V Y D U T Y 2 5 0 M M C H I N A
D
R
O
P
F
O
R
G
E
D
S
T
E
E
L
1
0
"
1. Determine the moment of the 40-N force about the bolt at A when
the wrench is in position a) and in position b).

316
4.1 Moments in Two-Dimensional Force Systems Example 1, page 2 of 3
A
40 N
2
y
x
H E A V Y D U T Y 2 5 0 M M C H I N A
Calculate the moment about A.
M

A
= +(40 N) (200 mm) = 8000 Nmm = 8 Nm Ans. +
Force distance to line of action of force
= magnitude of moment
4
Sign convention: a counterclockwise
rotation is called positive.
3
Observation: The force points in the negative y
direction but M

A
is still positive. The sign
convention for moment is distinct from the sign
convention for force.
5

317
4.1 Moments in Two-Dimensional Force Systems Example 1, page 3 of 3
200 mm
A
Position b)
40 N
6
y
x
A clockwise rotation
would occur, if the bolt
offered no resistance.
D
R
O
P
F
O
R
G
E
D
S
T
E
E
L
1
0
"
7
D
R
O
P
F
O
R
G
E
D
S
T
E
E
L
1
0
"
Calculate the moment about A.
M

A
= (40 N) (200 mm) = 8000 Nmm = 8 Nm Ans. +
8
40 N
D
R
O
P
F
O
R
G
E
D
S
T
E
E
L
1
0
"
A
Negative moment because the clockwise
rotation of the wrench is opposite what we
called positive moment.
Observation: The force points in
the positive x direction, but the
moment is still negative. This is
not a contradiction. Two sign
conventions exist, one for
moments and one for forces.
9

318
4.1 Moments in Two-Dimensional Force Systems Example 2, page 1 of 3
B
A
10 lb
4 ft 8 ft
2. Determine the moment of the 10-lb force
about a) end A and b) end B of the beam.

319
4.1 Moments in Two-Dimensional Force Systems Example 2, page 2 of 3
B
A
10 lb
4 ft
If the beam could rotate freely about
point A (no constraint present at B),
then a clockwise rotation would be
produced.
a) Moment about A 1
Rotation
about A
2
+
Sign convention: counterclockwise rotation is positive.
M

A= (10 lb) (4 ft) = 40 lbft Ans.
Minus sign because rotation of beam is clockwise.

320
4.1 Moments in Two-Dimensional Force Systems Example 2, page 3 of 3
B A
10 lb
8 ft
If the beam could rotate freely about
point B (no constraint present at A),
then a counterclockwise rotation
would be produced.
b) Moment about B 3
4 + M

B
= +(10 lb) (8 ft) = 80 lbft Ans.
Plus sign because rotation of beam is counterclockwise.
Observation: The same force can produce both a
positive or a negative moment it depends on the
location of the point about which the moment is
calculated.
5

321
4.1 Moments in Two-Dimensional Force Systems Example 3, page 1 of 4
200 mm
a)
A
b)
50 N
DROP FORGED STEEL 10"
3. Determine the moment of the force about
the bolt at A for cases a) and b).
200 mm
30
50 N
DROP FORGED STEEL 10"
A
String tied
to end of
wrench
A
DROP FORGED STEEL 10"
30
50 N
1 Case a)
Draw the line of action
of the force.
Draw a line from A perpendicular
to the line of action.
2
30
50 N
DROP FORGED STEEL 10"
A
30

322
4.1 Moments in Two-Dimensional Force Systems Example 3, page 2 of 4
200 mm
30
50 N
DROP FORGED STEEL 10"
A
string
Use trigonometry to calculate the "moment arm"
perpendicular distance d from A to the line of action.
3
30
50 N
DROP FORGED STEEL 10"
A
d = (200 mm) sin 30
= 100 mm
Calculate the moment.
M

A
= (50 N)(100 mm)

= 5000 Nmm
= Nm Ans.
4
+
Case b) 5
50 N
30
DROP FORGED STEEL 10"
A
Draw the line of action. 6

323
4.1 Moments in Two-Dimensional Force Systems Example 3, page 3 of 4
200 mm
Draw a line from A perpendicular to the
line of action and calculate the
perpendicular distance.
7
DROP FORGED STEEL 10"
A
d = (200 mm) sin 30
= 100 mm
Calculate the moment.
M

A
= (50 N)(100 mm)
= 5 Nm Ans.
8
+
30
50 N

324
4.1 Moments in Two-Dimensional Force Systems Example 3, page 4 of 4
Rod attached to
end of wrench
This observation is called the "principle of
transmissibility" the force can "slide" along
its line of action without affecting the
moment produced.
10
30
30
50 N
DROP FORGED STEEL 10"
A
string
9 Observation: Same moment no matter where
the force acts along its line of action.
30
50 N
DROP FORGED STEEL 10"
A
DROP FORGED STEEL 10"
A
30
50 N
DROP FORGED STEEL 10"
A
50 N

325
4.1 Moments in Two-Dimensional Force Systems Example 4, page 1 of 4
4 kN
y
x
4. Determine the moment of the 4-kN force about corner A of the plate a) by
using the definition M

A
= Fd, and b) by using the components of the force.
60
4 m
3 m
A
D
B
C

326
4.1 Moments in Two-Dimensional Force Systems Example 4, page 2 of 4
4 kN
y
x
60
4 m
3 m
A
Part a): Use geometry to compute d in M

A
= Fd. 1
60
D
C
B
E
d
AE = 4 m EB
= 4 m 1.732 m
= 2.268 m
5
Distance EB = = 1.732 m
4
Draw the line of action of the
force.
2
Draw a line
from A to
the line of
action.
3
d = AE sin 60
= (2.268 m) sin 60
= 1.964 m
6
Calculate moment.
M

A
= F d

= (4 kN) (1.964 m)
= 7.86 kNm Ans.
7
+
tan 60
3 m
60

327
4.1 Moments in Two-Dimensional Force Systems Example 4, page 3 of 4
4 kN
y
x
60
4 m
3 m
A
Part b): Express the force in terms of
rectangular components.
8
(4 kN) sin 60 = 3.464 kN
D
B
C
(4 kN) cos 60 = 2 kN
Tends to rotate the
plate clockwise
about A, so
negative.
10
+
Calculate moment.
M

A
= Fd = + (3.464 kN)(4m) (2 kN)(3m) = 7.86 kNm Ans.
11
9
Same answer as computed before.

328
4.1 Moments in Two-Dimensional Force Systems Example 4, page 4 of 4
Observation: Calculating M

A
was much easier when using components
(part b) because we did not have to compute any distances they were
given. In general, since horizontal and vertical distances are often given
(or easily computed) in a problem statement, calculating moments by
using horizontal and vertical components of force is often the best
procedure to follow. It all depends, however, on what data are given.
H
E
A
V
Y
D
U
T
Y
2
5
0
M
M
C
H
IN
A
200 mm
A
40 kN
4 m
A
D
y
3 m
B
x
60
4 kN
C
12
In the problem below, use M

A
= Fd (express
force in components)
In this problem, use M

A
= Fd (do not express force in
components)
13
14

329
4.1 Moments in Two-Dimensional Force Systems Example 5, page 1 of 5
5. Boom AB and cable CB support a crate of weight W.
The force from the cable acting on the boom is known to
be 5 kN and is directed from B to C. Determine the
moment produced by this force about A.
W
5

m
A
B
C
5
12
13
40

330
4.1 Moments in Two-Dimensional Force Systems Example 5, page 2 of 5
Express the 5-kN force in terms of
components at B:
5 m
A
B
C
5
12
13
(5 kN)
13
5
13
12
(5 kN)
5 kN
40
1
40
40
B
A
(5 m) cos 40 = 3.830 m
(5 m) sin 40 = 3.214 m
Geometry 2
Calculate the moment.
M
A
= Fd = (5 kN)(3.830 m) (5 kN)(3.214 m) = 7.47 kNm Ans. (1) +
3
5
13 13
12

331
4.1 Moments in Two-Dimensional Force Systems Example 5, page 3 of 5
40
A
(5 kN)
5
13
B
5
5 kN
13
12
(5 kN)
13
12
C
Line of action
Alternative solution. Slide the force to C
along the line of action.
4

332
4.1 Moments in Two-Dimensional Force Systems Example 5, page 4 of 5
40
A
(5 kN)
5
13
B
(5 kN)
13
12
C
d

CA
Calculate the moment.
M

A
= Fd = (5 kN)d

CA
+ (5 kN)(0) (2) +
5
5
13 13
12
This component of force
passes through point A so
its moment arm is zero.
6
40
B
A
Geometry 7
12
5
13
40
(5 m) cos 40 = 3.830 m
(5 m) sin 40 = 3.214 m
d
d

CA
5 m
C
8 Use similar triangles:
=
Solve and get
d = 1.596 m
9
3.830 m
12
d
5
10
d

CA
= 3.214 m 1.596 m

= 1.618 m
11

333
4.1 Moments in Two-Dimensional Force Systems Example 5, page 5 of 5
Substituting d

CA
= 1.618 m in Eq. 2 gives
M

A
= (5 kN)(1.618 m) = 7.47 kNm
Same result as before.
+
13
12
Observation: Even though sliding the force
vector to point C saved us some work because
one of the force components did not then
appear in the moment equation, this advantage
was not sufficient to compensate for the
additional geometry calculations that were
required; the first solution was easier.
13
12

334
4.1 Moments in Two-Dimensional Force Systems Example 6, page 1 of 6
6. A window is propped open by a stick AB. If a 150-lbin. moment about the
hinge is required to keep the window open, determine the force in the stick.
Assume the force is directed along the axis of the stick.
24 in.
30 in.
12 in.
A
B
C

335
4.1 Moments in Two-Dimensional Force Systems Example 6, page 2 of 6
Express the force of the stick acting
on the window in terms of
components at A.
A
B
C
Line of action of F

AB
F

AB
F

AB
cos
F

AB
sin
1
C
B
A
F

AB
sin
F

AB
cos


d

2
d

1
Calculate M

C
.
M

C
= Fd = (F

AB
cos )d

1
+ (F

AB
sin )d

2
(1) +
2

336
4.1 Moments in Two-Dimensional Force Systems Example 6, page 3 of 6
Geometry
A
B
C
3
C
B
= 49.458
A
d

2
= 30 in. 7.800 in.
= 22.200 in.
d

1
= (12 in.) sin 49.458
= 9.119 in.
Law of cosines for triangle ABC (calculate )
24
2
= 30
2
+12
2
2(30)(12) cos
Solving gives
= 49.458
4
30 in.
24 in.
12 in.
12 in.
30 in.
(12 in.) cos 49.458 = 7.800 in.
7
5
6

337
4.1 Moments in Two-Dimensional Force Systems Example 6, page 4 of 6
Substitute = 49.458, d

1
= 9.119 in., d

2
= 22.200 in., and M

C
= 150 lbin
(the required moment) into the equation for M

C
, Eq. 1:
M

C
= (F

AB
cos )d

1
+ (F

AB
sin )d
2
(Eq. 1 repeated)
150 lbin = (F

AB
cos 49.458)(9.119 in.) + (F

AB
sin 49.458)(22.200 in.)
Solving gives

F

AB
= 6.58 lb Ans.
8

338
4.1 Moments in Two-Dimensional Force Systems Example 6, page 5 of 6
Alternative solution. Slide the force along its
line of action to B and express it in terms of
components.
A
B
C
F
BA
F

BA
cos
F

BA
sin
9
M

C
= (F

BA
sin )(30 in.) + (F

BA
cos )(0) +
30 in.
Calculate the moment about C.
Moment arm is zero (line of
action passes through C).
11
Eliminating the moment
caused by F

BA
cos is the
reason we moved the force
to point B.
12
10

339
4.1 Moments in Two-Dimensional Force Systems Example 6, page 6 of 6
Equating M

C
to the known value 150 lbin.
gives
150 lbin. = (F

BA
sin )(30 in.) + 0
and substituting = 49.458 from Eq. 1 and
then solving yields:
F

BA
= 6.58 lb
(Same answer as before.)
13
Observation: Because the 30-in. moment arm
was given, the alternative solution method was
slightly easier than the first solution method
(Note that in both methods, we have to use the
law of cosines to calculate ).
14

340

4.2 Moments in Three-Dimensional Force Systems
341
4.2 Moments in Three-Dimensional Force Systems Procedures and Strategies, page 1 of 2
A possible
choice for r F
x
z
y
r
r
Best choice for r because r has a simple form
(only one component an x component)
O
Procedures and Strategies for Solving Problems Involving
Moments in Three-Dimensional Force Systems
1. To calculate the moment of a force F about a point O,
a) Express F in rectangular component form.
b) Define a position vector r, with tail at point O and head at any point
along the line of action of F. If you have more than one possible
choice for r, choose the one that gives the simplest form for r.
Express r in rectangular component form.
c) Evaluate the cross product: M
O
= r x F (If you have a scientific
graphing calculator, use its built-in cross-product function).
2. To determine the shortest distance d between a point A and a given
line, assume that a force of unknown magnitude F acts along the line,
and then make use of the fact that two different formulas exist for
calculating the magnitude of the moment M
A
about A:
M
A
= Fd (1)
and
M
A
= r x F
where r is a positive vector with tail at A and head on the line. Then
a) find a unit vector u parallel to the line, and express it in rectangular
component form;
b) express the force as F = Fu,
r
u
F = Fu
d
A

342
4.2 Moments in Three-Dimensional Force Systems Procedures and Strategies, page 2 of 2
c) express r in rectangular component form,
d) compute the cross product
r x F = r x (Fu)
= F(r x u)
Then Eqs. 1 and 2 give
Fd = F r x u
Thus to calculate the shortest distance d from point
A to a line, just calculate the magnitude of the cross
product of r and u.
Cancel F.

343
4.2 Moments in Three-Dimensional Force Systems Problem Statement for Example 1
200 mm
A
F = 50 N
1. Use the cross-product definition of the moment of a force to determine the
moment of the force about point A. Also, compare the sign of the result with
that obtained from the scalar definition of positive moment, M = Fd.
B
H E A V Y D U T Y 2 5 0 M M C H I N A
+
30
y
x
z

344
4.2 Moments in Three-Dimensional Force Systems Problem Statement for Example 2
2. A force F = 20 N is applied to the end of a string of length L. The other end of
the string is tied to the handle of a wrench as shown. Use the cross-product
definition of the moment to determine the moment of F about point A. Discuss the
effect of distance L on your answer.
30
x
z
y
C
L
H E A V Y D U T Y 2 5 0 M M C H I N A
B
F = 20 N
A
200 mm

345
4.2 Moments in Three-Dimensional Force Systems Problem Statement for Example 3
3. A shower/bathtub grab bar is being pulled by a force F = 30 lb as
shown. Determine the moment of F about the support A. Also determine
the coordinate direction angles of the moment vector and interpret the
result.
16 in.
8 in.
5 in.
A
B
z
x
y
F = 30 lb
40
60

346
4.2 Moments in Three-Dimensional Force Systems Problem Statement for Example 4
4. A force F = 15 N acting parallel to the z axis is applied to the
handle of a socket wrench to turn a bolt at A. Determine the
moment of the force about the point A. Also, state which
component of the moment tends to turn the bolt.
100 mm
80 mm
A
B
x
y
z
F = 15 N
C

347
4.2 Moments in Three-Dimensional Force Systems Problem Statement for Example 5
5. Pulley B is used to drive pulley C. Determine the resultant
moment about bearing A produced by the belt forces acting on
pulley B. Also, interpret your result.
z
x
y
30
Belt forces
A
B
C
D
E
F
Q = 55 N
P = 30 N
40 mm
Radius = 70 mm

348
4.2 Moments in Three-Dimensional Force Systems Problem Statement for Example 6
6. A child on a bicycle collides with a mailbox and
exerts the force F shown. If the base of the pole at O
will fail if the magnitude of the moment there exceeds
60 Nm, determine if the mailbox will fall over.
250 mm
75 mm
A
O
x
y
z
F = {80i + 12j 10k} N
900 mm

349
4.2 Moments in Three-Dimensional Force Systems Problem Statement for Example 7
7. Copper tubing emerges from the wall at A and is subjected to a
force F at its free end B. The tubing will fail if the magnitude of
the moment at A exceeds 3 Nm. Determine the largest value of
the force F that can be applied to the free end of the tubing.
30
40
250 mm
300 mm
A
B
x
z
y
F
35
200 mm

350
4.2 Moments in Three-Dimensional Force Systems Problem Statement for Example 8
8. Two forces, P = 60 N and Q = 80 N act on the vertices of a cube as
shown. Determine the moment of each force about point O, if the length
of each edge of the cube is 2 m. Also, determine the shortest distance
from O to the line BF.
x
z
y
A B
C
D
E F
G
O
P = 60 N
Q = 80 N
Each edge is
2 m long.

351
4.2 Moments in Three-Dimensional Force Systems Problem Statement for Example 9
9. Determine the moment about the screw at A of the force F = 2 N
applied to the sheet-metal bracket shown. Also, determine the shortest
distance from A to the line connecting B and C.
x
y
z
60 mm
100 mm
70 mm
80 mm
30 mm
60 mm 50 mm
A
B
C
F = 2 N
25

352
4.2 Moments in Three-Dimensional Force Systems Problem Statement for Example 10
10. If the tension in the cable BC is T = 80 lb,
determine the moment about point A of the cable force
acting on the frame at point B. Also, determine the
shortest distance from A to the line through B and C.
32 in.
18 in.
10 in.
A
B
x
y
z
T = 80 lb
C
12 in.

353
4.2 Moments in Three-Dimensional Force Systems Example 1, page 1 of 3
200 mm
200 mm
A
F = 50 N
1
1. Use the cross-product definition of the moment of a force to determine the
moment of the force about point A. Also, compare the sign of the result with
that obtained from the scalar definition of positive moment, M = Fd.
B
H E A V Y D U T Y 2 5 0 M M C H I N A
Introduce a position vector r
AB

with tail at A and head at B.
B
A
r
AB
= { 200i} mm
x
y
+
2 Express the force in rectangular components.
z
y
x
F = 50 N
30
(50 N) sin 30 = 25 N
(50 N) cos 30 = 43.30 N
F = {43.30i 25j} N (1)
H E A V Y D U T Y 2 5 0 M M C H I N A
y
x
z
z
30

354
4.2 Moments in Three-Dimensional Force Systems Example 1, page 2 of 3
6 M
A
= ( 200 mm)( 25 N)k
= {5000k} Nmm
= {5k} Nm Ans.
3 Use the vector cross product to compute the moment
about point A.
M
A
= r
AB
F
= { 200i} mm {43.30i 25j} N
= [ 200(43.30) i i + ( 200)( 25) i j ] Nmm
= 0, because cross
product of parallel
vectors is zero
= k, by the right-hand rule
4
j
k
i
A memory aid for cross products:
5
Assign a plus sign if the product is in
the order indicated by the arrowheads;
minus sign otherwise
k
i
j
i j = +k
i k = j
from i to j
thumb points out
of i-j plane
fingers of right hand
curl from i to j
i
k
j
Apply the
right-hand rule.

355
4.2 Moments in Three-Dimensional Force Systems Example 1, page 3 of 3
k
50 N
8 The 50-N force tends to rotate the
wrench counterclockwise about
the axis (through A) defined by
the unit vector k. By the
right-hand rule, this definition of
positive moment reduces to the
usual sign convention for positive
moment in coplanar problems.
7 Display the M
A
vector (a double-headed arrow)
z
y
x
M
A
= {+5k} Nm
y
50 N
M
A
= {+5k} Nm
x
+
direction of fingers
thumb points
out-of-plane
Ans.
H E A V Y D U T Y 2 5 0 M M C H I N A
H E A V Y D U T Y 2 5 0 M M C H I N A
A
A
z
30
30

356
4.2 Moments in Three-Dimensional Force Systems Example 2, page 1 of 4
1
2. A force F = 20 N is applied to the end of a string of length L. The other end of
the string is tied to the handle of a wrench as shown. Use the cross-product
definition of the moment to determine the moment of F about point A. Discuss the
effect of distance L on your answer.
Introduce a position vector r
AC

with head at C and tail at A.
F = 20 N
B
A
C
r
AC
z
y
x
H E A V Y D U T Y 2 5 0 M M C H I N A
30
x
z
y
C
L
H E A V Y D U T Y 2 5 0 M M C H I N A
B
F = 20 N
A
200 mm
30

357
4.2 Moments in Three-Dimensional Force Systems Example 2, page 2 of 4
200 mm
A
2
B
Determine the rectangular components of r
AC
.
C
y
z
x
r
AC
L cos 30
r
AC
= (L cos 30 + 200 mm)i (L sin 30)j (1) 5
3
L sin 30 4
L
x
y
z
C
F = 20 N
30
Determine the rectangular
components of the force F
6
F = (20 N) cos 30 i (20 N) sin 30 j (2)
H E A V Y D U T Y 2 5 0 M M C H I N A
30

358
4.2 Moments in Three-Dimensional Force Systems Example 2, page 3 of 4
7
M
A
= r
AC
F
= {( L cos 30 200)i L sin 30 j} mm { 20 cos 30 i 20 sin 30 j} N
= [( L cos 30 00)( 20 cos 30)(i i)
= 0
+ ( L cos 30 00)( 20 sin 30)(i j)
= k
+ ( L sin 30 )( 20 cos 30)(j i)
= k
+ ( L sin 30)( 20 sin 30)(j j) ] Nmm
= 0

j
k
i
= k[( L cos 30 200)( 20 sin 30) ( L sin 30)( 20 cos 30)] Nmm
= k[( L cos 30)( 20 sin 30) 200( 20 sin 30)
( L sin 30)( 20 cos 30)] Nmm
The terms involving L drop out
= {2000k} Nmm = {2k} Nm Ans.
Use the cross product to compute the moment.

359
4.2 Moments in Three-Dimensional Force Systems Example 2, page 4 of 4
Ans.
30
30
H E A V Y D U T Y 2 5 0 M M C H I N A
Note: All position vectors
and force vectors are in
the xy plane.
z
x
y
3
2
1
F
r
A3
r
A1
r
A2
r
AC
C
Discussion: Distance L does not appear in the answer
for M
A
. Thus the result is valid for all values of L,
and so the head of the position vector can be located
anywhere (for example, points 1, 2, 3, or C in the
figure) on the line of action of the force.
B
8
A

360
4.2 Moments in Three-Dimensional Force Systems Example 3, page 1 of 6
3. A shower/bathtub grab bar is being pulled by a force F = 30 lb as
shown. Determine the moment of F about the support A. Also determine
the coordinate direction angles of the moment vector and interpret the
result.
16 in.
8 in.
5 in.
A
B
z
x
y
F = 30 lb
40
60

361
4.2 Moments in Three-Dimensional Force Systems Example 3, page 2 of 6
16 in.
8 in.
5 in.
A
B
z
x
y
40
60
Introduce a position vector with tail at A and head at B.
r
AB
= { 16i + 8j + 5k} in. (1)
1
r
AB
F = 30 lb

362
4.2 Moments in Three-Dimensional Force Systems Example 3, page 3 of 6
Determine the rectangular
components of the force F.
2
F

y
= (30 lb) cos 60
= 15 lb (2)
3
(30 lb) sin 60 4
F

z
= (30 lb)(sin 60)(sin 40)
= 16.70 lb (3)
5
F

x
= (30 lb)(sin 60)(cos 40)
= 19.90 lb (4)
6
Component form of F,
from Eqs. 2-4:
F = { 19.90i 15j + 16.70k} lb (5)
7
40
60
x
y
z
F = 30 lb

363
4.2 Moments in Three-Dimensional Force Systems Example 3, page 4 of 6
8
M
A
= r
AB
F
= { 16i + 8j + 5k } { 19.90i 15j + 16.70k}
Because both vectors each have three non-zero
components, evaluating the cross products is
easier if we use the determinant form.
9
i j k
M
A
= 16 8 5
19.90 15 16.70
Expand the determinant in terms of the first row,
remembering to insert the minus sign for the j term.
10
8 5 16 5 16 8
M
A
= i j + k
15 16.70 19.90 16.70 19.90 15
= i[8(16.70) 5( 15)] j[ 16(16.70) 5( 19.90)] + k[ 16( 15) 8( 19.90)]
= {208.6i +167.7j + 399.2k} lbin. Ans.
Calculate the moment.

364
4.2 Moments in Three-Dimensional Force Systems Example 3, page 5 of 6
Observation: The above procedure is tedious and error prone. A much
better approach is to use a calculator with a built-in function for evaluating
the cross product of two vectors. If such a calculator is available, all you
need to do is enter the components of the vectors and then let the calculator
perform the arithmetic. As a result, typical errors such as missing a minus
sign or mis-copying a number from one line to the next are avoided.
11
To determine the coordinate direction angles, first determine the
magnitude of the moment.
M

A
= (208.6)
2
+ (167.7)
2
+ (399.2)
2

= 480.6 lbin.
480.6
399.2
480.6
167.7
480.6
208.6
Coordinate direction angles
= cos
-1
= cos
-1
= 64.3 Ans.
= cos
-1
= cos
-1
= 69.6 Ans.
= cos
-1
= cos
-1
= 33.8 Ans.
z
M

A
M

A
M

A
M

A
M

A
y
M

A
x

365
4.2 Moments in Three-Dimensional Force Systems Example 3, page 6 of 6
16 in.
5 in.
A
B
z
x
y
40
60
64.3
69.6
33.8
M

A
= 480.6 lbin.
Interpretation: The force F = 30 lb
produces a moment of 480.6 lbin. about
point A. This moment tends to rotate the
grab bar about an axis defined by the
moment vector.
12
Axis of rotation
Ans.
F = 30 lb

366
4.2 Moments in Three-Dimensional Force Systems Example 4, page 1 of 3
4. A force F = 15 N acting parallel to the z axis is applied to the
handle of a socket wrench to turn a bolt at A. Determine the
moment of the force about the point A. Also, state which
component of the moment tends to turn the bolt.
100 mm
80 mm
A
B
x
y
z
F = 15 N
C

367
4.2 Moments in Three-Dimensional Force Systems Example 4, page 2 of 3
Introduce a position vector
with head at B and tail at A.
r
AB
= {80i 100j} mm
1
F = 15 N
80 mm
z
A
y
100 mm
B
x
Calculate the moment.
M
A
= r
AB
F
= (80i 100j) { 15k}

= 80( 15)(i k) + ( 100)( 15)(j k)
= j = i
= ( 1200)( j) + (1500)i
= {1500i + 1200j} Nmm
= {1.5i + 1.2j} Nm Ans.
2
i
k
j
(F points in negative z-direction.)
r
AB
C

368
4.2 Moments in Three-Dimensional Force Systems Example 4, page 3 of 3
Display the moment vector. 3
The component that tends to
rotate the shaft AC of the
wrench about the x axis
(and thus turn the bolt) is
M

A
= 1.5 Nm Ans.
4
100 mm
15 N
B
x
y
z
A
80 mm
M

A
= 1.5 Nm
x
M

A
= 1.2 Nm
y
M

A
x
C

369
4.2 Moments in Three-Dimensional Force Systems Example 5, page 1 of 6
5. Pulley B is used to drive pulley C. Determine the resultant
moment about bearing A produced by the belt forces acting on
pulley B. Also, interpret your result.
z
x
y
30
Belt forces
A
B
C
D
E
F
Q = 55 N
P = 30 N
40 mm
Radius = 70 mm

370
4.2 Moments in Three-Dimensional Force Systems Example 5, page 2 of 6
z
x
y
30
Belt forces
A
B
C
D
E
F
Q = 55 N
P = 30 N
40 mm
Radius = 70 mm
1 Introduce position vectors with
tails at A and heads at E and F.
r
AF
r
AE
2 From the figure,
r
AE
= {40i 70j} mm (1)

371
4.2 Moments in Three-Dimensional Force Systems Example 5, page 3 of 6
30
z
x
y
30
Belt forces
A
B C
D
E
F
Q = 55 N
P = 30 N
40 mm
Radius = 70 mm
3 To determine the components of r

AF
, consider a
view of the pulley from the positive x axis:
r
AF
r
AE
7 In component form, from Eqs. 2 and 3,
r
AF
= {40i + 60.62j 35k} mm (4)
Q = 55 N
P = 30 N
z
y
B
F
6
4
5
r
AFz
= (70 mm) cos 60 = 35 mm (3)
Angle = 90 30 = 60
r
AFy
= (70 mm) sin 60
= 60.62 mm (2)
Radius = 70 mm

372
4.2 Moments in Three-Dimensional Force Systems Example 5, page 4 of 6
z
x
y
30
Belt forces
A
B
C
D
E
F
Q = 55 N
P = 30 N
40 mm
Radius = 70 mm
r
AF
r
AE
8 Express the forces in rectangular
components.
Q = (55 N) sin 30j + (55 N) cos 30k
= {27.5j + 47.63k} N (5)
P = {30k} N (6)

373
4.2 Moments in Three-Dimensional Force Systems Example 5, page 5 of 6
M
A
= r
AF
Q + r
AE
P
9
i j k i j k
M
A
= 40 60.62 35 + 40 70 0
0 27.5 47.63 0 0 30
60.62 35 40 35 40 60.62
= i j + k
27.5 47.63 0 47.63 0 27.5
= i[60.62(47.63) ( 35)(27.5)] j[40(47.63) ( 35)(0)] + k[40(27.5) 60.62(0)]
+ i[ 70(30) 0(0)] j[40(30) 0(0)] + k[40(0) 70)(0)]
= {1750i 3105j + 1100k} Nmm
= {1.750i 3.105j + 1.100k} Nm Ans.
70 0 40 0 40 70
+ i j + k
0 30 0 30 0 0
Calculate the resultant moment.

374
4.2 Moments in Three-Dimensional Force Systems Example 5, page 6 of 6
40 mm
z
x
y
30
Belt forces
A
B
C
55 N
30 N
M

x
= 1.750 Nm
Interpretation: The belt forces acting on pulley B tend to
rotate the entire structure with a 3.73 Nm moment about an
axis defined by the direction of the moment vector M
A
.
The 1.750 Nm x component of M
A
is the component of
moment that rotates the shaft and drives pulley C.
11
Ans.
The magnitude of the moment is
M

A
= (1.750)
2
+ ( 3.105)
2
+ (1.100)
2
= 3.73 Nm
10

375
4.2 Moments in Three-Dimensional Force Systems Example 6, page 1 of 3
6. A child on a bicycle collides with a mailbox and
exerts the force F shown. If the base of the pole at O
will fail if the magnitude of the moment there exceeds
60 Nm, determine if the mailbox will fall over.
250 mm
75 mm
A
O
x
y
z
F = {80i + 12j 10k} N
900 mm

376
4.2 Moments in Three-Dimensional Force Systems Example 6, page 2 of 3
Introduce a position vector with tail at O and
head at A.
r
OA
= {250i + 900j + 75k} mm
1
i j k
= 250 900 75
80 12 10
M
O
= r
OA
F
2
r
OA
= i[900( 10) (12)(75)] j[250( 10) 80(75)] + k[250(12) 80(900)]
= { 9900i + 8500j 69000k} Nmm
= { 9.9i + 8.5j 69.0k} Nm
900 75 250 75 250 900
= i j + k
12 10 80 10 80 12
Calculate the moment.
250 mm
75 mm
A
O
x
y
z
F = {80i + 12j 10k} N
900 mm

377
4.2 Moments in Three-Dimensional Force Systems Example 6, page 3 of 3
Magnitude of moment
M

O
= ( 9.9)
2
+ (8.5)
2
+ ( 69.0)
2
= 70.2 Nm
3
Because 70.2 Nm exceeds the 60 Nm
maximum allowable moment, the
mailbox will fall over.
4
Ans.

378
4.2 Moments in Three-Dimensional Force Systems Example 7, page 1 of 6
7. Copper tubing emerges from the wall at A and is subjected to a
force F at its free end B. The tubing will fail if the magnitude of
the moment at A exceeds 3 Nm. Determine the largest value of
the force F that can be applied to the free end of the tubing.
30
40
250 mm
300 mm
A
B
x
z
y
F
35
200 mm

379
4.2 Moments in Three-Dimensional Force Systems Example 7, page 2 of 6
250 mm
200 mm
300 mm
35
A
B
x
z
y
F
1 Introduce a position vector r
AB
with tail at A and head at B.
r

AB
30
40
35

380
4.2 Moments in Three-Dimensional Force Systems Example 7, page 3 of 6
250 mm
200 mm
300 mm
A
B
x
z
y
F
2 Determine the
components of r

AB
.
r

AB
3 r

ABx
= 200 mm + 250 mm = 450 mm (1)
250 mm
D
C
30
40
35

381
4.2 Moments in Three-Dimensional Force Systems Example 7, page 4 of 6
7 In component form, from Eqs. 1-3,
r

AB
= {450i 172.1j + 245.7k} mm (4)
r

ABz
= (300 mm) cos 35 = 245.7 mm (3) 6
4
r

ABy
= (300 mm) sin 35
= 172.1 mm (2)
5
To determine the y and z components of r
AB
,
consider a view from the positive x axis.
r

AB
35
300 mm
y
z
x, B, D
C, A

382
4.2 Moments in Three-Dimensional Force Systems Example 7, page 5 of 6
8 Determine the components of F.
F sin 40 10
F

z
= F sin 40 sin 30

= 0.3214F (6)
11 B
x
z
y
F
F

y
= F cos 40
= 0.7660F (5)
9
F

x
= F sin 40 cos 30

= 0.5567F (7)
12
13 In component form, from Eqs. 5-7,
F = F{0.5567i 0.7660j + 0.3214k} (8)
30
40

383
4.2 Moments in Three-Dimensional Force Systems Example 7, page 6 of 6
M
A
= r
AB
F
14
i j k
= 450 172.1 245.7
0.5567F 0.7660F 0.3214F
172.1 245.7 450 245.7 450 172.1
= i j + k
0.7660F 0.3214F 0.5567F 0.3214F 0.5567F 0.7660F
= (132.9F)i (7.8F)j (248.9F)k
M

A
= (132.9F)
2
+ ( 7.8F)
2
+ ( 248.9F)
2
= F (132.9)
2
+ ( 7.8)
2
+ ( 248.9)
2
= 282.3F Nmm
= 0.2823F Nm
15
Equate M

A
to the largest allowable moment, 3 Nm:
M

A
= 3 Nm
0.2823F Nm
Solving gives
F = 10.6 N Ans.
16
Calculate the resultant moment. Compute the magnitude.

384
4.2 Moments in Three-Dimensional Force Systems Example 8, page 1 of 6
8. Two forces, P = 60 N and Q = 80 N act on the vertices of a cube as
shown. Determine the moment of each force about point O, if the length
of each edge of the cube is 2 m. Also, determine the shortest distance
from O to the line BF.
x
z
y
A B
C
D
E F
G
O
P = 60 N
Q = 80 N
Each edge is
2 m long.

385
4.2 Moments in Three-Dimensional Force Systems Example 8, page 2 of 6
x
z
y
A B
C
D
E F
G
O
P = 60 N
Q = 80 N
Each edge is
2 m long.
To determine the
moment of the force
P about point O, we
have several choices
of position vector
(Recall that the head
of the vector can lie
anywhere on the line
of action of P).
1
Select the position vector with
the simpler form.
r
OD
= {2i + 2j +2k} m
r
OE
= {2j} m (1)
2
Simpler (only one component)
r
OD
r
OE

386
4.2 Moments in Three-Dimensional Force Systems Example 8, page 3 of 6
x
z
y
A B
C
D
E F
G
O
P = 60 N
Q = 80 N
Each edge is
2 m long.
Determine the components of
the force P by introducing a
position vector r
DE
:
r
DE
= { 2i 2k} m
3
P = (60 N)
= (60 N)
= { 42.43i 42.43k} N (2)
4
r
DE
2i 2k
( 2)
2
+ ( 2)
2
r
DE
r

DE
The force P has magnitude 60 N and
points in the direction from D to E, so

387
4.2 Moments in Three-Dimensional Force Systems Example 8, page 4 of 6
5 Calculate the moment.
M
OP
= r
OE
P
= {2j} { 42.43i 42.43k}
= (2)( 42.43)(j i) (2)(42.43)(j k)
= { 84.86i + 84.86k} Nm Ans.
j
k
i
= k = i
Since r
OE
has only one component, it is easier to
calculate cross products of base vectors individually
rather than use the determinant approach to calculating
the cross product.

388
4.2 Moments in Three-Dimensional Force Systems Example 8, page 5 of 6
Calculate the moment.
M

OQ
= r

OB
Q
8
i j k
= 2 0 2
0 56.57 56.57
0 2 2 2 2 0
= i j + k
56.57 56.57 0 56.57 0 56.57
= {113.14i 113.14j 113.14k} Nm (3) Ans.
To determine the moment of the force Q
about point O, we could introduce either
position vector r
OB
or r
OF
. Let's
arbitrarily choose r
OB
.
r
OB
= {2i + 2k} m
6
Determine the components of
force Q by noting it has
magnitude 80 N and points from
F to B so
Q = (80 N)
= (80 N)
= { 56.57j + 56.57k} N
7
2j + 2k
( 2)
2
+ (2)
2
r
FB
r

FB
x
z
y
A B
C
D
E F
G
O
P = 60 N
Q = 80 N
r
FB
r
OB
r
OF
Each edge is
2 m long.

389
4.2 Moments in Three-Dimensional Force Systems Example 8, page 6 of 6
x
z
y
A B
C
D
E F
G
O
P = 60 N
Q = 80 N
Each edge is
2 m long.
Finally, to determine the
shortest distance from point O
to line FB, consider the plane
defined by O and FB:
9
From the figure it is clear that of the various distances
d

1
, d

2
, d

3
, ..., from O to line FB, the perpendicular
distance d is the shortest. But we also know that, by
the scalar definition of moment, the moment of Q
about point O equals the perpendicular distance from O
to the line of action of Q times Q. That is,
M

QO
= Qd
Thus,
d =
=

= 2.45 m Ans.
10
Q = 80 N
B
F
O
d

1
d

2
d

3
d
(113.14)
2
+ ( 113.14)
2
+ ( 113.14)
2
80
M

QO
Q
d

390
4.2 Moments in Three-Dimensional Force Systems Example 9, page 1 of 6
9. Determine the moment about the screw at A of the force F = 2 N
applied to the sheet-metal bracket shown. Also, determine the shortest
distance from A to the line connecting B and C.
x
y
z
60 mm
100 mm
70 mm
80 mm
30 mm
60 mm 50 mm
A
B
C
F = 2 N
25

391
4.2 Moments in Three-Dimensional Force Systems Example 9, page 2 of 6
To calculate the moment
we could use either the
position vector r
AB
or r
AC
.
1
r
AC
has a single component,
r
AC
= (50 mm + 60 mm)i
= { 110i} mm (1)
while r
AB
has two components, so let's
choose the simpler, r
AC
.
2
x
y
z
A
C
F = 2 N
r
AC
r
AB
B
25
50 mm 60 mm

392
4.2 Moments in Three-Dimensional Force Systems Example 9, page 3 of 6
25
B
25
x
30 mm
B
A
Next, we need to calculate the components
of the force, F. To do this, we first need to
calculate the coordinates of point B. By
inspection, the x coordinate of B is 60 mm.
3
To find the y and z
coordinates of point B,
consider a view from
the positive x-axis.
4
y coordinate of B
= 30 mm 70 mm 42.26 mm
= 142.26 mm
5
z coordinate of B
= 90.63 mm + 80 mm
= 170.63 mm
6
y
z
60 mm
100 mm
70 mm
80 mm
60 mm 50 mm
A
C
F = 2 N
100 mm
80 mm
30 mm
70 mm
(100 mm) sin 25 = 42.26 mm
(100 mm) cos 25
= 90.63 mm
z
y

393
4.2 Moments in Three-Dimensional Force Systems Example 9, page 4 of 6
To determine the components of the force, introduce the
position vector r
BC
.
r
BC
= ( 50 mm 60 mm)i
+ [0 ( 142.26 mm)]j
+ (0 170.63 mm)k
= { 110i + 142.26j 170.63k} mm (2)
7
F = (2 N)
= (2 N)
= { 0.887i + 1.148j 1.377k} N
8
110i + 142.26j 170.63k
( 110)
2
+ (142.26)
2
+ ( 170.63)
2
r
BC
r

BC
y
z
60 mm
100 mm
70 mm
80 mm
60 mm 50 mm
A
C
F = 2 N
C coordinates
( 50 mm, 0, 0)
B coordinates (60, 142.26 mm, 170.63 mm)
r
BC
B
25
The force F has magnitude 2 N and
points from B to C so
x
30 mm

394
4.2 Moments in Three-Dimensional Force Systems Example 9, page 5 of 6
9
M
A
= r
AC
F
= { 110i} { 0.887i + 1.148j 1.377k}
= ( 110)( 0.887)(i i) + ( 110)(1.148)(i j) + ( 110)( 1.377)(i k)
= { 151.4j 126.3k} Nmm (3) Ans.
= j = k = 0
k
j
i
Calculate the moment.

395
4.2 Moments in Three-Dimensional Force Systems Example 9, page 6 of 6
To determine the shortest distance
between point A and line BC, consider the
plane formed by A and BC.
10
Magnitude of moment about A = Fd
or,
( 151.4)
2
+ ( 126.3)
2
N mm = (2 N)d
Solving gives,
d = 98.6 mm Ans.
F = 2 N
d
A
B
C
11
by Eq. 3
y
z
60 mm
100 mm
70 mm
80 mm
60 mm 50 mm
A
C
F = 2 N
B
25
x
30 mm

396
4.2 Moments in Three-Dimensional Force Systems Example 10, page 1 of 5
10. If the tension in the cable BC is T = 80 lb,
determine the moment about point A of the cable force
acting on the frame at point B. Also, determine the
shortest distance from A to the line through B and C.
32 in.
18 in.
10 in.
A
B
x
y
z
T = 80 lb
C
12 in.

397
4.2 Moments in Three-Dimensional Force Systems Example 10, page 2 of 5
To calculate the
moment, we could
use either position
vector r
AB
or r
AC
.
1
Since r
AC
has only one
component while r

AB
has three,
let's use r
AC
.
r
AC
= {12j} in. (1)
2
32 in.
18 in.
10 in.
A
B
x
y
z
T = 80 lb
C
r
AC
r
AB
12 in.

398
4.2 Moments in Three-Dimensional Force Systems Example 10, page 3 of 5
To determine the components of the force, T,
introduce the position vector r
BC
.
3
T = (80 lb)
= (80 lb)
= { 69.62i + 4.35j 39.16k} lb
4
32i + 2j 18k
( 32)
2
+ 2
2
+ ( 18)
2
r
BC
r

BC
r
BC
= (0 32 in.)i + (12 in. 10 in.)j + (0 18 in.)k
= { 32i + 2j 18k} in.
32 in.
18 in.
10 in.
A
B
x
y
z
T = 80 lb
C
r
BC
The force T has magnitude 80 lb and
points from B to C so
12 in.

399
4.2 Moments in Three-Dimensional Force Systems Example 10, page 4 of 5
5 Calculate the moment.
M
A
= r
AC
T
= 12j { 69.62i + 4.35j 39.16k}
= (12)( 69.62)(j i) + (12)(4.35)(j j) + (12)( 39.16)(j k)
= { 469.92i + 835.44k} lbin. Ans. (2)
= k = i = 0
k
j
i
by Eq. 1

400
4.2 Moments in Three-Dimensional Force Systems Example 10, page 5 of 5
Finally, to determine the shortest distance
between point A and line BC, consider the
plane formed by A and BC.
6
Magnitude of moment about A = Td
or,
( 469.92)
2
+ (835.44)
2
= (80 lb)d
Solving gives,
d = 11.98 in. Ans.
7
32 in.
18 in.
10 in.
A
B
x
y
z
T = 80 lb
C
T = 80 lb
d
Extension of BC
A
B
C
by Eq. 2
12 in.

401

4.3 Moment of a Couple
402
4.3 Moment of a Couple Procedures and Strategies, page 1 of 1
C
d
B
A
F
e
F
Procedures and Strategies for Solving Problems
Involving Couples
Calculate the moment of a couple by summing
moments of the two forces about any point.
The result is independent of which point you
choose, but often you can save work by
choosing the point to lie on the line of action of
one of the forces.
M
A
= Fd
M
B
= Fd
M
C
= Fe + F(e + d)
= Fd
+
+
+

403
4.3 Moment of a Couple Problem Statement for Example 1
1. Two swimmers on opposite sides of a boat
attempt to turn the boat by pushing as shown.
Determine the couple moment about
a) point A on the bow,
b) point B on the stern, and
c) point C.

Also state what general principles your results
demonstrate.
50 N
B
C
A
50 N
1 m
1.5 m
2 m

404
4.3 Moment of a Couple Problem Statement for Example 2
30 N
20 N
30 N
20 N
A D
C B
30
4 m
8 m
30
2. Determine the magnitude and sense of the resultant couple
moment acting on the rectangular plate.

405
4.3 Moment of a Couple Problem Statement for Example 3
3. Determine the value of the force P such that the
resultant couple moment of the two couples acting on
the beam is 900 lbft clockwise.
200 lb
200 lb P
P
E
C
D B
A
5 ft
3
4
5
3
4
5
0.5 ft
7 ft 4 ft

406
4.3 Moment of a Couple Problem Statement for Example 4
4. The wrench applies a 10 Nm couple
moment to the bolt. To prevent the plate
from rotating, two 2-N forces are applied as
shown. Determine the distance s such that the
resultant couple moment acting on the plate
and bolt is zero.

2 N 2 N
B A
s
C
10 Nm
ctvx 0u1x
b
8
'

407
4.3 Moment of a Couple Problem Statement for Example 5
P
5. Two cords are wrapped around pegs attached to a
board as shown. Determine the value of such that P is
as small as possible while still producing a resultant
couple moment of zero. Also determine the value of P
corresponding to this value of . Neglect the size of the
pegs.
2 m
A B
D
C
80 N
P
80 N
4 m

408
4.3 Moment of a Couple Problem Statement for Example 6
A
B C
D
z
x
y
80 N
80 N
E
6. A plumber uses two pipe wrenches so that he can
loosen pipe BC from pipe AB without also
loosening pipe AB from the connection at the wall,
A. Determine the moment of the forces about a) A
and b) D. Also state what general principles your
results demonstrate.
250 mm 130 mm
175 mm

409
4.3 Moment of a Couple Problem Statement for Example 7
7. Determine the couple moment produced by the two
forces applied to the handle of the crank as shown.
G = 20 N
B
A
25
25
y
z
x
F = 20 N
40
40
300 mm

410
4.3 Moment of a Couple Problem Statement for Example 8
z
A
B
C
D
6 lb
6 lb
10 lb
E
10 lb
F
5 in.
20 in.
30
8. Determine the resultant couple moment of the two couples acting
on the block.
x
y

411
4.3 Moment of a Couple Problem Statement for Example 9
z
y
x
70 mm
F = {16i + 12k} N
9. Force F is applied to the doorknob as shown. Determine
where on the surface of the door a force G = F should be
applied so that the resulting couple moment has a y
component of 2 Nm but no x or z components.
650 mm
700 mm

412
4.3 Moment of a Couple Example 1, page 1 of 3
1. Two swimmers on opposite sides of a boat
attempt to turn the boat by pushing as shown.
Determine the couple moment about
a) point A on the bow,
b) point B on the stern, and
c) point C.

Also state what general principles your results
demonstrate.
50 N
B
C
A
50 N
1 m
1.5 m
2 m

413
4.3 Moment of a Couple Example 1, page 2 of 3
ns.
ns.
2
1 Calculate the moment about A.
M
A
= (50 N)(1 m) (50 N)(1 m + 2 m)
= 100 Nm
Calculate the moment about B.
M
B
= (50 N)(2 m + 1.5 m) (50 N)(1.5 m)
= 100 Nm
1.5 m
50 N
C
B
50 N
50 N
C
A
50 N
1 m
2 m
2 m

414
4.3 Moment of a Couple Example 1, page 3 of 3
ns.
3 Moment about C
M
C
= (50 N)(2 m)
= 100 Nm
ns.
General principles demonstrated:
1) The couple moment is the same about every
point, and
2) The calculation of the couple moment is
simplified if the moment is calculated about a
point on the line of action of one of the forces
making up the couple.

4
50 N
2 m
C
50 N

415
4.3 Moment of a Couple Example 2, page 1 of 3
30 N
Resolve the inclined forces into rectangular
components.
1
20 N
30 N
20 N
A D
C B
B C
D A
20 N 30 N
20 N
30 N
(20 N) sin30
= 10 N
(20 N) cos 30
= 17.32 N
30
(20 N) sin 30
= 10 N
(20 N) cos 30
= 17.32 N
30
4 m
8 m
30
2. Determine the magnitude and sense of the resultant couple
moment acting on the rectangular plate.
30

416
4.3 Moment of a Couple Example 2, page 2 of 3
30 N
8 m
10 N
30 N 10 N
A D
C B
Moment of the couple formed by the forces at A and C.
M
C
= (17.32 N)(4 m) (10 N)(8 m)
= 10.72 Nm (2)
This moment value would be the same for any other point besides C.
Moment of the couple formed by the forces at B and D.
M
D
= (30 N)(8 m)
= 240 Nm (1)
This moment value would be the same for any other point
besides D.
3
4
2 Calculate the moments
17.32 N
17.32 N
4 m

417
4.3 Moment of a Couple Example 2, page 3 of 3
A D
C B
Arbitrary location
Since both couple moments are the same about all
points, we can move them to any arbitrary point
we choose and then add them to get the resultant
couple moment.
M = M
C
+ M
D
= 10.72 + 240
= 229 Nm
5
6
No subscript because
valid for all points
ns.
229 Nm

418
4.3 Moment of a Couple Example 3, page 1 of 2
3. Determine the value of the force P such that the
resultant couple moment of the two couples acting on
the beam is 900 lbft clockwise.
200 lb
200 lb P
P
E
C
D B
A
5 ft
3
4
5
3
4
5
0.5 ft
7 ft 4 ft

419
4.3 Moment of a Couple Example 3, page 2 of 2
5 ft 7 ft
A
B D
C
E
160 lb
P
P
ns.
2 Compute the moment of the couple at C and E.
M
E
= (120 lb)(0.5 ft) + (160 lb)(5 ft + 7 ft)
= 1860 lbft (2)
3
Since the resultant couple moment is specified to
be 900 lbft clockwise, Eq. 3 becomes
900 lbft = P(5) + 1860
Solving gives

P = 552 lb
Since the couple moments are the same about all
points, we can move them both to any arbitrary
point we choose and then add them to get the
resultant couple moment:
M = P(5) + 1860 (3)
4
1
Compute the moment of the couple at B and D.
M
D
= P(5 ft) (1)
120 lb
160 lb
120 lb
0.5 ft

420
4.3 Moment of a Couple Example 4, page 1 of 2
C
10 Nm
ctvx 0u1x
b
8
'
4. The wrench applies a 10 Nm couple
moment to the bolt. To prevent the plate
from rotating, two 2-N forces are applied as
shown. Determine the distance s such that the
resultant couple moment acting on the plate
and bolt is zero.

2 N 2 N
B A
s

421
4.3 Moment of a Couple Example 4, page 2 of 2
ctvx 0u1x
b
8
'
To calculate the moment of the couple, sum the moments of
the two forces at A and B with respect to the point A:
M
A
= (2 N)s

Since couple moments are the same about any point, the two
couple moments of (2 N)s and 10 Nm can be considered to act
at the same point and thus added to give the resultant moment,
M (Note that no subscript is needed on M, since M is
independent of where the moment is calculated):
M = (2 N)s 10 Nm
1
2
Because the resultant moment, M, is to be zero, the latter
equation becomes
0 = (2 N)s 10
Solving gives
s = 5 m
3
ns.
s
A B
2 N 2 N
10 Nm
C

422
4.3 Moment of a Couple Example 5, page 1 of 5
P
5. Two cords are wrapped around pegs attached to a
board as shown. Determine the value of such that P is
as small as possible while still producing a resultant
couple moment of zero. Also determine the value of P
corresponding to this value of . Neglect the size of the
pegs.
2 m
A B
D
C
80 N
P
80 N
4 m

423
4.3 Moment of a Couple Example 5, page 2 of 5
Moment of the couple at B and C.
A
D
1
M
C
= (80 N)(2 m)

= 160 Nm (1)
2 To balance the 160 Nm couple moment computed in
Eq. 1, the moment of the couple at A and D must be
160 Nm. To achieve this value of couple moment
with the smallest possible value of force P, P must be
perpendicular to line AD
B
80 N
80 N
P
P
C
2 m

424
4.3 Moment of a Couple Example 5, page 3 of 5
Now use geometry to find .
A
D
3
4 Equal angles
P
P
tan
-1
= 63.4 (2)
5
2 m
4 m
C
4 m
distance AD =
= 4.472 m
6
7 Moment of couple A and D
M
D
= P distance AD

= P(4.472 m) (3)
(2 m)
2
+ (4 m)
2
ns.
C
P
P
D
A
B
2 m
2 m
4 m

425
4.3 Moment of a Couple Example 5, page 4 of 5
8 The resultant couple moment must be zero:
M = M
C
+ M
D
= 0 (4)
or,
160 + P(4.472 m) = 0
Solving gives
P = 35.8 N
Alternative solution. Do not assume that force P is
perpendicular to line AD. Calculate the couple moment of P
about point D in terms of the unknown angle .

ns.
9
P cos
P sin
A
D
P
P
80
cos + 2 sin
M
D
= (P cos )(2 m) + (P sin )(4 m) (5)
Now substitute the expressions for M
C
and M
D
into Eq. 4:
M
C
+ M
D
= 0 (Eq. 4 repeated)
by Eq. 1 by Eq. 5
160 (P cos )(2) + (P sin )(4)
Solving for P gives
P = (6)
4 m
2 m

426
4.3 Moment of a Couple Example 5, page 5 of 5
dP
d
= 0
From Eq. 6,
=
d
dP d
d
80
cos + 2 sin
= 0
= 0
(cos + 2 sin )
2
80 ( sin + 2 cos )
Thus
sin + 2 cos = 0
tan = 2
= tan
-1
2
= 63.4
This is the same result as Eq. 2.

so
10 To find the minimum value of P, use

427
4.3 Moment of a Couple Example 6, page 1 of 3
250 mm 130 mm
175 mm
6. A plumber uses two pipe wrenches so that he can
loosen pipe BC from pipe AB without also
loosening pipe AB from the connection at the wall,
A. Determine the moment of the forces about a) A
and b) D. Also state what general principles your
results demonstrate.
A
B C
D
z
x
y
80 N
80 N
E

428
4.3 Moment of a Couple Example 6, page 2 of 3
z
175 mm
250 mm 130 mm
r
BD
r
AD
E
80 N
80 N
A
y
Equal magnitude, opposite
sign, so cancel out
i
j
k
r
AD
= {130i 175j} mm (1)
r
AE
= {130 mm + 250 mm}i (175 mm}j
= {380i 175j} mm (2)


M
A
= r
AD
{80k} N + r
AE
{ 80k} N
= {130i 175j} {80k} + {380i 175j} { 80k}
= 130(80)i k 175(80)j k + 380( 80)i k 175( 80)j k
= j = j
= [130(80) + 380( 80)]( j)
= {20 000j} Nmm

= {20j} Nm
2 Use the cross product definition of moment.
Part a): Determine the moment about
point A. Introduce position vectors
from point A to points D and E
respectively.
1
x
D
C
B

429
4.3 Moment of a Couple Example 6, page 3 of 3
z
175 mm
250 mm
E
80 N
80 N
D
A
y
6
r
DE

M
D

The two forces (the couple) produce a moment that tends to rotate the
entire pipe assembly about the vertical axis (j component). The force
applied at E also produces a moment on pipe BC about the x axis, while
the force applied at D produces a moment on the other pipe, AB, about
the x axis. Since the forces have opposite senses, the moments have
opposite senses and thus the total moment applied (to the entire pipe)
about the x axis adds to zero (no i component in M
D
).
General principles demonstrated:
1) The couple moment is the same about every point, and
2) the calculation of the couple moment is simplified, if the moment is
calculated about a point on the line of action of one of the forces
making up the couple.
Calculate the moment. 4
i
j
k
r
DE
= {250i} mm
5
Part b): To determine the moment about
point D, first introduce a position vector
with tail at D and head at E.
3
x
C
B
M
D
= r
DE
{ 80k} N
= {250i} { 80k}
= 250( 80)i k
= j
= {20000j} N mm
= {20j} N m
Same result as in Part a).

430
4.3 Moment of a Couple Example 7, page 1 of 3
300 mm
7. Determine the couple moment produced by the two
forces applied to the handle of the crank as shown.
G = 20 N
B
A
25
25
y
z
x
F = 20 N
40
40

431
4.3 Moment of a Couple Example 7, page 2 of 3
Express the force F in rectangular components. 1
F
y
= (20 N) cos 40
= 15.32 N
2
(20 N) sin 40
= 12.86 N
3
4 Fz = (12.86 N) sin 25
= 5.43 N
Fx = (12.86 N) cos 25
= 11.66 N
5
6 In component form,
F = {11.66i 15.32j + 5.43k} N

F = 20 N
z
A
x
y
40
25

432
4.3 Moment of a Couple Example 7, page 3 of 3
i
k
Because the couple has both j and k
components, it tends to cause rotation
about both the y and z axes.
9
ns.
r
BA
F = 20 N
x
z
y
A
B
G = 20 N
25
25
40
40
300 mm
Introduce a position vector with tail at B and head at A
r
BA
= { 300i} mm
7 Calculate the couple moment.
M = r
BA
F
= { 300i} {11.66i 15.32j + 5.43k}
= 300(11.66)i i 300( 15.32)i j 300( 5.43)i k
= 0 = k = j

= {1629j + 4596k} Nmm
= {1.63j + 4.60k} Nm

8
j

433
4.3 Moment of a Couple Example 8, page 1 of 6
8. Determine the resultant couple moment of the two couples acting
on the block.
x
y
z
A
B
C
D
6 lb
6 lb
10 lb
E
10 lb
F
5 in.
20 in.
30

434
4.3 Moment of a Couple Example 8, page 2 of 6
1
2 Calculate the moment of the 6-lb couple.
M
6
= r
AC
{6i} lb
= {20k} {6i}
= 20(6) k i
= {120j} lbin.

k
i
j
j
10 lb
6 lb
6 lb
D
C
B
A
z
y
x
r
AC
30
E
10 lb
F
z
5 in.
20 in.
Introduce a position vector
from a point, A, on the line of
action of the { 6i} lb force, to
a point C, on the line of action
of the {+6i} lb force.
r
AC
= {20k} in.

435
4.3 Moment of a Couple Example 8, page 3 of 6
Introduce a position vector
from a point, D, on the line of
action of the { 10i} lb force,
to a point, F, on the line of
action of the {+10i} lb force.
r
DF
= { 5j + 8.660k} in.
3
4 Calculate the moment of the 10-lb couple.
M
10
= r
DF
{10i}
= { 5j + 8.660k} {10i}
= 5(10)j i + 8.660(10)k i


= {86.60j + 50k} lbin.
k
i
j
= k = j
r
DF
tan 30
5 in.
= 8.660 in.
x
y
z
A
B
C
D
6 lb
6 lb
10 lb
E
10 lb
F
5 in.
D, C
F, E
View from positive x axis
looking back on yz plane
30
30
5 in.
20 in.

436
4.3 Moment of a Couple Example 8, page 4 of 6
Compute the resultant couple moment from Eqs. 1
and 2,
M
R
= M
6
+ M
10
= {120j} + {86.60j + 50k}
= {206.6j + 50k} lbin.
The two couples combine to produce a rotation of
the block about the axis defined by the M
R
vector.
Since M
R
is the same for all points, it can be
considered to act at an arbitrary point on top of the
block.
x
y
5
ns.
M
R
F E
D
C
z
A B
M
6
M
10
30

437
4.3 Moment of a Couple Example 8, page 5 of 6
Similarly, the 10-lb couple produces a couple moment with
magnitude 10 lb distance DF = 10 lb 10 in. = 100 lbin
and with direction perpendicular to the plane CDFE.
7
= 10 in.
5 in.
sin 30
6 lb
6 lb
D
C
B
A
y
x
120 lbin
5 in.
100 lbin
5 in.
100 lbin
x
y
A
B
C
D
10 lb
F, E
D, C
5 in.
View from positive x axis looking back on yz plane
Alternative solution that does not use the cross product
definition of a moment:
Using the right hand rule and the scalar equation M = Fd,
we can see that the couple moment produced by the 6-lb
couple has magnitude 6 lb 20 in. = 120 lbin. and can be
represented as a vector perpendicular to the plane of the
forces (plane ABDC) and pointing up.
6
30
30
E F
z
z
E
10 lb F
30
20 in.
20 in.

438
4.3 Moment of a Couple Example 8, page 6 of 6
M
R
= 86.6j + 50.0k + 120j
= {206.6j + 50k} lbin.
Same answer as before.
12 In component form,
(100 lbin.)(cos 60) = 50.0 lbin.
10
90 30 = 60
y
z
x
D, C
F, E
100 lbin
(100 lbin)(sin 60) = 86.6 lbin
30
View from positive x axis looking back on yz plane 9
Moment vector perpendicular
to top surface (plane ABDC)
of block
11
30
To add the 100 lbin. couple moment and the 120 lbin. couple
moment vectorially, we first have to express the 100 lbin.
couple moment, which is perpendicular to FD, in terms of its
y and z components.
8

439
4.3 Moment of a Couple Example 9, page 1 of 5
650 mm
700 mm
z
y
x
70 mm
F = {16i + 12k} N
9. Force F is applied to the doorknob as shown. Determine
where on the surface of the door a force G = F should be
applied so that the resulting couple moment has a y
component of 2 Nm but no x or z components.

440
4.3 Moment of a Couple Example 9, page 2 of 5
F = {16i + 12k} N
Introduce a force G.
G = F
= {16i + 12k} N
= { 16i 12k} N
G acts at a point B(x
B
, y
B
, 0) on the surface
of the door. G and F form a couple.

1
Draw a position vector r
BA
from point B on the surface of the door
to point A on the doorknob, and determine its rectangular
components.
r
BA
= (700 mm x
B
)i + (650 mm y
B
)j + (70 mm 0)k

2
B(x
B
, y
B
, 0)
G = F
70 mm
x
y
z
r
BA
A
650 mm
700 mm

441
4.3 Moment of a Couple Example 9, page 3 of 5
i [(650 y
B
)(12) 70(0)] j[(700 x
B
)(12) 70(16)] + k[(700 x
B
)(0) (650 y
B
)(16)]
(650 y
B
) 70
0 12

=
=
Calculate the couple moment.
M = r
BA
F

i j k
(700 x
B
) (650 y
B
) 70
16 0 12
i =
=
3
j
Use the fact that we know Mx = 0, so
Mx = (650 y
B
)(12) = 0 (2)
and Mz = 0, so
Mz = (650 y
B
)(16) = 0 (3)
4
(700 x
B
) 70
16 12

(700 x
B
) (650 y
B
)
16 0

+ k
i [(650 y
B
)(12)] j [( 12x
B
+ 7280)] + k [ (650 y
B
)(16)] (1)

442
4.3 Moment of a Couple Example 9, page 4 of 5
5 Solving Eq. 2 gives
y
B
= 650 mm ns.
This value of y
B
also satisfies Eq. 3:
(650 y
B
)(16) = 0 (Eq. 3 repeated)
The y component of M, from Eq. 1, is
M
y
= ( 12x
B
+ 7280) Nmm (4)
Since M
y
was specified at the beginning of the problem as
M
y
= 2 Nm = 2000 Nmm
Eq. 4 becomes
2000 = ( 12x
B
+ 7280)
Solving gives
x
B
= 440 mm ns.
650

443
4.3 Moment of a Couple Example 9, page 5 of 5
440 mm
G
Thus force G is applied at the same
height as F but closer to the hinges.
G = { 16i 12k} N
6
M = { 2j} Nm
The couple moment
tends to open the door.
7
B
F = {16i + 12k} N
x
y
z
650 mm
700 mm

444

4.4 Moment of a Force About a Line
445
4.4 Moment of a Force About a Line Procedures and Strategies, page 1 of 1
F
r
u
y
x
z
Line L
Procedures and Strategies for Solving Problems
Involving Moment of a Force About a Line
1. Express the force F in rectangular component form.
2. Find the unit vector u along the line and express it in
rectangular component form.
3. Construct a position vector r with tail at any point on the
line and head at any point on the line of action of the
force .
4. Evaluate the scalar triple product:
M
L
= u r x F
Note: The choice of sense of u is arbitrary. If M
L
turns out
to be positive, then the moment vector M
L
u has the same
sense as u; if M
L
turns out to be negative, then M
L
u has the
opposite sense as u .
Moment vector M
L
u

446
4.4 Moment of a Force About a Line Problem Statement for Example 1
1. Force F is applied to the end of gearshift
lever CDE. Determine the moment of F
about shaft AB. State which way the lever
will rotate about AB.
F = {2i j + 0.5k} lb
E
C
D
z
y
x
B
A
5 in.
6 in.
40

447
4.4 Moment of a Force About a Line Problem Statement for Example 2
2. Thread is pulled off a spool as shown.
Determine the moment of the force F
about the axis of the spool.
30 mm
x
F = 0.25 N
100 mm
y
z
40
60

448
4.4 Moment of a Force About a Line Problem Statement for Example 3
0.15 m
0.25 m
0.3 m
F = {3i + 10j 5k } N
z
y
x
3. The lug wrench is used to turn the lug nut holding the wheel
on the hub. Determine the moment of the force about the axis of
the bolt to which the nut is fastened.

449
4.4 Moment of a Force About a Line Problem Statement for Example 4
4. Forces P
1
, P
2
, P
3
, each of magnitude P, act on the edges of a
cube of side "a" as shown. Determine the moment of each force
about diagonal CB. Also express each moment in vector form.
x
y
z
B
A
C
H
F
G
P
1
= P
P
2
= P
P
3
= P
a
Force P
2
lies on the z axis.

450
4.4 Moment of a Force About a Line Problem Statement for Example 5
y
x
z
C
A
B
D
F = 6 kN
20 m
12 m
15 m
10 m
18 m
5. Determine the moment about the line through
points A and B of the force shown. Express the
result in vector form.

451
4.4 Moment of a Force About a Line Problem Statement for Example 6
6. An electrical conduit is held in place by brackets A, D
and E. The effect of the bracket at A on the conduit can be
represented by the force F as shown. Determine the
resulting moment that F produces that tends to twist the
portion CG of the conduit about its axis.
y
z
A
B
D
E
C
G
F = { 20i +12j + 16k} N
3 m
2 m
x
3.5 m
2.5 m

452
4.4 Moment of a Force About a Line Example 1, page 1 of 3
The moment M
L
of a force F about a line L is
M
L
= u r F
where u is a unit vector along L, and r is a
position vector from any point on L to any
point on the line of action of F.
A
B
x
y
z
D
C
E
1
F = {2i j + 0.5k} lb
1. Force F is applied to the end of gearshift
lever CDE. Determine the moment of F
about shaft AB. State which way the lever
will rotate about AB.
F = {2i j + 0.5k} lb
E
C
D
z
y
x
B
A
2 Here, the unit vector, u, is just the base
vector, k.

u k
5 in.
5 in.
6 in.
40

453
4.4 Moment of a Force About a Line Example 1, page 2 of 3
3
Choose position vector r as r
CE
:
r = r
CE
= (6 in.)(sin 40)i + [5 in. + (6 in.)(cos 40)]j
= {3.857i + 9.596j} in.

4 The best way to evaluate the scalar triple product is to use
a calculator that has built-in functions for the dot and cross
products. If such a calculator is not available, then
evaluate the triple product by using a determinant in which
the successive rows are u, r, and F.
M
AB
= u r

CE
F
0 0 1
3.857 9.596 0
2 1 0.5
=
=
9.596 0
1 0.5
0 0
3.857 0
2 0.5
1
3.857 9.596
2 1
+
u k
r
F
F = {2i j + 0.5k} lb
E
C
D
z
y
x
B
A
u k
r

CE
= 0 0 + (1)[3.857( 1) 9.596(2)]
= 23.0 lbin.
40
6 in.
5 in.

454
4.4 Moment of a Force About a Line Example 1, page 3 of 3
A
B
x
y
z
D
C
E
6 We could have saved a little work by noting that
M
AB
= u r
CE
F
= k r
CE
F
= M
C
(moment of F about C)
= k component of M
C
So we only need to calculate M
C
and look at its k
component; we didn't need to evaluate the determinant for
the scalar triple product, u r
CE
F.
5 The negative sign in M

AB
= 23.0 lbin.
indicates that M
AB
has a sense opposite to
u. This moment will cause the lever CDE
to rotate clockwise about AB, when
viewed from end A.
u
23.0 lbin.
ns.

455
4.4 Moment of a Force About a Line Example 2, page 1 of 3
The moment M
L
of a force F about a line L is
M
L
= u r F

where u is a unit vector along L and r is a
position vector from any point on L to any
point on the line of action of F.
2. Thread is pulled off a spool as shown.
Determine the moment of the force F
about the axis of the spool.
1
30 mm
x
F = 0.25 N
100 mm
y
z
40
60

456
4.4 Moment of a Force About a Line Example 2, page 2 of 3
3 The best choice
(simplest form) for r
is r = {30i} mm.
The unit vector
u is just the base
vector, u j.
2
4 F

y
= (0.25 N) cos 60 = 0.1250 N
(0.25 N) sin 60 = 0.2165 N 5
F

z = (0.2165 N) sin 40 = 0.1392 N 6
7 F

x = (0.2165 N) cos 40 = 0.1658 N
In vector form,
F = {0.1658i + 0.1250j 0.1392k} N
8
F = 0.25 N
40
60
30 mm
y
z
x

457
4.4 Moment of a Force About a Line Example 2, page 3 of 3
9 M
axis of bobbin
= u r F
0
0 0
0.1250 0.1392
=
30 0
0.1658 0.1392
1
30 0
0.1658 0.1250
+ 0
0 1 0
30 0 0
0.1658 0.125 0.1392
=
0 [30( 0.1392) 0(0.1658)] + 0 =
4.18 Nmm = ns.

458
4.4 Moment of a Force About a Line Example 3, page 1 of 2
The moment M
L
of a force F about a line L is
M
L
= u r F

where u is a unit vector along L and r is a
position vector from any point on L to any
point on the line of action of F.
1
0.15 m
0.25 m
0.3 m
F = {3i + 10j 5k } N
z
y
x
3. The lug wrench is used to turn the lug nut holding the wheel
on the hub. Determine the moment of the force about the axis of
the bolt to which the nut is fastened.

459
4.4 Moment of a Force About a Line Example 3, page 2 of 2
2 Since we want the moment about the bolt axis, which
is the x axis, the unit vector u is the base vector:
u i
Choose r as the position vector from A to B:
r = r
AB
= {0.3i + 0.15j + 0.25k} m
4
1[0.15(5) 0.25(10)] 0 + 0
1.75 Nm Ans.
1 0 0
0.3 0.15 0.25
3 10 5
+ 0 0
0.15 0.25
10 5
M
bolt
= u r
AB
F
= i {0.3i + 0.15j + 0.25k} {3i + 10j 5k}
=
1
=
0.3 0.25
3 5
0.3 0.15
3 10
x
=
=
u
A
B
3
x
y
z
F = {3i + 10j 5k} N
0.3 m
0.25 m
0.15 m

460
4.4 Moment of a Force About a Line Example 4, page 1 of 5
4. Forces P
1
, P
2
, P
3
, each of magnitude P, act on the edges of a
cube of side "a" as shown. Determine the moment of each force
about diagonal CB. Also express each moment in vector form.
x
y
z
The moment M
L
of a force F about a line L is
M
L
= u r F

where u is a unit vector along L and r is a
position vector from any point on L to any
point on the line of action of F.
B
A
C
H
F
G
P
1
= P
P
2
= P
P
3
= P
a
1
Force P
2
lies on the z axis.

461
4.4 Moment of a Force About a Line Example 4, page 2 of 5
2 To determine the unit vector u, first define the position vector r
CB
.
r
CB
= ai aj
Thus
r
CB
r
CB
ai aj

i j

x
y
z
B
A
C
H
F
G
P
1
= P
P
2
= P
P
3
= P
a
2
+ ( a
2
)
2 2
=
=
= u
u
r
CB
a

462
4.4 Moment of a Force About a Line Example 4, page 3 of 5
a
z
x
y
3 To calculate the moment of P
1
about line CB, the
best choice (simplest form) for r is r

CG
(Note
that G lies on the line of action of P
1
).
B
A
C
H
F
G
P
1
= P
u
Vector form of the force P
1
= Pi 4
5 For force P
1
,
The vector representation of the moment about CB is
just the product of the magnitude of the moment, M
CB
,
times the unit vector u, since u is parallel to CB. Thus

6
=
=
=
1
aP
ns.
M
CB
= u r
CG
P
1
0 a
0 0
0
0 0 a
P 0 0
=
0 + [0(0) a) P)] + 0
0 a
P 0
+ 0
0 0
P 0
2
2
2 2
1
2
1 1
2
r = r

CG


= ( a)k
r

CG
2
aP aP
2
M

CB
= u =
2
i j
2

=
aP
2
ns.
1
M

CB
( i + j)

463
4.4 Moment of a Force About a Line Example 4, page 4 of 5
( i + j )
a
z
x
y
7 To calculate the moment of P
2
about line CB, a
good choice for r is r
CA
(Note that A is on the
line of action of P
2
).
B
A
C
H
F
G
P
2
= P u
9 For force P
2
,
Vector form 10
=
=
=
1
aP ns.
M
CB
= u r
CA
P
2
a 0
0 P
0
0 a 0
0 0 P
=
[ a) P) 0(0)] 0 + 0
0 0
0 P
+ 0
0 a
0 0
2
2
2 2
1
2
1 1
2
r = r
CA
= aj
r
CA

2
aP
r
BA
would be
an equally
good choice

r
BA
Vector form of
the force
P
2
= Pk

aP
2
2
aP
ns.
M
CB

2
j i
2
u =
M
CB
=
=
1
8

464
4.4 Moment of a Force About a Line Example 4, page 5 of 5
11 To determine the moment of P
3
about line CB, note that
the line of action of P
3
passes through point B. Thus
the force does not tend to rotate the cube about line CB:
B
A
C
H
F
G
P
3
= P
M
CB
= 0 ns.
and in vector form
M
CB
= 0 ns.
In mathematical terms, we could choose a position
vector of zero length (from B to B), r = 0.
This would give
MCB = u r F
= u 0 F
= 0
a
z
x
y

465
4.4 Moment of a Force About a Line Example 5, page 1 of 4
The moment M
L
of a force F about a line L is

M
L
= u r F
where u is a unit vector along L and r is a
position vector from any point on L to any
point on the line of action of F.
1
y
x
z
C
A
B
D
F = 6 kN
20 m
12 m
15 m
10 m
18 m
5. Determine the moment about the line through
points A and B of the force shown. Express the
result in vector form.

466
4.4 Moment of a Force About a Line Example 5, page 2 of 4
To determine the unit vector u along AB, first introduce
position vector r
AB
.
r
AB
= (0 20 m)j + (0 18 m)k
= { 20j 18k} m
Then,
u =
=
= 0.7433j 0.6690k
2
The best choice (simplest form) for r is r
BC
r r
BC
= [10 m ( 20 m)]j
= {30j} m
Note that point C lies on the line of action of the force.
3
u
r
AB
( 20)
2
+ ( )
2
18 m
10 m
15 m
12 m
20 m
F = 6 kN
D
B
A
C
z
x
y
r
BC
r
AB
r
AB
20j 18k

467
4.4 Moment of a Force About a Line Example 5, page 3 of 4
To express the force in rectangular components, first
introduce a position vector from D to C.
r
DC
= { 15i + 10j 12k} m
Then the force is,



15i + 10j 12k

= { 4.1558i + 2.7705j 3.3247k} kN
4
( 15)
2
+ (10)
2
+ ( 12)
2
= (6 kN)
= (6 kN) F
18 m
10 m
15 m
12 m
20 m
F = 6 kN
D
B
A
C
z
x
y
r
DC
r
DC
r
DC

468
4.4 Moment of a Force About a Line Example 5, page 4 of 4
5
0 + 0.7433[0( 3.3247) 0( 4.1558)] 0.6690[0(2.7705) 30( 4.1558)]
=
0 0.7433 0.6690
0 30 0
4.1558 2.7705 3.3247
( 0. 33)
30 0
2.7705 3.3247

M
AB
= u r
BC
F

ns. 83.41 kN.m
0
=
=
=
0 0
4.1558 3.3247

+ 0.6690)
0 30
4.1558 2.7705

6 Vector form
M
AB
= M
AB
u
= ( .41){ 0.7433j 0.6690k}
= {62.0j + 55.8k} kNm ns.
M
AB
is the moment in the direction of
line AB, which is defined by the unit
vector, u.


469
4.4 Moment of a Force About a Line Example 6, page 1 of 3
6. An electrical conduit is held in place by brackets A, D
and E. The effect of the bracket at A on the conduit can be
represented by the force F as shown. Determine the
resulting moment that F produces that tends to twist the
portion CG of the conduit about its axis.
y
z
The moment M
L
of a force F about a line L is
M
L
= u r F

where u is a unit vector along L and r is a
position vector from any point on L to any
point on the line of action of F.
A
B
D
1
E
C
G
F = { 20i +12j + 16k} N
3 m
2 m
x
3.5 m
2.5 m

470
4.4 Moment of a Force About a Line Example 6, page 2 of 3
To determine the unit vector u, first
introduce a position vector from C to G.
r
CG
= {3i + 2.5j} m
Then the unit vector along CG is,
r
CG
r
CG
3i + 2.5j


= {0.7682i + 0.6402j}
2
(3)
2
+ (2.5)
2

=
r
CA
Choose the position vector r as
r r
CA

= { 3.5j + 2k} m
3
2.5 m
3.5 m
x
2 m
3 m
F = { 20i +12j
+ 16k} N
G
C
E
D
B
A
z
y
u
u
r

CG
=

471
4.4 Moment of a Force About a Line Example 6, page 3 of 3
4
=
0.7682 0.6402 0
0 3.5 2
20 12 16
0.6402
3.5 2
12 16

M
CG
= u r

CA
F

0.7682
= 0.7682[ 3.5(16) 2(12)] 0.6402 0(16) (2)( 20)
= 87.1 Nm ns.
=
0 2
20 16

+ 0
0 3.5
20 12


472

4.5 Equivalent Force-Couple Systems
473
4.5 Equivalent Force-Couple Systems Procedures and Strategies, page 1 of 2
M = Fd
F
A
B
A
B
d
Equivalent
F
Procedures and Strategies for Solving Problems Involving
Equivalent Force-Couple Systems
1. To move a force from a point A on a rigid-body to a
point B and maintain equivalence, first calculate the
moment M of the force about B. Then at B apply the
force and a couple moment of magnitude M.

474
4.5 Equivalent Force-Couple Systems Procedures and Strategies, page 2 of 2
O
A
Equivalent
M
R
Equivalent
d
O
R
d =
M
R
F
1
F
2
F
3
2. To reduce a system of forces to a single equivalent
force and couple moment, apply the procedure
described in paragraph 1 above to move each force to a
common point. Then add the forces to obtain a single
resultant force R and add the couple moments to obtain
a single resultant couple moment M.
3. To replace a system of coplanar forces by a single
force that is statically equivalent, first apply the
procedure described in paragraph 2 above to produce a
resultant force R and couple moment M at a point O.
Then move the force R to a new line of action such that
R produces a moment, about O, of magnitude M.

475
4.5 Equivalent Force-Couple Systems Problem Statement for Example 1
1. Replace the force at A by an equivalent
force and couple moment at point O.
y
x
4 m
6 m
20 N
A
4
3
5
0

476
4.5 Equivalent Force-Couple Systems Problem Statement for Example 2
2. The 60-N force acts at point A on the lever as shown.
Replace the force at A by a force and couple moment
acting at point O that will have an equivalent effect.
A
60 N
60
O
200 mm

477
4.5 Equivalent Force-Couple Systems Problem Statement for Example 3
3. Replace the 2-lb force acting on the end of the bottle
opener by an equivalent force and couple moment
acting on the underside of the bottle cap at A. Use your
results to explain how a bottle opener works.
A
2 lb
B
3 in.

478
4.5 Equivalent Force-Couple Systems Problem Statement for Example 4
4. Replace the given forces and couple moment by a
resultant force and couple moment at A.
A
120 lb
260 lb
800 lbft
7 ft 4 ft 2 ft
5
12
13

479
4.5 Equivalent Force-Couple Systems Problem Statement for Example 5
5. Replace the force F = 3 kN acting on corner A of the
block by an equivalent force and couple moment acting
at the center C of the block.
A
y
x
C
z
F = 3 kN
150 mm
150 mm
100 mm 100 mm

480
4.5 Equivalent Force-Couple Systems Problem Statement for Example 6
B
C
x
y
z
4 lb
7 lb
A
48 in.
6. Replace the forces acting on the ice auger by an
equivalent force and couple moment acting at A.
6 in.

481
4.5 Equivalent Force-Couple Systems Problem Statement for Example 7
7. Replace the forces and couple moment by a single force and
specify where it acts.
A B C D
2 ft 8 ft 4 ft 3 ft
3 kip
4 kip
20 kipft
40

482
4.5 Equivalent Force-Couple Systems Problem Statement for Example 8
8. Replace the forces acting on the frame by a single force
and specify where its line of action intersects
a) member BCD and b) member AB.
400 N
800 N
600 N
300 N
x
y
A
B
C D
E
2 m
2 m
4 m 4 m

483
4.5 Equivalent Force-Couple Systems Problem Statement for Example 9
9. Determine the value of force P such that the line of
action of the resultant of the forces acting on the truss
passes through the support at H. Also determine the
magnitude of the resultant.
A B C D E
I
J
H G F
8 ft
160 lb 200 lb 180 lb
260 lb
P
30
6 ft 6 ft 6 ft 6 ft

484
4.5 Equivalent Force-Couple Systems Problem Statement for Example 10
10. A machine part is loaded as shown. The part is to be attached to
a supporting structure by a single bolt. Determine the equation of
the line defining possible positions of the bolt for which the given
loading would not cause the part to rotate. Also, determine the
magnitude and direction of the resultant force.
x
y
0.3 m
0.5 m
90 Nm
20 Nm
O
60 N
80 N
0.6 m 0.4 m
60

485
4.5 Equivalent Force-Couple Systems Problem Statement for Example 11
11. The rectangular foundation mat supports the four
column loads shown. Determine the magnitude,
direction, and point of application of a single force that
would be equivalent to the given system of forces.
A
B
C
D
y
20 kip
15 kip
11 kip
z
x
30 kip
10 ft
5 ft
8 ft
3 ft

486
4.5 Equivalent Force-Couple Systems Problem Statement for Example 12
12. Three signs are supported by an arch over a highway and are
acted upon by the wind forces shown. Replace the forces by an
equivalent force and specify its point of application.
6 m
C
4 m
B
850 N
400 N
A
O
3.5 m
y
300 N
4 m
1 m
z
5 m
x

487
4.5 Equivalent Force-Couple Systems Problem Statement for Example 13
13. Three forces act on the pipe assembly. Determine the
magnitude of forces P and Q if the resultant of all three
forces is to act on point A. Also determine the magnitude
of the resultant.
B
A
C
D
O
P
Q
y
x
z
200 N
3 m
2 m
1.5 m
1 m

488
4.5 Equivalent Force-Couple Systems Problem Statement for Example 14
14. The end plate of a pressurized tank is held in place by forces from three bolts.
Determine the required value of bolt-force P and angle if the resultant of the
three bolt forces is to act through the center of the plate at O.
y
x
z
P 500 N
B
A
700 N
O
0.5 m
40
C

489
4.5 Equivalent Force-Couple Systems Problem Statement for Example 15
z
6 m
A
O
y
x
B
60 N
40 N
140 N
15. Replace the system of forces by a wrench.
Determine the pitch and axis of the wrench.
12 m
4 m

490
4.5 Equivalent Force-Couple Systems Problem Statement for Example 16
F

1
4 lb
M

1
60 lb in.
1.5 in.
M

2
80 lb in.
30
F

2
6 lb
z
6 in.
1 in.
3 in.
D
x
16. In a machining operation, holes are simultaneously drilled at points
A and B of the wedge. The drill at A produces a force and couple
moment perpendicular to the planar surface at A. The force and couple
moment at B are similarly perpendicular to the planar surface at B.
Replace the forces and couple moments by a wrench. Determine
a) the magnitude R of the resultant force,
b) the pitch of the wrench,
c) the axis of the wrench, and
d) the point where the axis intersects the x-z plane.
E
B
A
C
y
4 in. 4 in.

491
4.5 Equivalent Force-Couple Systems Example 1, page 1 of 1
1. Replace the force at A by an equivalent
force and couple moment at point O.
y
x
4 m
6 m
20 N
A
4
3
5
1 Express the force in rectangular components.
A
20 N
6 m
4 m
x
y
3
4
5
0
0
(20 N)
5
16 N =
4
3
= 12 N
5
(20 N)
Calculate the moment about O. 2
M
O
= (16 N)(4 m) + (12 N)(6 m)
= 136 Nm
3
Display the equivalent force and couple moment
at point O.
0
5
3
4
A
20 N
x
y
ns.
136 Nm

492
4.5 Equivalent Force-Couple Systems Example 2, page 1 of 1
2. The 60-N force acts at point A on the lever as shown.
Replace the force at A by a force and couple moment
acting at point O that will have an equivalent effect.
A
1 Calculate the moment about O.
M
O
= (60 N)(200 mm) cos 60
= 6000 Nmm
= 6 Nm
Display the equivalent force
and couple moment at point O.
2
60 N
60
60
60 N
A
A
60 N
6 N m
ns.
(200 mm) cos 60
O
O
O
200 mm
200 mm

493
4.5 Equivalent Force-Couple Systems Example 3, page 1 of 1
3. Replace the 2-lb force acting on the end of the bottle
opener by an equivalent force and couple moment
acting on the underside of the bottle cap at A. Use your
results to explain how a bottle opener works.
1 Calculate the couple moment about point A.
M
A
= (2 lb)(3 in.)
= 6 lbin.
Display the equivalent force
and couple moment at point A.
2
ns.
A
2 lb
B
B
3 The bottle opener works by pulling (with a 2-lb
force) on the edge of the cap while
simultaneously twisting (with a 6 lbin. moment)
the entire cap away from the top of the bottle.
6 lbin. ns.
3 in.
2 lb

494
4.5 Equivalent Force-Couple Systems Example 4, page 1 of 2
4. Replace the given forces and couple moment by a
resultant force and couple moment at A.
2 Calculate the resultant force
A
120 lb
260 lb
800 lbft
7 ft 4 ft 2 ft
Express the inclined force in rectangular
components.
1
2 ft 4 ft 7 ft
800 lbft
120 lb
13
12
5
A
Rx = Fx: Rx = 100 lb

R
y
= F
y
: R
y
= 240 lb 20 lb
= 360 lb
= 360 lb
5
12
13
5
13
(260 lb) =100 lb
240 lb = (260 lb)
13
12
Calculate the resultant couple moment about A. 3
M
R
A
= M
A
= (240 lb) (2 ft + 4 ft + 7 ft)
+ (120 lb) (4 ft + 7 ft) 800 lbft
= 3640 lbft

495
4.5 Equivalent Force-Couple Systems Example 4, page 2 of 2
Determine the magnitude and direction of the
resultant force.
4
ns.
R = (360)
2
+ (100)
2
= 374 lb
= tan
-1
100
360
= 74.5
ns.
360 lb
100 lb
5 Display the equivalent force and couple
moment at A.
A
374 lb
3640 lbft
ns.
74.5

496
4.5 Equivalent Force-Couple Systems Example 5, page 1 of 3
5. Replace the force F = 3 kN acting on corner A of the
block by an equivalent force and couple moment acting
at the center C of the block.
A
y
x
C
z
F = 3 kN
150 mm
150 mm
100 mm 100 mm

497
4.5 Equivalent Force-Couple Systems Example 5, page 2 of 3
Calculate the couple moment about C.
A
y
x
C
z
F = 3 kN
150 mm
150 mm
1
M

C
= r
CA
F
= {100i 150k}mm 3j}kN

= 100( i j 150( k j
= k = i

= { 450i 300k}kNmm
= { 450i 300k}N m Ans.
i
j
k
r
CA
100 mm 100 mm

498
4.5 Equivalent Force-Couple Systems Example 5, page 3 of 3
Display the equivalent force and couple moment at C.
A
y
x
C
z
150 mm
150 mm
2
F = { 3j}kN Ans.
M
C
= { 450i 300k}Nm
100 mm 100 mm

499
4.5 Equivalent Force-Couple Systems Example 6, page 1 of 4
B
C
x
y
z
4 lb
7 lb
A
48 in.
6. Replace the forces acting on the ice auger by an
equivalent force and couple moment acting at A.
6 in.

500
4.5 Equivalent Force-Couple Systems Example 6, page 2 of 4
B
C
x
4 lb
7 lb
A
48 in.
Rx = Fx: Rx = 0

R
y
= F
y
: R
y
= 4 lb
Rz = Fz: Rz = 7 lb

Calculate the resultant force 1
z
6 in.
y

501
4.5 Equivalent Force-Couple Systems Example 6, page 3 of 4
B
C
x
4 lb
7 lb
A
48 in.
M
A
R
= M
A

= r
AC
{ 4j} lb + r
AB
{7k} lb

2
k
j
i
r
AC
r
AB
z
6 in.
Calculate the resultant couple moment about A.
= 0, because r
AC
and { 4j} are parallel,
or, what amounts to the same thing,
the line of action of the { 4j} lb force
passes throughout point A.
= 0 + { 6i + 48j} in. {7k} lb
= 6(7)i k + 48(7)j k
= j = i
= {336i + 42j} lbin.
y

502
4.5 Equivalent Force-Couple Systems Example 6, page 4 of 4
B
C
x
y
A
M
A
R
= {336i + 42j} lbin.
R = { j + 7k} lb
ns.
z
3

503
4.5 Equivalent Force-Couple Systems Example 7, page 1 of 3
7. Replace the forces and couple moment by a single force and
specify where it acts.
A B C D
2 ft 8 ft 4 ft 3 ft
3 kip
4 kip
20 kipft
40

504
4.5 Equivalent Force-Couple Systems Example 7, page 2 of 3
A B
C
D
2 ft 8 ft 4 ft 3 ft
3 kip
20 kip ft
Rx = Fx: Rx = 3.064 kip = 3.064 kip

R
y
= F
y
: R
y
= 3 kip 2.571 kip = 5.571 kip

R = (3.064)
2
+ (5.571)
2

= 6.358 kip
Calculate the resultant force 2
1 Resolve the inclined force into
rectangular components.
y
(4 kip) cos 40 = 3.064 kip
(4 kip) sin 40 = 2.571 kip
40
3.064 kip
5.571 kip
= tan
-1
3.064
5.571
= 61.2
R
4 kip
3
ns.
ns.

505
4.5 Equivalent Force-Couple Systems Example 7, page 3 of 3
4
A B
C
D
2 ft 8 ft 4 ft 3 ft
3 kip
4 kip
20 kipft
We equate the moment of the new
system, about point A, to the moment
of the original force-couple system
(The choice of point A for summing
moments is arbitrary; any other point
would work as well, except that we
must use the same point for both the
original system and the new system.)
6
5 This is a new force-couple system that we want to make
equivalent to the original force-couple system. We
already know that the forces are equivalent because R
is the resultant of the forces in the original system.
Now we have to make sure that the moment is also
equivalent. We do this by placing R at some unknown
distance d from the left end and then choosing d so that
the moment of this new system is the same as that of
the original system.
3.064 kip
2.571 kip
40
= 61.2
7
5.571 kip
3.064 kip
D
C A
M
A
R
= M
A

or,
(5.571 kip)d = 20 kipft (3 kip)(2 ft) (2.571 kip)(2 ft + 8 ft)
Solving gives
d = 2.10 ft Ans.
d
R = 6.358 kip
This is the original force-couple moment
system.

506
4.5 Equivalent Force-Couple Systems Example 8, page 1 of 5
8. Replace the forces acting on the frame by a single force
and specify where its line of action intersects
a) member BCD and b) member AB.
1 Determine the resultant force.
Rx = Fx: Rx = 800 N = 800 N

R
y
= F
y
: R
y
= 400 N 600 N 300 N
= 1300 N
= 1300 N
400 N
800 N
600 N
300 N
x
y
A
B
C D
E
2 m
2 m
R = (800)
2
+ (1300)
2
= 1526 N
= tan
-1
= 58.4
1300 N
800 N
1300
800
Ans.
Ans.
4 m 4 m

507
4.5 Equivalent Force-Couple Systems Example 8, page 2 of 5
Choose d so that the moment about B of the resultant
force equals the moment of the original force system.
(The choice of point B was arbitrary.)
2 Part a) To determine where the line of
action of the resultant force intersects
member BCD, place the force on BCD,
an unknown distance d from point B.
400 N
800 N
600 N 300 N
x
A
B
C D
E
3
M
B
R
= M
B
or
(1300 N)d = (600 N)(4 m) (300 N)(4 m + 4 m) (800 N)(2 m)
Solving gives
d = 4.92 m Ans.
E
D
C
B
A
1300 N
d
R = 1526 N
800 N
58.4
2 m
4 m 4 m
2 m

508
4.5 Equivalent Force-Couple Systems Example 8, page 3 of 5
Choose d' so that the moment about B of the resultant
force equals the moment of the original force system.
4 Part b) To determine where the line of
action of the resultant force intersects
member AB, place the force a distance
d' from point B.
400 N
800 N
600 N
300 N
x
A
B
C
D
E
5
M
B
R
= M
B
or
(800 N)d' = (600 N)(4 m) (300 N)(4 m + 4 m) (800 N)(2 m)
Solving gives
d' = 8.0 m Ans.
E
D
B
A
1300 N
R = 1526 N
800 N
58.4
d'
2 m
2 m
4 m 4 m

509
4.5 Equivalent Force-Couple Systems Example 8, page 4 of 5
The intersection of the line of action with a line drawn
through A and B occurs below point B.
A
B D
E
6
800 N
58.4
1300 N
R = 1526 N
4 m
d' = 8 m

510
4.5 Equivalent Force-Couple Systems Example 8, page 5 of 5
7 Alternative solution for part b. Once we know where
the line of action intersects member BCD, we can
use geometry to find the intersection with AB.
E
D C B
A
1300 N
d = 4.92 m
line of action of
resultant force
G
d'
4.92
tan 58.4 =
Solving gives,
d' = 8.0 m
(same result as before)
From triangle CBG, 8
58.4
d'

511
4.5 Equivalent Force-Couple Systems Example 9, page 1 of 4
6 ft 6 ft
9. Determine the value of force P such that the line of
action of the resultant of the forces acting on the truss
passes through the support at H. Also determine the
magnitude of the resultant.
A B C D E
I
J
H G F
8 ft
160 lb 200 lb 180 lb
260 lb
P
30
6 ft 6 ft

512
4.5 Equivalent Force-Couple Systems Example 9, page 2 of 4
30
A
B C D E
I J H G F
6 ft 6 ft 6 ft 6 ft
8 ft
160 lb 200 lb 180 lb
260 lb
P
Rx = Fx: Rx = 130 lb P

R
y
= F
y
: R
y
= 225.2 lb 160 lb 200 lb 180 lb
= 765.2 lb
= 765.2 lb (2)
Determine the resultant force. 1
(260 lb) cos 30
= 225.2 lb
(260 lb) sin 30
= 130 lb

513
4.5 Equivalent Force-Couple Systems Example 9, page 3 of 4
M
H
R
= M
H
:
0 = (225.2 lb)(6 ft + 6 ft) (130 lb)(8 ft)
+ (160 lb)(6 ft) (180 lb)(6 ft) + P(8)
Solving gives
P = 192.8 lb
= 192.8 lb
F G H
J
I
E D C B
Resultant R
225.2 lb
Rx = 130 lb P
= 130 lb ( 192.8 lb)
= 322.8 lb
R
y
= 765.2 lb
ns.
ns.
A
B C D E
I
J
H G F
6 ft 6 ft 6 ft 6 ft
8 ft
160 lb 200 lb 180 lb
P
Choose force P so that the
moment about H of the
resultant force equals the
moment of the original force
system about H. Any other
point besides H could be
used, but H has the advantage
that the moment of the
resultant R is zero, since R is
known (as part of the
statement of the problem) to
pass through H.
2
130 lb
3

514
4.5 Equivalent Force-Couple Systems Example 9, page 4 of 4
Magnitude of resultant.
From Eqs. 1 and 2,
R = (322.8)
2
+ ( 765.2)
2
= 831 lb ns.
4

515
4.5 Equivalent Force-Couple Systems Example 10, page 1 of 4
10. A machine part is loaded as shown. The part is to be attached to
a supporting structure by a single bolt. Determine the equation of
the line defining possible positions of the bolt for which the given
loading would not cause the part to rotate. Also, determine the
magnitude and direction of the resultant force.
x
y
0.3 m
0.5 m
90 Nm
20 Nm
O
60 N
80 N
0.6 m 0.4 m
60

516
4.5 Equivalent Force-Couple Systems Example 10, page 2 of 4
x
y
90 Nm
20 Nm
O
60 N
Rx = Fx: Rx = 40 N = 40 N

R
y
= F
y
: R
y
= 69.28 N 60 N
= 129.28 N
= 129.28 N
Determine the resultant force. 1
Ans.
Ans.
129.28
40
40 N
129.28 N
R = (40)
2
+ (129.28)
2
= 135.33 N
= tan
-1
= 72.8
80 N
60
(80 N) cos 60 = 40 N
(80 N) sin 60 = 69.28 N

517
4.5 Equivalent Force-Couple Systems Example 10, page 3 of 4
x
0.5 m
y
x
y
90 Nm
20 Nm
O
60 N
Rotation is caused by moment.
If the given force-couple
system is replaced by a single
equivalent force (the resultant)
and a specified line of action,
then the moment would be zero
about any point on the line of
action. So the line of action is
the line on which the bolt
should be placed to prevent
rotation.
2
80 N
60
40 N
3 To find the equation of the line of action, place the resultant at a
general point (x, y). Then equate moments about, say, point O for the
resultant (the figure on the right) and the original loading (the figure
on the left):
M
O
R
= M
O
or
(40 N)y (129.28 N)x = 90 Nm 20 Nm + (40 N)(0.3 m + 0.5 m)
(69.28 N)(0.4 m) (60 N)(0.4 m + 0.6 m)

40 N
O
y
x
69.28 N
0.4 m
0.3 m (x, y)
129.28 N
R
72.8
0.6 m

518
4.5 Equivalent Force-Couple Systems Example 10, page 4 of 4
Solving for y gives the equation of a line
y = 3.232 x + 0.357 Ans.
This line defines the possible locations of
the bolt.
0
y
x
0.4 m
0.3 m
All points at the top of the machine part have a y coordinate of
y = 0.3 m + 0.5 m
= 0.8 m.
Substituting y = 0.8 m into the equation of the line for the bolt
locations,
y = 3.32 x + 0.357
and solving for x gives
x = 0.133 m
4
5
0.5 m
0.357 m
0.6 m
0.133 m

519
4.5 Equivalent Force-Couple Systems Example 11, page 1 of 4
R
y
= F
y
: R
y
= 30 kip 15 kip 20 kip 11 kip
= 76 kip
= 76 kip
11. The rectangular foundation mat supports the four
column loads shown. Determine the magnitude,
direction, and point of application of a single force that
would be equivalent to the given system of forces.
1
A
Determine the resultant.
B
C
D
y
20 kip
15 kip
11 kip
z
x
30 kip
10 ft
5 ft
8 ft
3 ft

520
4.5 Equivalent Force-Couple Systems Example 11, page 2 of 4
3 First, equate moments about the x axis. We can use
either the scalar definition of moment, M = Fd, or the
vector product definition. Let's use the scalar definition.
2
x
z
y
A
B
C
D
(x, 0, z)
x z
R = 76 kip
z
30 kip 11 kip
15 kip
C, B
x, A
8 ft 3 ft
z
C, B
y
R = 76 kip
x, A
z
View from positive x axis
20 kip
y
To make the resultant
force R equivalent to
the original system of
forces, place R at the
point (x, 0, z) and then
determine values of x
and z such that R
produces the same
moments about the x
and z axes as the given
forces produce.
D
D
3 ft
8 ft
5 ft
10 ft
30 kip
x
z
11 kip
15 kip
20 kip
y
D
C
B
A
Mx
R
= Fd: (76 kip)z = (20 kip)(8 ft + 3 ft) + (15 kip)(8 ft + 3 ft)
+ (11 kip)(3 ft ) + (30 kip)(0) (1)
4

521
4.5 Equivalent Force-Couple Systems Example 11, page 3 of 4
R = 76 kip
z x
(x, 0, z)
D
C
B
A
y
z
x
x
y
C
z, B, A
D C
30 kip
View from positive z axis
x
R = 76 kip
y
D
5 ft 10 ft
z, B, A
15 kip
20 kip 11 kip
Use two-dimensional views. 6
M

z
R
= Fd: (76 kip)x = (20 kip)(10 ft) (11 kip)(10 ft + 5 ft) (2) 7
5 Solving Eq. 1 gives
z = 5.5 ft Ans.
Next, equate moments about the z axis.
A
B
C
D
y
20 kip
15 kip
11 kip
z
x
30 kip
10 ft
5 ft
8 ft
3 ft

522
4.5 Equivalent Force-Couple Systems Example 11, page 4 of 4
8 Solving Eq. 2 gives
x = 4.80 ft Ans.

523
4.5 Equivalent Force-Couple Systems Example 12, page 1 of 3
Rz = Fz: Rz = 300 N 850 N 400 N

= 1550 N
= 1.55 kN
12. Three signs are supported by an arch over a highway and are
acted upon by the wind forces shown. Replace the forces by an
equivalent force and specify its point of application.
Ans.
Determine the resultant. 1
6 m
C
4 m
B
850 N
400 N
A
O
3.5 m
y
300 N
4 m
1 m
z
5 m
x

524
4.5 Equivalent Force-Couple Systems Example 12, page 2 of 3
A
B
C
(x, y, 0)
1550 N
y
z
y
y
O
x
5 m
z
1 m
4 m
300 N
y
3.5 m
O
A
400 N
850 N
B
4 m
C
6 m
x
x
4 Mx
R
= Fd: (1550 N)y = (850 N)(6 m) (400 N)(4 m) (300 N)(3.5 m) (1)
Place the resultant R at the arbitrary point (x, y, 0) and then
determine values of x and y such that R produces the same
moments about the x and y axes as the given forces produce.
2
3 First equate moments about the x axis.
B
C
A
850 N
400 N
300 N
3.5 m
4 m
6 m
z
x,O
y
x,O
y
R = 1550 N
z
View from the
positive x axis

525
4.5 Equivalent Force-Couple Systems Example 12, page 3 of 3
Solving Eq. 1 gives
y = 5.0 m Ans.
5
x
=
1 m
R = 1550 N
y, O
x
y,O
400 N 850 N
300 N
C B A
View from the
positive y axis
Next, equate moments about the y axis. 6
Ans.
Mx
R
= Fd: (1550 N)x = (300 N)(1 m) + (850 N)(1 m + 5 m) + (400 N)(1 m + 5 m + 4 m)
Solving gives
x = 6.06 m
7
x
5 m 4 m

526
4.5 Equivalent Force-Couple Systems Example 13, page 1 of 6
13. Three forces act on the pipe assembly. Determine the
magnitude of forces P and Q if the resultant of all three
forces is to act on point A. Also determine the magnitude
of the resultant.
B
A
C
D
O
P
Q
y
x
z
200 N
3 m
2 m
1.5 m
1 m
1 Express the resultant R in terms of P and Q.
R = F
y
: R = P + Q + 200 N (1)

527
4.5 Equivalent Force-Couple Systems Example 13, page 2 of 6
Place the resultant R at point A and then determine values of
P and Q such that R produces the same moment about the x
and z axes as the given forces, P, Q, and 200 N, produce.
B
C
D
P
Q
x
z
200 N
r
OD
r
OC
r
OB
y
2
R = ( P Q 200 N)j
We can use either the scalar definition of moment M = Fd,
or the vector definition, M = r F. Let's use the vector
definition and calculate moments with respect to point O.
r
OA
z
D
A
C
3
Introduce position vectors, all with tails at point O.
r
OB
= {3i} m
r
OC
= {5i + 1.5k} m
r
OD
= {5i + 2.5k} m
r
OA
= {3i + 1.5k} m
x
4
y
A
3 m
2 m
1 m
1.5 m
3 m
1.5 m
O
O
528
4.5 Equivalent Force-Couple Systems Example 13, page 3 of 6
For equivalence, equate the moment of the resultant R
about point O to the moment of the given forces about O.
5
M
O
R
= M
O
: r
OA
R = r
OB
{ Pj} + r
OC
{ Qj} + r
OD
{ 200j}
or
{3i + 1.5k} ( P Q 200)j = {3i} { Pj} + {5i + 1.5k} { Qj} + {5i + 2.5k} { 200j}
3( P Q 200)i j + 1.5( P Q 200)k j = 3( P)i j + 5( Q)i j + 1.5( Q)k j + 5( 200)i j + 2.5( 200)k j
= k = i = k = k = i = k = i

k
j
i
Collecting coefficients of i, j, and k gives
1.5(P + Q + 200)i 3(P + Q + 200)k = (1.5Q + 500)i + ( 3P 5Q 1000)k
Equating coefficients of i gives
1.5(P + Q + 200) = 1.5Q + 500 (2)
Equating coefficients of k gives
3(P + Q + 200) = 3P 5Q 1000 (3)
6

529
4.5 Equivalent Force-Couple Systems Example 13, page 4 of 6
P and Q were defined as
downward directed
forces.
8
Using these values in Eq. 1 gives
R = P + Q + 200 N


= 133.3 N
Solving Eqs. 2 and 3 gives, 7
ns. P = 133.3 N
Q = 200 N = 200 N
133.3
200
ns.
ns.

530
4.5 Equivalent Force-Couple Systems Example 13, page 5 of 6
Alternative solution: the computation are simplified
somewhat if we sum moments about point A instead
of point O.
Equate the moment of R about point A to
the moment of the given forces about A.
11
200 N
r
AC
D
A
r
AD C
Q
9
r
AB
P
Introduce position vectors, all with
tails at point A.
r
AB
= { 1.5k} m
r
AC
= {2i} m
r
AD
= {2i + k} m
10
x
R
z
y
3 m
2 m
1 m
1.5 m
O
B
A
C
D
O
y
B

531
4.5 Equivalent Force-Couple Systems Example 13, page 6 of 6
Equating coefficients of i gives
0 = 1.5P + 200 (4)
Equating coefficients of k gives
0 = 2Q 400 (5)
Solving Eqs. 4 and 5 gives,
P =
Q = N
Same result as before.
M
A
R
= M
A
: 0 = r
AB
{ Pj} + r
AC
{ Qj} + r
AD
{ 200j}
or
0 = { 1.5k} { Pj} + {2i} { Qj} + {2i + k} { 200j}
0 = 5( P)k j + 2( Q)i j + 2( 200)i j + ( 200)k j
= i = k = k = i

0i + 0j = ( 1.5P + 200)i + ( 2Q 400)k
Because R passes
through point A, the
moment is zero.
12
k
j
i

532
4.5 Equivalent Force-Couple Systems Example 14, page 1 of 4
14. The end plate of a pressurized tank is held in place by forces from three bolts.
Determine the required value of bolt-force P and angle if the resultant of the
three bolt forces is to act through the center of the plate at O.
y
x
z
P 500 N
B
A
700 N
O
0.5 m
40
C

533
4.5 Equivalent Force-Couple Systems Example 14, page 2 of 4
1 Place the resultant, R, at point O and then equate the moment of R about O (which is
zero because R passes through O) to the moment of the given forces about O.
2
y
x
z
P 500 N
B
A
700 N
O
0.5 m
C
r
OC
r
OB
r
OA
Introduce position vectors,
all with tails at point O.
r
OA
= {0.5j} m
r
OB
= {0.5 cos 40i 0.5 sin 40j} m = {0.3830i 0.3214j} m
r
OC
= { 0.5 cos i 0.5 sin j} m
y
z
B
A
O
0.5 m
C
R
x
40

534
4.5 Equivalent Force-Couple Systems Example 14, page 3 of 4
Equate moments about O. 3
M
O
R
= M
O
: 0 = r
OA
{ 700k} N + r
OB
{ 500k} N + r
OC
{ Pk} N
or
0 = {0.5j} { 700k} + {0.3830i 0.3214j} { 500k} + { 0.5 cos i 0.5 sin j} { Pk}
0 = 0.5( 700)j k + 0.3830( 500)i k 0.3214( 500)j k 0.5 cos ( P)i k 0.5 sin ( P)j k
= i = j = i = j = i

k
j
i
Collecting coefficients of i and j gives
0i + 0j = [0.5( 700) 0.3214( 500) 0.5 sin ( P)]i + [0.3830(500) 0.5 cos (P)]j (1)

Equating coefficients of i gives
0 = [0.5( 700) 0.3214( 500) 0.5 sin ( P)]
or, after some arithmetic,
P sin = 378.6 (2)
4
R passes through O, so
produces zero moment.

535
4.5 Equivalent Force-Couple Systems Example 14, page 4 of 4
5 Equating coefficients of j in Eq. 1 gives
0 = 0.3830(500) 0.5 cos (P)
or, after some arithmetic,
P cos = 383.0 (3)
Eqs. 2 and 3 are best solved by using a calculator capable of solving simultaneous
nonlinear equations. Alternatively, dividing Eq. 2 by Eq. 3 gives
6
P sin 378.6
P cos 383.0
tan
Solving for gives,
= 44.7
=
Substituting for in Eq. 3 gives,
P cos = 383.0
44.7
Solving gives
P = 539 N
ns.
ns.

536
4.5 Equivalent Force-Couple Systems Example 15, page 1 of 6
z
6 m
A
O
y
x
B
60 N
40 N
140 N
15. Replace the system of forces by a wrench.
Determine the pitch and axis of the wrench.
12 m
4 m

537
4.5 Equivalent Force-Couple Systems Example 15, page 2 of 6
14
r

AB
r

AB
Thus the vector form of the 140-N force is
F

AB
= (140 N)
= (140 N)

= {40i + 120j 60k} N (1)
4
2
+ 12
2
+ ( 6)
2
4i + 12j 6k
4 m
12 m
r

AB
B
x
y
A 6 m
z
2
1 Express the 140-N force in terms of
rectangular components. First introduce a
position vector r

AB
from A to B.
r

AB
{4i + 12j 6k} m
4
Forces along x and z axes
3 The resultant of all three forces is
R F
{40i + 120j 60k} N {40i} N + {60k} N
120j (2)
140 N
40 N
60 N

538
4.5 Equivalent Force-Couple Systems Example 15, page 3 of 6
60 N
z
6 m
A
O
40 N
y
x
B
C
D
Moment arm is zero.
(40 N force passes through O)
M
R
O
r

OA
F

AB
+ r

OA
(60k) + 0 ( 40i)
(6k) (40i +120j 60k) + (6k) (60k)
{ 720i + 240j} N m
6
Moment resultant about O: 5
F

AB
r

OA
{6k} m
12 m
6(40) k i + 6(120) k j + 6( 60) k k + 6(60) k k
j
i
0 0

539
4.5 Equivalent Force-Couple Systems Example 15, page 4 of 6
Express M
R
O
in terms of components parallel M and
perpendicular M to R.
7
By definition,
pitch of wrench, p,
M = { 720i} N m
R {120j} N
M {240j} N m
M
R
O
{ 720i +240j} N m
x
z
y
8
M
R
240
120
2 m Ans.
||
||
||

540
4.5 Equivalent Force-Couple Systems Example 15, page 5 of 6
10
Old line of action of R
(Intersection of new line
of action with x-z plane)
New position of R
New line of action of R
Old position of R
R
r

OP
{xi + zk} m
O
r

OP
P(x, 0, z)
x(120) i j + z(120) k j 720i
k i
(120z)i + (120x)k 720i + 0k
Equating coefficients of i gives
120z = 720
z = = 6 m
Equating coefficients of k gives
120x = 0
x = 0
720
120
Now move R to a new line of action such that in the
new location R will produce a moment (about O)
equal to M .
Equate the moment of R in the new position to M
r
OP
R M
or
{xi + zk} {120j} 720i
M = { 720i} N m
R {120j} N
x
z
y
9

541
4.5 Equivalent Force-Couple Systems Example 15, page 6 of 6
6 m
Ans.
Ans.
||
R {120j} N
M {240j} N m
x
z
y
O
Because a couple moment has the same value
about all points it can be moved to the new
line of action of R to form the wrench.
The axis of the wrench is a vertical line
(same direction as R {120j}) passing
through the point (0, 0, 6 m)
12
11

542
4.5 Equivalent Force-Couple Systems Example 16, page 1 of 14
F

1
4 lb
M

1
60 lb in.
1.5 in.
M

2
80 lb in.
30
F

2
6 lb
z
6 in.
1 in.
3 in.
D
x
16. In a machining operation, holes are simultaneously drilled at points
A and B of the wedge. The drill at A produces a force and couple
moment perpendicular to the planar surface at A. The force and couple
moment at B are similarly perpendicular to the planar surface at B.
Replace the forces and couple moments by a wrench. Determine
a) the magnitude R of the resultant force,
b) the pitch of the wrench,
c) the axis of the wrench, and
d) the point where the axis intersects the x-z plane.
E
B
A
C
y
4 in. 4 in.

543
4.5 Equivalent Force-Couple Systems Example 16, page 2 of 14
Express the forces and couple moments in rectangular components.
90 30
View from positive x axis
1
Equal
2
E
x, D
C
z
30
A
30
E
C
A
x, D
z
Geometry
M

2
60 lb in.
F

1
4 lb
60
y
y
60
60
F

1
(4 lb) sin 60j (4 lb) cos 60k
{ 3.464j 2k} lb (1)
M

1
(60 lb in.) sin 60j (60 lb in.) cos 60k
{ 51.962j 30k} lb in. (2)
3

544
4.5 Equivalent Force-Couple Systems Example 16, page 3 of 14
z
B
y
M

2
80 lb in.
F

2
6 lb
x
4 F

2
{ 6i} lb (3)
M

2
{ 80i} lb in. (4)

545
4.5 Equivalent Force-Couple Systems Example 16, page 4 of 14
4 in.
z
C
4 in.
E
A
B
y
B
30
E
(3 in.) cos 30 2.598 in.
A
1.5 in.
x, D
1 in.
6 in.
3 in.
y
C
z
x
Next, determine the coordinates of points A and B.
A

y
(3 in.) sin 30 1.5 in.
A

z (6 in. + 1 in.) 2.598 in.
4.402 in.
6
B

y
1.5 in.
B

z 1 in.
A

x 4 in.
B

x 4 in. + 4 in.
8 in.
8
7
5

546
4.5 Equivalent Force-Couple Systems Example 16, page 5 of 14
z
A
B
y
x
O
Introduce position vectors from O, the origin of coordinates, to A and B.
r

OA
A

xi + A

y
j + A

zk
{4i + 1.5j + 4.402k} in. (5)
9
10
r

OB
B

xi + B

y
j + B

zk
{8i + 1.5j + k} in. (6)
11

547
4.5 Equivalent Force-Couple Systems Example 16, page 6 of 14
Resultant force
R F: R { 3.464j 2k} + { 6i}
{ 6i 3.464j 2k} lb (7)
12
Resultant moment about point O
M
R
O
M

O
: M
R
O
r

OA
F

1
+ r

OB
F

2
13
Carrying out the cross products and simplifying gives
M
R
O
{12.249i + 2j 4.856k} (8)
i j k i j k
M
R
O
= 4 1.5 4.402 + 8 1.5 1
0 3.464 2 6 0 0

548
4.5 Equivalent Force-Couple Systems Example 16, page 7 of 14
Display R and M
O
with respect to the xyz axes. 14
x
y
z
O
R
M
R
O
{12.249i + 2j 4.856k}
R { 6i 3.464j 2k} lb

549
4.5 Equivalent Force-Couple Systems Example 16, page 8 of 14
O
M
R
O
R
Two intersecting vectors define a plane. Display the plane defined by R and M
R
O
.
15
P Q
R S

550
4.5 Equivalent Force-Couple Systems Example 16, page 9 of 14
O
M
R
O
R
P
Q
R S
M
u
16
Unit vector in R direction
By Eq. 7 for R,
( 6)
2
+ ( 3.464)
2
+ ( 2)
2
17
0.832i 0.480j 0.277k (10)
7.211 lb Ans. (9)
u
R
R
R
u
R
R
6i 3.464j 2k
7.211
Thus
M
Express M
R
O
in terms of components
parallel M and perpendicular M to R.
||
||
M 9.806 lb in. (11)
{12.249i + 2j 4.856k}{ 0.832i 0.480j 0.277k}
Performing the multiplications and
then simplifying gives
18
M component of M
R
O
in direction of u
M
R
O
u
||
||
19
M M u
( 9.806){ 0.832i 0.480j 0.277k}
{8.159i + 4.707j + 2.716k} lb in. (12)
In vector form, since M lies in the
direction of the unit vector u,
||
|| ||

551
4.5 Equivalent Force-Couple Systems Example 16, page 10 of 14
The component of M
R
O
perpendicular to R can now be
computed by starting with the vector sum
{12.249i + 2j 4.856k} {8.159i + 4.707j + 2.716k}
{4.090i 2.707j 7.572k} lb in. (13)
and rearranging to get
20
M = M
R
O
M
M
R
O
M M
by Eq.8
by Eq.12
||
||

552
4.5 Equivalent Force-Couple Systems Example 16, page 11 of 14
Now we move the force R to a new line of
action such that R will produce a moment
about O exactly equal to M
21
Old position of R
Old line of action of R
New line of action of R
New position of R
(below plane SPQR)
O
R
P
Q
R S
M

||
M
O
R
Parallel

553
4.5 Equivalent Force-Couple Systems Example 16, page 12 of 14
View in terms of xyz axes
O
R
Old line of action of R
New line of action of R
y
x
22
P(x, 0, z)-intersection of new line
of action with the x-z plane
New position of R
Old position of R
Parallel
r

OP
z
Because the force R, in its new position, is to create a
moment about O equal to M , we can write
r

OP
R M
23
{xi + zk} { 6i 3.464j 2k} 4.090i 2.707j 7.572k
3.464zi + (2x 6z)j 3.464xk 4.090i 2.707j 7.572k
Performing the multiplications and simplifying gives
or,
R
O
by Eq.7
by Eq. 13

554
4.5 Equivalent Force-Couple Systems Example 16, page 13 of 14
Equating coefficients of i gives
3.464z 4.090 (14)
Similarly for j and k
2x 6z 2.707 (15)
3.464x 7.572 (16)
These are three equations in only two unknowns, but the
equations are not all independent. Eqs 14 and 16 imply
Substituting these values into the left hand side of Eq.15 gives
2x 6z 2(2.186) 6(1.181) 2.714
Thus Eq.15 is satisfied (Round-off, error leads to 2.714 rather than 2.707).
24
z
4.090
3.464
1.181 (17)
x
7.572
3.464
2.186 (18)

555
4.5 Equivalent Force-Couple Systems Example 16, page 14 of 14
Summary: The effect of the drilling operations on the two
surfaces of the wedge is to push the wedge down the
wrench axis (direction of u) while simultaneously causing
the wedge to tend to rotate counterclockwise (as viewed
from a position above the x-z plane) about the wrench axis.
Because a couple moment is the same about all
points, we can move M to point P. Similarly
we can slide R to P along R's line of action, by
the principle of transmissibility.
25
28
The axis of the wrench is a line passing through the
point (2.186 in., 0, 1.181 in.) with direction, by Eq. 10,
u 0.832i 0.480j 0.277k
26
R Ru

(7.211 lb)u
by Eq. 9
M M u

( 9.805 lb in.)u
Pitch of wrench
M
R
9.806 lb in.
7.211 lb
1.360 in.
27
by Eq. 11
y
z
x
2.186 in.
1.181 in.
P
u
Ans.
Ans.
||
||
||
||

556

4.6 Distributed Loads on Beams
557
4.6 Distributed Loads on Beams Procedures and Strategies, page 1 of 1
xw(x) dx
w(x) dx
Area = wL
L
L
R = wL
w
wo
R =
woL
1
2
3 3
2L L
L
Procedures and Strategies for Solving Problems Involving Distributed
Loads Acting on Beams
1. To calculate the resultant R of a distributed load, calculate the
area under the load curve w(x) either by integration or by using the
formulas for area (for a rectangle or triangle, for example):
R = w(x) dx = Area
2. To locate the line of action, locate the centroid of the area under
the load curve either by integration or by using the formulas for
locating centroids of simple shapes.
x
R
=

558
4.6 Distributed Loads on Beams Problem Statement for Example 1
1. Determine a) the magnitude and location of the resultant
of the distributed load, and b) the reactions at the supports.
4 m 2 m
B
A
2 kN/m

559
4.6 Distributed Loads on Beams Problem Statement for Example 2
2. Determine a) the magnitude and location of the resultant of
the distributed load, and b) the reactions at the support.
6 m
4 kN/m
B
A

560
4.6 Distributed Loads on Beams Problem Statement for Example 3
3. Determine a) the magnitude and location of the resultant
of the distributed load, and b) the reactions at the supports.
50 lb/ft
200 lb/ft
A
B
12 ft

561
4.6 Distributed Loads on Beams Problem Statement for Example 4
A
300 N/m
900 N/m
250 N/m
B
3 m 1.5 m
4. Determine the reactions at the supports.

562
4.6 Distributed Loads on Beams Problem Statement for Example 5
3 ft 9 ft
A
2 kip/ft
2 kip/ft
5. Determine the reactions at the support.

563
4.6 Distributed Loads on Beams Problem Statement for Example 6
A
B
w = w

o sin
w
x
L
6. Determine the reactions at the supports.
x
L

564
4.6 Distributed Loads on Beams Problem Statement for Example 7
A B
x
w
4 kN/m
40 kN 14.4 kN/m
3 m 5 m
7. Determine the reactions at the supports.
w = 2 x
2
2x + 4 kN/m

565
4.6 Distributed Loads on Beams Problem Statement for Example 8
3 m
x
8. The lift force acting on an airplane wing can be modeled by
the equation shown. Determine the force and moment at the
point where the wing is attached to the fuselage.
p(x) = [1500 10(x
2
+ 4)] N/m
A

566
4.6 Distributed Loads on Beams Problem Statement for Example 9
B
A
9. Determine the reactions at the supports.
120 lb/ft
360 lb/ft
9 ft 4 ft

567
4.6 Distributed Loads on Beams Problem Statement for Example 10
2 ft 2 ft
3 kip ft
4 kip/ft
A
B
10. Determine the reactions at the supports.
6 ft

568
4.6 Distributed Loads on Beams Problem Statement for Example 11
2 m
A B
11. Determine w

1
and w

2
so that the beam is in equilibrium.
8 kN
w

1
w

2
4 m

569
4.6 Distributed Loads on Beams Problem Statement for Example 12
3 kN/m
8 kN m
w

1
w

2
A B
C
0.4 m
1 m
4 m
1 m
2 m
0.3 m
12. The forces from the supports are approximately represented as
triangular distributed loads. Determine the values of w

1
and w

2
.

570
4.6 Distributed Loads on Beams Example 1, page 1 of 3
B
A
2 kN/m
A
B
6 m
Distributed load diagram. The magnitude R
of the resultant of the distributed load equals
the area under the load curve.
1
2 R = Area
= (2 kN/m)(6 m)
= 12 kN Ans.
2 kN/m
1. Determine a) the magnitude and location of the resultant
of the distributed load, and b) the reactions at the supports.
4 m 2 m

571
4.6 Distributed Loads on Beams Example 1, page 2 of 3
A

x
B

y
A

y
3 m
6 m
The line of action of the resultant, R, passes through the centroid of the load area.
3 m
R = 12 kN
3
A
B
4 Centroid of rectangle lies
= 3 m from A Ans
6 m
2
5 Free-body diagram
A
R = 12 kN
B
4 m 2 m

572
4.6 Distributed Loads on Beams Example 1, page 3 of 3
Equations of equilibrium,
F

x = 0: A

x = 0
F

y
= 0: A

y
+ B

y
12 kN = 0
M

A
= 0: (4 m)(B

y
) (3 m)(12 kN) =0
Solving gives
A

x = 0 Ans.
A

y
= 3 kN Ans.
B

y
= 9 kN Ans.
+
+
6
+

573
4.6 Distributed Loads on Beams Example 2, page 1 of 3
4 kN/m
B
A
6 m
B
A
4 kN/m
Distributed load diagram. The magnitude R of
the resultant of the distributed load equals the
area under the load curve.
1
R = Area
= (4 kN/m)(6 m)
= 12 kN Ans.
1
2
2
2. Determine a) the magnitude and location of the resultant of
the distributed load, and b) the reactions at the support.
6 m

574
4.6 Distributed Loads on Beams Example 2, page 2 of 3
R = 12 kN
B
A
A

x
A

y
M

A
6 m
3
2 m
4 Centroid of triangle lies
(6 m) = 2 m from B Ans.
1
3
5
2 m
B
A
The line of action of the resultant, R, passes
through the centroid of the triangle.
R = 12 kN
x
y
Free-body diagram. Since the support at A is fixed,
three reactions are present: A

x, A

y
, and M

A
.

575
4.6 Distributed Loads on Beams Example 2, page 3 of 3
6
+
Equations of equilibrium:
F

x = 0: A

x = 0
F

y
= 0: A

y
12 kN = 0
M

A
= 0: (12 kN)(4 m) + M

A
= 0
Solving gives
A

x = 0 Ans.
A

y
= 12 kN Ans.

M

A
= 48 kN m Ans.
+
+

576
4.6 Distributed Loads on Beams Example 3, page 1 of 3
3. Determine a) the magnitude and location of the resultant
of the distributed load, and b) the reactions at the supports.
50 lb/ft
200 lb/ft
A
B
12 ft
12 ft
A B
F

2
= Area

2
= (150 lb/ft)(12 ft)
= 900 lb
F

1
= Area

1
= (50 lb/ft)(12 ft)
= 600 lb
Distributed load diagram. The trapezoid loading area can
be divided into rectangular and triangular loadings.
50 lb/ft
150 lb/ft
1
(2)
(1)
1
2
2
3

577
4.6 Distributed Loads on Beams Example 3, page 2 of 3
A
The lines of action of F

1
and F

2
pass through the centroids of the
rectangular and triangular loading areas respectively.
B
F

1
= 600 lb
R
F

2
= 900 lb
4 ft
6 ft
A single resultant, R, can be
calculated as:
R = F

y
= F

1
+ F

2


= 600 lb + 900 lb
= 1500 lb Ans.
+
To be equivalent, the moment about A produced by R must equal
the sum of the moments about A produced by F

1
and F

2
.
x

R
R = (6 ft)F

1
(4 ft)F

2
where x

R
is the distance of R from the support A.
Hence,
x

R
(1500 lb) = (6 ft)(600 lb) (4 ft)(900 lb)
x

R
= 4.8 ft Ans.
5
Centroid of triangle lies
(12 ft) = 4 ft from A
1
3
6
Centroid of rectangle lies (12 ft) = 6 ft from A

1
2
7
4
8
x

R
+

578
4.6 Distributed Loads on Beams Example 3, page 3 of 3
12 ft
A
R = 1500 lb
B
Free- body diagram
A

x
A

y B

y
Equations of equilibrium
F

x = 0: A

x = 0
F

y
= 0: A

y
+ B

y
1500 lb = 0
M

A
= 0: (12 ft)(B

y
) (1500 lb)(4.8 ft) = 0
Solving gives
A

x = 0 Ans.
A

y
= 900 lb Ans.
B

y
= 600 lb Ans.
+
+
9
10
x

R
= 4.8 ft
+

579
4.6 Distributed Loads on Beams Example 4, page 1 of 3
A
300 N/m
900 N/m
250 N/m
B
3 m 1.5 m
4. Determine the reactions at the supports.
B
3 m
A
1.5 m
Distributed load diagram. 1
(1)
(2)
(3)
(900 N/m 300 N/m) = 600 N/m
250 N/m
300 N/m
Resultant of load No. 1
F

1
= Area

1
= (300 N/m)(3 m)
= 900 N
Resultant of load No. 2
F

2
= Area

2
= (600 N/m)(3 m)
= 900 N
1
2
Resultant of load No. 3
F

3
= Area

3
= (250 N/m)(1.5 m)
= 375 N
2
3
4

580
4.6 Distributed Loads on Beams Example 4, page 2 of 3
5
F

1
F

2
F

3
1.5 m
2 m
3.75 m
Centroid of rectangle 1 lies
= 1.5 m from A
3 m
2
Centroid of rectangle 3 lies
3 m + = 3.75 m from A
1.5 m
2
Centroid of triangle 2 lies
(3 m) = 2 m from A
2
3
6
7
8
9 Free body diagram
B

y
A

x
A

y
1.5 m
2 m
3.75 m
4.5 m
(2)
3 m
A
(3)
1.5 m
(1)
B
The lines of action of resultants F

1
, F

2
, and F

3
pass through the
centroid of their loading areas.
F

3
= 375 N F

1
= 900 N
A
F

2
= 900 N
B

581
4.6 Distributed Loads on Beams Example 4, page 3 of 3
Equations of equilibrium
F

x = 0: A

x = 0
F

y
= 0: A

y
+ B

y
900 N 900 N 375 N 0
M

A
= (900 N)(1.5 m) (900 N)(2 m) (375 N)(3.75 m) + B

y
(4.5 m) = 0
Solving gives
A

x = 0 Ans.
A

y
= 1162 N Ans.
B

y
= 1012 N Ans.
+
+
10
+

582
4.6 Distributed Loads on Beams Example 5, page 1 of 3
3 ft 9 ft
A
2 kip/ft
2 kip/ft
5. Determine the reactions at the support.
A
9 ft 3 ft
1 Distributed load diagram
(1)
(2)
2 kip/ft
2 kip/ft
2 F

1
= Area

1
= (12 ft)(2 kip/ft)
= 12 kip
1
2
F

2
= Area

2
= (9 ft)(2 kip/ft)
= 9 kip
1
2
3

583
4.6 Distributed Loads on Beams Example 5, page 2 of 3
M

A
A

x
A

y
9 ft
12 ft
9 ft
F

2
The lines of actions of the resultants pass through the centroids of the loading areas.
A
F

1
1
Centroid of triangle 1 lies
(12 ft) = 4 ft from A
3
Centroid of triangle 2 lies
3 ft + (9 ft) = 9 ft from A
4 ft
2
3
4
5
6
7
4 ft
3 ft
9 ft
F

2
= 9 kip
A
F

1
= 12 kip
Free- body diagram

584
4.6 Distributed Loads on Beams Example 5, page 3 of 3
+
Equations of equilibrium
F

x = 0: A

x = 0
F

y
= 0: A

y
12 kip + 9 kip = 0
M

A
= 0: M

A
12 kip(4 ft) + 9 kip(9 ft) = 0
Solving gives
A

x = 0 Ans.
A

y
= 3 kip Ans.
M

A
= 33 kip ft Ans.
+
+
8

585
4.6 Distributed Loads on Beams Example 6, page 1 of 3
A
B
w = w

o sin
w
x
L
6. Determine the reactions at the supports.
Distributed load diagram. The magnitude and location of the
resultant of the given load will be determined by integration.
A
w
x
B
x
L
dA = w dx
The resultant force R = dA = w dx
R = w

o sin dx = w

o[ cos ] =
1
2
A

L

0

0

L
x
L
w = w

o sin
L
x
x
L
L
L
x

L

0
2w

oL
dx

586
4.6 Distributed Loads on Beams Example 6, page 2 of 3
R
A
B
L
2
L
2
A

x
A

y
B

y
C
R =
Free- body diagram
3
4
L
0
x

R
2w

oL
2w

oL
L
A
B
Resultant force diagram. The location x

R
of R measured from A is

x

R
=


=


=
L
2
x dA
dA
x w

o sin
x
L
dx

587
4.6 Distributed Loads on Beams Example 6, page 3 of 3
2w

oL
2w

oL
w

oL
w

oL
+
2
L
Equations of equilibrium
F

x = 0: A

x = 0
F

y
= 0: A

y
+ B

y
= 0
M

A
= 0: B

y
L ( ) = 0
Solving gives

A

x = 0 Ans.
A

y
= Ans.
B

y
= Ans.
+
+
5

588
4.6 Distributed Loads on Beams Example 7, page 1 of 2
A B
x
w
4 kN/m
40 kN 14.4 kN/m
3 m 5 m
7. Determine the reactions at the supports.
Distributed load diagram. The magnitude and location of the force
represented by the distributed load will be determined by integration.
A
w
B
x
x
dx
8 m
dA = w dx
4 kN/m
14.4 kN/m
1
w = 2 x
2
2x + 4 kN/m
w = 2 x
2
2x + 4 kN/m

589
4.6 Distributed Loads on Beams Example 7, page 2 of 2
40 kN
F = 60.44 kN
2x x
2
2x + 4 dx
The force F = dA = w dx
F = 2 x
2
2x + 4 dx = 60.44 kN
and F acts through the point defined by x

1
:
x

1
= = = = 5.05 m
dA
x dA
2
Equations of equilibrium
F

x = 0: A

x = 0
F

y
= 0: A

y
+ B

y
40 kN 60.44 kN = 0
M

A
= 0: B

y
(8 m) 40 kN(3 m) 60.44 kN(5.05 m) = 0
Solving gives
A

x = 0 Ans.
A

y
= 47.3 kN Ans.
B

y
= 53.1 kN Ans.
+
+
40 kN
3 m
8 m
5.05 m
A
A

x
A

y
Free- body diagram
B

y
B
F = 60.44 kN
3
4

8
0

0

8
+
305.09
60.44 60.44
A B
3 m
x

1

590
4.6 Distributed Loads on Beams Example 8, page 1 of 3
Distributed load diagram. The magnitude and location of
the resultant force will be determine by integration.
p(x) = [1500 10(x
2
+ 4)] N/m
dA = p(x) dx
dx
x
3 m
1
A
A
3 m
x
8. The lift force acting on an airplane wing can be modeled by
the equation shown. Determine the force and moment at the
point where the wing is attached to the fuselage.
p(x) = [1500 10(x
2
+ 4)] N/m

591
4.6 Distributed Loads on Beams Example 8, page 2 of 3
F

R
_
A
Free- body diagram
4290 N
A
A

x
M

A
A

y
1.484 m
3 m
3 m
2
3
A

3

0

0

3
4290
6368
x
The resultant force R is equal to the area under the loading curve.

R = dA = [1500 10(x
2
+ 4)] dx = 4290 N
The line of action of R passes through the point defined by x
R
:
x
R
= = = = 1.484 m
x dA
dA
x[1500 10(x
2
+ 4)] dx
4290

592
4.6 Distributed Loads on Beams Example 8, page 3 of 3
+
Equations of equilibrium
F

x = 0: A

x = 0
F

y
= 0: A

y
+ 4290 N = 0
M

A
= 0: M

A
+ (4290 N)(1.484 m) = 0
Solving gives
A

x = 0 Ans.
A

y
= 4290 N = 4.29 kN Ans.
M
A
= 6366 N m = 6.37 kN m Ans.
+
+
4

593
4.6 Distributed Loads on Beams Example 9, page 1 of 3
9 ft 4 ft
B
A
9. Determine the reactions at the supports.
120 lb/ft
360 lb/ft
Distributed load diagram. The distance x will be determined later.
A
B
x
9 ft x
1
120 lb/ft
360 lb/ft
(1)
(2)
F

1
= Area

1
= (120 lb/ft)(x)
2
1
F

2
= Area

2
= (360 lb/ft)(9 ft x)
2
1
2
3

594
4.6 Distributed Loads on Beams Example 9, page 2 of 3
4
x
120 lb/ft 360 lb/ft
9 x
1
2
1
2
5
0.75 ft
1
Centroid of triangle 1 lies (2.25 m) = 0.75 m from A
3
2
Centroid of rectangle 2 lies
2.25 ft + (6.75 ft) = 6.75 ft from A
3
6
7
9 ft 2.25 ft = 6.75 f t
From similar triangles, =
Solving gives
x = 2.25 ft.
The force represented by upper triangle F

1
= (120 lb/ft)(2.25 ft) = 135 lb.
The force represented by lower triangle F

2
= (360 lb/ft)(9 ft 2.25 ft) = 1215 lb.
The lines of action of these forces pass through the centroids of the corresponding areas.
F

1
= 135 lb
6.75 ft
2.25 ft
A
F

2
= 1215 lb
B

595
4.6 Distributed Loads on Beams Example 9, page 3 of 3
Equations of equilibrium
F

x = 0: A

x = 0
F

y
= 0: A

y
135 lb + 1215 lb B

y
= 0
M

A
= 0: (135 lb)(0.75 ft) + (1215 lb)(6.75 ft) B

y
(13 ft) = 0
Solving we get
A

x = 0 Ans.
A

y
= 457 lb Ans.
B

y
= 623 lb Ans.
+
+
13 ft
6.75 ft
135 lb
Free-body diagram
A
0.75 ft
A

x
A

y
1215 lb
B
B

y
8
9
+

596
4.6 Distributed Loads on Beams Example 10, page 1 of 3
2 ft 2 ft
3 kip ft
4 kip/ft
A
B
6 ft
A
B
1 Distributed load diagram
4 kip/ft
Resultant
F = Area
= (4 kip/ft)(6 ft)
= 12 kip
2
2
1
10. Determine the reactions at the supports.
6 ft

597
4.6 Distributed Loads on Beams Example 10, page 2 of 3
6 ft
The line of action of the resultant F passes through the centroid of the loading area.
A
B
Free-body diagram
A
A

x
A

y
F = 12 kip
B
B

y
3 kip ft
2
Centroid of triangle lies
(6 ft) = 4 ft from A
3
4
5
4 ft
4 kip/ft
4 ft
2 ft 6 ft + 2 ft = 8 ft
F
3

598
4.6 Distributed Loads on Beams Example 10, page 3 of 3
Equations of equilibrium
F

x = 0: A

x = 0
F

y
= 0: A

y
12 kip + B

y
= 0
M

A
= 0: (12 kip)(4 ft) (3 kip ft) + B

y
(6 ft + 2 ft + 2 ft) = 0
Solving gives
A

x = 0 Ans.
A

y
= 6.90 kip Ans.
B

y
= 5.10 kip Ans.
+
+
6
+

599
4.6 Distributed Loads on Beams Example 11, page 1 of 3
2 m
Distributed load diagram. The trapezoidal area can
be divided into rectangular and triangular areas.
8 kN
6 m
A B
A B
1
w

1
(1)
(2)
Resultant of load No. 2
F

2
= Area

2
= (w

2
w

1
)(6 m)
= 3(w

2
w

1
)
Resultant of load No. 1
F

1
= Area

1
= (w

1
)(6 m)
= 6w

1
2
1
2
3
w

2
w

1
11. Determine w

1
and w

2
so that the beam is in equilibrium.
8 kN
w

1
w

2
4 m

600
4.6 Distributed Loads on Beams Example 11, page 2 of 3
3(w

2
w

1
)
8 kN
The lines of action of the resultants pass through the
centroids of their areas.
A
6w

1
B
4
Centroid of rectangle 1 lies
= 3m from A
6
2
6 m
2
Centroid of triangle 2 lies
(6 m) = 4 m from A
3
4 m
5
7
4 m
3(w

2
w

1
)
3 m
A
6w

1
B
8 kN
Free-body diagram
3 m

601
4.6 Distributed Loads on Beams Example 11, page 3 of 3
Equations of equilibrium
F

y
= 0: 6w

1
+ 3(w

2
w

1
) 8 kN = 0
M

A
= 0: 6w

1
(3 m) + 3(w

2
w

1
)(4 m) 8 kN(4 m) = 0
Solving gives
w

1
= 0 Ans.
w

2
=
8
3
kN/m Ans.
+
8
+

602
4.6 Distributed Loads on Beams Example 12, page 1 of 3
3 kN/m
8 kN m
w

1
w

2
Distributed load diagram.
A B
C
0.4 m
1 m
4 m
1 m
2 m
0.3 m
A
C
B
1
3 kN/m
w

1
w

2
F

3
= Area

3
= (3 kN/m)(4 m)
= 12 kN
F

2
= Area

2
= w

2
(0.3 m)
= 0.15w

2
2
3
1
2
(1)
(2)
(3)
F

1
= Area

1
= w

1
(0.4 m)
= 0.2w

1
1
2
4
12. The forces from the supports are approximately represented as
triangular distributed loads. Determine the values of w

1
and w

2
.

603
4.6 Distributed Loads on Beams Example 12, page 2 of 3
0.15 w

2
8.5 m
6.4 m
3.4 m
0.267 m
0.2 w

1
A
12 kN
8 kN m
C
B
5
3
Centroid of triangle 2 lies
0.4 m + 1 m + 4 m + 1 m + 2 m + (0.3 m) = 8.5 m from A
1
Centroid of rectangle lies
0.4 m + 1 m + = 3.4 m from A
2
4 m
6
7 Centroid of triangle 1 lies
(0.4 m) = 0.267 m from A
2
3
8
The lines of action of the resultant forces pass
through the centroids of their loading areas.

604
4.6 Distributed Loads on Beams Example 12, page 3 of 3
0.2 w

1
A
C
8 kN m
B
Free- body diagram 9
10
+
Equations of equilibrium
F

y
= 0: 0.2w

1
+ 0.15w

2
12 kN = 0
M

A
= 0: (0.2w

1
)(0.267 m) (12 kN)(3.4 m) 8 kN m + (0.15w

2
)(8.5 m) = 0
Solving gives
w

1
= 32.3 kN/m Ans.
w

2
= 36.9 kN/m Ans.
+
0.15 w

2
12 kN
3.4 m
6.4 m
8.5 m
0.267 m

605

5. Equilibrium of a Rigid Body
606

5.1 Constraints and Static Determinacy
607
5.1 Constraints and Static Determinacy Procedures and Strategies, page 1 of 1
Procedures and Strategies for Solving Problems Involving Constraints and
Static Determinacy
To determine the type of constraints,
1. draw a free-body diagram but do not include the applied forces they do not
affect the type of constraint and
2. count the number of reaction components:
a) If the number of components is two or less, then the body is partially
constrained.
b) If the number of components is three or more, and if either 1) all force
components are parallel, or 2) the lines of action of all force components
pass through the same point and no reaction couple-moment is present, then
the body is improperly constrained.
c) If the body is neither partially nor improperly constrained, then it is
properly constrained .
To determine if a body that is known to be in equilibrium under a set of applied loads
is statically determinate or indeterminate,
1. draw a free-body diagram.
2. count the number of reaction components:
a) If the number is greater than three, the body is indeterminate.
b) If the number is three or less, write three equilibrium equations and attempt
to solve them. If a unique solution can be found, then the body is determinate.
Otherwise it is indeterminate.
A
B
C
A
A
B
C
A
B
B
C
C
R
C
R
B
R
A
R
Ay
R
C
R
Ax
x
y

608
5.1 Constraints and Static Determinacy Problem Statement for Example 1
2 kN
C
B A
45
500 N
B A
C B
A
200 lb
2 kN m
C
B
A
a)
c)
d)
b)
1. In each case, determine if the beam is partially,
improperly, or properly constrained.

609
5.1 Constraints and Static Determinacy Problem Statement for Example 2
6 kN m
C
B A
3 kN
D
B
C
A
45
A
B
C
300 N
b)
a)
c)
45
2. In each case, determine if the rigid body is partially,
improperly, or properly constrained.
4 m 4 m

610
5.1 Constraints and Static Determinacy Problem Statement for Example 3
D
10 kN
C
B
A
70 lb
D
C
B
A
10 kN
D C
B
A
A B
C D
400 lb
3. In each case, determine if the rectangular plate is
partially, improperly, or properly constrained.
a)
d)
c)
b)

611
5.1 Constraints and Static Determinacy Problem Statement for Example 4
4 kN
D C
B A
200 N
C
B A
45
45
45
D
900 N
C
B
A
a)
b)
c)
4. In each case, determine if the rectangular plate is
partially, improperly, or properly constrained.

612
5.1 Constraints and Static Determinacy Problem Statement for Example 5
B
C
A
200 N
O
45
Semicircles with
common center at O
D
6 N
20 N
A
C
B
a)
b)
5. In each case, determine if the rigid body is
partially, improperly, or properly constrained.

613
5.1 Constraints and Static Determinacy Problem Statement for Example 6
C B A
10 lb
D
Collar slides freely
on smooth rod
Pin slides freely
in smooth slot
C
50 N
A
B
a)
b)
45
45
45
6. In each case, determine if the rigid body is
partially, improperly, or properly constrained.

614
5.1 Constraints and Static Determinacy Problem Statement for Example 7
20 kN
3 m
C
B
A
D
3 m
45 45
45 45
7. Determine if the rigid body is partially,
improperly, or properly constrained.

615
5.1 Constraints and Static Determinacy Problem Statement for Example 8
8. In each case, determine if the rigid body is
statically determinate or indeterminate.
a)
4 ft 3 ft 5 ft
b)
4 ft
A
C
B
4 kip
4 ft 3 ft 5 ft
A
C
B
6 kip
4 ft

616
5.1 Constraints and Static Determinacy Problem Statement for Example 9
2 m 2 m
2 kN 2 kN
D
A C B
2 m
b)
1 m
2 m
4 m
a)
400 N
D
C
B
A
9. In each case, determine if the rigid body
is statically determinate or indeterminate.

617
5.1 Constraints and Static Determinacy Problem Statement for Example 10
B
C
D
E
45
10. In each case, determine if the rigid body
is statically determinate or indeterminate.
2 m
3 m 3 m
2 m
a)
Collar slides freely
on smooth rod
E
5 kN
D
A
C
B
3 m 3 m
6 m
b)
D
600 N 400 N
C
B A
40 N
2 m
1
3
6 m
1 m 4 m
2 m
c)
A

618
5.1 Constraints and Static Determinacy Example 1, page 1 of 9
2 kN
C
B A
45
500 N
B A
C B
A
200 lb
2 kN m
C
B
A
a)
c)
d)
b)
1. In each case, determine if the beam is partially,
improperly, or properly constrained.

619
5.1 Constraints and Static Determinacy Example 1, page 2 of 9
Definitions: a rigid body in two dimensions is said to be
Partially constrained, if it has two or fewer reaction
components (There are not enough reactions to prevent
motion under all possible loading conditions);
Improperly constrained, if it has three or more reaction
components and either a) the reaction forces are all parallel,
or b) the lines of action of the reaction forces intersect at a
common point, and no couple moment is present (There are
enough reactions, but they are not properly arranged to
prevent motion under all possible loading conditions); and
Properly constrained, if it is neither partially nor
improperly constrained (There are enough reactions and they
are properly arranged to prevent motion under all possible
loading conditions):

620
5.1 Constraints and Static Determinacy Example 1, page 3 of 9
A B
C
2 kN
C
B A
R

A
R

B
R

C
a)
Two or fewer reactions?
No. Conclusion: not
partially constrained.
2
1 Show the reactions. But do not show the applied 2-kN
force: the reaction forces, not the applied forces,
determine the constraints of the body.

621
5.1 Constraints and Static Determinacy Example 1, page 4 of 9
R

C
R

B
R

A
A B
C
A C
All reactions parallel?
Yes. Conclusion:
improperly constrained Ans.
(Horizontal translation not prevented.)
3

622
5.1 Constraints and Static Determinacy Example 1, page 5 of 9
B
R

Bx
R

By
M

B
A B
500 N
b)
Two or fewer reactions?
No. Conclusion: not
partially constrained.
2
Show the reactions. 1
45

623
5.1 Constraints and Static Determinacy Example 1, page 6 of 9
M

B
R

By
R

Bx
B
B
R

Bx
R

By
M

B
All reactions parallel?
No. Conclusion: can't translate
(There is no direction in which the
body could translate without the
translation being opposed by one
or more of the reaction forces).
3
Lines of action intersect
in common point?
Yes but a moment
reaction is also present
and it prevents rotation.
Conclusion: can't
rotate.
4
Because the body is neither
partially constrained nor
improperly constrained (can
neither translate nor rotate), it is
properly constrained Ans.
All motion is prevented.
5

624
5.1 Constraints and Static Determinacy Example 1, page 7 of 9
200 lb
R

By
R

Bx
A
B C
R

A
A
B
c)
Two or fewer reactions?
No. Conclusion: not
partially constrained.
2
Show the reactions. 1

625
5.1 Constraints and Static Determinacy Example 1, page 8 of 9
B
A
R

A
R

Bx
R

By
R

By
R

Bx
R

A
A
B
All reactions parallel?
No. Conclusion: can't translate
(There is no direction in which the
body could translate without the
translation being opposed by one
or more of the reaction forces).
3
Lines of action intersect
in common point?
No. Conclusion: can't
rotate (For example,
R
A
prevents rotation
about point B).
4
Because the body is neither partially
constrained nor improperly
constrained (can neither translate
nor rotate), it is
properly constrained Ans.
All motion is prevented.
5

626
5.1 Constraints and Static Determinacy Example 1, page 9 of 9
A B
C
R

A
A B
2 kN m
R

C
C
C B A
d)
Two or fewer reactions?
Yes. Conclusion:
partially constrained Ans.
(Horizontal translation is not prevented.)
2
1 Show the reactions.

627
5.1 Constraints and Static Determinacy Example 2, page 1 of 9
6 kN m
C
B A
3 kN
D
B
C
A
45
A
B
C
300 N
b)
a)
c)
45
2. In each case, determine if the rigid body is partially,
improperly, or properly constrained.
4 m 4 m

628
5.1 Constraints and Static Determinacy Example 2, page 2 of 9
Definitions: a rigid body in two dimensions is said to be
Partially constrained, if it has two or fewer reaction
components (There are not enough reactions to prevent
motion under all possible loading conditions);
Improperly constrained, if it has three or more reaction
components and either a) the reaction forces are all parallel,
or b) the lines of action of the reaction forces intersect at a
common point, and no couple moment is present (There are
enough reactions, but they are not properly arranged to
prevent motion under all possible loading conditions); and
Properly constrained, if it is neither partially nor
improperly constrained (There are enough reactions and they
are properly arranged to prevent motion under all possible
loading conditions):

629
5.1 Constraints and Static Determinacy Example 2, page 3 of 9
A B
C
C A
R

A
R

C
6 kN m
M

C
a)
Two or fewer reactions?
No. Conclusion: not
partially constrained.
2
Show the reactions. 1

630
5.1 Constraints and Static Determinacy Example 2, page 4 of 9
C
A
R

A
R

C
M

C
M

C
R

C
R

A
A
C
All reactions parallel?
No. Conclusion: can't translate
(There is no direction in which the
body could translate without the
translation being opposed by one
or more of the reaction forces).
3
Lines of action intersect in
common point? Yes (at
point A) but a moment
reaction is also present and
it prevents rotation.
Conclusion: can't rotate.
4
Because the body is neither
partially constrained nor improperly
constrained (can neither translate
nor rotate), it is
properly constrained Ans.
All motion is prevented.
5

631
5.1 Constraints and Static Determinacy Example 2, page 5 of 9
b)
A
C
B
D
3 kN
1 Show the reactions.
2 Two or fewer reactions?
No. Conclusion: not
partially constrained.
M

C
R
Dx
R

A
D
R
C
R
Dy
C A

632
5.1 Constraints and Static Determinacy Example 2, page 6 of 9
5 Because the body is neither
partially constrained nor improperly
constrained (can neither translate
nor rotate), it is
properly constrained Ans.
All motion is prevented.
4 Lines of action intersect in common
point? No. Conclusion: can't
rotate.
3 All reactions parallel?
No. Conclusion: can't translate
(There is no direction in which the
body could translate without the
translation being opposed by one
or more of the reaction forces).
R

A
A C
R
C
D
R

A
M

C
M

C
R
Dx
R

A
D
R
C
R
Dy
C A
R
Dx
R
Dy

633
5.1 Constraints and Static Determinacy Example 2, page 7 of 9
A
B C
R

B
R

A
R

C
4 m 4 m
Two or fewer reactions?
No. Conclusion: not
partially constrained.
2
c)
300 N
C
B
A
1 Show the reactions.
45
45
45 45
4 m 4 m

634
5.1 Constraints and Static Determinacy Example 2, page 8 of 9
R

C
R

A
R

B
C
B A
4 m 4 m
All reactions parallel?
No. Conclusion: can't translate
(There is no direction in which the
body could translate without the
translation being opposed by one
or more of the reaction forces).
3
45 45

635
5.1 Constraints and Static Determinacy Example 2, page 9 of 9
A
B C
R

B
R

A
R

C
D
D
C
B
A
4 m 4 m
Lines of action intersect in common point?
Yes. Conclusion:
improperly constrained Ans.
Small (actually infinitesimal) rotation
about D is not prevented. The short links
at A and C would have to change
orientation before they could restrain the
rotation; they could not restrain the
rotation in their original position.
4
45 45
45
45

636
5.1 Constraints and Static Determinacy Example 3, page 1 of 9
D
10 kN
C
B
A
70 lb
D
C
B
A
10 kN
D C
B
A
A B
C D
400 lb
3. In each case, determine if the rectangular plate is
partially, improperly, or properly constrained.
a)
d)
c)
b)

637
5.1 Constraints and Static Determinacy Example 3, page 2 of 9
Definitions: a rigid body in two dimensions is said to be
Partially constrained, if it has two or fewer reaction
components (There are not enough reactions to prevent
motion under all possible loading conditions);
Improperly constrained, if it has three or more reaction
components and either a) the reaction forces are all parallel,
or b) the lines of action of the reaction forces intersect at a
common point, and no couple moment is present (There are
enough reactions, but they are not properly arranged to
prevent motion under all possible loading conditions); and
Properly constrained, if it is neither partially nor
improperly constrained (There are enough reactions and they
are properly arranged to prevent motion under all possible
loading conditions):

638
5.1 Constraints and Static Determinacy Example 3, page 3 of 9
A
B
C
10 kN
C
A
D
B
R

D
R

Ay
R

Ax
D
a)
Two or fewer reactions?
No. Conclusion: not
partially constrained.
2
Show the reactions. But do not show the
applied 10-kN force: the reaction forces, not
the applied forces, determine the constraints of
the body.
1

639
5.1 Constraints and Static Determinacy Example 3, page 4 of 9
R

Ax
R

Ay
R

D
B
D
A
C
C
A
D
B
R

D
R

Ay
R

Ax
All reactions parallel?
No. Conclusion: can't
translate (There is no
direction in which the
body could translate
without the translation
being opposed by one
or more of the reaction
forces).
3
Lines of action intersect
in common point?
No. Conclusion: can't
rotate (For example,
R
D
prevents rotation
about A).
4
Because the body is neither
partially constrained nor
improperly constrained (can
neither translate nor rotate), it is
properly constrained Ans.
All motion is prevented.
5

640
5.1 Constraints and Static Determinacy Example 3, page 5 of 9
b)
A
B
C
D
70 lb
D
C
B A
F

A
F

D
D
C
B
A
Two or fewer reactions?
Yes. Conclusion:
partially constrained Ans.
Small (actually infinitesimal) rotation about B is not
prevented.
2
1 Show the reactions.

641
5.1 Constraints and Static Determinacy Example 3, page 6 of 9
A
B
C D
10 kN
C D
A
B
R

D
R

B
R

A
c)
Two or fewer reactions?
No. Conclusion: not
partially constrained.
2
Show the reactions. 1

642
5.1 Constraints and Static Determinacy Example 3, page 7 of 9
R

A
R

B
R

D
B
A
D C
C D
A
B
R

D
R

B
R

A
All reactions parallel?
No. Conclusion: can't translate
(There is no direction in which the
body could translate without the
translation being opposed by one
or more of the reaction forces).
Lines of action intersect
in common point?
No. Conclusion: can't
rotate (For example,
R
D
prevents rotation
about A).
Because the body is neither
partially constrained nor
improperly constrained (can
neither translate nor rotate), it is
properly constrained Ans.
All motion is prevented.
5
4
3

643
5.1 Constraints and Static Determinacy Example 3, page 8 of 9
A B
C D
400 lb
C D
A B
R

D
R

B
R

A
R

C
d)
Two or fewer reactions?
No. Conclusion: not
partially constrained.
2
Show the reactions. 1

644
5.1 Constraints and Static Determinacy Example 3, page 9 of 9
R

C
R

A
R

B
R

D
B A
D C
C D
A B
R

D
R

B
R

A
R

C
All reactions parallel?
No. Conclusion: can't translate
(There is no direction in which the
body could translate without the
translation being opposed by one
or more of the reaction forces).
Lines of action intersect
in common point?
No. Conclusion: can't
rotate (For example,
R
D
prevents rotation
about A).
Because the body is neither
partially constrained nor
improperly constrained (can
neither translate nor rotate), it is
properly constrained Ans.
All motion is prevented.
5
4
3

645
5.1 Constraints and Static Determinacy Example 4, page 1 of 8
4 kN
D C
B A
200 N
C
B A
45
45
45
D
900 N
C
B
A
a)
b)
c)
4. In each case, determine if the rectangular plate is
partially, improperly, or properly constrained.

646
5.1 Constraints and Static Determinacy Example 4, page 2 of 8
Definitions: a rigid body in two dimensions is said to be
Partially constrained, if it has two or fewer reaction
components (There are not enough reactions to prevent
motion under all possible loading conditions);
Improperly constrained, if it has three or more reaction
components and either a) the reaction forces are all parallel,
or b) the lines of action of the reaction forces intersect at a
common point, and no couple moment is present (There are
enough reactions, but they are not properly arranged to
prevent motion under all possible loading conditions); and
Properly constrained, if it is neither partially nor
improperly constrained (There are enough reactions and they
are properly arranged to prevent motion under all possible
loading conditions):

647
5.1 Constraints and Static Determinacy Example 4, page 3 of 8
A B
C D
C D
A
B
R

D
R

B
R

A
R

C
4 kN
a)
Two or fewer reactions?
No. Conclusion: not
partially constrained.
2
Show the reactions. 1

648
5.1 Constraints and Static Determinacy Example 4, page 4 of 8
R

C
R

A
R

B
R

D
B
A
D C
D C
B
A
All reactions parallel?
Yes. Conclusion:
improperly constrained Ans.
(Horizontal translation
is not prevented.)
3

649
5.1 Constraints and Static Determinacy Example 4, page 5 of 8
A B
C
C
A B
R

B
R

A
R

C
200 N
R

C
C
R

A R

B
b)
Two or fewer reactions?
No. Conclusion: not
partially constrained.
2
All reactions parallel?
No. Conclusion: can't translate
(There is no direction in which the
body could translate without the
translation being opposed by one
or more of the reaction forces).
3
Show the reactions. 1

650
5.1 Constraints and Static Determinacy Example 4, page 6 of 8
R

C
C
R

A R

B
B A
A
C
B
D
D
Lines of action intersect in common point?
Yes. Conclusion:
improperly constrained Ans.
Small (actually infinitesimal) rotation about
D is not prevented.
4

651
5.1 Constraints and Static Determinacy Example 4, page 7 of 8
C
A
B
D
R

D
R

A
R

B
Two or fewer reactions?
No. Conclusion: not
partially constrained.
2
c)
A
B
C
900 N
D
Show the reactions. 1
45
45
45
45 45
45

652
5.1 Constraints and Static Determinacy Example 4, page 8 of 8
A
B
C
D
C
A
B
D
R

D
R

A
R

B
All reactions parallel?
Yes. Conclusion:
improperly constrained Ans.
Translation in a direction 45 from the
horizontal is not prevented. The short
links at A, B, and D would have to
change orientation before they could
restrain the translation; they could not
restrain the translation in their original
position.
3
45
45
45
45

653
5.1 Constraints and Static Determinacy Example 5, page 1 of 7
B
C
A
200 N
O
45
Semicircles with
common center at O
D
6 N
20 N
A
C
B
a)
b)
5. In each case, determine if the rigid body is
partially, improperly, or properly constrained.

654
5.1 Constraints and Static Determinacy Example 5, page 2 of 7
Definitions: a rigid body in two dimensions is said to be
Partially constrained, if it has two or fewer reaction
components (There are not enough reactions to prevent
motion under all possible loading conditions);
Improperly constrained, if it has three or more reaction
components and either a) the reaction forces are all parallel,
or b) the lines of action of the reaction forces intersect at a
common point, and no couple moment is present (There are
enough reactions, but they are not properly arranged to
prevent motion under all possible loading conditions); and
Properly constrained, if it is neither partially nor
improperly constrained (There are enough reactions and they
are properly arranged to prevent motion under all possible
loading conditions):

655
5.1 Constraints and Static Determinacy Example 5, page 3 of 7
B
C
A
200 N
O
R

C
R

A
O
B
R

B
A C
Two or fewer reactions?
No. Conclusion: not
partially constrained.
2
a)
Semicircles with
common center at O
1 Show the reactions. But do not show the
applied 200-N force: the reaction forces, not
the applied forces, determine the constraints of
the body.
45
45

656
5.1 Constraints and Static Determinacy Example 5, page 4 of 7
3 All reactions parallel?
No. Conclusion: can't translate
(There is no direction in which the
body could translate without the
translation being opposed by one
or more of the reaction forces).
R

C
R

A
O
B
R

B
A C
45

657
5.1 Constraints and Static Determinacy Example 5, page 5 of 7
O
R

C
R

A
O
B
R

B
A C
Lines of action intersect in common point?
Yes. Conclusion:
improperly constrained Ans.
Rotation about O is not prevented.
4
45
45

658
5.1 Constraints and Static Determinacy Example 5, page 6 of 7
B
C
A
20 N
6 N
D
A
C
R

A
R

C
R

D
D
b)
Two or fewer reactions?
No. Conclusion: not
partially constrained.
2
1
Show the reactions.

659
5.1 Constraints and Static Determinacy Example 5, page 7 of 7
R

C
R

A
C
A
D
R

D
R

D
D
A
C
R

A
R

C
All reactions parallel?
No. Conclusion: can't translate
(There is no direction in which
the body could translate
without the translation being
opposed by one or more of the
reaction forces).
Lines of action intersect
in common point?
No. Conclusion: can't
rotate (For example,
RD prevents rotation
about A).
3
4
Because the body is neither
partially constrained nor
improperly constrained (can
neither translate nor rotate), it is
properly constrained Ans.
All motion is prevented.
5

660
5.1 Constraints and Static Determinacy Example 6, page 1 of 7
C B A
10 lb
D
Collar slides freely
on smooth rod
Pin slides freely
in smooth slot
C
50 N
A
B
a)
b)
45
45
45
6. In each case, determine if the rigid body is
partially, improperly, or properly constrained.

661
5.1 Constraints and Static Determinacy Example 6, page 2 of 7
Definitions: a rigid body in two dimensions is said to be
Partially constrained, if it has two or fewer reaction
components (There are not enough reactions to prevent
motion under all possible loading conditions);
Improperly constrained, if it has three or more reaction
components and either a) the reaction forces are all parallel,
or b) the lines of action of the reaction forces intersect at a
common point, and no couple moment is present (There are
enough reactions, but they are not properly arranged to
prevent motion under all possible loading conditions); and
Properly constrained, if it is neither partially nor
improperly constrained (There are enough reactions and they
are properly arranged to prevent motion under all possible
loading conditions):

662
5.1 Constraints and Static Determinacy Example 6, page 3 of 7
D
10 lb
A B C
C B A
D
R

D
R

C
R

B
R

A
a)
Two or fewer reactions?
No. Conclusion: not
partially constrained.
2
Show the reactions. 1

663
5.1 Constraints and Static Determinacy Example 6, page 4 of 7
D
A B
C
C B A
D
R

D
R

C
R

B
R

A
All reactions parallel?
Yes. Conclusion:
improperly constrained Ans.
(Horizontal translation
is not prevented.)
3

664
5.1 Constraints and Static Determinacy Example 6, page 5 of 7
B
A
50 N
C
Pin slides freely
in smooth slot
Collar slides freely
on smooth rod
b)
45
45
45

665
5.1 Constraints and Static Determinacy Example 6, page 6 of 7
C
B
A
R

C
R

B
R

A
R

A
R

B
R

C
A
B
C
Two or fewer reactions?
No. Conclusion: not
partially constrained.
2
All reactions parallel?
No. Conclusion: can't translate
(There is no direction in which the
body could translate without the
translation being opposed by one
or more of the reaction forces).
3
Show the reactions. 1
45
45
45
45
45
45
45
45

666
5.1 Constraints and Static Determinacy Example 6, page 7 of 7
C
B
A
R

C
R

B
R

A
C
A
B
Lines of action intersect in common point?
Yes. Conclusion:
improperly constrained Ans.
Small (actually infinitesimal) rotation
about B is not prevented. The collar at A
and the pin at C would have to move
slightly before they could restrain the
rotation; they could not restrain the
rotation in their original position.
4
45
45
45
45
45
45

667
5.1 Constraints and Static Determinacy Example 7, page 1 of 4
20 kN
3 m
C
B
A
D
3 m
45 45
45 45
7. Determine if the rigid body is partially,
improperly, or properly constrained.

668
5.1 Constraints and Static Determinacy Example 7, page 2 of 5
Definitions: a rigid body in two dimensions is said to be
Partially constrained, if it has two or fewer reaction
components (There are not enough reactions to prevent
motion under all possible loading conditions);
Improperly constrained, if it has three or more reaction
components and either a) the reaction forces are all parallel,
or b) the lines of action of the reaction forces intersect at a
common point, and no couple moment is present (There are
enough reactions, but they are not properly arranged to
prevent motion under all possible loading conditions); and
Properly constrained, if it is neither partially nor
improperly constrained (There are enough reactions and they
are properly arranged to prevent motion under all possible
loading conditions):

669
5.1 Constraints and Static Determinacy Example 7, page 3 of 4
3 m
3 m
R

A
R

B
R

C
R

D
D
A
B
C
3 m
3 m
R

A
R

B
R

C
R

D
D
A
B
C
Two or fewer reactions?
No. Conclusion: not
partially constrained.
2
All reactions parallel?
No. Conclusion: can't translate
(There is no direction in which the
body could translate without the
translation being opposed by one
or more of the reaction forces).
3
Show the reactions. 1
45
45
45
45
45
45
45
45

670
5.1 Constraints and Static Determinacy Example 7, page 4 of 4
3 m
D
A
B
C
3 m
E
E
3 m
3 m
R

A
R

B
R

C
R

D
D
A
B
C
Lines of action intersect in common point?
Yes. Conclusion:
improperly constrained Ans.
Small (actually infinitesimal) rotation about E is not
prevented. The short links at A, B, C, and D would
have to change orientation before they could restrain
the rotation; they could not restrain the rotation in
their original position.
4
45
45
45
45
45
45
45 45

671
5.1 Constraints and Static Determinacy Example 8, page 1 of 7
8. In each case, determine if the rigid body is
statically determinate or indeterminate.
a)
4 ft 3 ft 5 ft
b)
4 ft
A
C
B
4 kip
4 ft 3 ft 5 ft
A
C
B
6 kip
4 ft

672
5.1 Constraints and Static Determinacy Example 8, page 2 of 7
Definitions: a rigid body in equilibrium under a given loading is said to be
Statically determinate, if a unique set of reaction components can
be found by solving the equations of statics alone (No additional
equations such as are introduced in a course in mechanics of
materials are needed); or
Statically indeterminate, if not all reaction components can be
found uniquely by solving the equations of statics alone.

673
5.1 Constraints and Static Determinacy Example 8, page 3 of 7
Draw a free-body diagram. 1 There are four unknown reactions: Ax A
y
, B
y
, and C
y
.
But there are at most three equations of equilibrium
for a body in two dimensions. Because there are
more unknowns than equations, a unique solution
cannot be found; the rigid body is
statically indeterminate. Ans.
Observation No. 1: In situations such as this, it is
tempting to write four equilibrium equations by, for
example, writing two force equations and moment
equations about two different points. But this
approach will not work, as will now be shown.
2
3
4 ft
6 kip
B
C
A
5 ft 3 ft 4 ft
a)
4 ft 3 ft 5 ft
A

y
C

y
B

y
B
C
A
A

x
6 kip
4 ft

674
5.1 Constraints and Static Determinacy Example 8, page 4 of 7
Write four equilibrium equations:
Fx = 0: Ax + 6 kip = 0 (1)
F
y
= 0: A
y
+ B
y
+ C
y
= 0 (2)
M
A
= 0: B
y
(5 ft + 3 ft) + C
y
(5 ft + 3 ft + 4 ft)
(6 kip)(4 ft) = 0 (3)
M
C
= 0: Ax(4 ft) A
y
(5 ft + 3 ft + 4 ft) B
y
(4 ft) = 0 (4)
4
+
+
6 kip
A

x
A
C
B
B

y
C

y
A

y
5 ft 3 ft 4 ft
We will now show that Eq. 4 is not independent of Eqs. 1-3. Multiply Eq. 1 by
4, Eq. 2 by 12, Eq. 3 by 1 and add the results to obtain
4(Ax + 6) 12(Ay + By + Cy) + [By(5 + 3) + Cy(5 + 3 + 4) (6)(4)] = 0
The last equation simplifies to
Ax(4) Ay(12) By(4) + Cy( 12 + 5 + 3 + 4) + [4(6) 6(4)] = 0
which is equivalent to Eq. 4. This is a general result: You can write as many
equilibrium equations as you want, but at most only three of them will be
independent (for two-dimensional problems).
4 ft
5
+
+

675
5.1 Constraints and Static Determinacy Example 8, page 5 of 7
Observation No. 2: The difficulty is not that we can't find a solution. We can find
as many solutions as we want. For example, here are two:
Solution No. 1: Ax = 6 kip, A
y
= 3 kip, B
y
= 3 kip, and C
y
= 0.
Solution No. 2: Ax = 6 kip, A
y
= 2 kip, B
y
= 0, and C
y
= 2 kip.
The difficulty is that we can't find the unique solution, because it is not
determined by the equations of statics alone, which is another way of saying that
the structure is statically indeterminate. To find the unique solution, we would
have to supplement the equations of statics by introducing an equation describing
material behavior the subject of a course in mechanics of materials.
6

676
5.1 Constraints and Static Determinacy Example 8, page 6 of 7
Draw a free-body diagram. 1
4 kip
B
C
A
4 ft
b)
5 ft 3 ft 4 ft
4 ft
A
C B
4 kip
B

y
C

y
A

y
5 ft 3 ft 4 ft
2 There are three unknown reactions, A
y
, B
y
, and C
y
, and
for a body in two dimensions, there are at most three
equations of equilibrium, so it appears that the body may
be statically determinate. However, let's write the
equilibrium equations as a check:
Fx = 0: 0 = 0 (1)
F
y
= 0: A
y
+ B
y
+ C
y
4 kip = 0 (2)
M
A
= 0: B
y
(5 ft + 3 ft) + C
y
(5 ft + 3 ft + 4 ft)
(4 kip)(5 ft) = 0 (3)
Only two of the equations are independent, since Eq. 1 is
0 = 0. Conclusion: we do not have enough equations to
solve for the three reactions, so the rigid body is
statically indeterminate. Ans.
+
+
+

677
5.1 Constraints and Static Determinacy Example 8, page 7 of 7
3 Observation: The reason that the body is
statically indeterminate even though there are
only three unknown reactions is that the
structure is improperly constrained (all
reactions are parallel). In general, an
improperly constrained body (in equilibrium
under a set of loads) is always statically
indeterminate because one of the equilibrium
equations is always trivial; for example, the
sum of forces in the direction perpendicular to
the (parallel) reactions reduces to 0 = 0, or the
sum of the moments about a point where all
the reaction forces intersect reduces to 0 = 0.
A partially constrained body (in equilibrium
under a set of loads), on the other hand, may
or may not be statically determinate.
2 kN
4 kN
Partially constrained and
statically indeterminate
Partially constrained and
statically determinate

678
5.1 Constraints and Static Determinacy Example 9, page 1 of 4
2 m 2 m
2 kN 2 kN
D
A C B
2 m
b)
1 m
2 m
4 m
a)
400 N
D
C
B
A
9. In each case, determine if the rigid body
is statically determinate or indeterminate.

679
5.1 Constraints and Static Determinacy Example 9, page 2 of 4
Definitions: a rigid body in equilibrium under a given loading is said to be
Statically determinate, if a unique set of reaction components can
be found by solving the equations of statics alone (No additional
equations such as are introduced in a course in mechanics of
materials are needed); or
Statically indeterminate, if not all reaction components can be
found uniquely by solving the equations of statics alone.

680
5.1 Constraints and Static Determinacy Example 9, page 3 of 4
A
B
C
D
400 N a)
Draw a free-body diagram. 1
+
+
There are three unknowns reactions, A
y
, B
y
, and Cx, so the body will
be statically determinate if the equilibrium equations are independent.
Consider the following equilibrium equations:
Fx = 0: Cx + 400 N = 0 (1)
F
y
= 0: A
y
+ B
y
= 0 (2)
M
A
= 0: B
y
(4 m) + Cx(2 m) (400 N)(2 m + 1 m) = 0 (3)
Solving these equations yields the unique solution
A
y
= 100 N, B
y
= 100 m, and Cx = 400 N
Thus the body is
statically determinate. Ans.
C

x
B

y
A

y
2
4 m
2 m
1 m
1 m
2 m
400 N D
B
A
4 m
+

681
5.1 Constraints and Static Determinacy Example 9, page 4 of 4
B C A
D
2 kN 2 kN
2 m 2 m
D
A
C B
D

y
2 kN 2 kN
A

y
A

x
2 m 2 m 2 m
M

A
2 m
b)
2
There are four unknowns reactions, Ax, A
y
, D
y
,
and M
A
, and at most three equations of equilibrium
exist, so the body is
statically indeterminate. Ans.
1 Draw a free-body diagram.

682
5.1 Constraints and Static Determinacy Example 10, page 1 of 6
2 m
3 m 3 m
2 m
a)
Collar slides freely
on smooth rod
E
5 kN
D
A
C
B
3 m 3 m
6 m
b)
D
600 N 400 N
C
B A
40 N
2 m
1
3
6 m
1 m 4 m
2 m
c)
A
B
C
D
E
45
10. In each case, determine if the rigid body
is statically determinate or indeterminate.

683
5.1 Constraints and Static Determinacy Example 10, page 2 of 6
Definitions: a rigid body in equilibrium under a given loading is said to be
Statically determinate, if a unique set of reaction components can
be found by solving the equations of statics alone (No additional
equations such as are introduced in a course in mechanics of
materials are needed); or
Statically indeterminate, if not all reaction components can be
found uniquely by solving the equations of statics alone.

684
5.1 Constraints and Static Determinacy Example 10, page 3 of 6
There are four unknown reactions, Ax, C
y
, D
y
, and F
E
, and
at most three equations of equilibrium exist, so the body is
statically indeterminate. Ans.
2
45
45
B
C
A
D
5 kN
E
Collar slides freely
on smooth rod
a)
2 m
3 m 3 m
2 m
A

x
C

y
F

E
C
2 m
3 m 3 m
2 m
E
5 kN
A
B
Draw a free-body diagram. 1
D
D

y

685
5.1 Constraints and Static Determinacy Example 10, page 4 of 6
A B
C
400 N 600 N
D
b)
6 m
3 m 3 m
+
+
+
There are two unknown reactions, C
y
and D
y
. Writing equilibrium equations gives
Fx = 0: 0 = 0 (1)
F
y
= 0: C
y
600 N 400 N + D
y
= 0 (2)
M
D
= 0: C
y
(3 m + 3m) (600 N)(3 m) = 0 (3)
Solving these equations gives the unique solution C
y
= 300 N and D
y
= 700 N, so
the body is
statically determinate. Ans.
2
Draw a free-body diagram.
3 m 3 m
6 m
600 N 400 N
B A
1
C
C

y
D
D

y
Observation: There are only two
unknown reactions because the
body is partially constrained.
However, the loading happens to
be such that the two nontrivial
equations of equilibrium can be
satisfied. If the loading consisted
of a horizontal load, then Eq. 1
could not be satisfied, and the
body would not be in equilibrium.
3

686
5.1 Constraints and Static Determinacy Example 10, page 5 of 6
40 N
2 m
1
3
6 m
1 m 4 m
2 m
c)
A
B
C
C
B
A
2 m
4 m 1 m
6 m
3
1
2 m
40 N
Draw a free-body diagram. 1
2 There are three unknown reactions, F
A
, F
B
, and F
C
.
Writing equilibrium equations gives
Fx = 0: F
A
cos F
C
+ (40 N) cos = 0 (1)
F
y
= 0: F
A
sin F
B
+ (40 N) sin = 0 (2)
M
A
= 0: F
B
(4 m) F
C
(2 m)
+ (40 N) sin (4 m + 2 m)
(40 N) cos (6 m 2 m) = 0 (3)
+
+
F
A
F
B
F
C
D D
3 Geometry
1
3
1
2
D
A
E
E
E
5
10
cos = 2/
sin = 1/
cos = 1/
sin = 3/
10
10
5
5
+

687
5.1 Constraints and Static Determinacy Example 10, page 6 of 6
C
B
A
2 m
4 m 1 m
6 m
3
1
40 N
4 If a calculator is used to invert the matrix of
coefficients in the equations, the calculator will
return an error message. That is, the equations
are redundant and do not have a unique
solution. Thus the body is
statically indeterminate. Ans.
The source of the redundancy can be seen by
extending the lines of action of all the forces
and observing that they intersect in a common
point. Thus summing moments about this
common point would produce the trivial
equilibrium equation, 0 = 0. Hence this body
has only two independent equations of
equilibrium.
F
A
F
B
F
C
D
2 m
E
2
1

688

5.2 Rigid Bodies and Two-Dimensional Force Systems
689
5.2 Rigid Bodies and Two-Dimensional Force Systems Procedures and Strategies, page 1 of 1
Procedures and Strategies for Solving Problems Involving
Equilibrium of a Rigid Body in a Two-Dimensional Force System
1. Draw a free-body diagram by showing all external forces:
a) applied loads and couple moments
b) weight of the body if the weight or mass is given
c) reactions from the supports. If a constraint prevents motion in a certain
direction, then show a reaction force acting in that direction on the
free-body diagram. If a constraint prevents rotation about a certain axis,
then show a reaction couple-moment on the free-body diagram.

2. Introduce sufficient dimensions on the diagram to allow for calculating the
moments of forces.
3. Write three equations of equilibrium.
Notes:
a) In writing moment equilibrium equations, remember that a couple moment
can be considered to act anywhere on the free-body diagram.
b) Do not show internal forces on a free-body diagram because they occur in
equal and opposite sense and thus will cancel out.
c) At most, three equations of equilibrium are independent. If you find that
you have more unknowns than independent equations, then look for an error
in your free-body diagram you may have made a mistake in representing
the reaction forces from the supports. Do not write an additional equilibrium
equation by, for example, summing moments about a new point. Such an
equation will not be independent of the equilibrium equations that you have
already written.
A
B
P
Ax
Bx
A
y
M
A
Ax
A
y
P
A
P
P

690
5.2 Rigid Bodies and Two-Dimensional Force Systems Problem Statement for Example 1
40 lb
1. Determine the reactions at A and B. The
weight of the rod is negligible.
3 ft
A
3 ft B
30
30

691
5.2 Rigid Bodies and Two-Dimensional Force Systems Problem Statement for Example 2
A
B
3 kip
4 ft 4 ft
2. Determine the reactions at A and B. The
weight of the rod is negligible.
5 ft

692
5.2 Rigid Bodies and Two-Dimensional Force Systems Problem Statement for Example 3
3. Determine the reactions at supports A and C.
30
4 ft
5 ft
B
400 lb
A
C

693
5.2 Rigid Bodies and Two-Dimensional Force Systems Problem Statement for Example 4
4. The automobile shown is being pushed up the incline at
a constant velocity by a force, P, from a tow truck. The
2,600-lb weight of the car acts at the center of gravity, O,
and the friction forces acting on the wheels are negligible.
Determine the force P and the forces from the ground
acting on the individual tires.
55 in.
O
12 in.
20 in.
A
B
P
15 in.
10

694
5.2 Rigid Bodies and Two-Dimensional Force Systems Problem Statement for Example 5
4 kN
D
A
B
C
E
5 kNm
20
3 m
2 m
5. Plate ABCD is supported by cord EC and a pin at A.
Determine the tension in the cord and the horizontal and
vertical reactions at A. The weight of the plate is negligible.

695
5.2 Rigid Bodies and Two-Dimensional Force Systems Problem Statement for Example 6
O
A
B
C
60 lb
3

f
t
45
6. Member ABC has the shape of a quarter circle centered at O.
Determine the reactions at each of the roller supports.
45

696
5.2 Rigid Bodies and Two-Dimensional Force Systems Problem Statement for Example 7
7. The end of pole DA is buried deep in the ground. The
tension in cable BE is 2 kip and in cable CF is 3 kip.
Determine the reactions at A.
6 kip
C
B
A E F
D
2 ft
5 ft
2 ft
8 ft 12 ft

697
5.2 Rigid Bodies and Two-Dimensional Force Systems Problem Statement for Example 8
8. A smooth slot has been cut in the plate shown, and
a pin, C, fixed to a support behind the plate, fits in
the slot. Determine the forces acting on the plate at
the supports A and B and from pin C.
2 lb
B
C
4 lbft
D
A
15
3 ft 2 ft
1.5 ft
1.5 ft

698
5.2 Rigid Bodies and Two-Dimensional Force Systems Problem Statement for Example 9
100 mm
A
E
C
D
B
12 Nm
9. Pin C is rigidly attached to the plate and can slide freely
in the slot cut in member DB. Determine the reaction
force at the pin support at A and the force transmitted at C.
425 mm
30

699
5.2 Rigid Bodies and Two-Dimensional Force Systems Problem Statement for Example 10
2 m 3 m 2 m
3 kNm
A B D
C
10. Member ABD is supported by a pin at A and a cord attached at
B and D. The cord passes over a frictionless pulley at C.
Determine the reaction at A and the tension in the cord.
5 kN
40
60

700
5.2 Rigid Bodies and Two-Dimensional Force Systems Problem Statement for Example 11
B
A
25
11. The rigid bar AB is supported by two rollers attached to its
ends at A and B. If the bar is in equilibrium in the position
shown, determine the inclination, , of the inclined plane.
20 lb
C
3 in.
7 in.

701
5.2 Rigid Bodies and Two-Dimensional Force Systems Problem Statement for Example 12
A
C
B
D
12. The uniform square plate ABC of mass 10 kg is supported
by a vertical cord at B. Determine the tension in the cord and
the forces from the walls if the walls are smooth at the contact
points A and C.
20

702
5.2 Rigid Bodies in Two-Dimensional Force Systems Example 1, page 1 of 1
The force components at A can be assumed to act in any
direction we choose. Choosing them in the direction of the
inclined coordinate system simplifies the calculation a little.
40 lb
B

y
A

x
2
A

y
y
3 ft
3 ft
40 lb
Free-body diagram of rod AB.
1. Determine the reactions at A and
B. The weight of the rod is
negligible.
1
3 ft
x
3
A
4
Solving these equations simultaneously gives
A

x = 0 Ans.
A
y
= 20.0 lb Ans.
B
y
= 20.0 lb Ans.
+
5
+
+
Equilibrium equations for rod AB:
F

x = 0: A

x = 0
F
y
= 0: A

y
40 lb + B

y
= 0
M
A
= 0: 40 lb)(3 ft) + B

y
(3 ft + 3 ft) = 0
The force from the roller must be perpendicular
to the plane upon which the roller rests.
3 ft B
30
30
A
B

703
5.2 Rigid Bodies in Two-Dimensional Force Systems Example 2, page 1 of 1
A
B
3 kip
Free-body diagram of member AB 1
4 ft 4 ft
2. Determine the reactions at A and B. The
weight of the rod is negligible.
4 ft 4 ft
3 kip
B

y
5 ft
B

x
The force from the roller at A
must be perpendicular to the plane
upon which the roller rests.
2
Equilibrium equations for member AB:
Fx = 0: A

x + B

x = 0
F
y
= 0: 3 kip + B

y
= 0
M
B
= 0: A

x(5 ft) + (3 kip)(4 ft) = 0
4
+
+
+
The pin support at B
provides two force
components.
3
Solving these equations simultaneously gives
A

x = 2.4 kip Ans.
B

x = 2.4 kip Ans.
B

y = 3.0 kip Ans.
5
A
Ax
B
5 ft

704
5.2 Rigid Bodies in Two-Dimensional Force Systems Example 3, page 1 of 3
3. Determine the reactions at supports A and C.
1 Free-body diagram of member ABC
2 The force from the roller
is perpendicular to the
inclined plane. The
angle will be
determined later.
3 The pin support provides two
components of reaction.
B
400 lb
A
C
F

A
C

y
C

x
30
4 ft
5 ft
B
400 lb
A
C

705
5.2 Rigid Bodies in Two-Dimensional Force Systems Example 3, page 2 of 3
Equilibrium equations for member ABC:
Free-body diagram showing horizontal and vertical components
of the unknown reaction force F

A
.
F

A
sin
5
4
F

A
cos
A
C

y
C
C

x
F

x = 0: F

A
cos + C

x = 0 (1)
F

y
= 0: F

A
sin + C

y
400 lb = 0 (2)
M
C
= 0: F

A
cos (4 ft) + (400 lb)(5 ft)
F

A
sin 5 ft 4 ft)/tan 30] = 0 (3)
+
+
+
30
400 lb
B
5 ft (4 ft)/tan 30
4 ft

706
5.2 Rigid Bodies in Two-Dimensional Force Systems Example 3, page 3 of 3
C

y
= 260 lb
C
|C

x| = 81.1 lb
B
400 lb
60
A
F

A
= 162.2 lb
60
30
30
Substituting = 60 in Equations 1, 2, and 3
and solving gives
F

A = 162.2 lb Ans.
C

x = 81.1 lb Ans.
C

y = 260 lb Ans.
We had assumed that C

x pointed to the right, but
solving the equilibrium equations gave a negative
value for C

x, so our assumption was wrong. C

x
points to left as shown.
Free-body diagram showing forces
with correct senses
9
8
Geometry
7
6
B
A
= 90 30 =

707
5.2 Rigid Bodies in Two-Dimensional Force Systems Example 4, page 1 of 2
4. The automobile shown is being pushed up the incline at
a constant velocity by a force, P, from a tow truck. The
2,600-lb weight of the car acts at the center of gravity, O,
and the friction forces acting on the wheels are negligible.
Determine the force P and the forces from the ground
acting on the individual tires.
1 Free-body diagram of car
2
It is convenient to use x and y coordinates that
are parallel and perpendicular to the incline,
because the forces from the tow truck and
from the ground acting on the tires are parallel
and perpendicular to the incline.
3 If F

B
is the force acting on each front
wheel, then 2F

B
is the resultant force
acting on the body.
55 in.
O
12 in.
20 in.
A
B
O
A
B
y
x
2

F

A
2

F

B
2,600 lb
P
P
15 in.
10

708
5.2 Rigid Bodies in Two-Dimensional Force Systems Example 4, page 2 of 2
Solving simultaneously, with = 80, gives
P = 451 lb Ans.
F

A = 300 lb Ans.
F

B = 980 lb Ans.
= 90 10 = 80
Geometry 5
P (2,600 lb) cos = 0
2F

A
+ 2F

B
(2,600 lb) sin = 0
P(8 in.) 2F

A
(55 in.) + 2F

B
(15 in.) = 0
Equilibrium equations for the car:
F

x = 0:
F

y
= 0:
M

O
= 0:
+
+
+
6
10
20 in 12 in = 8 in
15 in.
P
55 in.
2,600 lb
2

F

B
2

F

A
y
B
A
O
Free-body diagram of car, with dimensions shown 4
x
10
20 in.
12 in.

709
5.2 Rigid Bodies in Two-Dimensional Force Systems Example 5, page 1 of 3
5. Plate ABCD is supported by cord EC and a pin at A.
Determine the tension in the cord and the horizontal and
vertical reactions at A. The weight of the plate is negligible.
4 kN
D
A
B
C
E
5 kNm
20
3 m
2 m

710
5.2 Rigid Bodies in Two-Dimensional Force Systems Example 5, page 2 of 3
B
4 kN
D
C
A

y
A

x
T
A
5 kNm
4
The 5 kNm couple-moment at corner B can be
considered to act anywhere; in particular, then, we can
consider it to act at point A, about which we are
summing moments. So we just insert "5 kNm" into the
equation. The minus sign signifies that the 5 kNm
couple-moment has a sense opposite to what we have
chosen to call a positive sense.
20
3 m
2 m
1 Free-body diagram of plate ABCD
2 The tension T points away from the
plate and is collinear with the cord.
3 Equilibrium equations for plate ABCD
F

x = 0:
F
y
= 0:
M
A
= 0:
+
+
+
A

x + 4 kN T cos 20 = 0
A

y
+ T sin 20 = 0
4 kN)(2 m) + (T cos 20)(2 m) + (T sin 20)(3 m) 5 kNm = 0

711
5.2 Rigid Bodies in Two-Dimensional Force Systems Example 5, page 3 of 3
Initially we arbitrarily assumed A

y
points upward.
Solving the equilibrium equations gave A

y
= 1.53 kN.
The minus sign shows that our assumption was wrong,
and the correct sense of A
y
is down, as shown on this
free body.
7
C
A
A

y
= 1.53 kN
A

x = 0.20 kN
B
5 kN m
Free-body diagram of plate ABCD showing
correct senses of forces.
4 kN
D
6
T = 4.47 kN
Solving these equations simultaneously gives
T = 4.47 kN Ans.
A

x = 0.20 kN Ans.
A

y = 1.53 kN Ans.
5
20

712
5.2 Rigid Bodies in Two-Dimensional Force Systems Example 6, page 1 of 2
O
A
B
C
60 lb
3

f
t
45
6. Member ABC has the shape of a quarter circle centered at O.
Determine the reactions at each of the roller supports.
45

713
5.2 Rigid Bodies in Two-Dimensional Force Systems Example 6, page 2 of 2
Free-body diagram of member ABC
The force from the
roller must be
perpendicular to the
plane upon which
the roller rests. This
is true at all three
points, A, B, and C.
F

x = 0:
F

y
= 0:
M

O
= 0:
60 lb F

B
sin 45 = 0
F

A
+ F

B
cos 45 F

C
= 0
(60 lb)(3 ft) F

C
(3 ft) = 0
Equilibrium equations for plate member ABC
1
2 3
Solving these equations simultaneously gives
F
A
= 0 Ans.
F
B
= 84.9 lb Ans.
F
C
= 60.0 lb Ans.
4
60 lb
O
F

A
F

B
F

C
A
B
C
45
3 ft
The lines of action of the forces F
A

and F
B
go through point O, so F
A

and F
B
have no moment about O.
+
+
+

714
5.2 Rigid Bodies in Two-Dimensional Force Systems Example 7, page 1 of 3
6 kip
C
B
A E F
D
2 ft
5 ft
2 ft
8 ft 12 ft
7. The end of pole DA is buried deep in the ground. The
tension in cable BE is 2 kip and in cable CF is 3 kip.
Determine the reactions at A.

715
5.2 Rigid Bodies in Two-Dimensional Force Systems Example 7, page 2 of 3
D
6 kip
C
B
T

CF
= 3 kip
T

BE
= 2 kip
M
A

x
A

y
1
2
3
4
5
C
B
A
E
F
( )
= tan
-1
= tan
-1

( )
= 58.00
= 59.74
2 ft
5 ft
2 ft
2 ft
5 ft
8 ft
Free-body diagram of pole ABCD
The tensions
point away from
the pole and are
collinear with the
cables.
Because the end of the pole is buried
in the ground, not only do two force
components A

x and A

y
act at the end
of the pole, but also a couple moment
M acts to prevent the pole from
rotating.
Equilibrium equations for pole ABCD
Geometry
F

x = 0:
F

y
= 0:
M

A
= 0:
+
+
+
A

x (2 kip) sin + (3 kip) sin + 6 kip = 0
A

y
(2 kip) cos (3 kip) cos = 0
M + (2 kip)(sin )(5 ft) (3 kip)(sin (5 ft + 2 ft)
(6 kip)(5 ft + 2 ft + 2 ft) = 0
12 ft
A
8 ft
5 ft
5 ft + 2 ft
12 ft

716
5.2 Rigid Bodies in Two-Dimensional Force Systems Example 7, page 3 of 3
Initially we assumed that A

x acted to the
right. Solving the equilibrium equations
gave a negative value for A

x, so the
correct sense of A

x must be to the left as
shown here.
Free-body diagram showing the
correct sense of the forces.
Solving these equations, with = 58.00 and = 59.74, gives
A

x = 6.9 kip Ans.
A

y = 2.6 kip Ans.
M = 63.7 kipft Ans.
6
6 kip
3 kip
2 kip
B
C
D
A

y
= 2.6 kip
A

x = 6.9 kip
M = 63.7 kipft
7
8

717
5.2 Rigid Bodies in Two-Dimensional Force Systems Example 8, page 1 of 3
8. A smooth slot has been cut in the plate shown, and
a pin, C, fixed to a support behind the plate, fits in
the slot. Determine the forces acting on the plate at
the supports A and B and from pin C.
2 lb
B
C
4 lbft
D
A
15
3 ft 2 ft
1.5 ft
1.5 ft

718
5.2 Rigid Bodies in Two-Dimensional Force Systems Example 8, page 2 of 3
The 4 lbft couple moment at corner D can be
considered to act anywhere; in particular, then, we
can consider it to act at point A, about which we are
summing moments. So we just insert "4 lbft" into
the equation.
5
4 lbft
A
C
B
D
2 lb
F

B N

C

F

A
Free-body diagram of plate ABD 1
Because the slot is smooth, no
friction force acts on it from the pin.
The only force from the pin must be
the normal force, N
C
. That is, the
force is perpendicular to the slot.
Here its direction has been arbitrarily
assumed to be up and to the right.
2
The force from the roller supports at B and A
are perpendicular to the surface upon which
the roller rests.
3
2 lb F

B
+ N

C
cos = 0
F

A
+ N

C
sin = 0
4 lbft (N

C
cos )(1.5 ft ) + (N

C
sin (3 ft)
+ F

B
(1.5 ft + 1.5 ft) = 0
Equilibrium equations for plate ABD:
F

x = 0:
F

y
= 0:
M

A
= 0: +
+
4
+
3 ft 2 ft
1.5 ft
1.5 ft

719
5.2 Rigid Bodies in Two-Dimensional Force Systems Example 8, page 3 of 3
= 90 15 = 75
A
F

A
= 2.94 lb
2 lb
F

B
= 1.212 lb
B
N

C
= 3.04 lb
C
4 lbft
D
Initially we arbitrarily assumed N

C
points upward
and to the right. Solving the equilibrium
equations gave N

C
= 3.04 lb. The minus sign
shows that our assumption was wrong, and the
correct direction of N

C
is down and to the left as
shown on this free body.
9
C 15
Free-body diagram of plate ABD showing correct senses of forces
Substituting the value = 75 into the equilibrium
equations and solving simultaneously, gives
F

A = 2.94 lb Ans.
F

B = 1.212 lb Ans.
N

C = 3.04 lb Ans.
Geometry
7
8
6
15

720
5.2 Rigid Bodies in Two-Dimensional Force Systems Example 9, page 1 of 3
100 mm
A
E
C
D
B
12 Nm
9. Pin C is rigidly attached to the plate and can slide freely
in the slot cut in member DB. Determine the reaction
force at the pin support at A and the force transmitted at C.
425 mm
30

721
5.2 Rigid Bodies in Two-Dimensional Force Systems Example 9, page 2 of 3
Free-body diagram of plate ACE
Because the slot is smooth, no friction force acts on
it from the pin. The only force from the pin must
be the normal force, that is, the force is
perpendicular to the slot. Here its direction has
been arbitrarily assumed to be up and to the left.
Equilibrium equations for plate ACE
100 mm
E
12 Nm
C
N

C
A

x
A

y
1
2
3
F

x = 0:
F

y
= 0:
M

A
= 0:
+
+
+
A

x N

C
sin = 0
A

y
N

C
cos = 0
12 Nm (N

C
sin (0.425 m) + (N

C
cos )(0.1 m) = 0
D
30
425 mm

722
5.2 Rigid Bodies in Two-Dimensional Force Systems Example 9, page 3 of 3
6 Free-body diagram of plate ACE showing correct
senses of forces
N
C
= 40.1 N
C
A

x = 20.1 N
A

y
= 34.7 N
D
E
12 Nm
The sense of the force
vector has been reversed
because solving the
equilibrium equations
gives a minus sign.
7
30
30
A
Substituting = 30 into the equilibrium equations and
solving simultaneously gives
A

x = 20.1 N Ans.
A

y
= 34.7 N Ans.
N
C
= 40.1 N Ans.
Geometry
5
4
C
D
= 90 60 = 30
90 30 = 60

723
5.2 Rigid Bodies in Two-Dimensional Force Systems Example 10, page 1 of 2
2 m 3 m 2 m
3 kNm
A B D
C
10. Member ABD is supported by a pin at A and a cord attached at
B and D. The cord passes over a frictionless pulley at C.
Determine the reaction at A and the tension in the cord.
Moment equilibrium for pulley C: The moment about the
center produced by T

1
must equal the moment produced by
T

2
. That is, T

1
r = T

2
r. Canceling the r's gives T

1
= T

2
. In
fact, it's always true that for a frictionless pulley, the
tensions on the two sides of the pulley must be equal.
1
C
Radius r
T

1
T

2
5 kN
40
60

724
5.2 Rigid Bodies in Two-Dimensional Force Systems Example 10, page 2 of 2
Free-body diagram of beam ABD Because the tensions on the two sides
of pulley C are equal, the same tension
T acts on point B as on point D.
2 3
3 m
D B
3 kNm
A
T
T
40
60
A

y
A

x
5 kN
4 m
Solving these equations simultaneously gives
A

x = 1.269 kN Ans.
A

y = 2.20 kN Ans.
T = 4.77 kN Ans.
5
Equilibrium equations for beam ABD 4
+
+
F

x = 0:
F

y
= 0:
M

A
= 0: +
A

x + T cos 60 T cos 40 = 0
A

y
+ T sin 60 +T sin 40 5 = 0
3 kNm + (T sin 60)(4 m)
+ (T sin 40)(4 m + 3 m)
(5 kN)(4 m + 3 m) = 0

725
5.2 Rigid Bodies in Two-Dimensional Force Systems Example 11, page 1 of 3
25
11. The rigid bar AB is supported by two rollers attached to its
ends at A and B. If the bar is in equilibrium in the position
shown, determine the inclination, , of the inclined plane.
20 lb
C
3 in.
7 in.
B
A

726
5.2 Rigid Bodies in Two-Dimensional Force Systems Example 11, page 2 of 3
Free-body diagram of bar AB
The force acting on roller A is perpendicular
to the wall; the same is true for roller B.
Equilibrium equations for rod AB:
20 lb
25
F

B
1
2
3
F

A
F

x = 0:
F

y
= 0:
M

B
= 0:
+
+
+
F

B
cos + F

A
cos 20 lb = 0 (1)
F

B
sin + F

A
sin = 0 (2)
(F

A
cos )(3 in. + 7 in.) (20 lb)(7 in.) = 0 (3)
C
A
B
3 in.
7 in.

727
5.2 Rigid Bodies in Two-Dimensional Force Systems Example 11, page 3 of 3
90 25 = 65
= 90 65 = 25
90
= 90 (90 )
=
F

B
sin (15.447 lb) sin 25
F

B
cos 20 lb (15.447 lb) cos 25
=
tan
Solving for gives
= 47.4 Ans.
Substituting = 25 in Eq. 3 and solving gives F

A
= 15.447 lb.
Substituting = 25, = , and F

A
= 15.447 lb in Eqs. 1 and 2 gives
F

B
cos + (15.447 lb) cos 25 = 20 lb (4)

F

B
sin + (15.447 lb) sin 25 = 0 (5)
Eqs. 4 and 5 can be solved with a calculator that can handle two
simultaneous nonlinear equations.
Alternatively, rewrite Eqs. 4 and 5 so that the constant terms are
on the right:
F
B
cos = 20 lb (15.447 lb) cos 25 (6)
F
B
sin = (15.447 lb) sin 25 (7)
Now eliminate F

B
by dividing Eq. 7 by Eq. 6:
6
Geometry
5
4
C
25
A
B

728
5.2 Rigid Bodies in Two-Dimensional Force Systems Example 12, page 1 of 3
A
C
B
D
12. The uniform square plate ABC of mass 10 kg is supported
by a vertical cord at B. Determine the tension in the cord and
the forces from the walls if the walls are smooth at the contact
points A and C.
20

729
5.2 Rigid Bodies in Two-Dimensional Force Systems Example 12, page 2 of 3
Free-body diagram of plate ABC
Since the wall is smooth, no friction force acts. Only
the normal force F
C
is present at C. Similarly only
the normal force F
A
is present at A.
Weight = mg = (10 kg)(9.81m/s
2
) = 98.1 N
Equilibrium equations for plate ABC:
F

x = 0:
F

y
= 0:
M

A
= 0:
F

A
F

C
= 0
F
BD
98.1 N = 0
(F
BD
)(d

1
) + (F

C
)(d
3
) (98.1 N)(d
2
) = 0
+
+
+
(1)
(2)
(3)
1
2
3
4
A
B
C
G
F

A
F
C
F
BD
d
1
d
2
d
3
Center of Mass

730
5.2 Rigid Bodies in Two-Dimensional Force Systems Example 12, page 3 of 3
d

1
= L sin 20
d
2
A
B
G
C
45
20
90 20 45 = 25
L cos 45
d
2
= (L cos 45)(cos 25)
C
B
d
3
A
L
2
+ L
2
= L 2
d
3
= (L 2) sin 25
20
45
20
25
L
L
L
L
Geometry - calculation of d
3
Geometry - calculation of d

2
Geometry - calculation of d

1
Substituting these expressions for d
1
, d
2
, and
d
3
into Eq. 3, canceling L, and solving Eqs. 1,
2, and 3 simultaneously gives
F

A
= 49.0 N Ans.
F

C
= 49.0 N Ans.
F
BD
= 98.1 N Ans.
7
8
6
5
B
d
1
A
C

731

5.3 Rigid Bodies and Three-Dimensional Force Systems
732
5.3 Rigid Bodies and Three-Dimensional Force Systems Procedures and Strategies, page 1 of 1
Procedures and Strategies for Solving
Problems Involving Equilibrium of a Rigid
Body in a Three-Dimensional Force System
1. Draw a free-body diagram, paying special
attention to the reactions from the supports. If a
constraint prevents motion in a certain direction,
then show a reaction force acting in that direction
on the free-body diagram. If a constraint prevents
rotation about a certain axis, then show a reaction
couple-moment on the free-body diagram.
2. Write six equations of equilibrium:
Fx = 0 F
y
= 0 Fz = 0
Mx = 0 M
y
= 0 Mz = 0
Notes:
a) If the problem is such that distances and force
components are easy to determine, then
calculate moments in scalar form (M = Fd).
Otherwise use the cross product definition of
moments ( M = r F).
b) In some problems, you can save work by
summing moments about an axis other than a
coordinate axis.
A
B
C
D
E
A
B
C
D
Ax
Bx
E
y
Ez
x
y
z
P
M
y
F
y
Mx
Fx
Journal bearing
Ball and socket
support
D
y
P
E
Short links
Ex

733
5.3 Rigid Bodies and Three-Dimensional Force Systems Problem Statement for Example 1
1. For the rigid frame shown, determine the
reactions at the knife-edge supports A, B, D.
Neglect the weight of the frame.
A
B
C
D
E
y
z
200 mm
150 mm
400 mm
150 mm
600 mm 300 mm
24 N
80 N
x

734
5.3 Rigid Bodies and Three-Dimensional Force Systems Problem Statement for Example 2
y
z
x
10 kg
z
x
O
300 mm
A
E
D
B C
2. A 10-kg block rests on top of a triangular plate of negligible
weight. The plate is supported by vertical wires AD, OB, and EC.
Determine the x and z coordinates of the 10-kg block if the
tensions in wires AD and CE both equal 20 N. Also determine
the tension in wire OB.

400 mm

735
5.3 Rigid Bodies and Three-Dimensional Force Systems Problem Statement for Example 3
18 in.
Radius = 8 in.
3. To hoist the 180-lb load, a vertical force P is applied to the
crank of the windlass shown. The bearing at A exerts forces
normal and parallel to the shaft; the bearing at B exerts only
forces normal to the shaft no axial force. Determine the
magnitude of P and the forces exerted by the bearings when
the crank is in the position shown.

C
60
View as seen from
the right end.
Bearing A
Bearing B
C
P
10 in.
180 lb
24 in. 12 in.
P
B

736
5.3 Rigid Bodies and Three-Dimensional Force Systems Problem Statement for Example 4
G
H
E
O
C D
A
B
z
x
y
20 lb
42 lb
30
30
5 ft
6 ft
6 ft
4. The box is supported by short links at the
corners. Determine the forces in the links.
4 ft

737
5.3 Rigid Bodies and Three-Dimensional Force Systems Problem Statement for Example 5
5 m
4 m
z
x
y
D
F = {20i 40j + 30k} N
B
C
Cord
5. A force F = {20i 40j 30k} N acts at the midpoint D of rod ADB.
End B of the rod rests on a smooth horizontal plate and is attached to
cord BC. End A is attached to a ball-and-socket support. Determine
the force in the cord and the reactions at A and B.

A
3 m

738
5.3 Rigid Bodies and Three-Dimensional Force Systems Problem Statement for Example 6
1 m
2 m
4 m
3 m
2 m
3 m
2 m
C
B
F
E
D
y
x
z
A
50 kg
6. The boom AB is supported by a ball-and-socket at A
and guy wires CF and DE. Determine the components
of the reactions at A and the forces in the guy wires.


739
5.3 Rigid Bodies and Three-Dimensional Force Systems Problem Statement for Example 7
400 mm
250 mm
z
x
y
D
B A
C
30 mm
30 mm
7. The uniform 15-kg lid of the box is supported by a stick CD
and short hinges at A and B. Assuming that hinge B transmits
no axial force and that the line of action of the force from the
stick coincides with the long axis of the stick, determine the
reactions at the hinges and the force transmitted by the stick at C.

50
250 mm

740
5.3 Rigid Bodies and Three-Dimensional Force Systems Problem Statement for Example 8
Cord
O
A
C
B
Cord
60
4 ft
30
8. The uniform bar AB weighs 20-lb, is 4-ft long, and is supported
by two cords and by smooth surfaces at A and B. Determine the
forces in the cords and the reactions at A and B.
z
x
y

741
5.3 Rigid Bodies in Three-Dimensional Force Systems Example 1, page 1 of 5
1. For the rigid frame shown, determine the
reactions at the knife-edge supports A, B, D.
Neglect the weight of the frame.
A
B
C
D
E
y
z
200 mm
150 mm
400 mm
150 mm
600 mm 300 mm
24 N
80 N
x

742
5.3 Rigid Bodies in Three-Dimensional Force Systems Example 1, page 2 of 5
Sum forces
F

y
= 0: F

A
+ F

B
+ F

D
80 24 N = 0 (1)

+
3
Free-body diagram of frame. 1
2 Since all forces are vertical and dimensions are given in
coordinate directions, a scalar rather than a vector
approach is probably best.

A
D
E
y
z
x
24 N
80 N
F
B
F
D
F
A
C
B

743
5.3 Rigid Bodies in Three-Dimensional Force Systems Example 1, page 3 of 5
80 N
F
B
F
D
FA
24 N
y
z
B E
x,D,A
C
150 mm
View from the positive x axis 5
To calculate moments acting about the x axis, consider the view seen by an
observer located on the positive x axis and looking back at the yz plane.
4
M

x = 0: F

B
(150 mm + 150 mm) + 24 N (200 mm)
+ 80 N (150 mm) = 0
Solving gives
F

B
= 56.0 N Ans.
B
Sum moments about the x axis. 6
200 mm
150 mm
D
+
200 mm
x
A
C
F
D E
y
z
150 mm
150 mm
400 mm 600 mm 300 mm
24 N
80 N
F
A
F
B

744
5.3 Rigid Bodies in Three-Dimensional Force Systems Example 1, page 4 of 5
A
C
F
D
E
y
150 mm
150 mm
400 mm 600 mm 300 mm
24 N
80 N
F
A
F
B
D
B
80 N
F
A
24 N FB = 56.0 N
y
z,A
E
C
F
D
9
To calculate moments about the z axis, consider the view seen by an observer
located on the positive z axis and looking back at the xy plane.
7
M

z = 0: 80 (400 mm) + 56 N (400 mm)
+ F

D
(400 mm + 600 mm)
24 N (400 mm + 600 mm
+ 300 mm) = 0
Solving gives
F

D
= 40.8 N Ans.
+
x
z
Sum moments about the z axis.
D
8 View from the positive z axis.
300 mm 600 mm 400 mm
200 mm
x

745
5.3 Rigid Bodies in Three-Dimensional Force Systems Example 1, page 5 of 5
Using F

B
= 56.0 N and F

D
= 40.8 N in Eq. 1 gives

F

A
+ F

B
+ F

D
80 N 60 N = 0 (Eq. 1 repeated)
Solving gives

F

A
= 7.2 N Ans.
10
40.8 N 56.0 N

746
5.3 Rigid Bodies in Three-Dimensional Force Systems Example 2, page 1 of 4
y
z
x
10 kg
z
x
O
300 mm
A
E
D
B C
2. A 10-kg block rests on top of a triangular plate of negligible
weight. The plate is supported by vertical wires AD, OB, and EC.
Determine the x and z coordinates of the 10-kg block if the
tensions in wires AD and CE both equal 20 N. Also determine
the tension in wire OB.

400 mm

747
5.3 Rigid Bodies in Three-Dimensional Force Systems Example 2, page 2 of 4
z
x
O
300 mm
400 mm
A
E
C
20 N
T
OB 20 N
y
Weight = (10 kg)(9.81 m/s
2
)
= 98.10 N
Since all forces are parallel to a single coordinate (y) axis
and dimensions are given parallel to coordinate directions, a
scalar rather vector approach is probably best.

2
1 Free-body diagram of plate and 10-kg block.
3
+
Sum forces
F

y
= 0: T

OB
+ 20 N + 20 N 98.10 N 0

Solving gives
T

OB
= 58.1 N Ans
10 kg
z
x

748
5.3 Rigid Bodies in Three-Dimensional Force Systems Example 2, page 3 of 4
View from the positive x axis 6
98.10 N
z
z
x
300 mm
400 mm
+
z
x
O
300 mm
D
E
y
20 N
T
OB
20 N
20 N
y
98.10 N
z x, E, O
D
5 To calculate moments about
the x axis, consider the view
seen by an observer located
on the positive x axis and
looking back at the yz plane.
Sum moments about the positive x axis.
M

x = 0: (98.10 N)(z) (20 N)(300 mm) = 0
Solving gives
z = 61.2 mm Ans.
7
To calculate x and z, we need to use moment
equations of equilibrium.
4

749
5.3 Rigid Bodies in Three-Dimensional Force Systems Example 2, page 4 of 4
z
x
O
300 mm
D
E
y
20 N
T
OB
20 N
T
OB
98.10 N
x
z, D, O
98.10 N
x
20 N
20 N 8 To calculate moments about the z axis,
consider the view seen by an observer located
on the positive z axis and looking back at the
xy plane.
View from the positive z axis 9
Sum moments about the z axis.
M

z = 0: 20 N (400 mm) (98.10 N)x = 0
Solving gives
x = 81.6 mm Ans.
10
E
400 mm
z
x
400 mm
+

750
5.3 Rigid Bodies in Three-Dimensional Force Systems Example 3, page 1 of 6
Bearing A
Bearing B
C
P
10 in.
180 lb
24 in. 12 in.
P
B
18 in.
Radius = 8 in.
3. To hoist the 180-lb load, a vertical force P is applied to the
crank of the windlass shown. The bearing at A exerts forces
normal and parallel to the shaft; the bearing at B exerts only
forces normal to the shaft no axial force. Determine the
magnitude of P and the forces exerted by the bearings when
the crank is in the position shown.

C
60
View as seen from
the right end.

751
5.3 Rigid Bodies in Three-Dimensional Force Systems Example 3, page 2 of 6
A
B
C
P
24 in.
No x component of force is
shown because the bearing B
transmits no axial force
+
Sum forces.
F

x = 0: Ax = 0 (1)
F

y
= 0: A

y
+ B

y
180 lb + P 0 2)

F

z = 0: A

z + B

z = 0 (3)
+
1 Free-body diagram of windlass
3
+
2
12 in. 10 in.
180 lb
Az
Ax
Ay
Bz
By
x
y
z

752
5.3 Rigid Bodies in Three-Dimensional Force Systems Example 3, page 3 of 6
P
x,B,A
18 in.
Radius
= 8 in.
(18 in.) cos 60
= 9 in.
z
180 lb
y
Az
Ax
Ay
Bz
By
x
y
z
A B
C
P
24 in.
View from the positive x axis 5
To calculate moments acting
about the x axis, consider the
view seen by an observer
located on the positive x axis
and looking back at the yz
plane.
4
M

x = 0: (180 lb)(8 in.) P(9 in.) = 0
Solving gives
P = 160 lb Ans.
6
60
12 in. 10 in. 24 in.
180 lb
+

753
5.3 Rigid Bodies in Three-Dimensional Force Systems Example 3, page 4 of 6
Az
y, A B
x
Bz
36 in.
Az
Ax
Ay
Bz
By
x
y
z
A B
C
P
View from the positive y axis 8
To calculate the moments acting about the y axis, consider
the view seen by an observer located on the positive y axis
and looking back at the xz plane.
7
M

y
= 0: B

z (36 in.) = 0
So
B

z = 0 Ans.
9 Sum moments about the y axis.
10 in.
180 lb
+
12 in. 24 in.

754
5.3 Rigid Bodies in Three-Dimensional Force Systems Example 3, page 5 of 6
Ay
y
z,A
B
x
180 lb
24 in.
By
P = 160 lb
Az
Ax
Ay
Bz
By
x
y
z
A
B
C
P
24 in.
Sum moments about the z axis.
M

z = 0: (180 lb)(12 in.) + B

y
(12 in. + 24 in.)
(160 lb)(12 in. + 24 in. + 10 in.) = 0

Solving gives

B

y
= 144.4 lb Ans.
12
View from the positive z axis 11
To calculate the moments about the z axis, consider
the view seen by an observer located on the positive
z axis and looking back at the xy plane.
10
10 in. 12 in.
12 in. 10 in.
180 lb
+

755
5.3 Rigid Bodies in Three-Dimensional Force Systems Example 3, page 6 of 6
Substituting P = 160 lb, B

y
= 144.4 lb, and B

z = 0
in Eqs. 1, 2, and 3 and solving gives

A

x = 0 Ans.
A

y
= 164.4 lb Ans.
A

z = 0 Ans.
13

756
5.3 Rigid Bodies in Three-Dimensional Force Systems Example 4, page 1 of 6
G
H
E
O
C D
A
B
z
x
y
20 lb
42 lb
30
30
5 ft
6 ft
6 ft
4. The box is supported by short links at the
corners. Determine the forces in the links.
4 ft

757
5.3 Rigid Bodies in Three-Dimensional Force Systems Example 4, page 2 of 6
G
H
E
O
C D
A
B
z
x
y
20 lb
42 lb
5 ft
6 ft
6 ft
F
A
F
B
F
E
F
G
F
C
F
O
+
F

y
= 0: F

C
sin 30 + F

G
sin 30 F

A
F
O
F

E
0 (2)

F

z = 0: F

B
+ 42 lb = 0 (3)
+
1 Free-body diagram of box
2
+
Sum forces.
F

x = 0: 20 lb F

C
cos 30 F

G
cos 30 = 0 (1)



4 ft
30
30

758
5.3 Rigid Bodies in Three-Dimensional Force Systems Example 4, page 3 of 6
G H
E
O
C D
A
B
z
x
y
20 lb
42 lb
5 ft
6 ft
6 ft
F
A
F
B
F
E
F
G
F
C
F
O
42 lb
D,C
B,A
x,E,O
H,G
F
A
F
C
sin 30
5 ft
y
12 ft
View from the positive x axis 4
To calculate the moments about the x axis, consider the view seen by an
observer located on the positive x axis and looking back at the yz plane.
3
Mx = 0: F
A
(12 ft) F

C
sin 30 (12 ft) + 42 lb (5 ft) = 0 (4)
5 Sum moments about the x axis.
4 ft
30
30
+
z

759
5.3 Rigid Bodies in Three-Dimensional Force Systems Example 4, page 4 of 6
G
H
E
C D
A B
z
x
y
20 lb
42 lb
5 ft
6 ft
6 ft
F
A
F
B
F
E
F
G
F
C
F
O
42 lb
D,B
y,G,O
F
C
cos 30
F
B
C,A
4 ft
z
20 lb
x
6 ft
6 ft
View from the positive y axis 7
To calculate the moments about
the y axis, consider the view seen
by an observer located on the
positive y axis and looking back
at the xz plane.
6
M

y
= 0: F

C
cos 30 (6 ft + 6 ft) F

B
(4 ft)
20 lb (6 ft) 42 lb (4 ft) = 0 (5)
4 ft
8 Sum moments about the y axis.
H,E
O
30
30
+

760
5.3 Rigid Bodies in Three-Dimensional Force Systems Example 4, page 5 of 6
G
H
E
O
C D
A
B
z
x
y
20 lb
42 lb
5 ft
6 ft
6 ft
F
A
F
B
F
E
F
G
F
C
F
O
Sum the moments about the z axis.
Mz = 0: F
G
cos 30 (5 ft) + F
C
cos 30 (5 ft)
20 lb (5 ft) F

E
(4 ft) 0 (6)
9 To calculate moments about the z axis, consider the view
seen by an observer located on the positive z axis and
looking back at the xy plane.
4 ft
10
11
View from the positive z axis
F
G
cos 30
B,E
D,H C,G 20 lb
F
E
5 ft
x
4 ft
z,A,O
y
F
C
cos 30
30
30
+

761
5.3 Rigid Bodies in Three-Dimensional Force Systems Example 4, page 6 of 6
Solving the six equilibrium equations
20 F

C
cos 30 F

G
cos 30 = 0 (1)
F

C
sin 30 + F

G
sin 30 F

A
F
O
F

E
= 0 2

F

B
+ 42 = 0 (3)
(42)(5) (12 F

C
) sin 30 + 12 F

A
= 0 (4)
(12 F

C
) cos 30 4F

B
+ (6)(20) (42)(4) = 0 (5)

(5 F

C
) cos 30 + (5 F

G
) cos 30 (20)(5) 4F

E
= 0 (6)
gives
F
A
= 11.7 lb Ans
F
B
= 42.0 lb Ans.
F
C
= 11.5 lb Ans.
F
E
= 0 Ans.
F
G
= 11.5 lb Ans.
F

O
= 23.3 lb Ans.

12

762
5.3 Rigid Bodies in Three-Dimensional Force Systems Example 5, page 1 of 6
5 m
4 m
z
x
y
D
F = {20i 40j + 30k} N
B
C
Cord
5. A force F = {20i 40j 30k} N acts at the midpoint D of rod ADB.
End B of the rod rests on a smooth horizontal plate and is attached to
cord BC. End A is attached to a ball-and-socket support. Determine
the force in the cord and the reactions at A and B.

A
3 m

763
5.3 Rigid Bodies in Three-Dimensional Force Systems Example 5, page 2 of 6
5 m
4 m
O
z
x
A
y
D
F = {20i 40j + 30k} N
B
Bj
Tk
A
y
j
Azk
Axi
3 m
Normal force from plate 4
Tension in cord 3
1 Free-body diagram of rod
5 Sum forces.
F = 0: A

xi + A

y
j + A

zk + Bj
+ {20i 40j + 30k}N Tk = 0
Collecting the coefficients of i, j, and k gives
(A

x + 20)i + (A

y
+ B 40)j + (A

z + 30 T)k = 0
Equating coefficients to zero gives
F

x = 0: A

x + 20 = 0 (1)
F

y
= 0: A

y
+ B 40 = 0 (2)
F

z = 0: A

z + 30 T = 0 (3)
6
2 Ax, A
y
, and Az are the forces from the
ball-and-socket joint at A. No moment is
transmitted at a ball-and-socket joint.

764
5.3 Rigid Bodies in Three-Dimensional Force Systems Example 5, page 3 of 6
7
8
Next, calculate moments. To do so,
first introduce position vectors from A
to B and from A to D.
3 m
Axi
Azk
A
y
j
Tk
Bj
B
F = {20i 40j + 30k} N
D
y
A
x
z
O
4 m
5 m
{2.5i 1.5j + 2k} m
=

=
2
=
2
r

AB
9 r
AD
= position vector from A to
midpoint of rod
r
AB
= {5i 3j + 4k} m
{5i 3j + 4k}

765
5.3 Rigid Bodies in Three-Dimensional Force Systems Example 5, page 4 of 6
10
3 4 5 4 5 3
+ i j + k
B T 0 T 0 B
= i[( 1.5)(30) ( 40)(2)] j[2.5(30) 20(2)] + k[2.5( 40) 20( 1.5)]
+ i[( 3)( T) B(4)] j[5( T) 0(4)] + k[5(B) 0)( 3)]
1.5 2 2.5 2 2.5 1.5
= i j + k
40 30 20 30 20 40
i j k i j k
0 = 2.5 1.5 2 + 5 3 4
20 40 30 0 B T
Sum moments about point A.
M
A
= 0: r
AD
F + r
AB
{Bj k} = 0

or,
{2.5i 1.5j + 2k} {20i 40j 30k} + { i 3j+ 4k} {Bj k} = 0
Evaluating the cross products gives

766
5.3 Rigid Bodies in Three-Dimensional Force Systems Example 5, page 5 of 6
11 Carrying out the multiplications and collecting coefficients of i,
j, and k gives
0 = (35 + 3T 4B)i + ( 35 + 5T)j + ( 70 + 5B)k
Equating each coefficient to zero gives
M

x = 0: 35 + 3T 4B = 0 (4)

M

y = 0: 35 + 5T = 0 (5)
M

z = 0: 70 + 5B = 0 (6)
Solving Eq. 5 gives
T = 7 N Ans.
and solving either Eq. 4 or Eq. 6 (one of the equations are
redundant) gives
B = 14 N Ans.
Eqs. 1, 2, and 3 are
F

x = 0: A

x + 20 = 0 (Eq. 1 repeated)
F

y = 0: A

y + B 40 = 0 (Eq. 2 repeated)
F

z = 0: A

z + 30 T = 0 (Eq. 3 repeated)
Substituting T = 14 N and B = 8 N and solving Eqs.
1, 2, and 3 gives
Ax = 20 N Ans.
A
y
= 26 N Ans.
Az = 23 N Ans.
12

767
5.3 Rigid Bodies in Three-Dimensional Force Systems Example 5, page 6 of 6
O
z
x
y
D
F
Bj
Tk
A
y
j
Azk
Axi
x'
z'
y'
Why is one of the moment equations, M

x = 0 and M

z = 0, redundant? Because the
lines of action of all forces pass through the axis of the rod. Thus if, instead of x, y,
and z axes, we had chosen x', y', and z' axes as shown above, then since all forces
intersect the x' axis, the moment equation M

x' = 0 would reduce to 0 = 0. That is,
one of the moment equations is not an independent equation, and there are only two
non-trivial moment equations: M

y'
= 0 and M

z' = 0. General conclusion: In three
dimensions, there are at most six independent equations of equilibrium.
13

768
5.3 Rigid Bodies in Three-Dimensional Force Systems Example 6, page 1 of 6
1 m
2 m
4 m
3 m
2 m
3 m
2 m
C
B
F
E
D
y
x
z
A
50 kg
6. The boom AB is supported by a ball-and-socket at A
and guy wires CF and DE. Determine the components
of the reactions at A and the forces in the guy wires.


769
5.3 Rigid Bodies in Three-Dimensional Force Systems Example 6, page 2 of 6
A
z
x
y
E
F
B
F
FC
F
ED
Azk
Axi
A
y
j
4
x, y, and z components of force
at a ball-and-socket joint
2
1 Free-body diagram
3 Sum forces.
F = 0: F

FC
+ F
ED
+ A

xi + A

y
j + A

zk {490.5j} N = 0 (1)

Next we have to express F

FC
and F

ED
in rectangular components.
Cable force supporting 50-kg block
= 50 kg 9.81 m/s
2
= 490.5 N

770
5.3 Rigid Bodies in Three-Dimensional Force Systems Example 6, page 3 of 6
F
ED
5 Introduce position vectors in the same direction as the forces.
r

FC
= ( 4 m 0)i + (2 m 0)j + (0 2 m 2 m)k
= { 4i + 2j 4k}m
r

ED
= (1 m 0)i + (3 m 0)j + (0 2 m)k
= {i + 3j 2k}m
6



= (F

FC
)( )
= F

FC
{ 0.6667i + 0.3333j 0.6667k} (2)


4i + 2j 4k
( 4)
2
+ (2)
2
+ ( 4)
2


( ) = F

FC

Force = (magnitude)(unit vector):
F

FC
r

FC
r

FC
50 kg
A
z
x
y
3 m
1 m
2 m
4 m
D
E
F
B
C
2 m
3 m
2 m
r
FC
r
ED
F
FC

771
5.3 Rigid Bodies in Three-Dimensional Force Systems Example 6, page 4 of 6
7

Substitute Eqs. 2 and 3 into the equilibrium equation, Eq. 1:
F

FC
+ F

ED
+ A

xi + A

y
j + A

zk (490.5)j = 0 (Eq. 1 repeated)

F

ED
{0.2673i + 0.8018j 0.5345k}
F

FC
{ 0.6667i + 0.3333j 0.6667k}
Collecting coefficients of i, then of j, and then of k gives
( 0.6667F

FC
+ 0.2673F

ED
+ A

x)i + (0.3333F

FC
+ 0.8018F

ED

+ A

y
490.5)j + ( 0.6667F

FC
0.5345F

ED
+ A

z)k = 0
Equating each coefficient to zero gives
F

x = 0: 0.6667F

FC
+ 0.2673F

ED
+ A

x

= 0 (4)

F

y
= 0: 0.3333F

FC
+ 0.8018F

ED
+ A

y
490.5 = 0 (5)
F

z = 0: 0.6667F

FC
+ 0.5345F

ED
+ A

z

= 0 (6)

i + 3j 2k
(1)
2
+ (3)
2
+ ( 2)
2


( ) = F

ED

= F

ED
{0.2673i + 0.8018j 0.5345k) (3)







= (F

ED
)( ) F
ED
r

ED
r

ED

772
5.3 Rigid Bodies in Three-Dimensional Force Systems Example 6, page 5 of 6
Ax
Az
A
y
r
AB
r
AF
A
z
x
E
F
B
2 m
2 m
3 m
F
FC
F
ED
r
AE
y


8

To calculate moments, first introduce position vectors
r

AE
= {2k} m
r

AF
= (2m + 2m)k = {4k} m
r

AB
= (2m + 2m + 3m)k = {7k} m


9 Sum moments.
M

A
= 0: r

AE
F

ED
+ r

AF
F

FC
+ r

AB
{ 490.5j} = 0
or,
{2k} F
ED
{0.2673i 0.8018j 0.5345k}
+ {4k} F
FC
{ 0.6667i + 0.3333j 0.6667k}
+ {7k} { 490.5j} = 0
Expanding the cross products gives
2F
ED
(0.2673)k i + 2F
ED
(0.8018)k j + 2F
ED
( 0.5345)k k + 4F
FC
( 0.6667)k i
+ 4F

FC
(0.3333)k j + 4F

FC
( 0.6667)k k + 7( 490.5 k j = 0

= j = i = 0
= j
= i
= 0 = i

773
5.3 Rigid Bodies in Three-Dimensional Force Systems Example 6, page 6 of 6
10 Carrying out the multiplications and collecting coefficients of i and then j gives
( 1.6036F
ED
1.3332F
FC
+ 3433.5)i + (0.5334F
ED
2.6668F
FC
)j = 0
Equating each coefficient to zero gives
M

x = 0: .6036F
ED
1.3332F
FC
+ 3433.5 = 0 (7)
M

y
= 0: 0.5334F
ED
2.6668F
FC
= 0 (8)
Solving Eqs. 7 and 8 simultaneously gives
F
ED
= 1835.841 N = 1.836 kN Ans.
F
FC
= 367.196 N = 0.367 kN Ans.
Using these results in Eqs. 4, 5, and 6 gives
A

x = 245.911 N = 0.246 kN Ans.
A

y
= 1103.863 N = 1.104 kN Ans.
A

z = 736.448 N = 0.736 kN Ans.

774
5.3 Rigid Bodies in Three-Dimensional Force Systems Example 7, page 1 of 8
400 mm
250 mm
z
x
y
D
B A
C
30 mm
30 mm
7. The uniform 15-kg lid of the box is supported by a stick CD
and short hinges at A and B. Assuming that hinge B transmits
no axial force and that the line of action of the force from the
stick coincides with the long axis of the stick, determine the
reactions at the hinges and the force transmitted by the stick at C.

50
250 mm

775
5.3 Rigid Bodies in Three-Dimensional Force Systems Example 7, page 2 of 8
z
x
y
400 mm
A
y
j
Axi
Azk
B
y
j
Bzk
B A
30 mm 30 mm 440 mm
400 mm
E (center
of lid)
200 mm
2
400 mm
=
r
DC
F
C
D
Bx = 0 because we are
told that hinge B
transmits no axial force
Weight = (15 kg)(9.81 m/s
2
) = 147.15 N
r

DC
r
DC
F
C
= F

C

The force F

C
must be expressed in rectangular components. We know
that, in terms of the position vector r

DC
:
6
4
5
Sum forces.
F = 0: A

xi + A

y
j + A

zk + B

y
j + B

zk + F
C
{147.15j} N = 0 (1)


1 Free-body diagram of lid
3 F
C
= force from
stick acting on
lid at point C.
C
2
250 mm
50

776
5.3 Rigid Bodies in Three-Dimensional Force Systems Example 7, page 3 of 8
y
x
z
500 mm
250 mm
D B A
C
400 sin 50 mm
= 306.4 mm
400 mm
y
x, B, A
400 cos 50 mm
= 257.1 mm
C
8 Thus, Eq. 2 gives

F
C
= F

C
{ 250i + 306.4j 142.9k}

( 250)
2
+ (306.4)
2
+ ( 142.9)
2
= F

C

= F

C
{ 0 5946i + 0.7287j 0.3399k} (3)
r

DC
r

DC
r

DC
View of yz plane as seen from positive x axis
View of xy plane as seen from positive z axis
7 The force F

C
must be expressed in rectangular
components. We know that, in terms of the
position vector r

DC
:
Here,
r

DC
= coordinates of C coordinates of D
= {(250 500)i + (306.4 0)j + (257.1 400)k} m
F

C
= F

C
(2)
r
DC
r

DC
z
50
400 mm
D

777
5.3 Rigid Bodies in Three-Dimensional Force Systems Example 7, page 4 of 8
9 This expression for F
C
can now be substituted into Eq. 1:
A

xi + A

y
j + A

zk + B

y
j + B

zk + F
C
{147.15j} = 0 (Eq. 1 repeated)
F
C
{ 0.5946i + 0.7287j 0.3399k
Collecting coefficients of i, then j, and then k gives
(A

x 0.5946 F

C
)i + (A

y
+ B

y
+ 0.7287 F

C
147.15)j
+ (A

z + B

z 0.3399 F

C
)k = 0
Equating each coefficient to zero gives
F

x = 0: A

x 0.5946F
C
= 0 (4)

F

y
= 0: A

y
+ B

y
+ 0.7287F
C
147.15 = 0 (5)
F

z = 0: A

z + B

z 0.3399F
C
= 0 (6)


778
5.3 Rigid Bodies in Three-Dimensional Force Systems Example 7, page 5 of 8
D
=
440 mm
2
220 mm
440 mm
A
B
Bzk
B
y
j
y
x
z
E
r
AE
147.15 N
r
AB
10

Next, moment equations must be written. If we choose point A as the point about which
to sum moments, then the resulting equations will be simplified, since A

x, A

y
, and A

z will
not appear. Thus we need the position vectors from A to the lines of action of the forces.
11 From the figures,

r

AE
= {220i +153.2j 128.6k} mm (7)
r

AB
= {440i} mm (8)
View of the yz plane as seen
from the positive x axis.
C
z
C
400 mm
50
200 sin 50 mm
= 153.2 mm
400 mm
y
x, B, A
200 cos 50 mm
= 128.6 mm
E
2
= 200 mm
Center of lid

779
5.3 Rigid Bodies in Three-Dimensional Force Systems Example 7, page 6 of 8
D
r
AD
400 mm
440 mm 30 mm
A
B
Bzk
B
y
j
y
x
z
E
13

We can use position vector r
AD
from A to D (to
calculate the moment of the force F
C
) because the
line of action of F
C
passes through D.
r

AD
= {(440 + 30)i +400k} mm (9)
F
C
C
Line of action of force F
C
12

780
5.3 Rigid Bodies in Three-Dimensional Force Systems Example 7, page 7 of 8



14 Sum moments about A.
M

A
= r
AE
{ 147.15j}N + r

AB
{B

y
j + Bzk} + r
AD
F

C
Substituting from Eqs. 8, 9, and 10 for r
AE
, r

AB
, and r
AD
and from Eq. 3 for F

C
gives

M

A
= {220i 153.2j 128.6k} { 147.15j} + {440i} {B

y
j

zk}
+ {470i 400k} F
C
{ 0 5946i + 0.7287j 0 k}
Expanding the cross products gives
M

A
= 220( 147.15)i j + 153.2( 147.15)j j + 128.6( 147.15)k j + 440(B

y
)i j + 440(B

z)i k
+ 470( 0.5946F

C
)i i + 470(0.7287F

C
i j + 470( 0.3399F

C
)i k
+ 400( 0.5946F

C
)k i + 400(0.7287F

C
)k j + 400( 0.3399F

C
)k k

= 0 = k
= i
= k = j
= 0 = k = j
= j
= i = 0

781
5.3 Rigid Bodies in Three-Dimensional Force Systems Example 7, page 8 of 8
15 Carrying out the multiplications and then collecting coefficients of i, j, and k gives
M
A
= (18923.490 291.480F
C
)i + ( 440Bz 78.087F
C
)j + ( 32373.000 + 440B
y
+ 342.489F
C
)k
Setting coefficients of i, j, and k to zero gives
18923.490 291.480F
C
= 0 (10)
440B

z 78.087F

C
= 0 (11)
32373.000 + 440B
y
+ 342.489F
C
= 0 (12)
Solving Eqs. 10, 11, and 12 simultaneously gives
F

C
= 64.922 N Ans.
B

y
= 23.041 N Ans.
B

z = 11.522 N Ans.
Substituting these results in Eqs. 4, 5, and 6, and then solving gives
A

x = 38.6 N Ans.
A

y
= 76.8 N Ans.
A

z = 33.6 N Ans.

782
5.3 Rigid Bodies in Three-Dimensional Force Systems Example 8, page 1 of 8
Cord
O
A
C
B
Cord
60
4 ft
30
8. The uniform bar AB weighs 20-lb, is 4-ft long, and is supported
by two cords and by smooth surfaces at A and B. Determine the
forces in the cords and the reactions at A and B.
z
x
y

783
5.3 Rigid Bodies in Three-Dimensional Force Systems Example 8, page 2 of 8
M (midpoint of AB)
N
B
k (normal force only, "smooth" surface so
no friction force no force parallel to the wall)
T
A
N
A
j (normal force only, because surface is smooth)
{ 20j} lb
(weight)
Tension in cord
1 Free-body diagram of AB
2 Sum forces.
F = 0: N

B
k + T
B
i + T
A
+ N

A
j + { 20j} lb = 0 (1)


B
A
O
T
B
i (tension in cord)
x
y
z

784
5.3 Rigid Bodies in Three-Dimensional Force Systems Example 8, page 3 of 8
T
A
r
AO
2
1.7321i k
r
AO
r
AO
T
A
= (T
A
)(unit vector along AO)
= T
A

= T
A
= 0.8660T
A
i 0.5T
A
k
7
Since the x and z components of r
OA
are
now known, r
OA
can be expressed as
r
AO
= { 1.7321i k} ft. (2)
x component of r
AO
= (2 ft) cos 30
= 1.7321 ft
4
z component of r
AO
= (2 ft) sin 30

= 1 ft
5

6
To find the rectangular
components of the force T
A
,
introduce a position vector r
AO

from A to O.
Magnitude r
AO
= 4 ft cos 60
= 2 ft
3
Cord
B
C
A
O
Cord
y
x
z
4 ft
60
30

785
5.3 Rigid Bodies in Three-Dimensional Force Systems Example 8, page 4 of 8

The expression that has just been found for T
A
in terms of
rectangular components can now be substituted into the sum of
forces equation, Eq. 1:
N
B
k + T

B
i + T
A
+ N
A
j 20j = 0 (Eq. 1 repeated)
0.8660T
A
i 0.5T
A
k
Collecting coefficients of i, then j, and then k gives
(T
B
0.8660T
A
)i + ( N
A
20)j + (N

B
0.5T
A
)k = 0
Each coefficient must equal zero so
T

B
0.8660T
A
= 0 (3)
N

A
20 = 0 (4)
N

B
0.5T
A
= 0 (5)
Three equations, four unknowns. An additional equation is needed
and will be obtained by writing a moment equation.





8

786
5.3 Rigid Bodies in Three-Dimensional Force Systems Example 8, page 5 of 8
1
2
1
2
MM' is the line of action of the weight
of the rod. By similar triangles AMM'
and ABO, point M' must lie at the
midpoint of OA (because M lies at the
midpoint of the rod). Thus
r
OM'
= r

OA
=
{
1.7321i + k}

= {0.8660i + 0.5k} ft (8)
12
The position vector from O to A, r
OA
, can be found by
noting that it is just the negative of the position vector from
A to O, r
AO
, and we have already found r
AO
( Eq. 2):
r
OA
= r
AO
Eq. 2

= { 1.7321i k}

= {1.7321i + k}ft (7)
r
OB
= (2ft + 2ft) sin 60j

= {3.4641j} ft (6)
10
11
To write the moment equation, first
introduce position vectors from O
to B, from O to A, and from O to
the line of action of the weight.
9
y
2 ft
2 ft
r
OA
M'
M
60
r
OM'
z
B
x
O
A
r
OB

787
5.3 Rigid Bodies in Three-Dimensional Force Systems Example 8, page 6 of 8
= 0 because r

OA
and T
A
are parallel
Sum moments about the origin O.
M
O
= 0: r

OB
N
B
k + T
B
i) + r
OM'
{ 20j} + (r
OA
T
A
) + r
OA
N
A
j = 0
{3.4641j} N
B
k + T
B
i) + {0.8660i + 0.5k} { 20j} + {1.7321i + k} N
A
j = 0
13


N
A
j
T
B
i
N
B
k
x
r
OA
y
{ 20j} lb
M'
M
r
OM' T
A
z
B
O
A
r
OB

788
5.3 Rigid Bodies in Three-Dimensional Force Systems Example 8, page 7 of 8



Carrying out the cross products gives
3.4641N
B
j k + 3.4641T

B
j i + 0.8660( 20)i j + 0.5( 20)k j + 1.7321N

A
i j + N
A
k j = 0
Carrying out the multiplications and collecting coefficients of i, then j, and then k gives
(3.4641N

B
+ 10 N
A
)i + ( 3.4641T
B
17.3200 + 1.7321N
A
)k = 0
Equating coefficients of i and then of k to zero gives
3.4641N
B
+ 10 N
A
= 0 (9)
3.4641T
B
17.3200 + 1.7321N
A
= 0 (10)
These equations must be solved simultaneously with Eqs. 3, 4, and 5:
T
B
0.8660T
A
= 0 (Eq. 3 repeated)
N
A
20 = 0 (Eq. 4 repeated)
N
B
0.5T
A
= 0 (Eq. 5s repeated)
= i = k = k
= i = k
= i
14

789
5.3 Rigid Bodies in Three-Dimensional Force Systems Example 8, page 8 of 8
Eqs. 9, 10, 3, 4, and 5 constitute five equations in four
unknowns. One equation must be redundant. Solving
these equations gives
N

A
= 20 lb Ans.
N
B
= 2.89 lb Ans.
T
B
= 5 lb Ans.
T
A
= 5.77 lb Ans.
15

790

6. Structural Applications
791

6.1 Frames and Machines
792
6.1 Frames and Machines Procedures and Strategies, page 1 of 3
T
1
T
2
T
2
A
B
C
B
Procedures and Strategies for Solving Problems Involving
Analysis of Frames and Machine
To solve problems involving frictionless pulleys,
1. Identify each rope and assign a unique label
such as T
1
, T
2
, ...., for the tension throughout the
length of the rope. Label the tensions on both
sides of pulleys with the same T
i
, since tension
is the same on both sides of a frictionless
pulley.
2. Draw free-body diagrams and write equilibrium
equations for various pulleys until you get
enough equations to solve for all the unknowns.
In drawing the diagrams, remember that tension
acting on a portion of a rope is always directed
away from the rope.
A
T
1
T
2
T
1
T
1
T
1

793
6.1 Frames and Machines Procedures and Strategies, page 2 of 3
To solve problems involving frames and machines other than pulleys,
1. Look for two-force members. Do not draw free-body diagram of these
members.
2. Select one of the multi-force members that is acted upon by forces you are
to determine that is, forces which were requested in the problem statement
and draw a free-body diagram of this member.
a) On the diagram, show the force from any connected two-force member
as a single unknown magnitude (not as two unknown components) and a
known line of action (passing through the two load points of the two-force
member).
b) Assign the senses of all unknown forces arbitrarily if you guess
wrong, at the end of the solution process the forces will be found to be
negative.
3. Count the number of unknowns. If it exceeds the number of equilibrium
equations available, draw a free-body diagram of another multi-force
member. This time, however, you cannot assign the senses of all the forces
arbitrarily forces that have already been shown on another free-body
diagram must now be shown with the opposite sense on the new diagram.
4. Continue to count the number of unknowns and add free-body diagrams
until you have enough equations to solve for all the unknowns. Remember
that sometimes you can save work by using a free body of the whole frame.

794
6.1 Frames and Machines Procedures and Strategies, page 3 of 3
A
Q
B
C
D
P
Ax
A
y
Bx
B
y
Bx
B
y
C
y
P
Q
5. Write equilibrium equations. Try to find
points for summing moments that will lead to
an equation with a single unknown. A point
where the lines of action of several forces
intersect would be an example.
Note: Special care must be given to situations where
three or more members are connected by a pin. One
approach is to assign the pin arbitrarily to one of the
free bodies. An alternative approach is to draw a
separate free-body diagram of the pin. Drawing a
separate free-body diagram increases the number of
unknowns but will help you understand which forces
act on which bodies.
Free-body diagram of AB with pin included at B
F
BD
(Force from
two-force member
BD acting on pin B)
Free-body diagram of BC without including pin at B
Force F
BD
is not present since
it acts on pin B, but pin B is
not part of this free body
B
B
A
C

795
6.1 Frames and Machines Problem Statement for Example 1
0.25 in.
A
D
B
C
E
18 lb
18 lb
1. A pair of pliers is used to hold a short, smooth
rod in position. Determine the force that the pliers
applies to the rod.
1.5 in. 3 in.

796
6.1 Frames and Machines Problem Statement for Example 2
A B
C
F
E D
20 lb
20 lb
2. Determine the force that the nutcracker
applies to the top and bottom of the nut.
1.5 in. 5 in.
1 in

797
6.1 Frames and Machines Problem Statement for Example 3
3. Determine the support force at A and the
tension in the cord. The pulleys are frictionless,
and their weight can be neglected.
90 kg
A
B
C
D

798
6.1 Frames and Machines Problem Statement for Example 4
400 lb
W
4. Determine the values of the weight W and the
support forces at A and B. The pulleys are frictionless,
and their weight can be neglected.
C
D
A
B

799
6.1 Frames and Machines Problem Statement for Example 5
400 lb
5. Determine the force P and the force in the connecting
bar EF. The pulleys are frictionless and their weight can
be neglected.
A B
E
D
C
F
P

800
6.1 Frames and Machines Problem Statement for Example 6
A
C D
E
F
B G
H
6. Determine the values of the reactions at the
supports, A and B. The pulleys are frictionless,
and their weight can be neglected.
100 kg

801
6.1 Frames and Machines Problem Statement for Example 7
7. While standing on a 50-lb platform, a 150-lb man supports
himself by pulling with force P on the rope. Determine the value of
P and the force that the man's feet exert on the platform. Assume
that the pulley is frictionless, the platform remains horizontal, and
the inclination of the rope can be neglected.

802
6.1 Frames and Machines Problem Statement for Example 8
3 ft
8. A 180-lb man supports himself, while standing in the middle of a
20-lb platform, by pulling on the ropes. Determine the force that the
man's feet exert on the platform. The pulleys are frictionless, each
weighs 10 lb, and the platform is of uniform density.
E F
A
B
D
C
F= 0
3 ft

803
6.1 Frames and Machines Problem Statement for Example 9
A
D C
B
E
10 kip
9. The frame is pin-connected at D and at the supports
A and B. Determine the reactions at A and B.
9 ft
6 ft 6 ft

804
6.1 Frames and Machines Problem Statement for Example 10
10. A trailer is connected to a truck by a ball-and-socket
trailer hitch B. Determine the force of each tire on the
ground, when the truck and trailer are parked.
2 ft
1.5 ft
D C
B
A
900 lb
5000 lb
4.5 ft
4 ft
6 ft

805
6.1 Frames and Machines Problem Statement for Example 11
2.5 in
400 psi
M
C
B
A
11. Determine the couple moment M applied to the crankshaft
that will keep the piston motionless when a 400 psi pressure acts
on the top of the piston. The diameter of the piston is 3 in., and
the piston slides with negligible friction in the cylinder.
8 in.
4 in.

806
6.1 Frames and Machines Problem Statement for Example 12
3 in.
12. Determine the force in spring AD for the wheel
suspension-system shown. The ground exerts a
900-lb vertical force on the wheel.
A B
E D
C
900 lb
10 in.
8 in. 7 in.
6 in.

807
6.1 Frames and Machines Problem Statement for Example 13
A C
B
3 kN 2 kN
40
60
13. Determine the reactions at A and C.
2 m
3 m

808
6.1 Frames and Machines Problem Statement for Example 14
B
A
C
Pin-connection
Smooth floor
30 30
P
14. Determine the force P required to keep the two 20-lb, uniform rods
of length L in equilibrium. The rods are prevented from moving out of
the plane of the figure.

809
6.1 Frames and Machines Problem Statement for Example 15
15. Determine the resultant force applied to the
material at A by the crushing mechanism. Neglect
friction acting on the sides of the piston.
300 N
C
D
B
A
200 mm
500 mm
270 mm
325 mm

810
6.1 Frames and Machines Problem Statement for Example 16
Beam 1
Beam 2
Beam 3
D C
G
B
A
E
F
4 kN
6 m 3 m 3 m
16. Determine the forces transmitted by the rollers. Horizontal
motion is prevented by the short links at A, E, and F.

811
6.1 Frames and Machines Problem Statement for Example 17
400 lb
45 45
17. Determine the resultant force
transmitted by the pin at D in the tongs.
A
B C
D
E F
400 lb
4 in.
8 in. 8 in.
18 in.
14 in.

812
6.1 Frames and Machines Problem Statement for Example 18
18. Determine all forces acting on member FCB.
The 2-kip force is applied to pin F.
F
A C
B
D E
4 kipft
2 kip
6 ft
3 ft
8 ft 4 ft 3 ft

813
6.1 Frames and Machines Problem Statement for Example 19
19. Determine all forces acting on pin D.
Members CD and DE are quarter-circle arcs.
A
B
C
D
E
14 in. 14 in.
40 lb
14 in.
20 in.

814
6.1 Frames and Machines Problem Statement for Example 20
B A
C
D
100 lb
20. Members AB, BC, and BED are connected at pin B.
Determine the reactions at the supports A, C, and D.
60
E
30 in.
25 in.
25 in. 25 in. 50 in.

815
6.1 Frames and Machines Problem Statement for Example 21
100 lb
A B
E
C
D
21. Pin C is attached to member BD and slides freely
in the slot cut in member AE. Determine the tension
in the cord BE and the force transmitted by pin C.
5 ft
5 ft
3 ft 3 ft

816
6.1 Frames and Machines Problem Statement for Example 22
30 kg
A B
C
E D
300 mm 300 mm
200 mm
200 mm
22. The sleeve C is pinned to bar AE and can slide feely
on the smooth bar BD. Calculate reaction components
from the supports at D and E.

817
6.1 Frames and Machines Problem Statement for Example 23
A
40 lb
E
D
C B
0.5 ft
23. Determine all the forces acting on pin B.
3 ft 3 ft
1 ft
2 ft

818
6.1 Frames and Machines Problem Statement for Example 24
24. The smooth cylinder is supported by two pinned
members held together by spring AB. The floor upon
which the members rest is smooth. Determine the
unstretched length of the spring.
5 in.-diameter
100 lb
k = 25 lb/in.
O
C
D E
A
B
24 in.
6 in. 6 in.

819
6.1 Frames and Machines Problem Statement for Example 25
25. Determine all forces acting on pin D.
C
B
A
D
20 kg
Radius = 100 mm
300 mm
250 mm
400 mm

820
6.1 Frames and Machines Problem Statement for Example 26
6-in. dia.
6-in. dia.
3.5 ft
2 ft 2 ft
15 lb
26. Motor E and pulley B weigh 25 lb and 3 lb respectively
and are supported by the two-dimensional frame shown.
When the motor is turned off, the belt tension is 2 lb. The
15-lb force acts on member ADG, not on pin A. Determine
the tension in cable FG and all forces acting on member ADG.
A
B
C
D
E
F
G
H
3.5 ft

821
6.1 Frames and Machines Example 1, page 1 of 2
0.25 in.
A
D
B
C
E
18 lb
18 lb
1. A pair of pliers is used to hold a short, smooth
rod in position. Determine the force that the pliers
applies to the rod.
1.5 in. 3 in.

822
6.1 Frames and Machines Example 1, page 2 of 2
Free-body diagram of member DBC 1
Since the rod is smooth, no
friction force acts between the
rod and the jaw of the pliers;
the only force is the normal
force.
2
Equilibrium equations for member DBC: 3
Fx = 0: B

x = 0
F
y
= 0: F
D
+ B
y
18 lb = 0
M
B
= 0: F
D
(1.5 in.) (18 lb)(3 in.) = 0
+
+
+
Solving these equations simultaneously gives
Bx = 0 Ans.
B
y
= 54 lb Ans.
F
D
= 36 lb Ans.
C
18 lb
B
y
Bx
F
D
D
3 in. 1.5 in.
B 0.25 in.

823
6.1 Frames and Machines Example 2, page 1 of 3
A B
C
F
E D
20 lb
20 lb
2. Determine the force that the nutcracker
applies to the top and bottom of the nut.
Free-body diagram of member AD. Note that it is a 2-force member (a
member loaded at two and only two points). Generally, you do not need to
draw a free-body diagram of a two-force member; it is shown here to illustrate
what we can conclude about the direction of the forces acting on the member.
Because AD is a two-force member, Ax is zero (If
it were not zero, the sum of moments about D
wouldn't be zero, and the body would spin).
Because AD is a two-force member, Dx is zero (If
it were not zero, the sum of moments about A
wouldn't be zero, and the body would spin).
Because AD is a two-force member, the force at D
must be equal and opposite to the force at A.
1
2
3
4
D
A
A
y
Ax = 0
D
y
= A
y

Dx = 0
1.5 in. 5 in.
1 in

824
6.1 Frames and Machines Example 2, page 2 of 3
Bx
B
y
A
y
20 lb
A B
C
6
5
A
y
pointed up on the free body of AD, so by Newton's Third
Law, it must point down on the neighboring body, ABC.
Free-body diagram of ABC
1.5 in. 5 in.
Equilibrium equations for member ABC. 7
Fx = 0: Bx = 0:
F
y
= 0: A
y
+ B
y
20 lb = 0
M
B
= 0: A
y
(1.5 in.) (20 lb)(5 in.) = 0
+
+
+
Solving these equations simultaneously gives
Bx = 0 Ans.
A
y
= 66.7 lb Ans.
B
y
= 86.7 lb Ans.

825
6.1 Frames and Machines Example 2, page 3 of 3
Note that the resultant force applied
to the nut is zero, since the forces
acting on the top and bottom cancel.
8
Equilibrium equation for the nut:
10
9
Free-body diagram of nut
B
y
= 86.67 lb
E
y
F
y
= -86.7 + E
y
= 0
Solving gives
E
y
= 86.7 lb Ans.
+

826
6.1 Frames and Machines Example 3, page 1 of 2
T
T
T
90 kg
B
A
3. Determine the support force at A and the
tension in the cord. The pulleys are frictionless,
and their weight can be neglected.
Tension is same on
both sides of pulley C.
Tension is same on
both sides of pulley D.
1
2
90 kg
A
B
C
D
D
C

827
6.1 Frames and Machines Example 3, page 2 of 2
Free-body diagram of pulley D. This
free body is chosen first because only
one unknown force, T, is present.
Weight of block = mg

= (90 kg)(9.81 m/s
2
)
= 882.9 N
F
y
= 0: 3T 882.9 N = 0
Solving gives
T = 294.3 N Ans.
+
Equilibrium equation for pulley D:
Free-body diagram of pulley C
Equilibrium equation for pulley C
+ F
y
= 0: F
A
2(294.3 N) = 0
Solving gives
F
A
= 589 N Ans.
3
5
6
7
D
T
Tension forces are directed
away from the body.
4
C
F
A
T = 294.3 N T = 294.3 N
T T
90 kg

828
6.1 Frames and Machines Example 4, page 1 of 2
400 lb
T = 400 lb
T = 400 lb
T* = W
T* = W
C
D
C
D
B
A
A
B
400 lb
W
4. Determine the values of the weight W and the
support forces at A and B. The pulleys are frictionless,
and their weight can be neglected.
Tension is same,
400 lb, on both
sides of pulley C.
Tension is same, T* = W,
on both sides of pulley D.
Note that this is a different
cord than the cord that goes
around pulley C. That is
why the tension is labeled
T* rather than T.
1
2
W

829
6.1 Frames and Machines Example 4, page 2 of 2
Equilibrium equation for pulley D:
Fy = 0: 400 2W = 0
Solving gives
W = 200 lb Ans.
Thus the support force at A is 200 lb Ans.
5
+
Free-body diagram of pulley C.
Equilibrium equation for pulley C:
Fy = 0: FB 2(400 lb) = 0
Solving gives
FB = 800 lb Ans.
7
6
C
F
B
T = 400 lb T = 400 lb
T = 400 lb
3
T* = W T* = W
D
4 The tension forces are
directed away from the body.
Free-body diagram of pulley D. This free
body is chosen first because only one
unknown force (W) is present.
+

830
6.1 Frames and Machines Example 5, page 1 of 3
400 lb
5. Determine the force P and the force in the connecting
bar EF. The pulleys are frictionless and their weight can
be neglected.
A B
E
D
C
F
P

831
6.1 Frames and Machines Example 5, page 2 of 3
400 lb
A B
E
D
C
F
P
Tension is same, P, on
both sides of pulley D.
P
P
P
P
P
P
Tension is same, P, on
both sides of pulley F.
Tension is same, P, on
both sides of pulley C.
P
Tension is same, P, on
both sides of pulley E .
4
3
1
2

832
6.1 Frames and Machines Example 5, page 3 of 3
F
8
7
Solving Eqs. 1 and 2 simultaneously gives
P = 100 lb Ans.
F
EF
= 200 lb Ans.
+ F
y
= 0: 2P F
EF
= 0 (2)
Free-body diagram of pulley E. This free body is
chosen because it involves only the forces P and F
EF
.
F
EF
P PP
E
6
P
(1) F
y
= 0: 2P + F
EF
400 lb = 0 +
5 Free-body diagram of pulley F. This free body is
chosen because it involves only the force P and the
force in the bar EF. The support forces at A and B,
which we have not been asked to calculate, do not
appear.
F
EF P
400 lb

833
6.1 Frames and Machines Example 6, page 1 of 2
A
C D
E
F
B G
H
6. Determine the values of the reactions at the
supports, A and B. The pulleys are frictionless,
and their weight can be neglected.
Tensions on
both sides of
pulley F are
equal.
Tensions on both
sides of pulleys C,
D, and E are equal.
Note that this is a
different cord than
the cord wrapped
around pulley F, so
the label T* is used
instead of T.
1
2
E
A
100 kg
F
D
H
C
T T
T*
T*
T*
T*
100 kg
B G

834
6.1 Frames and Machines Example 6, page 2 of 2
Equilibrium equation for pulley E:
Fy = 0: 2T* 490.5 N = 0
Solving gives
T* = 245.25 N
Reaction at A is T* = 245 Ans.
6
+
5 Free-body diagram for pulley E
E
T*
T = 490.5 N
T
F
+
4
3 Free-body diagram of pulley F. This
free body is chosen first because only
one unknown force, T, is present.
Tension in cable supporting the
weight = mg = (100 kg)(9.81 m/s
2
)
= 981 N
Equilibrium equation for pulley F:
Fy = 0: 2T 981 N = 0
Solving gives
T = 490.5 N
Reaction at B = T = 490.5 N Ans.
T*
T

835
6.1 Frames and Machines Example 7, page 1 of 2
7. While standing on a 50-lb platform, a 150-lb man supports
himself by pulling with force P on the rope. Determine the value of
P and the force that the man's feet exert on the platform. Assume
that the pulley is frictionless, the platform remains horizontal, and
the inclination of the rope can be neglected.
Tension is same, P, on
both sides of pulley.
1

836
6.1 Frames and Machines Example 7, page 2 of 2
Weight of platform = 50 lb
P = 100 lb
Free-body diagram of platform. Note carefully that the
150-lb weight of the man is not included in this free
body because the man is not part of the free body.
4
F
feet
Weight of man
= 150 lb
Center of
gravity of man
P P
2 Free-body diagram of man and platform.
This free body is chosen first because only
one unknown force (P) is present.
+ F
y
= 2P 150 lb 50 lb = 0
Solving gives
P = 100 lb Ans.
3 Equilibrium equation for man and platform:
Weight of platform = 50 lb
+ F
y
= 100 lb F
feet
50 lb = 0
Solving gives
F
feet
= 50 lb Ans.
5 Equilibrium equation for platform:

837
6.1 Frames and Machines Example 8, page 1 of 3
8. A 180-lb man supports himself, while standing in the middle of a
20-lb platform, by pulling on the ropes. Determine the force that the
man's feet exert on the platform. The pulleys are frictionless, each
weighs 10 lb, and the platform is of uniform density.
Tension T is
same on both
sides of pulleys
C and D.
Tension T* is
same on both
sides of pulleys
A and B.
1
2
E F
A
B
D
C
F= 0
F= 0
T
T*
3 ft 3 ft
C
D
E F
A
B

838
6.1 Frames and Machines Example 8, page 2 of 3
F
feet
Weight of platform = 20 lb
O
3 ft 3 ft
E F
Equilibrium equations for platform:
F
y
= 0: T* + T F
feet
20 lb = 0 (1)
M
O
= 0: 3T* + 3T = 0 (2)
Solving Eq. 2 gives T = T*, a result we expected because
of symmetry.
Free-body diagram of platform. This free body
is chosen first because it will allow us to show
that T = T*. Note carefully that the weight of the
man is not shown because the man is not part of
the free body.
4
3
+
+
T* T
5 Now we ask, "Is there another free-body diagram that
we can choose that will not introduce any new
unknowns?" The answer is "Yes." Pass a section
through the ropes as shown in the figure below, and
draw the free-body diagram of the part of the system
below the section.
D
C
A
B
3 ft 3 ft
E F
F= 0

839
6.1 Frames and Machines Example 8, page 3 of 3
F= 0
3 ft 3 ft
E F
C B
T T T T* T* T*
T T T
Because of symmetry, T* = T.
Equilibrium equation for part below section:
Free-body diagram of part below section.
7
6
Solving gives
T = 36.67 lb.
Using this result in Eq. 1 gives
F
feet
= 53.3 lb Ans.
F
y
= 0: 6T 180 lb 20 lb 20 lb = 0 +
Weight of man = 180 lb
Weight of platform = 20 lb
Weight of two pulleys = 2(10 lb) = 20 lb

840
6.1 Frames and Machines Example 9, page 1 of 3
A
D C
B
E
10 kip
Free-body diagram of member DEB
DEB is a two-force
member (loaded at
two and only two
points, and no couple
moment acts).
No equilibrium equation needs to be written. Because DEB
is a two-force member, we know that the forces at B and D
are equal, opposite, and have a line of action passing
through B and D. Usually, it is unnecessary to draw a
free-body diagram of a two-force member; it was drawn
here to emphasize that the line of action of the forces is
known.
1
2
3
D
B
E
9. The frame is pin-connected at D and at the supports
A and B. Determine the reactions at A and B.
F
BD
F
BD
9 ft
6 ft 6 ft
6 ft
9 ft

841
6.1 Frames and Machines Example 9, page 2 of 3
Geometry
B
9 ft
F
BD
Ax
A
y
Because BD is a two-force member, only one
unknown (the magnitude of F
BD
) is shown on the
free-body diagram at D.
D
C
10 kip
A
5
4 Free-body diagram of member ACD
8 Solving Eqs. 1, 2, and 3 simultaneously gives
Ax = 5.0 kip Ans.
A
y
= 7.5 kip Ans.
F
BD
= 9.014 kip
6 ft 6 ft
6
9
B
Equilibrium equationd for member ACD:
Fx = 0: 10 kip + Ax F
BD
sin = 0 (1)
F
y
= 0: A
y
+ F
BD
cos = 0 (2)
M
B
= 0: (10 kip)(9 ft) A
y
(6 ft + 6 ft) = 0 (3) +
+
+
6
7
D
6
= tan
-1
9
= 33.69
E

842
6.1 Frames and Machines Example 9, page 3 of 3
The horizontal and vertical components of F
BD
are
equal to the components of the force from the support
acting on the frame at B.
9
F
BD
= 9.014 kip
B
y
= (9.014 kip) cos 33.69 = 7.5 kip Ans.
Bx = (9.014 kip) sin 33.69 = 5.0 kip Ans.
= 33.69
B

843
6.1 Frames and Machines Example 10, page 1 of 3
4.5 ft
4 ft
6 ft
10. A trailer is connected to a truck by a ball-and-socket
trailer hitch B. Determine the force of each tire on the
ground, when the truck and trailer are parked.
2 ft
1.5 ft
D C
B
A
900 lb
5000 lb

844
6.1 Frames and Machines Example 10, page 2 of 3
Free-body diagram of truck and trailer
If F
D
represents the force acting
on one front wheel, then 2F
D

represents the resultant acting on
both front wheels.
Equilibrium equations for truck and trailer:
Two equations in three
unknowns, so an additional
equation is needed.
1
2
3
4
F
y
= 0:
M
A
= 0:
+
+
2F
A
900 lb + 2F
C
5000 lb + 2F
D
= 0 (1)
(900 lb)(1.5 ft) + 2F
C
(1.5 ft + 10 ft)
5000 lb (1.5 ft +10 ft + 4.5 ft)
+ 2F
D
(1.5 ft +10 ft + 4.5 ft + 2 ft) = 0 (2)
D C
B
A
900 lb
2 ft
1.5 ft
2F
D
2F
C
2F
A
5000 lb
10 ft 4.5 ft

845
6.1 Frames and Machines Example 10, page 3 of 3
Free-body diagram of trailer alone
Equilibrium equation for trailer alone:
5
6
M
B
= 0: F
A
(1.5 ft + 6 ft) + (900 lb)(6 ft) = 0 (3) +
B
A
900 lb
1.5 ft
2F
A
F
B
Solving gives

F
A
= 360 lb Ans.
Substituting this result into Eqs. 1 and 2 and solving
simultaneously gives
F
C
= 915 lb Ans.
F
D
= 1675 lb Ans.
6 ft

846
6.1 Frames and Machines Example 11, page 1 of 3
2.5 in
400 psi
M
C
B
A
11. Determine the couple moment M applied to the crankshaft
that will keep the piston motionless when a 400 psi pressure acts
on the top of the piston. The diameter of the piston is 3 in., and
the piston slides with negligible friction in the cylinder.
8 in.
4 in.

847
6.1 Frames and Machines Example 11, page 2 of 3
2
+ M
A
= 0: N(8 in. + 4 in.) M = 0 (1)
N
Ax
A
y
Resultant of pressure = (400 psi)( (3 in./2)
2
= 2827 lb
C
B
A
M
5
4
3
1
Equilibrium equation for the entire mechanism
(Because we weren't asked to determine Ax and
A
y
, we don't bother to write the equilibrium
equations for the horizontal and vertical forces):
Because member AB is not a two-force member
(a couple moment acts on it), the line of action
of the reaction force acting at A is not known.
Thus two unknown components, Ax and A
y
, are
shown.
Since friction is negligible, only the normal
force, N, acts on the side of the piston.
Free-body diagram of entire mechanism
8 in.
4 in.

848
6.1 Frames and Machines Example 11, page 3 of 3
( )
Solving Eqs. 2 and 3, with = 17.35, gives
F
BC
= 2,960 lb
N = 883 lb
Substituting N = 883 lb in Eq. 1 and solving gives
M = 10,600 lbin. Ans.
F
BC
B
= tan
-1
Geometry
C
2827 lb
N
C
Free-body diagram of piston 6
8
Fx = N + F
BC
sin = 0 (2)
F
y
= F
BC
cos 2827 lb = 0 (3)
= 17.35
+
+
Equilibrium equation for the piston 7
8 in.
2.5 in.
9
2.5 in.
8 in.

849
6.1 Frames and Machines Example 12, page 1 of 3
3 in.
12. Determine the force in spring AD for the wheel
suspension-system shown. The ground exerts a
900-lb vertical force on the wheel.
A B
E D
C
900 lb
10 in.
8 in. 7 in.
6 in.

850
6.1 Frames and Machines Example 12, page 2 of 3
F
AB
Ex
E
y
900 lb
B
E
+
+
+
Fx = 0: Ex F
AB
= 0

F
y
= 0: E
y
+ 900 lb = 0
M
E
= 0: F
AB
(10 in.) (900 lb)(8 in.) = 0
Solving simultaneously gives
Ex = 720 lb
E
y
= 900 lb
F
AB
= 720 lb
4
3
2
1
Equilibrium equations for the wheel and axle:
Because member AB is a
two-force member, only one
unknown force component, F
AB
,
is shown acting at B.
Free-body diagram of wheel and axle
Because member EDC is not a
two-force member (it is loaded at
three points), two unknown
components, Ex and E
y
, are
shown acting at E.
8 in.
10 in.

851
6.1 Frames and Machines Example 12, page 3 of 3
M
C
= 0: (900 lb)(7 in. + 3 in.) + F

spring
(3in.) = 0
Solving gives
F

spring
= 3,000 lb Ans.
+
Equilibrium equation for member CDE
Free-body diagram of member CDE
7
5
D
C
E
C
y
Cx
Ex
E
y
= 900 lb
3 in.
Newton's third law says that E
y
should be
directed downward on this diagram because it
was directed upwards on the free-body diagram
of the wheel and axle. But we found that E
y

was negative, so we now reverse its downward
direction.
6
F

spring
7 in.

852
6.1 Frames and Machines Example 13, page 1 of 3
A C
B
3 kN 2 kN
40
60
13. Determine the reactions at A and C.
2 m
3 m

853
6.1 Frames and Machines Example 13, page 2 of 3
Cx
C
y A
y
Ax
2 kN
A
B
3 kN
C
40
60
+
+
+
Fx = 0: Ax + Cx + 2 kN 3 kN = 0
F
y
= 0: A
y
+ C
y
= 0
M
A
= 0: (2 kN)(3 m) + (3 kN)(3 m) + C
y
(L
1
+ L
2
) = 0
4
2
1
Three equations but four unknowns, so
another free-body diagram is needed.
Equilibrium equation for the entire frame
Free-body diagram of entire frame
2 m + 3 m = 5 m
( )
L
2
= = 2.887 m
L
1
= = 5.959 m
( )
60
40
C
B
A
(1)
(2)
(3)
3 Geometry
L
1
L
2
2 m
3 m
L
1
L
2
5 m
tan 40
tan 60
5 m

854
6.1 Frames and Machines Example 13, page 3 of 3
3 kN
B
B
y
Bx
L
2
= 2.887 m
60
2 m
3 m
Solving Eqs. 1, 2, 3 and 4 gives

Ax = 0.396 kN Ans.
A
y
= 0.339 kN Ans.
Cx = 1.396 kN Ans.
C
y
= 0.339 kN Ans.
Since we were not asked to find Bx and B
y
, point B is a
good point for summing moments because Bx and B
y

will not appear in the equation:
Free-body diagram of member BC
6
5
M
B
= 0: C
y
(2.887 m) + Cx(3 m + 2 m) (3 kN)(2 m) = 0 + (4)
L
2
C
y
Cx
C

855
6.1 Frames and Machines Example 14, page 1 of 3
B
A
C
Pin-connection
Smooth floor
30 30
P
14. Determine the force P required to keep the two 20-lb, uniform rods
of length L in equilibrium. The rods are prevented from moving out of
the plane of the figure.

856
6.1 Frames and Machines Example 14, page 2 of 3
cos 30
Solving gives (note that L cancels out)
N = 20 lb
P
N
B
Weight = 20 lb
A
2
1
Because the floor is smooth,
no friction force is shown;
only a normal force is present.
Free-body diagram of the two rods together
30 30
Weight = 20 lb
Cx
C
y
C
3 Equilibrium equation for the two rods together. Because
four unknown forces are present in the free-body
diagram, we know that we must draw an additional
diagram to get enough equations. Note also that we were
not asked to compute Cx and C
y
, so we would like to
save work, if we can, by not writing any equilibrium
equations involving these quantities. Of course, if it
turns out later that we need the values of Cx and C
y
to
determine P, then we can return to this free body and
write the equations at that time.
( ) 20 lb 3 cos 30 + (20 lb) cos 30
N 4 cos 30 = 0
+ M
C
= 0:
2
L
2
L
2
L
2
L
( )
( )

857
6.1 Frames and Machines Example 14, page 3 of 3
Equilibrium equation for rod AB. Summing moments about B
eliminates Bx and B
y
and gives us one equation in one
unknown, P. Thus we don't have to compute forces Cx, C
y
, Bx,
and B
y
.
Free-body diagram of rod AB
5
4
N = 20 lb
A
P
20 lb
B
Bx
B
y
M
B
= 0: P(L sin 30) + (20 lb) cos 30 (20 lb)(L cos 30) = 0
Solving gives
P = 17.32 lb Ans.
+
L
2
cos 30
30
L sin 30
L
2

858
6.1 Frames and Machines Example 15, page 1 of 2
15. Determine the resultant force applied to the
material at A by the crushing mechanism. Neglect
friction acting on the sides of the piston.
Free-body diagram of piston (This is a good free body
to use first, since it shows F

crush
, the quantity that we
want to calculate.)
Normal force
from the wall
Ax and A
y
are the
force components
from member
ABD acting on
the piston.
Equilibrium equations for piston. Two equations, but
four unknowns, so additional equations are needed.
1
3
2
4
Fx = 0: N
A
+ Ax = 0 (1)
F
y
= 0: F

crush
+ A
y
= 0 (2) +
+
300 N
C
D
B
A
A
y
Ax
F

crush
N
A
A
200 mm
500 mm
270 mm
325 mm

859
6.1 Frames and Machines Example 15, page 2 of 2
C
B
D
A
BC is a
two-force
member.
Fx = 0: Ax + F
BC
cos (300 N) cos = 0 (3)
F
y
= 0: A
y
F
BC
sin + (300 N) sin = 0 (4)
M
B
= 0: Ax(270 mm) + A
y
(200 mm)
+ (300 N) (325 mm) = 0 (5)
(
200 mm
270 mm
)
= tan
-1
= 36.53
200 mm
270 mm
500 mm
F
BC
Ax
A
y
D
300 N
(
500 mm
200 mm
)
6
7
Geometry
5
Free-body diagram of member ABD
Equilibrium equations for member ABD
Substituting = 68.20 and = 36.53 into
Eqs. 3, 4, and 5 and solving Eqs. 1-5
simultaneously gives
Ax = N
A
= 16.5 N
F
BC
= 693 N
A
y
= 465 N
F

crush
= 465 N Ans.
8
B
A
+
+
+
= tan
-1
= 68.20
270 mm
325 mm
200 mm

860
6.1 Frames and Machines Example 16, page 1 of 3
16. Determine the forces transmitted by the rollers. Horizontal
motion is prevented by the short links at A, E, and F.
Beam 1
Beam 2
Beam 3
D C
G
B
A
E
F
4 kN
6 m 3 m 3 m

861
6.1 Frames and Machines Example 16, page 2 of 3
Fx = 0: Ex = 0 (4)
F
y
= 0: C
y
+ D
y
1 kN 3 kN = 0 (5)
M
C
= 0: D
y
(6 m) (1 kN)(6 m + 6 m) = 0 (6)
Solving simultaneously gives
Ex = 0
C
y
= 2 kN Ans.
D
y
= 2 kN Ans.
+
C
6 m 6 m
+
+
4 Equilibrium equations for beam 2:
Beam 2
Ex
E
A
y
= 1 kN
D
y
D
3 Free-body diagram of beam 2
C
y
B
y
= 3 kN
Ax
3 m
A
y B
y
4 kN
A
Beam 1
B
+
+
+
(1)
(2)
(3)
Fx = 0: Ax = 0
F
y
= 0: A
y
+ B
y
4 kN = 0
M
A
= 0: B
y
(9 m + 3 m) (4 kN)(9 m) = 0
Solving simultaneously gives
Ax = 0
A
y
= 1 kN Ans.
B
y
= 3 kN Ans.
2
1
Equilibrium equations for beam 1:
Free-body diagram of beam 1 (This is a good
free body to begin with, since only three
unknown forces are present).
9 m

862
6.1 Frames and Machines Example 16, page 3 of 3
Fx = 0: Fx = 0 (7)
F
y
= 0: F
y
2 kN 2 kN + G
y
= 0 (8)
M
G
= 0: (2 kN)(6 m) F
y
(6 m + 6 m) = 0 (9)
Solving simultaneously gives
Fx = 0
F
y
= 1 kN Ans.
G
y
= 3 kN Ans.
+
6 m 6 m
+
+
6 Equilibrium equations for beam 3
Fx
F
y
F
D
Beam 3
D
y
= 2 kN
G
y
G
C
y
= 2 kN
Free-body diagram of beam 3 5

863
6.1 Frames and Machines Example 17, page 1 of 4
400 lb
45 45
17. Determine the resultant force
transmitted by the pin at D in the tongs.
A
B C
D
E F
400 lb
4 in.
8 in. 8 in.
18 in.
14 in.

864
6.1 Frames and Machines Example 17, page 2 of 4
F
AB
cos 45 + Dx + Fx = 0 (1)
F
AB
sin 45 + D
y
F
y
= 0 (2)
Fx(18 in.) F
y
(8 in.) F
AB
cos 45(14 in.) (3)
F
AB
sin 45(4 in.) = 0
14 in.
45
F
AB
D
y
Dx
F
y
Fx F
B
D
+
+
+
Fx = 0:
F
y
= 0:
M
D
= 0:
5
4
3
2
1
Equilibrium equations for member BDF:
AB is a two-force member, so only one unknown,
F
AB
, is shown (the magnitude of the force is
unknown; the line of action of the force is known).
Free-body diagram of member BDF (This is a
good free body to begin with, since it shows the
components of force transmitted by pin D).
The tongs must depend on a friction
force to be able to lift the weight;
both a normal force, Fx, and a friction
force, F
y
, are present.
Three equations but five unknowns so at
least one more free body is needed (to obtain
more equations of equilibrium). 4 in.
18 in.
8 in.

865
6.1 Frames and Machines Example 17, page 3 of 4
7
10
+
+
Equilibrium equations for connection A:
Free-body diagram of connection A. 6
8
45
+
+
8 in. 8 in.
E
F
y
= 0: E
y
+ F
y
400 lb = 0 (6)
M
E
= 0: (400 lb)(8 in.) + F
y
(16 in.) = 0 (7)
Solving gives
E
y
= 200 lb
F
y
= 200 lb
Equilibrium equations for block EF:
Fx = 0: F
AB
cos 45 + F
AC
cos 45 = 0 (4)
F
y
= 0: 400 lb F
AB
sin 45 F
AC
sin 45 = 0 (5)
Solving gives
F
AB
= 282.8 lb
F
AC
= 282.8 lb
Since both AB and AC
are two-force members,
only two unknowns
(magnitudes of F
AB
and
F
AC
) appear on this free
body.
9 Free-body diagram of block EF
400 lb
E
y
Ex
F
y
Fx
45
400 lb
F
AB
A
F
AC

866
6.1 Frames and Machines Example 17, page 4 of 4
Substituting F
AB
= 282.8 lb and F
y
= 200 lb
into Eqs. 1, 2, and 3 and solving gives
Dx = 488.9 lb
D
y
= 0.0 lb
Fx = 288.9 lb
The resultant force transmitted at D is, then,

11
=
= 489 lb Ans.
( 488.9 lb)
2
+ (0.0)
2
Dx
2
+ D
y
2

867
6.1 Frames and Machines Example 18, page 1 of 5
18. Determine all forces acting on member FCB.
The 2-kip force is applied to pin F.
F
A C
B
D E
4 kipft
2 kip
6 ft
3 ft
8 ft 4 ft 3 ft

868
6.1 Frames and Machines Example 18, page 2 of 5
Three unknowns, F
y
, C
y
, and B
y
, remain to
be determined, so at least one additional
free-body diagram is needed.
1
2
3
4
B
C
F
F
y
Fx
C
y
Cx
B
y
+
+
+
(1)
(2)
(3)
6 ft
Free-body diagram of member FCB not including the pin at F.
Fx and F
y
are forces from the pin
acting on member FCB. The force
from member FD and the 2-kip force
are not shown since they act on the
pin, not on member FCB.
Equilibrium equations for member FCB:
Fx = 0: Fx + Cx = 0
F
y
= 0: F
y
+ C
y
+ B
y
= 0
M
C
= 0: Fx (6 ft) = 0
Eqs. 1 and 3 imply
Fx = 0 Ans.
Cx = 0 Ans.

869
6.1 Frames and Machines Example 18, page 3 of 5
Two unknowns, C
y
and B
y
, remain to be determined.
Substituting = 33.69 in Eqs. 4 and 5 and solving
simultaneously gives
F
DF
= 3.606 kip
F
y
= 3 kip Ans.
10
9
C
D
Free-body diagram of pin at F (This is a good choice
for the next free body to use, since it will allow us to
calculate F
y
).
Equilibrium equations for pin at F:
Fx = 0: 2 kip + F
DF
sin = 0
F
y
= 0: F
y
F
DF
cos = 0
DF is a two-force
member, so DF's line
of action is known.
Geometry 8
7
6
5
= tan
-1
= 33.69
F
( )
2 kip
F
DF
Fx = 0
F
y
+
+
(4)
(5)
6 ft
4 ft
4 ft
6 ft

870
6.1 Frames and Machines Example 18, page 4 of 5
Equilibrium equation for entire frame:
M
E
= 0: 4 kipft + (2 kip)(6 ft)
B
y
(7 ft) = 0 (6)
Solving gives
B
y
= 1.143 kip Ans.
Substituting B
y
= 1.143 kip and F
y
= 3 kip into Eq. 2
F
y
+ C
y
+ B
y
= 0 (Eq. 2 repeated)
and solving gives
C
y
= 1.857 kip Ans.
12
+
Ex
E
y
Free-body diagram of entire frame (This free-body
diagram will allow us to determine B
y
).
11
4 kipft
2 kip
C
B
A
6 ft
E
F
D
B
y
7 ft

871
6.1 Frames and Machines Example 18, page 5 of 5
F
y
= 3 kip
C
y
= 1.857 kip
B
y
= 1.143 kip
Free-body diagram of member FCB (not
including pin F) showing correct senses
13
B
C
F
Cx = 0
Fx = 0

872
6.1 Frames and Machines Example 19, page 1 of 3
19. Determine all forces acting on pin D.
Members CD and DE are quarter-circle arcs.
Three-dimensional free-body diagram of pin D
DE is a two-force member, so only one
unknown component, F
DE
, is shown (the
line of action of the force is known to
pass through E and D).
CD is also a two-force member (The line of action
of F
CD
is known to pass through C and D).
BD is also a two-force member.
Since member BD is horizontal,
force F

BD
is also horizontal.
1
2
3
4
A
B
C
D
E
14 in. 14 in.
F
BD
F
CD
40 lb
F
DE
x
y
z
14 in.
20 in.

873
6.1 Frames and Machines Example 19, page 2 of 3
F
BD
F
CD
F
DE
5 Two-dimensional free-body diagram of pin D
(1)
(2)
+
+
7
6
Two equations but three unknowns so at least one more
free body is needed (to obtain another equation of
equilibrium).
Equilibrium equations for pin D:
Fx = 0: F
BD
+ F
CD
cos 45 F
DE
cos 45 = 0
F
y
= 0: F
CD
sin 45 F
DE
sin 45 = 0
D
E D
45 45
14 in.
14 in.

874
6.1 Frames and Machines Example 19, page 3 of 3
A
To simplify the calculation of moments, FDE has been
expressed in terms of horizontal and vertical components.
Free-body diagram of pin D showing
correct senses of forces
11
Free-body diagram of portion ABCD of
frame including pin at D
9
8
Equilibrium equation for ABCD:
M
A
= 0: (40 lb)(20 in.) + F
DE
(cos 45)(20 in.)
F
DE
(sin 45)(14 in.) = 0 (3)
Solving Eq. 3 gives
F
DE
= 188.6 lb Ans.
Using this result in Eqs. 1 and 2 and solving simultaneously gives
F
BD
= 267 lb Ans.
F
CD
= 188.6 lb Ans.
+
10
40 lb
F
DE
cos 45
B D
F
DE
sin 45
C
F
CD
= 188.6 lb
F
DE
= 188.6 lb
F
BD
= 267 lb
Ax
A
y
14 in.
20 in.
y
x
z

875
6.1 Frames and Machines Example 20, page 1 of 3
B A
C
D
100 lb
20. Members AB, BC, and BED are connected at pin B.
Determine the reactions at the supports A, C, and D.
60
E
30 in.
25 in.
25 in. 25 in. 50 in.

876
6.1 Frames and Machines Example 20, page 2 of 3
Free-body diagram of entire frame
BD is a two-force member, so only one unknown
the magnitude of the force is shown; the line of
action of the force is known.
BC is also a two-force member.
Equilibrium equations for entire frame:
Fx = 0: 100 cos 60 F
BD
cos
F
BC
cos = 0 (1)
F
y
= 0: 100 sin 60 + A
y
F
BD
sin
+ F
BC
sin = 0 (2)
M
B
= 0: A
y
(25 in. + 25 in.)
+ (100 sin 60)(25 in.) = 0 (3)
+
+
+
1
2
3
4
100 lb
C
D
60
A
y
F
BC
F
BD
E
B
A
Three equations and three unknowns so no more
free bodies are needed.
5
25 in. 25 in. 50 in.

877
6.1 Frames and Machines Example 20, page 3 of 3
50 in.
30 in.
25 in.
30 in.
50 in.
25 in.
50 in.
= tan
-1
( ) = 30.96
= tan
-1
( ) = 26.57
Geometry 6
Substituting = 30.96 and = 26.57 in Eqs. 1, 2,
and 3 and solving simultaneously gives
F
BC
= 74.5 lb Ans.
F
BD
= 19.40 lb Ans.
A
y
= 43.3 lb Ans.
7
B
C
E
D

878
6.1 Frames and Machines Example 21, page 1 of 3
100 lb
A B
E
C
D
21. Pin C is attached to member BD and slides freely
in the slot cut in member AE. Determine the tension
in the cord BE and the force transmitted by pin C.
5 ft
5 ft
3 ft 3 ft

879
6.1 Frames and Machines Example 21, page 2 of 3
Free-body diagram of member
AE, including pin at E.
Since we assumed that the pin at E is
part of the free body, then the tension
T in the cord must be shown (if, as
seems reasonable, we assume that the
cord is attached to the pin)
Because the pin slides freely in
the slot, only a normal
component, F
C
, is present (No
friction acts).
Equilibrium equation for member AE (Since we weren't asked to
calculate Ex and E
y
, we won't write any additional equilibrium
equations that would introduce Ex and E
y
as additional
unknowns):
M
E
= 0: Ax(5 ft + 5 ft) + F
C
L
CE
= 0 (1)
1
2
3
4
E
C
A
T
E
y
Ex
F
C
Ax
+
C
= tan
-1
( ) = 59.04
= 90 = 30.96
L
CE
= = 5.831 ft
5 Geometry
E
6 We need another free-body one from
which we can calculate Ax easily.
5 ft
3 ft
L
CE
5 ft
5 ft
3 ft
5 ft
sin
5 ft

880
6.1 Frames and Machines Example 21, page 3 of 3
D
y
Dx
D
E
7
Free-body diagram of entire frame (We
can calculate Ax from this free body).
A
Ax
B
100 lb
+
11
T
B
100 lb
F
DE
D
3 ft
D
y
3 ft
F
C
= 171.5 lb
10 The force F
C

acting on the
pin acts at
the angle
previously
calculated.
Dx
= 30.96
C
Equilibrium equation for member DCB
M
D
= 0: (171.5 lb)(cos 30.96)(5 ft)
+ (171.5 lb)(sin 30.96)(3 ft)
+ (100 lb)(5 ft + 5 ft) T(3 ft + 3 ft) = 0
Solving gives
T = 88.2 lb Ans.
9 Free-body diagram of member DCB, including pin at
D (We can calculate T easily from this free body)
10 ft
5 ft
5 ft
C
Equilibrium equation for entire frame:
M
D
= 0: Ax(10 ft) + (100 lb)(10 ft) = 0 (2)
Solving gives Ax = 100 lb and using this value in Eq. 1
gives
F
C
= 171.5 lb Ans.
+
8

881
6.1 Frames and Machines Example 22, page 1 of 4
22. The sleeve C is pinned to bar AE and can slide feely
on the smooth bar BD. Calculate reaction components
from the supports at D and E.
30 kg
A B
C
E D
300 mm 300 mm
200 mm
200 mm

882
6.1 Frames and Machines Example 22, page 2 of 4
Free-body diagram of entire frame
Equilibrium equation for entire frame
Fx = 0: Dx + Ex = 0 (1)
F
y
= 0: D
y
+ E
y
294.3 N = 0 (2)
M
D
= 0: E
y
(300 mm + 300 mm)
(294.3 N)(300 mm) = 0 (3)
Dx and Ex remain
unknown, so at least
one more free-body
diagram is needed.
1
2
3
+
+
+
E
B
D
A
C
Weight = mg

= (30 kg)(9.81 m/s
2
)
= 294.3 N
Solving Eqs. 2 and 3 gives
D
y
= 147.2 N Ans.
E
y
= 147.2 N Ans.
Dx
D
y
E
y
Ex
300 mm 300 mm
400 mm

883
6.1 Frames and Machines Example 22, page 3 of 4
D
y
= 147.2 N
Dx
D
4
Free-body diagram of member DCB Member AB is a two-force member so only one
unknown (the magnitude) is shown.
Since the sleeve slides freely, it exerts a normal force, F

C
,
only (no friction force parallel to member DCB). Note that
F

C
does not lie in the direction of member ACB.
Equilibrium equations for member DC:
F
y
= 0: 147.2 N F
C
sin = 0 (4)
M
B
= 0: Dx(400 mm) + (F
C
)L

BC

(147.2 N) (600 mm) = 0 (5)
7
Geometry 8
6
5
B
F
AB
F
C
C
+
+
= tan
-1
( ) = 33.69
= 90 = 56.31
L

BC
= = 360.56 mm
B
C
L

BC
400 mm
600 mm
300 mm
200 mm 300 mm
200 mm
cos 33.69
300 mm

884
6.1 Frames and Machines Example 22, page 4 of 4
Substituting = 56.31 and L

BC
= 360.56 mm into
Eqs. 4 and 5 and solving simultaneously gives
F
C
= 176.9 N
Dx = 61.3 N Ans.
Then Eq. 1 gives
Ex = Dx
Ex = 61.3 N Ans.
9

885
6.1 Frames and Machines Example 23, page 1 of 4
A
40 lb
E
D
C B
0.5 ft
23. Determine all the forces acting on pin B.
3
B
y
B`x and B`
y
represent the force
components from the pulley.
4
B`
y
Three-dimensional free-body diagram of pin B
BE is a two-force
member, so only one
unknown, F
BE
, (the
magnitude of the
force) is shown. The
line of action of the
force is known.
B`x
1
F
BE
Bx
2 Bx and B
y
represent
the force components
from member ABC.
3 ft 3 ft
1 ft
2 ft
x
z
y

886
6.1 Frames and Machines Example 23, page 2 of 4
Geometry
Fx = 0: Bx + B`x + F
BE
cos = 0
F
y
= 0: B
y
+ B`
y
+ F
BE
sin = 0
= tan
-1
( ) = 26.57
+
+
Two-dimensional free-body diagram of pin B
Equilibrium equations for pin B:
5
6
7
B
y
B`
y
Bx B`x
F
BE
E
B
C
(1)
(2)
Two equations but five unknowns so at least one more
free body is needed (to obtain additional equations of
equilibrium).
8
B`
y
B`x
40 lb
40 lb
B
Fx = 0: B`x + 40 lb = 0
F
y
= 0: 40 B`
y
= 0
Solving simultaneously gives
B`x = 40.0 lb Ans.
B`
y
= 40.0 lb Ans.
9
10
Free-body diagram of pulley, not including pin at B
B`x and B`
y
represent the forces
from the pin acting on the pulley.
Equilibrium equations for the pulley:
+
+
11
3 ft
1.5 ft
3 ft.
1.5 ft.

887
6.1 Frames and Machines Example 23, page 3 of 4
Free-body diagram of member ABC not including pin at B
This free body introduces three additional
unknowns (Cx, C
y
, and A
y
), so it looks like a
poor choice. But by expressing Bx in terms of
Cx through Eq. 5, we only need one more free
body to complete the solution.
Equilibrium equation for member ABC:
Fx = 0: Bx + Cx = 0 (5)
Bx and B
y
represent the forces from
the pin acting on member ABC.
15
14
13
12
A
B
C
A
y
B
y
Bx
C
y
Cx
+

888
6.1 Frames and Machines Example 23, page 4 of 4
= 26.57
F
BE
40 lb
Cx
Dx
D
y
+
B
y
= 46.7 lb
B`x = 40 lb
B`
y
= 40 lb
F
BE
= 14.91 lb
Bx = 26.7 lb
C
y
x
y
z
1 ft
2 ft
Equilibrium equation for member DCE:
M
D
= 0: Cx(2 ft) + (40 lb)(2 ft + 0.5 ft)
+ (F
BE
cos )(2 ft + 0.5 ft + 1 ft) = 0 (6)
Substituting B`

x = 40 lb in Eq. 1 and then solving Eqs. 1, 5,
and 6 simultaneously gives
Cx = 26.7 lb
Bx = 26.7 lb Ans.
F
BE
= 14.91 lb Ans.
Substituting B`
y
= 40 lb and F
BE
= 14.91 lb in Eq. 2 and
then solving for B
y
gives
B
y
= 46.7 lb Ans.
Free-body diagram of pin B with correct sense of forces 19
Free-body diagram of member DCE
The tension in the
cord is 40 lb on both
sides of the pulley, so
the tension force
acting on member
DCE is 40 lb
18
17
16
D
C
0.5 ft
E

889
6.1 Frames and Machines Example 24, page 1 of 4
24. The smooth cylinder is supported by two pinned
members held together by spring AB. The floor upon
which the members rest is smooth. Determine the
unstretched length of the spring.
The unstretched length of the spring is
related to the force in the spring:
F
spring
= k (extension of spring)
= k (L
stretched
L
unstretched
)
= (25 lb/in.)[(6 in. + 6 in.) L
unstretched
] (1)
So to calculate L
unstretched
we first must calculate F
spring
.
5 in.-diameter
100 lb
1
k = 25 lb/in.
O
C
D
E
A
B
24 in.
6 in. 6 in.

890
6.1 Frames and Machines Example 24, page 2 of 4
2
3
4
7
L
CD
F
D
Cx
C
y
C
F
spring
D
B
F
B
+
Radius = = 2.5 in.
D
O
C
24 in.
6 in.
L
CD
5 Geometry
6 By similar triangles, =
Solving gives L
CD
= 10 in.
24 in.
6 in.
2.5 in.
6 in. 24 in.
L
CD
2
5 in.
Free-body diagram of member BCD
Since the cylinder is smooth, only a force F

D

normal to the member is present (no friction
force parallel to the member)
Equilibrium equation for member BCD. Since we weren't
asked to calculate Cx and C
y
, we won't write equilibrium
equations involving them. If we find later that we need to
know these values, we can always return to this free body
and write additional equations.
M
C
= 0: F
D
(L
CD
) + F
B
(6 in.) F
spring
(24 in.) = 0 (2)
To calculate F
spring
from Eq. 2, we need first to calculate
F

D
and F

B
.

891
6.1 Frames and Machines Example 24, page 3 of 4
Equilibrium equation for the cylinder:
Fx = 0: F
D
sin F

E
sin = 0
F
y
= 0: F
D
cos + F

E
cos 100 lb = 0
Substituting = 75.96 in Eqs. 3 and 4 and
solving gives
F

E
= 206.1 lb
F
D
= 206.1 lb
Geometry 10
11
9
Free-body diagram of the cylinder (This free
body will give us F

D
).
8
100 lb
O
F
D
F

E
(3)
(4) +
+
6 in.
24 in.
O
C
D
= tan
-1
( ) = 75.96
24 in.
6 in.

892
6.1 Frames and Machines Example 24, page 4 of 4
100 lb
F
A
F
B
+
6 in. 6 in.
A
B
Free-body diagram of the entire structure
(This free body will give us F
B
).
Substituting L
CD
= 10 in., F
B
= 50 lb, and F
D
= 206.1 lb into Eq. 2:
F
D
(L
CD
) + F
B
(6 in.) F
spring
(24 in.) = 0 (Eq. 2 repeated)
and solving gives
F
spring
= 98.38 lb
Substituting F
spring
= 98.38 lb into Eq. 1:
F
spring
= (25 lb/in.)(12 in L
unstretched
) (Eq. 1 repeated)
and solving gives
L
unstretched
= 8.07 in. Ans.
Equilibrium equation for the entire structure:
M
A
= 0: (100 lb)(6 in.) + F
B
(6 in. + 6 in.) = 0
Solving gives
F
B
= 50 lb
14
13 12

893
6.1 Frames and Machines Example 25, page 1 of 3
25. Determine all forces acting on pin D.
D
y
Dx
F
CD
F
BD
C
B
A
D
20 kg
Radius = 100 mm
4
3
2
1
Similiarly, CD is a
two-force member, so only
one unknown (the
magnitude of the force) is
shown; the direction of the
force is known.
BD is a two-force member,
so only one unknown
component (the magnitude
of the force) is shown ; the
line of action is known.
Dx and D
y
represent
the force components
from the pulley.
Three-dimensional free-body diagram of pin D
x
y
z
300 mm
250 mm
400 mm

894
6.1 Frames and Machines Example 25, page 2 of 3
Equilibrium equations for the pulley:
Fx = 0: Dx (196.2 N) cos = 0
F
y
= 0: D
y
196.2 N + (196.2 N) sin = 0
The tension is the same on
both sides of the pulley.
Free-body diagram of pulley
10
9
8
D
D
y
Dx
Weight = mg
= (20 kg)(9.81m/s
2
)
= 196.2N
T = 196.2 N
(3)
(4)
+
+
+
+
F
CD
F
BD
Dx
D
y
6
5 Two-dimensional free-body diagram of pin D
Equilibrium equation for pin D:
Fx = 0: F
BD
+ Dx + F
CD
cos = 0 (1)
F
y
= 0: F
CD
sin + D
y
= 0 (2)
= tan
-1
( ) = 32.01
D
B
C
32
7 Geometry
400
250
400
250

895
6.1 Frames and Machines Example 25, page 3 of 3
z
y
x
Radius = 100
400 + 100 sin
300 100 cos
100 sin
D
A
11 Geometry
12 From the figure above, we see that must satisfy the
following equation:
tan = ( )
Solving numerically gives = 25.33
300
300 100 cos
100 cos
400
400 + 100 sin
Substituting = 25.33 in Eqs. 3 and 4 and
solving simultaneously gives
Dx = 177.3 N Ans.
D
y
= 112.3 N Ans.
Substituting these values and = 32.01 into
Eqs. 1 and 2 and solving simultaneously gives
F
BD
= 2.2 N Ans.
F
CD
= 212 N Ans.
Free-body diagram of pin D with correct
sense of forces
14
13
F
BD
= 2.2 N
F
CD
= 212 N
D
y
= 112 N
Dx = 177 N

896
6.1 Frames and Machines Example 26, page 1 of 7
6-in. dia.
6-in. dia.
3.5 ft
2 ft 2 ft
15 lb
26. Motor E and pulley B weigh 25 lb and 3 lb respectively
and are supported by the two-dimensional frame shown.
When the motor is turned off, the belt tension is 2 lb. The
15-lb force acts on member ADG, not on pin A. Determine
the tension in cable FG and all forces acting on member ADG.
A
B
C
D
E
F
G
H
3.5 ft

897
6.1 Frames and Machines Example 26, page 2 of 7
Free-body diagram of member ADG, not including pin G and not
including pin A (Since we are asked to find all the forces acting
on member ADG, ADG is the best choice for the first free-body
diagram).
Forces Gx and G
y
are from the pin G
acting on the member ADG. The force
from the cable does not appear here,
since the cable is attached to the pin, and
the pin is not part of the free body.
Equilibrium equations for ADG:
Fx = 0: 15 lb + Ax + Dx + Gx = 0
F
y
= 0: A
y
+ D
y
+ G
y
= 0
M
G
= 0: 15 lb(3.5 ft + 3.5 ft) Ax(3.5 ft + 3.5 ft) Dx(3.5 ft) = 0
1
2
3
+
+
+
15 lb
Gx
Dx
Ax
G
y
D
y
A
y
(1)
(2)
(3)
A
D
G
3.5 ft
3.5 ft

898
6.1 Frames and Machines Example 26, page 3 of 7
= tan
-1
( ) = 41.19
Here the forces Gx and G
y
from
member ADG are equal and opposite
those specified in the free-body
diagram for member ADG.
Free-body diagram of pin G.
Equilibrium equations for pin G:
Fx = 0: Gx + G`x + T cos = 0
F
y
= 0: G
y
+ G`
y
+ T sin = 0
Geometry
G
9
+
+
4
H
3.5
F
G'x and G'
y
are
forces from the
pin support.
8
6
4
7
Gx
G`
y
G
y
T
The tension T comes from the cable.
Since we are asked to determine T,
pin G is a good choice for the next
free-body diagram.
G`x
5
(4)
(5)
3.5
4

899
6.1 Frames and Machines Example 26, page 4 of 7
H
D
15 lb A
E
Weight = 25 lb
F
3.5 ft
6-in dia
Only the forces G'x and G'
y

from the support are shown.
The forces Gx, G
y
, and T are
internal forces for this free
body and so are not shown.
11
G`x
G`
y
2 ft
G
H
y
2 ft
6-in dia
B
C
3.5 ft
Weight = 3 lb
Free-body diagram of entire body.
Using this free body next will allow
us to determine G'x and G'
y
.
10
+
+
12 Equilibrium equations for the entire body
(Only two equations are needed, because we
don't need to calculate the reaction H
y
.):
Fx = 0: 15 lb + G`x = 0 (6)

M
H
= 0: 15 lb(3.5 ft + 3.5 ft)
+ 3 lb(2 ft) 25 lb(2 ft)
G`
y
(2 ft + 2 ft) = 0 (7)
Solving Eqs. 6 and 7 gives
G`x = 15 lb
G`
y
= 12.25 lb

900
6.1 Frames and Machines Example 26, page 5 of 7
B
C
Cx
C
y
14
13
2 ft 0.25 ft
= 1.75 ft
4 ft
Equilibrium equation for the upper shelf and pulley:
M
C
= 0: A
y
(4 ft) + (3 lb)(1.75 ft + 0.25 ft) + (2 lb)(1.75 ft)
+ (2 lb)(1.75 ft + 0.25 ft + 0.25 ft) = 0
Solving gives
A
y
= 3.5 lb Ans.
15
+
Free-body diagram of upper shelf and pulley.
This free body will allow us to calculate A
y
.
The diameter of the pulley is 6 in. so
the radius is 3 in. (= 0.25 ft).
Weight = 3 lb
A
y
Ax
A
2 lb 2 lb
6 in. dia

901
6.1 Frames and Machines Example 26, page 6 of 7
16
F
y
Fx
17
6 in. dia (= 0.5 ft)
2 lb
E
4 ft
1.75 ft
F D
D
y
Dx
2 lb
Weight = 25 lb
1.75 ft
This free body of the lower shelf and motor
will allow us to calculate D
y
.
Since pin F is not
included in the
free-body, the
tension of the cable
is not shown
(Although it doesn't
really affect our
calculation, since
we sum moments
about point F and
no force at F would
enter the equation).
2 ft
Recall that Eq. 5 is
G
y
+ G`
y
+ T sin = 0 (Eq. 5 repeated)
Substituting G
y
= 14 lb, G`
y
= 12.25 lb, and
= 41.19 and then solving gives
T = 39.86 lb Ans.
Recall that Eq. 2 is
A
y
+ D
y
+ G
y
= 0 (Eq. 2 repeated)
Substituting A
y
= 3.5 lb and D
y
= 10.5 lb and
then solving gives
G
y
= 14 lb Ans.
Equilibrium equation for the lower shelf and motor:
M
F
= 0: D
y
(4 ft) + (25 lb)(2 ft)
(2lb)(1.75 ft + 0.5 ft)
(2 lb)(1.75 ft) = 0
Solving gives
D
y
= 10.5 lb Ans.
18 20
19
+

902
6.1 Frames and Machines Example 26, page 7 of 7
Ax = 0
A
y
= 3.5 lb
Free-body diagram of member
ADG not including pin G (correct
sense of forces are shown).
D
y
= 10.5 lb
G
y
= 14.0 lb
15 lb
Force G'x, G'
y
, and T are not
shown, since they act on pin
G, not on member ADG.
Gx = 15.0 lb
Dx = 30.0 lb
24
23
21
22
Recall that Eq. 4 is
Gx + G`x + T cos = 0 (Eq. 4 repeated)
Substituting G`x = 15.0 lb, T = 39.86 lb and = 41.19 and
then solving gives
Gx = 15 lb Ans.
Finally recall that Eqs. 1 and 3 are
15 lb + Ax + Dx + Gx = 0 (Eq. 1 repeated)
15 lb(7 ft) Ax(7 ft) Dx(3.5 ft) = 0 (Eq. 3 repeated)
Substituting Gx = 15.0 lb and solving simultaneously gives
Ax = 0 Ans.
Dx = 30.0 lb Ans.

903

6.2 Trusses: Method of Joints and Zero-Force Members
904
6.2 Trusses: Method of Joints and Zero-Force Members Procedures and Strategies, page 1 of 2
C
B
A
10 kN
H
G F
E
D
C B
A
4 kip
2 kip 2 kip
30 60
60
K
J
I
H
G
F
E
D
C B
A
2 kip
2 kip
x
y
Summing forces in the
(inclined) y direction will
give an equation with a single
unknown, F
IF
, so F
IF
can be
found easily.
Procedures and Strategies for Solving Problems
Involving Trusses: the Method of Joints
1. Examine the truss and try to find a joint at which
you can calculate at least one member force.
Such joints occur where
a) two members meet and an external load is
present,
b) two members meet at a support (the reaction
forces at the support must first be found by
using a free-body analysis of the entire truss),
and
c) three members meet and two are collinear.
2. Draw a free-body diagram of the joint and show
all unknown forces to be tensile (directed away
from the joint). Write the Fx = 0 = F
y

equilibrium equations and solve for the unknown
member force or forces. If a member force turns
out to be negative, then the force is compressive.

905
6.2 Trusses: Method of Joints and Zero-Force Members Procedures and Strategies, page 2 of 2
3. Examine the truss again, as you did in Step 1, but
take into account that one or more member forces
are known because you just now found them.
Repeat Step 2, being careful to show compressive
forces pointing towards the joint, tensile forces away
from the joint. Proceed in this manner joint-by-joint
through the entire truss.
4. You can save work by identifying zero-force
members by inspection. The common situations in
which zero-force members occur are where
a) two members meet at an unloaded joint, and
b) three members meet at an unloaded joint, and
two of the members are collinear the
non-collinear member is a zero-force member.
G F E D
C
B
A
4 kN
M
L
J
I
H
G
F E
D C
B
A
2 kip
4 kN
60

906
6.2 Trusses: Method of Joints and Zero-Force Members Problem Statement for Example 1
1. Determine the force in each member of the truss and
state whether the force is tension or compression.
10 kN
A
B
C
5 m
3 m

907
6.2 Trusses: Method of Joints and Zero-Force Members Problem Statement for Example 2
2. Determine the force in each member of the truss and state
whether the force is tension or compression.
2 kip 2 kip 4 kip
14 ft
10 ft 10 ft 10 ft 10 ft
A
B C
D
E
F
G H

908
6.2 Trusses: Method of Joints and Zero-Force Members Problem Statement for Example 3
3. Determine the force in each member of
the truss and state whether the force is
tension or compression.
10 ft
12 ft
60
30
400 lb
900 lb
A
B
C
D
E

909
6.2 Trusses: Method of Joints and Zero-Force Members Problem Statement for Example 4
30
4. Determine the force in each member of the truss and state whether
the force is tension or compression. The truss is symmetric.
2 kip
2 kip
2 kip
2 kip
2 kip
6 ft 6 ft 6 ft 6 ft 6 ft 6 ft
A
B C
D
E
F
G
H
I J
K
60
60

910
6.2 Trusses: Method of Joints and Zero-Force Members Problem Statement for Example 5
5. Determine the force in each member of the truss and
state whether the force is tension or compression.
4 ft 2 ft 4 ft
2.5 ft
1 ft
100 lb
A
B
C
D
E
F

911
6.2 Trusses: Method of Joints and Zero-Force Members Problem Statement for Example 6
6. Determine the force in each member of the truss and
state whether the force is tension or compression.
2 kip
A
B C D
E F
G
H
I
J
K
L
16 ft 16 ft
14 ft

912
6.2 Trusses: Method of Joints and Zero-Force Members Problem Statement for Example 7
7. Determine the force in each member and state
whether the force is tension or compression.
4 kN
A
B
C
D
E F G
H
I
J
2 m 2 m
60

913
6.2 Trusses: Method of Joints and Zero-Force Members Example 1, page 1 of 3
Free-body diagram of entire truss. Calculating the reactions
is a good place to start because they are usually easy to
compute, and they can be used in the equilibrium equations
for the joints where the reactions act.
3 m
5 m
C
B
A
10 kN
1
Equilibrium equations for entire truss
F

x = 0: A

x + 10 kN = 0 (1)
F

y
= 0: A

y
+ B

y
= 0 (2)
M

A
= 0: (10 kN)(3 m) + B

y
(5 m) = 0 (3)
Solving these equations simultaneously gives
A

x = 10 kN, A

y
= 6 kN, and B

y
= 6 kN
2
+
+
+
Ax
A
y
B
y
3 m
5 m
C
B
A
10 kN
1. Determine the force in each member of the truss and
state whether the force is tension or compression.

914
6.2 Trusses: Method of Joints and Zero-Force Members Example 1, page 2 of 3
Equilibrium equations for joint C. It is a good idea
to assume all members in tension (forces point away
from the joint, not towards it). Then, after solving
the equilibrium equations, you will know
immediately that any member force found to be
negative must be compression.
F

x = 0: 10 kN F

AC
sin = 0 (4)

F

y
= 0: F

AC
cos F

BC
= 0 (5)
4
+
+
10 kN
C
F
BC
F
AC
5 m
3 m
A
B
C
Geometry 5 Free-body diagram of joint C 3
= tan
-1
( ) = 59.04
5
3
Using = 59.04 in Eqs. 4 and 5 and solving
simultaneously gives
F

AC
= 11.66 kN (T) Ans.
and
F

BC
= 6.0 kN = 6.0 kN (C) Ans.
Writing "(T)" after the numerical value shows that
the member is in tension. We had arbitrarily
assumed member BC to be in tension. We then
found that the member force was negative, so we
know that our assumption was wrong. Member BC
is in compression, and we show this by writing a
positive "6.0" followed by "(C)".
6

915
6.2 Trusses: Method of Joints and Zero-Force Members Example 1, page 3 of 3
B
y
= 6 kN
B
F
BC
= 6 kN
F
AB
A
C
B
10 kN
6.0 (C)
0
1
1
.
6
6

(
T
)
Free-body diagram of joint B 7
An "Answer diagram" summarizes the analysis
of the entire truss (All forces are in kN).
9
10 Equilibrium equation for joint B
F

x = 0: F
AB
= 0
Solving gives
F
AB
= 0 Ans.
The force F
BC
is directed toward
the joint because member BC is
known to be in compression.
8
6
6
10
+

916
6.2 Trusses: Method of Joints and Zero-Force Members Example 2, page 1 of 7
2. Determine the force in each member of the truss and state
whether the force is tension or compression.
2 kip 2 kip 4 kip
14 ft
10 ft 10 ft 10 ft 10 ft
A
B C
D
E
F
G H

917
6.2 Trusses: Method of Joints and Zero-Force Members Example 2, page 2 of 7
H G F
E
D
C B
A
10 ft 10 ft 10 ft 10 ft
14 ft
4 kip 2 kip 2 kip
Ax
A
y
E
y
Free-body diagram of entire truss.
Calculating the reactions is usually a
good way to start the analysis.
1
Equilibrium equations for entire truss
F

x = 0: A

x = 0 (1)
F

y
= 0: A

y
+ E

y
kip kip kip = 0 (2)
M

A
= 0: 2 kip)(10 ft) (4 kip)(20 ft) (2 kip)(30 ft) + E

y
(40 ft) = 0 (3)
Solving simultaneously gives
A

x = 0, A

y
= 4.0 kip, and E

y
= 4.0 kip.
2
+
+
+

918
6.2 Trusses: Method of Joints and Zero-Force Members Example 2, page 3 of 7
Free-body diagram of joint E. This joint is
chosen because only two unknown forces are
present. Thus we know that we can solve for
these forces because two equations of
equilibrium are available for the joint. Note
also that we assume that both unknown forces
are in tension (directed away from the joint).
Using = 54.46 in Eqs. 4 and 5 and solving
simultaneously gives
F

DE
= 2.857 kip (T) Ans.
F

EH
= 4.916 kip = 4.916 kip (C) Ans.
We arbitrarily assumed member EH to be in tension. We
then found that the member force was negative, so we
know that our assumption was wrong. Member EH is in
compression, and we show this by writing a positive
"4.916" followed by "(C)".
Equilibrium equations for joint E
F

x = 0: F

DE
F

EH
cos = 0 (4)
F

y
= 0: F

EH
sin 4 kip = 0 (5)
= tan
-1
( ) = 54.46
Geometry
E
y
= 4 kip
E
+
+
F
EH
F
DE
H
E
D
14 ft
10 ft
3
6
7
4
5
14
10

919
6.2 Trusses: Method of Joints and Zero-Force Members Example 2, page 4 of 7
Use a free-body diagram of joint H next because only
two member forces are unknown.
F

EH
= 4.916 kip (C)
F

DE
= 2.857 kip (T)
Equilibrium equations for joint H
F

x = 0: F

GH
(4.916 kip) cos 54.46 = 0 (6)
F

y
= 0: F

DH
+ (4.916 kip) sin 54.46 = 0 (7)
Solving simultaneously gives
F

GH
= 2.858 kip = 2.858 kip (C) Ans.
and
F

DH
= 4.0 kip (T) Ans.
As before, we assume
that the unknown
member forces (GH and
DH in this instance) are
tension, so are directed
away from the joint.
The force in member
EH has already been
found to be 4.916 kip
compression, so it is
directed towards the
joint, not away from it.
Free-body diagram of joint H
F
A
G H
B C
D
E
2 kip
2 kip
4 kip
H
= 54.46
F

EH
= 4.916 kip (C)
F
GH
F
DH
8 9
10
11
+
+

920
6.2 Trusses: Method of Joints and Zero-Force Members Example 2, page 5 of 7
4 kip
2 kip
2 kip
E
D C B
H G
A
F
F

EH
= 4.916 kip (C)
F

DE
= 2.857 kip (T)
Use a free-body
diagram of joint
D because only
two member
forces are
unknown.
Free-body diagram of joint D
As before, we assume that the unknown
member forces are tension, so are directed away
from the joint. The forces in members DH and
DE have already been found to be tension and
so are directed away from the joint.
F
GH
= 2.858 kip(C)
F
DE
= 2.857 kip
F
DG
F
DH
= 4.0 kip
F
CD
2 kip
= 54.46
12
13
14
+
+
15 Equilibrium equations for joint D
F

x = 0: F

CD
F

DG
cos(54.46) + 2.857 kip = 0 (8)
F

y
= 0: F

DG
sin(54.46) + 4.0 kip 2 kip = 0 (9)
Solving simultaneously gives
F

CD
= 4.286 kip (T) Ans.
and
F

DG
= 2.458 kip = 2.458 kip (C) Ans.

921
6.2 Trusses: Method of Joints and Zero-Force Members Example 2, page 6 of 7
F

DE
= 2.857 kip (T)
F

EH
= 4.916 kip (C)
F
A
G H
B C
D
E
2 kip
2 kip
4 kip
F
GH
= 2.857 kip(C)
F
DG
= 2.458 kip (C)
F
DH
= 4.0 kip (T)
F
CD
= 4.286 kip (T)
Use a free-body diagram of
joint C because only two
member forces are
unknown.
Free-body diagram of joint C
The unknown forces in
members CG and BC are
assumed to be tension, so
are directed away from
the joint. The force in
member CD has already
been found to be 4.286
kip (T).
Equilibrium equations for joint C.
F

x = 0: F

BC
+ 4.286 kip = 0 (10)
F

y
= 0: F

CG
4 kip = 0 (11)
Solving gives
F

BC
= 4.286 kip (T) Ans.
and
F

CG
= 4 kip (T) Ans.
4 kip
C
F
CG
F
CD
= 4.286 kip (T)
F
BC
16
17
18
19
+
+

922
6.2 Trusses: Method of Joints and Zero-Force Members Example 2, page 7 of 7
H G F
E
D
C B
A
4 2 2
All remaining bar forces follow from symmetry.
Answer diagram
4
.
9
2

(
C
)
2.86 (C) 2.86 (C)
4
.
0
0

(
T
)
4
.
0

(
T
)
4
.
0

(
T
)
4
.
9
2

(
C
)
2
.
4
6

(
C
)
4.29 (T) 4.29 (T)
2
.
4
6

(
C
)
2.86 (T)
2.86 (T)
All forces in kips.
20
4 4

923
6.2 Trusses: Method of Joints and Zero-Force Members Example 3, page 1 of 4
3. Determine the force in each member of
the truss and state whether the force is
tension or compression.
10 ft
12 ft
60
30
400 lb
900 lb
A
B C
D
E

924
6.2 Trusses: Method of Joints and Zero-Force Members Example 3, page 2 of 4
900 lb
400 lb
10 ft
12 ft
60
30
B C
D
E
Ex
E
y
Because AB is a two-force member, the line of
action of F
AB
must pass through A and B.
Free body-diagram of entire truss
Equilibrium equations for entire truss
F

x = 0: F

AB
sin 60 + E

x + (900 lb) cos 30 = 0 (1)
F

y
= 0: F

AB
cos 60 + E

y
+ (900 lb) sin 30 400 lb = 0 (2)
M
C
= 0: (400 lb)(10 ft + F

AB
cos 60(10 ft) + E

x(12 ft) = 0 (3)
Solving simultaneously gives

F

AB
= 347.8 lb
E

x = 478.2 lb,
E

y
= 123.9 lb
2
1
3
+
+
+
F
AB

925
6.2 Trusses: Method of Joints and Zero-Force Members Example 3, page 3 of 4
Free-body diagram of joint D. Joint D is chosen
because only two member forces are unknown there.
Equilibrium equations for joint D
F

x = 0: F

DE
= 0 (4)
F

y
= 0: F

BD
400 lb = 0 (5)
Solving gives
F

BD
= 400 lb (T) Ans.
F

DE
= 0 Ans.
Free-body diagram of joint C. Joint C is chosen because
only two member forces are unknown there.
Equilibrium equations for joint C
F

x = 0: F

BC
+779.4 lb = 0 (6)
F

y
= 0: F

CE
+ 450 lb = 0 (7)
Solving gives
F

BC
= 779.4 lb (T) Ans.
F

CE
= 450.0 lb (T) Ans.
D
400 lb
F
DE
F
BD
F
CE
C
F
BC
(900 lb) sin 30 = 450.0 lb
(900 lb) cos 30= 779.4 lb
4
5
6
7
8
9
The unknown forces
have been assumed to
be tension.
The unknown
forces have
been assumed
to be tension.
+
+
+
+

926
6.2 Trusses: Method of Joints and Zero-Force Members Example 3, page 4 of 4
Free-body diagram of joint B. Only one
member force is unknown at this joint.
Equilibrium equations for joint B
F

x =0: F

BE
cos (347.8 lb) sin 60
+ 779.4 lb = 0 (8)
= tan
-1
( )
= 50.19
Geometry
Using = 50.19 in Eq. 8 and then solving gives
F

BE
= 746.9 lb = 747 lb (C) Ans.
Answer diagram (all forces in lb)
B
F
AB
= 347.8 lb
F
BD
= 400 lb
F
BE
F
BC
= 779.4 lb
60
124
478
E
D
C B
60
400
30
900 lb
C B
E
10 ft
12 ft
400 (T)
779 (T)
450 (T)
747 (C)
10
11
12
13
14
+
10
12
0
347.8 lb

927
6.2 Trusses: Method of Joints and Zero-Force Members Example 4, page 1 of 9
30
4. Determine the force in each member of the truss and state whether
the force is tension or compression. The truss is symmetric.
2 kip
2 kip
2 kip
2 kip
2 kip
6 ft 6 ft 6 ft 6 ft 6 ft 6 ft
A
B C
D
E
F
G
H
I J
K
60
60

928
6.2 Trusses: Method of Joints and Zero-Force Members Example 4, page 2 of 9
60
K
J I
H
G
F
E
D
C B
A
6 ft 6 ft 6 ft 6 ft 6 ft 6 ft
2 kip
2 kip
2 kip
2 kip
2 kip
Ax
A
y
D
y
Free-body diagram of entire truss
Equilibrium equations for entire truss
F

x = 0: A

x = 0
F

y
= 0: A

y
+ D

y
2 kip
2 kip 2 kip 2 kip 2 kip = 0
M

A
= 0: 2(kip)(6 ft) 2 kip(12 ft)
2(kip)(18 ft) 2 kip(24 ft)
2(kip)(30 ft) + D

y
(36 ft) = 0
Solving simultaneously gives
A

x = 0
A

y
= 5 kip
D

y
= 5 kip
+
+
+
2
1
30 60

929
6.2 Trusses: Method of Joints and Zero-Force Members Example 4, page 3 of 9
D
y
= 5 kip
A
y
= 5 kip
Ax = 0
2 kip
2 kip
2 kip
2 kip
2 kip
A
B C
D
E
F
G
H
I J
K
60
Use a free-body diagram of joint A because
only two unknown member forces are
present.
Free-body diagram of joint A.
Equilibrium equations for joint A
F

x = 0: F

AB
+ F

AE
cos 30 = 0

F

y
= 0: F

AE
sin 30 + 5 kip = 0
Solving simultaneously gives
F

AB
= 8.660 kip (T) Ans.
F

AE
= 10 kip = 10 kip (C) Ans.
A
y
= 5 kip
A
30
F
AE
F
AB
3
4
5
+
+
30 60

930
6.2 Trusses: Method of Joints and Zero-Force Members Example 4, page 4 of 9
Three unknown member forces are present at joint I, but
two of them, F

EI
and F

IK
, are collinear, so summing
forces perpendicular to F

EI
and F

IK
would give an
equation with F

FI
as the only unknown.
Free-body diagram of joint I
Geometry of members at joint I
Equilibrium equations for joint I
F

y
= 0: F

FI
sin 90 (2 kip) sin 60 = 0
Solving gives
F

FI
= 1.732 = 1.732 kip (C) Ans.
= 30 + 60
= 90
So member FI is
perpendicular to the x axis.
Thus the member force F

FI

lies on the y axis.
60
K
J I
H
G
F
E
D
C B
A
2 kip
2 kip
2 kip
2 kip
2 kip
Ax = 0
A
y
= 5 kip
D
y
= 5 kip
2 kip
I
y
x
F
IK
F
EI
F
FI
30
60 E
I
A
F
30
= 60
60
6
7
8
9
+
30 60
60
x
y

931
6.2 Trusses: Method of Joints and Zero-Force Members Example 4, page 5 of 9
Use the same technique at
joint F as was used at joint I:
sum forces perpendicular to
collinear members BF and FK.
Geometry of members at joint F
Free-body diagram of joint F
Equilibrium equations for joint F
F

y
= 0: F

EF
sin 60 (1.732 kip) sin 60 = 0
Solving simultaneously gives
F

EF
= 1.732 kip (T) Ans.
60
K
J I
H
G
F
E
D
C B
A
2 kip
2 kip
2 kip
2 kip
2 kip
Ax = 0
A
y
= 5 kip
D
y
= 5 kip
F
E
60
60
I
B
60
= 180 (60 + 60)
= 60
10
11
12
13
F

FI
= 1.732 kip (C)
(already known)
F
60
y
x
F

FI
= 1.732 kip (C)

F
BF
F
EF
F
FK
60
+
30 60

932
6.2 Trusses: Method of Joints and Zero-Force Members Example 4, page 6 of 9
F

AE
= 10.0 kip (C)
(already known)
At joint E, now only two member
forces, F

EI
and F

EB
, are unknown.
Free-body diagram of joint E
Equilibrium equations for joint E
F

x = 0: (10 kip) cos 30 + F

EI
cos 30 + F

BE
cos 30 + 1.732 kip = 0
F

y
= 0: (10 kip) sin 30 + F

EI
sin 30 F

BE
sin 30 2 kip = 0
Solving simultaneously gives
F

EI
= 9.0 kip = 9.0 kip (C) Ans.
F

BE
= 3.0 kip = 3.0 kip (C) Ans.
60
K
J I
H
G
F
E
D
C B
A
2 kip
2 kip
2 kip
2 kip
2 kip
Ax = 0
A
y
= 5 kip
D
y
= 5 kip
F

EF
= 1.732 kip (T)
(already known)
2 kip
E
60
30
30
30
F

EF
= 1.732 kip (T)

F
BE
F
EI
F
AE
= 10 kip(C)
14
15
16
+
+
30 60

933
6.2 Trusses: Method of Joints and Zero-Force Members Example 4, page 7 of 9
60
K
J I
H
G
F
E
D
C B
A
2 kip
2 kip
2 kip
2 kip
2 kip
Ax = 0
A
y
= 5 kip
D
y
= 5 kip
F

BE
= 3 kip (C)
(already known)
F

AB
= 8.660 kip (T)
(already known)
Free-body diagram of joint B
At joint B, now only two member forces, F

BF
and
F

BC
, are unknown.
Equilibrium equations for joint B
F

x = 0: 3 kip) cos 30 8.660 kip + F

BF
cos 60 + F

BC
= 0
F

y
= 0: 3 kip) sin 30 + F

BF
sin 60 = 0
Solving simultaneously gives
F

BF
= 1.732 kip (T) Ans.
F

BC
= 5.196 kip (T) Ans.
B
60 30
F
BC
F
BF
F
BE
= 3 kip(C)
F

AB
= 8.660 kip (T)
17
18
19
30 60
+
+

934
6.2 Trusses: Method of Joints and Zero-Force Members Example 4, page 8 of 9
The remaining unknown member forces, F

IK
and F

FK
,
can be found by re-using the free-body diagrams of
joints I and F.
Free-body diagram of joint I
Equilibrium equations for joint I
F

x = 0: 9.0 kip (2 kip) cos 60 + F

IK
= 0
Solving gives
F

IK
= 8.0 kip = 8.0 kip (C) Ans.
Free-body diagram of joint F
Equilibrium equation for joint F
F

x = 0: F

FK
1.732 kip (1.732 kip) cos 60
(1.732 kip) cos 60 = 0
Solving gives
F

FK
= 3.464 kip (T) Ans.
F

FI
= 1.732 kip (C)
F

EF
= 1.732 kip (T)
F

BF
= 1.732 kip (T)
F

EI
= 9.0 kip (C)
F
FI
= 1.732 kip (C)
F
IK
x
y
I
2 kip
60
F
FK
y
F
x
20
22
23
24
21
+
+

935
6.2 Trusses: Method of Joints and Zero-Force Members Example 4, page 9 of 9
K
J I
H
G F
E
D
C
B
A
By symmetry, all forces on the right half of
the truss are also known.
Answer diagram
All forces in kips
2
2
2
2
2
8.66 (T) 8.66 (T) 5.20 (T)
1
0
.
0
0

(
C
)
9
.
0
0

(
C
)
8
.
0
0

(
C
)
8
.
0
0

(
C
)
9
.
0
0

(
C
)
1
0
.
0
0

(
C
)
3
.
0
0

(
C
)
1.732 (T)
1.732 (T)
1
.
7
3
2

(
T
)
1
.
7
3
2

(
T
)
3
.
0
0

(
C
)
1
.
7
3
2

(
T
)
1
.
7
3
2

(
T
)
3
.
4
6

(
T
)
3
.
4
6

(
T
)
25
5 5

936
6.2 Trusses: Method of Joints and Zero-Force Members Example 5, page 1 of 8
5. Determine the force in each member of the truss and
state whether the force is tension or compression.
4 ft 2 ft 4 ft
2.5 ft
1 ft
100 lb
A
B
C
D
E
F

937
6.2 Trusses: Method of Joints and Zero-Force Members Example 5, page 2 of 8
Free-body diagram of entire truss.
Equilibrium equations for entire truss
F

x = 0: A

x + 100 lb = 0
F

y
= 0: A

y
+ F

y
= 0
M

A
= 0: 100 lb)(1 ft + 2.5 ft) + F

y
(4 ft +2 ft + 4 ft) = 0
Solving these equations simultaneously gives
A

x = 100 lb
A

y
= 35 lb
F

y
= 35 lb
F
E
D
C
B
A
100 lb
1 ft
2.5 ft
4 ft 2 ft 4 ft
Ax
A
y
F
y
1
2
+
+
+

938
6.2 Trusses: Method of Joints and Zero-Force Members Example 5, page 3 of 8
Free-body diagram
of entire truss.
Only two unknown member
forces act at joint F. A

x = 100 lb
F

y
= 35 lb
Free-body diagram of joint F.
Equilibrium equations for joint F
F

x = 0: F

EF
cos F

DF
sin = 0
F

y
= 0: F

EF
sin + F

DF
cos + 35 lb = 0
Geometry
Solving the equilibrium equations with
= 32.01 and = 48.81 gives
F

EF
= 165.10 lb (T) Ans.
F

DF
= 186.03 lb = 186.03 lb (C) Ans.
= tan
-1
( ) = 32.01
= tan
-1
( ) = 48.81
A
y
= 35 lb
2.5 ft
1 ft
100 lb
A
B
C
D
E
F
2 ft 4 ft 4 ft
F
F

y
= 35 lb
F
EF
F
DF
E
F
4 ft
3.5 ft
2.5 ft
4 ft
D
+
+
3
5
6
7
8
4
2.5 ft
4 ft
3.5 ft
4 ft

939
6.2 Trusses: Method of Joints and Zero-Force Members Example 5, page 4 of 8
Free-body diagram of joint A
By symmetry, the angles and at joint A are the
same as we calculated at joint F. Thus
= 32.01
= 48.81
Equilibrium equations for joint A
F

x = 0: 100 lb + F

AC
cos + F

AB
sin = 0
F

y
= 0: F

AC
sin + F

AB
cos 35 lb = 0
Solving the above equations with = 32.01
and = 48.81 gives
F

AC
= 247.70 lb (T) Ans.
F

AB
= 146.23 lb = 146.23 lb (C) Ans.
A
A
y
= 35 lb
A

x = 100 lb

+
+
10
11
F

AC
F

AB

9
4 ft 4 ft 2 ft
F
E
D
C
B
A
100 lb
1 ft
2.5 ft
A
y
= 35 lb F

y
= 35 lb
A

x = 100 lb
Only two unknown member
forces act at joint A.
Free-body diagram
of entire truss.

940
6.2 Trusses: Method of Joints and Zero-Force Members Example 5, page 5 of 8
Free-body diagram of entire truss
Only two unknown member
forces act at joint D.
F

DF
= 186.03 lb (C)
(already known)
Free-body diagram of joint D
Equilibrium equations for joint D.
F

x = 0: F

BD
(186.03 lb) sin 48.81 = 0
F

y
= 0: F

DE
+ (186.03 lb) cos 48.81 = 0
Solving these equations gives
F

BD
= 139.99 lb = 139.99 lb (C) Ans.
F

DE
= 122.51 lb (T) Ans.
F
E C
A
100 lb
A
y
= 35 lb
F

y
= 35 lb
A

x = 100 lb
B D
D
48.81
48.81
F
DE
F
BD
F

DF
= 186.03 lb (C)
13
14
15
+
+
12

941
6.2 Trusses: Method of Joints and Zero-Force Members Example 5, page 6 of 8
Equilibrium equations for joint C.
F

x = 0: (247.70 lb) cos 32.01 + F

CE
= 0
F

y
= 0: (247.70 lb) sin 32.01 + F

BC
= 0
Solving these equations gives
F

CE
= 210.04 lb (T) Ans.
F

BC
= 131.23 lb (T) Ans.
Free-body diagram of joint C.
F

AC
= 247.70 lb (T)
Only two unknown member
forces act at joint C
F

AC
= 247.70 lb (T)
(already known)
Free-body diagram of entire truss
A

x = 100 lb
F

y
= 35 lb
A
y
= 35 lb
100 lb
A
C E
F
17
18
19
C
32.01
+
+
F
BC
F
CE
16
D B

942
6.2 Trusses: Method of Joints and Zero-Force Members Example 5, page 7 of 8
Free-body diagram of entire truss
F

DE
= 122.51 lb (T)
(already known)
F

EF
= 165.10 lb (T)
(already known)
F

CE
= 210.04 lb (T)
(already known)
At joint E, member BE
is the only unknown
member force.
Free-body diagram of joint E.
Equilibrium equation for joint E.
F

x = 0: F

BE
cos 210.04 lb + (165.10 lb) cos 32.01 = 0
Geometry
Substituting = 26.57 in the equation for joint E
and solving gives
F

BE
= 78.31 lb = 78.31 lb (C) Ans.
= tan
-1
( ) = 26.57
A

x = 100 lb
F

y
= 35 lb
A
y
= 35 lb
100 lb
A
C E
F
B D
E
32.01
32.01
F

DE
= 122.51 lb (T)
F

EF
= 165.10 lb (T)
F

CE
= 210.04 lb (T)
F
BE
C
E
B
2 ft
1 ft
21
22
23
24
25
+
20
2 ft
1 ft

943
6.2 Trusses: Method of Joints and Zero-Force Members Example 5, page 8 of 8
35 35
100
100
A
B
C
D
E
F
140 (C)
210 (T)
123 (T)
1
8
6

(
C
)
1
6
5

(
T
)
131 (T)
1
4
6

(
C
)
2
4
8

(
T
)
7
8
(
C
)
Answer diagram
All forces in lb
26

944
6.2 Trusses: Method of Joints and Zero-Force Members Example 6, page 1 of 9
6. Determine the force in each member of the truss and state
whether the force is tension or compression.
2 kip
A
B C D
E F
G
H
I
J
K
L
16 ft 16 ft
14 ft

945
6.2 Trusses: Method of Joints and Zero-Force Members Example 6, page 2 of 9
Free-body diagram of entire truss
Equilibrium equations for entire truss
F

x = 0: A

x = 0
F

y
= 0: A

y
+ G

y
2 kip = 0
M

A
= 0: 2 kip(16 ft) + G

y
(16 ft + 16 ft) = 0
Solving simultaneously gives
A

x = 0
A

y
= 1 kip
G

y
= 1 kip.
L
K
J
I
H
G
F
E
D C B
A
2 kip
1
2
+
+
+
14 ft
16 ft 16 ft
Ax
A
y
G
y

946
6.2 Trusses: Method of Joints and Zero-Force Members Example 6, page 3 of 9
Free-body
diagram of joint G
Only two unknown member
forces act at joint G.
Equilibrium equations for joint G
F

x = 0: F

FG
F
GL
cos = 0
F

y
= 0: F
GL
sin + 1 kip = 0
Solving simultaneously gives

F

FG
= 1.143 kip (T) Ans.
F
GL
= 1.518 kip = 1.518 kip (C) Ans.
= tan
-1
( ) = 41.19
Geometry
G

y
= 1 kip
F
GL
F

FG
A

y
= 1 kip
2 kip
A
G
H
I
J
K
L
16 ft 16 ft
D
J
G
G

y
= 1 kip
G
14 ft
16 ft
4
3
5
6
7
+
+
B C D
E
F
14 ft
14 ft
16 ft

947
6.2 Trusses: Method of Joints and Zero-Force Members Example 6, page 4 of 9
16 ft 16 ft
L
H
G A
A

y
= 1 kip
G

y
= 1 kip
K
J
I
2 kip
14 ft
At joint F, no external forces
act, three members meet, and
two of these members are
collinear. So FL is a
zero-force member, as will
now be shown.
Free-body diagram of joint F
F

FG
= 1.143 kip
(already known)
Equilibrium equation for joint F
F

y
= 0: F
FL
sin = 0
Since sin 0, it follows that
F
FL
= 0 Ans.
F
F
FL
F
EF
8
10 9
+
B C D
E
F

948
6.2 Trusses: Method of Joints and Zero-Force Members Example 6, page 5 of 9
14 ft
2 kip
I
J
K
A

y
= 1 kip
A G
H
L
16 ft 16 ft
At joint L, no external forces act, three
members meet, and two of these
members are collinear. So EL is a
zero-force member:
F
EL
= 0 Ans.
Member LF has been omitted
because it is a zero-force member.
12
11
B C D
E
F
G

y
= 1 kip

949
6.2 Trusses: Method of Joints and Zero-Force Members Example 6, page 6 of 9
16 ft 16 ft
L
H
G A
A

y
= 1 kip
K
J
I
2 kip
14 ft
Consideration of joint E shows that EK is a zero-force member:
F
EK
= 0 Ans.
But then consideration of joint K shows that DK is also a
zero-force member:
F
DK
= 0 Ans.
Members EL and FL have been omitted
because they are zero-force members.
14 13
B C D
E
F
G

y
= 1 kip

950
6.2 Trusses: Method of Joints and Zero-Force Members Example 6, page 7 of 9
Because of symmetry, the members in
the left half of the truss must also be
zero-force and so can be omitted, too.
Consideration of joint D shows that DJ
must be a zero-force member:
F

DJ
= 0 Ans.
All zero-force members in the right
half of the truss have been omitted.
You cannot conclude that member
DJ is a zero-force member by
looking at end J. (Instead, look at
end D.)
L
H
G
A
A

y
= 1 kip
K
J
I
2 kip
16
18
15
17
B C D
E
F
G

y
= 1 kip

951
6.2 Trusses: Method of Joints and Zero-Force Members Example 6, page 8 of 9
Free-body diagram of joint L
We have previously shown that
F
GL
= 1.518 kip (C) Ans.
Consideration of free-body diagrams of K and L
show that
F
KL
= F
KJ
= 1.518 kip (C) Ans.
We have previously shown that
F

FG
= 1.143 kip (T) Ans.
Consideration of free-body diagrams of all joints in the lower
chord, B, C, D, E, and F, shows that all member forces there
must equal 1.143 kip (T).
Zero-force member DJ
has been omitted.
Free-body diagram joint F
By symmetry,
F

AH
= 1.518 kip = F

HI
= F

IJ
Ans
21
20
2 kip
I
J
K
A

y
= 1 kip
A
G
H
L
L
F
KL
F
KL
= F
GL
F
GL
23
F
F
EF
= F
FG
22
24
19
B C D
E
F
G

y
= 1 kip

952
6.2 Trusses: Method of Joints and Zero-Force Members Example 6, page 9 of 9
L
K
J
I
H
G A
2
0
0 0
0
0
0
0
0
0
1 1
1.143 (T)
1.518 (C)
1.518 (C)
Answer diagram
All forces in kips
25
B F C
D
E

953
6.2 Trusses: Method of Joints and Zero-Force Members Example 7, page 1 of 7
7. Determine the force in each member and
state whether the force is tension or
compression.
Equilibrium equations for entire truss
F

x = 0: A

x = 0
F

y
= 0: A

y
+ C

y
4 kN = 0
M

A
= 0: ( 4 kN)(2 m) + C

y
(2 m + 2 m) = 0
Solving simultaneously gives
A

x = 0, A

y
= 2 kN, and C

y
= 2 kN.
4 kN
A
B
C
D E F
G
H I J
2 m 2 m
60
60
J I
H
G
F E
D
C
B A
4 kN
Free-body diagram
of entire truss
1
2
Ax
A
y
C
y
+
+
+
2 m 2 m

954
6.2 Trusses: Method of Joints and Zero-Force Members Example 7, page 2 of 7
C
y
= 2 kN
A
y
= 2 kN
Ax
4 kN
A B
C
D
E F G
H
I
J
60
Free-body diagram of entire truss.
Two members meet at joint J, they are not collinear,
and no external force acts at the joint, so members IJ
and FJ must be zero-force members.
Free-body diagram of joint J
Equilibrium equations for joint J
F

x = 0: F

IJ
F

FJ
cos 60 = 0
F

y
= 0: F

FJ
sin 60 = 0
Solving simultaneously gives
F

IJ
= 0 Ans.
F

FJ
= 0 Ans.
J
60
3
4
5
F
FJ
F
IJ
+
+

955
6.2 Trusses: Method of Joints and Zero-Force Members Example 7, page 3 of 7
Two members meet at the joint, they are not
collinear, and no external forces act, so the
members carry zero force.
Members IJ and FJ have been omitted
because they are zero-force members.
The same argument at G shows FG and
CG are zero-force members.
C
y
= 2 kN A
y
= 2 kN
Ax
4 kN
A B
C
D
E F
G
H
I
J
7
6
8

956
6.2 Trusses: Method of Joints and Zero-Force Members Example 7, page 4 of 7
All members identified
as zero-force have been
omitted.
Free-body diagram of joint C
Equilibrium equations for joint C
F

x = 0: F

BC
F

CF
cos 60 = 0
F

y
= 0: F

CF
sin 60 + 2 kN = 0
Solving simultaneously gives
F

CF
= 2.309 kN = 2.309 kN (C) Ans.
F

BC
= 1.155 kN (T) Ans.
By symmetry,
F

AE
= 2.309 kN (C)
F

AB
= 1.155 kN (T)
11
I
F E
C
B
A
4 kN
A
y
= 2 kN
C
y
= 2 kN
C
y
= 2 kN
C
60
12
10
9
+
+
F

BC
F

CF

957
6.2 Trusses: Method of Joints and Zero-Force Members Example 7, page 5 of 7
B
Free-body diagram of joint B
F

AB
= 1.155 kN (T)
F

BC
= 1.155 kN (T)
Equilibrium equations for joint B
F

x = 0: F

BE
cos 60 + F

BF
cos 60 1.155 kN + 1.155 kN = 0
F

y
= 0: F

BE
sin 60 + F

BF
sin 60 = 0
Solving simultaneously gives
F
BE
= 0 Ans
F

BF
= 0 Ans.
60 60
F
BE
F
BF
14
13
+
+
C
y
= 2 kN
A
y
= 2 kN
4 kN
A
B
C
E F
I
F

AB
= 1.155 kN (T)
(already known)
F

BC
= 1.155 kN (T)
(already known)

958
6.2 Trusses: Method of Joints and Zero-Force Members Example 7, page 6 of 7
17
C
y
= 2 kN
A
y
= 2 kN
4 kN
A
B
C
E
F
I
F
y
x
Zero-force members
BE and BF have
been omitted.
At joint F, no external force
acts, three members meet, and
two of these members are
collinear, so member EF is a
zero-force member.
Free-body diagram of joint F
F

EF
= 0
(zero-force member)
F

CF
= 2.309 kN (C)
Equilibrium equations for joint F
F

y
= 0: F

FI
+ 2.309 kN (C) = 0
Solving gives
F

FI
= 2.309 kN = 2.309 kN (C) Ans.
Then by symmetry
F
EI
= F
FI
= 2.309 kN (C) Ans.
F
FI
18
16
15
+

959
6.2 Trusses: Method of Joints and Zero-Force Members Example 7, page 7 of 7
2 2
4
A B
C
D
E F G
H
I
J
Answer diagram
All forces in kN
0 0
0
0 0
0 0
0
0
2
.
3
1

(
C
)
2
.
3
1

(
C
)
2
.
3
1

(
C
)
2
.
3
1

(
C
)
1.155 (T)
1.155 (T)
19

960

6.3 Trusses: Method of Sections
961
6.3 Trusses: Method of Sections Procedures and Strategies, page 1 of 2
A
B C
D
E
F
2 kN 2 kN
Section
A
B
E
2 kN
F
EF
F
EC
F
BC
R
Ay
Find from the free-body diagram
of the entire truss.
Procedures and Strategies for Solving Problems
Involving Trusses: the Method of Sections
1. Pass a section through members where member forces
are to be calculated.
2. Draw the free-body diagram of the portion of the truss
on one side of the section (You can decide which side to
choose by noting which side will be easier to analyze, for
example, which side has fewer forces acting on it). On the
diagram, be sure to include the (unknown) forces from the
members through which the section passed.
3. Write equations of equilibrium for the portion of the
truss that you have isolated in the free-body diagram.
4. If support reaction-forces appear in the equations of
equilibrium, then you must draw a free-body diagram of
the entire truss, write equilibrium equations and solve for
the support reactions. You can then use these values in the
equations you wrote in Step 3.

962
6.3 Trusses: Method of Sections Procedures and Strategies, page 2 of 2
A
B
C
D
E
F
4 kN
A
B
D
E
F
BC
F
BE
F
AE
F
DE
R
Dy
Find from the free-body
diagram of the entire truss.
M
E
= 0
F
BC
is the only
unknown in the
equation.
Section
Notes:
a) Look for special situations in which you can save
work. For example, if two of three unknown forces at
a section are parallel, then summing forces in a
direction perpendicular to the two parallel forces will
give an equation involving only one unknown the
non-parallel force. Similarly, if the lines of action of
two unknown forces at a section intersect, then
summing moments about the point of intersection
will eliminate these forces from the moment equation.
b) The method of sections is ideally suited for
problems in which only a small number (one to three)
member forces are to be determined. The method can
be used, however, in combination with the method of
joints to analyze complicated trusses.
c) Sections need not be straight lines. Sometimes using
curved sections is advantageous.

963
6.3 Trusses: Method of Sections Problem Statement for Example 1
5 m
C
D
E
F
G H
5 m
5 m
B
3 m
A
1. Determine the force in members AC, CD, and DF, and
state whether the force is tension or compression.
4 kN
2 kN
6 kN

964
6.3 Trusses: Method of Sections Problem Statement for Example 2
2. Determine the force in members CD, CH, and GH, and state whether the force is tension or compression.
12 ft 12 ft 12 ft 12 ft
800 lb 800 lb 800 lb
A
F
G
H
B
C D
E
8 ft

965
6.3 Trusses: Method of Sections Problem Statement for Example 3
4 m 4 m 4 m
3. The diagonal members are not connected to each other where they cross. Determine the force
in members BG, CF, and FG, and state whether the force is tension or compression.
E
3 kN
F G
H
D
C B A
3 kN 3 kN
2.5 m

966
6.3 Trusses: Method of Sections Problem Statement for Example 4
3 m
1.5 m
3 m
6 m
3 m
3 m
3 m
4. Determine the force in members CE, EF, HF, and CF, and state whether the force is tension or compression.
I
G
E
C
A
B
D
F
H
J
6 kN
4 kN

967
6.3 Trusses: Method of Sections Problem Statement for Example 5
5. Determine the force in members RS, LS, FL, and EF, and state whether the force is tension or compression.
N O P Q R S T
H I J K L
M
A
B C D E F
G
2 m
3 m
2 m
3 m 3 m 3 m 3 m 3 m
4 kN 4 kN
4 kN

968
6.3 Trusses: Method of Sections Problem Statement for Example 6
5 m
5 m
T
N
A
B
C D E F G H I J K L
M
O P
Q
R S
U
V W
X
10 kN
12 panels @ 4 m each
6. Determine the force in members TU, EF, and EU. State whether the force is tension or compression.

969
6.3 Trusses: Method of Sections Problem Statement for Example 7
7. Determine the force in
members KM, LM, and
DK. State whether the
force is tension or
compression.
2 m 2 m 2 m 2 m 2 m 2 m 2 m 2 m 2 m 2 m
I
6 m
2 kN
2 kN
2 kN
2 kN
2 kN
2 kN
2 kN
H
L
M
K J
I
G
F
E D C B
A

970
6.3 Trusses: Method of Sections Problem Statement for Example 8
5 ft 5 ft 5 ft 5 ft 5 ft
J I H G
F
E D
C B
A
Cable
8. Determine the force in members GH,
CD, and CH. State whether the force is
tension or compression. Also, find the
tension in the cable.
5 kip
3 ft
30

971
6.3 Trusses: Method of Sections Example 1, page 1 of 2
5 m
C
D
E
F
G
H
5 m
5 m
B
3 m
A
1. Determine the force in members AC, CD, and DF, and
state whether the force is tension or compression.
1 Pass a section through the three
members whose forces are to be
determined.
A
B
C D
E
F
G
H
4 kN
2 kN
6 kN
6 kN
2 kN
4 kN

972
6.3 Trusses: Method of Sections Example 1, page 2 of 2
F

x = 0: 2 kN + 4 kN + F

CD
= 0
F

y
= 0: F

AC
F

DF
= 0
M

D
= 0: F

AC
(3 m) (2 kN)(5 m + 5 m)
(4 kN)(5 m) = 0
Solving simultaneously gives
F

AC
= 13.33 kN (T) Ans.
F

CD
= 6.0 kN = 6.0 kN (C) Ans.
F

DF
= 13.33 kN = 13.33 kN (C) Ans.
+
+
+
We had assumed member CD to be in tension.
Calculations showed that F

CD
is negative, so our
assumption was wrong: CD must be in compression.
Similarly DF must be in compression.
5
2 kN
4 kN
C
Equations of equilibrium for the portion of the
truss (Note that moments are summed about
point D, even though point D is not part of the
free body):
4
F

AC
F

CD
F

DF
At each cut through a
member, a force is shown
to represent the effect of
the portion of the member
on one side of the section
pulling on the portion on
the other side. It is
convenient to always
assume the force to be
tension.
3
Free-body diagram of portion of truss above the section 2
3 m
5 m
H
G
F
E
D
5 m

973
6.3 Trusses: Method of Sections Example 2, page 1 of 3
2. Determine the force in members CD, CH, and GH, and state whether the force is tension or compression.
12 ft 12 ft 12 ft 12 ft
800 lb 800 lb 800 lb
A
F
G
H
B
C D
E
800 lb
A
B
800 lb
C
F
G
D
800 lb
H
E
1 Pass a section through the three
members whose forces are to be
determined.
8 ft

974
6.3 Trusses: Method of Sections Example 2, page 2 of 3
D
800 lb
H
E
F

GH
F

CH
F

CD
E

y
2 Free-body diagram of portion of truss to right of section
At each cut through a member, a force is shown. 3
4 Equations of equilibrium for the portion of the
truss:
F

x = 0: F

GH
F

CH
sin F

CD
= 0 (1)
F

y
= 0: F

CH
cos 800 lb + E

y
= 0 (2)
M

H
= 0: F

CD
(8 ft) + E

y
(12 ft) = 0 (3)
12 ft
+
+
+
5
Three equations but four unknowns, so another
equation is needed.
8 ft
12 ft
6
Geometry
= tan
-1
= 56.31
H
D
C
8 ft
12 ft
8 ft

975
6.3 Trusses: Method of Sections Example 2, page 3 of 3
800 lb
12 ft 12 ft
7
A

y
B
800 lb
C
Free-body diagram of entire truss.
F
G
12 ft
D
800 lb
H
E

y
E
A

x
A
12 ft
8
+
9
Equilibrium equation for entire truss. This will give the needed fourth equation.
M

A
= 0: (800 lb)(12 ft) (800 lb)(2 12 ft) (800 lb)(3 12 ft) + E

y
(4 12 ft) = 0
Solving gives E

y
= 1,200 lb.
Substituting E

y
= 1,200 lb into Eqs. 1, 2, and 3 and solving simultaneously gives
F

CD
= 1,800 lb (T) Ans.
F

CH
= 721 lb (T) Ans.
F

GH
= 2,400 lb = 2,400 lb (C) Ans.
8 ft

976
6.3 Trusses: Method of Sections Example 3, page 1 of 3
4 m 4 m 4 m
3. The diagonal members are not connected to each other where they cross. Determine the force
in members BG, CF, and FG, and state whether the force is tension or compression.
E
3 kN
F G
H
D
C B A
3 kN 3 kN
B A
3 kN
3 kN
E
F
C
3 kN
G
D
H
1 Pass a section through the three members
whose forces are to be determined.
2.5 m

977
6.3 Trusses: Method of Sections Example 3, page 2 of 3
F

FG
F

BG
F

CF
2
3
At each cut through a member, a force is shown
4 m
4 Equations of equilibrium for the portion of the truss:
F

x = 0: F

FG
F

BG
sin F

CF
sin = 0 (1)
F

y
= 0: F

BG
cos + F

CF
cos + D

y
3 kN = 0 (2)
M

G
= 0: F

CF
sin (2.5 m) + D

y
(4 m) = 0 (3)

+
5
Three equations but four unknowns, so
another equation is needed.
4 m
C
G
F
6
= tan
-1
= 58.0
Geometry
D

y
+
+
2.5 m
2.5 m
2.5 m
4 m
G
3 kN
C
Free-body diagram of portion of truss to right of section
D
H

978
6.3 Trusses: Method of Sections Example 3, page 3 of 3
Free-body diagram of entire truss (This will give the needed fourth equation).
3 kN
3 kN
E

x
7
F
E
A B
E

y
D

y
3 kN
H
G
C
D
8
+
Equilibrium equation for entire truss.
M

E
= 0: (3 kN)(4 m) (3 kN)(2 4 m) + D

y
(3 4 m) = 0
Solving gives D

y
= 3.0 kN. Then substituting = 58.0 and D

y
= 3.0 kN into Eqs. 1, 2, and 3 and solving
simultaneously gives
F

BG
= 5.66 kN (T) Ans.
F

CF
= 5.66 kN (T) Ans.
F

FG
= 9.6 kN = 9.6 kN (C) Ans.
2.5 m
4 m 4 m 4 m

979
6.3 Trusses: Method of Sections Example 4, page 1 of 4
3 m
1.5 m
3 m
6 m
3 m
3 m
3 m
4. Determine the force in members CE, EF, HF, and CF, and state whether the force is tension or compression.
I
G
E
C
A
B
D
F
H
J
6 kN
4 kN
6 m
4 kN
6 kN
Pass a section through at
least some of the members
whose forces are to be
determined. The general
idea is to choose as few
members as possible --three
in this instance-- because
each time a member is cut
by a section, an additional
unknown is introduced into
the equilibrium equations.
1
I
J
H G
E
F
C
D
B
A

980
6.3 Trusses: Method of Sections Example 4, page 2 of 4
3 m
3 m
I
G
E
F
H
J
6 kN
4 kN
L

GH
L

EF
2 Free-body diagram of portion of truss above section (Using
the upper portion of the truss rather than the lower
eliminates the need to calculate the reactions at the bottom
of the truss).
3 Equations of equilibrium for the portion of the truss:
M
G
= 0: (6 kN)(3 m) + F
EF
(3 m) F

FH
cos (L

GH
) = 0 (1)
M

F
= 0: (6 kN)(2 3 m) 4 kN)(3 m) + F

CE
cos (L

EF
) = 0 (2)
F

x = 0: F

CE
sin + F

EF
+ F

FH
sin + 4 kN + 6 kN = 0 (3)
+
+
+
F

EF
F

CE
F

FH

981
6.3 Trusses: Method of Sections Example 4, page 3 of 4
E
C
D
G
I
F
H
J
B
A
4 Geometry
5
3 m
3 m
3 m
3 m
3 m
3 m
3 m
3 m
3 m
3 m
3 m
3 m
3 m
(6 m) tan
(3 m) tan
= tan
-1
= tan
-1
= 7.125
L

GH
= 3 m + (3 m) tan + (3 m) tan = 3.75 m
L

EF
= 3 m + (6 m) tan + (6 m) tan = 4.50 m
6 Substituting these values for , L
GH
, and L
EF
into Eqs. 1, 2, and
3 and solving simultaneously gives:
F

CE
= 10.75 kN Ans.
F

EF
= kN = 7.33 kN (C) Ans.
F

FH
= 10.75 kN = 10.75 kN (C) Ans.
B' 1.5 m 1.5 m
1.5 m
4 3 m JB'
BB'

982
6.3 Trusses: Method of Sections Example 4, page 4 of 4
8 Equilibrium equations for joint F
F

x = 0: F

CF
cos + F

DF
sin 7.125 + (10.75 kN)(sin 7.125) 7.33 kN = 0 (4)
F

y
= 0: F

CF
sin F

DF
cos 7.125 (10.75 kN)(cos 7.125) = 0 (5)
+
+
L

EF
= 4.50 m
(3 m) tan
E
F
D
C
9 Geometry
= tan
-1
= 31.61
10 Substituting = 31.608 into Eqs. 4 and 5 and solving simultaneously gives:
F

DF
= 14.97 kN = 14.97 kN (C)
F

CF
= 7.99 kN (T) Ans.
F

EF
= 7.33 kN (C)
F

CF
F

DF
F

FH
= 10.75 kN (C)
3 m
(3 m) tan 7.125 + 4.50 m
3 m
= 7.125
7 Free-body diagram of joint F.
This free body will enable us to
calculate the remaining unknown
force the force in member CF.
F

983
6.3 Trusses: Method of Sections Example 5, page 1 of 4
G
M
1 Pass a section through the four
members whose forces are to be
determined. It does not appear
possible to find a section that cuts
only three of these members.
5. Determine the force in members RS, LS, FL, and EF, and state whether the force is tension or compression.
N O P Q R S T
H I J K L
M
A
B C D E F
G
2 m
3 m
2 m
3 m 3 m 3 m 3 m 3 m
4 kN 4 kN
4 kN
C
N O
A
H
B
I L
P Q R S
4 kN 4 kN
D
J
4 kN
E
F
K
T

984
6.3 Trusses: Method of Sections Example 5, page 2 of 4
3 m
F
S
2 m
G
T
M
2 m
2 Free body diagram of truss portion to right of section line
F

RS
F

LS
F

FL
F

EF
G

y
3 Equations of equilibrium for the portion of the truss:
M

S
= 0: F

EF
(2 2 m) + G

y
(3 m) = 0 (1)
M

F
= 0: F

RS
(2 2 m) + G

y
(3 m) = 0 (2)
F

y
= 0: F

FL
F

LS
+ G

y
= 0 (3) +
+
+
Three equations with five unknowns so two more
equations are needed.
4

985
6.3 Trusses: Method of Sections Example 5, page 3 of 4
4 kN
3 m
P
3 m
A
3 m
B C
N
H
O
I J
3 m 3 m
D
4 kN
3 m
4 kN
E
F
Q S R
K L
2 m
G
T
M
2 m
5 Free-body diagram of entire truss (This free body will enable us to calculate the reaction at G).
6 Equation of equilibrium for the entire truss.
M

A
= 0: (4 kN)(2 3 m) (4 kN)(3 3 m) (4 kN)(4 3 m) + G

y
(18 m) = 0 (4)
Solving gives
G

y
= 6 kN
Substituting G

y
= 6 kN into Eqs. 1 and 2 and solving gives:
F

EF
= 4.5 kN (T) Ans.
F
RS
= 4.5 kN = 4.5 kN (C) Ans.
A

x
A

y G

y
+

986
6.3 Trusses: Method of Sections Example 5, page 4 of 4
S
7 Free-body diagram of joint S. This free body will
enable us to calculate the force in member LS.
F

LS
F

MS
F

ST
F

RS
= 4.5 kN (C)
8 Equations of equilibrium for joint S. Note that there
are three unknowns but only two equations.
F

x = 0: 4.5 kN + F

ST
+ F

MS
cos = 0 (5)
F

y
= 0: F

LS
F

MS
sin = 0 (6)
+
+
9 Geometry
S T
M
2 m
3 m
= tan
-1
= 33.69
10 Free body diagram of joint T
F

ST
F

MT
T
11 Two members meet at joint T, they are not collinear and no
external force acts at joint T, so members ST and MT are zero-
force members.
Substituting F

ST
= 0 in Eq. 5 and solving Eqs. 5 and 6
simultaneously gives:
F

MS
= 5.41 kN = 5.41 kN (C)
F

LS
= 3.0 kN (T) Ans.
12 Substituting F

LS
= 3.0 kN and G

y
= 6 kN into Eq. 3 and
solving gives:
F

FL
= 3.0 kN = 3.0 kN (C) Ans.
2 m
3 m

987
6.3 Trusses: Method of Sections Example 6, page 1 of 4
5 m
5 m
T
N
A
B
C D E F G H I J K L
M
O P
Q
R S
U
V W
X
10 kN
12 panels @ 4 m each
6. Determine the force in members TU, EF, and EU. State whether the force is tension or compression.
S
Q
R P O
N
A
C
B E D
10 kN
H G F J I
M
L K
T
U V W
X
1 Even though we were not asked to determine the force in member EP,
we have to pass the section through it because we must make the
section go completely through the truss.

988
6.3 Trusses: Method of Sections Example 6, page 2 of 4
T
F

EF
F

EP
F

EU
F

TU
A

y
A

x
5 m
5 m
4 m 4 m 4 m 4 m
2 Free-body diagram of portion of the truss to the left of the section
3 Equations of equilibrium for the portion of the truss:
F

x = 0: A

x + F

TU
+ F

EP
cos + F

EF
= 0 (1)
F

y
= 0: A

y
+ F

EU
+ F

EP
sin = 0 (2)
M

E
= 0: A

y
(4 4 m) F

TU
(2 5 m) = 0 (3)
+
+
+
4 Three equations with six unknowns so three more
equations are needed.
5 Geometry
P
F
E
4 m
5 m
4 m
5 m
= tan
-1
= 51.34
O
N
A
B
C D E

989
6.3 Trusses: Method of Sections Example 6, page 3 of 4
12 panels @ 4 m each
C
B
A
N
T
R
Q
P O
10 kN
G F E D J I H
S
L K
M
X
U V W
6 Free-body diagram of entire truss (This free body will enable us to calculate the reactions at support A).
A

x
A

y
M

y
7 Equations of equilibrium for entire truss. Note that we
only write two equations because we only need to
calculate Ax and Ay, since only Ax and Ay appear in
Eqs. 1, 2, and 3.
F

x = 0: A

x = 0 (4)
M

M
= 0: (10 kN)(6 4 m) A

y
(12 4 m) = 0 (5)
Solving gives A

x = 0 and A

y
= 5 kN.
Consideration of joint F shows that member FP is a zero-force
member, so F

FP
= 0.
But if member FP is removed (because it is a zero-force
member), consideration of joint P shows that member EP is also
a zero-force member, so F

EP
= 0.
8
+
+

990
6.3 Trusses: Method of Sections Example 6, page 4 of 4
9 Substituting
= 51.34,
A

x = 0,
A

y
= 5 kN,
and
F

EP
= 0
into Eqs. 1, 2, and 3, and solving gives:
F

TU
= 8 kN = 8 kN (C) Ans.
F

EF
= 8 kN (T) Ans.
F

EU
= 5 kN = 5 kN (C) Ans.

991
6.3 Trusses: Method of Sections Example 7, page 1 of 6
7. Determine the force in
members KM, LM, and
DK. State whether the
force is tension or
compression.
1 We choose a section that cuts at least some of the
members whose forces are to be determined. But the
section should cut as few other members as possible,
since each time a member is cut, an additional
unknown appears in the equilibrium equations.
L
2 kN
2 kN
A
G
2 kN
2 kN
B C
I
D
K J
M
E
H
2 kN
2 kN
F
2 kN
I
2 m 2 m 2 m 2 m 2 m 2 m 2 m 2 m 2 m 2 m
I
6 m
2 kN
2 kN
2 kN
2 kN
2 kN
2 kN
2 kN
H
L
M
K J
I
G
F
E D C B
A

992
6.3 Trusses: Method of Sections Example 7, page 2 of 6
E D
K
L
2 kN
F
H
2 kN
2 kN
F

KM
F

ML
F

CD
F

y
2 Free-body diagram of portion of truss to right of section.
It is not essential but we can save some work if we use
the principle of transmissibility as shown in Step 3.
F

CD
D
K
F

y
2 kN
E
H
L
2 kN
F
2 kN
F

KM
cos
F

KM
sin
F

ML
cos
F

ML
sin
3 Same free body as in Step 2, but now the force F

KM
has been
moved along its line of action to joint D (principle of
transmissibility) and then expressed in terms of vertical and
horizontal components. Similarly F

ML
is moved to joint F.
4 Equations of equilibrium for free body in Step 3. Note that
because we were not asked to determine F

CD
, we choose two
moment equations in which F

CD
does not appear.
M

F
= 0: (2 kN)(2 m) + (2 kN)(3 2 m) F

KM
sin (4 2 m) = 0 (1)
M

D
= 0: (2 kN)(2 m) (2 kN)(3 2 m) (2kN)(4 2 m) + F

y
(4 2 m) + F

ML
sin (4 2 m) = 0 (2)
5 Two equations but three unknown forces, so another
equilibrium equation is needed.
+
+
2 m 2 m 2 m 2 m

993
6.3 Trusses: Method of Sections Example 7, page 3 of 6
6 m
6 Geometry
= tan
-1
(

2 m
) = 71.56
= tan
-1
(

2 m + 8 m
) = 30.96
2 m
8 m
6 m
6 m
M
D
F

994
6.3 Trusses: Method of Sections Example 7, page 4 of 6
C D
I
2 kN
L
J K
M
2 kN
F
E
F

y
2 kN
H
2 kN
8 Equilibrium equation for entire truss
M

A
= 0: (2 kN)(2 m) (2 kN)(3 2 m) (2 kN)(5 2 m)
(2 kN)(7 2 m) (2 kN)(9 2 m) (2 kN)(10 2 m) + F

y
(10 2 m) = 0 (3)
9 Solving gives F

y
= 7 kN. Substituting F

y
= 7 kN, = 71.56, and
= 30.96 in Eqs. 1 and 2, and solving simultaneously gives:
F

KM
= 2.11 kN (T) Ans.
F

ML
= 5.83 kN = 5.83 kN (C) Ans.
+
I
2 m 2 m 2 m 2 m 2 m 2 m 2 m 2 m 2 m 2 m
Free-body diagram of entire
truss. This free body will
enable us to calculate the
reaction at support F.
7
2 kN
2 kN
A A

x
A

y
2 kN
G
B

995
6.3 Trusses: Method of Sections Example 7, page 5 of 6
F

DK
10 Free-body diagram of joint K (This free body will enable
us to calculate the force in member DK).
11 Since there are only two unknown forces, F

KL
and F

DK
,
we could write force-equilibrium equations in the x
and y directions and then solve them simultaneously.
However, we can save work by noticing that a
zero-force member is present.
2 kN
L
2 kN
A
G
2 kN
B C
I
D
K J
2 kN
2 kN
M
E
H
2 kN
2 kN
F
A

x
A

y
F

y
12 Free- body diagram of entire truss
13 Consideration of joint K shows that KL must
be a zero-force member, so F

KL
= 0.
I
K
y
x
F

KM
= 2.11 kN (T)
F

KL

996
6.3 Trusses: Method of Sections Example 7, page 6 of 6
14 Free-body diagram of joint K (repeated)
F

y
= 2.11 kN F

DK
= 0 (4)
Solving gives
F

DK
= 2.11 kN (T) Ans.
+
F

KM
= 2.11 kN (T)
y
F

KL
F

DK
K
x

997
6.3 Trusses: Method of Sections Example 8, page 1 of 3
A
B
F
C
D E
G H I
Cable
J
30
1 The section must pass through the
cable. Otherwise the portion of the
truss to the left of the section could
not be isolated as a free body.
5 ft 5 ft 5 ft 5 ft 5 ft
J I H G
F
E D
C B
A
Cable
8. Determine the force in members GH,
CD, and CH. State whether the force is
tension or compression. Also, find the
tension in the cable.
5 kip
3 ft
5 kip
30

998
6.3 Trusses: Method of Sections Example 8, page 2 of 3
5 kip
A
B
F
C
G
2 Free-body diagram of portion of truss to left of section
T
F

GH
F

CH
F

CD
3 The tension in the cable is
one of the unknowns.
4 Equations of equilibrium for the portion of the truss:
M

C
= 0: T cos 30(3 ft) F

GH
(3 ft) + (5 kip)(2 5 ft) = 0 (1)
M

G
= 0: (5 kip)(2 5 ft) + F

CD
(3 ft) + F

CH
sin (3 ft) = 0 (2)
F

y
= 0: 5 kip + T sin 30 + F

CH
cos 0 (3) +
+
+
C
G
H
5 Geometry
3 ft
5 ft
= tan
-1
(

3 ft
) = 59.04
6 Three equations but four
unknown forces, so another
equilibrium equation is needed.
3 ft
5 ft 5 ft
5 ft
30

999
6.3 Trusses: Method of Sections Example 8, page 3 of 3
J

x
8
5 kip
7
Equation of equilibrium for the entire truss. Only one equation is
used because we need to calculate T only; the reactions at J are not
needed.
M

J
= 0: (5 kip)(5 5 ft) T sin 30(3 5 ft) = 0 (4)
T = 16.67 kip Ans.
Substituting = 59.04 and T = 16.67 kip into Eqs. 1, 2, and 3 and
solving simultaneously gives:
F

GH
= 2.23 kip (T) Ans.
F

CD
= 11.11 kip = 11.11 kip (C) Ans.
F

CH
= 6.48 kip = 6.48 kip (C) Ans.
+
9
5 ft 5 ft 5 ft 5 ft 5 ft
3 ft
30
E
Free-body diagram of entire truss
A
B
F
C D
G
T
H
J

y
I J

1000

6.4 Space Trusses
1001
6.4 Space Trusses Procedures and Strategies, page 1 of 1
Procedures and Strategies for Solving Problems
Involving Space Trusses
Space truss problems can be solved by the same
methods the method of joints and the method of
sections as are used in solving planar trusses.
The only difference are 1) There are three rather
than two equilibrium equations for each joint; and
2) in planar truss problems you can sometimes save
work by looking for a point through which the
lines of action of several unknown force pass and
then summing moments about this point because
the unknown forces will not appear in the moment
equation. However, in space truss problem, you
look for a line through which the lines of action of
several unknown forces pass, and then you sum
moments about the line, rather than about a point.

1002
6.4 Space Trusses Problem Statement for Example 1
1. The truss is supported by short links at B and D and by a
ball and socket at C. Determine the force in each member,
and state whether the force is tension or compression.
A
B
D
C
y
z
5 m
4 m 4 m
7 m
200 N
2 m
x

1003
6.4 Space Trusses Problem Statement for Example 2
y
z
3 m
3 m
4 m
5 m
x
700 N
4 m
A
B
C
D
E
2. Determine the force in each member of the space truss, and
state whether the force is tension or compression. The truss is
supported by short links at C and D and by ball-and-socket
supports at A and E.

1004
6.4 Space Trusses Problem Statement for Example 3
D
3. The truss is supported by ball-and-socket
joints at A, C, D, and F. Determine the force
in each member and state whether the force
is tension or compression.
x
y
z
10 ft
8 ft
6 ft
B
C
F
E
D
A
5 kip
4 kip
2 kip

1005
6.4 Space Trusses Problem Statement for Example 4
5 m
4 m
8 m
z
y
x
A
800 N
400 N
2 m
B
C
D
4. Determine the force in each member of the space
truss. State whether the force is tension or
compression. The supports at C and D are short
links; at A a ball-and-socket support is present.

1006
6.4 Space Trusses Problem Statement for Example 5
3 m 5 m 3 m
4 m
2 m
4 m
x
y
z
800 N 800 N
800 N 800 N
A
B
C
D
E
F
G
H
8 m
5. The truss is supported by ball-and-socket
joints at A, B, C, and D. Determine the forces
in members FE and EC. State whether the
forces are tension or compression.

1007
6.4 Space Trusses Example 1, page 1 of 6
1. The truss is supported by short links at B and D and by a
ball and socket at C. Determine the force in each member,
and state whether the force is tension or compression.
A
B
D
C
y
z
5 m
4 m 4 m
7 m
200 N
2 m
x

1008
6.4 Space Trusses Example 1, page 2 of 6
A
F
AD F
AB
F
AC
At joint A, only three unknown forces are
present, so begin the analysis there.
Free-body diagram of joint A.
Equilibrium equation
F
AD
+ F
AC
+ F
AB
{200j}N= 0 (1)
1
2
200 N
A
B
D
C
y
z
5 m
4 m 4 m
7 m
200 N
2 m
x

1009
6.4 Space Trusses Example 1, page 3 of 6
Express the forces in component form.
F
AD
= F
AD
unit vector pointing from A to D
= F
AD
= F
AD
( 0.4216i + 0.5270j 0.7379k) (2)
F
AC
= F
AC
(0.4216i + 0.5270j 0.7379k) (3)
F
AB
= F
AB
= F
AB
(0.3939j 0.9191k) (4)
4i + 5j 7k
4
2
+ 5
2
+ 7
2
3j 7k
3
2
+ 7
2
3
F
AC
and F
AD
differ only in sign of x component
A
B
D
C
y
z
5 m
4 m 4 m
7 m
200 N
2 m
x

1010
6.4 Space Trusses Example 1, page 4 of 6
Substitute the component forms of the force vectors into Eq.1:
F
AD
+ F
AC
+ F
AB
200j = 0 (Eq. 1 repeated)
0.4216F
AD
i + 0.5270F
AD
j 0.7379F
AD
k
+ 0.4216F
AC
i + 0.5270F
AC
j 0.7379F
AC
k)
+ 0.3939F
AB
j 0.9191F
AB
k 200j = 0
Or, in scalar form,
Fx = 0: 0.4216F
AD
+ 0.4216F
AC
= 0 (5)
F
y
= 0: 0.5270F
AD
+ 0.5270F
AC
+ 0.3939 F
AB

200 = 0 (6)
Fz = 0: 0.7379F
AD
0.7379F
AC

0.9191F
AB
= 0 (7)
Solving gives
F
AD
= 474.3616 N (T) Ans. (8)
F
AC
= 474.3616 N (T) Ans. (9)
F
AB
= 761.5773 N = 761.5773 N (C) Ans. (10)
4

1011
6.4 Space Trusses Example 1, page 5 of 6
B
F
BA
= F
AB
(Force of member AB on joint B is equal
and opposite to force on joint A.)
= F
AB
(0.9191j 0.3939k), by Eq. 4
= (300j 700k) N
F
BD
Free-body diagram of joint B. Now that the force
in member AB is known, the member forces F
BD

and F
BC
can be found.
761.5773 N, by Eq. 10
5
Bzk (Reaction force)
4i + 2j
4
2
+ 2
2
F
BC
= F
BC
= F
BC
(0.8944i + 0.4472j)
4i + 2j
4
2
+ 2
2
F
BD
= F
BD
= F
BD
( 0.8944i + 0.4472j)
6 Summing the forces in the x and y directions gives
Fx = 0: F
BC
(0.8944) F
BD
(0.8944) = 0
F
y
= 0: F
BC
(0.4472) + F
BD
(0.4472) + 300 = 0
7
+
+
Solving gives
F
BC
= 335.4102 N = 335.4102 N (C) Ans.
F
BD
= 335.4102 N = 335.4102 N (C) Ans. (11)
A
B
D
C
y
z
5 m
4 m 4 m
7 m
200 N
2 m
x

1012
6.4 Space Trusses Example 1, page 6 of 6
Dzk
F
DA
= F
AD
= F
AD
( 0.4216i + 0.5270j 0.7379k), by Eq. 2
474.3616 N, by Eq. 8
= (200i 250j + 350k) N
Reaction forces
D
Free-body diagram of joint D. Now that the force
in member BD is known, the remaining unknown
member force F
DC
can be found.
Summing the x components of the forces acting on
joint D gives
Fx = 0: 200 300 + F
DC
= 0
Solving gives
F
DC
= 100 N (T) Ans.
8
9
+
F
DC
= F
DC
i
D
y
j
F
DB
= F
BD

= ( 335.4102)( 0.8944i + 0.4472j)
= 300i + j
by Eq. 11
A
B
D
C
y
z
5 m
4 m 4 m
7 m
200 N
2 m
x

1013
6.4 Space Trusses Example 2, page 1 of 4
y
z
3 m
3 m
4 m
5 m
x
700 N
4 m
A
B
C
D
E
D
F
DE
F
DB
Dzk
(Reaction force
from link at D)
2. Determine the force in each member of the space truss, and
state whether the force is tension or compression. The truss is
supported by short links at C and D and by ball-and-socket
supports at A and E.
Only three unknowns two member forces and a
reaction force are present at D, so a free-body
diagram of D is a good place to start.
Free-body diagram of joint D
Equilibrium equation
F
DB
+ F
DE
{700j}N+ Dzk = 0 (1)
1
700 N

1014
6.4 Space Trusses Example 2, page 2 of 4
Express the forces F
DB
and F
DE
in component form.
F
DB
= F
DB
unit vector pointing from D to B
= F
DB
= F
DB
( 0.7071i + 0.5657j 0.4243k) (2)
F
DE
= F
DE
i (3)
Substitute the component form of the force vectors in Eqs. 2
and 3 into Eq. 1:
F
DB
+ F
DE
700j + Dzk = 0 (Eq. 1 repeated)
or
0.7071F
DB
i + 0.5657 F
DB
j 0.4243F
DB
k
F
DE
i 700j + Dzk = 0
In scalar form,
Fx = 0: 0.7071F
DB
F
DE
= 0
F
y
= 0: 0.5657F
DB
700 = 0
Fz = 0: 0.4243F
DB
+ Dz

= 0
5i + 4j 3k
5
2
+ 4
2
+ 3
2
Solving gives
F
DB
= 1237.4369 N (T) Ans. (4)
F
DE
= 875 N

= 875 N (C) Ans. (5)
Dz = 525 N
2
3
y
z
3 m
3 m
4 m
5 m
x
4 m
A
B
C
D
E
700 N

1015
6.4 Space Trusses Example 2, page 3 of 4
B
F
BA
F
BE
F
BC
F
BD
Now that the force in member DB is known,
there are only three unknowns at joint B.
Free-body diagram of joint B
Equilibrium equation
F
BE
+ F
BC
+ F
BA
+ F
BD
= 0 (6)
Express each force in component form
F
BE
= F
BE
= F
BE
( 0.8j + 0.6k) (7)
F
BC
= F
BC

= F
BC
( 0.8j 0.6k) (8)
4j + 3k
4
2
+ 3
2

4j 3k
4
2
+ 3
2

F
BA
= F
BA
i (9)
F
BD
= F
DB
= F
DB
( 0.7071i + 0.5657j 0.4243k), by Eq. 2
= {875i 700j + 525k} N (10)
1237.4369 N, by Eq. 4
Force of member
BD on joint B is
equal and opposite
force of member
BD on joint D.
4
5
y
z
3 m
3 m
4 m
5 m
x
700 N
4 m
A
B
C
D
E

1016
6.4 Space Trusses Example 2, page 4 of 4
Substitute the component form of the force vectors in Eq. 7-10
into Eq. 6:
F
BE
+ F
BC
+ F
BA
+ F
BD
= 0 (Eq. 6 repeated)
or,
0.8F
BE
j + 0.6F
BE
k 0.8F
BC
j 0.6F
BC
k
F
BA
i 875i 700j + 525k = 0
In scalar form,
Fx = 0: F
BA
+
875 = 0
F
y
= 0: 0.8F
BE
0.8F
BC

700 = 0
Fz = 0: 0.6F
BE

0.6F
BC

525 = 0
Solving gives
F
BE
= 875 N = 875 N (C) Ans.
F
BA
= 875 N (T) Ans.
F
BC
= 0 Ans.
6
y
z
F
EC
F
BC
= 0
C
y
j
x, C
3 m
4 m
Free-body diagram of joint C.
Fz = 0: F
EC
+ 0 = 0
Thus
F
EC
= 0 Ans.
7
B

1017
6.4 Space Trusses Example 3, page 1 of 6
D
3. The truss is supported by ball-and-socket
joints at A, C, D, and F. Determine the force
in each member and state whether the force
is tension or compression.
x
y
z
10 ft
8 ft
6 ft
B
C
F
E
D
A
5 kip
4 kip
2 kip

1018
6.4 Space Trusses Example 3, page 2 of 6
4
6 ft
B
5 kip
4 kip
2 kip
F
BA
F
BE
F
BC
1
At joint B, only three unknown forces are
present, so begin the analysis there.
Free-body diagram of joint B
2
Equilibrium equation:
F
BA
+ F
BE
+ F
BC
+ {5i} kip {4j} kip
+ 2k} kip = 0 (1)
3
D
x
y
z
10 ft
8 ft
6 ft
B
C
F
E
D
A
5 kip
4 kip
2 kip
Express the forces in component form
F
BA
= F
BA
j (2)
F
BE
= F
BE
k (3)
F
BC
= F
BC
(4)
= F
BC
(0.8575i + 0.5145j) (5)
10i + 6j
10
2
+ 6
2


1019
6.4 Space Trusses Example 3, page 3 of 6
Substitute the component form of the force vectors in Eqs.
2-5 into Eq. 1:
F
BA
+ F
BE
+ F
BC
+ 5i 4j
+ 2k = 0 (Eq. 1 repeated)
F
BA
j F
BE
k + 0.8575F
BC
i + 0.5145F
BC
j
+ 5i 4j + 2k = 0
In scalar form,
Fx = 0: 0.8575F
BC
+ 5 = 0 (6)
F
y
= 0: F
BA
+ 0.5145F
BC
4 = 0 (7)
Fz = 0: F
BE
+ 2 = 0 (8)
Solving gives
F
BA
= 7 kips (T) Ans. (9)
F
BC
= 5.8310 kip (T)

= 5.8310 kip (C) Ans. (10)
F
BE
= 2 kip (T) Ans. (11)
5

1020
6.4 Space Trusses Example 3, page 4 of 6
Equilibrium equation
F
EB
+ F
ED
+ F
EF
+ F
EC
= 0 (12)
6 ft
D
x
y
z
10 ft
8 ft
6 ft
B
C
F
E
D
A
6
5 kip
4 kip
2 kip
E
F
ED
F
EF
F
EB
F
EC
Free-body diagram of joint E. Now that the force
in member BE is known, the forces in members
ED, EC, and EF can be found.

1021
6.4 Space Trusses Example 3, page 5 of 6
5 kip
4 kip
2 kip
x
y
z
10 ft
8 ft
6 ft
B
C
F
E
D
A
Express each force in component form.
F
EB
= force in direction opposite to F
BE
= F
BE
= { 2}k kip (13)
F
ED
= F
ED
j (14)
F
EF
= F
EF
= F
EF
(0.8575i + 0.5145j) (15)
F
EC
= F
EC
= F
EC
(0.7071i + 0.4243j
+ 0.5657k) (16)
by Eqs. 3 and 11
10i + 6j
10
2
+ 6
2

10i + 6j + 8k
10
2
+ 6
2
+ 8
2
7

1022
6.4 Space Trusses Example 3, page 6 of 6
Substitute the component form of the force vector in Eqs. 13-16 into Eq. 12:
F
EB
+ F
ED
+ F
EF
+ F
EC
= 0 (Eq. 12 repeated)
2k + F
ED
j + 0.8575F
EF
i + 0.5145F
EF
j + 0.7071F
EC
i
+ 0.4243F
EC
j + 0.5657F
EC
k = 0
In scalar form,
Fx = 0: 0.8575F
EF
+ 0.7071F
EC
= 0
F
y
= 0: F
ED
+ 0.5145F
EF
+ 0.4243F
EC
= 0
Fz = 0: 2 + 0.5657F
EC
= 0
Solving gives
F
EC
= 3.54 kip = 3.54 kip (C) Ans.
F
ED
= 0 Ans.
F
EF
= 2.92 kip (T) Ans.
8

1023
6.4 Space Trusses Example 4, page 1 of 7
5 m
4 m
8 m
z
y
x
A
2 m
B
C
D
4. Determine the force in each member of the space
truss. State whether the force is tension or
compression. The supports at C and D are short
links; at A a ball-and-socket support is present.
800 N
400 N

1024
6.4 Space Trusses Example 4, page 2 of 7
B
F
BA
F
BD
F
BC
Joint B is the only joint with no more
than three unknown forces, so begin
with a free-body diagram of joint B.
Free-body diagram of joint B
1
2
Equilibrium equation
F
BA
+ F
BD
+ F
BC
{800j}N + (400k}N= 0 (1)
5 m
4 m
8 m
z
y
x
A
2 m
B
C
D
800 N
400 N
800 N
400 N

1025
6.4 Space Trusses Example 4, page 3 of 7
Express the forces in component form.
F
BA
= F
BA
unit vector pointing from B to A
= F
BA
= F
BA
( 0.3714i 0.9285j) (2)
F
BC
= F
BC
= F
BC
(0.7682i 0.6402j) (3)
F
BD
= F
BD
= F
BD
(0.6838i 0.5698j + 0.4558k) (4)
2i 5j
2
2
+ 5
2

6i 5j
6
2
+ 5
2

6i 5j + 4k
6
2
+ 5
2
+ 4
2
3
5 m
4 m
8 m
z
y
x
A
2 m
B
C
D
800 N
400 N

1026
6.4 Space Trusses Example 4, page 4 of 7
Substituting the component form of the forces in Eqs. 2, 3, and 4 into
Eq. 1 gives
F
BA
+ F
BD
+ F
BC
800j + 400k = 0 (Eq. 1 repeated)
or, in scalar form,
Fx = 0: 0.3714F
BA
+ 0.7682F
BC
+ 0.6838F
BD
= 0 (5)
F
y
= 0: 0.9285F
BA
0.6402F
BC
0.5698F
BD
800 = 0 (6)
Fz = 0: 0.4558 F
BD
+ 400 = 0 (7)
Solving Eqs. 5, 6, and 7 simultaneously gives
F
BA
= 646.2198 N = 646.2198 N (C) Ans. (8).
F
BC
= 468.6150 N (T) Ans. (9)
F
BD
= 877.4964 N = 877.4964 N (C) Ans. (10)
4
5

1027
6.4 Space Trusses Example 4, page 5 of 7
C
F
CB
= F
BC

= (468.6150 N)(0.7682i 0.6402j), by Eq. 3
= 360i + 300j
C
y
j
F
CD
k
F
CA
i
Now that the force in member BC is known, there are
only three unknowns at joint C.
Free-body diagram of joint C
(Reaction from link C)
6
Equilibrium equations:
Fx = 0: F
CA
360 = 0
F
y
= 0: C
y
+ 300 = 0
Fz = 0: F
CD
= 0
Solving simultaneously gives
F
CA
= 360 N
= 360 N (C) Ans. (11)
F
CD
= 0 Ans. (12)
C
y
= 300 N (13)
5 m
4 m
8 m
z
y
x
A
2 m
B
C
D
7
800 N
400 N

1028
6.4 Space Trusses Example 4, page 6 of 7
Dxi
Reactions from links
8i 4k
8
2
+ 4
2
F
DA
= F
DA
= F
DA
( 0.8944i 0.4472k ) (16)
12
9
10
800 N
400 N
D
D
y
j
F
DA
F
DB
F
CD
= 0
Now that the forces in members CD and BD are known,
a free-body diagram of joint D will give an equation for
the remaining unknown member force, F
DA
.
Free-body diagram of joint D
Equilibrium equation:
F
DA
+ F
DB
+ Dxi + D
y
j = 0 (14)
Express the forces in component form.
F
DB
= F
BD

= F
BD
(0.6838i 0.5698j + 0.4558k) (Eq. 4 repeated)
= {600i 500j + 400k} N (15)
8
11
5 m
4 m
8 m
z
y
x
A
2 m
B
C
D
877.4964 N, by Eq. 10

1029
6.4 Space Trusses Example 4, page 7 of 7
Substitute the component form of the force vectors in Eqs. 15 and
16 into Eq. 14:
F
DA
+ F
DB
+ Dxi + D
y
j = 0 (Eq. 14 repeated)
or
0.8944F
DA
i 0.4472F
DA
k + 600i 500j
+ 400k + Dxi + D
y
j = 0
Then the summation of the z components gives
Fz = 0: 0.4472F
DA
+ 400 = 0
Solving gives
F
DA
= 894 N (T) Ans.
13

1030
6.4 Space Trusses Example 5, page 1 of 5
3 m
5 m 3 m
4 m
2 m
4 m
x
y
z
800 N 800 N
800 N 800 N
A
B
C
D
E
F
G
H
8 m
5. The truss is supported by ball-and-socket
joints at A, B, C, and D. Determine the forces
in members FE and EC. State whether the
forces are tension or compression.

1031
6.4 Space Trusses Example 5, page 2 of 5
3 m 5 m 3 m
4 m
2 m
4 m
x
y
z
800 N 800 N
800 N
800 N
A
B
C
D
E
F
G H
Section
8 m
Because we have been asked to find only two
member forces, the method of sections will be
used. Pass a section through the truss that cuts
members DF, CF, FG, and FE.
1

1032
6.4 Space Trusses Example 5, page 3 of 5
y
z
800 N
C
D
F
8 m
x
F
BD
3 m
F
FE
F
FG
(parallel to the z axis)
Free-body diagram of portion of truss on
side of section that includes joint F.
2
F
EF
can be computed by summing moments about the z axis:
Mz = 0: (800 N)(3 m) F
FE
(8 m) = 0
Solving gives
F
FE
= 300 N = 300 N (C) Ans.
3
+

1033
6.4 Space Trusses Example 5, page 4 of 5
3 m 5 m 3 m
4 m
2 m
4 m
x
y
z
800 N 800 N
800 N
800 N
A
B
C
D
E
F
G H
8 m
To calculate the force in member EC, pass a
section through the truss that cuts members EC,
EF, EA, and EH.
4
Section

1034
6.4 Space Trusses Example 5, page 5 of 5
6 m
x
y
800 N
A
C
E
F
EC
F
FE
= 300 N (C)
F
EC
F
EA
8 m
8 m
z
3 m
B
Free-body diagram of portion of truss on
side of section that includes joint E
(Parallel to line AB)
Sum moments about AB, calling a moment positive if it
produces a counterclockwise moment about AB, as
viewed from A looking back at B.
M
AB
= (800 N)(3 m) 300 N)(8 m) + 6.8716 F
EC
= 0
Solving gives
F
EC
= 0 Ans.
F
EC
can be found by summing moments about line AB. To
sum moments, first we have to express F
EC
in component
form.
F
EC
= F
EC
= F
EC
( 0.6247i 0.6247j 0.4685k)
The moment of F
EC
about the line AB is given by
M
AB
= u r
BE

F
EC
= k { 3i + 8j + 6k} m
F
EC
( 0.6247i 0.6247j 0.4685k)
= 6.8716 F
EC
8i 8j 6k
8
2
+ 8
2
+ 6
2
6
5
7
Unit vector
parallel to AB
Position
vector
from B
to E
+

1035

6.5 Cables: Concentrated Loads
1036
6.5 Cables: Concentrated Loads Procedures and Strategies, page 1 of 3
3 m 3 m 3 m 3 m
2 kN
3 kN
1.6 kN
4 m
3 m 3 m
2 kN
3 kN
4 m
A
B
C
D
E
E
D
C
3 m 3 m 3 m 3 m
2 kN
3 kN
1.6 kN
A
B
C
D
E
T
BC
Ax
A
y
E
y
Ex
Ex
E
y
Section
Procedures and Strategies for Solving Problems Involving
Cables With Concentrated Loads
1. Pass sections through the cable at
points where something about the
cable geometry is known. For
example, consider the following
situations:
a) The location of a load point is
known. Pass a section through
the cable close to the point, draw
a free-body diagram of either the
portion of the cable to the left or
to the right of the section, and
write M = 0 about the point.
This gives an equation involving
the reaction components at one
of the supports. Then writing
three equilibrium equations for a
free body of the entire cable will
allow you to solve
simultaneously for all reaction
components
Fx = 0
F
y
= 0
M
B
= 0
M
C
= 0 Equation
involving Ex and E
y

only
Three
equations
involving Ex,
E
y
, Ax, and A
y
.

1037
6.5 Cables: Concentrated Loads Procedures and Strategies, page 2 of 3
3 m 3 m 3 m 3 m
2 kN
3 kN
1.6 kN
2 kN
3 kN
A
B
C
D
E
E
D
C
Ex
E
y
Section
T
CBy
T
BCx
1.5 m
T
CBy
B
C
T
CBx
1.5 m
3 m
M
B
= 0 Equation involving
T
CBx
and T
CBy
only.
Fx = 0
F
y
= 0
Two equations
involving Ex, E
y
,
T
CBx
, and T
CBy
.
b) The horizontal and vertical
distances between two adjacent
load points are known.
Consider a free body of the
cable segment between the load
points (Do not include the load
points in the free body.) and sum
moments about either end to
obtain an equation relating the x
and y components of the tension
in the cable segment. Next pass
a section through the segment,
and consider a free body of
either the part of the cable to the
left or right of the section. Write
Fx = 0 and Fy = 0. These
equations, together with three
equilibrium equations for a free
body of the entire cable, will
allow you to solve for the
support reactions.
T
BAx
T
BAy
Point on BC just to left of B
Point on BC just to right of C

1038
6.5 Cables: Concentrated Loads Procedures and Strategies, page 3 of 3
2 kN
E
D
Ex
E
y
T
DC
y
D
3 m
Now known
2. Once the support-reaction components are
known, you can determine the elevation of any
load point by passing a section through the cable
near the load point, considering a free-body
diagram of the part of the cable on either side of
the section, and then writing M = 0 about the
section. The unknown elevation will occur as a
moment arm in this equation and can thus be
found.
M
D
= 0 Equation involving y
D
.

1039
6.5 Cables: Concentrated Loads Problem Statement for Example 1
1.4 kN
D
B
C
A
2 kN
4 m 3 m 2 m
1. For the cable system shown, determine the
reactions at support A and the distance y
C
.
2 m
2.5 m
y
C

1040
6.5 Cables: Concentrated Loads Problem Statement for Example 2
3.5 kN
B
2 kN
P A
C
4 m
2. The horizontal force P is applied to end A of the
cable as shown. Determine the value of P and the
distance d required to keep the cable system in the
configuration shown. Also determine the total
length of the cable.
d
15 m
12 m

1041
6.5 Cables: Concentrated Loads Problem Statement for Example 3
C
500 N
150 N
B
6 m

3 m
A
D
3. The cable supports the 150 N and 500 N
loads shown. Determine the distance x
C
and
the tension in each segment of the cable.
4 m
x
C
2 m

1042
6.5 Cables: Concentrated Loads Problem Statement for Example 4
2.2 kN
E
B C
A
1.8 kN
1.2 kN
2 m
3 m
D
y

D
0.5 m
0.25 m
4. For the cable system shown, determine the distance y
C
for
which segment BC will be horizontal. Also determine y
D
.
2 m
y
C

1043
6.5 Cables: Concentrated Loads Problem Statement for Example 5
P

B
B
A
C
200 N
P

D
D
E
7 m
5 m
2 m
2 m
3 m
5. For the cable system shown,
determine the value of the forces P
B

and P
D
necessary to maintain the
given configuration.
4 m 4 m
2 m

1044
6.5 Cables: Concentrated Loads Problem Statement for Example 6
80 lb
50 lb
B
C
1.5 ft
A
D
6. For the cable system shown, determine
distance y
B
and the tension in each segment.
5 ft 3.5 ft
2 ft
3.5 ft
y
B

1045
6.5 Cables: Concentrated Loads Problem Statement for Example 7
2 kip
2 kip
2 kip
2 kip
A
F
18 ft 18 ft 18 ft 18 ft
18 ft
B
C
D
E
15 ft
35 ft
7. The cable supports the four forces shown.
Determine the maximum tension in the cable.

1046
6.5 Cables: Concentrated Loads Example 1, page 1 of 3
1.4 kN
D
B
C
A
A
A

x
1.4 kN
T

BC
A

y
B
2 kN
4 m 3 m
2 m
2 m
1. For the cable system shown, determine the
reactions at support A and the distance y
C
.
1 Strategy: Note that both
horizontal and vertical
distances between A and B are
known. Thus summing
moments about B, for a
free-body AB, would give an
equation involving Ax and A
y

only.
2 Free-body diagram of AB
+
Equilibrium equation
M
B
= 0: Ax(2.5 m) A
y
(2 m) = 0 (1)
3
2 m
2.5 m
y
C
2.5 m

1047
6.5 Cables: Concentrated Loads Example 1, page 2 of 3
1.4 kN
D
B
C
A
2 kN
D

x
D

y
A

y
A

x
2 m 3 m 4 m
Equilibrium equation
M
D
= 0: (1.4 kN)(3 m + 4 m)
+ (2 kN)(4 m)

y
(2 m + 3 m + 4 m) = 0 (2)
Solving gives
y
= 1.978 kN Ans.
Using this result in Eq. 1 gives
Ax(2.5)
y
(2) = 0 ( Eq. 1 repeated)
1.978 kN
Solving gives
Ax = 1.582 kN Ans.
6
We can get another equation involving the reaction at
A as the only unknown by summing moments about
D for a free body of the entire cable.
Free-body diagram of entire cable
4
5
+

1048
6.5 Cables: Concentrated Loads Example 1, page 3 of 3
1.4 kN
B
C
A
2 kN
A

y
1.978 kN
A

x 1.582 kN
T

CD
2 m 3 m
Now that the values of Ax and A
y
are known,
summing moments about C, for a free body ABC,
will give an equation with a single unknown, yc.
Free-body diagram of ABC

+
Equilibrium equation
M
C
= 0: (1.4 kN)(3 m)
(1.978 kN)(2 m + 3 m)
+ (1.582 kN) yc) = 0
Solving gives
yc = 3.60 m Ans.
9

7

8
y
C

1049
6.5 Cables: Concentrated Loads Example 2, page 1 of 3
15 m
3.5 kN
B
2 kN
P A
C
4 m
2. The horizontal force P is applied to end A of the
cable as shown. Determine the value of P and the
distance d required to keep the cable system in the
configuration shown. Also determine the total
length of the cable.
Strategy: Note that we know the
horizontal distance between points A
and B, and we can compute the vertical
distance between these points. Thus
summing moments about B for a free
body AB, will give an equation from
which P can be found.
1
d
12 m

1050
6.5 Cables: Concentrated Loads Example 2, page 2 of 3
Free-body diagram of ABC 5
Equilibrium equation for ABC
M
C
= 0: (2.667 kN)(15 m) + (2 kN)(4 m + d)
+ (3.5 kN)(d) = 0
Solving gives
d = 5.819 m Ans.
Free-body diagram of AB
6
+
+
15 m 12 m = 3 m
C
15 m
d
3.5 kN
B
2 kN
P = 2.667 kN
A
4 m
C

y
C

x
P
2 kN
T

BC
3.5 kN
A
B
4 m
2
Equilibrium equation for AB
M
B
= 0: (2 kN)(4 m) P(3 m) = 0
Solving gives
P = 2.667 kN Ans.
3
Now that the value of P is known, we can sum
moments about C, for a free-body of the whole
cable, to obtain an equation for d.
4

1051
6.5 Cables: Concentrated Loads Example 2, page 3 of 3
B
A
C
4 m
12 m
d 5.819 m
15 m 12 m 3 m
The total length of the cable can be
found by applying the Pythagorean
Theorem to segment AB and to BC
7
Geometry 8
L
Total
= L
AB
+ L
BC
= (4 m)
2
+ (3 m)
2
+ (5.819 m)
2
+ (12 m)
2
= 5 m + 13.34 m
= 18.34 m Ans.

1052
6.5 Cables: Concentrated Loads Example 3, page 1 of 5
x
C
2 m
4 m
2 m
C
500 N
150 N
B
6 m

3 m
150 N
B
A
A

y
A

x
T

BC
2
A
D
3. The cable supports the 150 N and 500 N
loads shown. Determine the distance x
C
and
the tension in each segment of the cable.
1 Strategy: Note that both the horizontal
and vertical distances between A and B
are known. Thus summing moments
about B, for a free-body AB would give
an equation involving the reaction
components at A (Ax and A
y
), only; no
other unknowns are present.
Free-body diagram of AB
Equilibrium equation
M
B
= 0: A
y
(4 m) x (2 m) = 0 (1)
3
+
4 m

1053
6.5 Cables: Concentrated Loads Example 3, page 2 of 5
C
500 N
150 N
B
A
D
A

y
A

x
6 m
D

y
D

x
2 m
Now summing moments about D, for a free body of the entire
cable, will give another equation for Ax and A
y
.
4
Free-body diagram of entire cable 5
Equilibrium equation
M
D
= 0:
x
(2 m + 6 m + 3 m) (150 N)(6 m + 3 m)
(500 N)(3 m) = 0
Solving gives
Ax = 259.09 N (2)
Substituting this in Eq. 1 gives
4 A
y
2 Ax = 0 (Eq. 1 repeated)
259.09 N
Solving gives
A
y
= 129.55 N (3)
6
+
3 m

1054
6.5 Cables: Concentrated Loads Example 3, page 3 of 5
9
We can get the tension in AB by considering a
free body of support A.
10
Free-body diagram of A for calculating tension T
AB
Summing horizontal and vertical forces gives
T
ABx
= 259.09 N (4)
T
ABy
= 129.55 N (5)
and the magnitude of the tension in AB is
T
AB
= (T
ABx
)
2
+ (T
ABy
)
2
= (259.09 N)
2
+ (129.55)
2
= 290 N Ans.
12
x

C
+
C
500 N
150 N
B
A
11
A

y
129.55 N
A

x 259.09 N
T

ABy
T

ABx
T

CD
A

y
129.55 N
A

x 259.09 N
6 m
T

AB
2 m
Now that Ax and A
y
are known, we can calculate x
C
by
summing moments about C for a free-body ABC.
Free-body diagram of ABC
7
8
Equilibrium equation
M
C
= 0: (150 N)(6 m) (259.09 N)(2 m + 6 m)
+ (129.54 N)(x
C
) = 0
Solving gives
x
C
= 9.05 m Ans.

1055
6.5 Cables: Concentrated Loads Example 3, page 4 of 5
B
T
ABy
129.55 N
T

BCy
T
ABx
259.09 N
T

BC
T

BCx
14
150 N
Free-bodies of B and C will yield the
values of the tension in BC and CD
13
Free-body diagram of B for
calculating tension T
BC
The magnitude of the tension in BC is
T
BC
= (T
BCx
)
2
+ (T
BCy
)
2


= (109.09 N)
2
+ (129.55 N)
2
= 169.4 N Ans.
+
Summing vertical forces gives
T
BCy
= 129.55 N
Summing horizontal forces gives
Fx = 0: 150 N T
BCx
+ 259.09 N = 0
Solving gives
T
BCx
= 109.09 N
15
16

1056
6.5 Cables: Concentrated Loads Example 3, page 5 of 5
C
500 N
17
T

BCy
129.55 N
T

CDy
T

CD
T
BCx
109.09 N
T

CDx
Free-body diagram of C for
calculating tension T
CD
Summing vertical forces gives
T
CDy
= 129.55 N
Summing horizontal forces gives
F
x
= 0: 500 N T
CDx
+ 109.09 N = 0
Solving gives
T
CDx
= 390.91 N
The magnitude of the tension in CD is
T
CD
= (T
CDx
)
2
+ (T
CDy
)
2
= (390.91 N)
2
+ (129.55)
2


= 412 N Ans.
+
C
500 N
150 N
B
6 m

3 m
A
D
4 m
x
C
2 m
1057
6.5 Cables: Concentrated Loads Example 4, page 1 of 6
2.2 kN
E
B C
A
1.8 kN
1.2 kN
2 m
3 m
D
y

D
0.5 m
0.25 m
4. For the cable system shown, determine the distance y
C
for
which segment BC will be horizontal. Also determine y
D
.
Strategy: We have to make use of the fact
that segment BC is horizontal. One way to
do this is to pass a section through BC and
then consider the portion of the cable to the
left of the section.
1
2 m
y
C

1058
6.5 Cables: Concentrated Loads Example 4, page 2 of 6
B
A
2.2 kN
A

y
A

x
2
T

BC
(horizontal)
Free-body diagram of AB
Equilibrium equation for AB (Because BC is
horizontal, we use the sum of vertical forces
so that the unknown tension T
BC
will not
appear in the equation.):
Fx = 0: A
y
2.2 kN = 0
Solving gives
A
y
= 2.2 kN (1)
3
+
2.2 kN
E
B C
A
1.8 kN
1.2 kN
2 m
3 m
D
y

D
0.5 m
0.25 m 2 m
y
C

1059
6.5 Cables: Concentrated Loads Example 4, page 3 of 6
2.2 kN
E
B
C
A
1.8 kN
D
1.2 kN
2 m 2 m
y

C
A

y
2.2 kN
A

x
E

x
E

y
3 m
0.5 m
Now that A
y
is known, we can solve for
Ax by summing moments about E for a
free-body diagram of the entire cable.
4
Free-body diagram of entire cable 5
+
6 Equilibrium equation for entire cable
M
E
= 0: (1.2 kN)(0.5 m)
+ (1.8 kN)(0.5 m + 2 m)
+ (2.2 kN)(0.5 m + 2 m + 2 m)
(2.2 kN)(0.5 m + 2 m + 2 m + 0.25 m)
x(3 m) = 0
Solving gives
Ax = 1.517 kN (2)
0.25 m

1060
6.5 Cables: Concentrated Loads Example 4, page 4 of 6
y
B
A

y
2.2 kN
2.2 kN
0.25 m
B
A
T

BC
A

x 1.517 kN
y

B
y

C
(because BC is horizontal)
Summing moments about B for a free body AB
will now give us the value of y
C
.
7
Free-body diagram of AB 8
Equilibrium equation for AB
M
B
= 0: (1.517 kN)(y
C
) (2.2 kN)(0.25 m) = 0
Solving gives
y
C
= 0.36 m Ans.
9
+
2.2 kN
E
B C
A
1.8 kN
1.2 kN
2 m
3 m
D
y

D
0.5 m
0.25 m 2 m
y
C

1061
6.5 Cables: Concentrated Loads Example 4, page 5 of 6
2.2 kN
B C
A
1.8 kN
D
1.2 kN
2 m 2 m 0.25 m
A

y
2.2 kN
A

x 1.517 kN
T

DE
y

D
Finally, summing moments about D
for a free body ABCD will give an
equation for y
D
.
Free-body diagram of ABCD
10
11
Equilibrium equation
M
D
= 0: (1.517 kN)(y
D
)
+ (2.2 kN)(2 m + 2 m)
+ (1.8 kN)(2 m)
(2.2 kN)(0.25 m + 2 m + 2 m) = 0
Solving gives
y
D
= 2.01 m Ans.
12
+

1062
6.5 Cables: Concentrated Loads Example 4, page 6 of 6
2.2 kN
E
B
C
A
1.8 kN
D
1.2 kN
0.36 m
The minus sign in y
D
= 2.01 m
indicates point D lies above point A,
not below it, as was assumed in
drawing the free-body diagram.
13
2.01 m
3 m

1063
6.5 Cables: Concentrated Loads Example 5, page 1 of 4
4 m
2 m
P

B
B
A
C
200 N
P

D
D
E
7 m
5 m
2 m
2 m
3 m
5. For the cable system shown,
determine the value of the forces P
B

and P
D
necessary to maintain the
given configuration.
Strategy: Equilibrium equations for a free
body of the entire cable would involve
six unknowns (the components of the
support reactions: Ax, A
y
, Ex, and E
y
;
plus P
B
and P
D
). Thus using a free body
of the entire cable does not look like a
good place to start.
1
4 m

1064
6.5 Cables: Concentrated Loads Example 5, page 2 of 4
P

B
B
C
200 N
P

D
D
5 m
A

y
A

x
E

y
E

x
4 m 4 m
2 m 3 m
2 m
2 m
D
C
B
C
2 m + 3 m 5 m
2
BC
DC
BC
DC
A better place to start is to observe that we
know the horizontal and vertical distances
between points B, C and D. Thus we can
calculate the angles that segments BC and DC
make with the horizontal, and then we can use
the equilibrium equation for connector C to find
the tension in BC and in CD. Once these
tensions are known, we can use the equilibrium
equations for segments AB and ED to determine
P
B
and P
D
.
Geometry 3
3 m
5 m 1
BC
= tan ( ) = 59.04 (1)
1 7 m
5 m
DC
= tan ( ) = 54.46 (2)
7 m
5 m
3 m
7 m

1065
6.5 Cables: Concentrated Loads Example 5, page 3 of 4
4 m
P

B
B
C
A

y
A

x
2 m
C
200 N
T

BC
T

DC
T

BC
177.46 N
A
4
6
BC
59.04

DC
54.46
BC
59.04
Free-body diagram of C
Equilibrium equations for C
Fx = 0: T
BC
cos 59.04 T
DC
cos 54.46 + 200 = 0
F
y
= 0: T
BC
sin 59.04 T
DC
sin 54.46 = 0
Solving gives
T
BC
= 177.46 N (3)
T
DC
= 187.02 N (4)
+
+
5
Free-body diagram of AB
Equilibrium equation for AB
M
A
= 0: P
B
(4 m) + (177.46 N)(cos 59.04)(2 m)
(177.46 N)(sin 59.04)(4 m) = 0
Solving gives
P
B
= 106.5 N Ans.
7
+

1066
6.5 Cables: Concentrated Loads Example 5, page 4 of 4
C
P

D
D
E
E

y
2 m
E

x
T

DC
187.02 N
DC
54.46
8
8 m
Free-body diagram of ED
M
E
= 0: P
D
(2 m) (187.02 N)(cos 54.46 )(2 m)
+ (187.02 N)(sin 54.46 )(8 m) = 0
Solving gives
P
D
= 500 N Ans.
9 +

1067
6.5 Cables: Concentrated Loads Example 6, page 1 of 8
80 lb
50 lb
B
C
1.5 ft
A
D
6. For the cable system shown, determine
distance y
B
and the tension in each segment.
Strategy: If we can compute the reactions
at supports A and D, then we can compute
the tensions in AB and CD. Let's start
with support D. Note that both horizontal
and vertical distances between C and D
are known. Thus summing moments
about C, for the free body CD, would give
an equation involving Dx and D
y
only.
1
5 ft 3.5 ft
2 ft
3.5 ft
y
B

1068
6.5 Cables: Concentrated Loads Example 6, page 2 of 8
80 lb
50 lb
B
C
A

x
A

y
D

y
D

x
2 ft
1.5 ft
50 lb
C
D

x
D

y
3.5 ft
D
T

BC
5 ft
2
5
Free-body diagram of CD
Equilibrium equation
M
C
= 0: D
y
(5 ft) Dx(3.5 ft) = 0 (1)

+
3
4 We can get another equation involving reactions at Dx and
D
y
as the only unknowns by summing moments about A
for a free body of the entire cable.
Free-body diagram of entire cable

6
+
Equation of equilibrium
M
A
= 0: D
y
(2 ft + 3.5 ft + 5 ft) Dx(1.5 ft)
(50 lb)(2 ft + 3.5 ft) 80 lb(2 ft) = 0 (2)
3.5 ft 5 ft

1069
6.5 Cables: Concentrated Loads Example 6, page 3 of 8
y
B
80 lb
50 lb
B
C
D
D

y
34.41 lb
D

x 49.15 lb
T

AB
9
Solving Eqs. 1 and 2 simultaneously gives
Dx = 49.15 lb (3)
D
y
= 34.41 lb (4)
Now that the values of Dx and D
y
are
known, summing moments about B, for a
free body BCD, will give an equation with
a single unknown, y
B
.
8
Free-body diagram BCD
M
B
= 0: (50 lb)(3.5 ft) + (34.41 lb)(3.5 ft + 5 ft) (49.15 lb)(y
B
) = 0
Solving gives
y
B
= 2.39 ft Ans (5)
+ 10
7
5 ft 3.5 ft
3.5 ft

1070
6.5 Cables: Concentrated Loads Example 6, page 4 of 8
We still must find the tension in each cable segment.
A free-body consisting of support D will give the
tension in CD.
11
Summing x forces and then y forces gives
T
CDx
= 49.15 lb (6)
T
CDy
= 34.41 lb (7)
The magnitude of the tension in CD is then
T
CD
= (T
CDx
)
2
+ (T
CDy
)
2
= (49.15 lb)
2
+ (34.41 lb)
2
= 60.0 lb Ans.
D

y
34.41 lb
D

x 49.15 lb
T

CDy
T

CDx
T

CD
12
D
Free-body diagram of D for determining tension T
CD
80 lb
50 lb
B
C
1.5 ft
A
D
5 ft 3.5 ft
2 ft
3.5 ft
y
B

1071
6.5 Cables: Concentrated Loads Example 6, page 5 of 8
50 lb
C
14
T
CDx
49.15 lb (Eq. 7)
T

CDy
34.41 lb (Eq. 6)
T

BCx
T

BCy
T

BC
A free-body diagram of connection C will give
the tension in cable segment BC.
13
Free-body diagram of C for determining tension T
BC
Summing horizontal forces gives
T
BCx
= 49.15 lb (8)
Summing vertical forces gives
F
y
= 0: 34.41 lb T
BCy
50 lb = 0
Solving gives
T
BCy
= 15.59 lb (9)
+
80 lb
50 lb
B
C
1.5 ft
A
D
5 ft 3.5 ft
2 ft
3.5 ft
y
B

1072
6.5 Cables: Concentrated Loads Example 6, page 6 of 8
15 Why did T
BCy
turn out
to be negative? Because
in drawing the tension
T
BC
on the free-body
diagram of C, we
assumed that B was
below C:
But this assumption was
wrong, as we should have
realized, since we have
already calculated y
B
and
found y
B
= 2.39 ft (Eq. 5).
The corrected free-body
diagram shows T
BC
pulling
up on C, which would lead
to a positive value of T
BCy
16
Free-body diagram of C
Corrected free-body diagram of C
B lies
above C
80 lb
50 lb
B
C
A
D
y

B
3.5 ft
80 lb
50 lb
B
C
3.5 ft
A
D
2.39 ft
C
CD

y
CD

x
50 lb
T

BC
(Pulls down on C)
50 lb
C
CD

y
CD

x
T

BC
(Pulls up on C)

1073
6.5 Cables: Concentrated Loads Example 6, page 7 of 8
80 lb
B
T

ABy
T

BCx
49.15 lb
T

ABx
T

BCy
+15.59 lb
T

AB
The magnitude of the tension in BC is
T
BC
= (T
BCx
)
2
+ (T
BCy
)
2
= (49.15 lb)
2
+ ( 15.59 lb)
2
= 51.56 lb Ans.
17
The tension in AB can be found from a free-body
diagram of connector B.
18
Free-body diagram of B for calculating T
AB
19
Since T
BCy
points down,
we are assuming correctly
that point C lies below B.
Summing x forces gives
T
ABx
= 49.15 lb
Summing y forces gives
T
ABy
= 15.59 lb + 80 lb
= 95.59 lb
The magnitude of the tension in AB is then
T
AB
= (T
ABx
)
2
+ (T
ABy
)
2
= (49.15 lb)
2
+ (95.59 lb)
2
= 107.5 lb Ans.
20

1074
6.5 Cables: Concentrated Loads Example 6, page 8 of 8
80 lb
B
A
A

y
A

x
T
BC
(unknown)



B
(known)
The tension in the cable segments could have
been calculated by other approaches, for
example, by calculating the angle of inclination
of each segment and then summing moments
about one end of the segment.
Example:
21
M
A
= 0 gives an equation that will give us the
value of T
BC
. Proceeding to the next cable segment,
BC, and summing moments about end B of that
segment would give the value of the tension in that
segment, etc.
80 lb
50 lb
B
C
1.5 ft
A
D
5 ft 3.5 ft
2 ft
3.5 ft
y
B

1075
6.5 Cables: Concentrated Loads Example 7, page 1 of 7
2 kip
2 kip
2 kip
2 kip
A
F
18 ft 18 ft 18 ft 18 ft
18 ft
B
C
D
E
15 ft
35 ft
7. The cable supports the four forces shown.
Determine the maximum tension in the cable.
Strategy: To find the maximum tension, Tmax in
the cable, we could find the tension in each of
the five segments and then pick the largest.
However, this is a tedious and time-consuming
approach, and we can find Tmax more easily if we
note two facts: 1) The horizontal component of
tension is the same in all cable segments, and 2)
the maximum tension occurs in the cable
segment with the maximum slope.
1

1076
6.5 Cables: Concentrated Loads Example 7, page 2 of 7
2 kip
B
T

BCx
T

ABx
T

BCy
T

ABy
T

BC
T

AB
T

horizontal
T
Demonstration that horizontal components
of tension are equal:
2
Free-body diagram of B
Fx = 0: T
ABx
+ T
BCx
= 0
Thus

T
ABx
= T
BCx
That is, the horizontal components are
equal. A similar argument holds for points
C, D, and E.
+
Demonstration that Tmax occurs in the segment
with the greatest slope:
3
For each cable segments,
T =

Since T
horizontal
is the same for all segments, it
follow that T will be largest where cos is
smallest, that is, where is the largest the
steepest slope.

T
horizontal

cos


1077
6.5 Cables: Concentrated Loads Example 7, page 3 of 7
15 ft
2 kip 2 kip
2 kip
2 kip
A
B
E
C
D
F
A

x
A

y

A
F

y
F

x
F
18 ft 18 ft 18 ft 18 ft 18 ft
35 ft
Thus the problem of finding Tmax has now
been reduced to determining whether the slope
is greater at A than at B.
Free-body diagram of entire cable
4
5 To determine
A
, we can find Ax and A
y

first. To do this, sum moments about F for
a free-body consisting of the entire cable.
Equilibrium equation for cable
M
F
= 0: Ax(15 ft)
y
(5 18 ft)
+ (2 kip)(4 18 ft)
+ (2 kip)(3 18 ft)
+ (2 kip)(2 18 ft)
+ (2 kip)(1 18 ft) = 0 (1)
6
+

1078
6.5 Cables: Concentrated Loads Example 7, page 4 of 7
2 kip
2 kip
2 kip
A
B
C
D
A

x
T

DE
A

y
18 ft 18 ft 18 ft
8
35 ft
Equilibrium equation for ABCD
M
D
= 0: Ax(35 ft)
y
(3 18 ft)
+ (2 kip) (2 18 ft)
+ (2 kip) (18 ft) = 0 (2)
Solving Eqs. 1 and 2 simultaneously gives
Ax = 4.154 kip (3)
y
= 4.692 kip (4)
9
+
To obtain another equation for Ax and
y
, pass a
section through segment DE and consider a free
body ABCD.
7
Free-body diagram of ABCD

1079
6.5 Cables: Concentrated Loads Example 7, page 5 of 7
2 kip
2 kip
2 kip
2 kip
A
F
B
C
D
E
F

y
F

x
A

y
4.692 kip
A

x 4.154 kip
Free-body diagram of entire cable
10 To find the slope at F, consider a free body of the
entire cable.
11
12
+
+
Equilibrium equations for entire cable
Fx = 0: 4.154 kip + Fx = 0
F
y
= 0: 4.692 kip + F
y
(4 2 kip) = 0
Solving gives
Fx = 4.154 kip (5)
F
y
= 3.308 kip (6)

1080
6.5 Cables: Concentrated Loads Example 7, page 6 of 7
A F
13
A

y
4.692 kip
A

x 4.154 kip
T

AB

A

A
F

y
3.308 kip
F

x 4.154 kip

F

F
T

FE
Compare slopes at A and F
Thus
A
is greater than
F
and it follows that Tmax occurs at A.
Tmax = magnitude of the reaction force at A
= (Ax)
2
+ (A
y
)
2
= (4.154 kip)
2
+ (4.692 kip)
2
= 6.27 kip Ans.
14
A
= tan ( ) = 48.5
1 4.692
4.154 4.154
3.308 1
F
= tan ( ) = 38.5

1081
6.5 Cables: Concentrated Loads Example 7, page 7 of 7
15 Of course, once we had calculated the components of the
reaction at F,
Fx = 4.154 kip (Eq. 5 repeated)
F
y
= 3.308 kip (Eq. 6 repeated)
we could have computed the tension in EF:
T
EF
= (4.154 kip)
2
+ (3.308 kip)
2
= 5.31 kip
and then we could have compared T
EF
with the tension in
T
AB
. That is, in this particular example, we didn't have to
compare slopes, but we did it to illustrate the principle
that Tmax occurs where the slope is a maximum.

1082

6.6 Cables: Uniform Loads
1083
6.6 Cables: Uniform Loads Procedures and Strategies, page 1 of 3
Procedures and Strategies for Solving Problems
Involving Cables With Uniform Loads
1. Draw a free-body diagram of the entire cable and write
three equilibrium equations involving the four reaction
components at the supports.
2. Obtain a fourth equation by using additional information
given in the problem, such as the following:
a) The location of the low point of the cable is
specified. In this case, pass a section through the low
point, consider a free body of the portion of the cable
on one side of the section, and write M = 0 about the
low point.
B
A
6 m
w kN/m
Section through low
point
2 m
B
6 m
w
2 m
Bx
B
y
To

1084
6.6 Cables: Uniform Loads Procedures and Strategies, page 2 of 3
B
A
w kN/m
B
Bx
B
y
T
T known, unknown, so one
additional unknown and two
additional equations.
B
Bx
B
y
T
known (slope = tan ), T
unknown, so one additional
unknown and two additional
equations.
b) The tension in the cable is known at a support. In
this case, draw a free-body diagram of a short piece
of cable at the support, and write Fx = 0 and F
y
= 0.
c) The slope of the cable at a support is known. In
this case, proceed as in case b).
3. Once the support reactions are known, you can solve for
all other quantities such as cable tensions or the sag by
considering free-body diagrams of the supports or of
portions of the cable found by passing a section through the
cable and isolating the portion on one side.

1085
6.6 Cables: Uniform Loads Procedures and Strategies, page 3 of 3
w
y
x
Cable
wx
2
2T o
Parabolic Curve
y =
Notes:
a) You can solve most uniform-load cable problems by
using free bodies and equilibrium equations only. You
do not need to use the equation for the cable shape,
y = wx
2
/(2To) (1)
unless the cable length, s, is part of the problem, in
which case you must use Eq. 1 together with the
equation for calculating s.
ds = [(dx)
2
+ (dy)
2
] (2)
In using Eq. 1, take into account that the origin of the
xy-coordinate system is at the low point of the
parabola, but the low point and hence the origin need
not by part of the cable, since the cable coincides with
only a part of the parabola.
b) You can find the maximum tension in the cable by
calculating the tension at the point where the slope is a
maximum.
c) In some problems you can save some work by using
the fact that the horizontal component, To, of the cable
tension is constant throughout the cable.

1086
6.6 Cables: Uniform Loads Problem Statement for Example 1
A B
400 ft
3800 ft
1. Each cable of the center span of the suspension
bridge supports a uniform load of 10 kip/ft along
the horizontal. If the span is 3,800 ft and the sag
400 ft, determine the maximum and minimum
tensions in the cable.

1087
6.6 Cables: Uniform Loads Problem Statement for Example 2
A
B
C
8 ft
h
50 ft 30 ft
2. A length of oil pipeline weighing 3200 lb is supported by a system
of cables as shown. Determine a) the distance h to the lowest point C
on the cable and b) the maximum tension in the cable.

1088
6.6 Cables: Uniform Loads Problem Statement for Example 3
A
B
3. An 18-lb washer load of wet beach-towels is hung on a clothesline
to dry. Each beach towel is 72-in. long, and the clothesline is tied to
supports located 75 in. above the ground. If the breaking strength of
the clothesline is 280 lb, determine if the clothesline can be tightened
enough to keep the middle towel from touching the ground.
75 in 75 in
28 ft

1089
6.6 Cables: Uniform Loads Problem Statement for Example 4
A
0.5 kg/m 0.5 kg/m
3 m s
C B
80 m 60 m
4. Chains AB and BC are attached to a roller support at B
as shown. The chains support beams that have mass per
length of 0.5 kg/m. Determine the maximum tension in
each chain and the sag s in chain BC.

1090
6.6 Cables: Uniform Loads Problem Statement for Example 5
40
A
0.2 kg/m
B
3 m
2 m
5. Cable AB supports a uniformly distributed mass of
0.2 kg/m. The slope of the cable at B is known to be
40. Determine the maximum tension in the cable and
the length of the cable.

1091
6.6 Cables: Uniform Loads Problem Statement for Example 6
C
4 ft
B
A
48 ft
w lb/ft
6 ft
6. Determine the largest uniform load, w lb/ft, that the cable
can support if it will fail at a tension of 3,000 lb. Also
determine the location of the low point C of the cable.

1092
6.6 Cables: Uniform Loads Problem Statement for Example 7
100 kg
5 kg/m
A
B
20 m
4 m
7. The cable system shown supports a uniformly
distributed mass of 5 kg/m along the horizontal.
Determine the tension at B and the length of portion AB
of the cable. Assume that the pulleys are frictionless.
1093
6.6 Cables: Uniform Loads Problem Statement for Example 8
22 kg/m
16 m
A
B
C
x
y
d

A
d

B
30 m
8. The chain AB supports a horizontal, uniform beam of
mass per length 22 kg/m. If the maximum allowable
tension in the chain is 7 kN, determine distances d
A
and
d
B
of the supports above the low point C of the chain.
Also determine the length of the chain.

1094
6.6 Cables: Uniform Loads Problem Statement for Example 9
C A B
10 kg
h
d
9. A 40-m length of rope has a uniformly distributed
mass of 0.1 kg/m and has one end fixed and the other
end attached to a cart as shown. Determine the distance
d and the sag h when the cable and cart are in
equilibrium under the force supplied by the 10-kg load.

1095
6.6 Cables: Uniform Loads Example 1, page 1 of 4
A B
400 ft
3800 ft
1. Each cable of the center span of the suspension
bridge supports a uniform load of 10 kip/ft along
the horizontal. If the span is 3,800 ft and the sag
400 ft, determine the maximum and minimum
tensions in the cable.

1096
6.6 Cables: Uniform Loads Example 1, page 2 of 4
The maximum tension occurs where the slope is
greatest points A and B.
The minimum tension occurs where the slope is least the
low point of the cable.
2
1
3800 ft
400 ft
B
A

1097
6.6 Cables: Uniform Loads Example 1, page 3 of 4
C
B
T

By
T

Bx
400 ft
T

o
(1900 ft)/2 950 ft
(3800 ft)/2 = 1900 ft
(Because of symmetry, we know
that the low point occurs at the
middle of the 3800-ft span.)
C
(Resultant acts through middle of
1900 ft span)
Resultant load
(10 kip/ft)(1900 ft)
= 19,000 kip
Tension at low point
(minimum tension)
Tension components at B 5
7
8
6
4
3 Free-body diagram of CB
3800 ft
400 ft
B A

1098
6.6 Cables: Uniform Loads Example 1, page 4 of 4
T

max
T

By
19,000 kip
B
Equilibrium equation
Fx = 0: T + T
Bx
= 0
F
y
= 0: T
By
19,000 kip = 0
M
B
= 0: (19,000 kip)(950 ft) T (400 ft) = 0
Solving gives
T = 45,125 kip (minimum tension in cable) Ans.
T
Bx
= 45,125 kip
T
By
= 19,000 kip
9
The maximum tension is the resultant of T
Bx
and T
By
:
T
Bx
= 45,125 kip
+
+
+
T

max = (45,125 kip)
2
+ (19,000 kip)
2
= 49,000 kip Ans.

1099
6.6 Cables: Uniform Loads Example 2, page 1 of 4
A
B
C
8 ft
h
50 ft 30 ft
2. A length of oil pipeline weighing 3200 lb is supported by a system
of cables as shown. Determine a) the distance h to the lowest point C
on the cable and b) the maximum tension in the cable.

1100
6.6 Cables: Uniform Loads Example 2, page 2 of 4
B

x
B

y
A

y
A

x
A
B
C
3200 lb
(80 ft)/2 40 ft
(Weight of pipeline acts through
midpoint of 30 ft + 50 ft 80-ft span,
not through low point, C)
8 ft
2
40 ft
The maximum tension occurs at B, where the
slope is largest, so let's draw a free-body
diagram of the entire system, including the
tension components at B:
1
Equilibrium equation
MA = 0: B
y
(40 ft + 40 ft)
Bx(8 ft) (3,200 lb)(40 ft) = 0 (1)

We could write two additional equilibrium
equation, but they would introduce two
additional unknowns, Ax and A
y
, so nothing
would be gained. An additional free-body
diagram involving Bx and B
y
is needed.
3
4
+

1101
6.6 Cables: Uniform Loads Example 2, page 3 of 4
50 ft
8
7 Free-body diagram of BC
Pass a vertical section through the low point, C, and
draw a free-body diagram of the portion of the
system to the right of C.
5
The 3200-lb weight spread over an 80-ft length
of pipe is equivalent to a uniformly distributed
load of 3200 lb/80 ft = 40 lb/ft.
6
+
+
+
9 Equilibrium equation for free-body BC
Fx = 0: T + Bx = 0 (2)
F
y
= 0: B
y
2000 lb = 0 (3)
M
B
= 0: (2000 lb)(25 ft) T (h + 8 ft) = 0 (4)
A
B
C
80 ft
B

x
B

y
A

y
A

x
50 ft
B

x
B
C
B

y
h + 8 ft
T

o
(50 ft)/2
25 ft
Weight distributed load length of pipe
40 lb/ft 50 ft
2000 lb
(T

o is horizontal
because C is
the minimum
point on the
curve.)
h
8 ft

1102
6.6 Cables: Uniform Loads Example 2, page 4 of 4
B
B

x 4000 lb
B

y
2000 lb
T

max
Eq. 4 is nonlinear, but Eqs. 1-3 are linear and can be easily
solved to give
Bx = 4000 lb
B
y
= 2000 lb
T = 4000 lb
Using the value of T = 4000 lb in Eq. 4 gives

(2000 lb)(25 ft) T (h + 8 ft) = 0 (Eq. 4 repeated)
Solving gives
h = 4.5 ft Ans.
The maximum tension is the resultant of Bx and B
y
: 11
10
Tmax = (4000 lb)
2
+ (2000 lb)
2
= 4470 lb Ans.

1103
6.6 Cables: Uniform Loads Example 3, page 1 of 2
A
B
The towel near the middle of the clothesline span will touch the
ground if the sag is 75 in. 72 in. = 3 in. We can now work the
problem either of two ways: 1) assume the sag = 3 in., compute
the corresponding maximum tension, and compare it to the
280-lb breaking strength; or 2) assume the maximum tension is
280 lb, compute the corresponding sag, and compare it to the
maximum permissible sag of 3 in.
3. An 18-lb washer load of wet beach-towels is hung on a clothesline
to dry. Each beach towel is 72-in. long, and the clothesline is tied to
supports located 75 in. above the ground. If the breaking strength of
the clothesline is 280 lb, determine if the clothesline can be tightened
enough to keep the middle towel from touching the ground.
1
75 in 75 in
28 ft

1104
6.6 Cables: Uniform Loads Example 3, page 2 of 2
(28 ft)/4 7 ft
(28 ft)/4 7 ft
B

x 252 lb
B
T

max
B

y
9 lb
B

y
B

x
T

o
3 in
(18 lb)/2
9 lb
Let's arbitrarily choose the first approach assume a sag
= 3 in. Then a free-body diagram of the right half of the
clothesline and hanging towels would appear like this:
Equilibrium equations
Fx = 0: T + Bx = 0 (1)
F
y
= 0: 9 lb + B
y
= 0 (2)
M
B
= 0: 9 lb(7 ft)
T (3 in. 1 ft/ 12 in.) = 0 (3)
Solving Eq. 1-3 gives
Bx = 252 lb
B
y
= 9 lb
T = 252 lb
The maximum tension occurs where the slope is
greatest at B (or A):
= (252 lb)
2
+ (9 lb)
2
= 252.2 lb
2
3
4
+
+
+
Since the maximum tension, 252.2 lb, is less than
the breaking strength, 280 lb, the clothesline can
be tightened enough to keep the towel from
touching the ground. Note, however, that the
poles supporting the clothesline must be
well-anchored in the ground since they must
resist a horizontal force of Bx = 252 lb acting at
the point where the clothesline is attached.
B

1105
6.6 Cables: Uniform Loads Example 4, page 1 of 4
A
0.5 kg/m 0.5 kg/m
3 m s
C B
80 m 60 m
4. Chains AB and BC are attached to a roller support at B
as shown. The chains support beams that have mass per
length of 0.5 kg/m. Determine the maximum tension in
each chain and the sag s in chain BC.
1 The maximum tension in chain AB occurs where the
slope is greatest, points A and B.

1106
6.6 Cables: Uniform Loads Example 4, page 2 of 4
We know the location of the low point of chain AB, so
let's choose a free-body diagram of the right half of
the chain because then we will know the distances
appearing in the moment equation.
2
Equilibrium equation
Fx = 0: T + Bx = 0 (1)
F
y
= 0: B
y
196.2 N = 0 (2)
M
B
= 0: T (3 m)
+ (196.2 N)(20 m) = 0 (3)
5
Solving Eqs. 1-3 gives
T = 1308 N
Bx = 1308 N
B
y
= 196.2 N
6
+
+
+
B
3 m
B

y
B

x
T

o
(40 m)/2 20 m
Resultant force
(0.5 kg/m)(9.81 m/s
2
)(40 m)
196.2 N
By symmetry, we know the low point occurs at mid span:
(80 m)/2 40 m
4
3 Sag distance is given:

1107
6.6 Cables: Uniform Loads Example 4, page 3 of 4
s
(60 m)/2 30 m
Resultant weight
(0.5 kg/m)(9.81 m/s
2
)(30 m)
147.15 N
(30 m)/2 15 m
T
'
o
B
'
x
B
'
y
T

max
B

x 1308 N
B

y
196.2 N
B
Maximum tension in AB
Next consider a free-body diagram of the
left half of chain BC:
Equilibrium equation
Fx = 0: Bx + T = 0 (4)
F
y
= 0: B
y
147.15 N = 0 (5)
M
B
= 0: T (s) (147.15 N)(15 m) = 0 (6)
+
+
+
= (1308 N)
2
+ (196.2 N)
2
= 1323 N Ans.
7
8 9
B

1108
6.6 Cables: Uniform Loads Example 4, page 4 of 4
B
147.15 N
196.2 N
T

o 1308 N
T
'
o
F

B
, vertical force
acting on roller support
Eqs. 4-6 are three equations in four unknowns, Bx ,
B
y
, T and s. An additional free-body is needed.
The free-body diagram below shows that no
horizontal force acts on the base of the roller
support at B.
10
11
T

max
B
'
x 1308 N
B
'
y
147.15 N
Equilibrium equation
Fx = 0: 1308 N + T = 0 (7)
Solving Eqs. 4-7 simultaneously gives
T = 1308 N
s = 1.688 m Ans.
Bx = 1308 N
B
y
= 147.15 N
12
+
Maximum tension in BC 13
Tmax = (1308 N)
2
+ (147.15 N)
2
= 1316 N Ans.

1109
6.6 Cables: Uniform Loads Example 5, page 1 of 5
40
A
0.2 kg/m
B
3 m
2 m
5. Cable AB supports a uniformly distributed mass of
0.2 kg/m. The slope of the cable at B is known to be
40. Determine the maximum tension in the cable and
the length of the cable.
The maximum tension occurs at B, where the slope of
the cable is the greatest.
1

1110
6.6 Cables: Uniform Loads Example 5, page 2 of 5
40
T

max cos 40
T

max sin 40
2 m
A

y
A

x
A
Weight = (0.2 kg/m)(9.81 m/s
2
)(3 m)

= 5.886 N
(3 m)/2 1.5 m
B
3 m
T

max
Free-body diagram 2
Equilibrium equation
M
A
= 0: (Tmax sin 40)(3 m)
(Tmax cos 40)(2 m)
(5.886 N)(1.5 m) = 0 (1)
Solving gives
Tmax = 22.28 N Ans.
+
To compute the length of the cable, we need
to use the equation of the cable curve,
y = (2)
wx
2
2T
3
4

1111
6.6 Cables: Uniform Loads Example 5, page 3 of 5
Here, w is the distributed load per horizontal meter,
w = (0.2 kg/m)(9.81 m/s
2
)
= 1.962 N/m (3)
The quantity T is the horizontal component of the cable tension.
Since T is the same at all points, we can evaluate it at support B:
T = Tmax cos 40
= (22.28 N) cos 40 (4)
Substituting Eqs. 3 and 4 in Eq. 2 gives


y = (Eq. 2 repeated)
or,
y = 0.057478x
2
(5)
2T
wx
2
1.962 N/m by Eq. 3
(22.28 N) cos 40 by Eq. 4
We will also need the slope,
= 2(0.057478x)
= 0.114956x (6)
dx
dy
5

1112
6.6 Cables: Uniform Loads Example 5, page 4 of 5
A(x

A
, y

A
)
B(x

B
, y

B
)
y
x
40
dy
40
dx
dy
dx
ds
3 m
y 0.05784 x
2
The length of the cable is
s
AB
= ds
= (dx)
2
+ (dy)
2
= 1 + ( )
2
dx
A
B
dy
dx
B
A
0.114956x by Eq. 6
(Change the variable of
integration from s to x)
6
dy
dx
Also at B, the slope is known:
= tan 40 (9)
x
B
x
A
x
A
x
B
Thus the length of the cable can be expressed as
s
AB
= 1 + (0.114956x)
2
dx (7)
To evaluate this integral, we have to find the values of x
A

and x
B
. From the figure, we see that
x
A
= x
B
3 m (8)

7

1113
6.6 Cables: Uniform Loads Example 5, page 5 of 5
Using Eq. 6 to evaluate the left-hand side of Eq. 9 gives
( )
B
= tan 40 (Eq. 9 repeated)
0.114956x
B
, by Eq. 6
Solving for x
B
gives
x
B
= (tan 40)/0.114956
= 7.299311 m
and using this result in Eq. 8 gives
x
A
= x
B
3 m
= 7.299311 3
= 4.299311 m
dy
dx
This integral is best evaluated numerically with the
integral function of a calculator.
The result is
s
AB
= 3.61 m Ans.
8
9
7.299311
4.299311
By Eq. 7, then, the length of the cable is
s
AB
= 1+ (0.114956x)
2
dx

1114
6.6 Cables: Uniform Loads Example 6, page 1 of 5
C
4 ft
B
A
48 ft
w lb/ft
6 ft
6. Determine the largest uniform load, w lb/ft, that the cable
can support if it will fail at a tension of 3,000 lb. Also
determine the location of the low point C of the cable.

1115
6.6 Cables: Uniform Loads Example 6, page 2 of 5
+
2 Moment equilibrium equation
M
B
= 0: (48w)(24 ft) (3000 lb)(sin )(48 ft)
+ (3000 lb)(cos )(4 ft) = 0 (1)
C
B
A
B

x
B

y
(3000 lb) sin
(3000 lb) cos
Resultant load
w lb/ft 48 ft
(48w) lb
48 ft
24 ft (48 ft)/2 24 ft
4 ft
6 ft
The maximum tension in the cable occurs where the
slope is greatest, point A. The cable will fail if the
tension there exceeds 3,000 lb. A free-body diagram of
the entire system, with a maximum tension of 3,000 lb at
A, would appear as below:
1
The line of action of the weight of the cable does
not pass through the low point C, because the ends
of the cable are at different elevations.
As part of our solution
to the problem, we will
calculate .

1116
6.6 Cables: Uniform Loads Example 6, page 3 of 5
C
B
A
B

x
B

y
(3000 lb) sin
(3000 lb) cos 4 ft
6 ft
Pass a vertical section through
the low point, C.
d
(Unknown distance)
3
Moment equilibrium equation
M
B
= 0: (48w)(24 ft) (3000 lb)(sin )(48 ft)
+ (3000 lb)(cos )(4 ft) = 0 (1)
We could write two additional equilibrium equation, but they
would introduce two additional unknowns, Bx and B
y
, so there
is no advantage gained. Instead, we need another free-body
diagram.
2
+

1117
6.6 Cables: Uniform Loads Example 6, page 4 of 5
C
A
(3000 lb) sin
(3000 lb) cos
Resultant
w d
4 ft + 6 ft 10 ft
T

o
d/2 d/2
d
Free-body diagram of portion to left of section 4


5
+
+
Equation of equilibrium
F
y
= 0: (3000 lb)(sin ) wd = 0 (2)
M
C
= 0: (3000 lb)(cos )(10 ft)
(3000 lb)(sin )d
+ (wd)(d/2) = 0 (3)
We could write a third equilibrium equation, but it
would introduce an additional unknown, T so no
advantage would be gained.
Eqs. 1-3 are three nonlinear equations in three unknowns,
, w, and d. These equations are best solved with a
calculator capable of solving simultaneous nonlinear
equations. Alternatively, proceed as follows. First note
that Eq. 2,
3000 sin wd = 0 (Eq. 2 repeated)
can be solved for d:
d = (3000 sin )/w (4)
This equation can be used to eliminate d from Eq. 3:
3000 cos (10) (3000 sin )d
+ wd
2
/2 = 0 (Eq. 3 repeated)
(3000 sin )/w
2
(3000 sin )/w
Multiplying through by w and combining terms gives
(30,000 cos )w (3,000 sin )
2
/2 = 0 (5)
6

1118
6.6 Cables: Uniform Loads Example 6, page 5 of 5

Next note that Eq.1,
(48)(24)w (3,000 sin )(48)
+ (3,000 cos )(4) = 0 (Eq. 1 repeated)
can be solved for w:
w = 125 sin 10.4167 cos (6)
and this equation can be used to eliminate w from Eq. 5:
(30,000 cos ) w (3,000 sin )
2
/2 = 0 (Eq. 5 repeated)
125 sin 10.4167 cos
Carrying out the multiplication gives
(3.75 10
6
) cos sin (0.312501 10
6
) cos
2

(4.5 10
6
) sin
2
= 0 (7)
Dividing both sides by 10
6
gives
(3.75) cos sin (0.312501) cos
2

(4.5) sin
2
= 0 (8)
Solving this equation by trial and error gives
= 36.48
Using this value for in Eq. 6 then gives w:
w = 125 sin 10.4167 cos (Eq. 6 repeated)
= 125 sin 36.48 10.4167 cos 36.48
= 65.94 lb/ft Ans.
Eq. 4 then gives distance d:
d = (Eq. 4 repeated)
=
= 27.0 ft Ans.
w
3000 sin
3000 sin 36.48
65.94
7 8

1119
6.6 Cables: Uniform Loads Example 7, page 1 of 4
The 100-kg mass produces a cable
tension of 100 kg 9.81 m/s
2
= 981 N
at point A.
1
7. The cable system shown supports a uniformly
distributed mass of 5 kg/m along the horizontal.
Determine the tension at B and the length of portion AB
of the cable. Assume that the pulleys are frictionless.
4 m
20 m
B
A
5 kg/m
100 kg

1120
6.6 Cables: Uniform Loads Example 7, page 2 of 4
B
B

x
B

y
(981 N) sin
(981 N) cos
Resultant weight of cable between A and B
(20 m)(9.81 m/s
2
)(5 kg/m)
981 N
20 m
10 m
A
981 N
4 m
2 Free-body diagram
Equilibrium equation
Fx = 0: (981 N) cos + Bx = 0 (1)
F
y
= 0: (981 N) sin + B
y
981 N = 0 (2)
M
B
= 0: (981 N) cos (4 m)
(981 N) sin (20 m)
+ (981 N)(10 m) = 0 (3)
+
+
+
Eqs. 1-3 are best solved with a calculator that solves
simultaneous nonlinear equations. Alternatively, Eq. 3
involves only one unknown, , and can be solved by
trial-and-error to yield
= 40.67 (4)
Using this value in Eq. 1 and 2 then leads to
Bx = 744.1 N (5)
B
y
= 341.7 N (6)
3
4

1121
6.6 Cables: Uniform Loads Example 7, page 3 of 4
The tension at B is then
Bx
2
+ B
y
2
= (744.1 N)
2
+ (341.7 N)
2

= 818.8 N Ans.
To compute the length of the cable, we need to use the
equation of the cable curve,
2T
wx
2

y = (7)
Here, w is the distributed load per horizontal meter,
w = (5 kg/m)(9.81 m/s
2
)
= 49.05 N/m (8)
The quantity T is the horizontal component of the cable
tension. Since T is the same at all points, we can evaluate it
at support B:

T = Bx
744.1 N by Eq. 5 (9)
5
6
Substituting Eqs. 8 and 9 in Eq. 7 gives,
y = (Eq. 7 repeated)
or,
y = 0.03296 x
2
(10)
We will need the equation for the slope:
= 2(0.3296 x) = 0.06592 x (11)
wx
2
2T
49.05 N/m
744.1 N
dx
dy
The length of the cable is
s
AB
= ds
= (dx)
2
+ (dy)
2
or

s
AB
= 1 + ( )
2
dx (12)
A
B
(0.06592 x)
2
, by Eq. 11
ds
dx
dy
(Change the
variable of
integration
from s to x)
dx
dy
7
8
B
A
x
A
x
B

1122
6.6 Cables: Uniform Loads Example 7, page 4 of 4
x
y
B(x

B
, y

B
)
A(x

A
, y

A
)
20 m
y 0.03296x
2
40.67
dx
dy
40.67
To evaluate the integral for s
AB
, we have to find the values
of x
A
and x
B
.
From the figure, we see that
x
B
= x
A
+ 20 m (13)
Also at A, the slope is known, so
= tan 40.67
Using Eq. 11 to evaluate the left hand-side of this equation gives
9
10
dy
dx
Solving for x
A
gives
x
A
= 13.03436 m (14)
Using this result in Eq. 13 gives
x
B
= x
A
+ 20 m (Eq. 13 repeated)
= 13.03436 m + 20 m
= 6.96564 m (15)
By Eq. 12, then, the length of cable is
s
AB
= 1 + (0.06592 x)
2
dx
This integral is best evaluated numerically with the
integral function of a calculator. The result is
s
AB
= 21.7 m Ans.

6.96564
-13.03436
dy
dx
( )
A
= tan 40.67
0.06592x
A
, by Eq. 11
11

1123
6.6 Cables: Uniform Loads Example 8, page 1 of 6
22 kg/m
16 m
A
B
C
x
y
d

A
d

B
30 m
8. The chain AB supports a horizontal, uniform beam of
mass per length 22 kg/m. If the maximum allowable
tension in the chain is 7 kN, determine distances d
A
and
d
B
of the supports above the low point C of the chain.
Also determine the length of the chain.
The maximum tension occurs at support A,
where the slope of the chain is greatest.
1

1124
6.6 Cables: Uniform Loads Example 8, page 2 of 6
A
B
B

x
B

y
A

x
A

y
d

A
d

B
(46 m)/2 23 m
30 m + 16 m 46 m
Resultant weight
(22 kg/m)(9.81 m/s
2
)(46 m)
9928 N
9.928 kN
3
C
Free-body diagram of entire system.
Equation of equilibrium
M
B
= 0: Ax(d
A
d
B
) A
y
(46 m)
+ (9.928 kN)(23 m) = 0 (1)
We could write two additional equations of
equilibrium, but they would contain two additional
unknowns, Bx and B
y
, so nothing would be gained.
4
2
+

1125
6.6 Cables: Uniform Loads Example 8, page 3 of 6
22 kg/m
A
B
C
B

x
B

y
A

y
A

x
C
A
A

y
T

o
d

A
30 m
(30 m)/2 15 m
Resultant weight
(22 kg/m)(9.81 m/s
2
)(30 m)
6475 N
6.475 kN
d

A
30 m
To obtain an additional free-body diagram, pass a
vertical section through the low point of the chain.
A

x
5
Free-body diagram of portion of chain to left of section. 6

1126
6.6 Cables: Uniform Loads Example 8, page 4 of 6
Equation of equilibrium
F
x
= 0: Ax + T = 0 (2)
F
y
= 0: A
y
6.475 kN = 0 (3)
M
C
= 0: Ax(d
A
) A
y
(30 m)
+ 6.475 kN)(15 m) = 0 (4)
Equation 3 gives
A
y
= 6.475 kN (5)
Since we know that the maximum tension of 7 kN occurs
at A and is the resultant of Ax and A
y
, we have
Ax
2
+ A
y
2
= 7 kN
Substituting A
y
= 6.475 kN and solving for Ax gives
Ax = 2.660 kN (6)
Substituting for Ax and A
y
in Eqs. 2 and 4 gives
Ax + T = 0 (Eq. 2 repeated)

7
+
+
+
2.660 kN
Ax d
A
A
y
(30) + (6.475)(15) = 0 (Eq. 4 repeated)
2.660 kN 6.475 kN
Solving gives
T = 2.660 kN (7)
d
A
= 36.52 m (8) Ans.
Distance d
B
can now be found by substitution in Eq. 1:
Ax (d
A
d
B
) A
y
(46) + (9.928)(23) = 0
2.660 kN 36.52 m 6.475 kN
by Eq. 6 by Eq. 8 by Eq. 5
Solving gives
d
B
= 10.4 m Ans.
8

1127
6.6 Cables: Uniform Loads Example 8, page 5 of 6
To compute the length of the chain, we need to use the
equation of the chain curve:
y = (9)
Here, w is the distributed load per horizontal meter,
w = (22 kg/m)(9.81 m/s
2
)
= 215.8 N/m
= 0.2158 kN/m (10)
The quantity T has been given in Eq. 7
T = 2.660 kN (Eq. 7 repeated)
Substituting into Eq. 9 gives
0.2158 kN/m by Eq. 10
y = (Eq. 9 repeated)
2.660 kN by Eq. 7
or,
y = 0.040564 x
2
(11)
9
2T
wx
2
2T
wx
2
We will need the equation for the slope,
= 2(0.040564 x) = 0.08113 x (12)
dy
dx
(Change the variable of
integration from s to x)
dy
dx
ds
B
A
The length of the chain is
s
AB
= ds
= (dx)
2
+ (dy)
2
or
s
AB
= 1 + ( )
2
dx (13)
dy
dx
10
A
B
x
A
x
B

1128
6.6 Cables: Uniform Loads Example 8, page 6 of 6
x
y 0.040564 x
2
B(x

B
16 m)
y
A(x

A
30 m)
30 m 16 m
Since the location of the low point of the chain is
known, x
A
and x
B
are known:
11 The integral for the cable length, Eq. 13, can now be
written as
s
AB
= 1 + ( )
2
dx (Eq. 13 repeated)
This integral is best evaluated numerically with the integral
function of a calculator. The result is
s
AB
= 69.2 m Ans.
(0.08113x)
2
, by Eq. 12
-30
16
dy
dx
12

1129
6.6 Cables: Uniform Loads Example 9, page 1 of 4
C A B
10 kg
h
d
9. A 40-m length of rope has a uniformly distributed
mass of 0.1 kg/m and has one end fixed and the other
end attached to a cart as shown. Determine the distance
d and the sag h when the cable and cart are in
equilibrium under the force supplied by the 10-kg load.
The total weight of the rope is 40 m 0.1 kg/m 9.81 m/s
2
=
39.24 N, and the force of the cart acting horizontally on the
rope is 10 kg 9.81 m/s
2
= 98.1 N, or more than twice as much.
Thus it seems reasonable to assume that the sag, h, is small
compared to the span d, h d, and the weight of the rope is
well-approximated as a uniformly distributed load along the
horizontal of
w = (0.1 kg/m) (9.81 m/s
2
) = 0.981 N/m (1)
At the end of the problem, we can check the reasonableness of
these assumptions.
1

1130
6.6 Cables: Uniform Loads Example 9, page 2 of 4
B

x
Vertical force from
ground acting on cart
B

y
B

y
C

x
C

y
B

x T

o
(Horizontal
component
of tension)
(10 kg) (9.81 m/s
2
)
98.1 N
C
B
The equation describing the shape of the rope is
y = (2)
T is the horizontal component of tension in the rope
and can be found by considering free bodies of the
cart and rope.
wx
2
2T
2
Free-body diagram of cart 3
Free-body diagram of rope 5
The equation of the rope is now
0.981 N/m by Eq. 1
y = (Eq. 2 repeated)
98.1 N by Eq. 3
or,
y = 0.005 x
2
(4)
We will also need the equation for the slope,
= 2(0.005 x) = 0.01x (5)
2T
wx
2
+
4
6
F
x
= 0: 98.1 N + Bx = 0
Solving gives
Bx = 98.1 N = T (3)
dy
dx

1131
6.6 Cables: Uniform Loads Example 9, page 3 of 4
0.005d + 1 + (0.005d)
2
0.005d + 1 + (0.005d)
2
d/2 d/2
C B
h
x

C
d/2
x

B
d/2
x
dx
dy
C
B
B
C
ds
dx
dy
The equation for the length of the slope is
s
BC
= ds
= (dx)
2
+ (dy)
2
= 1 + ( )
2
dx (6)
x
C
and x
B
can be expressed in terms of the unknown, d:
7
The best way to solve for d in Eq. 7 is to use the solver on a
calculator and to use the calculator integral function to input
Eq. 7 in the solver. Alternatively, use a table of integrals to
evaluate the integral in Eq. 7 to get
40 = 1 + (0.005d)
2

+ (50) ln
Solving this equation by trial-and-error gives
d = 39.74 m Ans. (8)
d
2
x
B

x
C
8
y
d/2
-d/2
-d/2
d/2
dx
dy
Thus Eq. 6 can be written
s
BC
= 1 + ( )
2
dx (Eq. 6 repeated)
Rope length = 40 m (0.01 x) by Eq. 5
or,
40 = 1 + (0.01x)
2
dx (7)

1132
6.6 Cables: Uniform Loads Example 9, page 4 of 4
h
B C
y
x
The sag, h, can now be found from Eq. 4:
y = 0.005x
2
(Eq. 4 repeated)
h = by Eq. 8
39.74
2
The result is
h = 1.974 m Ans.
Note that the sag is much smaller than the span,
h = 1.974 m << 39.74 m = d, as we assumed in
the beginning of the problem.
9
d
2

1133

6.7 Cables: Catenaries
1134
6.7 Cables: Catenaries Procedures and Strategies, page 1 of 2
Procedures and Strategies for Solving Problems Involving Catenary Cables
To solve catenary-cable problems, you must determine the
"parameter of the catenary," c.
1. If the coordinates x and y of a point on the cable are
known, determine c from
y = c cosh (x/c) (1)
2. If the x coordinate of a point on the cable and the
corresponding cable length s are known, determine c from
s = sinh (x/c) (2)
3. If the y coordinate of a point on the cable and the length
s are known, determine c from
y
2
- s
2
= c
2
(3)
4. If the tension T is known at a point on the cable,
determine the y coordinate of the point from
T = wy (4)
Then if either x or s is known, then use either Eq. 1 or 3 to
solve for c.
s
y
x
T tension
in cable
T

o horizontal
component
of tension
(x, y)
c

1135
6.7 Cables: Catenaries Procedures and Strategies, page 2 of 2
5. If the slope of the cable is known at a point on the cable
with coordinate x, then differentiate Eq. 1 to obtain an
equation that can be solved for c.
Frequently, solving catenary-cable problem requires
finding roots of nonlinear algebraic equations. You can
usually handle these problems by using the solver on your
calculator, but some equations have multiple roots, and
you must determine which of the roots are physically
realistic.

1136
6.7 Cables: Catenaries Problem Statement for Example 1
A B
1. An electric power line of length 140 m and mass per
unit length of 3 kg/m is to be suspended between two
towers 120 m apart and of the same height. Determine
the sag and maximum tension in the power line.
120 m

1137
6.7 Cables: Catenaries Problem Statement for Example 2
C
400 ft
A B
40 ft
2. A cable is supported at two points 400 ft apart and at
the same elevation. If the sag is 40 ft and the weight per
unit length of the cable is 4 lb/ft, determine the length of
the cable and the tension at the low point, C.

1138
6.7 Cables: Catenaries Problem Statement for Example 3
A B
3. A 20-m chain is suspended between two points at
the same elevation and with a sag of 6 m as shown.
If the total mass of the chain 45 kg, determine the
distance between the supports. Also determine the
maximum tension.
6 m

1139
6.7 Cables: Catenaries Problem Statement for Example 4
A B
4. A certain cable will break if the maximum tension
exceeds 500 N. If the cable is 50-m long and has a
mass of 50 kg, determine the greatest span possible.
Also determine the sag.
Supports A and B are at the same elevation.

1140
6.7 Cables: Catenaries Problem Statement for Example 5
A B
F (force)
5. The cable is attached to a fixed support at A and a
moveable support at B. If the cable is 80-ft long, weighs
0.3 lb/ft, and spans 50 ft, determine the force F holding
the moveable support in place. Also determine the sag.

1141
6.7 Cables: Catenaries Problem Statement for Example 6
6. The cable is attached to a fixed support at
A and a moveable support B. If the cable is
40-m long and has a mass of 0.4 kg/m,
determine the span and sag.
A B
50 N

1142
6.7 Cables: Catenaries Problem Statement for Example 7
7. A cable goes over a frictionless pulley at B and
supports a block of mass m. The other end of the cable
is pulled by a horizontal force P. If the cable has a mass
per length of 0.3 kg/m, determine values of P and m that
will maintain the cable in the position shown.
m
P (horizontal)
A
B
10 m
5 m

1143
6.7 Cables: Catenaries Problem Statement for Example 8
C
20 m
A
B
8. A chain makes angles of 30 and 60 at its
supports as shown. Determine the location of the
low point C of the chain relative to A. Also
determine the tension at support A, if the cable has
a mass per length of 0.6 kg/m.
Supports A and B
are at different
elevation.
60
30

1144
6.7 Cables: Catenaries Problem Statement for Example 9
A
B
8 lb
C
9. A wire weighing 0.2 lb/ft is attached to a moveable
support at A and makes an angle of 55 at a fixed
support at B. Supports A and B are at different
elevations. Determine the location of the low point C
of the wire relative to support B. Also, determine the
tension in the wire at C.
55

1145
6.7 Cables: Catenaries Problem Statement for Example 10
A
B
C
1 m
10. Determine the location of the low point C relative to
the support at A. Also determine the tension at C, if the
mass of the cable per unit length is 1 kg/m.
10 m
2 m

1146
6.7 Cables: Catenaries Example 1, page 1 of 4
A B
1. An electric power line of length 140 m and mass per
unit length of 3 kg/m is to be suspended between two
towers 120 m apart and of the same height. Determine
the sag and maximum tension in the power line.
120 m

1147
6.7 Cables: Catenaries Example 1, page 2 of 4
(x, y) T

o horizontal
component
of tension
T tension
in cable
x
y
s
1 Many problems involving catenary cables can be
solved using the following formulas:
s = c sinh (x/c) (1)
y
2
s
2
= c
2
(2)
W = ws (3)
y = c cosh (x/c) (4)
T

o = wc (5)
T = wy (6)
where
w = weight of cable per unit length of cable,
and
W = weight of length of cable from low
point to a point a distance s along
the cable.
Other quantities are defined in the figure to the
right.
c

1148
6.7 Cables: Catenaries Example 1, page 3 of 4
A B
120 m
x

B
(120 m)/2
60 m
x
y
s

B
(140 m)/2
70 m
The distance, c, can be determined from Eq. 1:
s
B
= c sinh (x
B
/c) (Eq. 1 evaluated at point B)
or,
70 m = c sinh (60 m/c) (8)
This equation must be solved numerically for c. An initial
estimate for c, when the solver on a calculator is to be
used, could be
c = s
B
= 70 m
The solution to Eq. 8 is
c = 61.45 m
Another possible solution is c = 61.45 m, but this has no
physical meaning.
The sag, h, can be found from Eq. 2, provided that we
can determine the distance, c:
y
B
2
s
B
2
= c
2
(Eq. 2 evaluated at point B)
or,
(h + c)
2
(70 m)
2
= c
2
(7)
2
Sag h
c
y

B

1149
6.7 Cables: Catenaries Example 1, page 4 of 4
A B
x
y
Tension T

max
B
Substituting c = 61.45 m into Eq. 7 gives
(h + c)
2
(70 m) = c
2
(Eq. 7 repeated)
(h + 61.45 m)
2
(70 m)
2
= (61.45 m)
2
Solving gives the sag
h = 154.6 m
and
h = 31.70 m Ans.
The negative root has no physical meaning.
The maximum tension, Tmax, occurs where the cable
has its steepest slope, point B ( or point A).
The maximum tension can be calculated from Eq. 6:
Tmax = wy
B
(Eq. 6 evaluated at point B)
w is given.

= (3 kg/m)(9.81 m/s
2
) (31.70 m + 61.45 m)
= 2740 N
= 2.74 kN Ans.
5
3
4
y

B
h 31.70 m
c 61.45 m

1150
6.7 Cables: Catenaries Example 2, page 1 of 4
A B
40 ft
2. A cable is supported at two points 400 ft apart and at
the same elevation. If the sag is 40 ft and the weight per
unit length of the cable is 4 lb/ft, determine the length of
the cable and the tension at the low point, C.
C
400 ft

1151
6.7 Cables: Catenaries Example 2, page 2 of 4
(x, y) T

o horizontal
component
of tension
T tension
in cable
x
y
s
1 Many problems involving catenary cables can be
solved using the following formulas:
s = c sinh (x/c) (1)
y
2
s
2
= c
2
(2)
W = ws (3)
y = c cosh (x/c) (4)
T

o = wc (5)
T = wy (6)
where
w = weight of cable per unit length of cable,
and
W = weight of length of cable from low
point to a point a distance s along
the cable.
Other quantities are defined in the figure to the
right.
c

1152
6.7 Cables: Catenaries Example 2, page 3 of 4
3
c
c
y

B
A B
400 ft
x
x

B
(400 ft)/2
200 ft
y
s

B
A y B
Sag 40 ft
x
x

B
200 ft
The length of cable, s
B
, from the low point to point B can be
found from Eq. 1, provided that we can determine the
distance c:
s
B
= c sinh (x
B
/c) (Eq. 1 evaluated at point B)

= c sinh (200 ft/c) (7)
2 The distance c can be determined from Eq. 4:
y
B
= c cosh (x
B
/c) (Eq. 4 evaluated at point B)
or,
c + 40 ft = c cosh (200 ft/c) (8)
This equation must be solved numerically. An initial
estimate for c, when the solver on a calculator is to be
used, could be
c = sag = 40 ft.

1153
6.7 Cables: Catenaries Example 2, page 4 of 4
The solution to Eq. 8 is
c = 506.53 ft
Using this value of c in Eq. 7 gives
s
B
= c sinh (200 ft/c) (Eq. 7 repeated)

= (506.53 ft) sinh (200 ft/506.53 ft)

= 205.237 ft
By symmetry, the total length of cable is twice s
B
:
s
Total
= 2 205.237 ft

= 410 ft Ans.
Because the tension at the low point of the cable is
horizontal, it can be found from Eq. 5:
To = wc (Eq. 5 repeated)
w is given.
= (4 lb/ft)(506.53 ft)

= 2,025 lb Ans.
4

1154
6.7 Cables: Catenaries Example 3, page 1 of 4
A B
3. A 20-m chain is suspended between two points at
the same elevation and with a sag of 6 m as shown.
If the total mass of the chain 45 kg, determine the
distance between the supports. Also determine the
maximum tension.
6 m

1155
6.7 Cables: Catenaries Example 3, page 2 of 4
(x, y) T

o horizontal
component
of tension
T tension
in cable
x
y
s
1 Many problems involving catenary cables can be
solved using the following formulas:
s = c sinh (x/c) (1)
y
2
s
2
= c
2
(2)
W = ws (3)
y = c cosh (x/c) (4)
T

o = wc (5)
T = wy (6)
where
w = weight of cable per unit length of cable,
and
W = weight of length of cable from low
point to a point a distance s along
the cable.
Other quantities are defined in the figure to the
right.
c

1156
6.7 Cables: Catenaries Example 3, page 3 of 4
s

B
(20 m)/2 10 m
y
x
B
A
The distance between the supports is 2x
B
, and x
B
can be
found from Eq. 1, provided that we can determine the
distance c;
s
B
= c sinh (x
B
/c) (Eq. 1 evaluated at point B)
10 m
This equation can be solved explicitly for x
B
by
rearranging it as
sinh (x
B
/c) = 10 m/c
which implies
x
B
/c = sinh
-1
(10 m/c)
so
x
B
= c sinh
-1
(10 m/c) (7)
2
s

B
10 m
y
x
B A
The distance, c, can be determined from Eq. 2:
y
B
2
s
B
2
= c
2
(Eq. 2 evaluated at point B)
or
(6 m + c)
2
(10 m)
2
= c
2
or
36 + 12 c + c
2
100 = c
2
The c
2
terms cancel and the resulting linear equation
has the solution
c = 5.333 m
3
Sag 6 m
c
y

B c
x
B

1157
6.7 Cables: Catenaries Example 3, page 4 of 4
Total weight of the cable
Total length of the cable
T T

max
y
x
B
A
Substituting this value of c into Eq. 7 gives
x
B
= c sinh
-1
(10 m/c) (Eq. 7 repeated)
= (5.333 m) sinh
-1
(10 m/5.333 m)
= 7.393 m
Thus the distance between supports, 2x
B
, can be found:
2x
B
= 2(7.393 m)
= 14.786 m Ans.
The maximum tension occurs where the slope of the cable is a
maximum, at end B (or at A). Eq. 6 gives
Tmax = wy
B
(Eq. 6 evaluated at point B)
= y
B
= (6 m + 5.333 m)
= 250 N Ans.
4
20 m
(45 kg)(9.81 m/s
2
)
Sag 6 m
c 5.333 m
y
B

1158
6.7 Cables: Catenaries Example 4, page 1 of 6
A B
4. A certain cable will break if the maximum tension
exceeds 500 N. If the cable is 50-m long and has a
mass of 50 kg, determine the greatest span possible.
Also determine the sag.
Supports A and B are at the same elevation.

1159
6.7 Cables: Catenaries Example 4, page 2 of 6
(x, y) T

o horizontal
component
of tension
T tension
in cable
x
y
s
c
1
Many problems involving catenary cables can be
solved using the following formulas:
s = c sinh (x/c) (1)
y
2
s
2
= c
2
(2)
W = ws (3)
y = c cosh (x/c) (4)
T

o = wc (5)
T = wy (6)
where
w = weight of cable per unit length of cable,
and
W = weight of length of cable from low
point to a point a distance s along
the cable.
Other quantities are defined in the figure to the
right.

1160
6.7 Cables: Catenaries Example 4, page 3 of 6
A
B
y
x
T T

max
The maximum tension has been specified (500 N), so
a good place to start our solution is to see how we can
use the fact that Tmax = 500 N. Eq. 6 relates the
tension, T, to the y coordinate of a point on the curve:
T = wy (Eq. 6 repeated)
Since Tmax occurs where the curve has its steepest
slope, Eq. 6 gives Tmax at point B:
Tmax = wy
B
(Eq. 6 evaluated at point B)
2
=

=
= 50.97 m (7)
Thus because we know the maximum tension, we can
compute y
B
:
y
B
=
Tmax
w
Total length of the cable
Total weight of the cable
500 N
(50 kg)(9.81 m/s
2
)
50 m
3
Given
Tmax
y
B

1161
6.7 Cables: Catenaries Example 4, page 4 of 6
4 The distance between supports is 2x
B
, so we need to use the
value of y
B
to determine x
B
. This can be done by using Eq. 4,
provided that we can determine c:
y
B
= c cosh (x
B
/c) (Eq. 4 evaluated at point B)
We can solve this equation explicitly for x
B
by rewriting it as
cosh (x
B
/c) = y
B
/c
so
x
B
/c = cosh
-1
(y
B
/c)
or,
x
B
= c cosh
-1
(y
B
/c) (8)
A B
y
x
(By symmetry)
2 x
B
c
x
B
y
B

1162
6.7 Cables: Catenaries Example 4, page 5 of 6
A y
x
s

B
(50 m)/2 25 m
B
The distance, c, can be calculated from Eq. 2:
y
B
2
s
B
2
= c
2
(Eq. 2 evaluated at point B)
or,
(50.97 m)
2
(25 m)
2
= c
2
The solution is
c = 44.42 m
The negative root has no physical meaning.
Substituting c = 44.42 m and y
B
= 50.97 m in Eq. 8
gives
x
B
= c cosh
-1
(y
B
/c) (Eq. 8 repeated)
= (44.42 m) cosh
-1
(50.97 m/44.42 m)
= 23.836 m
So the distance between supports, 2x
B
, is known:
2x
B
= 2(23.836 m)
= 47.7 m Ans.
5
y
B
50.97 m
c

1163
6.7 Cables: Catenaries Example 4, page 6 of 6
A
y
x
Sag h
B
Since c and y
B
are known, the sag h can be computed:
h = y
B
c
= (50.97 m) (44.42 m)
= 6.55 m Ans.
6
y
B
50.97 m
c 44.42 m

1164
6.7 Cables: Catenaries Example 5, page 1 of 4
A B
F (force)
5. The cable is attached to a fixed support at A and a
moveable support at B. If the cable is 80-ft long, weighs
0.3 lb/ft, and spans 50 ft, determine the force F holding
the moveable support in place. Also determine the sag.

1165
6.7 Cables: Catenaries Example 5, page 2 of 4
(x, y) T

o horizontal
component
of tension
T tension
in cable
x
y
s
1 Many problems involving catenary cables can be
solved using the following formulas:
s = c sinh (x/c) (1)
y
2
s
2
= c
2
(2)
W = ws (3)
y = c cosh (x/c) (4)
T

o = wc (5)
T = wy (6)
where
w = weight of cable per unit length of cable,
and
W = weight of length of cable from low
point to a point a distance s along
the cable.
Other quantities are defined in the figure to the
right.
c

1166
6.7 Cables: Catenaries Example 5, page 3 of 4
c
A
x
B
y
50 ft
s

B
(80 ft)/2 40 ft
x

B
(50 ft)/2
25 ft
The force F acting on the moveable support at B equals the
horizontal component, T , of tension in the cable, F = T .
Eq. 5 can be used to calculate T , provided that we can
determine the distance, c:
T = wc (Eq. 5 repeated)
Given
= (0.3 lb/ft)c (7)
The distance, c, can be calculated from Eq. 1:
s
B
= c sinh (x
B
/c) (Eq. 1 evaluated at point B)
or,
40 ft = c sinh (25 ft/c) (8)
This equation must be solved numerically. An initial
estimate for c could be c = x
B
= 25 m, when finding c with
the use of the solver on a calculator. The solution to Eq. 8 is
c = 14.229 ft
The negative root has no physical meaning.
2

1167
6.7 Cables: Catenaries Example 5, page 4 of 4
Sag h
y
B
A
x
B
y
x

B
25 ft
3 Using c = 14.229 ft in Eq. 7 gives
T = (0.3 lb/ft)c (Eq. 7 repeated)
= (0.3 lb/ft)(14.229 ft)
= 4.27 lb
Since F = T , we have
F = 4.27 lb Ans.
The sag, h, can be calculated from Eq. 4 and the known
value of c:
h = y
B
c

By Eq. 4
= c cosh (x
B
/c) c
= (14.229 ft) cosh (25 ft/14.229 ft) 14.229 ft
= 28.2 ft Ans.
c 14.229 ft

1168
6.7 Cables: Catenaries Example 6, page 1 of 4
A B
50 N
6. The cable is attached to a fixed support at
A and a moveable support B. If the cable is
40-m long and has a mass of 0.4 kg/m,
determine the span and sag.

1169
6.7 Cables: Catenaries Example 6, page 2 of 4
(x, y) T

o horizontal
component
of tension
T tension
in cable
x
y
s
1 Many problems involving catenary cables can be
solved using the following formulas:
s = c sinh (x/c) (1)
y
2
s
2
= c
2
(2)
W = ws (3)
y = c cosh (x/c) (4)
T

o = wc (5)
T = wy (6)
where
w = weight of cable per unit length of cable,
and
W = weight of length of cable from low
point to a point a distance s along
the cable.
Other quantities are defined in the figure to the
right.
c

1170
6.7 Cables: Catenaries Example 6, page 3 of 4
c
x
B
2 x
B
A B
x
(by symmetry)
s

B
(40 m)/2 20 m
y
The span is 2x
B
. The coordinate, x
B
, can be found from
Eq. 1, provided that the distance c can be found:
s
B
= c sinh (x
B
/c) (Eq. 1 evaluated at point B)
This equation can be rewritten to give x
B
explicitly:
sinh (x
B
/c) = s
B
/c
so
x
B
/c = sinh
-1
(s
B
/c)
or
x
B
= c sinh
-1
(s
B
/c)
= c sinh
-1
(20 m/c) (7)
2

1171
6.7 Cables: Catenaries Example 6, page 4 of 4
Because the 50-N force acting on the moveable
support equals the horizontal component, T , of the
tension in the cable, Eq. 5 with T = 50 N can be used
to solve for c:
T = wc (Eq. 5 repeated)
or
50 N = (0.4 kg/m)(9.81 m/s
2
) c
Solving gives
c = 12.742 m
Using this value of c in Eq. 7 gives
x
B
= c sinh
-1
(20 m/c) (Eq. 7 repeated)
= (12.472 m) sinh
-1
(20 m/12.472 m)
= 15.708 m
So the span is
Span = 2x
B
= 2(15.708 m)
= 31.4 m Ans.
3

1172
6.7 Cables: Catenaries Example 7, page 1 of 4
7. A cable goes over a frictionless pulley at B and
supports a block of mass m. The other end of the cable
is pulled by a horizontal force P. If the cable has a mass
per length of 0.3 kg/m, determine values of P and m that
will maintain the cable in the position shown.
m
P (horizontal)
A
B
10 m
5 m

1173
6.7 Cables: Catenaries Example 7, page 2 of 4
(x, y) T

o horizontal
component
of tension
T tension
in cable
x
y
s
1 Many problems involving catenary cables can be
solved using the following formulas:
s = c sinh (x/c) (1)
y
2
s
2
= c
2
(2)
W = ws (3)
y = c cosh (x/c) (4)
T

o = wc (5)
T = wy (6)
where
w = weight of cable per unit length of cable,
and
W = weight of length of cable from low
point to a point a distance s along
the cable.
Other quantities are defined in the figure to the
right.
c

1174
6.7 Cables: Catenaries Example 7, page 3 of 4
B
A
P
y
x
5 m

The force P equals T , the horizontal component of
the cable tension given by Eq. 5:
T = wc (Eq. 5 repeated)
so, with T = P
P = wc (7)
Here,
w = (0.3 kg/m)(9.81 m/s
2
)
= 2.943 N/m (8)
The value of c in Eq. 7 can be found from Eq. 4:
y
B
= c cosh (x
B
/c) (Eq. 4 repeated)
or,
5 m + c = c cosh (10 m/c)
Solving numerically gives
c = 10.743 m
2
Using this value of c in Eq. 7 gives
P = wc (Eq. 7 repeated)
by Eq. 8
= (2.943 N/m)(10.743 m)
= 31.617 N Ans.
3
c
y
B
x
B
= 10 m

1175
6.7 Cables: Catenaries Example 7, page 4 of 4
mg
Pulley B
The cable tension at B must equal the weight, mg:
T
B
= mg
Thus the mass is
m = T
B
/g
by Eq. 6
= wy
B
/g
by Eq. 8
= (2.943 N/m)(5 m + 10.743 m) /9.81 N/m
2
= 4.72 kg Ans.
T
B
4
y
B
c 10.743 m
T

B
mg
P
A
y
B
x
5 m

1176
6.7 Cables: Catenaries Example 8, page 1 of 5
C
20 m
A
B
8. A chain makes angles of 30 and 60 at its
supports as shown. Determine the location of the
low point C of the chain relative to A. Also
determine the tension at support A, if the cable has
a mass per length of 0.6 kg/m.
Supports A and B
are at different
elevation.
60
30

1177
6.7 Cables: Catenaries Example 8, page 2 of 5
(x, y) T

o horizontal
component
of tension
T tension
in cable
x
y
s
1 Many problems involving catenary cables can be
solved using the following formulas:
s = c sinh (x/c) (1)
y
2
s
2
= c
2
(2)
W = ws (3)
y = c cosh (x/c) (4)
T

o = wc (5)
T = wy (6)
where
w = weight of cable per unit length of cable,
and
W = weight of length of cable from low
point to a point a distance s along
the cable.
Other quantities are defined in the figure to the
right.
c

1178
6.7 Cables: Catenaries Example 8, page 3 of 5
B (x

B
, y

B
)
A (x

A
, y

A
)
y
x
C
20 m
c
The geometric data are shown in the figure. To
determine the location of the low point C relative to A,
we need to determine the coordinates x
A
and y
A
. We
can get an equation for x
A
by using the fact that the
slope is known at A:
tan 30 =
by Eq. 4
=
= sinh (x
A
/c)
Solving for x
A
gives
x
A
= c sinh
-1
( tan 30) (7)
Similarly at point B, we have
x
B
= c sinh
-1
(tan 60) (8)
dx
dy (Slope is negative)
dy
dx
dx
d
c cosh (x/c)
A
A
2
A
30
30
60

1179
6.7 Cables: Catenaries Example 8, page 4 of 5
c
20 m
A (x

A
, y

A
)
C
y
x
B (x

B
, y

B
)
The coordinates x
A
and x
B
are related to the 20-m span
through the equation
x
B
x
A
= 20 m
or,
by Eq. 8 by Eq. 7
c sinh
-1
(tan 60) c sinh
-1
( tan 30) = 20
Since this equation is linear in c, it is easily solved to
give c = 10.717 m. Eq. 7 then gives
x
A
= c sinh
-1
( tan 30) (Eq. 7 repeated)
= (10.717 m) sinh
-1
( tan 30)
= 5.887 m Ans.
The y coordinate of point A can now be calculated from
Eq. 4:
y
A
= c cosh (x
A
/c) (Eq. 4 evaluated at point A)
= (10.717 m) cosh ( 5.887 m/10.717 m)
= 12.375 m (9)
3
30
60

1180
6.7 Cables: Catenaries Example 8, page 5 of 5
y
A
y
x
C
d
B
A
5.887 m
The vertical distance between support A and the low
point C is given by
d = y
A
c
by Eq. 9
= 12.375 m 10.717 m
= 1.658 m Ans.
The tension at A is given by Eq. 6:
T
A
= wy
A
(Eq. 6 evaluated at point A)
(w given) by Eq. 9
= (0.6 kg/m)(9.81 m/s
2
) (12.375 m)

= 72.8 N Ans.
4
c = 10.717 m

1181
6.7 Cables: Catenaries Example 9, page 1 of 4
A
B
8 lb
C
9. A wire weighing 0.2 lb/ft is attached to a moveable
support at A and makes an angle of 55 at a fixed
support at B. Supports A and B are at different
elevations. Determine the location of the low point C
of the wire relative to support B. Also, determine the
tension in the wire at C.
55

1182
6.7 Cables: Catenaries Example 9, page 2 of 4
(x, y) T

o horizontal
component
of tension
T tension
in cable
x
y
s
1 Many problems involving catenary cables can be
solved using the following formulas:
s = c sinh (x/c) (1)
y
2
s
2
= c
2
(2)
W = ws (3)
y = c cosh (x/c) (4)
T

o = wc (5)
T = wy (6)
where
w = weight of cable per unit length of cable,
and
W = weight of length of cable from low
point to a point a distance s along
the cable.
Other quantities are defined in the figure to the
right.
c

1183
6.7 Cables: Catenaries Example 9, page 3 of 4
To determine the location of the low point, C, relative to
the support at B, we need to determine the coordinates x
B

and y
B
. We can get an equation for x
B
by using the fact
that the slope is known at B:
tan 55 =
by Eq. 4
=
= sinh (x
B
/c)
Thus
x
B
= c sinh
-1
( tan 55) (7)
The value of c occurring in Eq. 7 can be found by
observing that the 8-lb force acting at support A equals T ,
the horizontal component of tension at A, so Eq. 5 gives
T = wc (Eq. 5 repeated)
8 lb 0.2 lb/ft
Solving gives
c = 40 ft (8)
B dx
dy
B
c cosh (x/c)
dx
d
2
C
B
w 0.2 lb/ft
A
T

o 8 lb
55
55
B
dx
dy
B (x

B
, y

B
)
A
x
y
C
c

1184
6.7 Cables: Catenaries Example 9, page 4 of 4
c = 40 ft
d
y

B
Using this result, c = 40 ft, in Eq. 7 gives
x
B
= c sinh
-1
(tan 55) (Eq. 7 repeated)
= (40 ft) sinh
-1
(tan 55)
= 46.169 ft Ans.
The vertical distance between B and C is
d = y
B
c
by Eq. 4
= c cosh (x
B
/c) c
= (40 ft) cosh (46.169 ft/40 ft) 40 ft
= 29.7 ft Ans.
Since point C is the low point of the cable, the tension
there is horizontal and so must equal the horizontal
component of tension at A which is known to be 8 lb.
That is,
T
C
= 8 lb Ans.
3
Free body diagram of portion AC of cable
C
8 lb
T

C
Vertical force
acting on support
A
Weight of portion
AC of cable
C
y
x
A
B
x

B
46.169 ft

1185
6.7 Cables: Catenaries Example 10, page 1 of 6
A
B
C
1 m
10. Determine the location of the low point C relative to
the support at A. Also determine the tension at C, if the
mass of the cable per unit length is 1 kg/m.
10 m
2 m

1186
6.7 Cables: Catenaries Example 10, page 2 of 6
(x, y) T

o horizontal
component
of tension
T tension
in cable
x
y
s
1 Many problems involving catenary cables can be
solved using the following formulas:
s = c sinh (x/c) (1)
y
2
s
2
= c
2
(2)
W = ws (3)
y = c cosh (x/c) (4)
T

o = wc (5)
T = wy (6)
where
w = weight of cable per unit length of cable,
and
W = weight of length of cable from low
point to a point a distance s along
the cable.
Other quantities are defined in the figure to the
right.
c

1187
6.7 Cables: Catenaries Example 10, page 3 of 6
Branch of cosh function
corresponding to the
principal value of the inverse
function
c
C 1 m
A(x

A
, y

A
)
B(x

B
, y

B
)
x
y
2 m + 1 m 3 m
10 m
The geometric data are shown in the figure.
To locate the low point, C, relative to support A, we
must determine the coordinate x
A
. We know that x
A
is
related to y
A
through Eq. 4:
y
A
= c cosh (x
A
/c) (Eq. 4 evaluated at point A)
And this can be solved for x
A
to give
x
A
= c cosh
-1
(y
A
/c) (7)
The minus sign must be inserted here to make x
A

negative because we will consider "cosh
-1
" to be the
principal value of the inverse cosh function, and the
principal value is always positive.
v
Note that cosh
-1
v is not defined for v < 1
1
cosh
-1
v
(principal value)
1
cosh u
u
2

1188
6.7 Cables: Catenaries Example 10, page 4 of 6
C
1 m
A
B
x
y
10 m
An equation analogous to Eq. 7 can be written for point B:
x
B
= c cosh
-1
(y
B
/c) (8)
Here no minus sign has to be inserted, because x
B
is positive.
The figure shows that
x
B
x
A
= 10 m
By Eq. 8 By Eq. 7
c cosh
-1
(y
B
/c) c cosh
-1
(y
A
/c) = 10 m
or
c cosh
-1

+ c cosh
-1
10 m = 0 (9)
This equation must be solved numerically for c. It can be difficult to
solve with the solver on a calculator because the cosh
-1
(u) function is
undefined for u 1, and, as a result, the solver may fail if, during the
solution procedure, an attempt is made to evaluate cosh
-1
(u) for u 1.
(
)
( )
3
y
A
3 m
c
y
B
c
3 m + c
1 m + c
c

1189
6.7 Cables: Catenaries Example 10, page 5 of 6
c = 7.044 m
1 + z
z
3 + z
z
3 + c
c
)
7.044 m
3 + 7.044 m
One approach to finding the solution of Eq. 9 is to define
the left-hand side of the equation as a function,
f(z) z cosh
-1
+ z cosh
-1
10
And then use the calculator to plot f(z) versus z. The point
z = zr where the f(z) curve crosses the z axis is the
approximate root of Eq. 9, that is, where f(zr) = 0.
Another approach to finding the solution of Eq. 9 is simply
to experiment with initial estimates for c while using the
solver on a calculator. For example, since the span was
given as 10 m, we might try c = 10 m, 50 m, 100 m as
successive initial estimates.
The solution to Eq. 9 is
c = 7.044 m
Eq. 7 then gives
x
A
= c cosh (y
A
/c) (Eq. 7 repeated)
= c cosh
= (7.044 m) cosh
= 15.5 m Ans.
( ) ( )
4
( )
(
y
x
B
A
C
3 m
y
A

1190
6.7 Cables: Catenaries Example 10, page 6 of 6
y
x
B
A
C
5 At the low point of the cable, the tension is horizontal, so
it can be calculated from Eq. 5:
To = wc (Eq. 5 repeated)
Given
= [(1 kg/m)(9.81 m/s
2
)](7.044 m)
= 69.1 N Ans.

To
c 7.044 m

1191

7. Friction
1192

7.1 Friction: Basic Applications
1193
7.1 Friction: Basic Applications Procedures and Strategies, page 1 of 1
Procedures and Strategies for Solving Problems Involving Friction:
Basic Applications
1. Draw free-body diagrams, taking care to choose the sense of the
friction forces so that they oppose actual or impending relative
motion at surfaces of contact.
2. Write equilibrium equations for each free body.
3. Count the number of unknowns and compare it with the number of
equilibrium equations.
a) If the numbers are equal, solve the equations and then compare
each friction force, f
i
, with the maximum possible friction force
that the surfaces can produce, f
i-max
N
i
. If f
i
< f
i-max
, then the
bodies are in equilibrium, if f
i
> f
i-max
, then the friction forces
are not large enough to keep the bodies from moving. The
equilibrium equations cannot be satisfied.
b) If the number of unknowns is greater than the number of
equilibrium equations, then you must obtain additional equations
by assuming slip impends at some surfaces (so f
i
= f
i-max
at
surface i) or perhaps tipping impends (thus, for example, the
location of a normal force is known to be at a corner). Solve the
entire set of equilibrium and friction equations, and then check
that your solution is physically possible, for example, that
f
i
f
i-max
everywhere. If the solution is possible, you are done.
Otherwise, you must consider a different case of impending
motion, for example, tipping rather than sliding.
W
f
N
W
N
P
P
Tipping does not
impend, so x > 0.
Assume sliding
impends, so f = fmax
( N)
Tipping impends,
so x = 0, and the
normal force N
acts at the corner.
W
P
x
f

1194
7.1 Friction: Basic Applications Problem Statement for Example 1
1. The uniform ladder is 2-m long and makes an angle
of = 60 with the floor. If the wall at B is smooth and
the coefficient of static friction at A is
A
= 0.3,
determine if the ladder can remain in the position shown.

A
B
Mass of ladder = 10 kg

1195
7.1 Friction: Basic Applications Problem Statement for Example 2
2. The uniform ladder is 2-m long and the wall at B
is smooth. If the coefficient of static friction at A is
A
= 0.2, determine the smallest angle for which
the ladder can remain in the position shown.

A
B
Mass of ladder = 10 kg

1196
7.1 Friction: Basic Applications Problem Statement for Example 3
3. The uniform ladder is 2-m long. The
coefficient of static friction at A is
A
= 0.6 and
at B is
B
= 0.4. Determine the smallest angle, ,
for which the ladder can remain in the position
shown.

A
B
Mass of ladder = 10 kg
6

1197
7.1 Friction: Basic Applications Problem Statement for Example 4
4. Four round pegs A, B, C, and D are attached to
the bracket and loosely straddle the vertical pole.
When a 100-N force is applied as shown, the
bracket rotates slightly and friction forces
develop between pegs B, C, and the pole. If the
coefficient of static friction between the pegs and
the pole is determine the smallest value of
for which the bracket will support the load.
Neglect the effect of the rotation of the bracket
on the distances shown.
100 N
A
B
C
D
300 mm
50 mm
100 mm

1198
7.1 Friction: Basic Applications Problem Statement for Example 5
5. Arm ABC acts as a brake on the wheel. The
coefficient of static friction at B is
B
= 0.4.
Determine the largest moment M that can act on the
wheel without causing it to slip.
O
M
A
B
400 mm
300 mm
C
100 N
Radius = 200 mm
200 mm

1199
7.1 Friction: Basic Applications Problem Statement for Example 6
6. The uniform block is initially at rest when a 10-lb
force is applied. The coefficient of static friction
between the block and the plane is = 0.6.
Determine if the block will move.
1 ft
2 ft
10 lb
20 lb
A
(weight)

1200
7.1 Friction: Basic Applications Problem Statement for Example 7
7. The uniform block is initially at rest when the force P
is applied. The coefficient of static friction between the
block and the plane is = 0.6. Determine the minimum
value of P that will cause the block to move.
2 ft
A
1 ft
(weight)
20 lb
B
P

1201
7.1 Friction: Basic Applications Problem Statement for Example 8
8. The cylinder is intially at rest when a horizontal
force P is applied. The coefficients of static friction
at A and B are

A
= 0.3 and

B
= 0.6. Determine
the minimum value of P that will cause the cylinder
to move.
0.3 m
P
20 kg
A
B
Radius = 0.2 m

1202
7.1 Friction: Basic Applications Problem Statement for Example 9
9. The small block B rests on top of the large block A. The
coefficients of static friction are shown in the figure.
Determine the smallest value of applied force P that will
keep block A from sliding down the inclined plane.
A
B
60 kg
10 kg
P
Cord
Frictionless
pulley
B
= 0.3
= 0.2
30

1203
7.1 Friction: Basic Applications Problem Statement for Example 10
10. The three blocks are stationary when the force P is applied. The
coefficients of static friction for each pair of surfaces are given in the
figure. Determine the smallest value of P for which motion will
occur. The blocks are sufficiently long that tipping will not occur.
C
A
B
10 kg
10 kg
10 kg

AB
= 0.8

BC
= 0.3

C
= 0.15
P

1204
7.1 Friction: Basic Applications Problem Statement for Example 11
25
11. The two cylinders shown are initially at rest when
horizontal forces of magnitude P/2 are applied to the ends
of the axle in the lower cylinder. The coefficients of
static friction for each pair of surfaces are given in the
figure. Determine the largest value of P that can be
applied without moving the cylinders up the inclined
plane.
A
B
C

B
= 0.4

C
= 0.5

A
= 0.6
P/2
Radius of each cylinder = 300 mm
Mass of each cylinder = 50 kg
P/2

1205
7.1 Friction: Basic Applications Example 1, page 1 of 2
Equations of equilibrium:
F

x = 0: N

B
f

A
= 0
F

y
= 0: N

A
98.1 N = 0
M

A
= 0: (98.1 N)(0.5 m) N

B
(1.732 m) = 0
+
+
+
6
1. The uniform ladder is 2-m long and makes an angle of = 60
with the floor. If the wall at B is smooth and the coefficient of
static friction at A is
A
= 0.3, determine if the ladder can remain
in the position shown.

A
B
Mass of ladder = 10 kg
1 m
1 m
N

B
N

A
f

A
(2 m)(sin 60) = 1.732 m
Results of solving the above
equations of equilibrium:
N

A
= 98.1 N
N

B
= 28.3 N
f

A
= 28.3 N
7
Free-body diagram
No friction force is
present because the
wall is smooth.
Because the ladder is
uniform, the weight
acts through the center.
Weight = mg
= (10 kg)(9.81 m/s
2
)
= 98.1 N
The friction force must
be drawn in a direction
opposing the
impending motion.
60
1
2
3
5
4 Impending motion
A
B
(1 m)(cos 60) = 0.5 m

1206
7.1 Friction: Basic Applications Example 1, page 2 of 2
Compute the maximum possible friction force that
the surfaces can develop at point A.
f

A-max


A
N

A
= (0.3)(98.1 N) = 29.4 N
To determine whether or not the ladder will stay in
the original position, the friction force found from the
equilibrium equations, f

A
= 28.3 N, must be
compared with the maximum force that the surfaces
at A can develop:
f

A-max
= 29.4 N
Since
f

A
= 28.3 N < 29.4 N = f

A-max

the surfaces are able to develop enough friction force
and the ladder will stay in equilibrium.
8

1207
7.1 Friction: Basic Applications Example 2, page 1 of 2
Equations of equilibrium:
F

x = 0: N

B
f

A
= 0 (1)
F

y
= 0: N

A
98.1 N = 0 (2)
M

A
= 0: (98.1 N)(1 m) cos N

B
(2 m) sin = 0 (3)
+
+
+
5
2. The uniform ladder is 2-m long and the wall at B
is smooth. If the coefficient of static friction at A is
A
= 0.2, determine the smallest angle for which
the ladder can remain in the position shown.

A
B
Mass of ladder = 10 kg
1 m
1 m
N

B
N

A
f

A

Three equations, but four unknowns: N

B
, f

A
, N

A
, and
An additional equation is needed.
6
Free-body diagram
No friction force is
present because the
wall is smooth.
The friction force
must be drawn in a
direction opposing the
motion.
1
2
4
3 Impending motion
(1 m) cos
(2 m) sin
Weight = mg
= (10 kg)(9.81 m/s
2
)
= 98.1 N


1208
7.1 Friction: Basic Applications Example 2, page 2 of 2
The fourth equation comes from the condition of
impending slip at point A, because if slip is just about to
occur, then the friction force, f

A
, is at its maximum value,
which is

A
N
A
:
f

A
f

A-max


A
N

A
= (0.2)N

A
(4)
Three of the four equations are linear but the moment
equation, Eq. 3, is nonlinear because cos and sin
appear.
M

A
= 0: (98.1 N)(1m) cos
- N

B
(2 m) sin = 0 (Eq. 3 repeated)
The easiest way to solve these equations is to use the
general equation solver on a calculator. Alternatively,
manipulate the equations as follows.
First note that Eq. 2 implies that
N

A
= 98.1 N
Then using this value for N

A
in Eq. 4 gives
f

A
= N

A
= (0.2)(98.1 N)
= 19.62 N
+
Using f

A
= 19.62 N in Eq. 1 gives
N

B
= f

A
= 19.62 N
Using N

B
= 19.62 N in Eq. 3 gives
(98.1 N) cos (19.62 N)(2) sin = 0
Dividing through by cos and rearranging gives

Replacing the left-hand side of this equation by tan
gives
tan = 2.5
which implies
68.2 Ans.
8
sin 98.1
cos (19.62)(2)
= = 2.5
7

1209
7.1 Friction: Basic Applications Example 3, page 1 of 2
Equations of equilibrium:
F

x = 0: N

B
f

A
= 0 (1)
F

y
= 0: N

A
+ f
B
98.1 N = 0 (2)
M

A
= 0: (98.1 N)(1 m) cos f

B
(2 m) cos
N

B
(2 m) sin = 0 (3)
+
+
Mass of ladder = 10 kg
B
A

3. The uniform ladder is 2-m long. The
coefficient of static friction at A is
A
= 0.6 and
at B is
B
= 0.4. Determine the smallest angle, ,
for which the ladder can remain in the position
shown.
+
5
1 m
1 m
N

B
N

A
f

A

Three equilibrium equations but five unknowns: N

A
,
N

B
, f

A
, f

B
, and two more equations are needed.
6
Free-body diagram
Friction force, f

B
, is
present and opposes
the possible motion.
1
3
4
(2 m) sin
f

B
Friction force
opposes the
possible motion.

(2 m) cos
(1 m) cos
Impending
motion
2

Weight = mg
= (10 kg)(9.81 m/s
2
)
= 98.1 N

1210
7.1 Friction: Basic Applications Example 3, page 2 of 2
The two additional equations come from the condition of
impending slip at points A and B, because if slip is just about
to occur, then the friction forces, f

A
and f

B
, are at their
maximum values,

A
N

A
and N

B
f

A
f

A-max


A
N

A
= 0.6N

A
(4)
f

B
f

B-max


B
N

B
= 0.4N

B
(5)
Four of the five equations are linear but the moment equation
Eq. 3 is nonlinear (sin and cos are present). To solve these
equations, use the general equation solver on your calculator or
manipulate the equation algebraically and use a trig identity
such as tan = sin /cos .
Results of solving the five equations (three equilibrium and
two friction equations) given above:
N

A
= 79.1 N
N

B
= 47.5 N
f

A
= 47.5 N
f

B
= 19.0 N
= 32.3 Ans.
7

1211
7.1 Friction: Basic Applications Example 4, page 1 of 3
4. Four round pegs A, B, C, and D are attached to
the bracket and loosely straddle the vertical pole.
When a 100-N force is applied as shown, the
bracket rotates slightly and friction forces
develop between pegs B, C, and the pole. If the
coefficient of static friction between the pegs and
the pole is determine the smallest value of
for which the bracket will support the load.
Neglect the effect of the rotation of the bracket
on the distances shown.
100 N
A
B
C
D
300 mm
50 mm
100 mm
Bracket rotates a
small amount
Peg A loses contact with the pole
Movement of bracket exaggerated for clarity
1
100 N
D
B
A
C
Peg D loses contact with the pole

1212
7.1 Friction: Basic Applications Example 4, page 2 of 3
2
P = 100 N
100 mm
f

B
N

B
N

C
f

C
Free-body diagram
300 mm 50 mm
Impending
motion of
bracket
3
The friction forces, f

B
and f

C
,
resist the motion by pushing the
bracket up.
4
As the bracket inclines slightly, the pegs at A and D
lose contact with the pole. That is why no forces
appear at A and D on the free-body diagram.
Equations of equilibrium:
F

x = 0: N

B
N

C
= 0 (1)
F

y
= 0: f

B
+ f

C
100 N = 0 (2)
M
C
= 0: (100 N)(300 mm) + f

B
(50 mm)
N

B
(100 mm) = 0 (3)
+
+
+
7
6
The normal forces,
N

B
and N

C
, are
directed from the
pole to the pegs.
5
B
C

1213
7.1 Friction: Basic Applications Example 4, page 3 of 3
There are only three equations of equilibrium but four
unknowns (f

B
, N

B
, f

C
, and N

C
), so at least one more
equation is needed. The additional equation comes from the
condition of impending slip at B, but if the bracket is going
to slip at B, it will also slip at C. So we have two additional
equations and one additional unknown, :
f

B
= f

B-max
N

B
(4)
f

C
= f

C-max
N

C
(5)
Solving Eqs. 1-5 gives the results below (Note that Eqs. 4
and 5 are nonlinear because multiplies N

B
and N

C
):
f

B
= 50 N
N

B
= 325 N
f

C
= 50 N
N

C
= 325 N
= 0.154 Ans.
8

1214
7.1 Friction: Basic Applications Example 5, page 1 of 2
M
O

y
O

x
200 mm
N

B
f

B
Free-body diagram of
wheel.
1
Impending motion
of point on outer
surface of wheel
The friction force f

B

opposes the motion.
2
3
Equation of moment equilibrium for the wheel
(Since we were not asked to compute the reactions
O

x and O

y
, we do not need to write the
force-equilibrium equations.):
M

O
= 0: f

B
(200 mm) M = 0 (1)
4
+
O
200 mm
Radius = 200 mm
100 N
C
300 mm
400 mm
B
A
M
O
5. Arm ABC acts as a brake on the wheel. The
coefficient of static friction at B is
B
= 0.4.
Determine the largest moment M that can act on the
wheel without causing it to slip.

1215
7.1 Friction: Basic Applications Example 5, page 2 of 2
The third equation follows from the condition
that slip impends at B:
f

B
f

B-max


B
N

B
= 0.4N

B
(3)
Solving Eqs. 1-3 simultaneously yields
f

B
= 87.5 N
N

B
= 218.8 N
M = 17 500 Nmm = 17.5 Nm Ans.

A

y
A

x
A
5 Free-body diagram of arm ABC.
f

B
N

B
200 mm
400 mm
300 mm
100 N
7
+
Equation of moment equilibrium for the brake (Since we
were not asked to compute the reactions A

x and A

y
, we do
not need to write the force-equilibrium equations.):
M

A
= 0: f

B
(200 mm) N

B
(400 mm)
+ (100 N)(300 mm + 400 mm) = 0 (2)
Thus far we have two equations but three unknowns (M, f

B
, and N

B
), so another equation is needed.
8
6 The sense of the friction
force on the brake can be
determined by Newton's
Third Law (equal and
opposite to the force on
the wheel).

1216
7.1 Friction: Basic Applications Example 6, page 1 of 4
6. The uniform block is initially at rest when a 10-lb
force is applied. The coefficient of static friction
between the block and the plane is = 0.6.
Determine if the block will move.
1 ft
2 ft
10 lb
20 lb
A
(weight)

1217
7.1 Friction: Basic Applications Example 6, page 2 of 4
The resultant of the
distributed force f
d
is f.
5
The distributed friction force f
d

opposes possible slip to the
right; the distributed normal
force N
d
opposes possible
tipping of the block.
As the block is pushed to the right by the 10-lb force, the floor opposes the
possible motion by providing a distributed reaction force. The component of
this reaction force parallel to the floor is the distributed friction force f
d
, and
the component normal to the floor is the distributed normal force N
d
.
3
2
The resultant of the distributed force
N
d
is N. Because N
d
opposes possible
tipping of the block, it is not uniform
but is greater near the right-hand side
of the base of the block to balance the
tendency to tip. Thus the resultant N
does not act at the middle of the base
but instead acts at some unknown
distance, x, from the middle.
6
B
N
d
f
d
1 Free-body diagram
A
20 lb
10 lb
1 ft
0.5 ft
B
N
f
4
Free-body diagram showing resultant forces
2 ft
A
20 lb
10 lb
x

1218
7.1 Friction: Basic Applications Example 6, page 3 of 4
Equations of equilibrium:
F

x = 0: 10 lb f = 0
F

y
= 0: N 20 lb = 0
M

A
= 0: (20 lb)(0.5 ft) (10 lb)(2 ft) + N(0.5 ft + x) = 0
Solving these equations gives
f = 10 lb
N = 20 lb
x = 1 ft

These are the values required if the system is to
stay in equilibrium, that is, not move. To
determine if the system can produce the 10-lb
friction force f required to keep the system in
equilibrium, we have to compare f with the
maximum possible value of the friction force:
f

max
N = (0.6)(20 lb) = 12 lb
Because f = 10 lb is less than the 12 lb maximum
possible force, the surfaces can develop enough
force to balance forces in the x direction (thus the
block will not slide to the right).
+
+
7
8
9
+

1219
7.1 Friction: Basic Applications Example 6, page 4 of 4
We next consider whether or not the block will tip.
Recall that solving the equilibrium equations gave the result
x = 1 ft. That is, to maintain equilibrium, the normal force
N must act at the location shown, 0.5 ft to the right of the
block. But this is impossible because N is the normal force
from the ground acting up on the block; the farthest N can
act is at the right hand corner, B. Thus the block will tip
because N cannot act far enough to the right to prevent it.
N (impossible location
because outside the base of
the block)
0.5 ft 0.5 ft
x = 1 ft
20 lb
10
f
A
B
10 lb

1220
7.1 Friction: Basic Applications Example 7, page 1 of 6
7. The uniform block is initially at rest when the force P
is applied. The coefficient of static friction between the
block and the plane is = 0.6. Determine the minimum
value of P that will cause the block to move.
2 ft
A
1 ft
(weight)
20 lb
B
P

1221
7.1 Friction: Basic Applications Example 7, page 2 of 6
P
1 ft
20 lb
A
Free-body diagram 1
f
d
N
d
P
20 lb
A
2 ft
Free-body diagram showing resultant forces 3
The resultant of the
distributed force f
d
is f.
f
4
As the block is pushed to the right by the force P, the floor opposes the
possible motion by providing a distributed reaction force. The component
of this reaction force parallel to the floor is the distributed friction force, f
d
,
and the component normal to the floor is the distributed normal force N
d
.
2
The resultant of the distributed force N
d

is N. Because N
d
opposes possible
tipping of the block, it is not uniform
but is greater near the right-hand side of
the base of the block to balance the
tendency to tip. Thus the resultant N
does not act at the middle of the base
but instead acts at some unknown
distance, x, from the middle.
5
N
B
B
0.5 ft x

1222
7.1 Friction: Basic Applications Example 7, page 3 of 6
1 ft
A
B
20 lb
(weight)
P
Equations of equilibrium:
F

x = 0: P f = 0 (1)
F

y
= 0: N 20 lb = 0 (2)
M

A
= 0: (20 lb)(0.5 ft) P(2 ft)
+ N(0.5 ft + x) = 0 (3)
Three equations but four unknowns (P, f, N and x),
so one more equation is needed.
The fourth equation comes from considering
possible impending motion. There are two cases to
consider: sliding and tipping.
Case 1: Sliding
6
7
8
9
+
+
+

1223
7.1 Friction: Basic Applications Example 7, page 4 of 6
P
20 lb
(weight)
B
A
We have to analyze each case separately. Let's (arbitrarily)
choose Case 1 first. If sliding impends, then
f = f

max N = 0.6N (4)
Solving Eqs. 1-4 simultaneously gives

P = 12 lb
N = 20 lb
f = 12 lb

x = 1.2 ft
Case 2: Tipping 10 11
12

1224
7.1 Friction: Basic Applications Example 7, page 5 of 6
B
20 lb
P = 12 lb
A
0.5 ft
x = 1.2 ft
N = 20 lb
f = 12 lb
But this diagram shows that the only way the equilibrium
equations for Case 1 can be satisfied is if the normal force N
lies to the right of the block (x = 1.2 ft). Since this is
impossible, the Case 1 assumption that sliding impends must
be incorrect.
Free body diagram for Case 1 (Sliding impends) 13
14

1225
7.1 Friction: Basic Applications Example 7, page 6 of 6
f
x
A
P
20 lb
B
Since the block is just about to tip, it loses contact with the
floor except at the corner B, where the normal force N is
concentrated. Since N acts at the corner, we know
x = 0.5 ft (5)
Solving the equilibrium equations, Eqs. 1, 2, and 3,
simultaneously with Eq. 5 gives

f = 5 lb
N = 20 lb
P = 5 lb Ans.
Since there were only two possibilities, sliding and tipping, and
we eliminated sliding, we know that the above result P = 5 lb is
correct. However, we can also check our work by verifying
that the friction force f is less than the maximum possible
value:
f = 5 lb < f

max N = (0.6)(20 lb) = 12 lb. (OK)
Free body diagram for Case 2 (Tipping impends) 15
16
0.5 ft 0.5 ft
N

1226
7.1 Friction: Basic Applications Example 8, page 1 of 3
8. The cylinder is initially at rest when a horizontal
force P is applied. The coefficients of static friction
at A and B are

A
= 0.3 and

B
= 0.6. Determine
the minimum value of P that will cause the cylinder
to move.
0.3 m
P
20 kg
A
B
Radius = 0.2 m
A
0.3 m
P
B
f

A
N

A
f

B
N

B
Weight = mg
= (20 kg)(9.81 m/s
2
)
= 196.2 N
Free-body diagram
Possible motion of
point B on cylinder.
Force P tends to
rotate the cylinder
clockwise.
The friction force from
the wall opposes the
motion of point B on
the cylinder.
Possible motion of point A.
The friction force from the floor
opposes the motion of point A on
the cylinder.
1
2
3
4
5
0.2 m

1227
7.1 Friction: Basic Applications Example 8, page 2 of 3
+
6
Case 2 Case 1
The cylinder rolls up the
wall without slipping.
The cylinder spins
about its center.
B
A
B
A
7 We have to analyze each case separately. Let's
(arbitrarily) choose Case 1 first. Thus if the cylinder
is about to slip about its center, then slip impends
simultaneously at points A and B, so
f

A
= f

A-max


A
N

A
= 0.3N

A
(4)
f

B
= f

B-max


B
N

B
= 0.6N

B
(5)
Solving Eqs. 1-5 simultaneously gives
P = 554 N
f

A
= 34.6 N
f

B
= 312 N
N

A
= 115 N
N

B
= 519 N
A negative normal force, N

A
, is impossible (The
floor can't pull down on the cylinder), so the
assumption of slip at both A and B must be wrong.
Equilibrium equations
F

x = 0: P + f

A
N

B
= 0 (1)
F

y
= 0: 196.2 N + f

B
+ N

A
= 0 (2)
M

A
= 0: f

B
(0.2 m) + N
B
(0.2 m) P(0.3 m) = 0 (3)
There are three equations and five unknowns (P, f

A
, N

A
, f

B
, N

B
),
so two more equations are needed. The two additional equations
come from considering possible impending motion. There are
two cases to consider:
+
+

1228
7.1 Friction: Basic Applications Example 8, page 3 of 3
Next consider Case 2 the cylinder is about to roll up the wall. Thus
the cylinder is about to lose contact with the floor at point A, and so
the friction and normal forces there are zero:
f

A
= 0 (6)
N

A
= 0 (7)
Solving the equilibrium equations, Eqs. 1, 2, and 3, simultaneously
with Eqs. 6 and 7 gives
f

A
= 0
N

A
= 0
f

B
= 196 N
N

B
= 392 N
P = 392 N Ans.
Since there were only two possibilities, spinning about the cylinder
center or rolling up the wall, and we eliminated spinning, the above
result P = 392 N must be correct. However, we can also check our
work by comparing the friction force, f

B
, with the maximum possible
value:
f

B
= 196 N < f

B-max


B
N

B
= (0.6)(392 N) = 235 N (OK)
8

1229
7.1 Friction: Basic Applications Example 9, page 1 of 4
9. The small block B rests on top of the large block A. The
coefficients of static friction are shown in the figure.
Determine the smallest value of applied force P that will
keep block A from sliding down the inclined plane.
A
B
60 kg
10 kg
P
Cord
Frictionless
pulley
B
= 0.3
= 0.2
30

1230
7.1 Friction: Basic Applications Example 9, page 2 of 4
Free-body diagram of block B
Tension in cord, T
Impending motion of
block B relative to
block A (If block A
moves down the plane,
block B must move up
the plane.)
Normal force
from block A
The friction force from block A opposes the
impending motion of block B up the incline.
The numerical value of
will be calculated later.
It's convenient to use an inclined
xy coordinate system.
P
B
30
x
y
N
B
f
B
1
4
3
5
2
Weight = mg
= (10 kg)(9.81 m/s
2
)
= 98.1 N
+
+
Equations of equilibrium for block B. We assume that the blocks will
not tip because they are much longer than they are high; thus no
moment equation is needed (Since no dimensions are given, we could
not write a moment equation even if we wanted to).
F

x =0: P T + f

B
+ (98.1 N) sin = 0 (1)
F

y
= 0: N

B
(98.1 N) cos = 0 (2)
6

1231
7.1 Friction: Basic Applications Example 9, page 3 of 4
60
30
= 90 60 = 30
x
y
Geometry 7
30
N
B
11
9
13
12
10
8
N
A
f
B
T
y
x
= 30
A
f
A
Weight = mg
= (60 kg)(9.81 m/s
2
)
= 588.6 N
[Weight of block A alone (Note
that the weight of block B is not
included because block B is not
part of this free-body. The effect
of the weight of block B is
transmitted through the normal
force, N

B
.)]
Impending motion
of block A relative
to block B.
Impending motion of block A
relative to inclined plane.
Normal force from
inclined plane
Friction force from
inclined plane opposes
motion of block A.
Friction force from block B opposes
motion of block A.
Free-body diagram of block A
30

1232
7.1 Friction: Basic Applications Example 9, page 4 of 4
14 Equilibrium equations for block A
F

x =0: (588.6 N) sin 30 f
A
f

B
T = 0 (3)
F

y
= 0: (588.6 N) cos 30 + N

A
N

B
= 0 (4)
Four equations in six unknowns (T, P, f

A
, N

A
, f

B
, N

B
). Two
more equations come from the condition of impending sliding
between the blocks and between block A and the plane:
f

A
f

A-max


A
N
A
= 0.2N
A
(5)
f

B
f

B-max


B
N

B
= 0.3N

B
(6)
Solving Eqs. 1-6 simultaneously gives
f

A
= 119 N
N

A
= 595 N
f

B
= 25 N
N

B
= 85 N
T = 150 N

P = 75 N Ans.
+
+
Free body diagram of block A repeated
f
A
A
x
y
T
f
B
N
A
N
B
30
= 30
588.6 N

1233
7.1 Friction: Basic Applications Example 10, page 1 of 8
10. The three blocks are stationary when the force P is applied. The
coefficients of static friction for each pair of surfaces are given in the
figure. Determine the smallest value of P for which motion will
occur. The blocks are sufficiently long that tipping will not occur.
Equilibrium equations for block A:
F

x = 0: P f

AB
= 0 (1)
F

y
= 0: N

AB
98.1 N = 0
The last equation gives
N

AB
= 98.1 N (2)
+
+
4
C
A
B
10 kg
10 kg
10 kg

AB
= 0.8

BC
= 0.3

C
= 0.15
P
P
A
N

AB
f

AB
Weight = mg
= (10 kg)(9.81 m/s
2
)
= 98.1 N
Impending motion of
block A relative to B
Friction opposes the motion
2
3
Free-body diagram of block A
1

1234
7.1 Friction: Basic Applications Example 10, page 2 of 8
Equilibrium equations for block B:
F

x = 0: f

AB
f

BC
= 0 (3)
F

y
= 0: 98.1 N 98.1 N + N

BC
= 0
The last equation gives
N

BC
= 196.2 N (4)
+
+
10
B
N

AB
= 98.1 N
f

AB
Impending motion of block B
relative to block A (An observer
on A would see B moving in this
direction.)
Friction force opposes
relative motion
8 9
Free-body diagram of block B
5
Weight = 98.1 N
f

BC
N

BC
6 Impending motion of
block B relative to C
7 Friction force opposes
relative motion

1235
7.1 Friction: Basic Applications Example 10, page 3 of 8
Equilibrium equations for block C:
F

x = 0: f

BC
f

C
= 0 (5)
F

y
= 0: 98.1 N 196.2 N + N

C
= 0
The last equation gives
N

C
= 294.3 N (6)
+
+
16
C
N

BC
= 196.2 N
f

BC
Impending motion of block C
relative to block B (An observer
on B would see C moving in this
direction.)
Friction force opposes
relative motion
14
15
Free-body diagram of block C 11
Weight = 98.1 N
f

C
N

C
12 Impending motion of
block C relative to
floor
13 Friction force opposes
relative motion

1236
7.1 Friction: Basic Applications Example 10, page 4 of 8
We now have six equilibrium equations but seven
unknowns (P, f

AB
, N

AB
, f

BC
, N

BC
, f

C
, N

C
), so another
equation is needed.
17 The seventh equation comes from the condition of
impending slip. We have to consider three cases:
18
A
B
C
Case 1
Impending
motion
Stationary
C
Stationary
B
A
Impending motion: blocks A and B move together
Case 2
C
B
A
Case 3
Impending
motion: blocks
A, B and C
move together

1237
7.1 Friction: Basic Applications Example 10, page 5 of 8
Slip impends so
f

AB
= f

AB-max



AB
N

AB
= (0.8)(98.1 N) = 78.5 N (7)
We have to check to see if the surfaces of contact between
blocks B and C develop enough friction force to keep block B
stationary.
20
Analyze each case separately. 19
Stationary
C
A
B
Case 1
For block B, Eq. 3 is
f

AB
f

BC
= 0
So for equilibrium,
f

BC
= f

AB
= 78.5 N
Let's compare this with the maximum possible friction
force:
f

BC-max


BC
N

BC

= (0.3)(196.2 N) = 58.9 N (8)
So the surfaces can develop only 58.9 N while 78.5 N
are needed for equilibrium. Thus block B will move,
contrary to our assumption for Case 1.
by Eq. 2
21
22

1238
7.1 Friction: Basic Applications Example 10, page 6 of 8
Slip impends so
f

BC
= f

BC-max


= 58.9 N
We have to check to see if the surfaces of contact between
block C and the ground develop enough friction force to
keep block C stationary.
23
For block C, Eq. 5 is
f

BC
f

C
= 0
So for equilibrium,
f

C
= f
BC
= 58.9 N
Compare this with the maximum possible friction force
f

C-max


C
N

C

= (0.15)(294.3 N) = 44.1 N (9)
So the surfaces can develop only 44.1 N while
58.9 N are needed for equilibrium. Thus block C will
move, contrary to our assumption for Case 2.
Case 2
C
B
A
Stationary
Impending
motion
together
by Eq. 8
by Eq. 6
24
25

1239
7.1 Friction: Basic Applications Example 10, page 7 of 8
Slip impends so
f

C
= f

C-max
= 44.1 N (10)
We don't have to check that the surfaces of contact
between blocks A and B and between B and C develop
enough friction to keep A and B in equilibrium, since there
were only three cases of possible motion, and we showed
that the first two cases were impossible. Nonetheless, we
can verify that our work is correct by showing that the
friction forces acting between A and B and between B and
C are less than their maximum possible values.
26 Eq. 5 gives f

BC
:
f

BC
f

C
= 0
Thus
f

BC
= f

C
= 44.1 N (11)
and so
by Eq. 8
44.1 N = f

BC
< f

BC-max
= 58.9 N (OK)
Eq. 3 gives f

AB
:
f

AB
f

BC
= 0
Thus
f

AB
= f

BC
= 44.1 N (12)
and so
by Eq. 7
44.1 N = f

AB
< f

AB-max
= 78.5 N (OK)
by Eq. 9
Case 3
B
C
A
Impending
motion together
by Eq. 10
27
28

1240
7.1 Friction: Basic Applications Example 10, page 8 of 8
A
B
C
Thus the surfaces of contact between blocks A
and B and between B and C can develop
enough friction to keep blocks A, B, and C
moving together as a unit.
Finally, we can calculate P from Eq. 1:
P f

AB = 0
or,

P = f

AB = 44.1 N Ans.
Why didn't we consider a case like this?
Impending motion of A relative to B
Impending motion of B relative to C
Answer: No matter what the impending motion is,
there are only seven unknown forces (f

AB, N

AB, f

BC, N

BC,
f

C, N

C, and P). Since these seven unknowns must
satisfy the six equations of equilibrium, the unknowns
can be chosen to satisfy only one additional equation
a friction equation. In the unlikely event that the
masses and 's just happen to have values such that the
seven forces simultaneously satisfy the six equilibrium
equations and two friction equations, then one of the
eight equations must be redundant.
Applying this reasoning to Case 4, we see that if forces
exist that satisfy Case 4's equations, then these forces
must be identical to the forces satisfying the equations
for Case 1 (slip between A and B) and Case 2 (slip
between B and C). Since solving Case 4 would give
the same answer as solving Case 1 (or Case 2), we
don't have to consider Case 4. A similar argument can
be made for other possible motions.
Case 4
Stationary
by Eq. 12
29
30
31
32

1241
7.1 Friction: Basic Applications Example 11, page 1 of 9
25
11. The two cylinders shown are initially at rest when
horizontal forces of magnitude P/2 are applied to the ends
of the axle in the lower cylinder. The coefficients of
static friction for each pair of surfaces are given in the
figure. Determine the largest value of P that can be
applied without moving the cylinders up the inclined
plane.
A
B
C

B
= 0.4

C
= 0.5

A
= 0.6
P/2
Radius of each cylinder = 300 mm
Mass of each cylinder = 50 kg
P/2

1242
7.1 Friction: Basic Applications Example 11, page 2 of 9
y
x
N
C
f
C
C
O
f
A
N
A
25
A
P
Free-body diagram of lower cylinder
The friction force from
the upper cylinder
opposes the relative
motion of point C on the
lower cylinder.
The friction force from the
plane opposes the motion
of point A on the cylinder.
Impending motion of point A on
cylinder. The x component of
the applied force, P cos , is
pushing the cylinder up the
plane.
It is convenient to use an
inclined xy coordinate-system.
Radius = 300 mm
The numerical values of and will be
calculated later.
Weight = mg
= (50 kg)(9.81 m/s
2
)
= 490.5 N
Impending motion of point C on lower
cylinder relative to upper cylinder (An
observer on the upper cylinder would
see this motion as the lower cylinder
moves).
1
6
8
7
2
4
3
5

1243
7.1 Friction: Basic Applications Example 11, page 3 of 9
Equilibrium equations for cylinder:
F

x = 0: (490.5 N) sin P cos + f

A
+ N

C
= 0 (1)
F

y
= 0: 90.5 N) cos P sin + f

C
+ N

A
= 0 (2)
M

O
= 0: f

A
(300 mm) f

C
(300 mm) = 0 (3)
Geometry
= 90 65
= 25
+
+
= 25
y
x
25
65
9
10
+
490.5 N
Radius = 300 mm
P
A
N
A
f
A
O
C
f
C
N
C
x
y
25
Free-body diagram of lower cylinder repeated

1244
7.1 Friction: Basic Applications Example 11, page 4 of 9
B
N
B
f
B
f
C
C
P
= 25
O
x
y
Weight = 490.5 N
Free-body diagram of upper cylinder
Radius = 300 mm
The friction force
from the plane
opposes the motion
up the plane.
Impending motion of point B as
normal force N

C
pushes the
upper cylinder up the plane.
Impending motion of point C on upper
cylinder relative to lower cylinder (An
observer on the lower cylinder would
see this motion as the upper cylinder
moves).
The friction force from the lower
cylinder opposes the relative
motion of point C on the upper
cylinder.
.
11
15
14
12
13
N
C

1245
7.1 Friction: Basic Applications Example 11, page 5 of 9
Equilibrium equations
F

x = 0: (490.5 N) sin 25 + f

B
N

C
= 0 (4)
F

y
= 0: (490.5 N) cos 25 f

C
+ N

B
= 0 (5)
M

O'
= 0: f

B
(300 mm) f

C
(300 mm) = 0 (6)
Thus far we have six equation but seven unknowns (P, f

A
, N

A
,
f

B
, N

B
, f

C
, N

C
), so another equation is needed.
The seventh equation comes from the condition of impending
slip. We have to consider only two cases:
1. slip occurs at point B (and simultaneously rolling
occurs about points A and C).
2. slip occurs at point C (and simultaneously rolling
occurs about points A and B).
Slip at point A will be discussed later.
+
+
16
17
+
N
C
Radius = 300 mm
490.5 N
y
x
O
25
P
C
f
C
f
B
N
B
B
Free-body diagram of upper cylinder repeated

1246
7.1 Friction: Basic Applications Example 11, page 6 of 9
Case 1
O'
O
A
B
C
A
B
C
O
O'
Displacement of point O
(Point O moves up the plane)
Before motion
After motion
Rolling without slipping (The radial
line OA on the lower cylinder
rotates through the same angle, , as
the radial line O'C on the upper
cylinder.)
Slip
For impending slip at B,
f

B
= f

B-max


B
N

B
= 0.4N

B
(7)
20
19
18

1247
7.1 Friction: Basic Applications Example 11, page 7 of 9
Solving Eqs. 1-7 simultaneously gives
f

A
= 296 N N

A
= 618 N
f

B
= 296 N N

B
= 741 N
f

C
= 296 N N

C
= 504 N
P = 1111 N
We must check that the surfaces at A and C can provide enough friction
force to prevent slip and allow rolling:
f

A
296 f

A-max


A
N

A
= (0.6)(618 N) = 371 N (OK)
f

C-max


C
N

C
= (0.5)(504 N) = 252 N (Not enough! We need
f

C
= 296 N for equilibrium.)
So the assumption of impending slip at B is wrong.
21
22

1248
7.1 Friction: Basic Applications Example 11, page 8 of 9
Case 2 (Slip at C, rolling at A
and B)
O'
O
A
B
C
A
B
C
O
O'
Displacement
of point O
Before motion
After motion
Rolling without
slipping
Slip
For impending slip at C,
f

C
= f

C-max


C
N

C
= 0.5N

C
(8)
26
24
23
Rolling without
slipping
25

1249
7.1 Friction: Basic Applications Example 11, page 9 of 9
Solving the six equilibrium equations, Eqs. 1-6, plus Eq. 8
yields
f

A
= 207 N N

A
= 624 N
f

B
= 207 N N

B
= 652 N
f

C
= 207 N N

C
= 415 N
P = 915 N Ans.
The above answers must be correct since we eliminated
the only other possible case where slip impends. But we
can check our results by verifying that the friction forces
at A and B are less than their maximum possible values.
f

A
= 207 N f

A-max


A
N

A
= (0.6)(624 N)
= 374 N (OK)
f

B
= 207 N f

B-max


B
N

B
= (0.4)(652 N)
= 261 N (OK)
What about slip occurring at point A only? Well if the
lower cylinder moves, then the upper cylinder must also
move. But the only way that the upper cylinder can move
is if either 1) it slips at point B, or 2) it slips at point C.
Thus the case of slip impending at point A alone is
impossible and does not have to be considered.
What about simultaneous slip at A and B? Answer: we
have already found values of the seven unknowns in the
problem that satisfy the six equilibrium equations and the
equation for slip at B. In the unlikely case that the seven
values happen to satisfy an eighth equation (slip at A), then
that equation must be redundant, and the solution for the
eight equations is the same as we have already found for
the seven equations.
An analogous statement can be made for the case of
simultaneous slip at A and C.
27 29
28

1250

7.2 Wedges
1251
7.2 Wedges Procedures and Strategies, page 1 of 1
Procedures and Strategies for Solving Problems
Involving Friction: Wedges
1. Draw a free-body diagram of the bodies involved and write
equilibrium equations.
2. Show friction forces opposing impending motion. If a wedge
is to be inserted or removed, then you know that slip impends
on the surfaces of the wedge, and thus f = f max ( ) on
these surfaces. If you do not know if slip impends, then solve
the friction-wedge problem by using the same approach used
for basic applications of friction equations..

1252
7.2 Wedges Problem Statement for Example 1
1. If the coefficient of static friction equals 0.3 for all
surfaces of contact, determine the smallest value of force P
necessary to raise the block A. Neglect the weight of the
wedge B.
A
300 kg
B
P
10

1253
7.2 Wedges Problem Statement for Example 2
10
10
E
C
D
A
B
2. Wedges A and B are to be glued together. Determine the
minimum coefficient of static friction required, if clamp
CDE is to be able to hold the wedges in the position shown,
while the glue dries.

1254
7.2 Wedges Problem Statement for Example 3
3. Determine the smallest values of forces P

1
and P

2

required to raise block A while preventing A from
moving horizontally. The coefficient of static friction
for all surfaces of contact is 0.3, and the weight of
wedges B and C is negligible compared to the weight
of block A.
2 kip
A
B
C
P
1
P
2
7

1255
7.2 Wedges Problem Statement for Example 4
4. If the coefficient of static friction for all surfaces of
contact is 0.25, determine the smallest value of the forces P
that will move wedge B upward.
200 kg
B
75 75 20 kg
20 kg
A C P
P

1256
7.2 Wedges Problem Statement for Example 5
5. The cylinder D, which is connected by a pin at A to the
triangular plate C, is being raised by the wedge B. Neglecting the
weight of the wedge and the plate, determine the minimum force
P necessary to raise the cylinder if the coefficient of static friction
is 0.3 for the surfaces of contact of the wedge.
400 lb
r = 10 in
D
A
C
B
10
P

1257
7.2 Wedges Problem Statement for Example 6
6. To split the log shown, a 120-lb force is applied to the top of the
wedge, which causes the wedge to be about to slip farther into the log.
Determine the friction and normal forces acting on the sides of the
wedge, if the coefficient of static friction is 0.6. Also determine if the
wedge will pop out of the log if the force is removed. Neglect the
weight of the wedge.
120 lb
Wedge angle = 8

1258
7.2 Wedges Problem Statement for Example 7
7. The end A of the beam needs to be raised slightly to make it level. If
the coefficient of static friction of the contact surfaces of the wedge is
0.3, determine the smallest value of the horizontal force P that will raise
end A. The weight and size of the wedge are negligible. Also, if the
force P is removed, determine if the wedge will remain in place, that is, is
the wedge self-locking?
8
P
60 lb/ft
B A
15 ft

1259
7.2 Wedges Example 1, page 1 of 4
1. If the coefficient of static friction equals 0.3 for all
surfaces of contact, determine the smallest value of force P
necessary to raise the block A. Neglect the weight of the
wedge B.
A
300 kg
B
P
10

1260
7.2 Wedges Example 1, page 2 of 4
A
(300 kg)(9.81 m/s
2
)
= 2.943 kN
f
A
f
AB
y
x
N
A
Free-body diagram
of block A
Impending motion of
left surface of block A
relative to wall
The friction force from the
wall opposes the relative
motion of block B.
Equations of equilibrium for block A
F

x = 0: N

A
f

AB
cos 10 N

AB
sin = 0 (1)
F

y
= 0: f

A
f

AB
sin 10 N

AB
cos 2.943 kN = 0 (2)
Impending motion so,
f

A
= f
A-max
N

A
= 0.3N

A
(3)
f

AB
= f
AB-max
N

AB
= 0.3N

AB
(4)
Impending motion of lower surface of block A relative
to block B (This is the motion that an observer sitting
on block B would see as he observes block A move
past.)
The friction force from block B opposes the relative
motion of block A.
1
2
3
6
4
5
+
+
N
AB
10

1261
7.2 Wedges Example 1, page 3 of 4
Geometry
Using = 10 in Eqs. 1- 4 and solving
simultaneously gives
f

A
= 0.523 N

N

A
= 1.743 N
f

AB
= 1.115 N
N

AB
= 3.716 N
The friction force f
AB
from
block A opposes the
relative motion of block B.
Impending motion
of top surface of
block B relative to
block A.
Impending motion of lower
surface of block B relative to
floor
Free-body diagram of wedge B
N

AB
= 3.716 N
f

AB
= 1.115 N
The friction force f
B
from the
floor opposes the relative motion
of block B.
10
80
P
B
f
B
N
B
10
7
8
10
11
13
12
9
= 90 80 = 10
10
+
+
14 Equations of equilibrium for block B
F

x = 0: f

B
+ (1.115 N) cos 10 + (3.716 N) sin 10 P = 0 (5)
F

y
= 0: N

B
+ (1.115 N) sin 10 (3.716 N) cos 10 = 0 (6)

1262
7.2 Wedges Example 1, page 4 of 4
Slip impends between block B and the floor, so
f

B
= f
B-max
N

B
= 0.3N

B
(7)
Solving Eqs. 5, 6, and 7 simultaneously gives
f

B
= 1.04 N
N

B
= 3.47 N
P = 2.78 N Ans.
15

1263
7.2 Wedges Example 2, page 1 of 3
10
10
E
C
D
A
B
2. Wedges A and B are to be glued together. Determine the
minimum coefficient of static friction required, if clamp
CDE is to be able to hold the wedges in the position shown,
while the glue dries.

1264
7.2 Wedges Example 2, page 2 of 3
Equations of equilibrium
F

x = 0: 2N sin 2f cos 10 = 0 (1)
F

y
= 0: N cos N cos f sin 10 + f sin 10 = 0 (2)
7
+
+
N
A
B
10
10
4
2
1
6
5
3
Impending motion of
lower surface of wedge
B relative to the lower
jaw of the clamp.
Impending motion of
upper surface of wedge A
relative to the upper jaw
of the clamp (The wedge
is just about to slip out
from the jaws of the
clamp.)
Because of symmetry, the normal force N and friction
force f acting on the bottom of the wedge are given the
same variable names as those on the top.
The friction force from the
lower jaw of the clamp opposes
the relative motion of block B.
The friction force from the
upper jaw of the clamp opposes
the relative motion of block A.
Free-body diagram
of wedges
N
f
f

1265
7.2 Wedges Example 2, page 3 of 3
10
=10
Eqs. 1 and 3 constitute two equations in three
unknowns (f, N, and ), so it appears that we can't
solve the problem. But we have not been asked to
find f and N, only . Thus solve Eq. 1 with = 10
for the ratio f/N:
= tan 10 (4)
Also solve Eq. 3 for f/N:
= (5)
Eqs. 4 and 5 imply that
= tan 10
= 0.176 Ans.
This result is independent of the clamp force N. That
is, no matter how strong the clamp spring is, if the
coefficient of friction is less than 0.176, then the
clamp will slip.
Since we have already used symmetry in
labeling the forces on the free-body diagram, the
F

y
equilibrium equation degenerates to 0 = 0
and gives us no new information.
Slip impends between both jaws of the clamp and
the wedge, and thus
f = f

max N (3)
8
10
11
Geometry 9
90 10 = 80
N
f
N
f

1266
7.2 Wedges Example 3, page 1 of 3
7
3. Determine the smallest values of forces P

1
and P

2

required to raise block A while preventing A from
moving horizontally. The coefficient of static friction
for all surfaces of contact is 0.3, and the weight of
wedges B and C is negligible compared to the weight
of block A.
Free-body diagram of block A
Equations of equilibrium for block A
F

x = 0: f

AB
= 0 (1)
Therefore, f

AB
= 0
F

y
= 0: N

AB
2 kip = 0 (2)
Solving gives
N

AB
= 2 kip
The friction force f
AB
has to be zero, since we know that block A is
not to move horizontally and no other horizontal force acts. In fact, we
could have just shown f

AB
= 0 on the free body initially.
2 kip
A
B
C
P
1
P
2
A
2 kip
1
2
3
+
+
f
AB
N
AB

1267
7.2 Wedges Example 3, page 2 of 3
7
Free-body diagram of block B
Impending motion
of lower surface of
block B relative to
block C.
The friction force
from block C
opposes the relative
motion of block B.
Equations of equilibrium for block B
F

x = 0: f

BC
cos 7 + N

BC
sin P

2
= 0 (3)
F

y
= 0: f

BC
sin 7 + N

BC
cos 2 kip = 0 (4)
Slip impends between blocks B and C, so
f

BC
= f

BC-max
N

BC
= 0.3N

BC
(5)
P
2
N
BC
x
y
N
AB
= 2 kip
f
BC
7
7
= 90 83 = 7
90 7 = 83
Solving Eqs. 3, 4, and 5 with = 7 gives

N
BC
= 2.092 kip
f
BC
= 0.628 kip = 628 lb
P
2
= 0.878 kip = 878 lb Ans.
Geometry 4
6
5
9
10
7
8
+
+
B

1268
7.2 Wedges Example 3, page 3 of 3
Free-body diagram of block C
The friction force f
BC
from
block B opposes the relative
motion of block C.
P
1
N
C
f
C
f
BC
N
BC
= 2.092 kip
x
y
= 7
7
The friction force f
C

from the floor
opposes the relative
motion of block B.
Equilibrium equations for block C
F

x
= 0: f

BC
cos 7 (2.092 kip) sin 7 + P

1
f

C
= 0 (6)
F

y
= 0: f

BC
sin 7 (2.092 kip) cos 7 + N

C
= 0 (7)
Slip impends between block C and the floor, so
f

C
= f

C-max
N

C
= 0.3N

C
(8)
Impending motion of upper
surface of C relative to B
Impending motion of lower
surface of C relative to the floor
Solving Eqs. 6, 7, and 8
simultaneously gives

f

C
= 0.6 kip = 600 lb
N

C
= 2 kip
P

1
= 1.478 kip Ans.
11
13
15
16
12
14
17
+
+
C

1269
7.2 Wedges Example 4, page 1 of 4
4. If the coefficient of static friction for all surfaces of
contact is 0.25, determine the smallest value of the forces P
that will move wedge B upward.
200 kg
B
75 75 20 kg
20 kg
A C P
P

1270
7.2 Wedges Example 4, page 2 of 4
Free-body diagram of block A
Weight = (20 kg)(9.81 m/s
2
)
= 196.2 N
Impending motion of right-side of block A
relative to block B (An observer on block B
would see block A move down.)
The friction force from block B opposes
the relative motion of block A.
Impending motion of
bottom of block A
relative to ground
The friction force from the floor
opposes the motion of block A.
Equations of equilibrium
F

x = 0: P f

A
f

AB
cos 75 N

AB
cos = 0 (1)
F

y
= 0: N

A
196.2 N + f

AB
sin 75 N

AB
sin = 0 (2)
Slip impends so,
f

A
= f

A-max
N

A
= 0.25N

A
(3)
f

AB
= f

AB-max
N

AB
= 0.25N

AB
(4)
Geometry
= 90 75
= 15
P
A
75
75
1
2
6
4
5
3
7
+
+
f
AB
N
A
f
A
75
N
AB

1271
7.2 Wedges Example 4, page 3 of 4
Free-body diagram of block B
Impending
motion of block
B relative to
block A
Impending motion of block B
relative to block C
f

BC
= f

AB
by symmetry (You can show this by
summing moments about the point where the
lines of action of N
AB
, N
BC
, and the weight
intersect)
N

BC
= N

AB
by symmetry (You can show this by
summing moments about the point where the lines
of action of f
AB
, f
BC
, and the weight intersect.)
The friction forces from
blocks A and C oppose
impending upward relative
motion of block B.
= 15
Weight = (200 kg)(9.81 m/s
2
)
= 1962 N
Equations of equilibrium
F

x = 0: N

AB
cos 15 N

AB
cos 15 + f

AB
cos 75 f

AB
cos 75 = 0 (5)
(Note that this equation reduces to 0 = 0. This happens because we have assumed symmetry
to conclude that f
BC
= f
AB
and N
BC
= N
AB
.)
F

y
= 0: N

AB
sin 15 + N

AB
sin 15 f

AB
sin 75 f

AB
sin 75 1962 N = 0 (6)
B
75 75
= 15
N

AB

8
10
9
f
AB
+
+

1272
7.2 Wedges Example 4, page 4 of 4
Solving Eqs. 1 4 and 6 simultaneously, with = 75, gives
f

A
= 294 N = 0.294 kN
N

A
= 1 180 N = 1.18 kN
f

AB
= 14 150 N = 14.15 kN
N

AB
= 56 600 N = 56.6 kN
P = 58 600 N = 58.6 kN Ans.
11

1273
7.2 Wedges Example 5, page 1 of 4
5. The cylinder D, which is connected by a pin at A to the
triangular plate C, is being raised by the wedge B. Neglecting the
weight of the wedge and the plate, determine the minimum force
P necessary to raise the cylinder if the coefficient of static friction
is 0.3 for the surfaces of contact of the wedge.
400 lb
r = 10 in
D
A
C
B
10
P

1274
7.2 Wedges Example 5, page 2 of 4
A
C
A
y
Ax
y
x
Resultant of
roller forces
Free-body diagram of triangular plate
Equilibrium equation for triangular plate
F

y
= 0: A

y
= 0 (1)
So, no vertical force is transmitted by pin A.
1
2
+

1275
7.2 Wedges Example 5, page 3 of 4
Equilibrium equations for cylinder D.
The equation for the sum of forces in the x-direction has not been
included because including it would introduce an additional
unknown, A

x, which we have not been asked to determine.
F

y
= 0: f

BD
sin + N

BD
cos 400 lb = 0 (2)
M

A
= 0: f

BD
(10 in.) = 0 (3)
Therefore f

BD
= 0, that is, no friction force acts on the cylinder.
Solving Eq. 2 with = 10 gives
N

BD
= 406.2 lb
Free-body diagram of cylinder D
Geometry
D
10
f
BD
10
80
400 lb
Ax
A
y
= 0
A
3
5
4
6
+
+
= 90 80 = 10
r = 10 in.
N

BD

1276
7.2 Wedges Example 5, page 4 of 4
Equilibrium equations for wedge B
F

x = 0: f

B
+ (406.2 lb) sin 10 P = 0 (4)
F

y
= 0: N

B
(406.2 lb) cos 10 = 0 (5)
Slip impends, so

f

B
= f
B-max
N

B
= 0.3N

B
(6)
Solving Eqs. 4, 5, and 6 simultaneously gives
f

B
= 120 lb
N

B
= 400 lb
P = 190.5 lb Ans.
Free-body diagram of wedge B
Impending motion
of wedge B relative
to the floor.
= 10
N

BD
= 406.2 lb
P
B
10
f
B
N
B
7
8
9
+
+

1277
7.2 Wedges Example 6, page 1 of 3
6. To split the log shown, a 120-lb force is applied to the top of the
wedge, which causes the wedge to be about to slip farther into the log.
Determine the friction and normal forces acting on the sides of the
wedge, if the coefficient of static friction is 0.6. Also determine if the
wedge will pop out of the log if the force is removed. Neglect the
weight of the wedge.
120 lb
Wedge angle = 8

1278
7.2 Wedges Example 6, page 2 of 3
Free-body diagram of wedge
Impending
motion of left side
of wedge
The friction
force from the
wood opposes
the motion of
the wedge.
Because of
symmetry, the
same variables,
N and f, are used
on the right side
of the wedge.
Equilibrium equation for the wedge
F

y
= 0: 2f cos 4 2N sin 120 lb = 0 (1)
Geometry
Slip impends, so
f = f

max N = 0.6N (2)
Solving Eqs. 1 and 2 simultaneously gives
f = 53.9 lb Ans.
N = 89.8 lb Ans.
= 90 86 = 4
90 4 = 86
4
4
4 4


f f
120 lb
N
N
1
2
3
4
5
6
7
+

1279
7.2 Wedges Example 6, page 3 of 3
P
4
4
4 4
f
f
N
N
Second part of the problem (Determine if the wedge
will pop out, if no force acts down on the top). To
determine if the wedge will pop out, let's first
determine what force would be needed to pull the
wedge out.
Free-body diagram of wedge based on assumption
that a force P is applied to pull the wedge out.
Impending
motion of wedge
(The wedge is
just about to pop
out.)
The friction force from the wood
opposes the motion of the wedge
(The force tries to keep the
wedge in the stump.)
Equilibrium equation for the wedge
F

y
= 0: 2f ' cos 4 + 2N' sin 4 + P = 0 (3)
Slip impends, so
f ' = f '

max N' = 0.6N' (4)
Substituting the expression for f ' of Eq. 4 into Eq. 3 and
solving for P gives
P = 2(0.6 cos 4 sin 4)N' = 1.058 N' (5)
Since the normal force N' always points towards the wedge,
it is always positive. Eq. 5 thus implies that the wedge will
pop out only if an upward force greater than or equal to 1.058
times the normal force is applied. For any smaller value of
P, the wedge will remain in place. Thus in particular for the
special case of P = 0 (no vertical force applied), the wedge
will remain in place.
8
9
10
11
12
+

1280
7.2 Wedges Example 7, page 1 of 6
7. The end A of the beam needs to be raised slightly to make it level. If
the coefficient of static friction of the contact surfaces of the wedge is
0.3, determine the smallest value of the horizontal force P that will raise
end A. The weight and size of the wedge are negligible. Also, if the
force P is removed, determine if the wedge will remain in place, that is, is
the wedge self-locking?
8
P
60 lb/ft
B A
15 ft

1281
7.2 Wedges Example 7, page 2 of 6
P
8
Free-body diagram of wedge
Impending motion of
top of wedge relative to
beam
Impending
motion of top
of wedge
relative to floor
Geometry
= 180 (90 +82)
= 8
90 8 = 82
Equilibrium equations for wedge
F

x = 0: P f
A
f

AB
cos 8 N

AB
sin = 0 (1)
F

y
= 0: N

A
+ f

AB
sin 8 N

AB
cos = 0 (2)
Slip impends so,
f

A
= f
A-max
N

A
= 0.3N
A
(3)
f

AB
= f
AB-max
N

AB
= 0.3N

AB
(4)
8
1
2
3
4
5
+
+
N
A
f
A
N
AB
y
f
AB

1282
7.2 Wedges Example 7, page 3 of 6
8
N
AB
f
AB
Bx
B
y
8
Impending motion of
beam relative to wedge
(An observer on the
wedge would see the
end of the beam move
up and to the left.)
The friction force
from the wedge
opposes the motion
of the beam.
Equilibrium equation of the beam
M

B
= 0: (60 lb/ft)(15 ft)(
15 ft
2
) N

AB
cos 8 (15 ft) + f

AB
sin 8 (15 ft) = 0 (5)
Solving Eqs. 1-5 simultaneously gives

f

A
= 135 lb
N

A
= 450 lb
f

AB
= 142 lb
N

AB
= 474 lb
P = 342 lb Ans.
+
6
7
8
15 ft
A
B
60 lb/ft

1283
7.2 Wedges Example 7, page 4 of 6
Second part of the problem: Determine if the
wedge is self-locking. To do so, reverse the
direction of force P and calculate the value of P
necessary to cause impending motion of the
wedge to the left.
9
15 ft
A B
60 lb/ft
P
8

1284
7.2 Wedges Example 7, page 5 of 6
y
x
f
AB
f
A
N
A
N
AB
8
P
Free-body diagram of wedge
Impending motion of
top surface of wedge
relative to beam (The
force P has been applied
to pull the wedge out.)
Impending motion of
bottom surface of
wedge relative to floor
Equilibrium equations for wedge
Fx = 0: P + f

A
+ f

AB
cos 8 N

AB
sin 8 = 0 (1)
F

y
= 0: N

A
f

AB
sin 8 N

AB
cos 8 = 0 (2)
Slip impends so,
f

A
= f
A-max
N

A
= 0.3N
A
(3)
f
AB
= f
B-max
N
AB
= 0.3N
AB
(4)
10
11
12
13
+
+
8

1285
7.2 Wedges Example 7, page 6 of 6
Free-body diagram of beam
Solving Eqs. 1-5 simultaneously gives

f

A
= 135 lb
N

A
= 450 lb
f

AB
= 131 lb
N

AB
= 436 lb,
P = 204 lb.
A force P of at least 204 lb is necessary to pull out the
wedge; any thing less than 204 lb is not enough the wedge
will remain in place. In particular, when no force is
appliced (P = 0), the wedge remains in place. Thus it is
self-locking.
14
17
+
60 lb/ft
B
A
B
y
Bx
f
AB
8
15 ft
16 Equilibrium equation of the beam
M

B
= 0: (60 lb/ft)(15 ft)(
15 ft
2
) N

AB
cos 8 (15 ft)
f

AB
sin 8 (15 ft) = 0 (5)
15 Impending motion of beam relative to
wedge (an observer on the wedge sees the
beam move down and to the right)
N
AB
8

1286

7.3 Square-Threaded Screws
1287
7.3 Square-Threaded Screws Procedures and Strategies, page 1 of 2
Procedures and Strategies for Solving Problems Involving
Friction: Square-Threaded Screws.
1. Make a sketch of the screw, showing F, the resultant
axial force acting on the screw excluding the force from
the supporting threads.
2. Compare the direction of F with the direction of the
motion of the screw relative to the supporting threads
a) If the force and motion are in opposite directions,
calculate the torque M applied to the screw from the
formula
M = Fr tan ( + )
in which
r = mean radius of the threads
= tan
-1
[L/(2 r)]
L = lead of the screw
= angle of friction (= tan
-1
)
= coefficient of friction.
Sense of rotation produced by torque M applied to screw
Direction of motion of screw
relative to supporting threads
M
F

1288
7.3 Square-Threaded Screws Procedures and Strategies, page 2 of 2
M
Direction of motion of screw
Direction of motion of screw
b) If the force and motion are in the same direction,
then determine if the screw is self-locking by
checking the condition for a screw to be self-locking:
tan
i) If the screw is self-locking and the motion of the
screw is in the same direction as the axial load,
then a torque must be applied to make the screw
move in the direction of the load. This torque can
be calculated from
M = Fr tan ( )
ii) If the screw is not self-locking and the
impending motion of the screw is in the same
direction as the axial load, then a torque must be
applied to prevent the screw from turning on its
own. This torque can be calculated from
M = Fr tan ( )
F
Self-locking screw
Non-self-locking screw
F M

1289
7.3 Square-Threaded Screws Problem Statement for Example 1
100 kg
1. The square-threaded jack is used to raise and lower
the 100-kg block. Determine a) the torque Mu required
to begin moving the block up, and b) the torque M
d

required to begin moving the block down. Also
determine if the block will remain stationary when the
torque is removed. The screw has lead L = 10 mm,
mean radius r = 8 mm, and is single-threaded. The
coefficient of static friction between the screw and the
supporting threads of the base is = 0.25.

1290
7.3 Square-Threaded Screws Problem Statement for Example 2
2. Determine the minimum number n of threads per
inch that will prevent the square-threaded jack screw
from turning by itself under its own weight. The mean
radius r of the thread is 0.5 in., and the coefficient of
static friction between the screw and the supporting
threads of the base is = 0.15. The screw is
single-threaded.

1291
7.3 Square-Threaded Screws Problem Statement for Example 3
B
A
M
3. An assembly consists of a tube, washer A, and
a square-threaded screw. The end of the screw is
threaded into the support B. The tube will be
crushed if the compressive load acting on it
exceeds 100 N. Determine the maximum value
of the torque M that can be safely applied to the
tube. The screw has lead L = 8 mm and mean
radius = 10 mm. The coefficient of static friction
between the screw and the threads in the support
A is = 0.3. Neglect the friction between the
washer and the tube and between the washer and
the head of the screw.

1292
7.3 Square-Threaded Screws Problem Statement for Example 4
Blocks
P
4. The C-clamp provides a compressive force of 900 N to
two blocks that are being glued together. Determine the
minimum force P necessary to loosen the clamp. The
clamp has a square-threaded screw with lead L = 7 mm
and mean radius r = 8 mm. The coefficient of static
friction between the screw and the supporting threads in
the frame is = 0.25.
100 mm

1293
7.3 Square-Threaded Screws Problem Statement for Example 5
5. A square-threaded bolt and nut are used to
connect three plates as shown. Determine the
torque that must be applied to the bolt and nut
to achieve a specified bolt tension of 60 kip.
The mean diameter of the thread is d = 1 in., the
lead is L = 0.1 in., and the coefficient of static
friction between the threads of the bolt and nut
is = 0.2. Friction between the washer and nut
can be neglected.

1294
7.3 Square-Threaded Screws Problem Statement for Example 6
C
B
W
150 mm
150 mm
D
6. The weight W = 6 kN can be raised by rotating the
square-threaded screw of the automobile jack, causing
joints A and C to move towards each other. Determine the
force P required to raise the load when = 30. The lead
of the thread is L = 4 mm, the mean diameter is d = 8 mm,
and the coefficient of static friction is = 0.2.
P
A
200 mm
150 mm
150 mm

1295
7.3 Square-Threaded Screws Problem Statement for Example 7
7. Joints A and D of the truss are connected by a cable with a
turnbuckle. Determine the minimum torque Me required to
loosen the turnbuckle. The turnbuckle has a square thread
with lead L = 1/8 in. and mean radius r = 1/4 in. The
coefficient of static friction between the turnbuckle frame and
the screws is = 0.4. Both screws are prevented from
rotating as the turnbuckle is turned.
Left-handed thread Right-handed thread
Enlarged view of turnbuckle
5 kip
A B
C
D
6 ft
6 ft
Me

1296
7.3 Square-Threaded Screws Problem Statement for Example 8
8. Determine the minimum torque Me applied to the turnbuckle
that will cause the 500-kg block to begin to move a) up and
b) down. Also, if no torque is applied, determine if the block
will remain stationary. The turnbuckle has a square thread
with lead L = 5 mm, mean radius r = 10 mm, and coefficient
of static friction between the turnbuckle frame and the screws
of = 0.24. Both screws are prevented from rotating as the
turnbuckle is turned.
Frictionless pulley
Cable
500 kg
Enlarged view of turnbuckle
Enlarged view of turnbuckle
Right-handed thread Left-handed thread

1297
7.3 Square-Threaded Screws Problem Statement for Example 9
5 kN 5 kN
9. The collar connects two shafts with threaded ends.
Both threads are single, square, and right-handed,
with lead L = 5 mm, mean radius r = 8 mm, and
coefficient of static friction = 0.4. The tension in
each shaft is 5 kN. Both screws are prevented from
turning as the collar turns. Determine the minimum
torque M required to turn the collar in either
direction.

1298
7.3 Square-Threaded Screws Example 1, page 1 of 5
100 kg
1. The square-threaded jack is used to raise and lower
the 100-kg block. Determine a) the torque Mu required
to begin moving the block up, and b) the torque M
d

required to begin moving the block down. Also
determine if the block will remain stationary when the
torque is removed. The screw has lead L = 10 mm,
mean radius r = 8 mm, and is single-threaded. The
coefficient of static friction between the screw and the
supporting threads of the base is = 0.25.

1299
7.3 Square-Threaded Screws Example 1, page 2 of 5
F
M
M
M
Direction of motion of screw
relative to supporting threads
Direction of motion of screw
Direction of motion of screw
Formulas For A Screw With A Single Square Thread
Let F be the resultant axial force acting on the screw,
exclusive of the force from the supporting threads). Then
for screw motion in a direction opposite to F:
M = Fr tan ( + ) (1)
in which M is the torque required to turn the screw; r is the
mean radius of the threads; = tan
-1
[L/(2 r)], where L is
the lead of the screw; is the angle of friction (= tan
-1
),
where is the coefficient of friction.
Condition for screw to be of self-locking:
tan (2)
Self-locking screw with motion in same direction as axial
load:
M = Fr tan ( ) (3)
Non-self-locking screw with motion in same direction as
axial load:
M = Fr tan ( ) (4)
in which M is the minimum torque that must be applied to
the screw to prevent it from turning on its own.
1 Sense of rotation produced by torque M applied to screw
F
F

1300
7.3 Square-Threaded Screws Example 1, page 3 of 5
M

u
3 Load on screw
(100 kg) (9.81 m/s
2
)
981 N
4 Sense of rotation produced by
torque M

u applied to screw
Impending
motion of
screw
Part a) Move load up.
Because the motion of the screw is opposite the
direction of the load on the screw, Eq. 1 can be used
to calculate the torque:
Mu = Fr tan ( + ) (Eq.1 repeated)
Here
F = 981 N
r = 0.008 m
= tan
-1
[L/(2 r)]
= tan
-1
[0.010 m/(2 (0.008 m))]
= 11.252
= tan
-1
,
= tan
-1
(0.25)
= 14.036
Substituting these results in Eq. 1 gives
Mu = 3.71 N m Ans.
2
5

1301
7.3 Square-Threaded Screws Example 1, page 4 of 5
M

d
Load 981 N
7
Sense of rotation
produced by torque
M

d
applied to screw
Impending
motion of
screw
8
6 Part b) Move load down.
Because the motion of the screw is in the same
direction as the load acting on the screw, the
appropriate formula to use to calculate the torque M
d
is
either Eq. 3 or Eq. 4. To decide which equation to use,
we have to determine if the screw is self-locking:
Condition for screw to be self-locking:
tan (Eq. 2 repeated)
0.25 tan 11.252= 0.199
Thus the inequality is satisfied and the screw is
self-locking. Eq. 3 then applies:
M
d
= Fr tan ( ) (Eq. 3 repeated)
Substituting F = 981 N, r = 0.008 m, = 11.252 and
= 14.036, as before, gives
M
d
= 0.38 N m Ans.
Note that this moment is considerably less than the 3.71
N m moment needed to raise the block.

1302
7.3 Square-Threaded Screws Example 1, page 5 of 5
9 If the torque is removed, does the block remain
stationary? Since we have shown that the screw is
self-locking, the
block remains stationary Ans.
Comment on "self-locking." The friction coefficient
was = 0.25. Suppose instead that it had been smaller,
say, = 0.15. Then consider the self-locking criterion:
tan (Eq. 2 repeated)
0.15 tan 11.252 = 0.199
Thus the inequality above is not satisfied, and the screw
is not self-locking. To keep the block from descending
rapidly under its own weight as the screw turns, we have
to apply a torque M in the same sense as if we were about
to raise the block. The smallest value of the torque that
will keep the block from coming down is given by Eq. 4:
M
d
= Fr tan ( ) (Eq. 4 repeated)
Substituting F = 981 N, r = 0.008 m, and = 11.252
as before, together with = tan
-1
0.15 = 8.531, gives
M
d
= 0.37 N m
100 kg
Sense of rotation (causes
the block to move up, or,
better stated, prevents the
block from moving down)
M
Impending
motion of
screw

1303
7.3 Square-Threaded Screws Example 2, page 1 of 3
2. Determine the minimum number n of threads per
inch that will prevent the square-threaded jack screw
from turning by itself under its own weight. The mean
radius r of the thread is 0.5 in., and the coefficient of
static friction between the screw and the supporting
threads of the base is = 0.15. The screw is
single-threaded.

1304
7.3 Square-Threaded Screws Example 2, page 2 of 3
F
M
M
M
Direction of motion of screw
relative to supporting threads
Direction of motion of screw
Direction of motion of screw
Formulas For A Screw With A Single Square Thread
Let F be the resultant axial force acting on the screw,
exclusive of the force from the supporting threads). Then
for screw motion in a direction opposite to F:
M = Fr tan ( + ) (1)
in which M is the torque required to turn the screw; r is the
mean radius of the threads; = tan
-1
[L/(2 r)], where L is
the lead of the screw; is the angle of friction (= tan
-1
),
where is the coefficient of friction.
Condition for screw to be of self-locking:
tan (2)
Self-locking screw with motion in same direction as axial
load:
M = Fr tan ( ) (3)
Non-self-locking screw with motion in same direction as
axial load:
M = Fr tan ( ) (4)
in which M is the minimum torque that must be applied to
the screw to prevent it from turning on its own.
1 Sense of rotation produced by torque M applied to screw
F
F

1305
7.3 Square-Threaded Screws Example 2, page 3 of 3
The jack will not turn by itself if it is
self-locking. The condition for self-locking is
tan (Eq. 2 repeated)
Here
tan = L/(2 r)
Using this result in Eq. 2 gives
L/(2 r) (5)

Because the screw is single-threaded, L is the
distance between two consecutive threads.
That is, L is the number of inches per thread. It
follows that the reciprocal of L is the number
of threads per inch:
1/L = n
Using this result in Eq. 5 gives
L/(2 r)
1/n
2 Solving for n gives
n 1/(2 r )
Substituting r = 0.5 in. and = 0.15 gives
n 2.122 threads/in.
Thus the minimum number is about
n = 2.12 threads/in. Ans.
3

1306
7.3 Square-Threaded Screws Example 3, page 1 of 4
B
A
M
3. An assembly consists of a tube, washer A, and
a square-threaded screw. The end of the screw is
threaded into the support B. The tube will be
crushed if the compressive load acting on it
exceeds 100 N. Determine the maximum value
of the torque M that can be safely applied to the
tube. The screw has lead L = 8 mm and mean
radius = 10 mm. The coefficient of static friction
between the screw and the threads in the support
A is = 0.3. Neglect the friction between the
washer and the tube and between the washer and
the head of the screw.

1307
7.3 Square-Threaded Screws Example 3, page 2 of 4
F
M
M
M
Direction of motion of screw
relative to supporting threads
Direction of motion of screw
Direction of motion of screw
Formulas For A Screw With A Single Square Thread
Let F be the resultant axial force acting on the screw,
exclusive of the force from the supporting threads). Then
for screw motion in a direction opposite to F:
M = Fr tan ( + ) (1)
in which M is the torque required to turn the screw; r is the
mean radius of the threads; = tan
-1
[L/(2 r)], where L is
the lead of the screw; is the angle of friction (= tan
-1
),
where is the coefficient of friction.
Condition for screw to be of self-locking:
tan (2)
Self-locking screw with motion in same direction as axial
load:
M = Fr tan ( ) (3)
Non-self-locking screw with motion in same direction as
axial load:
M = Fr tan ( ) (4)
in which M is the minimum torque that must be applied to
the screw to prevent it from turning on its own.
1 Sense of rotation produced by torque M applied to screw
F
F

1308
7.3 Square-Threaded Screws Example 3, page 3 of 4
M
100 N
100 N
Free-body diagram of tube
loaded to maximum capacity
2 The 100-N reaction force from the tube
pushes up on the washer and thus
produces a tensile load on the screw.
3
Impending
motion of
screw
Sense of rotation
produced by torque M
4
5
F 100 N

1309
7.3 Square-Threaded Screws Example 3, page 4 of 4
6 The motion of the screw is opposite the direction of the
100-N axial load, so Eq.1 applies:
M = Fr tan ( + ) (Eq. 1 repeated)
Here
F = 100 N
r = 0.01 m
= tan
-1
[L/(2 r)]
= tan
-1
[0.008 m/(2 0.01 m))]
= 7.256
= tan
-1

= tan
-1
(0.3)
16.699
Substituting these values in Eq. 1 gives
M = 0.444 N m Ans.

1310
7.3 Square-Threaded Screws Example 4, page 1 of 4
Blocks
P
4. The C-clamp provides a compressive force of 900 N to
two blocks that are being glued together. Determine the
minimum force P necessary to loosen the clamp. The
clamp has a square-threaded screw with lead L = 7 mm
and mean radius r = 8 mm. The coefficient of static
friction between the screw and the supporting threads in
the frame is = 0.25.
100 mm

1311
7.3 Square-Threaded Screws Example 4, page 2 of 4
F
M
M
M
Direction of motion of screw
relative to supporting threads
Direction of motion of screw
Direction of motion of screw
Formulas For A Screw With A Single Square Thread
Let F be the resultant axial force acting on the screw,
exclusive of the force from the supporting threads). Then
for screw motion in a direction opposite to F:
M = Fr tan ( + ) (1)
in which M is the torque required to turn the screw; r is the
mean radius of the threads; = tan
-1
[L/(2 r)], where L is
the lead of the screw; is the angle of friction (= tan
-1
),
where is the coefficient of friction.
Condition for screw to be of self-locking:
tan (2)
Self-locking screw with motion in same direction as axial
load:
M = Fr tan ( ) (3)
Non-self-locking screw with motion in same direction as
axial load:
M = Fr tan ( ) (4)
in which M is the minimum torque that must be applied to
the screw to prevent it from turning on its own.
1 Sense of rotation produced by torque M applied to screw
F
F

1312
7.3 Square-Threaded Screws Example 4, page 3 of 4
P
900 N 900 N
Free-body diagram of blocks 2
The reaction force from the block
produces an axial load on the screw,
3
Because the motion of the screw is in the same direction as
the load acting on the screw, the appropriate formula to use
to calculate the torque M is either Eq. 3 or Eq. 4.
6
Impending motion
of screw
5
Torque, M (0.1 m) P
(produced by force P)
4
F 900 N
100 mm

1313
7.3 Square-Threaded Screws Example 4, page 4 of 4
7 To decide whether to use Eq. 3 or Eq. 4, we have to
determine if the screw is self-locking.
Condition for a screw to be self-locking:
0.25
tan (Eq. 2 repeated)
L/(2 r) = 0.007 m/(2 0.008 m)) = 0.139
Thus the inequality is satisfied, and the screw is
self-locking. Eq. 3 applies:
M = Fr tan ( ) (Eq. 3 repeated)
Here
M = (0.1 m)P
F = 900 N
r = 0.008 m
= tan
-1
[L/(2 r)]
= tan
-1
[0.007 m/(2 0.008 m))]
= 7.928
= tan
-1

= tan
-1
(0.25)
14.036
Substituting these values in Eq. 3 and solving
for P gives
P = 7.70 N Ans.
8

1314
7.3 Square-Threaded Screws Example 5, page 1 of 4
5. A square-threaded bolt and nut are used to
connect three plates as shown. Determine the
torque that must be applied to the bolt and nut
to achieve a specified bolt tension of 60 kip.
The mean diameter of the thread is d = 1 in., the
lead is L = 0.1 in., and the coefficient of static
friction between the threads of the bolt and nut
is = 0.2. Friction between the washer and nut
can be neglected.

1315
7.3 Square-Threaded Screws Example 5, page 2 of 4
F
M
M
M
Direction of motion of screw
relative to supporting threads
Direction of motion of screw
Direction of motion of screw
Formulas For A Screw With A Single Square Thread
Let F be the resultant axial force acting on the screw,
exclusive of the force from the supporting threads). Then
for screw motion in a direction opposite to F:
M = Fr tan ( + ) (1)
in which M is the torque required to turn the screw; r is the
mean radius of the threads; = tan
-1
[L/(2 r)], where L is
the lead of the screw; is the angle of friction (= tan
-1
),
where is the coefficient of friction.
Condition for screw to be of self-locking:
tan (2)
Self-locking screw with motion in same direction as axial
load:
M = Fr tan ( ) (3)
Non-self-locking screw with motion in same direction as
axial load:
M = Fr tan ( ) (4)
in which M is the minimum torque that must be applied to
the screw to prevent it from turning on its own.
1 Sense of rotation produced by torque M applied to screw
F
F

1316
7.3 Square-Threaded Screws Example 5, page 3 of 4
F (force from washer on top)
F (force from washer on bottom)
Sense of rotation of nut
produced by torque M
applied to the nut
M
M
3
Free-body diagram of plates 4
Torques are applied to the bolt head
and to the nut to tighten the nut.
2

1317
7.3 Square-Threaded Screws Example 5, page 4 of 4
Impending motion
of screw
Sense of rotation of nut
F = 60 kip
M
The reaction force from the top
plate pushes the washer and the
head of the bolt up. This causes
an axial tensile load F in the
screw.
The motion of the nut is up so
the motion of the screw
relative to the supporting
screws of the nut is down.
5
6
M
7 The motion of the screw is opposite the
direction of the axial load F, so Eq.1 applies:.
M = Fr tan ( ) (Eq. 1 repeated)
Here
F = 60 kip
r = d/2
= 1.0 in./2
= 0.5 in.
= tan
-1
[L/(2 r)]
= tan
-1
[0.1 in./(2 0.5 in.))]
= 1.823
= tan
-1

= tan
-1
(0.2)
11.310
Substituting these values in Eq. 1 gives
M = 7.00 kip in. Ans.

1318
7.3 Square-Threaded Screws Example 6, page 1 of 6
C
B
W
150 mm
150 mm
D
6. The weight W = 6 kN can be raised by rotating the
square-threaded screw of the automobile jack, causing
joints A and C to move towards each other. Determine the
force P required to raise the load when = 30. The lead
of the thread is L = 4 mm, the mean diameter is d = 8 mm,
and the coefficient of static friction is = 0.2.
P
A
200 mm
150 mm
150 mm

1319
7.3 Square-Threaded Screws Example 6, page 2 of 6
F
M
M
M
Direction of motion of screw
relative to supporting threads
Direction of motion of screw
Direction of motion of screw
Formulas For A Screw With A Single Square Thread
Let F be the resultant axial force acting on the screw,
exclusive of the force from the supporting threads). Then
for screw motion in a direction opposite to F:
M = Fr tan ( + ) (1)
in which M is the torque required to turn the screw; r is the
mean radius of the threads; = tan
-1
[L/(2 r)], where L is
the lead of the screw; is the angle of friction (= tan
-1
),
where is the coefficient of friction.
Condition for screw to be of self-locking:
tan (2)
Self-locking screw with motion in same direction as axial
load:
M = Fr tan ( ) (3)
Non-self-locking screw with motion in same direction as
axial load:
M = Fr tan ( ) (4)
in which M is the minimum torque that must be applied to
the screw to prevent it from turning on its own.
1 Sense of rotation produced by torque M applied to screw
F
F

1320
7.3 Square-Threaded Screws Example 6, page 3 of 6
D
B
W
F

AB
F

BC
(force from floor
under jack)
F

AD
F

DC
W
2 The torque required to turn the screw depends on the
axial load acting on the screw. To find the axial
load, we have to compute the forces in the members
AB, BC, AD, and DC.
The forces in members AB and BC can be found
from a free-body diagram of bracket B:
Similarly using a free body diagram of bracket D
leads to
F
AD
= F
DC
= W/(2 sin ) (6)
+
+
Equilibrium equations
Fx = 0: F
AB
cos F
BC
cos = 0
F
y
= 0: F
AB
sin + F
BC
sin W = 0
Solving gives
F
AB
= F
BC
= W/(2 sin ) (5)
3
4
Free-body diagram of bracket D

1321
7.3 Square-Threaded Screws Example 6, page 4 of 6
A
F

AB
F

AD
F
5 Next, a free-body diagram of pin A gives the
force F of the screw acting on the pin:
A
W
B
C
D
F is the force of the screw acting on
the pin A. The equal and opposite
force of pin A acting on the screw
constitutes the load on the screw.
6
Equilibrium equations
W/(2 sin ) by Eq. 5
Fx = 0: F
AB
cos F
AD
cos + F = 0
W/(2 sin ) by Eq. 6
Solving gives
F = W/tan (7)
+

1322
7.3 Square-Threaded Screws Example 6, page 5 of 6
Load on screw F = W/tan
Impending motion
of screw relative
to supporting
threads at C
10
Sense of rotation
caused by torque M
9
7
Torque M produced by
force P (M P 0.2 m)
8
200 mm
A C
B
W
D
P
Because the motion of the screw is opposite the
direction of the load on the screw, Eq. 1 can be
used to calculate the torque.
11

1323
7.3 Square-Threaded Screws Example 6, page 6 of 6
Apply Eq. 1:
M = Fr tan ( + ) (Eq.1 repeated)
Here
M = 0.2P
F = W/tan
= (6000 N)/tan 30
= 10 392 N
r = (0.008 m)/2
= 0.004 m
= tan
-1
[L/(2 r)]
= tan
-1
[0.004 m/(2 (0.004 m))]
= 9.043 (8)
= tan
-1
,
= tan
-1
(0.2)
= 11.310
Substituting these results in Eq. 1 gives
P = 77.1 N Ans.
12 13 Comment: if a jack is to be safe, it must support the
weight being lifted, even when the operator of the
jack takes his hands off the handle. Is the present
jack safe? Answer: yes, if it is self-locking.
Condition for screw to be self-locking:
0.2
tan (Eq. 2 repeated)
tan 9.043 by Eq. 8
= 0.159
Because the inequality is satisfied, the jack is
self-locking and so is safe.

1324
7.3 Square-Threaded Screws Example 7, page 1 of 8
5 kip
A B
C
D
6 ft
6 ft
Me
7. Joints A and D of the truss are connected by a cable with a
turnbuckle. Determine the minimum torque Me required to
loosen the turnbuckle. The turnbuckle has a square thread
with lead L = 1/8 in. and mean radius r = 1/4 in. The
coefficient of static friction between the turnbuckle frame and
the screws is = 0.4. Both screws are prevented from
rotating as the turnbuckle is turned.
Left-handed thread Right-handed thread
Enlarged view of turnbuckle

1325
7.3 Square-Threaded Screws Example 7, page 2 of 8
F
M
M
M
Direction of motion of screw
relative to supporting threads
Direction of motion of screw
Direction of motion of screw
Formulas For A Screw With A Single Square Thread
Let F be the resultant axial force acting on the screw,
exclusive of the force from the supporting threads). Then
for screw motion in a direction opposite to F:
M = Fr tan ( + ) (1)
in which M is the torque required to turn the screw; r is the
mean radius of the threads; = tan
-1
[L/(2 r)], where L is
the lead of the screw; is the angle of friction (= tan
-1
),
where is the coefficient of friction.
Condition for screw to be of self-locking:
tan (2)
Self-locking screw with motion in same direction as axial
load:
M = Fr tan ( ) (3)
Non-self-locking screw with motion in same direction as
axial load:
M = Fr tan ( ) (4)
in which M is the minimum torque that must be applied to
the screw to prevent it from turning on its own.
1 Sense of rotation produced by torque M applied to screw
F
F

1326
7.3 Square-Threaded Screws Example 7, page 3 of 8
Me
B
A
5 kip
+
The torque required to loosen the turnbuckle
depends on the value of the axial force that acts
on the turnbuckle. Thus we first have to solve
for the force in truss member AD. Using the
method of joints, we begin with a free-body
diagram of joint B:
Equilibrium equation:
Fx = 0: F
AB
5 kip = 0
Thus
F
AB
= 5 kip
= 5 kip (C)
D C
5 kip
F

AB
B
F

BD
2

1327
7.3 Square-Threaded Screws Example 7, page 4 of 8
A
F

AC
Free-body diagram of joint A
Equilibrium equation:
Fx = 0: 5 kip + F
AD
sin 45 = 0
Thus
F
AD
= 7.071 kip (T) (5)
This is the axial force acting on the ends of the
turnbuckle.
3
F

AB
5 kip (C)
F

AD
Me
6 ft
6 ft
D
B A
5 kip
+
45
C

1328
7.3 Square-Threaded Screws Example 7, page 5 of 8
Right-handed thread
Axial load
Axial load
Axial load
To loosen the turnbuckle, we must turn it in
such a way that screw R moves to the right
relative to the turnbuckle frame.
Because screw R has a right-handed
thread, it will move to the right if an
external torque Me is applied to the
frame in the sense shown.
4
5
Note that the motion of screw
R is in the same direction as
the axial load acting on the
screw.
6
Axial load
L R
Direction of desired motion of
screw R relative to frame
R L
Impending motion of screw R
relative to frame
Sense of rotation of frame
produced by torque Me
Me

1329
7.3 Square-Threaded Screws Example 7, page 6 of 8
R L
Axial load
Impending motion of
screw relative to frame
Sense of rotation of frame
produced by torque M

e
M

e
Left-handed thread
Because screw L has a left-handed
thread, it will move left relative to the
frame if torque Me is applied to the
frame with the same sense as before.
Axial load
7
Just as was the case for screw R, the
impending motion of screw L is in the same
direction as the axial load. It follows that for
both screws either Eq. 3 or Eq. 4 applies. To
decide which equation to use, we have to
determine if the screws are self-locking.
8

1330
7.3 Square-Threaded Screws Example 7, page 7 of 8
Condition for screw to be self-locking:
tan (Eq. 2 repeated)
Here
= 0.4
and
tan = L/(2 r)
= (1/8 in.)/[2 (1/4 in.)]
= 0.080 (6)
Since
0.4
> tan
0.080
the inequality, Eq. 2, is satisfied and the screws are
self-locking. Thus Eq. 3 is the appropriate equation
for calculating M, the torque acting on each screw.
9 10 Apply Eq. 3.
M = Fr tan ( ) (Eq. 3 repeated)
Here
F = 7.071 kip (Eq. 5 repeated)
r = 1/4 in.
= tan
-1

= tan
-1
(0.4)
1.801
= tan
-1
(0.080) (by Eq. 6)
= 4.570
Substituting these values into Eq. 3 gives
M = 0.548 kip in (7)
Note: the torque M is not the same as the external
torque Me, as will now be shown.

1331
7.3 Square-Threaded Screws Example 7, page 8 of 8
L R
M 0.548 kip in. by Eq. 7
(Torque from screw R
resisting rotation of frame)
13 M 0.548 kip in. by Eq. 7
(Torque from screw L
resisting rotation of frame)
12
Torque applied externally to
turnbuckle to make it rotate
14
F 7.07 kip
M

e
F 7.07 kip
+
Moment equilibrium equation:
M
axial
= 0: M Me + M = 0
Thus
Me = M + M
= 0.548 kip in + 0.548 kip in
= 1.096 kip in Ans.
The torque M can now be related to the external torque
Me by considering a free-body diagram of the frame:
11
15

1332
7.3 Square-Threaded Screws Example 8, page 1 of 8
8. Determine the minimum torque Me applied to the turnbuckle
that will cause the 500-kg block to begin to move a) up and
b) down. Also, if no torque is applied, determine if the block
will remain stationary. The turnbuckle has a square thread
with lead L = 5 mm, mean radius r = 10 mm, and coefficient
of static friction between the turnbuckle frame and the screws
of = 0.24. Both screws are prevented from rotating as the
turnbuckle is turned.
Frictionless pulley
Cable
500 kg
Enlarged view of turnbuckle
Enlarged view of turnbuckle
Right-handed thread Left-handed thread

1333
7.3 Square-Threaded Screws Example 8, page 2 of 8
F
M
M
M
Direction of motion of screw
relative to supporting threads
Direction of motion of screw
Direction of motion of screw
Formulas For A Screw With A Single Square Thread
Let F be the resultant axial force acting on the screw,
exclusive of the force from the supporting threads). Then
for screw motion in a direction opposite to F:
M = Fr tan ( + ) (1)
in which M is the torque required to turn the screw; r is the
mean radius of the threads; = tan
-1
[L/(2 r)], where L is
the lead of the screw; is the angle of friction (= tan
-1
),
where is the coefficient of friction.
Condition for screw to be of self-locking:
tan (2)
Self-locking screw with motion in same direction as axial
load:
M = Fr tan ( ) (3)
Non-self-locking screw with motion in same direction as
axial load:
M = Fr tan ( ) (4)
in which M is the minimum torque that must be applied to
the screw to prevent it from turning on its own.
1 Sense of rotation produced by torque M applied to screw
F
F

1334
7.3 Square-Threaded Screws Example 8, page 3 of 8
Direction of desired motion of
screw R relative to frame
Impending motion of screw
R relative to frame
3
Note that the motion of
screw R is in a direction
opposite to the axial load
acting on the screw.
6
Because screw R has a right-handed thread, it
will move left if an external torque M

e is
applied to the frame in the sense shown.
4
Part a): 500-kg block moves up.
To make the block move up, we must rotate the
turnbuckle in such a way that screw R moves left
relative to the turnbuckle frame.
2
Right-handed thread
L R
L R
Axial load
Axial load
Axial load
Axial load
5 Sense of rotation of frame
produced by torque M

e
M

e

1335
7.3 Square-Threaded Screws Example 8, page 4 of 8
Axial load
Because screw L has a left-handed thread, it will
move right relative to the frame if torque Me is
applied with the same sense as before.
Axial load
R L
Impending motion of screw L
Sense of rotation of frame
produced by torque M

e
M

e
7
Thus the motion of each screw is in a direction opposite
to the axial load acting on the screw. It follows that the
appropriate formula (for square-threaded screws) to use
for both screws is Eq.1, which applies when screw
motion and axial load are in opposite directions.
8
Left-handed thread

1336
7.3 Square-Threaded Screws Example 8, page 5 of 8
M = Fr tan ( + ) (Eq. 1 repeated)
Here M is the torque acting on either screw L or
screw R. M is not Me (recall that Me is the torque
applied to the frame of the turnbuckle, not to the
individual screws).
F = tension in cable
= weight of 500-kg mass
= (500 kg)(9.81 m/s
2
)
= 4905 N (5)
r = 0.01 m (6)
= tan
-1
[L/(2 r)]
= tan
-1
[0.005 m/(2 (0.01 m))]
= 4.550 (7)
= tan
-1
,
= tan
-1
(0.24)
= 13.496 (8)
Substituting these results in Eq. 1 gives
M = 15.981 N m (9)
9

1337
7.3 Square-Threaded Screws Example 8, page 6 of 8
L R
M 15.981 N m
(Torque from screw R
acting on frame)
13
M 15.981 N m
(Torque from screw L
acting on frame)
11 Torque applied externally
to turnbuckle frame to
make it rotate
12
F 4905 N
M

e
Free-body diagram of frame of turnbuckle
10 The torque M can now be related to the torque
Me by considering a free-body diagram.
F 4905 N
14 Moment equilibrium:
M
axial
= 0: M + Me M = 0
Thus
Me = 2M (10)
= 2(15.981 N m)
= 32.0 N m Ans.
+

1338
7.3 Square-Threaded Screws Example 8, page 7 of 8
Axial load
R L
Axial load
Sense of rotation
produced by torque M

e
M

e
16
17 Impending motion of screw R
Impending motion of screw L
Part b): 500-kg block moves down.
To make the block move down, rotate the turnbuckle in the
opposite sense. The sense of M

e and the direction of
motions of the screws reverse from what we had before.
15
Because the motion of each screw is in the same direction as the axial
load acting on the screw, either Eq. 3 or Eq. 4 apply. To decide which,
equation to use we have to determine if the screws are self-locking:
Criterion of self-locking:
18
tan (Eq. 2 repeated)
0.24
tan 4.550 by Eq. 7
0.080

1339
7.3 Square-Threaded Screws Example 8, page 8 of 8
Thus the inequality is satisfied, and the screws are self-locking. Eq. 3
applies:
M = Fr tan ( ) (Eq. 3 repeated)
Substituting the previous values from Eqs. 5-9 (F = 4905 N,
r = 0.01 m, = 4.550, and = 13.496) gives
M = 7.721 N m
Writing a moment equilibrium equation as before gives Eq. 10 again:
Mu = 2M (Eq. 10 repeated)
= 2(7.721 N m)
= 15.44 N m Ans.
Finally, because we have shown that the turnbuckle is self-locking
( > tan in Eq. 2), we can conclude that
the weight will remain stationary when Me = 0 Ans.
19

1340
7.3 Square-Threaded Screws Example 9, page 1 of 7
5 kN 5 kN
9. The collar connects two shafts with threaded ends.
Both threads are single, square, and right-handed,
with lead L = 5 mm, mean radius r = 8 mm, and
coefficient of static friction = 0.4. The tension in
each shaft is 5 kN. Both screws are prevented from
turning as the collar turns. Determine the minimum
torque M required to turn the collar in either
direction.

1341
7.3 Square-Threaded Screws Example 9, page 2 of 7
F
M
M
M
Direction of motion of screw
relative to supporting threads
Direction of motion of screw
Direction of motion of screw
Formulas For A Screw With A Single Square Thread
Let F be the resultant axial force acting on the screw,
exclusive of the force from the supporting threads). Then
for screw motion in a direction opposite to F:
M = Fr tan ( + ) (1)
in which M is the torque required to turn the screw; r is the
mean radius of the threads; = tan
-1
[L/(2 r)], where L is
the lead of the screw; is the angle of friction (= tan
-1
),
where is the coefficient of friction.
Condition for screw to be of self-locking:
tan (2)
Self-locking screw with motion in same direction as axial
load:
M = Fr tan ( ) (3)
Non-self-locking screw with motion in same direction as
axial load:
M = Fr tan ( ) (4)
in which M is the minimum torque that must be applied to
the screw to prevent it from turning on its own.
1 Sense of rotation produced by torque M applied to screw
F
F

1342
7.3 Square-Threaded Screws Example 9, page 3 of 7
5 kN 5 kN
Apply a torque M

e to the collar with the
sense chosen arbitrarily as shown.
2
3 Sense of rotation of collar
produced by torque M

e
B
A
M

e
5 kN 5 kN
M

e
A B
Because screw B has a right-handed thread,
it will move left relative to the collar.
4
Impending motion of
screw B
Note that the motion of screw B is in a direction
opposite to the 5-kN axial load acting on the
screw. It follows that the appropriate formula to
use to calculate M
B
, the torque required to turn
screw B, is Eq. 1, which applies when screw
motion and axial force are in opposite direction.
5

1343
7.3 Square-Threaded Screws Example 9, page 4 of 7
Apply Eq. 1.
M
B
= Fr tan ( ) (Eq. 1 repeated)
Here M
B
is the torque acting on screw B. M
B
is not
Me, as will be shown shortly. The other parameters in
Eq. 1 have the following values:
F = 5000 N (5)
r = 0.008 m
= tan
-1
[L/(2 r)]
= tan
-1
[0.005 m/(2 (0.008))]
= 5.681 (6)
= tan
-1

= tan
-1
(0.4)
21.801 (7)
Substituting these values into Eq. 1 gives
M
B
= 20.807 N m (8)
6

1344
7.3 Square-Threaded Screws Example 9, page 5 of 7
5 kN 5 kN
Sense of rotation of collar
produced by torque M

e
Impending motion of screw A
A
M

e
B
Next consider screw A. Because screw A has a
right-handed thread, it will move left relative to the
collar when torque Me is applied.
7
8 Because the motion of screw A is in the same
direction as the axial load acting on the screw, the
appropriate formula to use to calculate M
A
, the torque
required to turn screw A, is either Eq. 3 or Eq. 4. To
decide which equation to use, we have to determine if
the screw is self-locking.

1345
7.3 Square-Threaded Screws Example 9, page 6 of 7
Condition for a screw to be self-locking:
0.4
tan (Eq. 2 repeated)
tan 5.68 by Eq. 6
= 0.099
Thus the inequality is satisfied and the screws are
self-locking. Eq. 3 is the appropriate equation for
calculating M
A
, the torque acting on screw A:
M
A
= Fr tan ( ) (Eq. 3 repeated)
Here
F = 5000 N
r = 0.008 m
= 21.801 (Eq. 7 repeated)
= 5.681 (Eq. 6 repeated)
Substituting these values into Eq. 3 gives
M
A
= 11.561 N m (9)
9

1346
7.3 Square-Threaded Screws Example 9, page 7 of 7
F 5 kN
M
A
11.561 N m by Eq. 9
(Torque from screw A
resisting rotation of collar)
11
M
B
20.807 N m by Eq. 8
(Torque from screw B
resisting rotation of collar)
13
B A
M

e
Free-body diagram of collar (alone, without screws A and B)
Torque applied externally to
collar to make it rotate
12
F 5 kN
Finally, the torque Me applied to the collar can be
related to M
A
and M
B
, the torques required to turn
the screws, by considering a free-body diagram:
10
14 Moment equilibrium equation:
M
axial
= 0: M
A
+ Me M
B
= 0
Thus
Me = M
A
+ M
B
= 11.561 N m + 20.807 N m
= 32.4 N m Ans.
+
We initially chose the sense of the torque Me arbitrarily. Had
we chosen Me with the opposite sense, however, it would have
made no difference in the final answer, because the new value
of M
B
would be equal to the old value of M
A
, and the new value
of M
A
would be equal to the old value of M
B
. That is, M
B
and
M
A
would merely have switched values, and their sum, which is
equal to Me, would have remained the same. Thus the torque
calculated, Me, is the minimum torque required to turn the
collar in either direction.
15

1347

7.4 Flat Belts
1348
7.4 Flat Belts Procedures and Strategies, page 1 of 1
T
1 T
2
Direction of
impending motion
Procedures and Strategies for Solving Problems Involving
Friction: Flat Belts
1. Draw free-body diagrams and write equilibrium equations.
2. Apply the equation
T
2
= T
1
e
in which
T
2
= belt tension in the direction of impending motion
T
1
= belt tension in the direction opposite the motion
coefficient of friction
= angle of wrap, measured in radians

1349
7.4 Flat Belts Problem Statement for Example 1
}
n turns
600 lb
5 lb
1. Determine the minimum number of turns of rope that will allow
the 5-lb force to support the 600-lb block, if the coefficient of
static friction is 0.15.

1350
7.4 Flat Belts Problem Statement for Example 2
P
150 lb
A
B
60
Rope is horizontal.
2. If the coefficient of static friction between the rope and the
fixed circular drums A and B is 0.2, determine the largest value
of the force P that can be applied without moving the 150-lb
weight upwards.

1351
7.4 Flat Belts Problem Statement for Example 3
A
B
C
P

80
o
15 lb-ft
D
15 in. 10 in.
6 in.
3. Determine the smallest force P applied to the handle of the
band brake that will prevent the drum from rotating when the
15 lbft moment is applied. The coefficient of static friction
is 0.25, and the weight of lever arm ABC can be neglected.

1352
7.4 Flat Belts Problem Statement for Example 4
4. The uniform beam ABC weighs 40 lb. The coefficient
of static friction between the cord and the fixed drum D is
0.3. Determine the smallest value of the weight W for
which the beam will remain horizontal.
D
A
B W C
4 ft 4 ft

1353
7.4 Flat Belts Problem Statement for Example 5
80 lb
A
B
W
70
50
5. Determine the largest value W of the weight
of block B for which neither block will move.
The coefficients of static friction are 0.2 between
the blocks and the planes, and 0.25 between the
cord and the drum.

1354
7.4 Flat Belts Problem Statement for Example 6
22 in.
B
T
7 in.
Driven pulley
A
200 lbin.
Driving pulley
4 in.
6. A motor attached to pulley A drives the pulley clockwise with a 200 lb-in. torque.
The flat belt then overcomes the resisting torque T at pulley B and rotates the pulley B
clockwise. Determine the minimum tension that can exist in the belt without causing
the belt to slip at pulley A. Also determine the corresponding resisting torque T. The
coefficient of static friction between the belt and the pulleys is 0.3.

1355
7.4 Flat Belts Problem Statement for Example 7
7. If the coefficient of static friction between the fixed
drums D and E and the ropes is 0.35, determine the
largest weight W that can be supported.
W
A
D
B
C
100 lb
50 lb
E

1356
7.4 Flat Belts Problem Statement for Example 8
80 mm
80 mm
6 Nm T
C
A
B
Driving pulley Driven pulley
8. Pulley A is rotating under the action of a 6 N-m torque. This motion is transmitted
through a flat belt to drive pulley B, which is in turn acted upon by a resisting torque T
(the "load" on pulley B). The coefficient of static friction between the belt and the
pulleys is 0.45. Determine a) the maximum possible value of T, b) the maximum force
in the belt, and c) the corresponding force required in the spring C.

1357
7.4 Flat Belts Example 1, page 1 of 2
1. Determine the minimum number of turns of rope that will allow
the 5-lb force to support the 600-lb block, if the coefficient of
static friction is 0.15.
}
n turns
600 lb
5 lb
5 lb
Tensions in the rope
Impending motion
(The motion can't be up, since the 5-lb force is too
small to lift the 600-lb block.)
600 lb
1
2

1358
7.4 Flat Belts Example 1, page 2 of 2
Since = 0.15, Eq. 1 becomes

600 lb = (5 lb) e
Solving gives = 31.917 radians. If n is the
number of turns of rope, then the number of
radians is 2 n. Equating this to gives an
equation for n:
2 n =
= 31.917 rad
Solving for n gives
n = 5.08 turns
Rounding off to the next higher integer then gives
n = 6 Ans.
Apply the equation for belt friction:
T
2
= T
1
e (1)
In this equation, T
2
is the tension in the direction
of impending motion
T
2
= 600 lb
The other tension, T
1
, is in the direction opposite
the impending motion, so
T
1
= 5 lb
3 4

1359
7.4 Flat Belts Example 2, page 1 of 2
2. If the coefficient of static friction between the rope and the
fixed circular drums A and B is 0.2, determine the largest value
of the force P that can be applied without moving the 150-lb
weight upwards.
P
150 lb
A
B
60
Rope is horizontal.
Impending motion
(The rope must move to
the right if the 150-lb
weight moves up)
150 lb
T

AB
=
A
Tensions in the rope on either side of drum A

1
Because slip impends between the rope and the drum, we can
apply the equation for belt friction:
T

2
= T

1
e (1)
where T
2
is the tension in the direction of impending motion.
In our particular problem,
T
2
= T
AB
and the tension opposing the impending motion is
T
1
= 150 lb
Using these results and = 0.2 and = 2 in Eq. 1 gives
T

AB
= (150)e
= 205.4 lb
0.2( /2)
2

1360
7.4 Flat Belts Example 2, page 2 of 2
B
60
Impending
motion
P
T

AB
= 205.4 lb
Flat-belt friction equation:
T

2
= T

1
e
so,
P = (205.4 lb)e (2)
4
60
60
30
60
Geometry 5
= 60 = rad
6 Using = 60 = rad in Eq. 2 gives
P = (205.4)e
= 253 lb Ans.
/
Tensions in the rope on
either side of drum B
3

1361
7.4 Flat Belts Example 3, page 1 of 3
6 in.
3. Determine the smallest force P applied to the handle of the
band brake that will prevent the drum from rotating when the
15 lbft moment is applied. The coefficient of static friction
is 0.25, and the weight of lever arm ABC can be neglected.
A
B
C
P

D

x
D

y
T

B
T

A
6 in.
D
Impending slip of
band relative to
drum (An observer
on the drum would
see the belt move
down.)
Free-body diagram of drum
3
+
Equilibrium equation for drum
M

D
= 0: T

A
(6 in.) T

B
(6 in.) 180 lbin. = 0 (1)
2
1
80
80
o
o
15 lb-ft
D
15 lbft = 15 lbft 12 in./ft
= 180 lbin.
15 in. 10 in.

1362
7.4 Flat Belts Example 3, page 2 of 3
80
80
10
10
Geometry 5
The belt-friction equation is in general
T

2
= T

1
e (2)
where T
2
is the tension in the direction
of impending slip. In our particular
problem,
T
2
= T
A

and the tension opposing the impending
motion is
T
1
= T
B
Eq. (2) becomes
T

A
= T

B
e (3)
Eq. 3 becomes, with 3.316 and 0.25
T
A
= T
B
e
0.25(3.316)

Solving Eqs. 1 and 4 simultaneously gives
T
A
= 53.237 lb
T
B
= 23.237 lb
4
= 180 + 10
= 190
= (190/180) rad
= 3.316 rad

1363
7.4 Flat Belts Example 3, page 3 of 3
+
7
Ax
T

A
= 53.237 lb
T

B
cos 80
6 Free-body diagram of lever arm ABC
15 in. 10 in.
P
B A
Equilibrium equation for lever arm
M

A
= 0: [(23.237 lb) sin 80](10 in.) P(10 in. + 15 in.) = 0 (5)
Solving gives
P = 9.15 lb Ans.
A
y
T

B
sin 80 = (23.237 lb) sin 80

1364
7.4 Flat Belts Example 4, page 1 of 3
4. The uniform beam ABC weighs 40 lb. The coefficient
of static friction between the cord and the fixed drum D is
0.3. Determine the smallest value of the weight W for
which the beam will remain horizontal.
Free-body diagram of block B
(Tension in the cord)
If end A of the beam
is about to move
down, then block B
is about to lose
contact with the
beam. Thus the
normal force N is
zero.
Equilibrium equation for block B
F

y
= 0: T

B
W = 0 (1)
D
A
B W C
4 ft 4 ft
W
N = 0
T
B
1
2
3
+

1365
7.4 Flat Belts Example 4, page 2 of 3
Free-body diagram of beam
T

A
is the tension in the cord.
The 40-lb weight of the beam acts
through the center of the span.
Equilibrium equation for the beam
M

C
= 0: (40 lb)(4 ft) T

A
(4 ft + 4 ft) = 0 (2)
Solving gives
T
A
= 20 lb
Because the cord is on the verge of slipping, we can
apply the equation for belt friction:
T

2
= T

1
e (3)
where T
2
= 20 lb is the tension in the direction of
impending slip and T
1
= T
B
is the tension opposite the
direction of impending motion
Tensions in
cord on either
side of drum
C B
A
4 ft 4 ft
T
A
= 20 lb
Impending
motion
40 lb
N = 0
C
y
Cx
y
x
4
5
6
7
8
9
+
T
B

1366
7.4 Flat Belts Example 4, page 3 of 3
T
B
10
T
A

=
Geometry
With and 0.3, Eq. 3 becomes
T

2
= T

1
e (Eq. 3 repeated)
20 lb 0.3
Solving gives
T
B
= 7.79 lb
Eq. 1 then gives
W = T
B
= 7.79 lb Ans.
11

1367
7.4 Flat Belts Example 5, page 1 of 4
80 lb
A
B
W
70
50
80 lb
A
T
A
N
A
f
A
y
x
5. Determine the largest value W of the weight
of block B for which neither block will move.
The coefficients of static friction are 0.2 between
the blocks and the planes, and 0.25 between the
cord and the drum.
Free-body diagram of block A
Impending motion of
block A (Since we are
to determine the
"largest value of the
weight" of block B,
block B must be about
to move down the
plane. Thus block A
must be about to
move up the plane.)
70
Equilibrium equations for block A:
Fx = 0: T
A
f
A
(80 lb) sin (1)
F

y
= 0: N
A
(80 lb) cos = 0 (2)
Slip impends, so

f
A
= f
A-max
= N
A
= (0.2)N
A
(3)
+
+
1
2
3

1368
7.4 Flat Belts Example 5, page 2 of 4
Geometry
Solving Eqs. 1, 2, and 3, with = 70 gives
T

A
= 80.65 lb
f

A
= 5.47 lb
N

A
= 27.36 lb
Tensions in cord on either side of drum
Impending motion
(Block B moves down plane)
Because the cord is about to slip over the drum, we can
apply the equation for belt friction:
T
2
= T
1
e (4)
where T
2
= T
B
is the tension in the direction of impending
slip, and T
1
= 80.65 lb is the tension opposite to the direction
of impending motion.
A
T
A
= 80.65 lb
T
B
4
5
6
7
drum
20
20
= 70
70

1369
7.4 Flat Belts Example 5, page 3 of 4
Substituting = 2 /3, T
2
= T
B
, T
1
= 80.65 lb,
and
drum
0.25 in Eq. 4 gives
T

B
= (80.65 lb) e
Evaluating the right hand side yields
T

B
= 136.1 lb
9
8
Geometry
= 70 + 50

= 120

= rad
70
20
70 50
50
70
50
40
2
3

1370
7.4 Flat Belts Example 5, page 4 of 4
Free-body diagram of block B Equilibrium equations for block B:
F

x' =0: N

B
W cos = 0 (5)

F

y'
= 0: 136.1 lb + f

B
W sin = 0 (6)
f

B
= f
B-max
= N

B
= (0.2)N

B
(7)
Solving Eqs. 5, 6, and 7, with = 50 gives
f

B
= 27.4 lb
N

B
= 137.2 lb
W = 213 lb Ans.
Geometry
B
= 50
40
50
B
50
N
B
T
B
= 136.1 lb
W
Impending
motion
y
x
f
B
10
12
11
13
+
+

1371
7.4 Flat Belts Example 6, page 1 of 4
6. A motor attached to pulley A drives the pulley clockwise with a 200 lb-in. torque.
The flat belt then overcomes the resisting torque T at pulley B and rotates the pulley B
clockwise. Determine the minimum tension that can exist in the belt without causing
the belt to slip at pulley A. Also determine the corresponding resisting torque T. The
coefficient of static friction between the belt and the pulleys is 0.3.
22 in.
B
T
7 in.
Driven pulley
A
200 lbin.
Driving pulley
4 in.

1372
7.4 Flat Belts Example 6, page 2 of 4
1
2
Equilibrium equation for pulley A
M

A
= 0: T
D
(4 in.) T
C
(4 in.) 200 lbin. = 0 (1)


+
3
Free-body diagram of pulley A
Impending motion of belt relative to pulley
(Since pulley A is driven by a clockwise
torque, the pulley would slip in a clockwise
sense relative to the belt. Thus an observer
at point D on the pulley would see the belt
move in the direction shown.)
200 lbin.
A
T
C
T
D
A

y
A

x
4 in.
C
D
Because the belt is about to slip on the pulley, the belt friction
equation applies:
T

2
= T

1
e
where T
2
= T
D
, the tension in the direction of impending motion,
and T
1
= T
C
, so
T
D
= T
C
e (2)

1373
7.4 Flat Belts Example 6, page 3 of 4
4 Geometry


4 in.
4 in.
22 in.
Radius = 7 in.
A B
C

Solving Eqs. 1 and 3 simultaneously gives
T
C
= 36.65 lb
T
D
= 86.65 lb Ans.
5
B

x
B

y
T
T
C
= 36.65 lb
T
D
= 86.65 lb
7 in.
Free-body diagram of pulley B 6
Equilibrium equation for pulley B
M

B
= 0: T + (36.65 lb)7 in. (86.65 lb)7 in. = 0
Solving gives
T = 350 lbin. Ans.
+
7
From triangle ABC,
cos (

2
) =
3 in.
22 in.
T
D
= T
C
e
0.3(2.868)
(3)
Solving gives = 164.33 = 2.868 rad.
So Eq. 2, with = 0.3, becomes
7 in. 4 in. = 3 in.

1374
7.4 Flat Belts Example 6, page 4 of 4
B
A
Finally, we must check that pulley B does not slip. But this follows
from the observation that pulley B has a larger angle of wrap than
pulley A. Thus it must be able to carry a maximum possible tension
larger than the 86.5 lb maximum tension that pulley A carries.
8
'
86.65 lb tension
36.65 lb tension
More precisely, because
' >
we know that
e
'
> e .

Multiplying through by 36.65 lb gives
(36.65 lb)e
'
> (36.65 lb)e
9
T
max-B
, the maximum
possible tension that
pulley B could support
without slipping.
86.65 lb, by Eq. 2
Thus T
max-B
> 86.65 lb and pulley B won't slip.

1375
7.4 Flat Belts Example 7, page 1 of 2
W
T
E
T
D
C
C
100 lb
50 lb
E
7. If the coefficient of static friction between the fixed
drums D and E and the ropes is 0.35, determine the
largest weight W that can be supported.
Free-body diagram of block C
Equilibrium equation for block C
F

y
= 0: T
E
+ T
D
W = 0 (1)
1
2
+
W
A
D
B

1376
7.4 Flat Belts Example 7, page 2 of 2
D
E
=
100 lb
Impending
motion
50 lb
T
E
Rope tensions acting on drum D
Impending motion (Since we are to determine the largest
value of weight of block C that can be supported, block C
must be about to move down, so the rope connected to C
must also be about to move down.)
Because slip is about to occur, the belt-friction equation
applies:
T

2
= T

1
e
Here, T
2
= T
D
, T
1
= 100 lb, 0.35, and so
T

D
= (100 lb)e
= 300.3 lb (2)
Using the results of Eqs. 2 and 3 in
Eq. 1 gives
W = 450 lb Ans.
Flat-belt friction equation:
T

2
= T

1
e

or,
T

E
= (50 lb)e
0.35
= 150.1 lb (3)
3
4
7
8
5
Rope tensions acting on
drum E (not drum D)
6
=
T
D

1377
7.4 Flat Belts Example 8, page 1 of 3
80 mm
80 mm
6 Nm T
C
A
B
Driving pulley Driven pulley
8. Pulley A is rotating under the action of a 6 N-m torque. This motion is transmitted
through a flat belt to drive pulley B, which is in turn acted upon by a resisting torque T
(the "load" on pulley B). The coefficient of static friction between the belt and the
pulleys is 0.45. Determine a) the maximum possible value of T, b) the maximum force
in the belt, and c) the corresponding force required in the spring C.

1378
7.4 Flat Belts Example 8, page 2 of 3
6 N-m
A
y
Ax
=
A
T
1
T
2
Free-body diagram of pulley A
Impending motion (Since the driving torque acting
on pulley A is clockwise, if slip is about to occur,
then pulley A will slip clockwise relative to the
belt. An observer located on point D on pulley A
would see the belt move in the direction shown.)
Equilibrium equations for pulley A
F

x = 0: A

x T

1
T

2
= 0 (1)
F
y
= 0: A
y
= 0 (2)
M

A
= 0: T

2
(0.08 m) T

1
(0.08 m) 6 Nm = 0 (3)
Flat-belt friction equation:
T

2
= T

1
e

= T

1
e (4)
Solving Eqs. 1-4 simultaneously gives
A

x = 123.21 N
A
y
= 0
T

1
= 24.11 N
T

2
= 99.11 N Ans.
1
2
3
4
+
+
80 mm
+
D

1379
7.4 Flat Belts Example 8, page 3 of 3
B
T
2
= 99.11 N
T
1
= 24.11 N
B
y
Bx
F
spring Ax = 123.21 N
Free-body diagram of member AC
Equilibrium equations for member AC
F

x = 0: F

spring
123.21 N = 0 (4)
Solving gives
F

spring
= 123.21 N Ans.
Equilibrium equations for pulley B
M

B
= 0: T + (24.11 N)(0.08 m) 99 (0.08 m) = 0
Solving gives
T = 6 Nm Ans.
That is, the maximum resisting torque equals the driving
torque.
Free-body diagram of pulley B
5
6
7
8
+
+
T
80 mm

1380

7.5 Thrust Bearings and Disks
1381
7.5 Thrust Bearings and Disks Procedures and Strategies, page 1 of 1
Procedures and Strategies for Solving Problems Involving
Frictional Forces on Thrust Bearings and Disks
R
1
R
2
M
P
Frictional force = p dA,
so torque = rp dA
1. If the axial force P can be assumed to be distributed
uniformly over the bearing surface, then you can calculate the
resisting torque acting on the shaft (or disk) from the equation
M =
2. If the force P is distributed in a non-uniform manner, then
you must calculate M by evaluating an integral:
M = r p dA
where the normal pressure p = p(r) varies with radial distance r
from the center of the disk and satisfies the equation
P = p dA
R
2
2
R
1
2
3
2 P R
2
3
R
1
3
r
Coefficient of
friction =

1382
7.5 Thrust Bearings and Disks Problem Statement for Example 1
70 mm
M P
1. A shaft rotates at constant angular velocity and is supported
by a collar bearing. Assuming that the normal pressure between
the collar and supporting surface is uniform, determine the
torque M required to overcome friction. The coefficient of
kinetic friction is
k
= 0.3, and the axial force is 1.5 kN.
120 mm

1383
7.5 Thrust Bearings and Disks Problem Statement for Example 2
S
S
A
B
C
D
2. A hand-held sander has a rotating disk with a circular piece of
sandpaper attached to the bottom of the disk. When the sander is
applied to the board ABCD, nails are driven in at corners A and C to
keep the board from rotating. Assuming that the downward force
exerted on the sander is P = 50 N and that this force is distributed
uniformly over the sander disk, determine the smallest pair of parallel
forces S from the nails required to keep board in place. The
coefficient of kinetic friction is
k
= 0.5.
M
P
Moment and force applied to
the sander
250 mm
350 mm
150 mm

1384
7.5 Thrust Bearings and Disks Problem Statement for Example 3
R
1
R
2
M P
3. Assuming that the contact pressure between the conical
thrust bearing and its support surface is uniform, determine
the torque M required to rotate the shaft at a constant
velocity. The coefficient of kinetic friction is
k
, and the
axial load is P.

1385
7.5 Thrust Bearings and Disks Problem Statement for Example 4
4 in.
M P M P
4. A clutch may be modeled as two opposing circular disks loaded by an axial
force P and a torque M. Given that P = 100 lb and the coefficient of static
friction is s = 0.5, determine the maximum torque that can be applied
without slipping if a) the clutch is new, and b) the clutch has experienced
wear proportional to the distance that a point on the clutch surface has
traveled.

1386
7.5 Thrust Bearings and Disks Problem Statement for Example 5
15 mm
30 mm
po
r (mm)
M
P
p = po
(30 mm r)
(15 mm)
5. Because of wear in the collar bearing surface, the pressure
transmitted decreases linearly with radial distance from the
center of the shaft as shown. If the coefficient of kinetic
friction is
k
= 0.3, and the axial force transmitted by the shaft
is P = 2 kN, determine the torque M required to maintain the
rotation of the shaft at a constant angular velocity.

1387
7.5 Thrust Bearings and Disks Example 1, page 1 of 1
70 mm
M P
1. A shaft rotates at constant angular velocity and is supported
by a collar bearing. Assuming that the normal pressure between
the collar and supporting surface is uniform, determine the
torque M required to overcome friction. The coefficient of
kinetic friction is
k
= 0.3, and the axial force is 1.5 kN.
Because the pressure between the surfaces of
contact is uniform, we can use the following
formula for disc friction:
M = (1)
Here
M =
= 21.9 N m Ans.
R
2
3
R
1
3
R
2
2
R
1
2
(120 mm/2)
3
(70 mm/2)
3
(120 mm/2)
2
(70 mm/2)
2
1
(0.3)(1.5 kN)
2
3
2
3
k
P
120 mm

1388
7.5 Thrust Bearings and Disks Example 2, page 1 of 2
S
S
A
B
C
D
2. A hand-held sander has a rotating disk with a circular piece of
sandpaper attached to the bottom of the disk. When the sander is
applied to the board ABCD, nails are driven in at corners A and C to
keep the board from rotating. Assuming that the downward force
exerted on the sander is P = 50 N and that this force is distributed
uniformly over the sander disk, determine the smallest pair of parallel
forces S from the nails required to keep board in place. The
coefficient of kinetic friction is
k
= 0.5.
M
P
Moment and force applied to
the sander
250 mm
350 mm
150 mm

1389
7.5 Thrust Bearings and Disks Example 2, page 2 of 2
S
S
M = 1250 N mm
A
B
C D
250 mm
350 mm
Because the pressure between the surface of contact is uniform, we can
use the following formula for disk friction:
M = (1)
Here
M = (0.5)(50 N)
= 1250 N mm
(150 mm/2)
3
(0)
3
(150 mm/2)
2
(0)
2
1
2 Free-body diagram of sander and board
ABCD (top view)
3 Forces S must be perpendicular to line AD, if S is to be as
small as possible and still produce the required moment:
4 Moment equilibrium
M
A
= S (250 mm)
2
+ (350 mm)
2
1250 N mm = 0
Solving gives
S = 2.91 N Ans.
+
R
2
3
R
1
3
R
2
2
R
1
2
2
3
k
P
2
3

1390
7.5 Thrust Bearings and Disks Example 3, page 1 of 5
R
1
R
2
M P
3. Assuming that the contact pressure between the conical
thrust bearing and its support surface is uniform, determine
the torque M required to rotate the shaft at a constant
velocity. The coefficient of kinetic friction is
k
, and the
axial load is P.

1391
7.5 Thrust Bearings and Disks Example 3, page 2 of 5
+
M P
M
f
x
Frictional forces from the
supporting surface produce
a resisting torque M
f
.
Free-body diagram of shaft.
p (Pressure from supporting surfaces; p is not yet known.)
Sum the forces in the x direction.
Fx = 0: P Rx = 0 (1)
Here Rx is the x component of the resultant
of the pressure p.

Sum the moments about the x axis.
Mx = 0: M M
f
= 0 (2)
+
1
2
p
y

1392
7.5 Thrust Bearings and Disks Example 3, page 3 of 5
R
2
R
1
r
s
r d
d
ds
3 Relate Rx to the pressure p. 4
The force on the differential area dA is p dA. Let
dRx equal the x-component of this force:
dRx = p dA sin (3)
Here
dA = r d ds
Express s in terms of r:
r/s = sin
Thus
ds = dr/sin
and so
dA = r d ds
= r d dr/sin
Hence by Eq.3 ,
dRx = p sin dA (Eq. 3 repeated)
= p sin (r d dr/sin )
= pr d dr (5)
5 Rx can be now found by integration,
Rx = dRx
= p r d dr
= p(R
2
2
R
1
2
) (6)
2
0 R
1
R
2
pr d dr, by Eq. 5 A
dRx
p dA

1393
7.5 Thrust Bearings and Disks Example 3, page 4 of 5
Substituting the expression for Rx in Eq. 6 into Eq. 1 gives
P Rx= 0 (Eq. 1 repeated)
Solving this equation for the (unknown) pressure p lets us express p in
terms of the axial force P:
p = (7)
p(R
2
2
R
1
2
), by Eq. 6
P
(R
2
2
R
1
2
)
6

1394
7.5 Thrust Bearings and Disks Example 3, page 5 of 5
r
r d
r
7 Calculate M
f
8 Normal force on dA = p dA
r d dr/sin by Eq. 4
9 Friction force =
k
normal force

=
k
pr d dr/sin
Torque produced by friction force:
dM
f
= (
k
pr d dr/sin r (8)
10 M
f
can now be found by integration.
M
f
= dM
f

= r
2
dr d
= (R
2
3
R
1
3
)
= Ans.

k
pr
2
d dr/sin by Eq. 8
k
p
sin
R
1

R
2

0
2
2
k
p
3 sin
P
(R
2
2
R
1
2
)
, by Eq. 7
k
P
3 sin
(R
2
3
R
1
3
)
(R
2
2
R
1
2
)
M, by Eq. 2

1395
7.5 Thrust Bearings and Disks Example 4, page 1 of 4
4 in.
M P M P
4. A clutch may be modeled as two opposing circular disks loaded by an axial
force P and a torque M. Given that P = 100 lb and the coefficient of static
friction is s = 0.5, determine the maximum torque that can be applied
without slipping if a) the clutch is new, and b) the clutch has experienced
wear proportional to the distance that a point on the clutch surface has
traveled.
Part a): If the clutch is new, the surface of the disks are flat and it is
reasonable to assume that the pressure transmitted between the disks is
uniform. In this case, we can use the following formula for the disk friction:
M = sP (1)
M = (0.5)(100 lb)
= 133.3 lb.in. Ans.
R
2
3
R
1
3
R
2
2
R
1
2
(4 in.)
3
0
(4 in.)
2
0
2
3
2
3
1

1396
7.5 Thrust Bearings and Disks Example 4, page 2 of 4
2 Part b): If the clutch is worn, the pressure between disks is
not uniform, and Eq. 1 can no longer be used.
If the wear on a disk is proportional to the distance
traveled by a point on the disk, then for a point at a
distance r from the center, the wear is proportional to the
circumference 2 r of the circle traced out by the point as
the shaft rotates. That is, wear is proportional to r. But if
the wear between the disks increases with r, it seems
reasonable to suppose that the pressure decreases with r.
The simplest way of modeling this decrease in pressure is
to assume that the pressure varies as 1/r, that is
p = k/r (2)
where k is a constant of proportionality to be determined.

1397
7.5 Thrust Bearings and Disks Example 4, page 3 of 4
M P
r
dr
r d
d
4 in.
3 Determine the value of k by requiring the resultant of
the pressure forces to equal the axial force P.
Normal force acting on differential
area:
dN = pr d dr
= k d dr (3)
4
5 Equate the axial force P to the resultant of the normal forces:
P = dN
= k d dr
= (25.133 in.)k
Solving for k gives
k = P/25.133 in.
(25.133 in.)r
k d dr, by Eq. 3
0
4
0
2

Substituting this result for k into Eq. 2 gives the relation
between the axial force P and the pressure
p = k/r (Eq. 2 repeated)
= (4)

P
k/r, by Eq. 2
dN
6

1398
7.5 Thrust Bearings and Disks Example 4, page 4 of 4
by Eq. 5
M P
r
4 in.
dN = pr d dr
7 Now compute the resultant moment produced by the friction forces.
df = friction force
= s dN
= spr d dr

(25.133 in.)r
P
, by Eq. 4
25.133 in.
sP d dr
= (5)
8 Equate the torque M applied by the shaft to the resisting
torque produced by the friction forces:
M = r df
= r d dr
=
= 100 lb in. Ans.
25.133 in.
sP d dr
0
4
0
2
25.133 in.
sP
25.133 in.
(0.5)(100 lb)
(50.265 in
2
)
Note that the worn clutch transmits appreciably less
torque without slipping than the new clutch
transmits (100 lb in. compared to 133 lb in.)
9

1399
7.5 Thrust Bearings and Disks Example 5, page 1 of 3
15 mm
30 mm
po
r (mm)
M
P
p = po
(30 mm r)
(15 mm)
5. Because of wear in the collar bearing surface, the pressure
transmitted decreases linearly with radial distance from the
center of the shaft as shown. If the coefficient of kinetic
friction is
k
= 0.3, and the axial force transmitted by the shaft
is P = 2 kN, determine the torque M required to maintain the
rotation of the shaft at a constant angular velocity.

1400
7.5 Thrust Bearings and Disks Example 5, page 2 of 3
M
P
15 mm
30 mm
r d
d
r
dr
Determine the value of p0 by requiring the resultant of the
pressure force to equal the axial force P.
1
dN = normal force on differential area, dA
= p dA
= p0 r d dr (1)
15 mm
(30 mm r)
2 Equate the axial force P (= 2 kN) to the resultant of the
normal forces:
2 kN = dN
= p0


r d dr
= p0 (942.478 mm
2
)
Solving for p0 gives
p0 = 2.122 N/mm
2
(2)
15 mm
30 mm
0
2
15 mm
(30 mm r)
dN
A

1401
7.5 Thrust Bearings and Disks Example 5, page 3 of 3
r
df
(30 mm r)
(30 mm r)
df = friction force
=
k
dN (3)
k
po
M
P
15 mm
30 mm
dN
3 Now compute the resultant moment produced by
friction forces.
5 Equate the torque M delivered by the shaft to the resisting
torque produced by friction
M = r df

= 12.38 N m Ans.
k
p0 r d dr, by Eq.3
15 mm
15 mm
=

15 mm
30 mm

0
2
(30 mm r) r
2
d dr
0.3 2.122 N/mm
2
, by Eq. 2
4
15 mm
p0 r d dr, by Eq. 1

1402

7.6 Journal Bearings
1403
7.6 Journal Bearings Procedures and Strategies, page 1 of 2
Procedures and Strategies for Solving Problems
Involving Frictional Forces on Journal Bearings
For problems involving a loosely fitting pulley (or wheel
or collar or similar device) on a fixed shaft, follow these
steps.
1. Calculate the angle of friction and radius r
f
of the
circle of friction from the equations
tan
-1
where is the coefficient of friction
r
f
= r sin where r is the radius of the shaft
(which, because the pulley fits "loosely", is
approximately equal to the inner radius of the
pulley)
2. Draw circles showing the positions and contact
point of the shaft and the central hole in the pulley,
when no rotation has occurred.
3. Draw circles showing the positions and contact
point of the shaft and the pulley hole, after rotation
has occurred.
F
F
Pulley
P
1
Fixed shaft
Contact point
F + F
P
2
Unbalanced forces
cause pulley to rotate to
new contact point.
F
Fixed shaft
Belt forces
acting on
pulley are
equal.
Pulley
Belt forces
acting on
pulley are
not equal.
For clarity, the difference in size between the
radius of the shaft and the inner radius of the
pulley has been exaggerated in the diagram.

1404
7.6 Journal Bearings Procedures and Strategies, page 2 of 2
4. Draw a sketch of the pulley, with the circle of friction inside. On
this sketch, show the friction force from the shaft opposing the
rotation. Furthermore, show the line of action of the resultant, say
R, of the normal and friction force passing through the contact
point and lying tangent to a circle of radius rf (the circle of friction).
5. Draw a free-body diagram of the pulley, expressing R in terms
of x and y components, Rx and R
y
.
6. Write equilibrium equations: Sum forces in the x and y
directions and sum moments about the center of the pulley.
Note that the moment of R is R r
f
, because R is tangent to
the circle of friction, which has radius r
f
. Also,
R = Rx
2
+ R
y
2
7. Solve the equations.
Note: The above discussion applies when the pulley fits loosely on
a fixed shaft. If instead the pulley is rigidly fixed to the shaft, but
the shaft fits loosely on a supporting bearing, then the steps above
can be followed, except that a) the shaft rotates to a new contact
point relative to the fixed bearing, and b) a free-body diagram of
the shaft rather than the pulley is used.
r
f
Line of action of
resultant, R, is
tangent to circle
of friction.
Frictional force
opposes rotation
F + F
F
N
R
f
P
2
Pulley
r
f
P
2
Pulley
R
y
R = Rx
2
+ R
y
2
Rx
Sense of
rotation of
pulley
Circle of friction
(not the shaft)
x
y

1405
7.6 Journal Bearings Problem Statement for Example 1
F
140 N
80 mm
1. The pulley has a radius of 80 mm and has
negligible weight. If the pulley fits loosely on a
12-mm-diameter fixed shaft, and the coefficient of
static friction is s = 0.2, determine the minimum
force F required to start the pulley rotating
clockwise.

1406
7.6 Journal Bearings Problem Statement for Example 2
135 N
140 N
80 mm
2. The pulley has a radius of 80 mm and has negligible
weight. If the pulley fits loosely on a 12-mm-diameter
fixed shaft, and the pulley rotates with constant angular
velocity counterclockwise, determine the coefficient of
kinetic friction,
k
.

1407
7.6 Journal Bearings Problem Statement for Example 3
130 mm
F
100 N
3. The force from the belt causes the 130-mm-radius
pulley to rotate counterclockwise with constant angular
velocity. The pulley fits loosely on a fixed shaft of
24-mm diameter. Determine the value of the belt force F
if the coefficient of kinetic friction is
k
= 0.3. Assume
that no slipping occurs between the belt and pulley.

1408
7.6 Journal Bearings Problem Statement for Example 4
8 in.
4 in.
20 lb 40.5 lb
4. The stepped pulley has radii of 4 in. and 8 in. and fits loosely on a
0.5-in.-diameter fixed shaft. If the given loads cause the pulley to
rotate clockwise with a constant angular velocity, determine the
normal and frictional forces from the shaft acting on the pulley. Also
find the coefficient of kinetic friction,
k
.

1409
7.6 Journal Bearings Problem Statement for Example 5
P
A
B
C
P
150 mm 150 mm
5. The couple forces P are intended to rotate the shaft and wind
the cable around the drum, thus raising the 5-kg mass. The
shaft has a diameter of 30 mm and fits loosely in the journal
bearing B. If the combined mass of the shaft handle A and
drum C is 15 kg, and the coefficient of static friction is s = 0.2,
determine the minimum value of P required to initiate upward
motion of the 5-kg mass. Assume the drum C is attached
rigidly to the shaft.
Diameter of drum = 200 mm
5 kg

1410
7.6 Journal Bearings Problem Statement for Example 6
10 in. 10 in.
A
B
100 lb W
6. Pulleys A and B are identical. Each has a radius of 10 in.,
weighs 10 lb and fits loosely on a 0.5-in.-diameter fixed shaft.
Determine the maximum value of the weight W that may be
supported without causing the pulleys to rotate. Also calculate
the value of the tension in the cord between the pulleys. The
coefficient of static friction is s = 0.2 .

1411
7.6 Journal Bearings Problem Statement for Example 7
100 kg
P
7. The 100-kg cart has four 200-mm-diameter wheels,
25-mm-diameter axles, and center of mass G. Each wheel has
a mass of 10 kg. The coefficient of kinetic friction is 0.06.
Determine the horizontal force P required to move the cart at
constant speed. Assume that the axles do not rotate, the wheels
fit loosely on the axles, and rolling resistance between the
wheels and the plane is negligible.
G
100 mm
400 mm 400 mm

1412
7.6 Journal Bearings Problem Statement for Example 8
8. The 100-kg cart has four 300-mm-diameter wheels,
30-mm-diameter axles, and center of mass G.
Determine the angle for which the cart will roll down
the incline at constant speed. Also determine the
reactions from the incline acting on each wheel. The
coefficient of kinetic friction is 0.2. Assume that
rolling resistance and the weight of the wheels are
negligible, the wheels fit loosely on the axles, and the
axles do not rotate.
G
200 mm
500 mm
500 mm

1413
7.6 Journal Bearings Example 1, page 1 of 3
F
140 N
80 mm
1. The pulley has a radius of 80 mm and has
negligible weight. If the pulley fits loosely on a
12-mm-diameter fixed shaft, and the coefficient of
static friction is s = 0.2, determine the minimum
force F required to start the pulley rotating
clockwise.
1 The angle of friction and the radius of the circle
of friction are
s = tan
-1

s
= tan
-1
0.2
= 11.3099
r
f
= r sin s
= (12/2 mm) sin 11.3099
= 1.1767 mm (1)

1414
7.6 Journal Bearings Example 1, page 2 of 3
140 N
2
Pulley
P
1
Fixed shaft
Contact
point
For clarity, the difference in size between the
radius of the shaft and the inner radius of the
pulley has been exaggerated in the diagram.
Suppose
F = 140 N
F
140 N
Pulley
P
1
New contact
point
P
2
Pulley rotates clockwise.
Position of pulley if rope
forces were equal
Rotated position of pulley caused by
increase in force F.
3

1415
7.6 Journal Bearings Example 1, page 3 of 3
Normal force N and friction force f
from shaft acting on pulley.
4
N
R
f
Line of action of
resultant, R, is tangent
to circle of friction.
Frictional force f
opposes clockwise
rotation of pulley
Pulley
P
2
Free-body diagram of pulley 5
R
y
R = Rx
2
+ R
y
2
Rx
r
f
O
F
140 N
80 mm
80 mm
Fx = 0: F + 140 N Rx = 0
F
y
= 0: R
y
= 0
M
O
= 0: (140 N)(80 mm) F(80 mm)

+ Rx
2
+ R
y
2

(r
f
) = 0
1.1767 mm, by Eq. 1
Solving gives
Rx = 284 N
R
y
= 0
F = 144.2 N Ans.
7 Equilibrium equations
+
+
+
Pulley
P
2
r
f
O
For convenience, express
R in terms of horizontal
and vertical components
(rather than in terms of N
and f).
6

1416
7.6 Journal Bearings Example 2, page 1 of 4
135 N
140 N
80 mm
2. The pulley has a radius of 80 mm and has negligible
weight. If the pulley fits loosely on a 12-mm-diameter
fixed shaft, and the pulley rotates with constant angular
velocity counterclockwise, determine the coefficient of
kinetic friction,
k
.
1 The angle of friction,
k
, and radius of the circle, r
f
,
can be expressed in terms of
k
:
k
= tan
-1
k
r
f
= r sin
k
= (12/2 mm) sin ( tan
-1
k
)

= 6 sin ( tan
-1
k
) (1)

1417
7.6 Journal Bearings Example 2, page 2 of 4
135 N
2
Pulley
P
1
Fixed shaft
Contact
point
For clarity, the difference in size between the
radius of the shaft and the inner radius of the
pulley has been exaggerated in the diagram.
135 N
140 N
Pulley
P
1
New contact
point
Pulley rotates counterclockwise.
Position of pulley if belt
forces were equal
Rotated position of pulley caused by
increase of belt force to 140 N.
3
Suppose that the
140 N force is
replaced by a 135
N force.
P
2

1418
7.6 Journal Bearings Example 2, page 3 of 4
Normal force N and friction force f
from shaft acting on pulley.
4
Line of action of resultant, R, is
tangent to circle of friction.
Frictional force opposes
counterclockwise
rotation of pulley.
P
2
R
f
N
r
f

1419
7.6 Journal Bearings Example 2, page 4 of 4
P
2
R
y
r
f
Rx
R = Rx
2
+ R
y
2
140 N
80 mm
80 mm
135 N
Free-body diagram of pulley 5
Fx = 0: 135 N + 140 N Rx = 0
F
y
= 0: R
y
= 0
M
O
= 0: (140 N)(80 mm) (135 N)(80 mm)

Rx
2
+ R
y
2

(r
f
) = 0
Solving gives
Rx = 275 N
R
y
= 0
r
f
= 1.455 mm
7 Equilibrium equations
+
+
+
8 Substituting the value of r
f
in Eq. 1 gives
r
f
= 6 sin (tan
-1
k
) (Eq. 1 repeated)
Thus
sin
-1
(1.455/6) = tan
-1
k
k
= tan [sin
-1
(1.455/6)]
= 0.25 Ans.
1.455 mm
6 For convenience, express R in
terms of horizontal and vertical
components (rather than in
terms of N and f).
O

1420
7.6 Journal Bearings Example 3, page 1 of 4
130 mm
1
F
100 N
3. The force from the belt causes the 130-mm-radius
pulley to rotate counterclockwise with constant angular
velocity. The pulley fits loosely on a fixed shaft of
24-mm diameter. Determine the value of the belt force F
if the coefficient of kinetic friction is
k
= 0.3. Assume
that no slipping occurs between the belt and pulley.
Calculate the angle of friction and the radius of
the circle of friction.
k
= tan
-1

k
= tan
-1
0.3
= 16.6992
r
f
= r sin
k
= (24/2 mm) sin 16.6992
= 3.4482 mm (1)

1421
7.6 Journal Bearings Example 3, page 2 of 4
Suppose F = 100 N
100 N
Pulley A
P
1
Fixed shaft
Contact point
100 N
Pulley A
P
1
New contact point
P
2
Pulley rotates counterclockwise.
F
For clarity, the difference in size between the radius
of the shaft and the inner radius of the pulley has
been exaggerated in the diagram.
2 Position of pulley if belt force were equal
3 Rotated position of pulley caused by a
decrease in value of force F.

1422
7.6 Journal Bearings Example 3, page 3 of 4
Pulley A
P
2
r
f
f
N
R
Frictional force opposes
counterclockwise rotation of
pulley.
Line of action of resultant, R, is tangent to circle of
friction.
Normal force N and friction force f
from shaft action on pulley
4

1423
7.6 Journal Bearings Example 3, page 4 of 4
100 N
Pulley A
P
2
F
r
f
R = Rx
2
+ R
y
2
130 mm
130 mm Rx
R
y
O
6 Equilibrium equations
Fx = 0: Rx 100 N = 0
F
y
= 0: R
y
F = 0
M
O
= 0: (100 N)(130 mm) F(130 mm)
Rx
2
+ R
y
2
(r
f
) = 0
Solving gives

Rx = 100 N
R
y
= 96.3 N
F = 96.3 N Ans.
+
+
+
3.4482 mm, by Eq. 1
5 For convenience, express R in
terms of horizontal and vertical
components (rather than in
terms of N and f).

1424
7.6 Journal Bearings Example 4, page 1 of 5
8 in.
4 in.
20 lb 40.5 lb
4. The stepped pulley has radii of 4 in. and 8 in. and fits
loosely on a 0.5-in.-diameter fixed shaft. If the given loads
cause the pulley to rotate clockwise with a constant angular
velocity, determine the normal and frictional forces from
the shaft acting on the pulley. Also find the coefficient of
kinetic friction,
k
.
1 The angle of friction,
k
, and radius of friction circle,
r
f
, can be expressed in terms of
k
:

k
= tan
-1

k
r
f
= r sin
k
= (0.5/2 in) sin (tan
-1

k
)
= 0.25 sin (tan
-1

k
) (1)

1425
7.6 Journal Bearings Example 4, page 2 of 5
P
1
Contact point
Fixed shaft
Pulley
40.5 lb 20 lb
Replace by 40 lb
P
2
New contact
point
Pulley rotates clockwise.
For clarity, the difference in size between the radius
of the shaft and the inner radius of the pulley has
been exaggerated in the diagram.
2 Position of pulley if belt force were
slightly altered to produce equal moments
(20 lb 8 in. = 40 lb 4 in.) about the
center of the pulley.
3 Rotated position of pulley caused by an increase of
belt force on the right to 40.5 lb.
8 in. 4 in.
P
1
40.5 lb 20 lb
8 in. 4 in.

1426
7.6 Journal Bearings Example 4, page 3 of 5
Line of action of resultant, R, is
tangent to circle of friction.
4
Frictional force f opposes the
clockwise rotation of the pulley.
Normal force N and friction force f
from shaft acting on pulley.
P
2
N
R
f
r
f

1427
7.6 Journal Bearings Example 4, page 4 of 5
O
R = Rx
2
+ R
y
2
Equilibrium equations
Fx = 0: Rx = 0
F
y
= 0: R
y
20 lb 40.5 lb = 0
M
O

= 0: (20 lb)(8 in.) (40.5 lb)(4 in.)
+ Rx
2
+ R
y
2

r
f
= 0
Solving gives

Rx = 0
R
y
= 60.5 lb
r
f
= 0.0331 in.
5 Free-body diagram of pulley
7
+
+
+
6 For convenience,
express R in terms of
horizontal and
vertical components
(rather than in terms
of N and f).
P
2
r
f
40.5 lb 20 lb
8 in. 4 in.
Rx
R
y

1428
7.6 Journal Bearings Example 4, page 5 of 5
9
R = Rx
2
+ R
y
2
= 0 + (60.5 lb)
2
= 60.5 lb
(line of action passes through P
2
and is tangent to the
friction circle)
8 Substituting the value of r
f
just computed into Eq. 1
gives
r
f
= 0.25 sin (tan
-1

k
) (Eq.1 repeated)
Thus,
sin
-1
(0.0331/0.25) = tan
-1

k
or

k
= tan sin
-1
(0.0331/0.25)
= 0.134 Ans.
The normal force N and friction force f can be
found by resolving the resultant R into components:
f = R cos
= (60.5 lb) cos 82.392
= 8.01 lb Ans.
N = R sin
= (60.5 lb) sin 82.392
= 60.0 lb Ans.
0.0331 in.
O
r
f
= 0.0331 in.
r
pulley
= 0.25 in.
N (line of action passes
through P
2
and center of
circle, O)
f
P
2
= cos
-1
(0.0331 in./0.25 in.)
= 82.392
Circle of friction
drawn to scale
relative to the
0.25-in. radius
of the hole in the
pulley

1429
7.6 Journal Bearings Example 5, page 1 of 4
P
A
B
C
P
150 mm 150 mm
5. The couple forces P are intended to rotate the shaft and wind
the cable around the drum, thus raising the 5-kg mass. The
shaft has a diameter of 30 mm and fits loosely in the journal
bearing B. If the combined mass of the shaft handle A and
drum C is 15 kg, and the coefficient of static friction is s = 0.2,
determine the minimum value of P required to initiate upward
motion of the 5-kg mass. Assume the drum C is attached
rigidly to the shaft.
Diameter of drum = 200 mm
1 The angle of friction and the radius of circle of
friction are
s = tan
-1
s

= tan
-1
0.2
= 11.3099
r
f
= r sin 11.3099
= (30 mm/2) sin 11.3099
= 2.9394 mm (1)
5 kg

1430
7.6 Journal Bearings Example 5, page 2 of 4
P
1
P
1
P
2
Fixed support bearing at B
Shaft (free to rotate
in support bearing)
Contact
point
New
contact
point
Rotation of shaft counterclockwise
For clarity, the difference in size between the
radius of the shaft and the radius of the support
bearing has been exaggerated in the diagram.
2 Position of the shaft, if external
forces were balanced so that no
friction force exists at point P
1
. That
is, no tendency exists to either raise
or lower the weight.
3 Rotated position of shaft caused by
increasing the forces P on the handle (The
shaft tends to "climb" the wall of the bearing
until it reaches a point where slipping
occurs).

1431
7.6 Journal Bearings Example 5, page 3 of 4
P
1
P
2
N
R
f
r
f
Frictional force opposes the
counterclockwise rotation of the shaft.
Line of action of
resultant, R, is tangent
to circle of friction
4 Normal force N and friction force f
from bearing acting on the shaft.

1432
7.6 Journal Bearings Example 5, page 4 of 4
R = Rx
2
+ R
y
2
Rx
R
y
P
Shaft
Drum C
O
P
150 mm 150 mm
Weight of handle and drum = (15 kg)(9.81 m/s
2
= 147.15 N
200/2 mm = 100 mm
r
f
Free body of shaft, handle A, and drum 5
Equilibrium equations
Fx = 0: Rx = 0
F
y
= 0: R
y
+ P P 147.15 N 49.05 N = 0
M
O
= 0: (2 P)(150 mm) (49.05 N)(100 mm)
(Rx
2
+ R
y
2
) (r
f
) = 0
Solving gives

Rx = 0
R
y
= 196.2 N
P = 18.3 N Ans.
7
+
+
+
2.9394 mm, by Eq. 1
Weight of block
= (5 kg)(9.81 m/s
2
= 49.05 N
6 For convenience, express R in
terms of horizontal and vertical
components (rather than in
terms of N and f).

1433
7.6 Journal Bearings Example 6, page 1 of 7
10 in. 10 in.
A
B
100 lb W
6. Pulleys A and B are identical. Each has a radius of 10 in.,
weighs 10 lb and fits loosely on a 0.5-in.-diameter fixed shaft.
Determine the maximum value of the weight W that may be
supported without causing the pulleys to rotate. Also calculate
the value of the tension in the cord between the pulleys. The
coefficient of static friction is s = 0.2 .
1 The angle of friction and the radius of the circle
of friction are
s = tan
-1
s

= tan
-1
0.2
= 11.3099
r
f
= r sin s
= (0.5 in./2) sin 11.3099
= 0.0490 in. (1)

1434
7.6 Journal Bearings Example 6, page 2 of 7
Pulley A
100 lb
Suppose
cord tension
is T = 100 lb
P
1
Fixed
shaft
Contact
point
Pulley A
100 lb
T > 100 lb
P
1
New
contact
point
P
2
Pulley rotates clockwise (We are to find the
maximum value of the weight W, so
impending motion of the cord is to the right).
For clarity, the difference in size between the
radius of the shaft and the inner radius of the
pulley has been exaggerated in the diagram.
Position of pulley A if cord
forces were equal
Position of pulley A caused by
increase in value of force T
2
3

1435
7.6 Journal Bearings Example 6, page 3 of 7
Pulley A
P
2
r
f
N R
f
Frictional force
opposes clockwise
rotation of pulley.
Line of action of resultant, R, is
tangent to circle of friction.
4 Normal force N and friction force f
from shaft acting on pulley A

1436
7.6 Journal Bearings Example 6, page 4 of 7
Pulley A
P
2
r
f
R
y
Rx
R = Rx
2
+ R
y
2
100 lb
T
10 in.
10 in.
10 lb
(Weight of the pulley)
O
5 Free-body diagram of pulley A
Equilibrium equations
Fx = 0: T Rx = 0
F
y
= 0: R
y
100 lb 10 lb = 0
M
O
= 0: (100 lb)(10 in.) T(10 in.)
(Rx
2
+ R
y
2
)(r
f
) = 0
Solving simultaneously gives

Rx

= 100.7309 lb
R
y
= 110.0000 lb
T = 100.7309 lb Ans.
7
+
+
+
0.0490 in., by Eq. 1
6 For convenience, express
R in terms of horizontal
and vertical components
(rather than in terms of N
and f).

1437
7.6 Journal Bearings Example 6, page 5 of 7
Pulley B
T = 100.7309 lb

Q
1
Fixed
shaft
Contact point
Suppose W = T = 100.7309 lb
Pulley B
T = 100.7309 lb
Q
1
New contact
point
W > 100.7309 lb
Q
2
Pulley rotates clockwise.
8
Position of pulley B if cord forces were equal 9 Rotated position of pulley B, caused by
increase in value of weight W

1438
7.6 Journal Bearings Example 6, page 6 of 7
Pulley B
Q
2
N
S
f
Frictional force
opposes clockwise
rotation of pulley.
Line of action of resultant, S, is
tangent to circle of friction.
r
f
10 Normal force N and friction force f
from shaft acting on pulley B

1439
7.6 Journal Bearings Example 6, page 7 of 7
r
f
Pulley B
Sx
S
y
Q
2
10 in.
10 in.
O
T = 100.7309 lb
W
10 lb
(Weight of the pulley)
S = Sx
2
+ S
y
2
11 Free-body diagram of pulley B
Write Equilibrium equations
Fx = 0: Sx

100.7309 lb = 0
F
y
= 0: S
y
W 10 lb = 0
M
O
= 0: (100.7309 lb)(10 in.) W(10 in.)
+ Sx
2
+ S
y
2
(r
f
) = 0
Solving Simultaneously gives

Sx = 100.7 lb
S
y
= 111.5 lb
W = 101.5 lb Ans.
12
+
+
+
0.0490 in., by Eq. 1

1440
7.6 Journal Bearings Example 7, page 1 of 5
100 kg
P
7. The 100-kg cart has four 200-mm-diameter wheels,
25-mm-diameter axles, and center of mass G. Each wheel has
a mass of 10 kg. The coefficient of kinetic friction is 0.06.
Determine the horizontal force P required to move the cart at
constant speed. Assume that the axles do not rotate, the wheels
fit loosely on the axles, and rolling resistance between the
wheels and the plane is negligible.
The angle of friction and the radius of the circle of
friction are
k
= tan
-1
s

= tan
-1
0.06 = 3.4336
r
f
= r sin
k

= (25/2 mm) sin 3.4336
= 0.7486 mm (1)
1
G
100 mm
400 mm 400 mm

1441
7.6 Journal Bearings Example 7, page 2 of 5
Weight of 100-kg
cart = 981 N
2G
1
2G
2
2F
1
2F
2
2 Free-body diagram of cart
Factor of "2"
because 2 rear
wheels.
Forces G
1
and G
2
normal to the
plane and F
1
and F
2
parallel to the
plane act on the four wheels.
3
P
G
100 mm
400 mm 400 mm
Equilibrium equations
Fx = 0: P 2F
1
2F
2
= 0 (1)
F
y
= 0: 2G
1
+ 2G
2
981 N 2(98.1 N) 2(98.1 N) = 0 (2)
M
A
= 0: 2(98.1 N)(400 mm) 2(98.1 N)(400 mm)
+ (2G
2
)(400 mm) (2G
1
)(400 mm) P(100 mm) = 0 (3)
+
+
+
A
Rolling resistance is negligible (The
coefficient of rolling resistance is
zero), so the force from the plane
acting on the wheel is concentrated at
a single point the point where the
wheel is tangent to the plane.
Weight of 2
10-kg wheels
= 2(98.1 N)

1442
7.6 Journal Bearings Example 7, page 3 of 5
Wheel
P
1
Contact point
Axle (does not
rotate)
Wheel
P
1
Clockwise rotation of wheel
P
2
New contact point
4 Relative position of axle and wheel when
cart is not moving (No friction force from
the plane acts on the wheel).
5 Relative position of axle and wheel when
cart is moving (As the friction force from
the plane increases from zero, it causes
the wheel to rotate about the contact point
P
1
initially. But this rotation causes the
contact point to shift up and to the left on
the axle. As the force increases further
and the wheel rotates more, the contact
point moves further left and up until
slipping occurs).
Friction force from plane
acting on wheel

1443
7.6 Journal Bearings Example 7, page 4 of 5
P
2
N
R
f
r
f
Frictional force opposes
clockwise rotation
Line of action of
resultant, R, is
tangent to circle of
friction.
Normal force N and friction
force f from axle acting on wheel.
6

1444
7.6 Journal Bearings Example 7, page 5 of 5
R
y1
R
x1
R = (R
x1
)
2
+ (R
y1
)
2
200/2 mm = 100 mm
7 Free-body diagram of a rear wheel
Equilibrium equations:
Fx = 0: R
x1

F
1
= 0 (4)
F
y
= 0: G
1
98.1 N R
y1
= 0 (5)
M
O

= 0: (F
1
)(100 mm) + (R
x1
)
2
+ (R
y1
)
2
(r
f
) = 0 (6)
A free-body diagram of a front wheel of the cart would appear
similar to the free-body diagram of the rear wheel. Thus the
equilibrium equations for a front wheel are identical to those
above except the subscript "1" is replaced by "2":
Fx = 0: R
x2

F
2
= 0 (7)
F
y
= 0: G
2
98.1 N R
y2
= 0 (8)
M
O

= 0: (F
2
)(100 mm) + (R
x2
)
2
+ (R
y2
)
2
(r
f
) = 0 (9)
Substituting the value r
f
= 0.7486 mm and then solving Eqs. 1-9
gives
R
x1
= F
1
= 1.83 N R
x2
= F
2
= 1.84 N G
1
= 342.89 N
G
2
= 343.81 N R
y1
= 244.79 N R
y2
= 245.71 N
P = 7.34 N Ans.
8
+
+
+
P
2
r
f
G
1
F
1
Weight of wheel
= 98.1 N
O

1445
7.6 Journal Bearings Example 8, page 1 of 5
8. The 100-kg cart has four 300-mm-diameter wheels,
30-mm-diameter axles, and center of mass G.
Determine the angle for which the cart will roll down
the incline at constant speed. Also determine the
reactions from the incline acting on each wheel. The
coefficient of kinetic friction is 0.2. Assume that
rolling resistance and the weight of the wheels are
negligible, the wheels fit loosely on the axles, and the
axles do not rotate.
1 The angle of friction and the radius of the
circle of friction are
k
= tan
-1
s
= tan
-1
0.08 = 4.5739
r
f
= r sin
k

= (30/2 mm) sin 4.5739
= 1.1962 mm (1)
G
200 mm
500 mm
500 mm

1446
7.6 Journal Bearings Example 8, page 2 of 5
x
y
Weight of 100 kg mass = 981 N
Factor of 2 to account
for 2 front wheels
2 Free-body diagram of cart.
Rolling resistance is negligible (The
coefficient of rolling resistance is
zero), so the force from the plane
acting on the wheel is concentrated
at a single point the point where
the wheel is tangent to the plane.
Equilibrium equations
Fx = 0: 2F
1
+ 2F
2
(981 N) sin = 0 (2)
F
y
= 0: 2G
1
+ 2G
2
(981 N) cos = 0 (3)
M
A
= 0: [(981 N) sin ](200 mm) [(981 N) cos ](500 mm)
+ (2G
2
)(500 mm + 500 mm) = 0 (4)

3
+
+
G
200 mm
500 mm
500 mm
A
+
2G
1
2G
2
2F
1
2F
2

1447
7.6 Journal Bearings Example 8, page 3 of 5
P
1
Contact point
between axle
and wheel
Axle (does not
rotate)
P
1
P
2
New
contact
point
Counterclockwise
rotation of wheel
4
Relative position of wheel and axle when cart is not
moving (We suppose that the cart is held in place by
a force that is parallel to the plane and is equal and
opposite to the component of the weight down the
plane). No friction force from the plane acts on the
wheel.
5
Wheel
Relative position of wheel and axle when
cart is moving (As the friction force from the
plane increases from zero, it causes the
wheel to rotate about the contact point P
1

initially. But this rotation causes the contact
point to shift up and to the right on the axle.
As the force increases further and the wheel
rotates more, the contact point moves further
right and up until slipping occurs).
Friction force from plane
acting on wheel

1448
7.6 Journal Bearings Example 8, page 4 of 5
P
2
N
R
f
Frictional force
opposes the
counterclockwise
rotation of the
wheel.
R is the resultant of the normal
force, N, and the frictional force, f,
from the axle acting on the wheel.
The line of action of R is tangent to
the circle of friction and passes
through the contact point P
2
.
6 Force from axle acting on wheel.
Circle of friction
(radius r
f
)
P
2
R
r
f
G
1
F
1
8 Free-body diagram of front wheel
r
f
We can save some work if we
use the fact that the wheel is a
two-force member. Thus the
force R from the axle must
have the same line of action as
the resultant of the forces F
1

and G
1
from the inclined plane.
That is, both lines of action
pass through P
2
and the point
of contact, A, with the inclined
plane. It follows from
geometry that F
1
and G
1
must
be related by the angle :
tan F
1
/G
1
(5)
7
9
A
Resultant of
forces F
1
and
G
1
from the
plane acting
on the wheel

1449
7.6 Journal Bearings Example 8, page 5 of 5
10 From geometry,
= sin
-1
(r
f
/150)
1.1962 mm, by Eq. 1
= 0.4569
Using this result in Eq. 5 gives
tan = F
1
/G
1
(Eq. 5 repeated)
or
tan 0.4569 = F
1
/G
1
(6)
Because the geometry is the same for both wheels,
tan 0.4569 = F
2
/G
2
(7)
Solving Eqs. 2-4, 6 and 7 gives
F
1
= 1.96 N, G
1
= 246 N
F
2
= 1.95 N, G
2
= 244 N
= 0.457 Ans.
P
2
R
r
f
300/2 mm = 150 mm
A

1450

7.7 Rolling Resistance
1451
7.7 Rolling Resistance Procedures and Strategies, page 1 of 1
Procedures and Strategies for Solving Problems Involving
Rolling Resistance
R (Resisting force
opposing the motion of the
wheel relative to the plane)
a
1. Locate the point where the resisting force from the plane
acts on the wheel. This point lies on the circumference
of the wheel at a distance, measured parallel to the plane,
given by the coefficient of rolling resistance, "a". The
resisting force is oriented so that its component parallel
to the plane opposes the motion of the wheel relative to
the plane. The line of action of the resisting force will
pass through center of the wheel if and only if the line of
action of the resultant of the other forces acting on the
wheel passes through the center.
2. For later use in calculating moments, determine the
horizontal and vertical distances between the center of
the wheel and the point of application of the resisting
force.
3. Solve the remainder of the problem in the same way you
would solve any two-dimensional rigid-body equilibrium
problem, drawing free-body diagrams and writing
equations of equilibrium.
Direction of rotation of wheel
r
O
Rx
R
y
a
A
A
Direction of motion
of center of wheel
relative to plane
(Plane may be
moving)
O
r
2
a
2

1452
7.7 Rolling Resistance Problem Statement for Example 1
100 mm
250 mm
850 mm
Radius = 120 mm
P
y
Px
G
1. The fertilizer spreader and the fertilizer it contains have a combined
mass of 40 kg and a center of gravity located at point G. If the coefficient
of rolling resistance for the tires is 5 mm, determine the horizontal and
vertical components, Px and P
y
, of the force that must be applied to the
handle to move the spreader at a constant speed.

1453
7.7 Rolling Resistance Problem Statement for Example 2
P
2. The mass of the piano is 200 kg. The coefficient of rolling
resistance of the casters is 0.4 mm. Determine the value of the
horizontal force P required to move the piano at constant speed.
Make the simplifying assumption that the resisting forces are
the same at all four casters.
Radius of the
casters = 14 mm

1454
7.7 Rolling Resistance Problem Statement for Example 3
15
F
8 in.
3. If the coefficient of rolling resistance is 0.3 in., determine the
magnitude of the horizontal force F required to push the 300-lb
drum up the inclined plane at constant velocity.

1455
7.7 Rolling Resistance Problem Statement for Example 4
4. A 100-lb steel-rim wheel of 24-in. diameter rolls
at constant velocity down the inclined plane. If the
coefficient of rolling resistance is 0.08 in.,
determine the angle .

1456
7.7 Rolling Resistance Problem Statement for Example 5
80 mm
100 kg
P
5. If the coefficient of rolling resistance at the top of the
cylinder is 0.4 mm and at bottom of the cylinder is 0.8 mm,
determine the horizontal force P required to start the block
moving to the left. The weight of the cylinder is negligible
compared to the weight of the block.

1457
7.7 Rolling Resistance Problem Statement for Example 6
4 ft
24 ft
Rope
100,000 lb
Diameter of
logs = 1 ft.
Force of men
(and perhaps
oxen)
6. The ancient Britons who constructed the prehistoric monument
Stonehenge moved massive stones over twenty miles from a quarry to
the monument site. One possible way that they might have done this is
to roll the stones over logs laid on the ground. Assuming that the
coefficient of rolling resistance for the top of a log is 0.01 ft and the
coefficient is 0.08 ft for the bottom, estimate the horizontal force
required to move the typical stone shown below. Make the simplifying
assumption that the resisting forces are the same for each log.
F

1458
7.7 Rolling Resistance Example 1, page 1 of 4
100 mm
250 mm
850 mm
Radius = 120 mm
P
y
Px
G
1. The fertilizer spreader and the fertilizer it contains have a combined
mass of 40 kg and a center of gravity located at point G. If the coefficient
of rolling resistance for the tires is 5 mm, determine the horizontal and
vertical components, Px and P
y
, of the force that must be applied to the
handle to move the spreader at a constant speed.

1459
7.7 Rolling Resistance Example 1, page 2 of 4
Rotation
of wheel
Direction of motion
of center of wheel
A
5 mm ( = coefficient of rolling resistance)
B
Locate the point where the resisting force
from the ground acts on the wheel.
As the wheel rolls to
the left, the ground
exerts a force, R,
opposing the motion.
R acts at a point B on the
circumference 5 mm
from a vertical line
through the center A of
the wheel.
1
2
3
R
A
B
5 mm
100 mm
(100 mm)
2
(5 mm)
2
= 99.875 mm
4 For later use in calculating moments, find the
vertical distance between A and B.

1460
7.7 Rolling Resistance Example 1, page 3 of 4
100 mm
250 mm
P
y
Px
G
5 mm
2Rx
2R
y
Components of force
from ground acting on
two wheels
A
B
Weight = 40 kg 9.81 m/s
2
= 392.4 N
Free-body diagram of spreader. 5
Equilibrium equations
Fx = 0: 2Rx Px = 0 (1)
F
y
= 0: 2R
y
+ P
y
392.4 N = 0 (2)
M
B
= 0 : Px(850 mm + 99.875 mm)
+ P
y
(250 mm + 5 mm)
(392.4 N)(100 mm + 5 mm) = 0 (3)
Three equations, four unknowns; an additional free
body is needed.
6
+
+
+ 850 mm
99.875 mm

1461
7.7 Rolling Resistance Example 1, page 4 of 4
A
B
5 mm
100 mm
A
y
Ax
Rx
R
y
Forces from spreader
acting on axle
7 Free body diagram of a wheel and axle Moment equation for wheel
M
A
= 0: Rx(99.875 mm) R
y
(5 mm) = 0 (4)
Solving equations 1-4 simultaneously gives
Rx = 7.1 N
R
y
= 141.9 N
Px = 14.2 N Ans.
P
y
= 108.7 N Ans.
8
+
99.875 mm

1462
7.7 Rolling Resistance Example 2, page 1 of 4
P
2. The mass of the piano is 200 kg. The coefficient of rolling
resistance of the casters is 0.4 mm. Determine the value of the
horizontal force P required to move the piano at constant speed.
Make the simplifying assumption that the resisting forces are
the same at all four casters.
Radius of the
casters = 14 mm

1463
7.7 Rolling Resistance Example 2, page 2 of 4
Rotation
of wheel
Direction of motion
of center of wheel
A
0.2 mm ( = coefficient
of rolling resistance)
B
Locate the point where the resisting force
from the floor acts on the wheel.
As the wheel rolls to the left,
the floor exerts a force, R,
opposing the motion.
R acts at a point B on the
circumference 0.2 mm from a
vertical line through the center
A of the wheel.
1
2
3
A
B
0.2 mm
14 mm
(14 mm)
2
(0.2 mm)
2
= 13.9986 mm
4 For later use in calculating
moments, find the vertical
distance between A and B.
R

1464
7.7 Rolling Resistance Example 2, page 3 of 4
Rx Rx
Rx Rx
R
y
R
y
R
y
R
y
Free-body diagram of piano
The resisting forces from floor
acting on caster are assumed to
be the same at each caster
Equilibrium equations.
Fx = 0: 4Rx P = 0 (1)
F
y
= 0: 4R
y
1962 N = 0 (2)
Two equations, three unknowns; an
additional free body is needed.
6
+
+
5
Weight = (200 kg)(9.81 m/s
2
) = 1962 N
P

1465
7.7 Rolling Resistance Example 2, page 4 of 4
Moment equation for caster 8
M
A
= 0 : Rx(13.9986 mm) R
y
(0.2 mm) = 0 (3)
Solving Eqs.1-3 simultaneously gives
Rx = 7.01 N
R
y
= 490.5 N
P = 28.0 N Ans.
+
A
B
0.2 mm
13.9986 mm
Rx
R
y
Forces from piano
acting on axle
Free-body diagram of a caster and axle 7
A
y
Ax

1466
7.7 Rolling Resistance Example 3, page 1 of 3
15
F
8 in.
3. If the coefficient of rolling resistance is 0.3 in., determine the
magnitude of the horizontal force F required to push the 300-lb
drum up the inclined plane at constant velocity.
15
A
B
R
0.3 in. ( = coefficient
of rolling resistance)
Rotation of
drum
Direction of
motion of
center of drum
Locate the point where the
resisting force from the inclined
plane acts on the drum.
As the wheel rolls up
the plane, the plane
exerts a force, R,
opposing the motion.
R acts at a point B on the circumference
0.3 in. from the center A of the drum.
Note that the 0.3 in. is measured parallel
to the inclined plane.
1
2
3

1467
7.7 Rolling Resistance Example 3, page 2 of 3
A
B
8 in.
0.3 in.
(8 in.)
2
(0.3 in.)
2
= 7.9944 in.
C
D
4 For use in calculating moments, find the
distance from A to B in the direction
perpendicular to the inclined plane.
F
8 in.
300 lb (Weight of drum)
7.9944 in.
A
B
C
D
Reference line
perpendicular to
inclined plane
Rx
R
y
y
x
Components
of force from
inclined plane
acting on
wheel
5 Free-body diagram of drum
Moment equilibrium equation 6
M
B
= 0: (300 cos lb)(0.3 in.)
+ (300 sin lb)(7.9944 in.)
(F cos )(7.9944 in.)
+ (F sin )(0.3 in) = 0 (1)
+
15
0.3 in.

1468
7.7 Rolling Resistance Example 3, page 3 of 3
A
B
C
D
90 15 = 75
Inclined
plane
= 90 75 = 15
7 Calculate
8
9 Substitute = 15 into Eq. 1:
(300 cos lb)(0.3 in.)
+ (300 sin lb)(7.9944 in.)
(F cos )(7.9944 in.)
+ (F sin )(0.3 in) = 0 (Eq. 1 repeated)
Solving gives
F = 92.6 lb Ans.
15

1469
7.7 Rolling Resistance Example 4, page 1 of 3
4. A 100-lb steel-rim wheel of 24-in. diameter rolls
at constant velocity down the inclined plane. If the
coefficient of rolling resistance is 0.08 in.,
determine the angle .
A
B
0.08 in. ( = coefficient
of rolling resistance)
Rotation of wheel
R
Direction of
motion of the
center of the
wheel
Locate the point where the resisting force
from the inclined plane acts as the wheel.
1
As the wheel rolls down the
plane, the plane exerts a force,
R, opposing the motion.
2
3
R acts at a point B on the circumference
0.08 in. from the center of the wheel,
measured parallel to the inclined plane.

1470
7.7 Rolling Resistance Example 4, page 2 of 3
A
B
0.08 in.
C
(12 in.)
2
(0.08 in.)
2
= 11.9997 in.
D
Radius = 24/2 in.

= 12 in.
For use in calculating moments, find the distance
from A to B in the direction perpendicular to the
inclined plane.
4
A
B
Rx
R
y
x
y
C
D
0.08 in.
Components of force
from inclined plane
acting on wheel
Free-body diagram of wheel 5
11.9997 in.
Equilibrium equations
Fx = 0: Rx (100 lb) sin = 0 (1)
F
y
= 0: R
y
(100 lb) cos = 0 (2)
M
A
= 0 : R
y
(0.08 in.) Rx(11.9997 in.) = 0 (3)
Solving gives
Rx = 0.667 lb
R
y
= 99.998 lb
= 0.382 (4)
+
+
+
6

1471
7.7 Rolling Resistance Example 4, page 3 of 3
A
C
D
Complement of
Geometry 7
But = 0.382, by Eq. 4 so,
= 0.382 Ans.
A
B
R
0.08 in.
Weight
12 in. = sin
-1 0.08 in.
12 in.
We could have saved same work by
noticing that the wheel is a two-force
body. Thus because the line of action
of the weight is a vertical line through
the center, the line of action of the
resisting force R must also be a vertical
line passing through the center. The
angle can be found from geometry, as
shown below.
= 0.382
8

1472
7.7 Rolling Resistance Example 5, page 1 of 4
80 mm
100 kg
P
5. If the coefficient of rolling resistance at the top of the
cylinder is 0.4 mm and at bottom of the cylinder is 0.8 mm,
determine the horizontal force P required to start the block
moving to the left. The weight of the cylinder is negligible
compared to the weight of the block.

1473
7.7 Rolling Resistance Example 5, page 2 of 4
100 kg P
0.4 mm (= coefficient of rolling resistance)
A
Rotation
of
wheel
0.8 mm (= coefficient of rolling resistance)
R
ground
Direction of
motion of center
of the wheel
relative to the
ground
Direction of motion
of center of the wheel
relative to the block
R
block
C
B
The resisting force from the ground opposes
the rolling of the cylinder on the ground.
3
The resisting force from the block opposes
the rolling of the cylinder on the block.
2
Locate the points where the resisting forces act on the cylinder. 1

1474
7.7 Rolling Resistance Example 5, page 3 of 4
A
C
B
D
E
80 mm
80 mm
0.8 mm
0.4 mm
(80 mm)
2
(0.8 mm)
2
= 79.9960 mm
(80 mm)
2
(0.4 mm)
2
= 79.9990 mm
For use in calculating moments, find the vertical distances
between the center of the wheel and the point of application of
the resisting forces.
4

1475
7.7 Rolling Resistance Example 5, page 4 of 4
0.4 mm
A
0.8 mm
R
ground-x
R
block-x
C
B
100 kg
F
C
R
ground-y
R
block-y
E
R
ground-x
R
ground-y
Weight = (100 kg)(9.81 m/s
2
) = 981 N
Free-body diagram of block and cylinder 5
Equilibrium equations
Fx = 0: R
ground-x
F = 0 (1)
F
y
= 0: R
ground-y
981 N = 0 (2)
Two equations, three unknowns: an additional
free-body is needed.
Moment equation
M
B
= 0: R
ground-x
(79.9990 mm + 79.9960 mm)
R
ground-y
(0.8 mm + 0.4 mm) = 0 (3)
Solving Eqs. 1-3 gives
R
groung-x
= 7.358 N
R
ground-y
= 981 N
F = 7.36 N Ans.
+
+
6
+
Free-body diagram of the cylinder 7
79.999 mm
79.996 mm

1476
7.7 Rolling Resistance Example 6, page 1 of 5
4 ft
24 ft
Rope
100,000 lb
Diameter of
logs = 1 ft.
Force of men
(and perhaps
oxen)
6. The ancient Britons who constructed the prehistoric monument
Stonehenge moved massive stones over twenty miles from a quarry to
the monument site. One possible way that they might have done this is
to roll the stones over logs laid on the ground. Assuming that the
coefficient of rolling resistance for the top of a log is 0.01 ft and the
coefficient is 0.08 ft for the bottom, estimate the horizontal force
required to move the typical stone shown below. Make the simplifying
assumption that the resisting forces are the same for each log.
F

1477
7.7 Rolling Resistance Example 6, page 2 of 5
Rotation
of log
A
0.08 ft
Coefficient of
rolling resistance
R
g
C
0.01 ft
B
Portion
of stone
Direction of
motion of stone
Coefficient of
rolling resistance
Locate the points where the resisting
forces act on the log.
2 As the log rolls on the
underside of the stone, the
stone exerts a force opposing
the rolling motion
Direction of motion of center
of log relative to ground
Direction of
motion of
center of log
relative to
stone
As the log rolls on the
ground, the stone exerts a
force opposing the rolling
motion.
3
1

1478
7.7 Rolling Resistance Example 6, page 3 of 5
A
C
B
0.08 ft
0.01 ft
0.5 ft
0.5 ft
(0.5 ft)
2
(0.01 ft)
2
= 0.4999 ft
(0.5 ft)
2
(0.08 ft)
2

= 0.4936 ft
For use in calculating moments, find the vertical distances from
the center to the points C and B where the resisting forces act.
4
Radius = (1 ft)/2 = 0.5 ft

1479
7.7 Rolling Resistance Example 6, page 4 of 5
100,000 lb
R
g-x
R
g-y
Free-body diagram of stone and logs
Equilibrium equations
Fx = 0: F 10R
g-x
= 0 (1)
F
y
= 0: 10R
g-y
100,000 lb = 0 (2)
Two equations, three unknowns: an additional free-body
is needed.
5
+
+
F
The same resisting force acts on each of the ten logs.

1480
7.7 Rolling Resistance Example 6, page 5 of 5
A
C
B
0.08 ft
0.01 ft
R
g-x
R
g-y
R
s-y
Rs-x
Free-body diagram of an individual wheel 6
Moment equilibrium
M
B
= 0: R
g-y
(0.01 ft + 0.08 ft) R
g-x
(0.4999 ft + 0.4936 ft) = 0 (3)
Solving Eqs. 1-3 gives
R
g-x
= 906 lb
R
g-y
= 10,000 lb
F = 9,060 lb Ans.
The force F required to move the 100,000-lb stone is large, even when the
ground is level. One scholar has estimated that as many as 600 men were
needed to move such a stone up one of the slopes lying between the quarry
and the monument site.
+
7
8
0.4999 ft
0.4936 ft
Force components
from the stone

1481

8. Internal Forces
1482

8.1 Internal Forces in Structural Members
1483
8.1 Internal Forces in Structural Members Procedures and Strategies, page 1 of 2
D
C
B
P
Q
A
D A
Dx
D
y
M
V
N
Procedures and Strategies for Solving Problems Involving
Internal Forces in Structural Members
1. Pass a section through the structural member at the point
where the internal forces are to be calculated.
2. Draw a free-body diagram of the portion of the member on the
side of the section that has the least number of loads acting on it,
and show the (unknown) internal forces and moments acting at
the section.
3. Write the equilibrium equations for the free body.


1484
8.1 Internal Forces in Structural Members Procedures and Strategies, page 2 of 2
D A
C
Q Dx
D
y
B
P
Bx
B
y
Dx
D
y
Cx
C
y
4. Solve for the forces from the support or from other
members by using the usual techniques for analyzing a
rigid body subject to a system of forces.
5. Substitute the results found in step 5 into the
equations of step 4, and solve for the internal forces.
Find Dx and D
y
by writing equilibrium
equations for these two free-body diagrams.

1485
8.1 Internal Forces in Structural Members Problem Statement for Example 1
D E
4 kN
4 m 4 m 2 m 2 m
1. Determine the normal force, shear force, and
moment at sections passing through a) B and b) D.
A
B C

1486
8.1 Internal Forces in Structural Members Problem Statement for Example 2
2. Determine the normal force, shear force, and moment at a
section passing through B.
6 ft 6 ft
300 lb/ft
C
B
A

1487
8.1 Internal Forces in Structural Members Problem Statement for Example 3
30 lb
70 lb
A
B
C
O
3. Determine the normal force, shear force, and moment
acting at a section passing through point B on the
quarter-circular rod shown.
4 ft
40

1488
8.1 Internal Forces in Structural Members Problem Statement for Example 4

2 ft


2 ft
A
B C
D
2 kip
2 kip
6 kipft (Torque applied at end)
4. Determine the internal torque at sections passing
through points a) B and b) D of the shaft.

1489
8.1 Internal Forces in Structural Members Problem Statement for Example 5
5. Determine the axial force, shear force, and moment at
sections passing through a) C and b) immediately to the left
of roller E, and c) immediately to the right of roller E.
5 m
F
4 kN
Hinge (pin connection)
5 m
E C D
4 m
A
B
2 m 1 m
5 kN m

1490
8.1 Internal Forces in Structural Members Problem Statement for Example 6
A
35 mm
6. Two wooden blocks have been glued together and a
compressive force of 80 N is applied by the clamp to press the
blocks together as the glue dries. Determine the normal force,
shear force, and moment at a vertical section through point A.

1491
8.1 Internal Forces in Structural Members Problem Statement for Example 7
100 lb
7. Determine the axial force, shear force, and moment at C.
5 ft 5 ft
A B
C
D
E

1492
8.1 Internal Forces in Structural Members Problem Statement for Example 8
A
C
2,000 lb
B
E
D
F G
H
6 ft
5 ft 5 ft 5 ft 5 ft
8. The spreader bar BCD is used to spread the load acting on the
2,000 lb beam EFGH while it is being lifted. Determine the normal
force, shear force, and moment at the midpoint C of the spreader bar.
Neglect the weight of the bar.

1493
8.1 Internal Forces in Structural Members Problem Statement for Example 9
9. The frame shown is pin-connected at D and rests on smooth
surfaces at A and G. Determine the normal force, shear force,
and moment acting at a section passing through point C.
100 kg
10 m
A
B
C
D
E
F
G
1 m 2 m 3 m 2 m 3 m 1 m

1494
8.1 Internal Forces in Structural Members Problem Statement for Example 10
10 ft
3 ft
10. In the floor-beam girder system shown, the four floor panels at the top
are simply supported at their ends by floor beams. The beams in turn
transmit forces to the girder ACI. Determine the axial force, shear force, and
moment in the girder at sections passing through a) point B and b) point J.



(side view)
Girder
(end view)
Floor beam
Floor panels
J
I
H G F E D
C B
A
2 kip
6 ft 10 ft 7 ft
2 ft 2 ft

1495
8.1 Internal Forces in Structural Members Example 1, page 1 of 4
D E
4 kN
4 m 4 m 2 m 2 m
1. Determine the normal force, shear force, and
moment at sections passing through a) B and b) D.
A
B C

1496
8.1 Internal Forces in Structural Members Example 1, page 2 of 4
B C E
A
B
2 m
V

B
M

B
N

B
2
Solving Eqs. 1, 2, and 3 gives

N

B
= 0 Ans.
V

B
= 4 kN Ans.

M

B
= 8 kNm Ans.
4
M

B
= 8 kNm
B
4 kN
2 m
A
V

B
= 4 kN
N

B
= 0
5
Pass a section through point B.
Free-body diagram of part of beam to left of B (This is a much
better choice of free body than the part of the beam to the right,
since we won't have to calculate the reactions at C and E.)
Free-body diagram showing correct senses of
internal forces and moment at B:
4 kN
4 kN
Equations of equilibrium for part of beam to left of B:
F

x = 0: N

B
= 0 (1)
F

y
= 0: V

B
4 kN = 0 (2)
M

B
= 0: (4 kN)(2 m) + M

B
= 0 (3)
3
+
+
+
Part a): Internal forces and moment at B
1
A

1497
8.1 Internal Forces in Structural Members Example 1, page 3 of 4
E
4 kN
Part b): Internal forces and moment at D
9
V

D M

D
N

D
7
Equations of equilibrium for part of beam to right of D:
F

x = 0: N

D
+ E

x = 0 (4)
F

y
= 0: V

D
E

y
= 0 (5)
M
D
= 0: M

D
+ E

y
(4 m) = 0 (6)
+
+
+
8
6
D
E
D
E

x
E

y
4 m
Pass a section through point D.
Free-body diagram of part of beam to right of D
Three equations but five unkowns.
Another free body is needed.
A
C

1498
8.1 Internal Forces in Structural Members Example 1, page 4 of 4
V

D
= 2 kN
M

D
= 8 kNm
N

D
= 0
13
12
E
D
2 kN
Using these results in Eqs. 4, 5, and 6 and solving gives

N

D
= 0 Ans.
V

D
= 2 kN Ans.
M

D
= 8 kNm Ans.
Free-body diagram showing correct senses
of internal forces and moment at D
Free-body diagram of entire beam 10
4 m 8 m
C

y E

y
E

x
D
E C
A
4 kN
11
+
+
Equilibrium equations for entire beam:
F

x = 0: E

x = 0 (7)

M

C
= 0: (4 kN)(4 m) + E

y
(8 m) = 0 (8)
Solving Eqs. 7 and 8 gives
E

x = 0 and E

y
= 2 kN


1499
8.1 Internal Forces in Structural Members Example 2, page 1 of 2
1
2
Equations of equilibrium for part of beam to right of B:
F

x = 0: N

B
= 0 (1)
Therefore
N

B
= 0 Ans.
F

y
= 0: V

B
+ C

y
300 lb/ft)(6 ft) = 0 (2)
M

B
= 0: M

B
(300 lb/ft)(6 ft)( )
+ C
y
(6 ft) = 0 (3)
Eqs. 2 and 3 involve three unknowns. An additional free-body
diagram is needed.
+
+
+
3
A
B
C
300 lb/ft
A
C
B
C
B
300 lb/ft
C

y
N

B
V

B
M

B
300 lb/ft
2. Determine the normal force, shear force, and moment at a
section passing through B.
Pass a section through point B.
Free-body diagram of part of
beam to right of B
Note that the resultant of the entire distributed
load, (300 lb/ft) (12 ft) = 3,600 lb, does not
act at B. The entire distributed load acts on
the entire 12-ft span; the free body shown
below has 300 lb/ft acting over only a 6-ft
span, and the resultant of this distributed load
acts halfway between B and C.
4
2
6 ft
6 ft 6 ft
6 ft

1500
8.1 Internal Forces in Structural Members Example 2, page 2 of 2
Equation of equilibrium for entire beam:
M

A
= 0: C

y
(12 ft) (300 lb/ft)(12 ft)( ) = 0 (4)
Solving Eq. 4 gives C

y
= 1800 lb.
Using C

y
= 1800 lb in Eqs. 2 and 3 and then solving gives

V

B
= 0 Ans.
M

B
= 5,400 lb ft Ans.

+
6
A
B
C
300 lb/ft
12 ft
C

y A

y
A

x
Free-body diagram of entire beam.
12 ft
2
5

1501
8.1 Internal Forces in Structural Members Example 3, page 1 of 2
+
3
N

B
V

B
M

B
30 lb
70 lb
A
B
C
O
3. Determine the normal force, shear force, and moment
acting at a section passing through point B on the
quarter-circular rod shown.
B
A
C
70 lb
30 lb
Pass a section through point B 1
C
30 lb
70 lb
B
40
O

4 ft
40
4 ft
Choosing point O for summing moments eliminates
both the 70-lb force and the shear V

B
from the moment
equation because their lines of action pass through O.
Free-body diagram of portion above B 2
Equilibrium equations for part of rod to right and above
section at B:
F

x = 0: N

B
sin V

B
cos 40 + 30 lb = 0 (1)
F

y
= 0: N

B
cos + V

B
sin 40 + 70 lb = 0 (2)
M

O
= 0: M

B
+ N

B
(4 ft) (30 lb)(4 ft) = 0 (3)
+
+

1502
8.1 Internal Forces in Structural Members Example 3, page 2 of 2
O
C
B
5 Substituting = 40 into Eqs. 1, 2, and 3 and solving
gives

N

B
= 72.9 lb Ans.
V

B
= .0 lb Ans.
M

B
= 171.6 lb Ans.

= 90 50 = 40
40
50
Geometry
4

1503
8.1 Internal Forces in Structural Members Example 4, page 1 of 6

2 ft


2 ft
A
B C
D
2 kip
2 kip
6 kipft (Torque applied at end)
4. Determine the internal torque at sections passing
through points a) B and b) D of the shaft.

1504
8.1 Internal Forces in Structural Members Example 4, page 2 of 6


2 ft


2 ft
A
B C
D
2 kip
2 kip
6 kipft (Torque applied at end)
Part a) point B
Pass a section through point B. 1

1505
8.1 Internal Forces in Structural Members Example 4, page 3 of 6

2 ft


2 ft
B
C
D
2 kip
2 kip
6 kipft (Torque applied at end)
2
x
M

B
Free-body diagram of part to right of B. This is a better choice
than using the part to the left because the reaction at the support
would have to be calculated, if we used the part to the left.
Equilibrium equation for part of shaft:
M

x = 0: M

B
(2 kip)(2 ft) (2 kip)(2 ft) + 6 kipft = 0
Solving gives

M

B
= 2 kipft Ans.
3
+
Torque from part
of the shaft to the
left of point B
acting on the part
to the right.
The choice of
sense of the torque
is arbitrary.

1506
8.1 Internal Forces in Structural Members Example 4, page 4 of 6

2 ft


2 ft
B
C
D
2 kip
2 kip
6 kipft (Torque applied at end)
x
M

B
= 2 kipft
Free-body diagram of part to
right of B showing correct sense
of torque at B
4

1507
8.1 Internal Forces in Structural Members Example 4, page 5 of 6


2 ft


2 ft
A
B C
D
2 kip
2 kip
6 kipft
Part b) point D
Pass a section through point D. 5

1508
8.1 Internal Forces in Structural Members Example 4, page 6 of 6
D
6 kipft
x
M

D
Free-body diagram of part to right of D 6
Equilibrium equation for part of shaft:
Mx = 0: M

D
+ 6 kipft = 0
Solving gives

M

D
= 6 kipft Ans.
+
7

1509
8.1 Internal Forces in Structural Members Example 5, page 1 of 6
Part a): point C
1
5 m 5 m
5. Determine the axial force, shear force, and moment at
sections passing through a) C and b) immediately to the left
of roller E, and c) immediately to the right of roller E.
Pass a section through point C.
4 m
F
Hinge
5 m
F
4 kN
Hinge (pin connection)
5 m
E C D
4 m
A
B
2 m 1 m
1 m 2 m
5 kN m
5 kN m
A
B
C
D E
4 kN

1510
8.1 Internal Forces in Structural Members Example 5, page 2 of 6
4
Equilibrium equations for part of beam:
F

x = 0: N

C
= 0 (1)

Therefore,
N

C
= 0 Ans.
F

y
= 0: V

C
+ E
y
4 kN = 0 (2)

M

C
= 0: M

C
+ E
y
(1 m + 5 m) (4 kN)(1 m + 5 m +5 m) = 0 (3)
+
+
+
3
C
5 m 5 m
E
D
Hinge
4 kN
1 m
V

C
M

C
N

C
Three equations but four unknowns so an additional equation is needed.
E
y
Free-body diagram of part of beam to right of section. This is a
better choice than the part to the left because only one
unknown reaction (the vertical force at E) occurs on the right
while three (two force components and a moment) occur at A.
2

1511
8.1 Internal Forces in Structural Members Example 5, page 3 of 6
A
B
C
D
E
4 kN
5
5 m 5 m
6
4 m
F
Hinge
1 m 2 m
To calculate the reaction at E, pass a
section immediately to the left of hinge D.
5 m
E
M

D
= 0
N

D
5 m
V

D
D
Hinge
4 kN
Free-body diagram of part of beam to right
of section
The moment is zero at a hinge
(a "hinge" is just another
name for a pin connection
between two parts of the
beam; no moment is
transmitted at a pin
connection). We will make
use of this fact by summing
moments about D.
7
E
y
5 kN m

1512
8.1 Internal Forces in Structural Members Example 5, page 4 of 6
9
Equilibrium equation for part of beam:
M

D
= 0: E
y
(5 m) 4(5 m +5 m) = 0 (4)
Note that V

D
does not appear in this equation because
its moment arm about D is infinitesimal.
Solving Eq. 4 gives
E
y
= 8 kN
Substituting E
y
= 8 kN in Eqs. 2 and 3 and solving
gives
V

C
= 4 kN Ans.
M

C
= 4 kNm Ans.
+
8
C
5 m 5 m
E
D
Hinge
4 kN
1 m
V

C
= 4kN
M

C
= 4 kNm
N

C
= 0
Free-body diagram of part of beam
showing correct senses of forces and
moment at C
E
y
= 8 kN

1513
8.1 Internal Forces in Structural Members Example 5, page 5 of 6
10
5 m 5 m
11
4 m
F
Hinge
1 m 2 m
Pass a section through the beam
immediately to the left of E.
5 m
E
M

E
N

E
V

E
4 kN
Free-body diagram of part of
beam to right of section
12
E
y
= 8 kN
Part b): point immediately to left of E.
Equilibrium equations for part to right of section:
F

x = 0: N

E
= 0 (5)

Therefore, N

E
= 0 Ans.
F

y
= 0: V

E
+ 8kN 4 kN = 0 (6)
Solving gives
V

E
= 4 kN Ans.

M

E
= 0: M

E
(4 kN)(5 m) = 0 (7)
Solving gives
M

E
= 20 kNm Ans.

+
+
+
Free-body diagram of part of beam showing correct
senses of forces and moment at section to left of E
5 m
E
y
= 8 kN
E
4 kN
V

E
= 4 kN
M

E
= 20 kNm
N

E
= 0
13
5 kN m
A
B
C
D
E
4 kN

1514
8.1 Internal Forces in Structural Members Example 5, page 6 of 6
14
5 m 5 m
15
4 m
F
Hinge
1 m 2 m
Pass a section through the beam
immediately to the right of E.
M

E
'
N

E
'
V

E
'
4 kN
Free-body diagram of part of
beam to right of section
16
Part c): point immediately to right of roller E.
Equilibrium equations for part to right of section:
F

x = 0: N

E
' = 0 (8)

Therefore, N

E
' = 0 Ans.
F

y
= 0: V

E
' 4 kN = 0 (9)
Solving gives
V

E
' = 4 kN Ans.

M

E
= 0: M

E
' (4 kN)(5 m) = 0 (10)
Solving gives
M

E
' = 20 kNm Ans.

+
+
+
Free-body diagram of part of beam showing correct
senses of forces and moment at section to right of E
4 kN
V

E
' = 4 kN
M

E
' = 20 kNm
N

E
' = 0
17
5 m
5 m
5 kN m
A
B
C
D E
4 kN

1515
8.1 Internal Forces in Structural Members Example 6, page 1 of 2
35 mm
A
A
35 mm
6. Two wooden blocks have been glued together and a
compressive force of 80 N is applied by the clamp to press the
blocks together as the glue dries. Determine the normal force,
shear force, and moment at a vertical section through point A.
Pass a section through point A. 1

1516
8.1 Internal Forces in Structural Members Example 6, page 2 of 2
3
Equilibrium equations for part of clamp:
F

x = 0: N

A
+ 80 N = 0 (1)
F

y
= 0: V

A
= 0 (2)

M

A
= 0: M

A
+ (80 N)(35 mm) = 0 (3)
Solving gives
N

A
= 80 N Ans.
V

A
= 0 Ans.
M

A
= 2800 Nmm = 2.8 Nm Ans.
+
+
+
4
V

A
M

A
N

A
The force from the block points towards
the clamp because the block is in
compression.
35 mm
A
Free-body diagram of part of clamp to right of
section. The block is not included in the free body.
2
80 N

1517
8.1 Internal Forces in Structural Members Example 7, page 1 of 3
A B
C
D
E
100 lb
1
7. Determine the axial force, shear force, and moment at C.
Pass a section through point C.
5 ft 5 ft
5 ft 5 ft
A
C
B
D
E
100 lb

1518
8.1 Internal Forces in Structural Members Example 7, page 2 of 3
5 ft 5 ft
Free-body diagram of part of member
above section at C
M

C
N

C
V

C
Equations of equilibrium for part of member:
F

x = 0: V

C
= 0 (1)
Therefore V

C
= 0 Ans.
F

y
= 0: N

C
+ F

DB
100 lb = 0 (2)
M

C
= 0: M

C
+ F

DB
(5ft) (100 lb)(5 ft +5 ft) = 0 (3) +
+
4
+
F

DB
3
DB is a two-force member so the line of
action of F

DB
is known to be vertical.
Four unknowns but only three equations so an
additional equation is needed.
5
C
D
E
100 lb
2

1519
8.1 Internal Forces in Structural Members Example 7, page 3 of 3
100 lb
6
5 ft
Free-body diagram of entire structure
Equations of equilibrium for entire structure:
M

A
= 0: F

DB
(5ft) (100 lb)(5 ft +5 ft) = 0 (4)
Solving gives

F

DB
= 200 lb
Substituting F

DB
= 200 lb in Eqs. 2 and 3 and solving
gives

N

C
= 100 lb Ans.

M

C
= 0 Ans.
+
8
F

DB
7 Because DB is a
two-force member, the
line of action of F

DB
is
known to be vertical.
B
A
C
D
5 ft
E
A

x
A

y

1520
8.1 Internal Forces in Structural Members Example 8, page 1 of 3
C
A
2,000 lb
B
E
D
F G
H
6 ft
5 ft 5 ft 5 ft 5 ft
8. The spreader bar BCD is used to spread the load acting on the
2,000 lb beam EFGH while it is being lifted. Determine the normal
force, shear force, and moment at the midpoint C of the spreader bar.
Neglect the weight of the bar.
Pass a section through point C. 1
H
G F
D
E
B
2,000 lb
A
C

1521
8.1 Internal Forces in Structural Members Example 8, page 2 of 3
E
F G
H
5 ft 5 ft 5 ft 5 ft
C
B
2 Free-body diagram
of part of spreader
bar to left of C
T

BA
and T

BF
are
the tensions in
the cables at B.
3
V

C
N

C
M

C

5 ft
T

BA
T

BF
Equilibrium equations for part of spreader bar to left of C:
F

x = 0: N

C
+ T

BA
cos = 0 (1)
F

y
= 0: T

BA
sin T

BF
V

C
= 0 (2)

M

C
= 0: T

BF
(5 ft) T

BA
(sin )(5 ft) + M

C
= 0 (3)
+
+
+
4
Three equations but five unknowns, so at least one more free
body must be used.
5
T

BF
T

DG
Free-body diagram of beam 6
2,000 lb
Equilibrium equations for beam:
M

G
= 0: T

BF
(5 ft+5 ft) + (2,000 lb)(5 ft) = 0 (4)
Solving gives
T

BF
= 1,000 lb
Another free body is needed, if we are to calculate the
value of T

BA
in Eqs. 1, 2, and 3.
7
+

1522
8.1 Internal Forces in Structural Members Example 8, page 3 of 3
Free-body diagram of
connection A
8
T

BA T

DA
Equilibrium equations for connection A:
F

x = 0: T

BA
cos + T

DA
cos = 0 (5)
F

y
= 0: 2,000 lb T

BA
sin T

DA
sin = 0 (6)
+
+
9
Geometry
10
Substituting = 50.19 in Eqs. 5 and 6 and solving gives

T

BA
= T

DA
= 1,302 lb
Substituting T

BA
= 1,302 lb and T

BF
= 1,000 lb into Eqs.
1, 2, and 3 and solving gives

N

C
= 833 lb Ans.
V

C
= 0 Ans.
M

C
= 0 Ans.
11


2,000 lb
A
A
B
5 ft
6 ft
C
= tan
-1
( ) = 50.19
6 ft
5 ft

1523
8.1 Internal Forces in Structural Members Example 9, page 1 of 5
Pass a section through C. 1
A
B
G
10 m
F
100 kg
E
C
D
1 m 3 m 2 m 3 m 2 m 1 m
G
F
E
D
C
B
A
10 m
100 kg
9. The frame shown is pin-connected at D and rests on smooth
surfaces at A and G. Determine the normal force, shear force,
and moment acting at a section passing through point C.

1524
8.1 Internal Forces in Structural Members Example 9, page 2 of 5
2 Free-body diagram of part CBA
Since the floor is smooth,
only a normal force is
present; friction is absent.
3
N

C
Equilibrium equations for part ABC:
F

x = 0: T + N

C
cos + V

C
cos = 0 (1)
F

y
= 0: F

A
+ N

C
sin V

C
sin = 0 (2)

M

A
= 0: T(L

AB
) V

C
( L

AC
) + M

C
= 0 (3)
Three equations and five unknowns, so at least one
more free body is needed. But first, the angles
and and distances L

AB
and L

AC
must be determined.
+
+
+
4

L

AC

A
B
C
T
L

AB
M

C
V

C
F

A
1 m2 m

1525
8.1 Internal Forces in Structural Members Example 9, page 3 of 5
D
B
A
C

L

AC
10 m
1 m2 m 3 m
L

AB
Geometry 5
B
A
C
D
10 m
F
G
E
Weight =
(100 kg)(9.81 m/s
2
)
= 981 N
F

A
F

G
8 m 4 m
Free-body diagram of entire frame. This free
body will enable us to calculate F
A
.
6
+
Equilibrium equation for entire frame:
M

G
= 0: F

A
(8 m + 4 m) + (981 N)(4 m) = 0 (4)
Solving this equation gives

F

A
= 327.0 N
One more free body is needed, since we now have four
equations but five unknowns.
7
= tan
-1
(
10 m
1 m + 2 m + 3 m
) = 59.04
L

AC
=
(1 m + 2 m)
cos
= 5.831 m
= 90 = 30.96
L
AB
= (1 m) tan = 1.667 m

1526
8.1 Internal Forces in Structural Members Example 9, page 4 of 5
B
A
D
10 m
F

A
= 327.0 N
6 m
Free-body diagram of member
ABCD. This free body will give
us an equation relating F

A
and T.
8
+
Equilibrium equations for member ABCD:
M

D
= 0: (327.0 N)(6 m) + T(10 m 1.667 m) = 0 (5)
Solving gives

T = 235.4 N
10
L

AB
= 1.667 m
T
D

y
D
x
Forces from the pin
connection at D
9

1527
8.1 Internal Forces in Structural Members Example 9, page 5 of 5
A
C
F

A
= 327 N
Free-body diagram showing correct sense of
internal moment and forces at C
Substituting

T = 235.4 N
F

A
= 327.0 N
= 59.04
= 30.96
L

AB
= 1.667 m

L

AC
= 5.831 m
into Eqs. 1, 2, and 3 and solving gives

N

C
= 402 N Ans.
V

C
= 33.6 N Ans
M

C
= 40.9 Nm Ans
11
N

C
= 402 N
M

C
= 40.9 Nm
V

C
= 33.6 N
12

1528
8.1 Internal Forces in Structural Members Example 10, page 1 of 8
10 ft
3 ft
10. In the floor-beam girder system shown, the four floor panels at the top
are simply supported at their ends by floor beams. The beams in turn
transmit forces to the girder ACI. Determine the axial force, shear force, and
moment in the girder at sections passing through a) point B and b) point J.



(side view)
Girder
(end view)
Floor beam
Floor panels
J
I
H G F E D
C B
A
2 kip
6 ft 10 ft 7 ft
2 ft 2 ft

1529
8.1 Internal Forces in Structural Members Example 10, page 2 of 8
2 kip
A
B C
D E F G H
I
J
Floor panels
Floor beam
(end view)
Girder
(side view)



Pass a section through the girder at B. 1
Part a) point B

1530
8.1 Internal Forces in Structural Members Example 10, page 3 of 8
A
B
D E
2
F
13 ft 7 ft
N

B
M

B
V

B
F

A
F

F
Free-body diagram of part of structure to left
of point B, including floor panels DE and EF
Force from floor beam F acting
on floor panel EF ("simply
supported" so no moment acts
at F, only a force)
3
+
+
4 Equilibrium equations for part of structure:
F

x = 0: N

B
= 0 (1)
F

y
= 0: F

A
V

B
+ F

F
= 0 (2)
M

B
= 0: F

A
(13 ft) + M

B
+ F

F
(7 ft) = 0 (3)
+
Three equations but five unknowns so at least one
more free body is needed.
5

1531
8.1 Internal Forces in Structural Members Example 10, page 4 of 8
E
6
F
F

E
F

F
Free-body diagram of floor panel EF:
Obviously, F

F
= 0 (Just
consider the sum of moments
about E).
7
Free-body diagram of entire structure (This free body will enable us to calculate F

A
): 8
A
D
B
E F G
I
H
2 kip
C
F

A
C

y
C

x
10 ft 7 ft 10 ft 6 ft
3 ft
2 ft 2 ft

1532
8.1 Internal Forces in Structural Members Example 10, page 5 of 8
A
B
D E
10
F
13 ft 7 ft
N

B
= 0
M

B
= 6.93 kipft
V

B
= 0.53 kip
F

A
F

F
= 0
Free-body diagram showing correct senses of
internal reactions at B
+
9 Equilibrium equation for entire structure:
M

C
= 0: F

A
(10 ft + 3 ft + 7 ft + 10 ft) (2 kip)(6 ft + 2ft) = 0 (4)
Solving gives
F

A
= 0.5333 kip
Substituting F

A
= 0.5333 kip in Eqs. 2 and 3 and solving gives
V

B
= 0.53 kip Ans.
M

B
= 6.93 kipft Ans.

1533
8.1 Internal Forces in Structural Members Example 10, page 6 of 8
2 kip
A
B C
D E F G H
I
J
Pass a section through the girder at J. 11
Part b) Internal reactions at section J

1534
8.1 Internal Forces in Structural Members Example 10, page 7 of 8
12
N

J
M

J
V

J
F

H
Free-body diagram of part of structure to
right of section at J, including floor beam H
Force from panel GH
acting on floor beam H
13
+
+
14 Equilibrium equations for part of structure:
F

x = 0: N

J
= 0 (5)
Therefore
N

J
= 0 Ans.
F

y
= 0: V

J
F

H
= 0 (6)
M

J
= 0: M

J
F

H
(4 ft) = 0 (7)
+
Three equations but four unknowns so another
free body is needed.
15
I
J
H
4 ft

1535
8.1 Internal Forces in Structural Members Example 10, page 8 of 8
G
16
H
F

G
F

H
Free-body diagram of floor panel GH.
This free body will enable us to
calculate F

H
.
2 kip
8 ft 2 ft
Equilibrium equation for floor panel GH:
M

G
= 0: (2 kip)(8 ft) + F

H
(8 ft + 2 ft) = 0
Solving gives
F

H
= 1.60 kip
Substituting F

H
= 1.60 kip in Eqs. 6 and 7 and solving gives
V

J
= 1.60 kip Ans.
M

J
= 6.40 kipft Ans.
17
+
18 Free-body diagram of part of beam to right
of J showing internal forces and moment
with correct senses.
V

J
= 1.60 kip
4 ft
M

J
= 6.40 kipft
N

J
= 0
J
F

H
= 1.6 kip
H
I

1536

8.2 Shear and Bending-Moment Diagrams: Equation Form
1537
8.2 Shear and Bending-Moment Diagrams: Equation Form Procedures and Strategies, page 1 of 2
M
P
x
x
x
P
x
R
A
R
B
A
B
A
B
A
R
A
V
M
F
y
= 0
M = 0
w
w
Procedures and Strategies for Solving Problems Involving
Constructing Shear and Bending-Moment Diagrams by Use
of Equations
1. Draw a free-body diagram of the beam and solve for the
reactions.
2. Introduce a coordinate x with origin at the left end of the beam.
(x will locate the position of various sections passed through the
beam, as described in the next steps).
3. Pass a section through the beam at a point between the left end
and the first "critical point" (a point at which a concentrated
force or couple moment acts or at which a distributed load
begins or ends).
4. Draw a free-body diagram of the part of the beam to the left of
the section and show the shear V acting downward and moment
M acting counterclockwise on the right end.
5. Write and solve equilibrium equations to find V and M as
functions of x.
6. Pass a new section through the beam at a point between the
current critical point and the next critical point.
7. Repeat steps 4-6 until the right end of the beam is reached.
8. Plot V vs. x and M vs. x by using the equations you have
derived for V and M.
P
x
R
A
A
F
y
= 0
M = 0
V

1538
8.2 Shear and Bending-Moment Diagrams: Equation Form Procedures and Strategies, page 2 of 2
P
x
R
A
A
V
M
F
y
= 0
M = 0
w
Note: By using free-body diagrams that grow
successively longer with each new section, your force
and moment equations change only by the addition of
one new term. This property helps you avoid errors in
writing the equations.

1539
8.2 Shear and Bending-Moment Diagrams: Equation Form Problem Statement for Example 1
3 ft 5 ft 7 ft
x
A B
9 kip 6 kip
1. Express the shear V and bending moment M as functions of x, the distance from
the left end of the beam to an arbitrary point on the beam. Plot V and M vs. x.

1540
8.2 Shear and Bending-Moment Diagrams: Equation Form Problem Statement for Example 2
14 m
x
A
2 N/m
20 N m
2. Express the shear V and bending moment M as functions of x, the distance from
the left end of the beam to an arbitrary point on the beam. Plot V and M vs. x.

1541
8.2 Shear and Bending-Moment Diagrams: Equation Form Problem Statement for Example 3
x
A
B
10 lb/ft
3 ft 10 ft 7 ft
3. Express the shear V and bending moment M as functions of x, the distance from
the left end of the beam to an arbitrary point on the beam. Plot V and M vs. x.

1542
8.2 Shear and Bending-Moment Diagrams: Equation Form Problem Statement for Example 4
4. Express the shear V and bending moment M as functions of x, the distance from
the left end of the beam to an arbitrary point on the beam. Plot V and M vs. x.
3 ft 5 ft 7 ft
x
A B
27 kip ft
18 kip ft

1543
8.2 Shear and Bending-Moment Diagrams: Equation Form Problem Statement for Example 5
5. Express the shear V and bending moment M in the horizontal portion ACDB of
the beam as functions of x, the distance from the left end of the beam to an
arbitrary point on the beam. Plot V and M versus x.
x
A B
4 kip
2 ft 2 ft 4 ft 2 ft 2 ft
4 kip
C D

1544
8.2 Shear and Bending-Moment Diagrams: Equation Form Problem Statement for Example 6
x
A
B
4 kN
C
Hinge
2 m 2 m 2 m 2 m
8 kN
6. Express the shear V and bending moment M as functions of x, the distance from
the left end of the beam to an arbitrary point on the beam. Plot V and M vs. x.

1545
8.2 Shear and Bending-Moment Diagrams: Equation Form Problem Statement for Example 7
x
A
B
20 kip
5 ft 5 ft 8 ft
Hinge
4 kip/ft
7. Express the shear V and bending moment M as functions of x, the distance from
the left end of the beam to an arbitrary point on the beam. Plot V and M vs. x.

1546
8.2 Shear and Bending-Moment Diagrams: Equation Form Problem Statement for Example 8
x
A
B
6 m 6 m
4 kN/m
2 kN/m
8. Express the shear V and bending moment M as functions of x, the distance from
the left end of the beam to an arbitrary point on the beam. Plot V and M vs. x.

1547
8.2 Shear and Bending-Moment Diagrams: Equation Form Example 1, page 1 of 6
3 ft 5 ft 7 ft
x
A B
9 kip 6 kip
7 ft
F

y
= 0: R

A
9 kip 6 kip + R

B
= 0
M

A
= 0: (9 kip)(3 ft) (6 kip)(3 ft + 5 ft) + R

B
(3 ft + 5 ft + 7 ft) = 0
Solving gives
R

A
= 10 kip and R

B
= 5 kip
5 ft
9 kip
Draw a free-body diagram and find the reactions.
+
3 ft R

A
+
1
A
6 kip
R

B
1. Express the shear V and bending moment M as functions of x, the distance from
the left end of the beam to an arbitrary point on the beam. Plot V and M vs. x.

1548
8.2 Shear and Bending-Moment Diagrams: Equation Form Example 1, page 2 of 6
x
9 kip
R

A
= 10 kip
A
6 kip
R

B
= 5 kip
B
Pass a section through the beam at a point between the left end and the 9-kip force. 2
7 ft 3 ft 5 ft
0 < x < 3 ft
Draw a free-body diagram of the portion of the beam to the left
of the section and find V and M at the section.
3
R

A
= 10 kip
A
x
M(sign convention: M positive counterclockwise, on right end of section)
V (sign convention: V positive down, on right end of section)
F

y
= 0: 10 kip V = 0
M

x = 0: (10 kip)x + M = 0
Solving gives
V = 10 kip and M = 10x kip ft
valid for 0 < x < 3 ft.
+
+

1549
8.2 Shear and Bending-Moment Diagrams: Equation Form Example 1, page 3 of 6
x
9 kip
R

A
= 10 kip
A
6 kip
R

B
= 5 kip
B
Pass a section through the beam at a point between the 9-kip force and the 6-kip force.
7 ft 3 ft 5 ft
3 ft < x < 8 ft
Draw a free-body diagram of the portion of the beam to the left
of the section and find V and M at the section.
R

A
= 10 kip
A
V
x
M
F

y
= 0: 10 kip 9 kip V = 0
M

x = 0: (10 kip)x + (9 kip)(x 3 ft) + M = 0
Solving gives
V = 1 kip (3)
M = (x + 27) kip ft (4)
valid for 3 ft < x < 8 ft.
+
+
4
3 ft (x 3 ft)
5
9 kip

1550
8.2 Shear and Bending-Moment Diagrams: Equation Form Example 1, page 4 of 6
x
9 kip
R

A
= 10 kip
A
6 kip
R

B
= 5 kip
B
Pass a section through the beam at a point between the 6-kip force and the right end of the beam.
7 ft 3 ft 5 ft
8 ft < x < 15 ft
Draw a free-body diagram of the portion of the beam to the left
of the section and find V and M at the section.
R

A
= 10 kip
A
V
x
M
F

y
= 0: 10 kip 9 kip 6 kip V = 0
M

x = 0: (10 kip)x + (9 kip)(x 3 ft)
+ (6 kip)(x 8 ft) + M = 0
Solving gives
V = 5 kip (5)
M = ( 5x + 75) kip ft (6)
valid for 8 ft < x < 15 ft.
+
+
3 ft
(x 8 ft)
6
7
5 ft
(x 3 ft)
9 kip 6 kip

1551
8.2 Shear and Bending-Moment Diagrams: Equation Form Example 1, page 5 of 6
Collect the results from Eqs. 1-6:
0 < x < 3 ft V = 10 kip
M = 10x kip ft
3 ft < x < 8 ft V = 1 kip
M = (x + 27) kip ft
8 ft < x < 15 ft V = 5 kip
M = ( 5x + 75) kip ft
8
Ans.

1552
8.2 Shear and Bending-Moment Diagrams: Equation Form Example 1, page 6 of 6
6 kip 9 kip
B
R

B
= 5 kip R

A
= 10 kip
A
7 ft 3 ft 5 ft
Plot V and M versus x. 9
V
(kip)
M
(kip ft)
10
1
x
30
35
x
5

1553
8.2 Shear and Bending-Moment Diagrams: Equation Form Example 2, page 1 of 3
14 m
x
A
F

y
= 0: R

A
28 N = 0
M

A
= 0: M

A
(28 N)(7 m) + 20 N m = 0
Solving gives
R

A
= 28 N and M

A
= 176 N m
Draw a free-body diagram and find the reactions.
+
+
1
2 N/m
20 N m
14 m
A
20 N m
2 N/m
R

A
M

A
= 7 m
14 m
2 Resultant = (2 N/m)(14 m) = 28 N
A couple-moment reaction must always be
included at a built-in end of a beam.
2
2. Express the shear V and bending moment M as functions of x, the distance from
the left end of the beam to an arbitrary point on the beam. Plot V and M vs. x.

1554
8.2 Shear and Bending-Moment Diagrams: Equation Form Example 2, page 2 of 3
2 N/m
Resultant = (2 N/m)(x)
Pass a section through the beam at an arbitrary point (located by x)
x
3
0 < x < 14 m
M

A
= 176 N m
R

A
= 28 N
F

y
= 0: 28 N 2x V = 0
M

x = 0: 176 N m x + ( )(2 N/m)(x) + M = 0
Solving gives

V = ( 2x + 28) N Ans.
M = ( x
2
+ x 176) N m Ans.
valid for 0 < x < 14 m.
Draw a free-body diagram of the portion of the beam to the left
of the section and find V and M at the section.
+
+
x
4
M

A
= 176 N m
R

A
= 28 N
V
M
20 N m
A
A
x
2
2
x
14 m

1555
8.2 Shear and Bending-Moment Diagrams: Equation Form Example 2, page 3 of 3
14 m
2 N/m
Plot V and M versus x.
M

A
= 176 N m
R

A
= 28 N
5
M
(N m)
V
(N)
x
x
28
20
176
20 N m
A

1556
8.2 Shear and Bending-Moment Diagrams: Equation Form Example 3, page 1 of 6
F

y
= 0: R

A
100 lb + R

B
= 0
M

A
= 0: (100 lb)(7 ft + 5 ft) + R

B
(7 ft + 10 ft + 3 ft) = 0
Solving gives
R

A
= 40 lb and R

B
= 60 lb
Draw a free-body diagram and find the reactions.
+
+
1
= 5 ft
10 ft
2
Resultant = (10 lb/ft)(10 ft) = 100 lb
x
A
B
10 lb/ft
A
B
R

A
R

B
7 ft 3 ft
3 ft 10 ft 7 ft
10 ft
3. Express the shear V and bending moment M as functions of x, the distance from
the left end of the beam to an arbitrary point on the beam. Plot V and M vs. x.

1557
8.2 Shear and Bending-Moment Diagrams: Equation Form Example 3, page 2 of 6
Draw a free-body diagram and find the reactions.
7 ft 3 ft
A
B
10 ft
10 lb/ft
x
2 Pass a section through the beam at a point between the left end
of the beam and the beginning of the distributed load.
x
R

A
= 40 lb
F

y
= 0: 40 lb V = 0
M

x = 0: (40 lb)x + M = 0
Solving gives
V = 40 lb (1)
M = (40x) lb ft (2)
valid for 0 < x < 7 ft.
V
+
M
+
3
R

A
= 40 lb R

B
= 60 lb
0 < x < 7 ft

1558
8.2 Shear and Bending-Moment Diagrams: Equation Form Example 3, page 3 of 6
Draw a free-body diagram of the portion of the beam to
the left of the section and solve for V and M at the section.
x
R

A
= 40 lb
F

y
= 0: 40 lb (10 lb/ft)(x 7 ft) V = 0
M

x = 0: (40 lb)x + [(10 lb/ft)(x 7 ft)]
( ) + M = 0
Solving gives
V = ( 10x + 110) lb (3)
M= ( 5x
2
+ 110x 245) lb ft (4)

valid for 7 ft < x < 17 ft.
V
+
M
+
Pass a section through the beam at a point between the
beginning and end of the distributed load.
4
10 ft 7 ft
A
R

A
= 40 lb
x
B
3 ft
R

B
= 60 lb
10 lb/ft
5
2
x 7 ft
Resultant = (10 lb/ft)(x 7 ft)
7 ft < x < 17 ft
7 ft x 7 ft
2
x 7 ft

1559
8.2 Shear and Bending-Moment Diagrams: Equation Form Example 3, page 4 of 6
Draw a free-body diagram of the portion of the beam to the left
of the section and solve for V and M at the section.
x
R

A
= 40 lb
V
M
Pass a section through the beam at a point between the right end
of the distributed load and the right end of the beam.
10 ft 7 ft
A
R

A
= 40 lb
x
B
3 ft
R

B
= 60 lb
10 lb/ft
17 ft < x < 20 ft
7 ft
x 17 ft
6
10 ft
= 5 ft
2
10 ft Resultant = (10 lb/ft)(10 ft) = 100 lb
7

1560
8.2 Shear and Bending-Moment Diagrams: Equation Form Example 3, page 5 of 6
F

y
= 0: 40 lb 100 lb V = 0
M

A
= 0: (40 lb)x + (100 lb)[(x 17 ft) + 5 ft] + M = 0
Solving gives
V = 60 lb (5)
M = ( 60x + 1200) lb ft (6)

valid for 17 ft < x < 20 ft.
+
+
9 Collect the results from Eqs. 1-6:
0 < x < 7 ft V = 40 lb
M = 40x lb ft
7 ft < x < 17 ft V = ( 10x + 110) lb
M = ( 5x
2
+ 110x 245) lb ft
17 ft < x < 20 ft V = 60 lb
M = ( 60x + 1200) lb ft
Ans.
8

1561
8.2 Shear and Bending-Moment Diagrams: Equation Form Example 3, page 6 of 6
7 ft 3 ft
A
B
10 ft
10 lb/ft
Plot V and M versus x.
R

A
= 40 lb
R

B
= 60 lb
10
V
(lb)
M
(lb ft)
60
40
x
x
280
360
180
40
60

1562
8.2 Shear and Bending-Moment Diagrams: Equation Form Example 4, page 1 of 6
4. Express the shear V and bending moment M as functions of x, the distance from
the left end of the beam to an arbitrary point on the beam. Plot V and M vs. x.
3 ft 5 ft 7 ft
x
A B
7 ft
F

y
= 0: R

A
+ R

B
= 0
M

A
= 0: R

B
(3 ft + 5 ft + 7 ft) 27 kip ft 18 kip ft = 0
Solving gives
R

A
= 3 kip = 3 kip
R

B
= 3 kip
5 ft
Draw a free-body diagram and find the reactions.
+
3 ft R

A
+
1
A
R

B
27 kip ft
18 kip ft
27 kip ft
18 kip ft

1563
8.2 Shear and Bending-Moment Diagrams: Equation Form Example 4, page 2 of 6
x
R

A
= 3 kip
A
R

B
= 3 kip
B
Pass a section through the beam at a point between the left end and the 27 kip ft
moment couple.
2
0 < x < 3 ft
Draw a free-body diagram of the portion of the beam to the left
of the section and find V and M at the section.
3
A
V
x
M
F

y
= 0: 3 kip V = 0
M

x = 0: (3 kip)x + M = 0
Solving gives
V = 3 kip (1)
M = 3x kip ft (2)
valid for 0 < x < 3 ft.
+
+
27 kip ft
18 kip ft
R

A
= 3 kip
7 ft 5 ft 3 ft

1564
8.2 Shear and Bending-Moment Diagrams: Equation Form Example 4, page 3 of 6
x
R

A
= 3 kip
A
R

B
= 3 kip
B
Pass a section through the beam at a point between the
27 kip ft and 18 kip ft moment couples.
3 ft < x < 8 ft
Draw a free-body diagram of the portion of the beam to the left
of the section and find V and M at the section.
A
V
x
M
F

y
= 0: 3 kip V = 0
M

x = 0: (3 kip)x 27 kip ft + M = 0
Solving gives
V = 3 kip (3)
M = ( 3x + 27) kip ft (4)
valid for 3 ft < x < 8 ft.
+
+
27 kip ft
18 kip ft
R

A
= 3 kip
27 kip ft
4
5
3 ft x 3 ft
3 ft 7 ft 5 ft

1565
8.2 Shear and Bending-Moment Diagrams: Equation Form Example 4, page 4 of 6
x
R

A
= 3 kip
A
R

B
= 3 kip
B
Pass a section through the beam at a point between the 18 kip ft
moment couple and the right end of the beam.
7 ft 3 ft 5 ft
3 ft < x < 8 ft
Draw a free-body diagram of the portion of the beam to the left
of the section and find V and M at the section.
A
V
x
M
F

y
= 0: 3 kip V = 0
M

x = 0: (3 kip)x 27 kip ft 18 kip ft + M = 0
Solving gives
V = 3 kip (5)
M = ( 3x + 45) kip ft (6)
valid for 8 ft < x < 15 ft.
+
+
27 kip ft
18 kip ft
R

A
= 3 kip
27 kip ft
3 ft
6
7
18 kip ft
5 ft

1566
8.2 Shear and Bending-Moment Diagrams: Equation Form Example 4, page 5 of 6
8 Collect the results from Eqs. 1-6:
0 < x < 3 ft V = 3 kip
M = 3x kip ft
3 ft < x < 8 ft V = 3 kip
M = ( 3x + 27) kip ft
8 ft < x < 15 ft V = 3 kip
M = ( 3x + 45) kip ft
Ans.

1567
8.2 Shear and Bending-Moment Diagrams: Equation Form Example 4, page 6 of 6
Plot V and M versus x. 9
V
(kip)
M
(kip ft)
3
x
x
18
21
3
18 kip ft
7 ft 3 ft
A
R

A
= 3 kip
5 ft
27 kip ft
R

B
= 3 kip
B
9
3

1568
8.2 Shear and Bending-Moment Diagrams: Equation Form Example 5, page 1 of 6
5. Express the shear V and bending moment M in the horizontal portion ACDB of
the beam as functions of x, the distance from the left end of the beam to an
arbitrary point on the beam. Plot V and M versus x.
x
A B
4 kip
F

y
= 0: R

A
4 kip 4 kip + R

B
= 0
M

A
= 0: (4 kip)(2 ft) (4 kip)(10 ft)
+ R

B
(12 ft) = 0
Solving gives
R

A
= 4 kip
R

B
= 4 kip
Draw a free-body diagram and find the reactions.
+
R

A
+
1
R

B
2 ft 2 ft 4 ft 2 ft 2 ft
4 kip
2 ft
4 kip
2 ft 2 ft
A
4 ft 2 ft
B
4 kip
C D

1569
8.2 Shear and Bending-Moment Diagrams: Equation Form Example 5, page 2 of 6
Pass a section through the beam at a point between the left end and the attachment point for the first arm.
2 ft
4 kip
2 ft 2 ft
A
4 ft 2 ft
B
4 kip
x
Draw a free-body diagram of the
portion of the beam to the left of the
section and find V and M at the
section. Note carefully that the 4-kip
force on the left arm does not act on
this free body.
2
R

A
= 4 kip
A
3
x
V
M
0 < x < 4 ft
F

y
= 0: 4 kip V = 0
M

x = 0: (4 kip)x + M = 0
Solving gives
V = 4 kip (1)
M = 4x kip ft (2)
valid for 0 < x < 4 ft.
+
+
R

A
= 4 kip
R

B
= 4 kip

1570
8.2 Shear and Bending-Moment Diagrams: Equation Form Example 5, page 3 of 6
Pass a section through the beam at a point between the attachment points of the two arms.
R

A
= 4 kip
R

B
= 4 kip
2 ft
4 kip
2 ft 2 ft
A
4 ft 2 ft
B
4 kip
x
Draw a free-body diagram of the portion of the beam to
the left of the section and find V and M at the section.
V
M
4 ft < x < 8 ft
F

y
= 0: 4 kip 4 kip V = 0
M

x = 0: (4 kip)x + (4 kip)(x 4 ft + 2 ft) + M = 0
Solving gives
V = 0 (3)
M = 8 kip ft (4)
valid for 4 ft < x < 8 ft.
+
+
4
5
2 ft
4 kip
2 ft
R

A
= 4 kip
A
x
x 4 ft

1571
8.2 Shear and Bending-Moment Diagrams: Equation Form Example 5, page 4 of 6
Pass a section through the beam at a point between the point
of attachment of the right arm and the right end of the beam.
R

A
= 4 kip
R

B
= 4 kip
2 ft
4 kip
2 ft 2 ft
A
4 ft 2 ft
B
4 kip
x
Draw a free-body diagram of the portion of the beam to
the left of the section and find V and M at the section.
V
M
8 ft < x < 12 ft
F

y
= 0: 4 kip 4 kip 4 kip V = 0
M

x = 0: (4 kip)x + (4 kip)(x 2 ft)
+ (4 kip)(x 10 ft) + M = 0
Solving gives
V = 4 kip (5)
M = ( 4x + 48) kip ft (6)
valid for 8 ft < x < 12 ft.
+
+
2 ft
4 kip
2 ft
R

A
= 4 kip
A
x
6
7
4 ft
x 10 ft
4 kip
2 ft
Note that the 4-kip force on the right arm acts on the free
body.
8

1572
8.2 Shear and Bending-Moment Diagrams: Equation Form Example 5, page 5 of 6
Collect the results from Eqs. 1-6:
0 < x < 4 ft V = 4 kip
M = 4x kip ft
4 ft < x < 8 ft V = 0 kip
M = 8 kip ft
8 ft < x < 12 ft V = 4 kip
M = 4(x 12) kip ft
Ans.
9

1573
8.2 Shear and Bending-Moment Diagrams: Equation Form Example 5, page 6 of 6
Plot V and M versus x.
V
(kip)
M
(kip ft)
4
x
x
2 ft 2 ft
4 kip
2 ft
R

A
= 4 kip
A
4 ft
R

B
= 4 kip
2 ft
B
4 kip
4
16
8
10
11 Note that the jumps in the diagrams occur at the
attachment points of the arms, not at the points where
the 4-kip external loads act.
8
16

1574
8.2 Shear and Bending-Moment Diagrams: Equation Form Example 6, page 1 of 8
6. Express the shear V and bending moment M as functions of x, the distance from
the left end of the beam to an arbitrary point on the beam. Plot V and M vs. x.
x
A
B
4 kN
F

y
= 0: R

A
+ R

B
4 kN 8 kN + R

C
= 0 (1)
M

A
= 0: R

B
(2 m) (4 kN)(2 m + 2 m)
(8 kN)(2 m + 2 m + 2 m)
+ R

C
(2 m + 2 m + 2 m + 2 m) = 0 (2)
Draw a free-body diagram and find the reactions.
+
+
1
C
Hinge
A
hinge
B
C
4 kN 8 kN
Two equations but three unknowns.
An additional equation is needed.
2
2 m 2 m 2 m 2 m
R

C
2 m 2 m 2 m 2 m
R

B
R

A
8 kN

1575
8.2 Shear and Bending-Moment Diagrams: Equation Form Example 6, page 2 of 8
Pass a section through the beam at a point
immediately to the right of the hinge.
A
B
4 kN
Draw a free-body diagram of the portion of
the beam to the right of the section.
+
8 kN
C
Hinge
2 m 2 m
C
8 kN
R

C
3
M = 0
V
4
Because the section is next to a hinge,
the moment is known to be zero there
(that's what we mean by a "hinge").
5
Write the equilibrium equation for the sum of moments about the hinge.
M

hinge
= 0: (8 kN)(2 m) + R

c(2 m + 2 m) = 0 (3)
6
Note that we don't use the equation F

y
= 0, because this equation
would introduce an additional unknown, the shear V at the hinge.
7
2 m 2 m 2 m 2 m

1576
8.2 Shear and Bending-Moment Diagrams: Equation Form Example 6, page 3 of 8
Pass a section through the beam at a point
between the left end and the reaction at B.
A
Hinge
B
C
4 kN 8 kN
8 Solving Eqs. 1-3 gives
R

A
= 8 kN = 8 kN
R

B
= 16 kN
R

C
= 4 kN
9
x
10 Draw a free-body diagram of the portion of the beam to
the left of the section and find V and M at the section.
A
R

A
= 8 kN
x
M
V
+
+
R

C
= 4 kN
2 m 2 m 2 m 2 m
R

B
= 16 kN R

A
= 8 kN
0 < x < 2 m
F

y
= 0: kN V = 0
M

x = 0: kN)x + = 0
Solving gives
V = kN (4)
M = ( x) kNm (5)
valid for 0 < x < 2 m.

1577
8.2 Shear and Bending-Moment Diagrams: Equation Form Example 6, page 4 of 8
Pass a section through the beam at a point
between the the reaction at B and the hinge.
A
Hinge
B
C
4 kN
8 kN
x
Draw a free-body diagram of the portion of the beam to
the left of the section and find V and M at the section.
A
R

A
= 8 kN
x
M
V
F

y
= 0: 8 kN + 16 kN V = 0
M

x = 0: (8 kN)x (16 kN)(x 2 m) + M = 0
Solving gives
V = 8 kN (6)
M = (8x 32) kNm (7)
valid for 2 m < x < 4 m.
+
+
11
12
2 ft
R

B
= 16 kN
B
2 m < x < 4 m
R

C
= 4 kN
2 m 2 m 2 m 2 m
R

B
= 16 kN R

A
= 8 kN

1578
8.2 Shear and Bending-Moment Diagrams: Equation Form Example 6, page 5 of 8
Pass a section through the beam at a point
between the hinge and the 8-kN force.
A
Hinge
B
C
4 kN 8 kN
x
A
M
V
B
13
14
Hinge
4 kN
x
4 m < x < 6 m
R

C
= 4 kN
2 m 2 m 2 m 2 m
R

B
= 16 kN R

A
= 8 kN
2 m
R

B
= 16 kN
2 m
F

y
= 0:
M

x
R

A
= 8 kN
Draw a free-body diagram of the portion of the beam to
the left of the section and find V and M at the section.
8 kN + 16 kN 4 kN V = 0
= 0: (8 kN)x 16 kN)(x 2 m)
+ (4 kN)(x 4 m) + M = 0
Solving gives
V = 4 kN (8)
M = (4x 16) kNm (9)
valid for 4 m < x < 6 m.

1579
8.2 Shear and Bending-Moment Diagrams: Equation Form Example 6, page 6 of 8
Solving gives
V = 4 kN (10)
M = ( 4x + 32) kNm (11)
valid for 6 m < x < 8 m.
Mx = 0: (8 kN)x (16 kN)(x 2 m)
+ (4 kN)(x 4 m)
+ (8 kN)(x 6 m) + M = 0
8 kN + 16 kN 4 kN
8 kN V = 0
Pass a section through the beam at a point between
the 8-kN force and end C of the beam.
A
Hinge
B
C
4 kN 8 kN
x
A
M
V
B
15
16
Hinge
4 kN
x
8 kN
2 m 2 m
6 m < x < 8 m
R

B
= 16 kN
2 m
F

y
= 0:
R

A
= 8 kN
Draw a free-body diagram of the portion of the beam to the
left of the section and find V and M at the section.
R

C
= 4 kN
2 m 2 m 2 m 2 m
R

B
= 16 kN R

A
= 8 kN

1580
8.2 Shear and Bending-Moment Diagrams: Equation Form Example 6, page 7 of 8
Collect the results from Eqs. 4-11:
0 < x < 2 m V = 8 kN
M = 8x kNm
2 m < x < 4 m V = 8 kN
M = (8x 32) kNm
4 m < x < 6 m V = 4 kN
M = (4x 16) kNm
6 m < x < 8 m V = 4 kN
M = ( 4x + 32) kNm
17
Ans.

1581
8.2 Shear and Bending-Moment Diagrams: Equation Form Example 6, page 8 of 8
Plot V and M versus x.
V
(kN)
M
(kN m)
4
x
x
8
18
A
x
B
Hinge
4 kN
C
8 kN
8
4
16
8
4
8
4
2m
R

C
= 4 kN
R

B
= 16 kN
2 m
R

A
= 8 kN
2 m 2 m

1582
8.2 Shear and Bending-Moment Diagrams: Equation Form Example 7, page 1 of 7
7. Express the shear V and bending moment M as functions of x, the distance from
the left end of the beam to an arbitrary point on the beam. Plot V and M vs. x.
x
A
B
20 kip
F

y
= 0: R

A
20 kip 32 kip + R

B
= 0 (1)
M

A
= 0: M

A
(20 kip)(5 ft) (32 kip)(5 ft + 5 ft + 4 ft)
+ R

B
(5 ft + 5 ft + 8 ft) = 0 (2)
Draw a free-body diagram and find the reactions.
+
R

A
1
R

B
5 ft 5 ft 8 ft
Hinge
Two equations but three unknowns.
An additional equation is needed.
4 kip/ft
20 kip
Hinge
B
= 4 ft
2
8 ft Resultant = (4 kip/ft)(8 ft) = 32 kip
A
M

A
2 A couple moment must always be
included at a built-in end of a beam.
3
+
5 ft 5 ft
8 ft

1583
8.2 Shear and Bending-Moment Diagrams: Equation Form Example 7, page 2 of 7
Pass a section through the beam at a point immediately to the right of the hinge.
Draw a free-body diagram of the portion of
the beam to the right of the section.
+
M = 0
4
Because the section is next to a hinge,
the moment is known to be zero there
(that's what we mean by a "hinge").
5
Write the equilibrium equation for the sum of moments about the hinge.
M

hinge
= 0: (32 kip)(4 ft) + R

B
(8 ft) = 0 (3)
6
Note that we don't use the equation F

y
= 0, because this equation
would introduce an additional unknown, the shear V at the hinge.
7
R

B
B
Resultant = (4 kip/ft)(8 ft) = 32 kip
Hinge
20 kip
M

A
A
R

A
5 ft 5 ft 8 ft
8 ft
Resultant = 32 kip
B
R

B
8
V
= 4 ft
8 ft
2
4 ft

1584
8.2 Shear and Bending-Moment Diagrams: Equation Form Example 7, page 3 of 7
Pass a section through the beam at a point between
the left end and the 20-kip force.
Solving Eqs. 1-3 gives
R

A
= 36 kip
R

B
= 16 kip
M

A
= 260 kip ft
9
x
10
Draw a free-body diagram of the portion of the beam to
the left of the section and find V and M at the section.
x
M
V
F

y
= 0: 36 kip V = 0
M

x = 0: 260 kip ft (36 kip)x + M = 0
Solving gives
V = 36 kip (4)
M = (36x 260) kip ft (5)
valid for 0 < x < 5 ft.
+
+
0 < x < 5 ft
8 ft
B
R

B
= 16 kip
5 ft
20 kip
Hinge
5 ft
M

A
= 260 kip ft
R

A
= 36 kip
A
11
A
R

A
= 36 kip
M

A
= 260 kip ft
4 kip/ft

1585
8.2 Shear and Bending-Moment Diagrams: Equation Form Example 7, page 4 of 7
Pass a section through the beam at a point between
the 20-kip force and the hinge.
x
12
Draw a free-body diagram of the portion of the beam to
the left of the section and find V and M at the section.
M
V
F

y
= 0: 36 kip 20 kip V = 0
M

x = 0: 260 kip ft (36 kip)x + 20 kip(x 5 ft) + M = 0
Solving gives
V = 16 kip (6)
M = (16x 160) kip ft (7)
valid for 5 ft < x < 10 ft.
+
+
5 ft < x < 10 ft
8 ft
B
R

B
= 16 kip
5 ft
20 kip
Hinge
5 ft
M

A
= 260 kip ft
R

A
= 36 kip
A
13
R

A
= 36 kip
5 ft
M

A
= 260 kip ft
A
20 kip
x
4 kip/ft

1586
8.2 Shear and Bending-Moment Diagrams: Equation Form Example 7, page 5 of 7
Pass a section through the beam at a point between
the hinge and the right end of the beam.
x
14
Draw a free-body diagram of the portion of the beam to
the left of the section and find V and M at the section.
M
V
F

y
= 0: 36 kip 20 kip
(4 kip/ft)(x 10 ft) V = 0
M

x = 0: 260 kip ft (36 kip)x + 20 kip(x 5 ft)
+ (4 kip/ft)(x 10 ft)( )
+ M = 0
Solving gives
V = 4x + 56 kip (8)
M = ( 2x
2
+ 56x 360) kip ft (9)
valid for 10 ft < x < 18 ft.
+
+
10 ft < x < 18 ft
8 ft
B
R

B
= 16 kip
5 ft
20 kip
Hinge
5 ft
M

A
= 260 kip ft
R

A
= 36 kip
A
15
Resultant = (4 kip/ft)(x 10 ft)
5 ft 5 ft
A
20 kip
Hinge
2
x 10 ft
M

A
= 260 kip ft
x
R

A
= 36 kip
2
x 10 ft
4 kip/ft

1587
8.2 Shear and Bending-Moment Diagrams: Equation Form Example 7, page 6 of 7
Collect the results from Eqs. 4-9:
0 < x < 5 ft V = 36 kip
M = (36x 260) kip ft
5 ft < x < 10 ft V = 16 kip
M = (16x 160) kip ft
10 ft < x < 18 ft V = ( 4x + 56) kip
M = ( 2x
2
+ 56x 360) kip ft
Ans.
16

1588
8.2 Shear and Bending-Moment Diagrams: Equation Form Example 7, page 7 of 7
Plot V and M versus x.
V
(kN)
M
(kN m)
x
x
16
x
16
8 ft 5 ft
R

A
= 36 kip
M

A
= 260 kip ft
A
Hinge
5 ft
20 kip
R

B
= 16 kip
B
36
260
80
32
17
36
16
4 kip/ft

1589
8.2 Shear and Bending-Moment Diagrams: Equation Form Example 8, page 1 of 8
x
A
B
Draw a free-body diagram and find the reactions.
R

A
1
R

B
6 m 6 m
4 kN/m
2 kN/m
6 m 6 m
A
B
2 kN/m
2 kN/m
Replace the trapezoidal distributed load by the sum of a rectangular and triangular load. 2
8. Express the shear V and bending moment M as functions of x, the distance from
the left end of the beam to an arbitrary point on the beam. Plot V and M vs. x.

1590
8.2 Shear and Bending-Moment Diagrams: Equation Form Example 8, page 2 of 8
R

A R

B
6 m 6 m
A
B
5
1
2
6
Resultant of rectangular load
= (12 m)(2 kN/m)
= 24 kN
Resultant of triangular load
= (12 m)(2 kN/m)
= 12 kN
3
4
12 m
2 kN/m
12 m
2 kN/m
6 m 4 m = 2 m
(6 m + 6 m) = 4 m
(acts through centroid of triangle)
1
3
F

y
= 0: R

A
24 kN 12 kN + R

B
= 0
M

A
= 0: (12 kN)(2 m) + R

B
(6 m) = 0
Solving gives
R

A
= 32 kN
R

B
= 4 kN
+
+

1591
8.2 Shear and Bending-Moment Diagrams: Equation Form Example 8, page 3 of 8
x
A
B
R

A
= 32 kN
R

B
= 4 kN
6 m
4 kN/m
2 kN/m
7 Pass a section through the beam at a point between the left end
and the support at A.
0 < x < 6 m
2 kN/m
Draw a free-body diagram of the portion of the beam to the left
of the section and solve for V and M.
8
V
M
x
= distributed load (kN/m) at location x
w
6 m

1592
8.2 Shear and Bending-Moment Diagrams: Equation Form Example 8, page 4 of 8
Before we can solve for V and M, we have to express w as a function of x. This can be done
by noting that w is a linear function of x and then using the slope-intercept equation for a line.
9
w
x
(0, 2 kN/m)
(x, w)
(12 m, 4 kN/m)
w = mx + b
= x + 2 kN/m
= + 2 (1)
4 kN/m 2 kN/m
12 m 0
x
6
Slope
Intercept
Now the distributed load on the free-body of length x can be
replaced by the resultant of a rectangular and triangular load.
10
x
2 kN/m
V
M
w
=
x
V
M

2
x
3
x
Resultant of rectangular load
= (2 kN/m)x
Resultant of triangular load
= (w 2)
11
12
2
x
w 2
Free-body diagram

1593
8.2 Shear and Bending-Moment Diagrams: Equation Form Example 8, page 5 of 8
F

y
= 0: (2 kN/m)x (w 2) V = 0
M

x = 0: (2 kN/m)(x)( ) + [ (w 2)( )] + M = 0
Replacing w in these equations by w = (x/6) + 2 from
Eq. 1 and solving gives
V = ( 2x) kN (2)
M = ( x
2
) kN m (3)
valid for 0 < x < 6 m.
+
+
13
3
x
x
2
x
2
x
2
x
2

12
x
3

36
Pass a section through the beam at a point between
the support at A and the support at B.
6 m 6 m
2 kN/m
6 m < x < 12 m
x
R

A
= 32 kN
A
R

B
= 4 kN
B
4 kN/m
14

1594
8.2 Shear and Bending-Moment Diagrams: Equation Form Example 8, page 6 of 8
6 m
2 kN/m
6 m < x < 12 m
R

A
= 32 kN
A
15
x 6 m
x
w(x)
V
M
We can save some work if we note that this free-body diagram is identical to the
previous one except that an additional vertical force of 32 kN is present. This
increases the shear in Eq. 2 by 32 kN and the moment in Eq. 3 by (32 kN)(x 6 m) so
V = ( 2x + 32) kN (4)
M = ( x
2
+ 32x 192) kN m (5)
valid for 6 m < x < 12 m.
16
36
x
2

12
x
3

Free-body diagram

1595
8.2 Shear and Bending-Moment Diagrams: Equation Form Example 8, page 7 of 8
Collect the results from Eqs. 4-11:
0 < x < 6 m V = ( x
2
2x) kN
M = ( x
3
x
2
) kN m
6 m < x < 12 m V = ( x
2
2x + 32) kN
M = ( x
3
x
2
+ 32x 192) kN m
Ans.
17
12
36
1
1
12
36
1
1

1596
8.2 Shear and Bending-Moment Diagrams: Equation Form Example 8, page 8 of 8
Plot V and M versus x.
V
(kN)
M
(kN m)
x
x
x
15
2.03
18
6 m 6 m
2 kN/m
R

A
= 32 kN
A B
4 kN/m
4
42
17
R

A
= 4 kN

1597

8.3 Shear and Bending-Moment Diagrams Constructed by Areas
1598
8.3 Shear and Bending-Moment Diagrams Constructed by Areas Procedures and Strategies, page 1 of 3
Procedures and Strategies for Solving Problems Involving
Constructing Shear and Bending-Moment Diagrams by Areas
1. You can construct the shear diagram by using the following
rules:
a) A downward concentrated load P causes a downward jump
of magnitude P in the shear diagram.
b) The change in shear between two "shear critical points"
(points at which a concentrated load acts or a distributed
load begins or ends) equals the negative of the area under the
distributed load curve, V = w dx.
c) The slope of the shear curve equals the negative of the
distributed load, dV/dx = w. Thus if w = constant, the
shear curve is a straight line; if w is linear, the shear curve is
parabolic; and if no distributed load is present (w = 0), the
shear curve is a horizontal line.
Application of the rules to construct a shear diagram: Draw a
free-body diagram of the beam, and solve for the reactions.
Then starting from the left end of the beam, proceed from
critical point to critical point, and apply rules a) and b) to
determine the values of the shear at the critical points. Apply
rule c) to determine the type of curve that connects the shear
values at the critical points on the shear diagram.
2 kN/m 4 kN
V(kN)
x(m)
4 4
10
Area of rectangle = 2 kN/m 3 m = 6 kN
Change in shear
= negative of area
= 6 kN
3 m 3 m

1599
8.3 Shear and Bending-Moment Diagrams Constructed by Areas Procedures and Strategies, page 2 of 3
2. You can construct the moment diagram by using
the following rules:
a) a clockwise couple moment M causes an
upward jump in the moment diagram.
b) The change in moment between two "moment
critical points" (shear critical points plus points
at which a couple moment acts) equals the area
under the shear curve, M = V dx.
c) The slope of the moment curve equals the
shear, dM/dx = V. Thus if V is linear, M is
parabolic; if V is constant, M is linear.
Application of the rules to construct a moment
diagram: Starting from the left end of the beam,
proceed from critical point to critical point, and
apply rules a) and b) to determine the values of
the moment at the critical points. Apply rule c) to
determine the type of curve that connects the
moment values at the critical points.
10 kN m
7 kN
7
11
3 m 3 m
7
V(kN)
x(m)
M(kN m)
x(m)
10
10
Area = 7 kN 3 m
= 21 kN m
= change in M

1600
8.3 Shear and Bending-Moment Diagrams Constructed by Areas Procedures and Strategies, page 3 of 3
Notes:
1) A point on the shear diagram at which V = 0
requires special consideration because dM/dx =
V = 0 implies that the moment M is a local
maximum or minimum there. First determine
perhaps by using similar triangles the
precise location where V = 0 on the shear
diagram. Then treat this point as an additional
critical point on the moment diagram and
calculate the moment there by applying rule b).
2) Both diagrams must close. If either diagram
does not close, check for a mistake in either
your calculation of the reactions or in your
calculation of V and M at successive critical
points.
8
9
M (kN m)
V(kN)
4 m 4 m
2 kN/m
2
6
2
x (m)
x(m)
(4 s)
s
Similar triangles:
=
(4 s) s
2 6

1601
8.3 Shear and Bending-Moment Diagrams Constructed by Areas Problem Statement for Example 1
A
C
6 kN
B
1. Draw the shear and moment diagrams for the beam.
2 m 4 m

1602
8.3 Shear and Bending-Moment Diagrams Constructed by Areas Problem Statement for Example 2
2. Draw the shear and moment diagrams for the beam.
2 kN 3 kN 4 kN 3 kN 2 kN
B A
2 m 2 m 2 m 2 m 2 m 2 m

1603
8.3 Shear and Bending-Moment Diagrams Constructed by Areas Problem Statement for Example 3
3. Draw the shear and moment diagrams for the beam.
30 lb/ft
A B
24 ft

1604
8.3 Shear and Bending-Moment Diagrams Constructed by Areas Problem Statement for Example 4
4. Draw the shear and bending moment diagrams for the beam.
2 kip/ft
B A
C
8 ft 8 ft

1605
8.3 Shear and Bending-Moment Diagrams Constructed by Areas Problem Statement for Example 5
5. Draw the shear and moment diagrams for the beam.
A C
B
2 kN/m
2 m 4 m

1606
8.3 Shear and Bending-Moment Diagrams Constructed by Areas Problem Statement for Example 6
6. Draw the shear and moment diagrams for the beam.
B A
C
10 m 20 m
60 kN m

1607
8.3 Shear and Bending-Moment Diagrams Constructed by Areas Problem Statement for Example 7
4 ft 4 ft 4 ft 4 ft
12 kip ft 8 kip ft
20 kip ft
E
D C
A B
7. Draw the shear and moment diagrams for the beam.

1608
8.3 Shear and Bending-Moment Diagrams Constructed by Areas Problem Statement for Example 8
D A 8 kN m C
2 kN/m
E
Hinge
4 kN
B
3 m 3 m 3 m 3 m
8. Draw the shear and moment diagrams for the beam.

1609
8.3 Shear and Bending-Moment Diagrams Constructed by Areas Problem Statement for Example 9
B A
C
D
Hinge
4 kN/m
15 kN m
9. Draw the shear and moment diagrams for the beam.
21 kN
3 m 6 m 3 m

1610
8.3 Shear and Bending-Moment Diagrams Constructed by Areas Example 1, page 1 of 8
2 m 4 m
2 m 4 m
A
C
6 kN
B
First determine the reactions by drawing a free-body diagram
of the whole beam and writing equilibrium equations.
1
1. Draw the shear and moment diagrams for the beam.
A
C
6 kN
B
A

x
A

y
B

y
Equilibrium equations
F

x 0: A

x 0
F

y
0: A

y
+ B

y
6 0
M

A
0: ( 6 kN) 2 m + B

y
(2 m + 4 m) 0
Solving gives

A

x 0
A

y
4 kN
B

y
2 kN
+
+
+

1611
8.3 Shear and Bending-Moment Diagrams Constructed by Areas Example 1, page 2 of 8
A C
6 kN
B
2 kN 4 kN
Begin plotting the shear diagram by noting that the 4-kN upward
reaction force at A causes a 4-kN upward jump at A.
3
x(m)
V(kN)
Free-body diagram showing reactions 2
4
4

1612
8.3 Shear and Bending-Moment Diagrams Constructed by Areas Example 1, page 3 of 8
A
C
6 kN
B
w 0
2 kN 4 kN
x(m)
V(kN)
2
4 4
No distributed load, w, acts between A and C,
so the change in shear is zero:
V negative of area under w curve
0
4
2 m 4 m

1613
8.3 Shear and Bending-Moment Diagrams Constructed by Areas Example 1, page 4 of 8
2 m 4 m
A
C
6 kN
B
2 kN 4 kN
x(m)
V(kN)
2
4
2 ( 4 6)
The 6-kN downward force causes
a 6-kN downward jump at C.
5
6 kN

1614
8.3 Shear and Bending-Moment Diagrams Constructed by Areas Example 1, page 5 of 8
4 m
A
C
6 kN
B
2 kN
4 kN
x(m)
V(kN)
2
4
6
No distributed load, w, acts between C
and B, so the change in shear is zero:
V negative of area under w curve
0
6
-2
-2
4
w 0
2 m

1615
8.3 Shear and Bending-Moment Diagrams Constructed by Areas Example 1, page 6 of 8
A
C
6 kN
B
2 kN
4 kN
x(m)
V(kN)
2
4
-2 -2
4
0
The 2-kN upward reaction at B causes a
2-kN upward jump in the shear at B
7
V 2 + 2 0. The shear diagram closes at
the end of the beam, as it must because the
internal shear V 2 must be balanced by the
2-kN external reaction, as shown in a free-body
diagram of a short length of beam at end B:
8
2 kN
B
2 kN
dx
(negative shear)
2 kN
2 m 4 m

1616
8.3 Shear and Bending-Moment Diagrams Constructed by Areas Example 1, page 7 of 8
A
C
6 kN
B
2 kN 4 kN
x(m)
V(kN)
2
4
-2 -2
4
x(m)
M(kNm)
C
The change in moment equals
the area under the shear curve:
M cross-hatched area
4 kN 2 m
8 kN m
So the moment at point C is
M

C
M

A
+ M
0 + 8 kN m
The curve connecting M

A
and M

C
must have constant slope,
since dM/dx shear 4 kN for all points between A and C. But
a "curve with constant slope" is simply a straight line.
11
10
A
B C
8
1
4
A
No moment
acts at the end
of the beam,
so the moment
diagram starts
at 0.
9
2 m 4 m

1617
8.3 Shear and Bending-Moment Diagrams Constructed by Areas Example 1, page 8 of 8
A
C
6 kN
B
2 kN 4 kN
x(m)
V(kN)
2
4
-2 -2
4
x(m)
M(kN m)
C
8
B
M

C
and M

B
are connected by a
straight line of slope

dM/dx shear 2 kN
13
The change in moment equals the area under the
shear curve:
M ( 2 kN)(6 m 2 m)
8 kN m
So the moment at point B is
M

B
M

C
+ M
8 + ( 8 kN m)
0 (The moment diagram must close.)
Maximum moment 8 kN m 14
2
1
12
A B
6 C
2 m 4 m
A

1618
8.3 Shear and Bending-Moment Diagrams Constructed by Areas Example 2, page 1 of 5
2. Draw the shear and moment diagrams for the beam.
Determine the reactions by drawing a free-body diagram of
the whole beam and writing equilibrium equations.
Equilibrium equations
F

x 0: A

x 0
F

y
0: A

y
+ B

y
2 kN 3 kN 4 kN
3 kN 2 kN 0
M

A
0: 2 kN 2 m 3 kN 4m
4 kN 6 m 3 kN 8 m
2 kN 10 m + B

y
12 m 0
Solving gives
A

x 0
A

y
7 kN
B

y
7 kN
2 kN 3 kN 4 kN 3 kN 2 kN
2 kN 3 kN 4 kN 3 kN 2 kN
B

y
A

x
A

y
B A
1
+
+
+
2 m 2 m 2 m 2 m 2 m 2 m
2 m 2 m 2 m 2 m 2 m 2 m

1619
8.3 Shear and Bending-Moment Diagrams Constructed by Areas Example 2, page 2 of 5
2
2 kN 3 kN 4 kN 3 kN 2 kN
C
B A
Free-body diagram showing reactions
No distributed load acts between A and C
so the shear is unchanged.
The 7-kN upward
reaction at A causes a
7-kN upward jump in
the shear diagram.
3
7 kN
7 kN
4
x(m)
V(kN)
7 7
2 m 2 m 2 m 2 m 2 m 2 m

1620
8.3 Shear and Bending-Moment Diagrams Constructed by Areas Example 2, page 3 of 5
2 kN 3 kN 4 kN 3 kN 2 kN
C
B A
7 kN 7 kN
x(m)
V(kN)
7
7
5 2 7
7
The remainder of the
shear diagram
consists of horizontal
portions ( V area
under load curve 0)
and jumps at
concentrated loads.
5
7 2 5
5 3 2
2 4 2
2 3 5
7 7 0
Diagram closes 6
2
7
3
4
2
5
2
2
5
3
2 m 2 m 2 m 2 m 2 m 2 m
2
C

1621
8.3 Shear and Bending-Moment Diagrams Constructed by Areas Example 2, page 4 of 5
2 kN 3 kN 4 kN 3 kN 2 kN
C
B A
7 kN 7 kN
x(m)
V(kN)
7
7
7
5
2
2
5
No moment acts at
the left end of the
beam so the moment
diagram starts at 0.
7
x(m)
M(kN m)
14
A
2
7
A
2
Since the shear is constant between A and C, dM/dx V 7 kN,
and the moment curve is a straight line.
9
7
1
C
The change in moment equals
the area under the shear curve:
M 7 kN 2 m 14 kN m
Thus the moment at C is
M

C
M

A
M 0 14 kN m
8
2 m 2 m 2 m 2 m 2 m 2 m
C B
B

1622
8.3 Shear and Bending-Moment Diagrams Constructed by Areas Example 2, page 5 of 5
6
2 kN 3 kN 4 kN 3 kN 2 kN
C
B
A
7 kN 7 kN
x(m)
V(kN)
7
7
7
5
2
2
5
x(m)
14
A
2
7
7
1
C
5
2
2
5
Maximum moment 28 kNm
Diagram closes
The remainder of the
moment diagram can be
found by adding areas
under the shear curve to
moment values.
10
14 10 24
24 4 28
12
11
7
1
5
1
28 4 24
24 10 14
14 14 0
Area
2 2
4
Area
( 7) 2
14
Area
( 5) 2
10
Area
( 2) 2
4
2
1
2
1
5
1
M(kN m)
Area
5 2
10
2 m 2 m 2 m 2 m 2 m 2 m

1623
8.3 Shear and Bending-Moment Diagrams Constructed by Areas Example 3, page 1 of 5
+
+
+
24 ft
3. Draw the shear and moment diagrams for the beam.
Determine the reactions by drawing a
free-body diagram of the whole beam and
writing equilibrium equations:
1
Equilibrium equations
F

x 0: A

x 0
F

y
0: A

y
+ B

y
(30 lb/ft) 24 ft 0
M

A
0: B

y
24 ft (30 lb/ft)(24 ft)(24/2 ft) 0
Solving gives
A

x 0
A

y
360 lb
B

y
360 lb
30 lb/ft
A B
B

y
A

x
A

y
A
B
30 lb/ft
24 ft

1624
8.3 Shear and Bending-Moment Diagrams Constructed by Areas Example 3, page 2 of 5
30 lb/ft
A
B
The 360-lb vertical reaction
at end B closes the diagram.
6
Free-body diagram of beam showing reactions 2
The 360-lb upward
reaction at A causes a
360-lb upward jump in
the shear diagram.
3
The curve connecting
V

A
and V

B
must be a
straight line since it
has constant slope:
dV/dx w
30
5
360 lb
360 lb
x(ft)
The change in shear from end A to end B equals
the negative of the area under the load curve:
V (area of rectangle under w curve)

(30 lb/ft)(24 ft) 720 lb
Thus the shear at B is
V

B V

A V
360 lb ( 720 lb) 360 lb
4
V(lb)
24 ft
1
30
360
B
360 + 360 0
A
360

1625
8.3 Shear and Bending-Moment Diagrams Constructed by Areas Example 3, page 3 of 5
360
B
A
360
Change in moment equals area under shear
curve:
M area of cross-hatched triangle
(1/2)(360 lb)(12 ft)
2160 lb ft
So
M

C
M

A
+ M
0 + 2160 lb ft
No moment
acts at the left
end of the
beam so the
moment
diagram starts
at 0.
Because of symmetry, the point where
V 0 is easily found to lie at the
middle of the beam span. This point
corresponds to a local maximum of the
moment, because there the derivative
of M is zero:
dM/dx V
0
7
8
9
C
M(lb ft)
x(ft) A
C
12
(24 ft)/2 12 ft
2160
30 lb/ft
A
B
360 lb
360 lb
x(ft)
V(lb)
24 ft

1626
8.3 Shear and Bending-Moment Diagrams Constructed by Areas Example 3, page 4 of 5
x(ft)
V(lb)
360
B
A
360
C
M(lb ft)
x(ft) A
C
12
2160
12
A
C
12
M(lb ft)
x(ft)
dM/dx V

C
0
(horizontal tangent)
Because the shear is a linear function
and dM/dx V, integration would show
that M is a quadratic function. But how
can you decide which of the two
possible quadratic curves is correct?
Answer: Draw short line segments
showing the slope of the curve at
the ends of the interval and choose
the curve that matches these slopes.
10
11
360
1
dM/dx V

A
360 lb

1627
8.3 Shear and Bending-Moment Diagrams Constructed by Areas Example 3, page 5 of 5
24
Maximum moment 2160 lb ft 14
Moment curve is quadratic with
zero slope at point C.
13
B
24
2160
Area (1/2)( 360 lb)(24 ft 12 ft)
2160 lb ft
So change in moment is
M 2160 lb ft
and
M

B
M

C
+ M
2160 lb ft 2160 lb ft
0 (diagram closes)
12
24 ft
360 lb
A
360 lb
B
30 lb/ft
x(ft)
V(lb)
360
B
A
360
C
12
A
C
12
M(lb ft)
x(ft)
dM/dx V

C
0
(horizontal tangent)

1628
8.3 Shear and Bending-Moment Diagrams Constructed by Areas Example 4, page 1 of 6
4. Draw the shear and bending moment diagrams for the beam.
Determine the reactions by drawing a
free-body diagram of the whole beam
and writing equilibrium equations:
2 kip/ft
B A
C
2 kip/ft
B
B

y
A

x
A

y
A
1
C
8 ft 8 ft
Equilibrium equations
F

x 0: A

x 0
F

y
0: A

y
B

y
(2 kip/ft) 8 ft 0
M

A
0: (8 ft + 8 ft) B

y
(2 kip/ft)
(8 ft)[(8 ft)/2] 0
Solving gives
A

x 0
A

y
12 kip
B

y
4 kip
+
+
+
8 ft 8 ft

1629
8.3 Shear and Bending-Moment Diagrams Constructed by Areas Example 4, page 2 of 6
8 ft 8 ft
2 kip/ft
B
A
C
The change in shear equals the negative
of the area under the load curve, w:
V (area of rectangle under w)
(2 kip/ft) 8 ft
16 kip
Thus the shear at C is
V

C
V

A
V
12 16
4
4
The curve connecting V

A
and
V

C
must be a straight line
since it has constant slope:
dV/dx w
2
5
The 12-kip
reaction at A
causes a jump in
the shear diagram.
3
Free-body diagram showing reactions 2
12 kip 4 kip
12
A
V(kip)
x(ft)
C
4
8
2
1

1630
8.3 Shear and Bending-Moment Diagrams Constructed by Areas Example 4, page 3 of 6
2 kip/ft
B A
C
12 kip 4 kip
12
A
V(kip)
x(ft)
C
4
The reaction at B causes a
jump in the shear curve
and closes the diagram.
Since the distributed load, w, between
C and B is zero, the change in shear is
zero:
V area under w curve
0 (no change in V)
6
7
4
B

1631
8.3 Shear and Bending-Moment Diagrams Constructed by Areas Example 4, page 4 of 6
One way to locate the V 0
point is to use similar triangles:
10
12
A C
s
4
s/12 (8 s)/4
Solving gives s 6
We have to locate the point where V 0 (that is, find s) because
the moment diagram has a local maximum or minimum there:
dM/dx V
0 (the condition for a maximum or minimum M)
9
x(ft)
B C
8 16
M(kip ft)
8
s
8 s
8 ft 8 ft
2 kip/ft
B A
C
12 kip 4 kip
12
A
V(kip)
x(ft)
C
4 4
B
Because no
moment reaction is
present at A, the
moment diagram
starts at 0.
8

1632
8.3 Shear and Bending-Moment Diagrams Constructed by Areas Example 4, page 5 of 6
x(ft)
C
8
M(kip ft)
8
s = 6
Because V is linear, M is
quadratic. Because dM/dx V,
M has a slope of 12 at the left
end of the curve and a slope of
zero at the right end.
13
Change in moment equals area
under shear curve:
M area of triangle
(1/2)(12 kip)(6 ft)
36 kip ft
11
M M

A
M
0 36 kip ft
12
8 s 8 6 2
6
dM/dx 12
dM/dx 0
A
36
8 ft 8 ft
12
1
2 kip/ft
B A
C
12 kip 4 kip
12
A
V(kip)
x(ft)
C
4 4
B

1633
8.3 Shear and Bending-Moment Diagrams Constructed by Areas Example 4, page 6 of 6
2 kip/ft
B A
C
12 kip 4 kip
12
A
V(kip)
x(ft)
C
4 4
B
x(ft)
C 8
M(kip ft)
8
6
A
36
Change in moment:
M area of rectangle
( 4)(16 8)
32 kip ft
So
M

B
M

C
M
32 ( 32)
0
16
Quadratic curve has zero
slope at the left end.
15
Maximum moment 18
The curve is a line with constant slope:
dM/dx V = 4
17
16
32
B
4
1
V 0
6
8 ft 8 ft
Change in moment:
M area of triangle
(1/2)( 4)(8 6)
4 kip ft
So
M

C
36 + M
36 4
32 kip ft
14

1634
8.3 Shear and Bending-Moment Diagrams Constructed by Areas Example 5, page 1 of 6
5. Draw the shear and moment diagrams for the beam.
Determine the reactions by drawing a
free-body diagram of the whole beam
and writing equilibrium equations:
1
A C
B
2 kN/m
2 m 4 m
2 kN/m
A
B
C
C

y
C

x
B

y
Equilibrium equations
F

x 0: C

x 0
F

y
0: C

y
+ B

y
(2 kN/m)(2 m + 4 m) 0
M
C
0: B

y
4 m + (2 kN/m)(2 m + 4 m)[(2 m + 4 m)/2] 0
Solving gives
B

y
9 kN
C

x 0
C

y
3 kN
+
+
+
2 m 4 m

1635
8.3 Shear and Bending-Moment Diagrams Constructed by Areas Example 5, page 2 of 6
A
C
2 kN/m
3 kN 9 kN
x(m)
V(kN)
A
B
2
1
2 m 4 m
Slope of shear curve equals negative
of load, w:
dV/dx w 2kN
Change in shear from A to B equals the negative
of the area under the load curve w:
V (area of cross-hatched rectangle)
(2kN/m)(2 m)
4 kN

4
No concentrated force
reaction at end A, so the
shear curve starts at 0.
Free-body diagram showing reactions 2
3
4 ( V

A
+ V 0 + [ 4 kN])
5
B

1636
8.3 Shear and Bending-Moment Diagrams Constructed by Areas Example 5, page 3 of 6
The end reaction
causes a 3-kN jump.
Slope of shear curve equals negative of distributed load, w:
dV/dx w 2
Change in shear from B to C
equals negative of area of
cross-hatched rectangle:
V (2kN/m)(4 m)
8
The 9-kN upward
reaction causes a
9-kN upward jump in
the shear curve.
9
6
7
8
V(kN)
A
B
1
2
x(m)
4
5 ( 4 + 9)
3 ( 5 8)
2
1
B
A C
2 kN/m
C
0 ( 3 + 3)
3
3 kN
9 kN
2 m 4 m
9 kN

1637
8.3 Shear and Bending-Moment Diagrams Constructed by Areas Example 5, page 4 of 6
V(kN)
A
B
x(m)
4
B
A C
2 kN/m
3 kN 9 kN
Change in moment equals area under shear
curve:
M area of triangle
(1/2)(2 m)( 4 kN) = 4 kN m
11
Because no moment
reaction acts at end A,
the moment diagram
starts at zero.
M

B
= M

A
+ M
= 0 + ( 4 kN m)
= 4 kN m
Because V is linear, M is quadratic.
Because dM/dx V, the M curve has a slope
of zero at the left end of the curve and a
slope of 4 at the right end.
12
10
5
2
3
6
x(m)
M(kN m)
2 m 4 m
A
dM/dx 0
1
4
B

1638
8.3 Shear and Bending-Moment Diagrams Constructed by Areas Example 5, page 5 of 6
C
2 kN/m
3 kN
9 kN
5
2
3
x(m)
M(kN m)
By similar triangles,
s/5 (4 - s)/3
Solving gives
s 2.5
Because V is linear, M is quadratic.
Because dM/dx = V, the M curve has a
slope of 5 at left end and zero at the right.
Change in moment:
M area of triangle (1/2)(2.5 m)(5 kN) = 6.25 kN m
We have to locate the point
where V 0, that is, find s.
13
16
14
15
C
s
4 s
4 m 2 m
A B
M M

B
+ M 4 kN m + 6.25 kN m 2.25 kN m
s 2.5
4
5
1
V(kN)
A
B
x(m)
4
B
A

1639
8.3 Shear and Bending-Moment Diagrams Constructed by Areas Example 5, page 6 of 6
V(kN)
A
B
x(m)
4
B
A C
2 kN/m
3 kN 9 kN
5
2
3
x(m)
M(kN m)
C
Because dM/dx = V, the M
curve has a horizontal slope at
the left end and a slope of 3 at
the right.
Change in moment:
M area of triangle
(1/2)(1.5 m)( 3 kN)
2.25 kN m
17
18
4 s 4 2.5 1.5
s = 2.5
dM/dx V 0
4 m 2 m
4
A
2
B
4.5
2.25
C
M

C
2.25 + M
2.25 + ( 2.25) 0
1
3

1640
8.3 Shear and Bending-Moment Diagrams Constructed by Areas Example 6, page 1 of 5
6. Draw the shear and moment diagrams for the beam.
Determine the reactions by drawing a
free-body diagram of the whole beam and
writing equilibrium equations.
1
Equilibrium equations
F

x 0: A

x 0
F

y
0: A

y
+ B

y
0
M

A
0: 60 kN m + B

y
(10 m + 20 m) 0
Solving gives
A

x 0
A

y
2 kN
B

y
2 kN

+
+
+
B A
C
A
C
B
A

y
A

x
B

y
10 m 20 m
60 kN m
10 m 20 m
60 kN m

1641
8.3 Shear and Bending-Moment Diagrams Constructed by Areas Example 6, page 2 of 5
No distributed load, w, acts on the beam so the
change in shear is zero:
V negative of area under w curve
0
4
The couple moment at C does not cause a jump in the shear diagram. 6
3
The 2-kN downward
reaction causes a 2-kN
downward jump in the
shear diagram.
5
Free-body diagram showing reactions 2
2 kN 2 kN
10 m 20 m
60 kN m
x(m)
V(kN)
30
10
A
B C
2
2
The 2-kN upward
reaction causes a 2-kN
upward jump in the
shear diagram.
B A
C

1642
8.3 Shear and Bending-Moment Diagrams Constructed by Areas Example 6, page 3 of 5
The change in moment equals the
area under the shear curve:
M (2 kN)(10 m)
20 kN m
The couple moment at C causes a jump of 60 kN m
in the moment diagram. But how can you decide if
the jump is up or down?
The moment curve is a
straight line between A and
C because
dM/dx V
2 kN
No moment at A so the
moment diagram starts at 0.
9
10
8
7
60 kN m
10 m 20 m
10
A
2
C
x(m)
30
B
2
V(kN)
M(kN m)
x(m)
20
A
10
30
B C
2 kN
2 kN
1
2
80?
40?
B A
C

1643
8.3 Shear and Bending-Moment Diagrams Constructed by Areas Example 6, page 4 of 5
60 kN m
2 kN 2 kN
Consider the moment equilibrium equation for a small
length of the beam containing the couple moment.
M

Left end
M + (M + M) + 60 kN m (V + V)( x) 0

Taking the limit as x 0 gives
M 60 kN m
General conclusion:
A counterclockwise couple moment applied to the beam produces a
downward jump in the moment diagram.
11 x
60 kN m
M
M + M
V
V + V
x
C
+
B A
C

1644
8.3 Shear and Bending-Moment Diagrams Constructed by Areas Example 6, page 5 of 5
2 kN 2 kN
20 m
60 kN m
x(m)
V(kN)
30 10
A B C
2 2
The moment curve is a straight line:
dM/dx V
2
The change in moment is
M area under shear curve
(2 kN)(30 m 10 m)
40 kN m
So
M

B
M

C
+ M
40 + 40
0
13
12
M(kN m)
A C
10
20
x(m)
B
30
20 60 40
1
2
60
10 m
B A
C

1645
8.3 Shear and Bending-Moment Diagrams Constructed by Areas Example 7, page 1 of 3
Determine the reactions by drawing a free-body
diagram of the whole beam and writing
equilibrium equations:
1
D E
20 kip ft
8 kip ft 12 kip ft
A

y
A

x
Equilibrium equations
F

x 0: A

x 0
F

y
0: A

y
0
M

A
0: M

A
20 kip ft 8 kip ft + 12 kip ft 0
Solving gives
A

x 0
A

y
0
M

A
16 kip ft
4 ft 4 ft 4 ft 4 ft
+
+
+
E D
12 kip ft
C B A
20 kip ft 8 kip ft M
A
B A
C
7. Draw the shear and moment diagrams for the beam.

1646
8.3 Shear and Bending-Moment Diagrams Constructed by Areas Example 7, page 2 of 3
3
4
2
x(ft)
V(kip)
V(x) 0
16 kip ft 8 kip ft 20 kip ft
A
B C
12 kip ft
D E
Free-body diagram showing reactions
Because the distributed load w is zero everywhere
and the change in shear equals the negative of the
area under the w curve, the shear never changes
from its starting value: 0.
Because the reaction force at
the end of the beam is zero,
the shear there is zero.

1647
8.3 Shear and Bending-Moment Diagrams Constructed by Areas Example 7, page 3 of 3
C
12 kip ft
D E
6
7
x(ft)
V(kip)
A
B
C
D
E
x(ft)
16
4 ( 16 + 20)
12 ( 4 + 8)
0 ( 12 12)
12
4
16
M(kip ft)
8
12
20
V(x) 0
16 kip ft 8 kip ft 20 kip ft
A
B
Free-body diagram showing reactions
Because the change in moment equals the area under the shear curve,
and the shear curve is V(x) 0, the moment does not change
between points where the couple moments are applied.
Clockwise couple moments produce upward
jumps; counterclockwise, downward jumps.
The counterclockwise
16 kip ft couple
moment causes a 16
kip ft downward
jump in the moment
diagram.
5

1648
8.3 Shear and Bending-Moment Diagrams Constructed by Areas Example 8, page 1 of 11
Determine the reactions by drawing free-body
diagrams and writing equilibrium equations.
Free-body diagram of whole beam
Writing the three equilibrium equations (below)
gives three equations but four unknowns: A

x, A

y
,
B

y
, and M

A
, so an additional free body is needed.
F

x 0: A

x 0 (1)
F

y
0: A

y
+ (2 kN/m)(3 m + 3 m)
4 kN + B

y
0 (2)
M

A
0: M

A
8 kN m
+ (2 kN/m)(3 m + 3 m)
(3 m) (4 kN)
(3 m + 3 m + 3 m)
+ B

y
(3 m + 3 m
+ 3 m + 3 m) 0 (3)
8. Draw the shear and moment diagrams for the beam.
D A
8 kN m C
2 kN/m
E
Hinge
4 kN
B
1
2
3
Hinge
2 kN/m
A 8 kN m C D E
B
4 kN
B

y
A

y
A

x
M

A
+
+
+
3 m 3 m 3 m 3 m
3 m 3 m 3 m 3 m

1649
8.3 Shear and Bending-Moment Diagrams Constructed by Areas Example 8, page 2 of 11
To obtain an additional free-body diagram, pass a section
through the beam close to the hinge. Then consider the
portion of the beam to the right of the section.
Free-body diagram of the portion of the
beam to the right of the section.
Free-body diagram of the whole beam (repeated)
M

D
0
(moment is zero
at a hinge)
5
6
4
Moment equilibrium for portion DEB of the beam:
M

Hinge
0: B

y
(3 m + 3 m) (4 kN)(3 m) 0 (4)
Using a free body extending from the hinge to end B gives an
equation involving only one unknown, B

y
.
Solving Eqs.1-4 gives
A

x 0, A

y
10 kN, B

y
2 kN, and M

A
16 kN m
7
8
D
Hinge
B
E
B

y
4 kN V

D
(unknown)
shear in beam
3 m
+
3 m
3 m 3 m 3 m 3 m
M

A
A

x
A

y
B

y
4 kN
B
E
D
C 8 kN m A
2 kN/m
Hinge

1650
8.3 Shear and Bending-Moment Diagrams Constructed by Areas Example 8, page 3 of 11
B
A C D
4 kN
Hinge
2 kN/m
8 kN m
E
An upward distributed load
is considered negative.
The couple moment
has no effect on the
shear curve.
The downward
reaction causes a
downward jump.
Free-body diagram showing reactions
12
13
10
9
V

D
V

A
+ V
10 kN + 12 kN
2 kN
Change in shear:
V area under load curve
( 2 kN/m)(3 m + 3 m)
12 kN
14
11
2 kN
10 kN
V(kN)
x(m)
A
D
2
1
10
Slope of shear curve:
dV/dx w
( 2)
2
15
2
3 m 3 m 3 m 3 m
16 kN m

1651
8.3 Shear and Bending-Moment Diagrams Constructed by Areas Example 8, page 4 of 11
4 kN
Hinge
2 kN/m
E
2 kN
10 kN
V(kN)
x(m)
A
D
10
2
Jump in shear caused by
concentrated forces.
No change in shear because distributed load is zero:
w 0
17
16
w 0
12
2
B
C
E
2
9
2
6
3
3 m 3 m 3 m 3 m
4
2
B
C
D
16 kN m
8 kN m
A

1652
8.3 Shear and Bending-Moment Diagrams Constructed by Areas Example 8, page 5 of 11
V(kN)
x(m)
A D
10
2
12
2
B E
2
2
6
To draw the moment diagram, we will need to calculate s,
the distance to the point where the shear curve crosses the
x-axis. By similar triangles, we have
s/10 (6-s)/2
Solving gives
s 5
18
A
D
s
10
6 s
2
s
6 s

1653
8.3 Shear and Bending-Moment Diagrams Constructed by Areas Example 8, page 6 of 11
4 kN
Hinge
2 kN/m
E
2 kN
10 kN
V(kN)
x(m) A
D
10
2
12
2
B
C
E
2
9
2
6
3
The clockwise couple moment at the end
causes an upward jump in the moment
diagram.
19
x(m)
M(kN m)
A
16
F
Change in moment from A to
C equals cross-hatched area.
To calculate this area, first use
similar triangles to calculate
distance h.
h/10 2/5
Solving gives
h 4
A
10
F
C
s 5
3
20
h
5 3 2
3 m 3 m 3 m 3 m
s 5
16 kN m
8 kN m
A
B
C
D

1654
8.3 Shear and Bending-Moment Diagrams Constructed by Areas Example 8, page 7 of 11
Area rectangle + triangle
(3 m)( 4 kN) + (1/2)(3 m)( 6 kN)
21 kN m
Thus the change in moment is
M 21 kN m
16 kN m
8 kN m
A
4 kN
Hinge
2 kN/m
E
2 kN
10 kN
V(kN)
x(m) A
D
10
2
12
2
B
C
E
2
9
2
6 3
3 m 3 m 3 m 3 m
x(m)
M(kN m)
A
16
F
Now that the value of h is known, the
cross-hatched area can be calculated:
3 C
4
M

C
M

A
+ M 6 kN m + ( 21 kN m) 5 kN m 22
4
C A 3
4
10 ( 4) 6
21
s 5
h 4
B
C
D

1655
8.3 Shear and Bending-Moment Diagrams Constructed by Areas Example 8, page 8 of 11
B
C D
4 kN
Hinge
2 kN/m
E
2 kN
10 kN
V(kN)
x(m) A
D
10
2
2
B C E
2
2
x(m)
M(kN m)
A
16
F
3
C
4
Clockwise couple moment
causes an upward jump.
24
dM/dx V so curve is quadratic with
slopes of 10 and 4 at the ends
23
3 m 3 m 3 m
4
1
10
1
8
5
3 m 2 m
1 m
3 m
16 kN m
8 kN m
A

1656
8.3 Shear and Bending-Moment Diagrams Constructed by Areas Example 8, page 9 of 11
B
C
D
4 kN
Hinge 2 kN/m
E
2 kN
10 kN
V(kN)
x(m)
A
D
10
2
2
B
C
E
2
2
x(m)
M(kN m)
A
16
F
3
C
4
3 m 3 m 3 m
5
3 m
1 m
dM/dx V, so
moment curve is
quadratic with slopes
of 4 and 0 at the ends
M

F
M

C
+ M
3 kN m + ( 4 kN m)
1 kN m
26
27
Change in moment:
M area of triangle
(1/2)( 4 kN)(2 m)
4 kN m
25
3 m
Slope 0
F
1
D
Slope 4
2 m
16 kN m
8 kN m
A

1657
8.3 Shear and Bending-Moment Diagrams Constructed by Areas Example 8, page 10 of 11
Change in moment:
M area of triangle
(1/2)(2 kN)(1 m)
1 kN m
28
dM/dx V, so
moment curve is
quadratic with slopes
of 0 and 2.
30
M

D
M

F
+ M
1 kN m + 1 kN m
0
(Actually, we knew in
advance that M

D
0,
because a hinge
exists at point D.)
29
F C A
5
1
M(kN m)
16
3
D
x(m)
2 m
Slope 2
16 kN m
8 kN m
A
4 kN
Hinge
2 kN/m
E
2 kN
10 kN
V(kN)
x(m) A
D
10
2
2
B
C
E
2
2
F
4
3 m 3 m 3 m
3 m
1 m
3 m
B
C
D

1658
8.3 Shear and Bending-Moment Diagrams Constructed by Areas Example 8, page 11 of 11
dM/dx 2 so
moment curve is a
straight line
dM/dx V 2, so
moment curve is a
straight line
M

E
M

D
+ M
M

D
+ area
0 + (2 kN)(3 m)
6 kN m
32
31
33
34
2 m
M

B
M

E
+ M

M

E
+ area
6 kN m + ( 2 kN)(3 m)
0
B
6
2
1
2
1
3
A
5
C
1
F
M(kN m)
16
D
x(m)
E
16 kN m
8 kN m
A
4 kN
Hinge
2 kN/m
E
2 kN
10 kN
V(kN)
x(m) A
D
10
2
2
B
C
E
2
2
F
4
3 m 3 m 3 m
3 m
1 m
3 m
B
C
D

1659
8.3 Shear and Bending-Moment Diagrams Constructed by Areas Example 9, page 1 of 11
Hinge
4 kN/m
15 kN m
Determine the reactions by drawing free-body diagrams
and writing equilibrium equations.
Free-body diagram of whole beam 2
1
Writing the three equilibrium equations (below) gives
three equations but four unknowns: A

x, A

y
, B

y
, and
M

B
, so an additional free body is needed.
3
9. Draw the shear and moment diagrams for the beam.
21 kN
A
Hinge
15 kN m C
D
21 kN
B B

x
B

y
A

y
3 m 6 m
M

B
3 m 3 m 6 m
4 kN/m
3 m
F

x 0: B

x 0 (1)
F

y
0: A

y
+ (4 kN/m)(3 m + 3 m)
21 kN + B

y
0 (2)
M

A
0: (4 kN/m)(3 m + 3 m)(3 m)
(21 kN)(3m) 15 kN m
+ B

y
(3 m + 3 m + 6 m)
+ M

B
0 (3)
+
+
+
B A
C
D

1660
8.3 Shear and Bending-Moment Diagrams Constructed by Areas Example 9, page 2 of 11
B
A
C
D
Hinge
4 kN/m
15 kN m
21 kN
A
Hinge
15 kN m
C
D
4 kN/m
21 kN
3 m
Free-body diagram of the portion of the beam
to the left of the section.
To obtain an additional free-body diagram,
pass a section through the beam close to
the hinge. Then consider the portion of the
beam to the left of the section.
5
6
Solving Eqs. 1-4 gives
A

y
4 kN
B

x 0
B

y
1 kN
M

B
6 kN m
Free-body diagram of whole beam (repeated) 4
8
B

x
B

y
M

B
6 m
A

y
3 m 3 m
D

x
V(unknown)
shear in beam
M 0 at hinge
Moment equilibrium
M

Hinge
0: (A

y
)(3 m + 3 m) (4 kN/m)(3 m + 3 m)(3m)
15 kN m + (21 kN)(3 m) 0 (4)

Using a free body extending from the hinge to end A gives an
equation involving only one unknown, A

y
.
7
+
3 m
A

y

1661
8.3 Shear and Bending-Moment Diagrams Constructed by Areas Example 9, page 3 of 11
Hinge
4 kN/m
15 kN m
21 kN
3 m 6 m
The 21-kN
concentrated load
causes a 21-kN jump
in the shear diagram
13
The 4-kN downward
reaction causes a
downward jump
10
Slope of shear curve equals
negative of distributed load, w:
dV/dx w ( 4) 4
12
V

C
V

A
+ V
V

A
+ [ cross-hatched area]
4 kN + [ ( 4 kN/m)(3 m)]
8 kN
11
Free-body diagram showing reactions 9
1 kN
x(m)
V(kN)
4 kN
An upward
distributed load is
considered negative.
8
A
C
13 ( 8 21)
4
1
3 m
6 kN m
21
4
B
A
C
D

1662
8.3 Shear and Bending-Moment Diagrams Constructed by Areas Example 9, page 4 of 11
Hinge
4 kN/m
15 kN m
21 kN
3 m 6 m
1 kN
x(m)
V(kN)
4 kN
8
A C
13
3 m
6 kN m
Slope of shear curve:
dV/dx w
( 4)
4
V

D
V

C
+ V
V

C
+ [ cross-hatched area]
13 kN + [ ( 4 kN/m)(3 m)]
1 kN
14
15
4
E D B
4
1
1
B
A
C
D

1663
8.3 Shear and Bending-Moment Diagrams Constructed by Areas Example 9, page 5 of 11
B
A
C
D
Hinge
4 kN/m
15 kN m
21 kN
3 m 6 m
1 kN
x(m)
V(kN)
4 kN
8
A C
13
3 m
6 kN m
4
E D B
1
Because the distributed load is zero,
the change in shear is zero.
16
The 1-kN upward reaction
at B closes the diagram.
17
w 0
1

1664
8.3 Shear and Bending-Moment Diagrams Constructed by Areas Example 9, page 6 of 11
B
A
C
D
Hinge
4 kN/m
15 kN m
21 kN
3 m 6 m
1 kN
x(m)
V(kN)
4 kN
8
A C
13
3 m
6 kN m
4
E D B
1 1
4
A
E
C
8
3 s
s
3 s
s
To draw the moment diagram, we will need to
calculate s, the distance to the point where the
shear crosses the x-axis. By similar triangles,
s/4 (3 s)/8
Solving gives
s 1
18

1665
8.3 Shear and Bending-Moment Diagrams Constructed by Areas Example 9, page 7 of 11
B
A
C
D
Hinge
4 kN/m
15 kN m
21 kN
3 m 6 m
1 kN
x(m)
V(kN)
4 kN
8
A C
13
3 m
6 kN m
4
E D B
1 1
2 m
1 m
No moment acts at the
left end so the moment
curve starts at zero.
dM/dx V, so the moment curve is
quadratic with slopes of 4 and 0.
21
19
M

E
M

A
+ M
M

A
+ area of triangle
0 + (1/2)( 4 kN)(1 m)
2 kN m
20
x(m)
M(kN m)
A E
2
1
slope 0
4

1666
8.3 Shear and Bending-Moment Diagrams Constructed by Areas Example 9, page 8 of 11
Hinge
4 kN/m
15 kN m
21 kN
3 m 6 m
1 kN
x(m)
V(kN)
4 kN
8
A
C
13
3 m
6 kN m
4
E D B
1 1
2 m
1 m
x(m)
M(kN m)
A E
2
Slope 0
M

C
M

E
+ M
M

E
+ area of triangle
2 kN m + (1/2)(8 kN)(2 m)
6 kN m
dM/dx V, so the moment curve is quadratic
with slopes of 0 and 8.
23
8
1
6
C
22
B
A
C
D

1667
8.3 Shear and Bending-Moment Diagrams Constructed by Areas Example 9, page 9 of 11
B
A
C
D
Hinge
4 kN/m
15 kN m
21 kN
1 kN
x(m)
V(kN)
4 kN
8
A C
13
6 kN m
4
E D B
1 1
1 m
x(m)
M(kN m)
A E
2
6
C
The clockwise 15 kN m couple moment causes an
upward jump in the moment curve.
24
3 m 3 m
21 ( 6 + 15)
15
2 m
6 m

1668
8.3 Shear and Bending-Moment Diagrams Constructed by Areas Example 9, page 10 of 11
dM/dx V, so moment curve is quadratic with
slopes of 13 and 1.
26
M

D
M

C
+ M
M

C
+ cross-hatched area
M

C
+ area of rectangle + area of triangle
21 + ( 1 kN)(3 m) + (1/2)( 13 ( 1))(3 m)
0
25
0
Slope 13
Slope 1
(A value of zero was expected because a
hinge is present at D.)
6 m
B
A
C
D
Hinge
4 kN/m
15 kN m
21 kN
1 kN
x(m)
V(kN)
4 kN
8
A C
13
6 kN m
4
E D B
1 1
1 m
x(m)
M(kN m)
A E
2
6
C
3 m 3 m
21
2 m

1669
8.3 Shear and Bending-Moment Diagrams Constructed by Areas Example 9, page 11 of 11
B
A
C
D
Hinge
4 kN/m
15 kN m
21 kN
1 kN
x(m)
V(kN)
4 kN
8
A C
13
6 kN m
4
E D B
1 1
1 m
x(m)
M(kN m)
A E
2
6
C
21
2 m
D
M

B
M

D
+ M
M

D
+ cross-hatched area
0 + ( 1 kN)(6 m)
6 kN m
27
The clockwise 6 kN m couple moment
at the end closes the diagram.
29
Maximum moment 21 kN m 30
dM/dx V 1 28
1
1
B
6
3 m 6 m

1670

9. Centroids and Mass Centers
1671

9.1 Centroids by Integration
1672
9.1 Centroids by Integration Procedures and Strategies, page 1 of 2
x
y
dx
y = f (x)
(x, y)
(x
el
, y
el
)
x
el
= x
Procedures and Strategies for Solving Problems
Involving Calculating Centroids by Integration
1. Determine the coordinates of the centroid by
evaluating integrals such as
xc =
For a planar area, the differential area dA is usually a
rectangular strip of finite length and differential width dx
(for a vertical strip) or dy (for a horizontal strip). Use a
vertical strip if the curve bounding the planar region is
given as a function of x, y = f(x). Use a horizontal strip
if the bounding curve is given as a function of y, x = g(y).
The integrand x
el
is the x coordinate of the centroid of the
strip. It must be expressed as a function of x for a
vertical strip and as a function of y for a horizontal strip.
x
el
dA
dA
dy
x
y
x = g(y)
(x
el
, y
el
)
(x, y)
x
el
= x/2
= g(y)/2

1673
9.1 Centroids by Integration Procedures and Strategies, page 2 of 2
dL
dx
dy
x
y
2. For a line (a wire), the area element dA is replaced by
dL = (dx)
2
+ (dy)
2
)
= 1+ (dy/dx)
2
dx
if the line is given as a function of x: y = f(x). Use
dL = (dx/dy)
2
+ 1 dy
if the line is given as function of y: x = g(y).
3. For volumes with some degree of symmetry (for example, a solid
of revolution), dA can be replaced by a circular disk of finite radius
and differential thickness.
4. Using the integral function on a scientific graphing calculator
simplifies the work and helps avoid errors.
z = f(x)
x
y
z
Radius = x
dy

1674
9.1 Centroids by Integration Problem Statement for Example 1
x
1. Locate the centroid of the plane area shown. Use a
differential element of thickness dx.
y
y = 3x
2
2 ft
12 ft

1675
9.1 Centroids by Integration Problem Statement for Example 2
x
y
y = a sin( )
2b
x
a
b
2. Locate the centroid of the plane area shown, if a = 3 m
and b = 1 m. Use a differential element of thickness dy.

1676
9.1 Centroids by Integration Problem Statement for Example 3
y
x
1 in.
3. Locate the centroid of the plane area shown.
1 in
13 in.
y = 4x
5
3x
2
+ 12x + 1

1677
9.1 Centroids by Integration Problem Statement for Example 4
xy = 1
x
y
4. Locate the centroid of the plane area shown.
0.5 m
2 m
0.5 m
2 m

1678
9.1 Centroids by Integration Problem Statement for Example 5
5. Locate the centroid of the plane area shown.
x
y =
y
y = x
2
+
x(13 x)
6
14 11x
3
6 m
2 m
1 m
4 m

1679
9.1 Centroids by Integration Problem Statement for Example 6
x = 3y
x = 4 y
2
y
x
6. Locate the centroid of the plane area shown.
1 m
3 m 1 m

1680
9.1 Centroids by Integration Problem Statement for Example 7
y
x
7. Locate the centroid of the plane area shown. Use a
differential element of thickness dx.
y =
h
b
x
h
b

1681
9.1 Centroids by Integration Problem Statement for Example 8
x = a[1 ( )
2
]
y
x
y
b
b
a
8. Locate the centroid of the plane area shown. Use a
differential element of thickness dy.

1682
9.1 Centroids by Integration Problem Statement for Example 9
9. A sign is made of 0.5 in. thick steel plate in the shape shown.
Determine the reactions at supports B and C.
x = 50 + (10) sin
Specific weight = 490
B
C
y
24
lb
ft
3
y
x
50 in.
72 in.

1683
9.1 Centroids by Integration Problem Statement for Example 10
y = 2x
2
x
y
10. Locate the centroid of the wire shown.
3 m
18 m

1684
9.1 Centroids by Integration Problem Statement for Example 11
x = 300[1 ( )
4
]
y
x
11. Locate the centroid of the wire shown.
y
200
300 mm
200 mm

1685
9.1 Centroids by Integration Problem Statement for Example 12
12. The rod is bent into the shape of a circular arc.
Determine the reactions at the support A.
3 ft
20
0.2 lb/ft
A

1686
9.1 Centroids by Integration Problem Statement for Example 13
y
x
625 ft
299 ft 299 ft
13. a) Locate the centroid of the Gateway Arch in St.
Louis, Missouri, USA. b) During the pre-dawn hours of
September 14, 1992, John C. Vincent of New Orleans,
Louisiana, USA, climbed up the outside of the Arch to the
top by using suction cups and then parachuted to the
ground. Estimate the length of his climb.
Approximate equation of centerline:
y = 639.9 ft (68.78 ft) cosh[(0.01003 ft
-1
)x]

1687
9.1 Centroids by Integration Problem Statement for Example 14
O
x
y
3 m
14. Locate the centroid of the cone shown.
z
Radius = 2 m

1688
9.1 Centroids by Integration Problem Statement for Example 15
y
x
One-eighth of a
sphere of radius "a"
a
15. Locate the centroid of the volume shown.
z

1689
9.1 Centroids by Integration Problem Statement for Example 16
(This curve is rotated about the
x-axis to generate the solid.)
16. Determine the x coordinate of the centroid of the solid
shown. The solid consists of the portion of the solid of
revolution bounded by the xz and yz planes.
z
y
x = a[1 (
z
b
)
2
]
x
a
b

1690
9.1 Centroids by Integration Problem Statement for Example 17
x
y
z
a a
b
b
17. Locate the centroid of the pyramid shown.
h

1691
9.1 Centroids by Integration Example 1, page 1 of 4
x
Definition of centroid coordinates

xc = (1)

yc = (2)
where (x
el
, y
el
) are the coordinates of the
centroid of the differential area
element dA.
1. Locate the centroid of the plane area shown. Use a
differential element of thickness dx.
1
y
el
dA
dA
x
el
dA
dA
y
y = 3x
2
2 ft
12 ft

1692
9.1 Centroids by Integration Example 1, page 2 of 4
Express the element area in terms of the coordinates of a
point (x, y) on the curve:
dA = height width (of rectangle)
or,

dA = y dx (3)
By choosing an element of width dx, we have also
implicitly chosen x to be the variable of integration.
3
(x, y)
x
x
dx = width
y = height
Locate the differential element so that it extends
from an arbitrary point (x, y) on the curve to an
opposite boundary of the crosshatched region.
y = 3x
2
2
y

1693
9.1 Centroids by Integration Example 1, page 3 of 4
Express the coordinates of the element
centroid in terms of the coordinates of the
point (x, y) on the curve. The x
coordinate of the element centroid is the
same as the x coordinate of the point on
the curve:
x
el
= x (4)
Since the centroid of the differential
element is located in the center of the
element, the y coordinate of the element
centroid is
y
el
= (5)
Since the variable of integration is x, we
now have to express dA and y
el
in terms of
x (As can be seen from Eq. 4, x
el
already is
a function of x). The point (x, y) on the
curve satisfies
y = 3x
2
(6)
Substituting the expression for y in Eq. 6
into the equations for dA (Eq. 3) and for y
el

(Eq. 5) gives
dA = y dx (Eq. 3 repeated)
= 3x
2
dx (7)
y
el
=
= (8)
Evaluate the integral in the denominator
of the equation for xc over the range (from
0 to 2):
dA = 3x
2
dx = 8 ft
2
(9)
dx
x
(x, y)
y = 3x
2
y 4
5
6
(x
el
, y
el
)
2 ft
y
2
0
y
2
2
y
3x
2
2
x

1694
9.1 Centroids by Integration Example 1, page 4 of 4
3x
2
2
Evaluate the integral in the numerator of the equation for xc
over the range from 0 to 2:
x
el
dA = x(3x
2
) dx = 12 ft
3
(10)
Evaluate the integral in the numerator of the equation for yc over
the range from 0 to 2:
y
el
dA = [ ](3x
2
) dx = 28.8 ft
3
(11)
Substitute the results given in Eqs. 9, 10, and 11 into the
definitions for xc and yc:
xc = =
12
8
= 1.5 ft Ans.
yc = =
28.8
8
= 3.6 ft Ans.
dA
x
el
dA
dA
y
el
dA
2
0
7
2
0

1695
9.1 Centroids by Integration Example 2, page 1 of 3
x
y = a sin( )
2b
2. Locate the centroid of the plane area shown, if a = 3 m
and b = 1 m. Use a differential element of thickness dy.
Definition of centroid coordinates

xc = (1)

yc = (2)
where (x
el
, y
el
) are the coordinates of the
centroid of the differential area
element dA.
x
el
dA
dA
dA
y
el
dA
x
x
y
dy
(x, y)
Express the element area in terms of the
coordinates of a point (x, y) on the curve:
dA = width height (of rectangle)
or,

dA = x dy (3)
By choosing an element of width dy, we have
also implicitly chosen y to be the variable of
integration.
Locate the differential element so that it
extends from an arbitrary point (x, y)
on the curve to an opposite boundary
of the crosshatched region.
3
2
1
y = a sin( )
2b
x
y
a
b
y
x

1696
9.1 Centroids by Integration Example 2, page 2 of 3
2
x
2b
y
a
1
2
2b
a
y
b
y
a
(x, y)
6
x
(x
el
, y
el
)
y
4 Express the coordinates of the
element centroid in terms of the
coordinates of the point (x, y) on
the curve. Since the centroid of
the differential element is
located in the center of the
element, the x coordinate of the
element centroid is:
x
el
= (4)
The y coordinate of the element
centroid is the same as the y
coordinate of the point on the
curve:
y
el
= y (5)
5
2b
3
0
y
3
a
y
x
y
a
x
y = a sin( )
2b
2b
x
Since the variable of integration is y, we now
have to express dA and x
el
in terms of y (As
can be seen from Eq. 5, y
el
already is expressed
as a function of y). The point (x, y) on the
curve satisfies
y = a sin ( )
Solving this equation for x gives
x = ( ) sin
-1
( ) (6)
Substituting the expression for x in Eq. 6 into
the equations for dA (Eq. 3) and for x
el
(Eq. 4)
gives
dA = x dy (Eq. 3 repeated)
= [( ) sin
-1
( )] dy (7)

x
el
= (Eq. 4 repeated)
= ( ) sin
-1
( )
= ( ) sin
-1
( ) (8)
2
x
Substitute a = 3 m and b = 1 m, and evaluate the integral in
the denominator of the equation for xc over the range from 0
to 3 (Use the integral function on your calculator):

dA = ( ) sin
-1
( ) dy = 1.090 m
2
(9)
dy

1697
9.1 Centroids by Integration Example 2, page 3 of 3
3
0
3
0
y
3
y
3
Substitute the results given in Eqs. 9, 10, and 11 into the definitions
for xc and yc:
xc = =
0.2841
1.090
= 0.261 m Ans.
yc = =
2.2500
1.090
= 2.06 m Ans.
And similarly evaluate the integral in the numerator of the equation for yc over the
range from 0 to 3 (Use the integral function on your calculator):
y
el
dA = y [( sin
-1
( )] dy = 2.2500 m
3
(11)
Similarly, evaluate the integral in the numerator of the equation for xc over the range
from 0 to 3 (Use the integral function on your calculator):

x
el
dA = [ ] [( sin
-1
( )] dy = 0.2841 m
3
(10)
dA
dA
x
el
dA
y
el
dA
7
sin
-1
(y/3)

1698
9.1 Centroids by Integration Example 3, page 1 of 5
y = 4x
5
3x
2
+ 12x + 1
14 y
y
y
x
1 in.
x
y = 4x
5
3x
2
+ 12x + 1
y
(x, y)
Locate the differential element so that it extends from
an arbitrary point (x, y) on the curve to an opposite
boundary of the crosshatched region.
2
Definition of centroid coordinates

xc = (1)

yc = (2)
where (x
el
, y
el
) are the coordinates of the
centroid of the differential area
element dA.
3. Locate the centroid of the plane area shown.
1
y
el
dA
dA
x
el
dA
dA
Express the element area in terms of the coordinates
of a point (x, y) on the curve:
dA = height width (of rectangle)
or,

dA = (14 y) dx (3)
By choosing an element of width dx, we have also
implicitly chosen x to be the variable of integration.
3
dx 1 in
13 in.
x
14 in.

1699
9.1 Centroids by Integration Example 3, page 2 of 5
y = 4x
5
3x
2
+ 12x + 1
Express the coordinates of the
element centroid in terms of the
coordinates of the point (x, y) on
the curve. The x coordinate of the
element centroid is the same as the
x coordinate of the point on the
curve:
x
el
= x (4)

Since the centroid of the differential
element is located in the center of
the element, the y coordinate of the
element centroid is:
y
el
= y +
= 7 + (5)
4
Since the variable of integration is x, we now have
to express dA and y
el
in terms of x (As can be seen
from Eq. 4, x
el
already is expressed as a function of
x). The point (x, y) on the curve satisfies
y = 4x
5
3x
2
+ 12x + 1 (6)
Substituting the expression for y from Eq. 6 into the
equations for dA (Eq. 3) and for y
el
(Eq. 5) gives
dA = (14 y) dx (Eq. 3 repeated)
= [14 (4x
5
3x
2
+ 12x + 1)] dx
= ( 4x
5
+ 3x
2
12x + 13) dx (7)
y
el
= 7 + (Eq. 5 repeated)
= 7 +
= (8)
x
y
x
(x, y)
4x
5
3x
2
+ 12x + 1
4x
5
3x
2
+ 12x + 15
2
2
y
14 y
(x
el
, y
el
)
dx
5
14 y
2
14 y
2
y
2
y
2
14 in

1700
9.1 Centroids by Integration Example 3, page 3 of 5
What if we had chosen a differential element dy wide instead of
dx? The figure shows that now the element area is
dA = x dy
Since dy is the variable of integration, we must express x as a
function of y. But x and y are related by
y = 4x
5
3x
2
+ 12x + 1
and this equation is difficult to invert, that is, to solve for x as a
function of y. So it is much easier to use a differential element
dx wide, because then we don't have to solve for x as a function
of y.
The general conclusion to draw is that whether we should choose
a differential element dx or dy wide depends on whether the
equation defining the boundary of the region can be more easily
written as a function of x or as a function of y.
6
dy
x
y = 4x
5
3x
2
+ 12x +1
x
y
7

1701
9.1 Centroids by Integration Example 3, page 4 of 5
y = 4x
5
3x
2
+ 12x + 1
x
1 in.
(x, y)
y
dx
And similarly evaluate the integral in the numerator of the equation for yc
over the range from 0 to 1 (Use the integral function on your calculator):
y
el
dA = [ ]( 4x
5
+ 3x
2
12x + 13) dx
= 69.849 in
3
(11)
4x
5
3x
2
+ 12x + 15
Similarly, evaluate the integral in the numerator of the equation for xc
over the range from 0 to 1 (Use the integral function on your calculator):
x
el
dA = x ( 4x
5
+ 3x
2
12x + 13) dx = 2.679 in
3
(10)
Return now to a differential element dx wide.
Evaluate the integral in the denominator of the equation for xc over
the range from 0 to 1 (Use the integral function on your calculator):
dA = ( 4x
5
+ 3x
2
12x + 13) dx = 7.333 in
2
(9)
1
0
1
0
11
10
2
8
1
0
9

1702
9.1 Centroids by Integration Example 3, page 5 of 5
Substitute the results given in Eqs. 9, 10, and 11 into the
definitions for xc and yc:
xc = =
2.679
7.333
= 0.37 in.
yc = =
69.849
7.333
= 9.52 in.
dA
12
x
el
dA
dA
y
el
dA
Ans.
Ans.

1703
9.1 Centroids by Integration Example 4, page 1 of 4
xy = 1
x
y
y
x
(x, y)
x
dy
Locate the differential element so that it
extends from an arbitrary point (x, y)
on the curve to an opposite boundary.
Definition of centroid coordinates

xc = (1)

yc = (2)
where (x
el
, y
el
) are the coordinates of the centroid of the
differential area element dA.
2
1
4. Locate the centroid of the plane area shown.
dA
y
el
dA
dA
x
el
dA
xy = 1
0.5 m
2 m
0.5 m
2 m
2 m
0.5 m
x
y

1704
9.1 Centroids by Integration Example 4, page 2 of 4
Express the element area in terms of the coordinates of a point
(x, y) on the curve:
dA = height width (of rectangle)
or,

dA = x dy (3)
By choosing an element of width dy, we have also implicitly
chosen y to be the variable of integration.
Express the coordinates of the element centroid in terms of the
coordinates of the point (x, y) on the curve. Since the centroid
of the differential element is located in the center of the element,
the x coordinate of the element centroid is
x
el
=
x
2
(4)
The y coordinate of the element is the same as the y coordinate
of the point on the curve:
y
el
= y (5)
x
x
4
dy
y
xy = 1
(x, y)
(x
el
, y
el
)
y
3

1705
9.1 Centroids by Integration Example 4, page 3 of 4
Evaluate the integral in the denominator of the equation for
xc over the range from 0.5 to 2:

dA =
1
y
dy = 1.3863 m
2
(9)
Since the variable of integration is y, we now have to
express dA and x
el
in terms of y (As can be seen from
Eq. 5, y
el
already is a function of y). The point (x, y) on
the curve satisfies
xy =1
Solving for x gives
x =
1
y
(6)
Substituting the expression for x in Eq. 6 into the
equations for dA (Eq. 3) and for x
el
(Eq. 4) gives
dA = x dy (Eq. 3 repeated)
=
1
y
dy (7)
x
el
=
x
2
(Eq. 4 repeated)
=
=
1
2y
(8)
1/y
2
5
2 m
6
0.5
2
0.5 m
y
xy = 1
dy
x

1706
9.1 Centroids by Integration Example 4, page 4 of 4
2 m
(x, 2)
y
(x, y)
xy = 1
x
10
2
0.5
0.5
2
Evaluate the integral in the numerator of the equation for yc
over the range from 0.5 to 2:
y
el
dA = y (
1
y
) dy = 1.5 m
3
(11)
Evaluate the integral in the numerator of the equation for xc
over the range from 0.5 to 2:
x
el
dA = (
1
2y
)(
1
y
) dy = 0.7500 m
3
(10)
Substitute the results given in Eqs. 9, 10, and 11 into the
definitions for xc and yc:
xc = =
0.7500
1.3863
= 0.541 m Ans.
yc = =
1.5
1.3863
= 1.082 m Ans.
dA
dA
y
el
dA
x
el
dA
9
8
7 What if we had chosen a vertical element rather
than a horizontal element? The figure below
shows that we would have to define the element
areas in two equations, one for the region where y
is constant (y = 2 m), and one for the region where
y varies. We would also have to evaluate two
integrals, one over each region. Thus using a
vertical element rather than a horizontal element
would almost double the amount of work required.
Nevertheless, it would give us the correct answer.

1707
9.1 Centroids by Integration Example 5, page 1 of 3
6 m
2 m
1 m
4 m
y2
y1
y
1
x
(x, y
2
)
(x, y
1
)
Express the element area in terms of the
coordinates of a point (x, y) on the curve:
dA = height width (of rectangle)
or,

dA = (y
2
y
1
) dx (3)
By choosing an element of width dx, we have
also implicitly chosen x to be the variable of
integration.
Locate the differential element so that it
extends from an arbitrary point on the
lower curve to a point directly above on
the upper curve.
Definition of centroid coordinates

xc = (1)

yc = (2)
where (x
el
, y
el
) are the coordinates of the
centroid of the differential area element dA.
3
2 1
5. Locate the centroid of the plane area shown.
dA
y
el
dA
dA
x
el
dA
x
y =
y
y = x
2
+
x(13 x)
6
14 11x
3
dx
x
y

1708
9.1 Centroids by Integration Example 5, page 2 of 3
y
2
+ y
1
7x
2
+ 35x 28
14 11x
x(13 x)
7x
2
+ 35x 28
[x(13 x)/6] + [x
2
+ (14 11x)/3]
12
(5x
2
9x + 28)
Since the variable of integration is x, we now have to express
dA and y
el
in terms of x (As can be seen from Eq. 4, x
el
already
is a function of x). The y coordinates of the top and bottom of
the element satisfy
y
2
=
y
1
= x
2
+
y
2
y
1
= [ ] [x
2
+ ]
= (6)
Substituting the expression for y in Eq. 6 into the equations for
dA (Eq. 3)
dA = (y
2

y
1
) dx (Eq. 3 repeated)
= dx (7)
and for y
el
(Eq. 5) gives
y
el
= (Eq. 5 repeated)

=
= (8)
(x, y
2
)
dx
(x
el
, y
el
)
x = x

el
(x, y
1
)
y
2
y
1
2
x
2
Express the coordinates of the element centroid in terms
of the coordinates of the point (x, y) on the curve. The
x coordinate of the element is the same as the x
coordinate of the point on the curve:
x
el
= x (4)
Since the centroid of the differential element is located
in the center of the element, the y coordinate of the
element centroid is average of the y coordinates of the
points on the top and bottom curves.
y
el
= (5)
4
y
2
+ y
1
y
2
6
5
6
2
6
6
3
3
14 11x x(13 x)
y
1
y
2
y
1

1709
9.1 Centroids by Integration Example 5, page 3 of 3
Evaluate the integral in the numerator of the equation for yc
over the range from 1 to 4:
y
el
dA = [ ][ ] dx
= 17.0625 m
3
(11)
Evaluate the integral in the numerator of the equation for xc over the
range from 1 to 4:
x
el
dA = x [ ] dx = 13.1250 m
3
(10)
(5x
2
9x + 28)
Substitute the results given in Eqs. 9, 10, and 11 into the
definitions for xc and yc:
xc = =
13.125
5.25
= 2.50 m Ans.
yc = =
17.0625
5.25
= 3.25 m Ans.
x
el
dA
dA
y
el
dA
dA
9
4
1
7x
2
+ 35x 28
12 6
7x
2
+ 35x 28
7x
2
+ 35x 28
Evaluate the integral in the denominator of the equation for xc over
the range from 1 to 4:
dA = dx = 5.2500 m
2
(9)
8
7
4
1
6
1
4
6
6
14 11x
y = x
2
+
3
dx
x
x(13 x)
y =
6
y
4 m
1 m

1710
9.1 Centroids by Integration Example 6, page 1 of 3
x = 3y
x = 4 y
2
y
x
Express the element area in terms of the
coordinates of a point (x, y) on the curve:
dA = width height (of rectangle)
or,

dA = (x
2
x
1
) dy (3)
By choosing an element of height dy, we
have also implicitly chosen y to be the
variable of integration.
Locate the differential element so that it
extends from an arbitrary point (x, y)
on the curve to an opposite boundary.
Definition of centroid coordinates

xc = (1)

yc = (2)
where (x
el
, y
el
) are the coordinates of
the centroid of the differential area
element dA.
3
2
1
6. Locate the centroid of the plane area shown.
dA
y
el
dA
dA
x
el
dA
(x
1
, y)
x
dy
(x
2
, y)
y
x = 4 y
2
x = 3y
1 m
3 m 1 m
y
x
1
x
2
x
1

1711
9.1 Centroids by Integration Example 6, page 2 of 3
Since the variable of integration is y, we now have to
express dA and x
el
in terms of y (As can be seen from
Eq. 5, y
el
already is a function of y). The point (x, y)
on the curve satisfies
x
1
= 3y
x
2
= 4 y
2

x
2
x
1
= (4 y
2
) (3y)
= y
2
3y + 4 (6)
Substituting the expression for x
2
x
1
in Eq. 6 into the
equations for dA (Eq. 3) gives
dA = (x
2
x
1
) dy (Eq. 3 repeated)
= ( y
2
3y + 4) dy (7)
and for x
el
(Eq. 4) gives
x
el
= (Eq. 4 repeated)
=
= (8)
2
(4 y
2
) + (3y)
x
2
x
1
y
2
x
y
2
+ 3y + 4
2
2
Express the coordinates of the element centroid in terms
of the coordinates of the point (x, y) on the curve. Since
the centroid of the differential element is located in the
center of the element, the x coordinate of the element
centroid is
x
el
= + x
1
= (4)
The y coordinate of the element is the same as the y
coordinate of the point on the curve:
y
el
= y (5)
x
2
x
1
x
2
x
1
x = 3y
y
2
2
(x
1
, y)
4
x = 4 y
2
(x
el
, y
el
)
(x
2
, y)
dy
x
2
x
1
5
x
2
x
1
x
1

1712
9.1 Centroids by Integration Example 6, page 3 of 3
Substitute the results given in Eqs. 9, 10, and 11
into the definitions for xc and yc:
xc = =
5.2667
2.1667
= 2.43 m Ans.
yc = =
0.75
2.1667
= 0.346 Ans.
Evaluate the integral in the numerator of the equation for yc over
the range from 0 to 1 (Use the integral function on your calculator):
y
el
dA = y( y
2
3y + 4) dy = 0.75 m
3
(11)
Evaluate the integral in the numerator of the equation for xc over the range
from 0 to 1 (Use the integral function on your calculator):
x
el
dA = [ ] [ y
2
3y + 4] dy = 5.2667 m
3
(10)
y
2
+ 3y + 4
Evaluate the integral in the denominator of the equation for
xc over the range from 0 to 1 (Use the integral function on
your calculator):

dA = ( y
2
3y + 4) dy = 2.1667 m
2
(9)
8
7
2
6
dA
9
x = 3y
x
el
dA
dA
y
el
dA
y
x
x = 4 y
2
1
0
0
1
0
1
1 m

1713
9.1 Centroids by Integration Example 7, page 1 of 3
h
b
x
y
y
x
Express the element area in terms of the
coordinates of a point (x, y) on the curve:
dA = height width (of rectangle)
or,

dA = y dx (3)
By choosing an element of width dx, we
have also implicitly chosen x to be the
variable of integration.
Locate the differential element so that it
extends from an arbitrary point (x, y)
on the curve to an opposite boundary.
Definition of centroid coordinates

xc = (1)

yc = (2)
where (x
el
, y
el
) are the coordinates of
the centroid of the differential area
element dA
3
2
1
7. Locate the centroid of the plane area shown. Use a
differential element of thickness dx.
dA
y
el
dA
dA
x
el
dA
(x, y)
dx
y =
h
b
x
y =
h
b
x
x
y

1714
9.1 Centroids by Integration Example 7, page 2 of 3
dx
x
(x
el
, y
el
)
y
y = x
(x, y)
y
2
y
2
h
2b
b
h
b
h
h
b
x
y
Since the variable of integration is x, we now have to express dA and
y
el
in terms of x (As can be seen from Eq. 4, x
el
already is a function
of x). The point (x, y) on the curve satisfies
y = x (6)
Substituting the expression for y in Eq. 6 into the equations for dA
(Eq. 3) and for y
el
(Eq. 5) gives
dA = ( x) dx (7)
y
el
= x (8)
Express the coordinates of the element centroid in terms of the
coordinates of the point (x, y) on the curve. The x coordinate
of the element is the same as the x coordinate of the point on
the curve:
x
el
= x (4)
Since the centroid of the differential element is located in the
center of the element, the y coordinate of the element centroid
is
y
el
= (5)
5
4

1715
9.1 Centroids by Integration Example 7, page 3 of 3
dA
x
el
dA
y
el
dA
dA
6
9
b
2
h/3
bh/2
bh
2
/6
bh/2
dx
x
y
b
Evaluate the integral in the denominator of the equation for
xc over the range from 0 to b:

dA =
h
b
x dx =
bh
2
(9)
Evaluate the integral in the numerator of the equation for
xc over the range from 0 to b:
x
el
dA = x(
h
b
x) dx = (10)
Evaluate the integral in the numerator of the equation for
yc over the range from 0 to b:
y
el
dA = (
h
2b
x)(
h
b
x) dx = (11)
bh
2 b
0
8
7
b
0
b
2
h
3
0
b
6
Substitute the results given in Eqs. 9, 10, and 11
into the definitions for xc and yc:
xc = = =
2b
3
Ans.
yc = = =
h
3
Ans.

1716
9.1 Centroids by Integration Example 8, page 1 of 3
a x
a
Express the element area in terms of the
coordinates of a point (x, y) on the curve:
dA = width height (of rectangle)
or,

dA = (a x) dy (3)
By choosing an element of width dy, we have
also implicitly chosen y to be the variable of
integration.
Locate the differential element so that it
extends from an arbitrary point (x, y) on
the curve to an opposite boundary of the
crosshatched region.
Definition of centroid coordinates

xc = (1)

yc = (2)
where (x
el
, y
el
) are the coordinates of
the centroid of the differential area
element dA.
3
2
1
8. Locate the centroid of the plane area shown. Use a
differential element of thickness dy.
dA
y
el
dA
dA
x
el
dA
(x, y)
(a, y)
dy
x = a[1 ( )
2
]
y
x
x
y
x = a[1 ( )
2
]
y
b
y
b
b
a
y
x

1717
9.1 Centroids by Integration Example 8, page 2 of 3
y
dy
a x
2
x
(a, y)
x = a[1 ( )
2
]
y
b
y
b
a + x
2
a + x
2
y
b
y
b
a[2 (y/b)
2
]
2
y
Since the variable of integration is y, we now have to express dA and
x
el
in terms of y (As can be seen from Eq. 5, y
el
already is a function
of y). The point (x, y) is on the curve so it satisfies
x = a[1 ( )
2
] (6)
Substituting the expression for x in Eq. 6 into the equation for dA
(Eq. 3) gives
dA = (a x dy (Eq. 3 repeated)
= (a a[1 ( )
2
] dy
= a( )
2
dy (7)
2
a + a[1 (y/b)
2
]
Substituting the expression for x in Eq. 6 into
the equation for x
el
(Eq. 4) gives
x
el
= (Eq. 4 repeated)
=
= (8)
Express the coordinates of the element centroid in terms of the
coordinates of the point (x, y) on the curve. Since the centroid
of the differential element is located in the center of the
element, the x coordinate of the element centroid is
x
el
= + x
= (4)
The y coordinate of the element is the same as the y coordinate
of the point on the curve:
y
el
= y (5)
(a x)
5
4
2
(x
el
, y
el
)
(a x)
(x, y)
x
a

1718
9.1 Centroids by Integration Example 8, page 3 of 3
7a
Substitute the results given in Eqs. 9, 10, and 11
into the definitions for xc and yc:
xc = = = Ans.
yc = = = Ans.
y
el
dA
x
el
dA
9
dA
dA
10
y
dy
x
y
b
3
ab
ab/3
(7/30)a
2
b
ab
2
/4
ab/3 4
3b
30
7
b
Evaluate the integral in the numerator of the equation for xc
over the range from 0 to b:
x
el
dA = [ ][a( )
2
] dy
= a
2
b (10)
Evaluate the integral in the numerator of the equation for yc over
the range from 0 to b:
y
el
dA = y[a )
2
] dy = (11)
Evaluate the integral in the denominator of the equation for x
c over the range from 0 to b:

dA = a( )
2
dy = (9)
8
6
7
[2a (a/b
2
) y
2
]
0
b
0
b
b
2
y
0
b
4
ab
2
b
y

1719
9.1 Centroids by Integration Example 9, page 1 of 3
+
x
c
9. A sign is made of 0.5 in. thick steel plate in the shape shown.
Determine the reactions at supports B and C.
x = 50 + (10) sin
Specific weight of steel = 490 lb/ft
3
Free-body diagram of plate
The weight acts through the
centroid of the plate.
Equilibrium equations for the plate
Fx = 0: Bx + Cx = 0 (1)
F
y
= 0: B
y
W = 0 (2)
M
B
= 0: (72 in.)Cx xcW = 0 (3)
Calculate the weight of the plate
by calculating the area. Also
calculate the distance xc to the
center of gravity.
B
C
C
B
Weight, W
+
+
Definition of centroid coordinate

xc = (1)

where x
el
is the coordinate of the
differential element dA.
dA
x
el
dA
1
2
3
4
y
24
Cx
B
y
Bx
y
x
x
5
x = 50 + (10) sin
y
24
50 in.
72 in.
72 in.

1720
9.1 Centroids by Integration Example 9, page 2 of 3
y
(x
el
, y
el
)
7
6
y
24
2
y
24
x = 50 + (10) sin
y
24
x
2
x
Since the variable of integration is y, we now have to express
dA and x
el
in terms of y (As can be seen from the equation of
the curve, x already is a function of y). Therefore
dA = x dy
= [50 + (10) sin dy (7)
x
el
=
x
2
=
= 25 + 5 sin (8)
50 + (10) sin( y/24)
Express the element area in terms of the coordinates of a
point (x, y) on the curve:
dA = width height (of rectangle)
or,

dA = x dy (4)
By choosing an element of width dy, we have also implicitly
chosen y to be the variable of integration.
y
Express the coordinate of the x-element centroid
in terms of the coordinates of the point (x, y) on
the curve. Since the centroid of the differential
element is located in the center of the element, the
x coordinate of the element centroid is
x
el
=
x
2
(5)
8
x
9
Locate the differential element so that it
extends from an arbitrary point (x, y)
on the curve to an opposite boundary.
(x, y)
dy

1721
9.1 Centroids by Integration Example 9, page 3 of 3
y
24
y
24
Ans.
Ans.
Ans.
xc = =
99,439
3753


= 26.5 in.
Substitute the results given in Eqs. 9 and 10 into the
definition for xc:
Thus the weight is
W = area thickness specific weight
= (3753 in
2
) (0.5 in.) (490 lb/ft
3
) (
1 ft
12 in.
)
3
= 532.1 lb
Substituting xc = 26.5 in. and W = 532.1 lb in the
equilibrium equations, Eqs. 1, 2, and 3, and solving gives
Bx = 196 lb
B
y
= 532 lb
Cx = 196 lb
Evaluate the integral in the numerator of the equation for
xc over the range from 0 to 72 (Use the integral function
on your calculator):
x
el
dA = [25 + 5 sin ][50 + (10) sin ]dy
= 99,439 in
3
(10)
12
72 in.
x
x
el
dA
dA
x = 50 + (10) sin
y
24
y
13
Evaluate the integral in the denominator of the equation
for xc over the range from 0 to 72 (Use the integral
function on your calculator):

dA = [50 + (10) sin dy = 3753 in
2

(9)
10
72
0
y
24
11
72
0

1722
9.1 Centroids by Integration Example 10, page 1 of 3
y = 2x
2
(x
el
, y
el
)
dL
dx
dy
dL
x
y
Since dy/dx is slightly easier to compute than
dx/dy, express dL in terms of dy/dx and dx:
dL = [(dx)
2
+ (dy)
2
]
= [1 + (
dy
dx
)
2
] (dx)
2
= [1 + (
dy
dx
)
2
] dx (5)
By expressing the length dL in terms of dx, we
have also implicitly chosen x to be the variable
of integration.
Locate the differential element at an
arbitrary point (x, y) on the curve and
express the centroidal coordinates of
the element in terms of x and y.
x
el
= x (3)
y
el
= y (4)
Definition of centroid coordinates


xc = (1)

yc = (2)
where (x
el
, y
el
) are the coordinates of
the centroid of the differential length
element dL.
3
2
1
10. Locate the centroid of the wire shown.
dL
y
el
dL
dL
x
el
dL
x
y
y = 2x
2
3 m
18 m
y
x

1723
9.1 Centroids by Integration Example 10, page 2 of 3
Evaluate the integral in the denominator of the equation for
xc over the range from 0 to 3:

dL = [1 + (4x)
2
] dx = 18.46 m (10)
Since the variable of integration is x, we now have to express
dL and y
el
in terms of x (As shown in Eq. 3, x
el
already is
expressed as a function of x, since x
el
= x). The point (x, y) on
the curve satisfies:
y = 2x
2
(6)
Thus

dy
dx
= 4x (7)
Substituting these expressions for y and dy/dx into the
equations for y
el
(Eq. 4) and dL (Eq. 5) gives
y
el
= y
= 2x
2
(8)
dL = [1 + (
dy
dx
)
2
] dx
= [1 + (4x)
2
] dx (9)
0
3
5
4
x
dL
(x, y)
y
y = 2x
2
3 m

1724
9.1 Centroids by Integration Example 10, page 3 of 3
Evaluate the integral in the numerator of the equation for xc over the range
from 0 to 3:
x
el
dL = x [1 + (4x)
2
] dx = 36.36 m
2
(11)
Evaluate the integral in the numerator of the equation for yc over
the range from 0 to 3:
y
el
dL = (2x
2
) [1 + (4x)
2
] dx = 163.11 m
2
(12)
Substitute the results given in Eqs. 10, 11, and 12 into the definitions for xc
and yc:
xc = =
36.36
18.46
= 1.97 m Ans.
yc = =
163.11
18.46
= 8.84 m Ans.
6
dL
y
el
dL
x
el
dL
dL
8
7
0
3
0
3

1725
9.1 Centroids by Integration Example 11, page 1 of 3
x = 300[1 ( )
4
]
y
x
dL
(x
el
, y
el
)
Definition of centroid coordinates


xc = (1)

yc = (2)
where (x
el
, y
el
) are the coordinates of the
centroid of the differential length element dL.
Locate the differential element at an arbitrary
point (x, y) on the curve and express the
centroidal coordinates of the element in terms
of x and y.
x
el
= x (3)
y
el
= y (4)
2
dL
dL
y
el
dL
x
el
dL
1
11. Locate the centroid of the wire shown.
x
y
y
200
x = 300[1 ( )
4
]
200
y
300 mm
200 mm
y
x

1726
9.1 Centroids by Integration Example 11, page 2 of 3
x = 300[1 ( )
4
]
200
y
200
y
4y
3
200
4
y
200
dy
x
y
dx
dx
dy dx
dy
dx
dy
dy
Since dx/dy is slightly easier to compute than dy/dx, express
dL in terms of dx/dy and dy:
dL = [(dx)
2
+ (dy)
2
]
= [( )
2
+ 1] (dy)
2
= [( )
2
+ 1] dy (5)
By expressing the length dL in terms of dy, we have also
implicitly chosen y to be the variable of integration.
3
dL
dx
(x
el
, y
el
)
y
dL
x
Since the variable of integration is y, we now have to
express dL and x
el
in terms of y (As shown in Eq. 4, y
el

already is expressed as a function of y, since y
el
= y). The
coordinates of the point (x, y) on the curve satisfy
x = 300[1 ( )
4
] (6)
Thus
= 300[ ]

= 7.5 10
-7
y
3
(7)
Substituting these expressions for x and dx/dy into the
equations for x
el
(Eq. 3) and dL (Eq. 5) gives
x
el
= x
= 300[1 ( )
4
] (8)
dL = [( )
2
+ 1] dy
= [( .5 10
-7
y
3
)
2
+ 1] dy (9)
4

1727
9.1 Centroids by Integration Example 11, page 3 of 3
Evaluate the integral in the numerator of the equation for yc over
the range from 0 to 200 (Use the integral function on your
calculator):
y
el
dL = y [( .5 10
-7
y
3
)
2
+ 1] dy = 54 861.7 m
2
(12)
Evaluate the integral in the numerator of the equation for xc over the
range from 0 to 200 (Use the integral function on your calculator):
x
el
dL = 300[1 ( )
4
] [( .5 10
-7
y
3
)
2
+ 1] dy
= 75 209.6 m
2
(11)
Evaluate the integral in the denominator of the equation for xc over the
range from 0 to 200 (Use the integral function on your calculator):

dL = [( .5 10
-7
y
3
)
2
+ 1] dy = 407.4 mm (10)
6
7
5
Substitute the results given in Eqs. 10, 11, and 12 into
the definitions for xc and yc:
xc = =
75 209.6
407.4
= 184.6 mm Ans.
yc = =
54 861.7
407.4
= 134.7 mm Ans.
dL
y
el
dL
x
el
dL
dL
8
y
dL
(x, y)
x
x = 300[1 ( )
4
]
200
y
y
200
200
0
200
0
200
0
200 mm

1728
9.1 Centroids by Integration Example 12, page 1 of 3
12. The rod is bent into the shape of a circular arc.
Determine the reactions at the support A.
If L represents the length
of the rod, the weight W
is (0.2 lb/ft)L.
The weight of the rod acts
through the centroid (center of
gravity), located by x
c
.
Equations of equilibrium for the rod.
Fx = 0: Ax = 0 (1)
Therefore Ax = 0 Ans.
F
y
= 0: A
y
(0.2)L = 0 (2)
M
A
= 0: M
A
(0.2 lb/ft)(L)(x
c
) = 0 (3)
The length L of the rod can be calculated without
using an integral:
L = (angle in radians) (radius)
= (180 20 )( /180) 3 ft
= 8.378 ft (4)
Substituting L = 8.378 ft into Eq. 2 and solving
gives
A
y
= 1.68 lb Ans.
3 ft
20
20
3
ft
xc
y
x
M
A
0.2 lb/ft
0.2 lb/ft
+
+
2
3
4
5
6
W
Free-body diagram of rod 1
A
y
Ax
A
+

1729
9.1 Centroids by Integration Example 12, page 2 of 3
Express the element length in terms of the polar
coordinate angle
dL = (3 ft) d (6)
To solve for the moment M
A
in Eq. 3 we have to
calculate the horizontal coordinate of the centroid:

xc = (5)

where x
el
is the horizontal coordinate of the centroid
of the differential length element dL.
x
el
dL
Locate the differential
element at an arbitrary point
on the curve.
x
(3 ft) cos
dL
A
y
8
d
3 ft
(x
el
, y
el
)
dL
20
We do not need to evaluate the integral in the
denominator of Eq. 5 for xc, since we already know
that
dL = L = 8.378 ft (8)
by Eq. 4.
Express the horizontal coordinate of the element in
terms of .
x
el
= 3 ft + (3 ft) cos (7)
11
10
9
7
3 ft
x
e1

1730
9.1 Centroids by Integration Example 12, page 3 of 3
To evaluate the integral in the numerator in the
expression for xc, we note the limits of
integration.
y
20
180
radians
x
el
dL = (3 + 3 cos )(3)d = 22.055 ft
2
(9)
Substitute the results given in Eq. 8 and 9 into the
definition of x
C
:

xc = =
22.055
8.378
= 2.632 ft (10)

Substituting this result in Eq. 3 and solving for M
A
gives
M
A
= 4.41 lbft Ans.
x
dL
x
el
dL
13
180
20
12

1731
9.1 Centroids by Integration Example 13, page 1 of 3
2
Definition of centroid coordinates

xc = (1)

yc = (2)
where (x
el
, y
el
) are the coordinates of the centroid of the
differential length element dL.
Because of symmetry about the y-axis, the centroid must lie
on the y-axis, that is,
xc = 0 Ans.
1
y
el
dL
dL
x
el
dL
dL
Approximate equation of centerline:
y = 639.9 ft (68.78 ft) cosh[(0.01003 ft
-1
)x]
y
x
625 ft
299 ft 299 ft
13. a) Locate the centroid of the Gateway Arch in St.
Louis, Missouri, USA. b) During the pre-dawn hours of
September 14, 1992, John C. Vincent of New Orleans,
Louisiana, USA, climbed up the outside of the Arch to the
top by using suction cups and then parachuted to the
ground. Estimate the length of his climb.
To avoid cluttering the equations with so many digits, define
a = 639.9 ft (3)
b = 68.78 ft (4)
c = 0.01003 ft
-1
(5)
Then the equation of the arch becomes
y = 639.9 ft (68.78 ft) cosh[(0.01003 ft
-1
)x]
= a b cosh(cx) (6)

1732
9.1 Centroids by Integration Example 13, page 2 of 3
dL
x
Since the variable of integration is x, we now have to
express dL and y
el
in terms of x. The point (x, y) on the
curve satisfies
y = a b cosh (cx) (9)
Thus
= bc sinh (cx) (10)
Substituting these expressions for y and into Eq. 7 for
y
el
and Eq. 8 for dL gives
y
el
= y
= a b cosh (cx) (11)
dL = [ 1 + ( )
2
] dx
= [1 + ( bc sinh (cx))
2
] dx (12)
Locate the differential element at a general point (x, y)
on the curve and express the y-centroidal coordinate of
the element in terms of the point on the curve.
y
el
= y (7)
3
5
dy
dx
dy
dx
dy
dx
dx
dy
dx
dy
y
Since dy/dx is easier to compute than dx/dy, express dL in
terms of dy/dx and dx:
dL = [(dx)
2
+ (dy)
2
]
= [ 1 + ( )
2
] (dx)
2


= [1 + ( )
2
] dx (8)
By expressing the length dL in terms of dx, we have also
implicitly chosen x to be the variable of integration.
dx
dL
dy
4
y
y = a b cosh (cx)

1733
9.1 Centroids by Integration Example 13, page 3 of 3
y = a b cosh (cx)
y
299 ft
x
dL
6 Evaluate the integral in the denominator of the equation for yc over the
range from 299 ft to +299 ft (Use the integral function on a calculator):
dL = [1 + ( bc sinh (cx))
2
] dx = 1,476 ft (13)
Similarly evaluating the numerator of the equation for yc gives
y
el
dL = (a b cosh (cx)) [1 + ( bc sinh (cx))
2
] dx
= 439,685 ft
2
(14)

Substitute the results given in Eqs. 13 and 14 into the definitions for yc.

yc = = = 298 ft Ans.
Since Mr. Vincent only climbed half of the total arc length, we must
divide the length given in Eq. 13 by 2:
Mr. Vincent's climb = = 738 ft Ans.
(Actually he climbed a little more than 738 ft, since he climbed on the
outside surface, not on the centerline of the cross section.)

y
el
dL
dL
299
-299
1,476
2
439,685
1,476
-299
299
299 ft

1734
9.1 Centroids by Integration Example 14, page 1 of 4
Definition of centroid coordinates

xc = (1)

yc = (2)

zc = (3)
where (x
el
, y
el
) are the coordinates of the centroid of
the differential element dV.
By symmetry, the centroid must lie on the y-axis, so
xc = 0 Ans.
zc = 0 Ans.
The centroidal coordinate yc remains to be calculated.
14. Locate the centroid of the cone shown.
z
el
dV
dV
dV
dV
y
el
dV
1
x
el
dV
z
Radius = 2 m
O
x
y
3 m

1735
9.1 Centroids by Integration Example 14, page 2 of 4
(0, y
el
, 0)
z
dy
3 The boundary of the disk
intersects the xy plane at an
arbitrary point P(x, y, 0).
The y coordinate of the
element centroid equals the
y coordinate of the point P
in the xy plane.
y
el
= y (5)
x
P(x, y, 0)
O
r
4
Use a differential element in the form of a
disk of thickness dy. The volume of the
disk is
dV = (area of circular base)
thickness of disk
= r
2
dy (4)
By choosing an element of width dy, we
have also implicitly chosen y to be the
variable of integration.
B
y
A
2
y

1736
9.1 Centroids by Integration Example 14, page 3 of 4
y
Since the variable of integration is y, we now have to
express dV and x in terms of y.
By similar triangles OCP and OBA, we have
=
Thus
x = (7)
Substituting x = into Eq. 6 gives
r = (8)
Thus the volume of the differential element in Eq. 4
becomes
dV = r
2
dy
= )
2
dy (9)
A B
2 m
7 Evaluate the integral in the denominator of the
equation for yc over the range from 0 to 3:
dV = )
2
dy = 4 m
3
(10)
0
3
P (x, y, 0)
y
z
O
x
C
dy
r
x
O
The distance from the y axis to
the boundary of the disk is x, so
r = x (6)
y
5
A
C
P
B
3 m
6
y
3
2y
3
2y
3
3
2y
2y
3
x
2
3 m
2 m
x
2y
3

1737
9.1 Centroids by Integration Example 14, page 4 of 4
Evaluate the integral in the numerator of the equation for yc:
y
el
dV = y [ )
2
] dy = 9 m
4
(11)
Using the results given by Eqs. 10 and 11 in the definition of yc yields

yc = = = 2.25 m Ans.
y
el
dV
dV
4
9
8
0
3
2y
3

1738
9.1 Centroids by Integration Example 15, page 1 of 4
Definition of centroidal coordinates
xc = (1)
yc = (2)
zc = (3)
where (x
el
, y
el
, z
el
) are the coordinates of the centroid
of the differential volume-element dV.
Because of symmetry, xc = yc = zc, so we only have
to compute one centroidal distance. Let's arbitrarily
choose to compute zc.
15. Locate the centroid of the volume shown.
y
el
dV
dV
1
x
el
dV
dV
y
el
dV
dV
z
y
x
One-eighth of a
sphere of radius "a"
a

1739
9.1 Centroids by Integration Example 15, page 2 of 4
4
y
4
The boundary of the quarter disk
intersects the yz plane at an
arbitrary point P(0, y, z).
The differential element is one-quarter of a disk of thickness dz.
The volume of the disk is
dV = (one-fourth of the area of a circle) thickness of disk
= dz (4)
Note that by choosing an element of width dz, we have also
implicitly chosen z to be the variable of integration.
3
2
(x
el
, y
el
, z
el
)
z
r
dz
x
The z coordinate of the element
centroid equals the z-coordinate of
the point P in the yz-plane.
z
el
= z (5)
P(0, y, z)
z
r
2

1740
9.1 Centroids by Integration Example 15, page 3 of 4
The distance from the z axis to the
boundary of the disk is y so
r = y (6)
a
z
y
r
z
y
5 Since the variable of integration is z, we now have to express y
and dV in terms of z. Because point P lies on a circle of radius
"a" in the yz plane, y and z must satisfy the equation of a
circle
y
2
+ z
2
= a
2
Solving this for y gives
y = a
2
z
2
(7)
Substituting y = a
2
z
2
into Eq. 6 gives
r = y

= a
2
z
2
(8)
Thus the volume of the differential element in Eq. 4 becomes
dV = dz

= dz

= dz (9)
dz
x
a
2
z
2
)
4
a
2
z
2

2
4
4
r
2
P (0, y, z)
6
[ ]

1741
9.1 Centroids by Integration Example 15, page 4 of 4
dz
y
x
Evaluate the integral in the numerator of the equation
for zc over the range from 0 to a:
z
el
dV = z[ ] dz = (11)
Using the results given by Eqs. 10 and 11 in the
definition of zc yields
zc = = =
3a
8
Ans.

Thus from symmetry,
xc = yc =
3a
8
Ans.

16 4 0
z
el
dV
9
dV
a
4
/16
a
3
/6
Evaluate the integral in the denominator of the
equation for yc over the range from 0 to a:
dV = dz = (10)
a
2
z
2
)
8
a
0
7
a
a
3
a
2
z
2
)
4 6
a
4
z
a

1742
9.1 Centroids by Integration Example 16, page 1 of 3
x = a[1 (
z
b
)
2
]
z
P(x, 0, z)
r
dx
y
The differential element is one-half of a disk of thickness dx.
The volume of the disk is
dV = (one-half of the area of a circle) thickness of disk
= dx (2)
By choosing an element of width dx, we have also implicitly
chosen x to be the variable of integration.
The boundary of the half disk intersects the xz
plane at an arbitrary point P(x, 0, z) on the
generating curve.
x
3
Definition of centroidal coordinates
xc = (1)
where x
el
is the coordinate of the centroid
of the differential volume-element dV.
2
r
2
2
dV
1
x
el
dV
x
b
a
x
x = a[1 (
z
b
)
2
]
y
z
16. Determine the x coordinate of the centroid of the solid
shown. The solid consists of the portion of the solid of
revolution bounded by the xz and yz planes.
(This curve is rotated about the
x-axis to generate the solid.)

1743
9.1 Centroids by Integration Example 16, page 2 of 3
The distance from the x-axis to the
boundary of the half-disk is z so
r = z (4)
r
z
P (x, 0, z)
5
x
z
4
(x
el
, y
el
, 0)
The x coordinate of the element centroid
equals the x coordinate of the point P in
the xy plane.
x
el
= x (3)
y
Since the variable of integration is x, we now have to
express z and dV in terms of x. Because point P lies
on the generating curve, x and z must satisfy the
equation of that curve
x = a[1 (
z
b
)
2
]
Solving this for z gives
z = b 1 (
x
a
) (5)
Substituting z = b [1 (
x
a
)] into Eq. 4 gives
r = z

= b 1 (
x
a
) (6)
Thus the volume of the differential element in Eq. 2
becomes
dV = dx
= dx
= dx (7)
x
r
2
b 1 (x/a) ]
2
b
2
x/a)
2
2
2
6

1744
9.1 Centroids by Integration Example 16, page 3 of 3
a
dx
y
Evaluate the integral in the numerator of the equation for xc over
the range from 0 to a:
x
el
dV = x[ ] dx = (9)
a
2
b
2
Using the results given by Eqs. 8 and 9 in the definition of xc
yields
xc = = =
a
3
Ans.
dV
x
el
dV
9
ab
2
/4
a
2
b
2
/12
b
2
x/a)
2 12
Evaluate the integral in the denominator of the equation for xc
over the range from 0 to a:
dV = dx = (8)
b
2
x/a)
8
x
7
2
ab
2
4
0
a
0
a
z
P(x, 0, z)

1745
9.1 Centroids by Integration Example 17, page 1 of 4
x
y
z
a a
b
b
17. Locate the centroid of the pyramid shown.
x
el
dV
y
el
dV
dV
dV
dV
z
el
dV
Definition of centroid coordinates

xc = (1)

yc = (2)

zc = (3)
where ( x
el
, y
el
) are the coordinates of the
centroid of the differential volume-element
dV.
By symmetry, the centroid must lie on the
y-axis, so
x

c = 0 Ans.
z

c = 0 Ans.
y

c remains to be calculated.
1
h

1746
9.1 Centroids by Integration Example 17, page 2 of 4
The y coordinate of the element centroid equals
the y coordinate of the point P in the xy plane.
y
el
= y (5)
5
Q(0, y, z)
z
b
a
B
O
y
dy
C
The boundary of the differential element
intersects the xy and xz planes at arbitrary points
P and Q on the sloping lines in those planes.
4
x
2 Because of symmetry, we need consider
only one-fourth of the pyramid.
A
(x
el
, y
el
, z
el
)
P(x, y, 0)
y
The differential element is a rectangular box of
thickness dy. The volume of the box is
dV

o = (area of base) thickness
= (xz) dy (4)
By choosing an element of width dy, we have also
implicitly chosen y to be the variable of integration.
3
h

1747
9.1 Centroids by Integration Example 17, page 3 of 4
y
O
z
a
C
x
h
x
h y
P(x, y, 0)
z
B
b
z Q(0, y, z)
x
O
y
h
A
y
6
A
h y
y
7
h y
h
y
h

x
a
Because the variable of integration is y, we now have
to dexpress dV in terms of y. Because point P lies on
the line AC in the xy plane, similar triangles can be
used to derive an equation relating x and y:
=

Solving for x gives
x = a(1 ) (6)
Because point Q lies on the line AB in the yz plane,
similar triangles can be used to derive an equation
relating z and y:
=

Solving for x gives
z = b(1 ) (7)
b
z

h
y
h
h y

1748
9.1 Centroids by Integration Example 17, page 4 of 4
Substituting Eqs. 6 and 7 in Eq. 4 and multiplying by
4 to get the volume of the whole pyramid (not just one
fourth) gives the volume element in terms of y.
dV = 4(dV

o)
= 4(xz)dy
= 4[a(1 )] [b(1 ] dy
= 4ab(1 )
2
dy (8)
z
y
h
x
Evaluate the integral in the denominator of the equation for yc
over the range from 0 to h:
dV = 4 ab(1 )
2
dy = (9)
Evaluate the integral in the numerator of the equation for yc:
y
el
dV = 4 y[ab(1 )
2
]dy = (10)

Using the results given by Eqs. 9 and 10 in the definition of yc
yields

yc = = =
h
4
Ans.
3
abh
2
y
el
dV
abh
2
/3
4abh/3
dV
9
8
y
h
3
4abh
h
y
h
y
h
y y
h
h
0
h
0

1749

9.2 Centroids: Method of Composite Parts
1750
9.2 Centroids: Method of Composite Parts Procedures and Strategies, page 1 of 1
X

c =
x

cW
W
=
X

c =
x

cA
A
Procedures and Strategies for Solving Problems Involving
Centroids: Method of Composite Parts
Geometric Problems
1. Divide the given region into a collection of simple shapes that
are listed in a table of geometric properties of lines, areas, or
volumes. Sometimes it is convenient to subtract one region from
another, when dividing the given region. For example, if a hole is
present in a rectangle, then a circular region should be subtracted
from a rectangular region.
2. Set up a coordinate system in which all centroidal coordinates are
expressed. Note that some coordinates may be negative.
3. Set up a table, and evaluate the formula (for area A)
Area A is replaced by L or V for lengths or volumes.
Weighted Averages
Decide what are the weights and what is to be averaged. The
weights alone go in the denominator of the weighted average
formula.

1751
9.2 Centroids: Method of Composite Parts Problem Statement for Example 1
1. Locate the center of mass of the system of particles.
y
x
A 5 kg
B 2 kg
C 4 kg
D 10 kg
4 m 2 m 3 m 5 m

1752
9.2 Centroids: Method of Composite Parts Problem Statement for Example 2
y
2 ft
3 ft
1 ft
4 ft
y
E
9 lb
O
2. Determine where a 9-lb weight should be placed on
the y axis so that the center of gravity of the system
lies at the origin O of the coordinate system.
x
7 lb
2 lb
5 lb
3 lb
A
B
C
D

1753
9.2 Centroids: Method of Composite Parts Problem Statement for Example 3
3. Determine the mass mo of the particle located at point A
so that the center of mass of the system lies at x = 2 m.
y
x
6 kg
12 kg
4 kg A
B
C
D
4 m 2 m 3 m
mo
1 m

1754
9.2 Centroids: Method of Composite Parts Problem Statement for Example 4
4. Locate the center of mass of the system of particles.
y
x
A
5 kg
B
3 kg
C
4 kg
70 m
60 m
100 m
40 m
65 m 80 m

1755
9.2 Centroids: Method of Composite Parts Problem Statement for Example 5
140 mm
80 mm
100 mm
A
B
C
D
x
y
30
5. Locate the centroid of the wire.

1756
9.2 Centroids: Method of Composite Parts Problem Statement for Example 6
6 in b
A
B
C
D
45
45
x
y
Radius of arc = 4 in.
6. The wire ABCD can rotate freely about the pin support at B. Determine
the length b for which the wire will remain horizontal.

1757
9.2 Centroids: Method of Composite Parts Problem Statement for Example 7
A
B
C
D
x
y
z
4 m
2 m
3 m
BC is a circular arc
lying in the xy plane.
7. Locate the centroid of the wire.

1758
9.2 Centroids: Method of Composite Parts Problem Statement for Example 8
8. Locate the centroid of the beam cross section shown.
70 mm 70 mm
y
x
25 mm
80 mm
20 mm
40 mm

1759
9.2 Centroids: Method of Composite Parts Problem Statement for Example 9
9. Locate the centroid of the crosshatched region.
y
4 m
2 m
x

1760
9.2 Centroids: Method of Composite Parts Problem Statement for Example 10
400 mm
160 mm
250 mm
500 mm
y
x
60 mm
10. Locate the centroid of the plate with hole.

1761
9.2 Centroids: Method of Composite Parts Problem Statement for Example 11
11. Determine distance h so that the centroid of the crosshatched
region lies at y = h.
2 in. 2 in.
y
x
h
4 in.
3 in. 3 in.

1762
9.2 Centroids: Method of Composite Parts Problem Statement for Example 12
12. Locate the centroid of the crosshatched region.
y
x
60 mm
78 mm
21 mm

1763
9.2 Centroids: Method of Composite Parts Problem Statement for Example 13
13. Determine the x coordinate of the
centroid of the crosshatched region.
y
x
15 in.
9 in. 12 in.
O
D
E F

1764
9.2 Centroids: Method of Composite Parts Problem Statement for Example 14
14. Four rolled-steel angles are welded to a plate to form a
girder with the cross section shown. Locate the centroid.
15 mm
y
x
700 mm
Area = 772 mm
2
Area = 932 mm
2
C
C
19.0 mm
25.1 mm

1765
9.2 Centroids: Method of Composite Parts Problem Statement for Example 15
15. Two rolled-steel angles and a plate are welded
together to form a beam with the cross section shown.
Locate the centroid.
y
x
15 mm
120 mm 120 mm
Area = 2430 mm
2
30.2 mm
C

1766
9.2 Centroids: Method of Composite Parts Problem Statement for Example 16
x
y
z
34 in
16. The door of an autoclave unit is composed of a curved surface
ABCD bounded by two angular sectors on the ends. The door is made
of sheet metal of uniform thickness. Locate the centroid of the door.
Neglect the irregularities in geometry near the holes.
A
B
C
D
60
28 in.

1767
9.2 Centroids: Method of Composite Parts Problem Statement for Example 17
10 in.
10 in.
24 in.
9 in.
r = 8 in.
z
x
9 in.
17. A bird bath has been made by cutting a hemispherical
cavity in the top of a wooden block. The designer is
concerned that the block may tip over. As part of a study of
the stability of the structure, you are asked to locate the center
of gravity.
y

1768
9.2 Centroids: Method of Composite Parts Problem Statement for Example 18
18. Ice cream was loosely packed into a cone, and
then a solid ball of ice cream of density 920 kg/m
3

was placed on top. Estimate the density of the
loosely packed ice cream, if the center of mass of the
entire ice-cream cone is 120 mm above the tip of the
cone.
x
y
z
130 mm
Radius of ball = 30 mm

1769
9.2 Centroids: Method of Composite Parts Problem Statement for Example 19
19. Locate the center of mass of the buoy shown. The
buoy is of uniform density.
350 mm
250 mm
500 mm
x
y
z
Hemisphere
Hemisphere
150 mm

1770
9.2 Centroids: Method of Composite Parts Problem Statement for Example 20
20. Locate the centroid of the bracket shown.
10 mm
60
60 mm
70 mm
50 mm 50 mm
40 mm
10 mm
8 mm
x
y
z
Circular arc

1771
9.2 Centroids: Method of Composite Parts Problem Statement for Example 21
21. Determine the greatest depth, h, of the circular hole of
diameter 180 mm for which the cube will remain in the
position shown. The cube is made of a material of uniform
density and is supported by a hinge A along the edge
perpendicular to the plane of the figure.
B
A
h
40
10 mm
10 mm
180 mm

1772
9.2 Method of Composite Parts Example 1, page 1 of 1
3 m 5 m
1 Definition of center of mass
X

c =

where x

c is the x coordinate of mass m.
x

cm
m
2 Set up a table.
Particle m (kg)
A
B
C
D
5
2
4
10
2
6
8
60
x

c (m) x

cm (kg m)
8
3 6
40
x

cm = 22 m = 21
3 x

c is a coordinate, not a distance,
so it can have a negative value.
4 The summation sign means add
all the numbers in the column.
5 X

c =

=
22
21
= 1.048 m Ans.
x

cm
m
1. Locate the center of mass of the system of particles.
y
x
A 5 kg
B 2 kg
C 4 kg
D 10 kg
4 m 2 m

1773
9.2 Method of Composite Parts Example 2, page 1 of 2
1 Place the 9-lb weight at an unknown distance y
from the origin.
Definition of center of gravity
Y

c =

(1)
where y

c is the coordinate of weight W.
y

cW
W
2
Set up a table.
Particle W (lb)
A
B
C
D
2
5
7
3
y

c (ft) y

cW (lb ft)
1
y

cW = 3 + 9y W = 26
3
4 Substitute the results from the table into the definition of Y

c.
Y

c =

=

26
(2)
y

cW
W
E 9
y
3
5 10
15
9y
7
15
3 + 9y
2. Determine where a 9-lb weight should be placed on
the y axis so that the center of gravity of the system
lies at the origin O of the coordinate system.
x
7 lb
2 lb
5 lb
3 lb
A
B
C
D
y
2 ft
3 ft
1 ft
4 ft
y
E
9 lb
O
x

1774
9.2 Method of Composite Parts Example 2, page 2 of 2
5 If the center of gravity is to be at the origin, then
Y

c = 0
6 Eq. 2 becomes
Yc =
and solving for y gives
y = 0.333 ft Ans.
3 + 9y
26
0
(Eq. 2 repeated)

1775
9.2 Method of Composite Parts Example 3, page 1 of 1
3. Determine the mass mo of the particle located at point A so
that the center of mass of the system lies at x = 2 m.
y
x
6 kg
12 kg 4 kg
A
B C
D
1 Definition of center of mass
X

c =

(1)
where x

c is the coordinate of mass m.
x

cm
m
2 Set up a table.
Particle m (kg)
A
B
C
D
12
4
6
x

c (m) x

cm (kg m)
12
x

cm = 32 + 4mo m = 22 + mo
3 Substitute the results from the table into Eq. 1:
X

c = = (2)
x

cm
m
4 m 2 m 3 m
2
6
4
1
8
36
Since the center of mass is to lie at x = 2 m, substitute X

c
= 2 m into Eq. 2 to get
2 =
Solving gives
mo = 2 kg
Since mass cannot be negative, this result shows that there
is no particle which can be placed at point A to make the
center of mass lie at 2 m. Ans.
22 + mo
32 + 4mo
mo
mo 4mo
22 + mo
1 m
32 + 4mo
1776
9.2 Method of Composite Parts Example 4, page 1 of 1
4. Locate the center of mass of the system of particles.
y
x
1
Definition of center of mass
X

c =

Y

c =

where (x

c, y

c) are the coordinates of mass m
x

cm
m
2 Set up a table.
Particle m (kg)
A
B
C
5
3
4
x

c (m) x

cm (kg m)
80
180
400
x

c m = 620 m = 12
70
y

c (m) y

cm (kg m)
65
40
210
325
y

cm = 375
A
5 kg
B
3 kg
C
4 kg
y

cm
m
100
60
400 160
X

c =

=
620
12
= 51.7 m Ans.
Y

c =

=
375
12
= 31.2 m Ans.
3
4
x

cm
m
y

cm
m
70 m
60 m
100 m
40 m
65 m 80 m

1777
9.2 Method of Composite Parts Example 5, page 1 of 3
5. Locate the centroid of the wire.
1
Definition of centroidal coordinates
X

c =

(1)
Y

c =

(2)
where (x

c, y

c) are the coordinates of the
centroid of line segment L.
x

cL
L
y

cL
L
140 mm
80 mm
100 mm
A
B
C
D
x
y
30

1778
9.2 Method of Composite Parts Example 5, page 2 of 3
Segment L ( mm )
AB
BC
CD
140
80
100
x

c ( mm ) x

cL ( mm
2
)
70
0
9800
x

cL = 5470 L = 320
40
y

c ( mm ) y

cL ( mm
2
)
80
25
3200
11200
y

cL = 11900
43.30
0
4330 2500
3
A
B
C
D
x
y
30
2 Locate the centroids of the individual segments.
Set up a table.
140
2
mm = 70 mm
mm = 40 mm
centroid of AB: ( 70, 80)
centroid of BC: (0, 40)
50 cos 30 = 43.30 mm
50 sin 30 = 25 mm
centroid of CD: (43.30, 25)
mm = 40 mm
mm = 50 mm
80
2
80
2
100
2

1779
9.2 Method of Composite Parts Example 5, page 3 of 3
X

c =

=

320
= 17.1 mm Ans.
Y

c =

= = 37.2 mm Ans.
4
x

cL
L
y

cL
L
5
11 900
5470
320

1780
9.2 Method of Composite Parts Example 6, page 1 of 3
6. The wire ABCD can rotate freely about the pin support at B. Determine
the length b for which the wire will remain horizontal.
6 in b
A
B
C D
45
45
1 Free body diagram
A
B
D
x
x
y
y
X

c
W, weight of entire wire acts through the center of gravity
(a distance X

c from the origin of coordinates at B)
B

x
B

y
2 Equilibrium equation
M

B
= WX

c = 0 (1)
Therefore
X

c = 0 (2)
So the length L must be chosen such that the centroidal
coordinate of the entire wire satisfies Eq. 2.
3 Definition of centroid
X

c =

(3)
where x

c is the centroidal coordinate of line segment L.
x

cL
L
+
Radius of arc = 4 in.

1781
9.2 Method of Composite Parts Example 6, page 2 of 3
r sin
(4 in.) sin
4
4
In our example, r = 4 in. and = 45 =

4
rad, so
L

c =

=

= 3.601 in.
Thus the coordinate of the centroid of the arc is
x

c = 6 in. + 3.601 in. = 9.601 in.
b
2
b
2
C
A
B
D
x
y
4 Locate the centroids of the individual segments.
5 in 5 in
L

c
Centroid of AB
Centroid of BD
Centroid of arc
5 Arc length = angle radius
= 2( ) (4 in)
= 6.283 in
Angle = 45 =

4
radians
A table of geometric properties of planar curves gives
the information shown below.

6
7
6 in.
4
Centroid Location
Circular arc segment
r sin
C
Length = 2 r
r
x
y

1782
9.2 Method of Composite Parts Example 6, page 3 of 3
110.323 b
2
/2
16.283 + b
Substitute the results from the table into the definition of the
centroidal distance X

c and set X

c = 0:
X

c =

=
or
0 =
Solving for b gives
b = 14.85 in Ans.
x

cL
L
8
110.323 b
2
/2
16.283 + b
Set up a table.
Segment L ( in.)
AB
BD
Arc
b
(6 + 4)
6.283 9.601 60.323
x

c (in.) x

cL ( in
2
)
x

c L = 110.323
L = 16.283 + b
5
b
2
/2
50
b/2
b
2
/2

1783
9.2 Method of Composite Parts Example 7, page 1 of 3
7. Locate the centroid of the wire.
1 Definition of centroidal coordinates
X

c =

Y

c =

Z

c =

where (x

c, y

c, z

c) are the centroidal coordinates of
line segment L.
x

cL
L
y

cL
L
A
B
C
D
x
y
z
z

cL
L
2 Locate the centroids of the individual segments.
3 Centroid of AB as viewed in xz plane
xz plane
4 m
2 m
3 m
BC is a circular arc
lying in the xy plane.
3 m
2 m
A
B
x
z
2
2
m = 1 m
3
2
m = 1.5 m
Centroid
4 Use the Pythagorean Theorem to calculate the length:
L

AB
= (3 m)
2
+ (2 m)
2
= 3.606 m

1784
9.2 Method of Composite Parts Example 7, page 2 of 3
Centroid of CD 5
6
A table of geometric properties of planar
curves gives the information shown below.
7 L

BC
=
= 4.712 m
8
3
2
m = 1.5 m
4
2
m = 2 m
4 m
Centroid
z
y
C
D
yz plane
9 L

CD
= (3 m)
2
+ (4 m)
2
= 5 m
Centroid of BC
C
2r
Length = r
2
r
Quarter circular segment
Centroid Location
x
y
B
C
x
y
= 1.910 m
1.910 m
r = 3 m
2(3 m)
(3 m)
3 m

1785
9.2 Method of Composite Parts Example 7, page 3 of 3
Segment
AB
BC
CD
L
3.606
4.712
5.000 0.000
1.500
x

c ( m )
1.500
1.910
0.000
y

c ( m ) z

c ( m )
2.000
1.000
x

cL ( m
2
)
5.409
0.000
y

cL ( m
2
)
9.000
7.500
0.000
z

cL ( m
2
)
10.000
3.606
L = 13.318 x

cL = 14.409 y

cL = 16.500 z

cL = 6.394
Set up a table. 10
X

c =

=

13.318
= .082 m Ans.
x

cL
L
14.409
Y

c =

=
16.500
13.318
= 1.239 m Ans.
y

cL
L
Z

c =

=

13.318
= 0.480 m Ans.
zcL
L
.394
11
12
13
1.910 0.000 9.000 0.000

1786
9.2 Method of Composite Parts Example 8, page 1 of 3
y

cA
A
2
By symmetry, X

c = 0 Ans.
70 mm 70 mm
y
x
25 mm
80 mm
3 Consider the beam cross-section to be composed of two rectangles.
20 mm
y
x
y
x
y
x
= +
40 mm
8. Locate the centroid of the beam cross section shown.
1 Definition of y coordinate of centroid.
Y

c =

where y

c is the y coordinate of composite part with area A.

1787
9.2 Method of Composite Parts Example 8, page 2 of 3
y
x
25 mm
80 mm
y
y
x
y
x
4 Area and centroid of upper rectangle
140 mm
5 Area A = (140 mm)(25 mm) = 3500 mm
2
25
2
mm = 12.5 mm
80 mm
C
y

c
6
y

c = 80 mm + 12.5 mm = 92.5 mm
7 Area and centroid of lower rectangle
x
80 mm
8 Area A = (80 mm)(40 mm) = 3200 mm
2
9
C
40 mm
y

c = 40 mm
y

c
80
2
mm = 40 mm
80
2
mm = 40 mm
25 mm
140 mm

1788
9.2 Method of Composite Parts Example 8, page 3 of 3
10 Set up a table.
Region
upper rectangle
lower rectangle
3500
3200
92.5
40
A ( mm
2
)
A = 6700
y

c ( mm ) y

cA ( mm
3
)
y

cA = 451 750
128 000
323 750
11 Y

c =

=

6700
= 67.4 mm Ans.
y

cA
A
451 750

1789
9.2 Method of Composite Parts Example 9, page 1 of 3
A
2
y

cA
Consider the shaded area to be composed of a rectangle and a semicircle.
y
4 m
2 m
x
By symmetry, X

c = 0 Ans.
y
x
y
x
y
x
=
+
3
9. Locate the centroid of the crosshatched region.
1
Definition of y coordinate of centroid.
Y

c =

where y

c is the y centroidal coordinate of the
composite part with area A.

1790
9.2 Method of Composite Parts Example 9, page 2 of 3
y
x
y
x
4
4 m
C
5 Area A = 4 m 4 m = 16 m
2
y

c
6
y

c = 2 m
7 Area and centroid of semicircle
2 m
C
4 m
4 m
8 A table of properties of planar regions
gives the information shown below.
4(2 m)
0.849 m 4r
3

3
2
(2 m)
2

Area =
r
2
= = 6.283 m
2
9 In our particular problem, r = 2 m, so the distance to the centroid of
the semicircle is
yc = 4 m + = 4 m + = 4 m + 0.849 m = 4.849 m
and

C
x
4r
3
r
y
Semicircular region
r
2
2
Area =
Area and centroid of rectangle
Centroid Location
2

1791
9.2 Method of Composite Parts Example 9, page 3 of 3
10 Set up a table.
Region
Rectangle
Semicircle
16
6.283
2
4.849
A ( mm
2
)
A = 22.283
y

c ( mm ) y

cA ( mm
3
)
y

cA = 62.466
30.466
32.000
11
y

cA
A
Y

c =

=
62.466
22.283
= 2.80 m Ans.

1792
9.2 Method of Composite Parts Example 10, page 1 of 2
x

cA
A
2
400 mm
160 mm
250 mm
500 mm
y
x
y

cA
A
y
x
Consider the shaded area to be composed of a rectangle minus a circle.
y
x
y
x
=
60 mm
10. Locate the centroid of the plate with hole.
1 Definition of y coordinate of centroid.
X

c =

Y

c =

where (x

c, y

c) are the coordinates of the centroid of the
composite part with area A.

1793
9.2 Method of Composite Parts Example 10, page 2 of 2
y
x
y
x
3 Area and centroid of rectangle
500
2
mm = 250 mm
250
2
mm = 125 mm
500 mm
C
Area A = (0.250 m)(0.500 m)

= 0.125 m
2
6
C
400 mm
160 mm
7
10
Area and centroid of circular hole
Area A = r
2


= (0.060 m)
2

= 0.011310 m
2
Area
rectangle
circular hole
0.125000
0.011310
A ( m
2
)
A = 0.113690
x

c ( m )
0.250
0.400
y

c ( m )
0.125
0.160
0.031250
0.004524
x

cA ( m
2
)
x

cA = 0.026726
0.015625
0.001810
y

cA ( m
2
)
y

cA = 0.013815
11 Set up a table.
12 X

c =

=
0.026726
0.113690
= 0.235 m Ans.
x

cA
A
Y

c =

=
0.013815
0.113690
= 0.122 m Ans.
y

cA
A
60 mm
4
5
x

c = 0.250 m
y

c = 0.125 m
8 x

c = 0.400 m
9 y

c = 0.160 m
250 mm

1794
9.2 Method of Composite Parts Example 11, page 1 of 3
11. Determine distance h so that the centroid of the crosshatched
region lies at y = h.
1
Definition of y coordinate of the centroid
Y

c =

where y

c is the y centroidal coordinate of the
composite part with area A.
y

c A
A
2
2 in. 2 in.
y
x
h
4 in.
By symmetry, X

c = 0.
3 in. 3 in.

1795
9.2 Method of Composite Parts Example 11, page 2 of 3
Consider the shaded area to be composed of two rectangles and two triangles.
y
x
y
x
y
x
= +
3
x x
+ +
y y
4
Area and centroid of rectangle 1
y
x
h
3 in. 3 in.
C
5
6
Area A = 6h
y

c =
h
2
7
y
x
h
2 in. 2 in.
8
9
Area and centroid of rectangle 2
y

c = h + 2 in.
Area A = (4 in.)(4 in.) = 16 in
2
C
h h h
h
2
4
2
in. = 2 in.
4
2
in. = 2 in.
Triangle 1 Triangle 2 Rectangle 2 Rectangle 1

1796
9.2 Method of Composite Parts Example 11, page 3 of 3
y
x
Area and centroid of triangle 1
h
y
x
4 in.
C
Area A =
1
2
(4 in.)(1 in.) = 2 in
2
10
11
12
13
C
y

c
14 Area and centroid of triangle 2
Same as triangle 1:
Area A = 2 in
2
y

c = h + 1.333 in.
Apply the formula for
the centroid of a
triangle.
4
3
in. = 1.333 in.
15
Set up table.
Region
Rectangle 1
Rectangle 2
A ( in
2
)
6h
16 h + 2 16h + 32
A = 20 + 6h y

c A = 3h
2
+ 20h + 37.332
y

c ( in )
h
2
3h
2
y

cA ( in
3
)
y

cA
A
3h
2
+ 20h + 37.332
Triangle 1
Triangle 2
2
2
h + 1.333
h + 1.333
2h + 2.666
2h + 2.666
3h
2
+ 20h + 37.332
16
17
1 in.
Y

c =

=

20 + 6h
h =

20 + 6h
y

c = h + 1.333 in.
Simplifying gives
3h
2
37.333 = 0
which has the root
h = 3.53 in. Ans.

1797
9.2 Method of Composite Parts Example 12, page 1 of 3
12. Locate the centroid of the crosshatched region.
1
Y

c = = 20 mm Ans.
We know that the centroid of a triangle lies one-third
the distance from the base to the opposite vertex.
Thus Y

c can be computed directly:
y
x
60 mm
78 mm
y
60 mm
x
C
21 mm
60 mm
3
Y

c

1798
9.2 Method of Composite Parts Example 12, page 2 of 3
Unfortunately the "one-third of the distance"
rule does not give us X

c.
y
x
C
2
3 It gives this.
h
h
3
X

c
4 But we want this.
y
x
6 Let's consider the given triangle to be composed of two right triangles, P and Q.
y
x
= +
y
x
P
Q
Thus to calculate X

c, we have to either integrate or consider
the triangle to be made up of simpler shapes and then apply
the definition of the coordinate of the centroid:
X

c =

where x

c is the x centroidal coordinate of the composite part
with area A.
x

cA
A
5

1799
9.2 Method of Composite Parts Example 12, page 3 of 3
y
x
y
x
P
Q
7
Area and centroid of region P
C
h = 21 mm
x

c
60 mm
8 Area A =
1
2
(0.021 m)(0.060 m)
= 0.000630 m
2
9 Apply the formula for the
centroid of a triangle

h
3
=
0.021
3
m = 0.007 m
x

c = 0.021 m 0.007 m = 0.014 m 10
60 mm
21 mm
57 mm
h
3
C
x

c
11
Area A =
1
2
(0.060 m)(0.057 m)
= 0.001710 m
2
15
Area and centroid of region Q
13
x

c = 0.021 m + 0.019 m = 0.040 m
Apply the formula for the centroid
of a triangle

h
3
=
0.057
3
m = 0.019 m
h
3
14
16 Set up a table.
Region
P
Q
A ( m
2
)
0.000630
0.001710
0.014
0.040
A = 0.002340
x

c ( m ) x

cA ( m
3
)
0.882 10
-5
6.840 10
-5
x

cA = 7.722 10
-5
18 X

c =


=

0.002340
= 0.033 m Ans.
A
x

cA 7.722 10
-5
78 mm
Base of triangle =
78 mm 21 mm =
57 mm
12

1800
9.2 Method of Composite Parts Example 13, page 1 of 3
13. Determine the x coordinate of the
centroid of the crosshatched region.
1
y
x
15 in.
9 in. 12 in.
y
x
We know the rule that the centroid of a triangle
is located one-third the distance (the altitude)
from a base to the opposite vertex. Thus we can
compute the three centroidal distances shown
below. But we can't compute X

c directly.
O
D
E F
O
D
E
F

3
h

F

3
h

D
h

E
h

F
X

c
Altitude for vertex D
Altitude for vertex E
Altitude for vertex F
Extension of side DE
This is the distance we want.
h

E

3
h

D
C

1801
9.2 Method of Composite Parts Example 13, page 2 of 3
x

cA
A
2
Consider the shaded region to be composed of
two right triangles, one subtracted from the
other.
y
x
3
y
x
y
x
=
P
Q
Definition of x coordinate of the centroid
X

c =

where x

c is the x centroidal coordinate of the
composite part with area A.

1802
9.2 Method of Composite Parts Example 13, page 3 of 3
10
Region
P
Q
A ( in
2
)
157.5
67.5
A = 90
x

c ( in. )
7
3
x

cA ( in
3
)
1102.5
202.5
x

cA = 900
X

c =


=
900
90
= 10 in. Ans.
A
x

cA
11
y
x
y
x
P
Q
4 Area and centroid of region P
5
6
Area A =
1
2
(15 in.)(21 in.) = 157.5 in
2
Apply the formula for the
centroid of a triangle
x

c =

3
in. = 7 in.
15 in.
x

c
9 in. + 12 in. = 21 in.
C
Area and centroid of region Q
Area A =
1
2
(15 in.)(9 in.) = 67.5 in
2
Apply the formula for the
centroid of a triangle
x

c =
9
3
in. = 3 in.
8
9
15 in.
9 in.
C
x

c
21
7
Set up a table. Note the minus sign for the area
of region Q (We are subtracting this region from
region P).

1803
9.2 Method of Composite Parts Example 14, page 1 of 2
15 mm
y
700 mm
Area = 772 mm
2
Area = 932 mm
2
C
C
19.0 mm
25.1 mm
14. Four rolled-steel angles are welded to a plate to form a
girder with the cross section shown. Locate the centroid.
1 By symmetry, X

c = 0
2 Definition of y coordinate of centroid
Y

c =


where y

c is the y centroidal coordinate of
the composite section with area A.
y

cA
A
3 Consider the cross section to be composed of
a plate and angles at the top and bottom.
y
x
y
x
y
x
y
x
= + +
Plate
Angles (top)
Angles (bottom)

1804
9.2 Method of Composite Parts Example 14, page 2 of 2
25.1 mm
C C
C C
19.0 mm
x
y
x
y
x
y
4 Area and centroid of plate
700 mm
15 mm
A = (15 mm)(700 mm) = 10500 mm
2
5
6
y

c = mm = 350 mm
7 Area and centroid of top angles
A = 932 mm
2
700 mm
8
9
Total A = 2 932 mm
2
= 1864 mm
2
y

c = 700 mm 19.0 mm = 681 mm
y

c
2
700
Area and centroid of bottom angles
A = 772 mm
2
10
11 Total A = 2 772 mm
2
= 1544 mm
2
12 y

c = 25.1 mm
Member A ( mm
2
) y

c ( mm ) y

cA ( mm
3
)
Plate
Angles (top)
Angles (bottom)
10500
1864
1544
350.00
681.00
25.10
3675000.00
1269384.00
38754.40
A = 13908
y

cA = 4983138.40
13 Set up a table.
14 Y

c =
=
= 358 mm Ans.
A
y

cA
13908
4983138.4

1805
9.2 Method of Composite Parts Example 15, page 1 of 2
1
15. Two rolled-steel angles and a plate are welded
together to form a beam with the cross section shown.
Locate the centroid.
y
x
15 mm
120 mm 120 mm
By symmetry, X

c = 0.
2 Definition of y coordinate of centroid
Y

c =
where y

c is the y centroidal coordinate of the
composite part with area A.
A
y

cA
3 Consider the section shown to be composed of two angles and a plate.
y
x
y
x
y
x
y
x
= + +
Angle 1 Angle 2 Plate
Area = 2430 mm
2
30.2 mm
C

1806
9.2 Method of Composite Parts Example 15, page 2 of 2
y

cA
A
246672
8460
30.2 mm 30.2 mm
A = 2430 mm
2
15 mm 15 mm
C C
y
x
y
x
y
x
Angle 1
Angle 2
Plate
Area and centroid of angles
4
y

c y

c
5 A = 2430 mm
2
y

c = 15 mm + 30.2 mm = 45.2 mm
6 Area and centroid of plate
15 mm
15
2
mm = 7.5 mm
120 mm 120 mm
7 A = ( 15 mm )( 240 mm ) = 3600 mm
2
y

c = 7.5 mm
8 Set up a table.
Member
Angle 1
Angle 2
Plate
2430
2430
3600
45.2
45.2
7.5
A ( mm
2
) y

c ( mm ) y

cA ( mm
3
)
A = 8460 y

cA = 246672
109836
109836
27000
9 Y

c = = = 29.2 mm Ans.

1807
9.2 Method of Composite Parts Example 16, page 1 of 5
1 By symmetry,
Z

c = = 17 in. Ans.
34 in.
2
2 Definition of centroidal coordinates
X

c =
Y

c =
where (x

c, y

c) are the centroidal coordinates of
the composite part with area A.
x

cA
A
A
y

cA
28 in.
60
D
C
B
A
34 in
z
y
x
16. The door of an autoclave unit is composed of a curved surface
ABCD bounded by two angular sectors on the ends. The door is made
of sheet metal of uniform thickness. Locate the centroid of the door.
Neglect the irregularities in geometry near the holes.

1808
9.2 Method of Composite Parts Example 16, page 2 of 5
x
y
z
x
y
z
x
y
z
x
y
= + +
P
Q
R
z
3 Consider the door to be composed of two circular sector
areas and a part of a circular cylinder.

1809
9.2 Method of Composite Parts Example 16, page 3 of 5
L

c
30
30
y

c
x

c
C
y
x
y
x
A table of properties of planar regions gives
the information shown below using the
entry for a circular sector area (Note that in
the formula equals half the angle of the arc).
4
5 In our particular problem for regions P and Q, = 60/2 = /6 radians
and r = 28 in., so
A = r
2
= ( /6)(28 in.)
2
= 410.501 in
2
L

c = = = 17.825 in.

x

c = L

c cos = (17.825 in.) cos ( /6) = 15.437 in.
y

c = L

c sin = (17.825 in.) sin ( /6) = 8.912 in.
60
2r sin
Region P (same as Q)
2(28 in.) sin ( /6)
( /6)
28 in.
Centroid Location
r
C
2r sin
y
x
Area = r
2
Circular sector region

1810
9.2 Method of Composite Parts Example 16, page 4 of 5
A table of properties of planar regions
gives the information shown below
using the entry for a circular arc
segment (Note that in the formula
equals half the angle of the arc).
In our particular problem for region R, = 60/2 = /6 radians and
r = 28 in., so
Area = arc length length of straight edge
= 2 r 34 in. = 2( /6)(28 in.) 34 in. = 996.932 in
2
Lc = = = 26.738 in.
x

c = L

c cos = (26.738 in.) cos ( /6) = 23.156 in.
y

c = L

c sin = (26.738 in.) sin ( /6) = 13.369 in.
z
x
y
Region R
6
60
28 in.
34 in. 34 in.
28 in.
z
x
y
y

c
x

c
C
L

c
7
L
/6
(28 in.) sin ( /6)
r sin
30
Circular arc segment
r sin
C
Length = 2 r
r
x
y
Centroid Location

1811
9.2 Method of Composite Parts Example 16, page 5 of 5
8
Region
P
Q
R
410.501
410.501
996.932
A ( in
2
) x

c ( in. ) y

c ( in. ) x

cA ( in
3
) y

cA ( in
3
)
8.912
8.912
13.369
A = 1817.934
x

cA = 35758.765 y

cA = 20644.754
3658.385
3658.385
13327.984
Set up a table.
15.437
15.437
23.156
6336.904
6336.904
23084.957
9 X

c = = = 19.7 in. Ans.
x

cA
A
35785.765
1817.934
Y

c = = = 11.4 in. Ans.
y

cA
A
20644.754
1817.934
10

1812
9.2 Method of Composite Parts Example 17, page 1 of 5
17. A bird bath has been made by cutting a hemispherical
cavity in the top of a wooden block. The designer is
concerned that the block may tip over. As part of a study of
the stability of the structure, you are asked to locate the center
of gravity.
1 Because the block is made of a single material of
approximately uniform density, the center of gravity
coincides with the centroid.
2 Because of symmetry, the centroid must lie on the
y-axis (which passes through the center of the block), so
X

c = 0 Ans.
Z

c = 0 Ans.
y
10 in.
10 in.
24 in.
9 in.
r = 8 in.
z
x
9 in.

1813
9.2 Method of Composite Parts Example 17, page 2 of 5
Consider the bird bath to be composed of two
simpler solid shapes, a block and a hemisphere.
5 Subtraction
y

cV
V
y
y
y
=
z z
z
x x x
3 Definition of y coordinate of centroid
Y

c =
where y

c is the y coordinate of the centroid of
the composite part with volume V.
4

1814
9.2 Method of Composite Parts Example 17, page 3 of 5
Volume Vand centroidal coordinate yc of block 6
V = (24 in.)(18 in.)(20 in.) = 8640 in
3
18 in.
x
z
24 in.
y
20 in.
y

c = in. = 12 in.
2
24

1815
9.2 Method of Composite Parts Example 17, page 4 of 5
A table of properties of solid volumes
gives the information shown below.
7
8 For our particular problem,
yc = 24 in. in.
= 21 in.
V = r
3
= (8 in.)
3
= 1072.33 in
3
2
3
2
3
y
24 in.
3
8
r = (8 in.) = 3 in.
8
3
yc
x
z
r = 8 in.
r
3
8
Volume = r
3

2
3
r
3
8
Hemisphere
z
y
x
C
r
Centroid Location

1816
9.2 Method of Composite Parts Example 17, page 5 of 5
7567.67
81161.07
V
y

cV
Y

c = = = 10.72 in. Ans. 10
y

cV = 81161.07
22518.93
103680.00
y

cV ( in
4
)
21.00
12.00
y

c ( in.)
1072.33
8640.00
V = 7567.67
V ( in
3
)
Hemisphere
Block
Solid
Set up a table. 9

1817
9.2 Method of Composite Parts Example 18, page 1 of 6
Radius of ball = 30 mm
Because of symmetry,
X

c = 0 Ans.
Z

c = 0 Ans.
1
Definition of y-coordinate of center of mass
Y

c =
where y

c is the y-coordinate of the center of
mass of the composite part with mass m.
2
y

cm
m
(1)
18. Ice cream was loosely packed into a cone, and
then a solid ball of ice cream of density 920 kg/m
3

was placed on top. Estimate the density of the
loosely packed ice cream, if the center of mass of the
entire ice-cream cone is 120 mm above the tip of the
cone.
x
y
z
130 mm

1818
9.2 Method of Composite Parts Example 18, page 2 of 6
3 Consider the ice-cream cone to be composed of
the sum of three solids of simpler shape.
z
x
y
z
x
y
=
+
Sphere
Cone
(Solid ice
cream)
Spherical cap
(Loosely
packed
ice
cream)
(Loosely
packed
ice
cream)
y
x
z
z
y
x

1819
9.2 Method of Composite Parts Example 18, page 3 of 6
y
x
4 Volume of sphere
Volume V = r
3
= (0.03 m)
3
= 1.13097 10
-4
m
3
5
4
3
4
3
C
CD = radius = 0.03 m
0.13 m
yc
O
B
D
By similar
triangles,
angles are
equal.
6
From triangle BCD,
BD = (0.03 m) cos
From triangle OBD,
BD = (0.13 m) tan
Thus must satisfy
(0.03 m) cos = (0.13 m) tan
Solving numerically gives
= 12.688 (2)
and so
yc = 0.13 m + (0.03 m) sin 12.688
= 0.13659 m (3)
7
Geometric relations between sphere and cone
Calculate the centroidal coordinate
of the center of the sphere.

1820
9.2 Method of Composite Parts Example 18, page 4 of 6
y
x
C
0.03 m
0.13 m
O
B
D
= 12.688
r
yc
A table of properties of solid volumes gives the
information shown below.
8
h
4
3
(0.03 m) cos 12.688
2
(0.13 m)
3
r
2
h
Volume of cone V =
=
= 1.16611 10
-4
m
3
h
4
9
10
For our problem,
yc = h

= 3h/4
= 3(0.13 m)/4
= 0.09750 m
G
Cone
h
4
h
z
y
C
r
Volume =
r
2
h
3
x
Centroid Location

1821
9.2 Method of Composite Parts Example 18, page 5 of 6
y
x
C
D
R
yc
x
3
R
3
(2 3 sin + 3 sin
3
V =
A table of properties of solid volumes gives the
information shown below.
11
G
z
Spherical cap
G
3R(1+ sin )
2
4(2+ sin )
y
1
R
z
0.13 m
4(2+ sin )
3R(1+ sin )
2
For our problem,
yc = 0.13 m R sin
0.12151 m
3R(1+ sin )
2
4(2+ sin )
= 12.688, by Eq. 2 R = 0.03 m
12
1
V =
= 0.38817 10
-4
m
3
R
3
(2 3 sin + 3 sin
3

3
0.03 m 12.688

1822
9.2 Method of Composite Parts Example 18, page 6 of 6
Solid V ( m
3
) Density, ( kg/m
3
) m = V ( kg )
m = 0.10405 + 7.7794 10
-5
c
y

c ( m ) y

cm ( kg m )
y

cm = 0.01421 + 0.66529 10
-5
c
Set up a table. 13
14 Mass = density volume
15 Y

c =
0.12 =
Solving gives
c = 643 kg/m
3
Ans.
y

cm
m
Sphere 1.13097 10
-4
920 0.10405 0.13659 0.01421
Cone 1.16611 10
-4
c 1.16611 10
-4
c 0.09750

Cap 0.38817 10
-4
c 0.38817 10
-4
c 0.12151 0.47167 10
-5
c
Center of mass is to lie 0.12 m
above the tip of the cone.
1.13696 10
-5
c
0.01421 + 0.66529 10
-5
c
0.10405 + 7.7794 10
-5
c

1823
9.2 Method of Composite Parts Example 19, page 1 of 9
19. Locate the center of mass of the buoy shown.
The buoy is of uniform density.
Definition of y coordinate of center of mass
Y

c =
where y

c is the y coordinate of the center of
mass of the composite part with mass m.
350 mm
250 mm
500 mm
x
y
z
Hemisphere
Hemisphere
y

cV
V
150 mm
1 Because of symmetry,
X

c = 0 Ans.
Z

c = 0 Ans.

1824
9.2 Method of Composite Parts Example 19, page 2 of 9
Consider the buoy to be composed of a
sum of solids that have simple shapes.
x
y
z
2
z
y
x
z
y
x
z
y
x
z
y
x
z
y
x
Cylinder
Large cone
Small cone
= + +
+
3 The frustum of a cone can be considered to be
a small cone subtracted from a large cone.
Bottom
hemisphere
Top
hemisphere

1825
9.2 Method of Composite Parts Example 19, page 3 of 9
150 mm
x
y
z
For our particular problem,
y
c
= 1100 mm + r
= 1100 mm + (150 mm)
= 1156.25 mm
Volume V = r
3
= (150 mm)
3
= 7.069 10
6
mm
3
z
x
y
r = 150 mm
r
8
3
500 mm + 250 mm + 350 mm
= 1100 mm
5
1100 mm
2
3
A table of properties of solid volumes
gives the information shown below.
4
3
8
8
3
yc
Top hemisphere
3
2
3
2
Volume = r
3

2
3
r
3
8
Hemisphere
z
y
x
C
r
Centroid Location

1826
9.2 Method of Composite Parts Example 19, page 4 of 9
r = 150 mm
x
yc = 750 mm + 175 mm
= 925 mm
V = area of base height of cylinder
= r
2
height
= (150 mm)
2
(350 mm)
= 24.740 10
6
mm
3
y
x
z
y
6
750 mm
175 mm
175 mm
350 mm
500 mm + 250 mm
= 750 mm
y

c
Cylinder

1827
9.2 Method of Composite Parts Example 19, page 5 of 9
x
y
z
Before we can calculate the volume of the
large cone, we must first find its altitude.
7
z
y
x
150 mm
500 mm
250 mm
A
B
C
D
E
250 mm
150 mm
500 mm
8 Triangles ABC and ADE are similar, so
=
Solving gives
s = 107.143 mm
s
150 mm
s + 250 mm
500 mm
s

1828
9.2 Method of Composite Parts Example 19, page 6 of 9
x
z
y
yc = 500 mm +
= 500 mm +
= 589.286 mm
V = ( r
2
h)/3
= [ (500 mm)
2
(357.143 mm)]/3
= 93.500 10
6
mm
3
250 mm
z
500 mm
x
y
r = 500 mm
10
A table of properties of solid volumes
gives the information shown below.
9
4
357.143 mm
4
h
y

c
4
h
G
500 mm
Large cone Cone
h
4
h
z
y
C
r
Volume =
r
2
h
3
x
Centroid Location
s = 107.143 mm
107.143 mm + 250 mm
= 357.143 mm
h = 357.143 mm

1829
9.2 Method of Composite Parts Example 19, page 7 of 9
12
z
y
x
z
y
500 mm + 250 mm
= 750 mm
750 mm
r = 150 mm
11 Using the same table of properties of solid volumes as was
used for the large cone gives the information shown below.
4
h
h
4
107.143 mm
4
yc = 750 mm +
= 750 mm +
= 776.786 mm
V = ( r
2
h)/3
= [ (150 mm)
2
(107.143 mm)]/3
= 2.524 10
6
mm
3
y

c
h
4
h = 107.143 mm
Small cone
Cone
h
4
h
z
y
C
r
Volume =
r
2
h
3
x
Centroid Location
107.143 mm

1830
9.2 Method of Composite Parts Example 19, page 8 of 9
x
z
y
500 mm
y
x
z
r = 500 mm

13 A table of properties of solid volumes
gives the information shown below.
8
3
r
2
3
2
3
3
8
8
3
For our particular problem,
y
c
= 500 mm r
= 500 mm (500 mm)
= 312.5 mm
V = r
3
= (500 mm)
3
= 261.799 10
6
mm
3
G
14
y

c
Bottom hemisphere
Volume = r
3

2
3
Volume = r
3

2
3
r
3
8
Hemisphere
z
y
x
C
r
Centroid Location

1831
9.2 Method of Composite Parts Example 19, page 9 of 9
15 Set up a table.
Volume, V ( mm
3
)
7.069 10
6
Solid
Top hemisphere
Cylinder
Large cone
Small cone
Bottom hemisphere
24.740 10
6
93.500 10
6
2.524 10
6
261.799 10
6
V = 384.584 10
6

y

c ( mm ) y

cV ( mm
4
)
1156.250
925.000
589.286
776.786
312.500
8.173 10
9
22.885 10
9
55.098 10
9
1.961 10
9
81.812 10
9
y

cV = 166.007 10
9
Y

c = = = 432 mm Ans.
y

cV
V
384.584 10
6

166.007 10
9
16

1832
9.2 Method of Composite Parts Example 20, page 1 of 9
10 mm
60
60 mm
70 mm
50 mm 50 mm
40 mm
10 mm
8 mm
x
y
z
Definition of centroidal coordinates
X

c =
Y

c =
Z

c =
where (x

c, y

c, z

c) are the coordinates of the
centroid of the composite part with volume V.
1
y

cV
V
x

cV
V
z

cV
V
Circular arc
20. Locate the centroid of the bracket shown.

1833
9.2 Method of Composite Parts Example 20, page 2 of 9
x
y
z
x
z
y
x
z
y
x
y
z
x
y
z
x
y
z
= + +
+
2
3
4
The bottom plate extends to the
far left side of the bracket.
The plate and hole can be considered to be a
solid cylinder subtracted from a uniform plate.
P
Q
R
S
T
Consider the bracket to be composed of a sum of solids that have simple shapes.

1834
9.2 Method of Composite Parts Example 20, page 3 of 9
Calculate dimensions of solids P, Q, and R.
x
y
z
y
60 mm
60 mm 35.0 mm = 25.0 mm
70 mm
(70 mm) cos 60 = 35.0 mm
70 mm + 25.0 mm = 95.0 mm
z
6
7
5
8
9
(70 mm) sin 60 = 60.622 mm
70 mm
R
Q
P
10 mm
25.0 mm
x

c =
= 5 mm
10 mm
2
60
10 Volume and centroidal coordinate of solid P.
C
25 mm
y
z
x
z

c =
= 12.5 mm
25 mm
2
y

c =
= 30.311 mm
60.622 mm
2
Volume V = (10 mm)(60.622 mm)(25 mm) = 1.516 10
4
mm
3
60.622 mm
11 View of solid P as seen from the positive x axis
C
60.622 mm

1835
9.2 Method of Composite Parts Example 20, page 4 of 9
x
y
z
10 mm
x

c =
= 5 mm
10 mm
2
Volume and centroidal coordinate of solid Q.
C
y
z
x
z

c = 25.0 mm + 11.667 mm = 36.667 mm
y

c = 60.622 mm 20.207 mm = 40.415 mm
Volume V = area of triangle thickness
= (60.622 mm)(35.0 mm)(10 mm)
= 1.061 10
4
mm
3
25 mm
35 mm
60.622 mm
C
z

c
y

c
60.622 mm
3
= 20.207 mm
35.0 mm
3
= 11.667 mm
35.0 mm 25.0 mm
Apply formula to locate centroid of triangle at
.
h
3
12
13
1
2
14
View of solid Q as seen
from the positive x axis
15
60.622 mm

1836
9.2 Method of Composite Parts Example 20, page 5 of 9
x
y
z
y
z
Radius = 70 mm 10 mm
25.0 mm
x

c =
= 5 mm
10 mm
2
Volume and centroidal coordinate of solid R.
C
70 mm
60 25 mm
z

c
y

c
L

c
16
C
17 View of solid R as seen from the positive x axis
60
2
= 30

1837
9.2 Method of Composite Parts Example 20, page 6 of 9
30
6
6
18
19
Centroid Location
r
C
2r sin
y
x
Area = r
2
Circular sector region
A table of properties of planar regions gives
the information below.

2(70 mm) sin
6
6
For our particular problem,
= = radians
r = 70 mm
Thus
Area = (70 mm)
2
= 2566 mm
2
Lc = = 44.563 mm
yc = Lc sin = (44.563 mm) sin 30 = 22.282 mm
zc = 25 mm + Lc cos
= 25 mm + (44.563 mm) cos 30
= 63.593 mm
Volume = area thickness
= (2566 mm
2
)(10 mm)
= 2.566 10
4
mm
3
3
y
z
Radius = 70 mm
25 mm
z

c
y

c
L

c
C
60
2
= 30

1838
9.2 Method of Composite Parts Example 20, page 7 of 9
x
y
z
Volume and centroidal coordinate of solid S (Recall that the
bottom plate extends to the far left side of the bracket.)
100 mm
95.0 mm
10 mm
x
z
= 47.5 mm
xc = 55 mm 10 mm = 45 mm
= 55 mm
100 mm + 10 mm
2
10 mm
10 mm
2
95.0 mm
2
y
Volume = (10 mm)(110 mm)(95.0 mm)
= 10.450 10
4
mm
3
C
C
10 mm
20
21
10 mm
zc =
yc = = 5 mm

1839
9.2 Method of Composite Parts Example 20, page 8 of 9
x
y
z
Volume and centroidal coordinate of solid T.
10 mm
x
z
10 mm
2
= 5 mm
y
(x

c, y

c, z c) = (50 mm, 5 mm, 40 mm)
Volume = area of circle height of cylinder
= (8 mm)
2
(10 mm)
= 2.011 10
3
mm
3
50 mm 50 mm
40 mm
8 mm
8 mm
50 mm
40 mm
22
23
10 mm


1840
9.2 Method of Composite Parts Example 20, page 9 of 9
Set up a table.
Volume, V ( mm
3
)
1.516 10
4
Solid
P
Q
R
S
T
1.061 10
4
2.566 10
4
10.450 10
4
201 10
4
V = 15.392 10
4
y

c ( mm ) x

cV ( mm
4
)
30.311
40.415
22.282
0.758 10
5

0.531 10
5

1.283 10
5
47.025 10
5

1.005 10
5
x

cV = 43.448 10
5

Y

c = = = 6.2 mm Ans.
y

cV
V
15.392 10
4
x

c ( mm )
5
5
5
45
50
5.000
5.000
z

c ( mm )
12.500
36.667
63.593
47.500
40.000
4.595 10
5
y

cV = 9.477 10
5

0.101 10
5

5.225 10
5
5.718 10
5

4.288 10
5
y

cV ( mm
4
) z

cV ( mm
4
)
1.895 10
5
3.891 10
5
16.318 10
5

49.638 10
5

0.840 10
5
z

cV = 70.902 10
5

43.448 10
5

15.392 10
4
X

c = = = 28.2 mm Ans.
V
x

cV
15.392 10
4
Z

c = = = 46.1 mm Ans.
V
z

cV
24
25
9.477 10
5

70.902 10
5


1841
9.2 Method of Composite Parts Example 21, page 1 of 4
21. Determine the greatest depth, h, of the circular hole of
diameter 180 mm for which the cube will remain in the
position shown. The cube is made of a material of uniform
density and is supported by a hinge A along the edge
perpendicular to the plane of the figure.
B
A
h
40
10 mm
10 mm
180 mm

1842
9.2 Method of Composite Parts Example 21, page 2 of 4
A
O
C
y
3
Geometry
= 100 mm
200 mm
2
4 Now the centroidal distance, Y

c, can be
calculated independent of distance h:
Yc = (100 mm) tan 50
= 119.175 mm (1)
Y

c
90 40 = 50
40
The center of gravity (same
as the centroid, C) lies
directly above point A.
1 Free-body diagram of cube with hole
O
(The reaction at
the roller is zero,
if the cube is just
about to tip.)
C
y
x
A

y
A

x
Moment equilibrium:
M

A
= 0: Wb = 0
Therefore b = 0. That is, the line of
action of the weight, W, must pass
through the corner of the cube, A.
2
40
B
A
h
b
The line of action of the
weight, W, of the cube
passes through the
centroid, C.
W
R
B
= 0
x
(one half of length of side of cube)
+

1843
9.2 Method of Composite Parts Example 21, page 3 of 4
5 Next, we have to derive an expression for Y

c in terms
of the depth h of the hole.
Consider the cube with a hole to consist of a solid
cylinder subtracted from a solid cube.
y y
y
x x x
z z
z
6
Volume V = (200 mm)
3
= 8 10
6
mm
3
Centroidal height yc = (200 mm)/2
= 100 mm
Volume V = (90 mm)
2
h
= (8100 h) mm
3
Centroidal height yc = h/2
=
Radius = 90 mm
h
200 mm

1844
9.2 Method of Composite Parts Example 21, page 4 of 4
Set up a table.
Volume, V ( mm
3
)
8 10
6
Solid
Cube
Cylinder
8100 h
V = 8 10
6
8100 h
Y

c = = (2)
y

cV
V
8 10
6
8100 h
8 10
8
4050 h
2
y

c ( mm )
100 8 10
8
y

cV = 8 10
8
4050 h
2

4050 h
2
y

cV ( mm
4
)
7
h
2
Substituting Yc = 119.175 mm from Eq. 1 into the left-hand side of Eq. 2 gives
119.175 =
Solving gives
h = 72.8 mm Ans.
8 10
8
4050 h
2
8 10
6
8100 h

1845

9.3 Theorems of Pappus and Guldinus
1846
9.3 Theorems of Pappus and Guldinus Procedures and Strategies, page 1 of 2
Procedures and Strategies for Solving Problems Involving
the Theorems of Pappus and Guldinus
To calculate the area of a solid of revolution,
1. identify the axis of revolution,
2. sketch the generating curve,
3. find the distance r
C
from the axis of revolution to the
centroid of the curve, and
4. Apply the formula for the area:
A = 2 r
C
L
where L is the length of the curve.
To calculate the volume of a solid of revolution,
1. identify the axis of revolution,
2. sketch the generating area,
3. find the distance r
C
from the axis of revolution to the
centroid of the area, and
4. apply the formula for the volume:
V = 2 r
C
A
where A is the generating area.
a
a
r
C
Centroid of curve
b
Centroid of region
r
C
b
Planar region of area A
Curve of length L
a
a
b
b

1847
9.3 Theorems of Pappus and Guldinus Procedures and Strategies, page 2 of 2
Notes:
1) If the curve or area is rotated an amount less than a complete
revolution, then the factor of 2 in the equations for the volume
and area must be replaced by (expressed in radians).
2) If the centroidal distance cannot be found from a table, then you
will have to calculate the distance by evaluating an integral. But you
can save work by noting that you do not have to calculate r
C
and L
independently you need calculate only the product r
C
L and that
can be found by noting that
x
C
=
implies
x
C
L = x
el
dL
So you only have to evaluate one integral, namely
x
el
dL
x
el
dL
dL
L

1848
9.3 Theorems of Pappus and Guldinus Problem Statement for Example 1
10 ft
y
x
3 ft
1. Determine the amount of paint required to paint
the inside and outside surfaces of the cone, if one
gallon of paint covers 300 ft
2
.

1849
9.3 Theorems of Pappus and Guldinus Problem Statement for Example 2
10 ft
y
x
3 ft
2. Determine the volume of the cone.

1850
9.3 Theorems of Pappus and Guldinus Problem Statement for Example 3
4 m
x
y
z
1 m
C
3. Determine the area of the
half-torus (half of a doughnut).

1851
9.3 Theorems of Pappus and Guldinus Problem Statement for Example 4
4. Determine the volume of the
half-torus (half of a doughnut).
4 m
x
y
z
1 m
C

1852
9.3 Theorems of Pappus and Guldinus Problem Statement for Example 5
2 m
3 m
x
y
O
5. Determine the area of the frustum of the cone.
4 m

1853
9.3 Theorems of Pappus and Guldinus Problem Statement for Example 6
6. Determine the volume of the frustum of the cone.
2 m
3 m
x
y
O
4 m

1854
9.3 Theorems of Pappus and Guldinus Problem Statement for Example 7
y
x
C
r
C
R
7. Determine the centroidal coordinate rc of a
semicircular arc of radius R, given that the area of a
sphere of radius R is known to be 4 R
2
.

1855
9.3 Theorems of Pappus and Guldinus Problem Statement for Example 8
y
x
C
r
C
Radius = R
8. Determine the centroidal coordinate rc of a
semicircular area of radius R, given that the volume of a
sphere is known to be (4/3) R
3
.

1856
9.3 Theorems of Pappus and Guldinus Problem Statement for Example 9
20 m
1 m 3 m
2 m
40
9. A concrete dam is to be constructed in the shape
shown. Determine the volume of concrete that
would be required.
3.5 m

1857
9.3 Theorems of Pappus and Guldinus Problem Statement for Example 10
10. The concrete steps shown are in the shape of a quarter
circle. Determine the amount of paint required to paint
the steps, if one liter of paint covers 1.5 m
2
.
190 mm
190 mm
260 mm
260 mm
y
x
z

1858
9.3 Theorems of Pappus and Guldinus Problem Statement for Example 11
11. The concrete steps shown are in the shape of a quarter circle.
Determine the total number of cubic meters of concrete required to
construct the steps.
190 mm
190 mm
260 mm
260 mm
y
x
z

1859
9.3 Theorems of Pappus and Guldinus Problem Statement for Example 12
20 mm
Front view Side view
12. Determine the mass of the steel V-belt pulley
shown. The density of the steel is 7840 kg/m
3
.
100 mm
15 mm
15 mm
70 mm
4 mm 4 mm
12 mm 10 mm 12 mm

1860
9.3 Theorems of Pappus and Guldinus Problem Statement for Example 13
x
y
1 m
z
13. Determine the area of the surface of
revolution generated by rotating the curve
y = z
4
, 0 z 1 m, about the z axis.

1861
9.3 Theorems of Pappus and Guldinus Problem Statement for Example 14
x
y
1 m
z
14. Determine the volume of the solid of
revolution generated by rotating the curve
y = z
4
, 0 z 1 m, about the z axis.

1862
9.3 Theorems of Pappus and Guldinus Problem Statement for Example 15
Radius = 20 mm
7 mm
15. A pharmaceutical company plans to put a coating
0.01 mm thick on the outside of the pill shown.
Determine the amount of coating material required.
1.5 mm

1863
9.3 Theorems of Pappus and Guldinus Problem Statement for Example 16
10 mm
5 mm
70 mm
60 mm
16. Determine the volume of the funnel.
5 mm

1864
9.3 Theorems of Pappus and Guldinus Problem Statement for Example 17
0.2 m
Signals parallel
to axis of dish
17. A satellite dish is shaped in the form of a paraboloid of
revolution to take advantage of the geometrical fact that all
signals traveling parallel to the axis of the paraboloid are
reflected through the focus. Determine the amount, in m
2
,
of reflecting material required to cover the inside surface of
the dish.
0.3 m
0.3 m

1865
9.3 Theorems of Pappus and Guldinus Problem Statement for Example 18
90 mm
80 mm
18. Determine the amount of coffee that
the coffee mug holds when full to the
brim. The radius of the rounded corners
and the rim is 15 mm.

1866
9.3 Theorems of Pappus and Guldinus Problem Statement for Example 19
Radius = 20 mm
19. Determine the capacity of the small bottle of lotion if the
bottle is filled half way up the neck.
17.5 mm
5 mm
15 mm

1867
9.3 Theorems of Pappus and Guldinus Example 1, page 1 of 2
10 ft
y
x
3 ft
3 ft
10 ft
C
= 1.5 ft
3 ft
2
x
y
1. Determine the amount of paint required to paint
the inside and outside surfaces of the cone, if one
gallon of paint covers 300 ft
2
.
1 The y axis is the axis of revolution.
The generating curve is a
straight line through the
origin.
2
The distance to the centroid of the line is
r
C
= 1.5 ft
3

1868
9.3 Theorems of Pappus and Guldinus Example 1, page 2 of 2
3 ft
10 ft
4 The length of the curve can be found from the Pythagorean theorem:
L = (10 ft)
2
+ (3 ft)
2
= 10.4403 ft
Applying the first theorem of Pappus-Guldinus gives the area:
A = 2 rcL
= 2 (1.5 ft)(10.4403 ft)
= 98.3975 ft
2
Calculate the volume of paint required:

Volume of paint = 2(98.3975 ft
2
)( )
= 0.656 gal Ans.
Because both the inside and outside surfaces must be painted, the value of
the computed area must be doubled.

1 gal
300 ft
2
L

1869
9.3 Theorems of Pappus and Guldinus Example 2, page 1 of 1
10 ft
y
x
3 ft
3 ft
10 ft
C
x
y
3 ft
3
rc = = 1 ft
2
1 The y axis is the axis of rotation.
The generating area is
a triangle
3 The centroid of a
triangle is located
one-third of the
distance from the base
to the opposite vertes.
4 The area of the triangle is
A = (1/2)(10 ft)(3 ft)
= 15 ft
2
5 Applying the second theorem of
Pappus-Guldinus gives the volume:
V = 2 rcA
= 2 ft)( 15 ft
2
)
= 94.2 ft
3
Ans.
2. Determine the volume of the cone.

1870
9.3 Theorems of Pappus and Guldinus Example 3, page 1 of 1
4 m
x
y
z
1 m
4 m
x
y
1 m
C
C
3. Determine the area of the
half-torus (half of a doughnut). 1 The axis of revolution is the x axis.
2 The generating curve
is a circle.
3 The distance to the centroid
is rc = 4 m.
4 The length of
the curve is the
circumference
of the circle:
L = 2 (1m)
= 6.2832 m
5 Applying the first theorem of
Pappus-Guldinus gives the area:
A = rcL
= (4 m)(6.2832 m)
= 79.0 m
2
Ans.
The angle of revolution is , not 2 , because
the figure is a half -torus.

1871
9.3 Theorems of Pappus and Guldinus Example 4, page 1 of 1
4 m
x
y
1 m
C
4. Determine the volume of the
half-torus (half of a doughnut).
1 The axis of revolution is the x axis
2 The generating area is the
region bounded by a circle
3 The distance to the centroid is rc = 4m
4 The area bounded by
the circle is
A = (1 m)
2

= m
2
5 Applying the second theorem of
Pappus-Guldinus gives the volume:
V = rcA
= (4 m)( m
2
)
= 39.5 m
3
Ans.
The angle of revolution is , not 2 , because
the figure is a half -torus.
4 m
x
y
z
1 m
C

1872
9.3 Theorems of Pappus and Guldinus Example 5, page 1 of 2
2 m
3 m
x
y
O
O
D
C
B
4 m
2 m
d
3 m
2 m
3 m
x
y
O
B
D
d
5. Determine the area of the frustum of the cone.
1 The y axis is the axis of rotation
2 The generating curve is the
straight line BD. The
horizontal coordinate d of
the lower end of the line
can be found by similar
triangles:
d
2 m
3 m
2 m + 4 m
=
Solving gives
d = 1 m
4 m
4 m

1873
9.3 Theorems of Pappus and Guldinus Example 5, page 2 of 2
2 m
3 m
x
y
3 m
x
y
O
B
C
D
1 m
rc = (1 m + 3 m)/2
= 2 m
B
C
D
3 m 1 m = 2 m
4 m
O
3 The distance to
the centroid is
4 The length of the generating curve BD is
L = (2 m)
2
+ (4 m)
2

= 4.4721 m
5 Applying the first theorem of Pappus-Guldinus gives the area:
A = 2 rcL
= 2 (2 m)(4.4721 m)
= 56.2 m
2
Ans.
4 m
B
D

1874
9.3 Theorems of Pappus and Guldinus Example 6, page 1 of 3
6. Determine the volume of the frustum of the cone.
2 m
3 m
x
y
O
4 m
O
D
B
4 m
2 m
d
3 m
1 The y axis is the axis of rotation
2 The generating area is the
trapezoid BDEF. The
horizontal coordinate d of
the lower end of the line
BD can be found by similar
triangles:
d
2 m
3 m
2 m + 4 m
=
Solving gives
d = 1 m
2 m
3 m
x
y
O
B
D
C
d
r
C
3 The distance rc to the centroid of the
area can be calculated by dividing the
crosshatched trapezoid into composite
parts and using the formula
rc = (1)
where x
el
is the centroidal coordinate
of the part with area A
el
.
A
el
x
el
A
el
E
F
4 m

1875
9.3 Theorems of Pappus and Guldinus Example 6, page 2 of 3
However, we can save some work by noting that the
second theorem of Pappus-Guldinus involves the
product of rc and the generating area A:
V = 2 rcA (2)
where V is the volume of the solid of revolution.
Solving Eq. 1 for the product of rc
and A
el
gives
rc A
el
= x
el
A
el
and substituting this result in Eq. 2 gives
V = 2 x
el
A
el
(3)
Thus we do not need to calculate rc and A
independently we need only evaluate the first
moment of the area, x
el
A
el
.
A = total area = sum of individual elements of area
4

1876
9.3 Theorems of Pappus and Guldinus Example 6, page 3 of 3
O
4 m
3 m
O
4 m
1 m
1 m
y
x
y
x
O
4 m
2 m
y
x
C
C
1 m
4 m
x
el
= 1 m + (1/3)(2 m)
= 1.6667 m
x
el
= 0.5 m
= +
5 To calculate x
el
A
el
, divide the trapezoidal region into
the sum of a rectangle and triangle, and set up a table.
6 Substituting the value of x
el
A
el
into Eq. 3 gives the volume of the solid:
V = 2 x
el
A
el
(Eq. 3 repeated)
= 54.5 m
3
Ans.
8.6668
Region A
el
(m
2
) x
el
(m) x
el
A
el
(m
3
)
Rectangle 4 0.5 2
Triangle 4 1.6667 6.6668
x
el
A
el
= 8.6668
A
el
= (4 m)(1 m)
= 4 m
2
A
el
= (1/2)(4 m)(2 m)
= 4 m
2

1877
9.3 Theorems of Pappus and Guldinus Example 7, page 1 of 1
y
x
C
r
C
R
7. Determine the centroidal coordinate rc of a
semicircular arc of radius R, given that the area of a
sphere of radius R is known to be 4 R
2
.
1 If the semicircle is revolved around the y axis,
a sphere of radius R is generated. The first
theorem of Pappus-Guldinus says that the
area of the sphere is given by
A = 2 rcL
Because we already know A (= 4 R
2
), we can
solve this equation for rc in terms of R and L:
rc = (1)
The length L in Eq. 1 is the circumference of
the semicircle:
L = R
Substituting this result in Eq. 1 gives
rc = (Eq. 1. repeated)
= Ans.

2 L
A
4 R
2
2
4 R
2
2 L
R
2R

1878
9.3 Theorems of Pappus and Guldinus Example 8, page 1 of 1
y
x
C
r
C
Radius = R
8. Determine the centroidal coordinate rc of a
semicircular area of radius R, given that the volume of a
sphere is known to be (4/3) R
3
.
1 If the semicircular area is revolved around the y axis,
a sphere of radius R is generated. The second
theorem of Pappus-Guldinus says that the volume of
the sphere is given by
V = 2 rcA (1)
Because we already know V (= (4/3) R
3
), we can
solve Eq. 1 for rc in terms of A and R:
rc = (2)
V
2 A
(4/3) R
3
The area A in Eq. 2 is the area bounded by the
semicircle:
R
2
Substituting this result in Eq. 2 gives
rc =

= Ans.
(4/3) R
3
2 A
2
A =
R
2
2
4R
3
2
(Eq. 2 repeated)

1879
9.3 Theorems of Pappus and Guldinus Example 9, page 1 of 4
20 m
1 m 3 m
2 m
40
9. A concrete dam is to be constructed in the shape
shown. Determine the volume of concrete that
would be required.
1 m 3 m
2 m
y
x
The y axis is the axis of revolution
2
The crosshatched region is the generating area.
1
3.5 m
3.5 m
20 m

1880
9.3 Theorems of Pappus and Guldinus Example 9, page 2 of 4
A = sum of element areas
The second theorem of Pappus-Guildinus gives the
volume as
V = 2 40/360)rcA (1)
where rc is the distance to the centroid of the generating
area, A is the magnitude of the area, and the
factor 40/360 accounts for the fact that the dam
corresponds to 40 rather than to a complete circle.
Thus we must calculate the product rcA. This product
may be found by dividing the crosshatched area into
composite parts and then using the formula
rc = (2)
where x
el
is the centroidal coordinate of the part with
area A
el
. Solving for the product rcA gives
rcA = x
el
A
el
Thus Eq. 1 can be written as
V = 2 40/360)(rcA) (Eq. 1 repeated)

= (2/9) x
el
A
el
(3)
and, to calculate V, we have to evaluate the sum x
el
A
el
.
3
A
el
x
el
A
el
Generating area divided into a
rectangle and two triangles
1 m 3 m
2 m
y
20 m
rc
3.5 m
x

1881
9.3 Theorems of Pappus and Guldinus Example 9, page 3 of 4
4 Calculate the areas and centroidal
distances, and set up a table.
1 m 3 m
2 m
y
x
20 m
rc
3 m
y
O
1 m 2 m
y
20 m
23 m
2 m
y
20 m
x
el
= 23 m + (3 m)/3 = 24 m
x
el
= 20 m + 2 m + 0.5 m = 22.5 m
x
el
= 20 m + (2 m)(2/3) = 21.333 m
=
+
+
O
O
O
O
Triangle 1
Triangle 2
Rectangle
3.5 m
3.5 m
3.5 m
3.5 m
A
el
= (1/2)(3.0 m) 3.5 m) = 5.25 m
2
A
el
= (3.5 m) m) = 3.5 m
2
A
el
= (1/2)(3.5 m) m)
= 3.5 m
2
x
x
x

1882
9.3 Theorems of Pappus and Guldinus Example 9, page 4 of 4
5 Table
6
Region A
el
(m
2
) x
el
(m) x
el
A
el
(m
3
)
Triangle 1 5.25 24 126
Rectangle 3.5 22.500 78.750
Triangle 2 3.5 21.333 74.665
x
el
A
el
= 279.415
Substituting the value of x
el
A
el
into Eq. 3 gives the
volume of the solid:
V = x
el
A
el
(Eq. 3 repeated)
279.415 m
3
= 195.1 m
3
Ans
2
9

1883
9.3 Theorems of Pappus and Guldinus Example 10, page 1 of 4
y
x
190 mm
190 mm
260 mm 260 mm
10. The concrete steps shown are in the shape of a quarter
circle. Determine the amount of paint required to paint
the steps, if one liter of paint covers 1.5 m
2
.
1 The y axis is the axis of revolution.
2 The generating
curve is a series of
four straight
line-segments.
190 mm
190 mm
260 mm
260 mm
y
x
z

1884
9.3 Theorems of Pappus and Guldinus Example 10, page 2 of 4
L = sum of segment lengths
The first theorem of Pappus-Guildinus gives the area as
A = 2 (90/360)rcL
= ( /2)rcL (1)
where rc is the distance to the centroid of the generating
curve, L is the length of the area, and the factor (90/360)
accounts for the fact that the steps are in the shape of a
quarter circle. Thus we must calculate the product rcL.
This product may be found from the formula for the centroid
of a composite curve made of a of collection of line
segments:
rc = (2)
where x
el
is the centroidal coordinate of the line segment
with length L
el
. Solving for the product rcL gives
rcL = x
el
L
el
Thus Eq. 1 can be written as
A = ( /2)(rcL) (Eq. 1 repeated)

= ( /2) x
el
L
el
(3)
and, to calculate A, we have to evaluate the sum x
el
L
el
.
y
x
190 mm
190 mm
260 mm 260 mm
3
x
el
L
el
L
el

1885
9.3 Theorems of Pappus and Guldinus Example 10, page 3 of 4
4 Calculate the centroidal coordinates
and lengths of the line segments, and
set up a table.
+
y
x
190 mm
190 mm
260 mm 260 mm
y
x
x
el
= 260 mm/2
= 130 mm
260 mm
380 mm
y
x
x
el
= 260 mm
y
x
190 mm
190 mm
260 mm 260 mm
190 mm
y
x
260 mm 260 mm
190 mm
x
el
= 260 mm + 260 mm
= 520 mm
x
el
= 260 mm + (260 mm)/2 = 390 mm
+
+
=
Line 1
Line 2
Line 3
Line 4
L
el
= 260 mm
L
el
= 190 mm
L
el
= 190 mm
L
el
= 260 mm

1886
9.3 Theorems of Pappus and Guldinus Example 10, page 4 of 4
6 Substituting the value of x
el
A
el
into Eq. 3 gives the area of the steps
A = x
el
L
el
(Eq. 3 repeated)
= 445 164 mm
2

283 400 mm
2
7 Amount of paint required = 445 164 mm
2
[1 m/(1000 mm)]
2
[1 liter of paint/1.5 m
2
of covered area]
= 0.297 liter Ans.
2
5 Table
Region L
el
(mm) x
el
(mm) x
el
L
el
(mm
2
)
Line 1 260 130 33 800
Line 2 190 260 49 400
Line 3 260 390 104 100
Line 4 190 520 98 800
x
el
L
el
= 283 400

1887
9.3 Theorems of Pappus and Guldinus Example 11, page 1 of 3
y
x
190 mm
190 mm
260 mm 260 mm
11. The concrete steps shown are in the shape of a quarter circle.
Determine the total number of cubic meters of concrete required to
construct the steps.
2 The generating area is the crosshatched region
shown.
The y axis is the axis of revolution 1
190 mm
190 mm
260 mm
260 mm
y
x
z

1888
9.3 Theorems of Pappus and Guldinus Example 11, page 2 of 3
3
A = sum of element areas
The second theorem of Pappus-Guildinus gives the
volume as
V = 2(90/360 rcA
= ( /2)rcA (1)
where rc is the distance to the centroid of the generating
area, A is the magnitude of the area, and the factor
(90/360) accounts for the fact that the steps form a
quarter circle. Thus we must calculate the product rcA.
This product may be found by dividing the crosshatched
area into composite parts and then using the formula
(2)
where x
el
is the centroidal coordinate of the part with
area A
el
. Solving for the product rcA gives
rcA = x
el
A
el
Thus Eq. 1 can be written as
V = ( /2)(rcA) (Eq. 1 repeated)

= ( /2) x
el
A
el
(3)
and, to calculate V, we have to evaluate the sum x
el
A
el
.
A
el
x
el
A
el
Generating area divided into two
rectangles
y
x
190 mm
190 mm
260 mm
260 mm
O
rc =

1889
9.3 Theorems of Pappus and Guldinus Example 11, page 3 of 3
4 Calculate the centroidal distances and areas
and set up a table.
5 Substituting the value of x
el
A
el
into Eq. 3 gives the volume of the solid
V = x
el
A
el
= 50 438 270 mm
3

= 0.0504 m
3
Ans.
32 110 000 mm
3
y
x
190 mm
190 mm
260 mm
260 mm
y
x
260 mm
y
x
260 mm 260 mm
O
O
O
=
+ 380 mm
2
x
el
= 260 mm/2 = 130 mm
x
el
= 260 mm + 260 mm/2 = 390 mm
Rectangle 1
Rectangle 2
190 mm
Region A
el
(mm
2
) x
el
(mm) x
el
A
el
(mm
3
)
Rectangle 1 98 800 130 12 844 000
Rectangle 2 49 400 390 19 266 000
x
el
A
el
= 32 110 000
A
el
= (190 mm)(260 mm)
= 49 400 mm
2
A
el
= (380 mm)(260 mm)
= 98 800 mm
2

1890
9.3 Theorems of Pappus and Guldinus Example 12, page 1 of 4
Front view Side view
12. Determine the mass of the steel V-belt pulley
shown. The density of the steel is 7840 kg/m
3
.
x
y
1 The x axis is the axis
of revolution:
2
The crosshatched area is
the generating area for
the right half of the
pulley.
100 mm
15 mm
15 mm
70 mm
4 mm 4 mm
12 mm 10 mm 12 mm
(20 mm)/2 = 10 mm
20 mm

1891
9.3 Theorems of Pappus and Guldinus Example 12, page 2 of 4
3
A = sum of element areas
The second theorem of Pappus-Guildinus gives the
volume as
V = 2 rcA (1)
where rc is the distance to the centroid of the generating
area, and A is the magnitude of the area. Thus we must
calculate the product rcA. This product may be found
by dividing the crosshatched area into composite parts
and then using the formula
rc = (2)
where x
el
is the centroidal coordinate of the part with
area A
el
. Solving for the product rcA gives
rcA = y
el
A
el
Thus Eq. 1 can be written as
V = 2 (rcA) (Eq. 1 repeated)

= 2 y
el
A
el
(3)
and, to calculate V, we have to evaluate the sum y
el
A
el
.
A
el
y
el
A
el
Generating area divided into two
rectangles, and a triangle
(70 mm 20 mm)/2 = 25 mm
15 mm
x
y
10 mm
4 mm
5 mm 12 mm

1892
9.3 Theorems of Pappus and Guldinus Example 12, page 3 of 4
A
el
= (1/2)(12 mm)(15 mm)
= 90 mm
2
A
el
= (4 mm)(15 mm)
= 60 mm
2
Generating area divided into
two rectangles, and a triangle
10 mm
25 mm
15 mm
x
y
10 mm
4 mm
5 mm 12 mm
12 mm
y
4 Calculate the centroidal distances
and areas for the rectangles and
triangle, and set up a table.
x
y
x
4 mm
x
y
5 mm + 12 mm + 4 mm
= 21 mm
25 mm
15 mm
10 mm + 5 mm = 35 mm
15 mm
35 mm
Rectangle 1
Triangle
Rectangle 2
=
+
+
y
el
= 10 mm + 25/2 mm
= 22.5 mm
y
el
= 35 mm + 15/3 mm
= 40 mm
y
el
= 35 mm + 15/2 mm
= 42.5 mm
A
el
= (21 mm)(25 mm)
= 525 mm
2

1893
9.3 Theorems of Pappus and Guldinus Example 12, page 4 of 4
Table 5
6 Substituting the value of y
el
A
el
into Eq. 3 gives half the volume of the
V-belt pulley
V = y
el
A
el
= 112 861.7 mm
3
= 0.000 112 861 7 m
3

17 962.5 mm
3
Total mass of the V-belt = total volume density
= (2 0.000 112 861 7 m
3
) 7840 kg/m
3
= 1.770 kg Ans.
7
Region A
el
(mm
2
) y
el
(mm) y
el
A
el
(mm
3
)
Rectangle 1 525 22.5 11 812.5
Triangle 90 40.0 3 600.0
Rectangle 2 60 42.5 2 550.0
y
el
A
el
= 17 962.5
Double the half-volume.

1894
9.3 Theorems of Pappus and Guldinus Example 13, page 1 of 3
y
z
13. Determine the area of the surface of
revolution generated by rotating the curve
y = z
4
, 0 z 1 m, about the z axis.
The z axis is the axis of revolution.
The generating curve is
y = z
4
, 0 z 1
1
2
1 m
1 m
x
y
1 m
z

1895
9.3 Theorems of Pappus and Guldinus Example 13, page 2 of 3
3
The first theorem of Pappus-Guildinus gives the area of
the surface of revolution as
A = 2 rcL (1)
where rc is the distance to the centroid of the generating
curve, and L is the length of the curve. Thus we must
calculate the product rcL. This product may be found
by considering the equation for the centroidal
coordinate:
rc =
L
where dL is an increment of curve length, and y
el
is the
coordinate of the increment. Solving for the product rcL
gives
rcL = y
el
dL
Thus Eq. 1 can be written as
A = 2 (rcL)

= 2 y
el
dL (2)
and, to calculate A, we have to evaluate a single integral,
y
el
dL.
y
el
dL
dL
C
dL
y
z
1 m
1 m
y
z
1 m
1 m
rc
y
el

1896
9.3 Theorems of Pappus and Guldinus Example 13, page 3 of 3
dL
dy
dz
5
Evaluating the integral by use of the integral
function key on a calculator gives
A = 3.44 m
2
Ans.
To evaluate the integral y
el
dL in Eq. 2, we use the
equation of the curve,
y = z
4
(3)
to express dL as a function of z. Thus
dL = (dy)
2
+ (dz)
2
= ( )
2
+ 1 dz (4)
and differentiating Eq. 3 gives
= 4z
3
so Eq. 4 can be written as
dL = (4z
3
)
2
+ 1 dz (5)
Substitute this expression for dL into Eq. 2
A = 2 y
el
dL (Eq. 2 repeated)
= 2

z
4
(4z
3
)
2
+ 1 dz
4
dz
dy
dz
dy
0
1
dL
y
z
y
el
= y = z
4
1 m

1897
9.3 Theorems of Pappus and Guldinus Example 14, page 1 of 3
x
y
1 m
z
y = z
4
1 The z axis is the axis of rotation.
2 The generating area is the
area under the y = z
4
curve.
14. Determine the volume of the solid of
revolution generated by rotating the curve
y = z
4
, 0 z 1 m, about the z axis.
y
z
1 m
1 m

1898
9.3 Theorems of Pappus and Guldinus Example 14, page 2 of 3
3
y
z
r
C
The second theorem of Pappus-Guildinus gives the
volume as
V = 2 rcA (1)
where rc is the distance to the centroid of the generating
area, and A is the magnitude of the area. Thus we must
calculate the product rcA. This product may be found
by considering the equation for the centroidal distance:
rc =
A
where dA is an increment of area, and y
el
is the
coordinate of the centroid of the incremental region.
Solving for the product rca gives
rcA = y
el
dA
Thus Eq. 1 can be written as
V = 2 (rcA)

= 2 y
el
dA (2)
and, to calculate V, we have to evaluate a single integral,
y
el
dA.
y
el
dA
dA
C
y
z
y = z
4
y = z
4
(x, y)
y
el

1899
9.3 Theorems of Pappus and Guldinus Example 14, page 3 of 3
y
z
y = z
4
(y, z)
y
el
= y/2 = z
4
/2
dz
4 To evaluate the integral y
el
dL in Eq. 2, we use the
equation of the curve,
y = z
4
to express dA as a function of z. Thus
dA = y dz
= z
4
dz
Substitute this expression for dA into Eq. 2
A = 2 y
el
dz (Eq. 2 repeated)
= 2
(
z
4
/2)(z
4
) dz
Evaluating the integral by use of the integral function key
on a calculator gives
V = 0.349 m
2
Ans.
0
1
1 m

1900
9.3 Theorems of Pappus and Guldinus Example 15, page 1 of 5
Radius = 20 mm
7 mm
15. A pharmaceutical company plans to put a coating
0.01 mm thick on the outside of the pill shown.
Determine the amount of coating material required.
1 The x axis is the axis of revolution.
The generating curve for half of the
pill surface is a composite curve
consisting of one straight line and a
circular arc. By symmetry, the total
surface area of the pill will be two
times the area generated by the curve
above.
2
x
y
3.5 mm
(1.5 mm)/2 = 0.75 mm
1.5 mm

1901
9.3 Theorems of Pappus and Guldinus Example 15, page 2 of 5
The first theorem of Pappus-Guildinus gives the area as
A = 2 rcL (1)
where rc is the distance to the centroid of the generating
curve and L is the length of the curve. Thus we must
calculate the product rcL. This product may be found
by dividing the curve into composite parts and then
using the formula
rc = (2)
where x
el
is the centroidal coordinate of the part with
length L
el
. Solving for the product rcL gives
rcL = y
el
L
el
Thus Eq. 1 can be written as
A = 2 (rcL) (Eq. 1 repeated)

= 2 y
el
L
el
(3)
and, to calculate A, we have to evaluate the sum y
el
L
el
.
3
Generating curve divided into a
straight line segment and an arc
x
y
3.5 mm
L
el
y
el
L
el
L
0.75 mm

1902
9.3 Theorems of Pappus and Guldinus Example 15, page 3 of 5
4 Calculate the centroidal distances and
the lengths, and set up a table.
y
=
3.5 mm
20 mm
x
y
y
el
= 3.5 mm
+
Arc
Straight line
x
y
3.5 mm
x
y
y
el
= 3.5 mm
Straight line
Radius = 20 mm
0.75 mm
x
5 For the straight line, the length and
coordinate of the centroid are easily
calculated..
L
el
= 0.75 mm
0.75 mm 0.75 mm

1903
9.3 Theorems of Pappus and Guldinus Example 15, page 4 of 5
6 To calculate x
el
L
el
for the arc,
use the information shown below,
which has been taken from a table
of properties of common
geometric shapes.
x
y
For our particular arc, r = 20 mm and
= (1/2) sin
-1
(3.5/20)
= 0.08795 rad
Thus
Length = 2 r

= 2(0.08795) 20 mm
= 3.5180 mm
rarc = (r sin
= (20 mm)(sin 0.08795)/(0.08795)

= 19.9742 mm
y
el
= rarc sin
19.9742 mm (sin 0.08795)
= 1.7545 mm 3.5 mm
20 mm
rarc
y
el
Arc
7
2
20 mm
3.5 mm
A
B
C
B C
A
Circular arc segment
r sin
C
Length = 2 r r
x
y
Centroid Location

1904
9.3 Theorems of Pappus and Guldinus Example 15, page 5 of 5
9 Substituting the value of y
el
A
el
into Eq. 3 gives the area of the solid
A = 2 2 y
el
L
el
= 110.5501 mm
2

where a factor of 2 has been inserted to account for the fact that we took advantage
of symmetry to calculate the area of only half of the body.
Amount of coating material required = 110.5501 mm
2
0.01 mm
= 1.106 mm
3
Ans.
8.7973 mm
2
8 Table
Region L
el
(m) y
el
(m) y
el
L
el
(m
2
)
Line 0.75 3.5 2.6250
Arc 3.5180 1.7545 6.1723
y
el
L
el
= 8.7973

1905
9.3 Theorems of Pappus and Guldinus Example 16, page 1 of 4
10 mm
5 mm
70 mm
60 mm
16. Determine the volume of the funnel.
x
y
The y axis is the axis of revolution 1
5 mm
2.5 mm
2 The generating area is the
crosshatched area shown.
5 mm
70 mm
60 mm
10 mm

1906
9.3 Theorems of Pappus and Guldinus Example 16, page 2 of 4
A
The second theorem of Pappus-Guildinus gives the
volume as
V = 2 rcA (1)
where rc is the distance to the centroid of the generating
area and A is the magnitude of the area. Thus we must
calculate the product rcA. This product may be found
by dividing the cross-hatched area into composite parts
and then using the formula
rc = (2)
where x
el
is the centroidal coordinate of the part with
area A
el
. Solving for the product rcA gives
rcA = x
el
A
el
Thus Eq. 1 can be written as
V = 2 (rcA) (Eq. 1 repeated)

= 2 x
el
A
el
(3)
and, to calculate V, we have to evaluate the sum x
el
A
el
.
3
A
el
x
el
A
el
Generating area divided into two
rectangles and two triangles
x
y
5 mm
2.5 mm
70 mm
60 mm
10 mm

1907
9.3 Theorems of Pappus and Guldinus Example 16, page 3 of 4
4 To calculate x
el
A
el
, divide the area
into the sum of two rectangles and
two triangles, and set up a table.
=
+
+
Triangle 1
Triangle 2
Rectangle 1
60 mm
x
y
60 mm
x
y
5 mm
10 mm
+
Rectangle 1
O
O
x
el
= 2.5 mm
5 mm
x
el
= 5 mm + (5 mm)/3
= 6.6667 mm
x
el
= 2.5 mm + (2.5 mm)/3
= 3.3333 mm
x
y
2.5 mm
x
y
2.5 mm
2.5 mm
70 mm
70 mm
Ael = (5 mm)(60 mm)
= 300 mm
2
A
el
= (2.5 mm)(70 mm)
= 175 mm
2
A
el
= (1/2)(5 mm)(60 mm)
= 150 mm
2
A
el
= (1/2)(2.5 mm)(70 mm)
= 87.5 mm
2
x
el
=1.25 mm
x
y
5 mm
2.5 mm
70 mm
60 mm
10 mm

1908
9.3 Theorems of Pappus and Guldinus Example 16, page 4 of 4
5
Substituting the value of x
el
A
el
into Eq. 3 gives the volume of the funnel
V = x
el
A
el
(Eq. 3 repeated)
= 14 200 mm
3
Ans.
2260.4187 mm
3
Table
6
Region A
el
(mm
2
) x
el
(mm) x
el
A
el
(mm
3
)
Rectangle 1 300 2.5 750
Rectangle 1 175 1.25 218.75
Triangle 1 150 6.6667 1000.0050
Triangle 2 87.5 3.3333 291.6637
x
el
A
el
= 2260.4187

1909
9.3 Theorems of Pappus and Guldinus Example 17, page 1 of 3
0.2 m
Signals parallel
to axis of dish
17. A satellite dish is shaped in the form of a paraboloid of
revolution to take advantage of the geometrical fact that all
signals traveling parallel to the axis of the paraboloid are
reflected through the focus. Determine the amount, in m
2
,
of reflecting material required to cover the inside surface of
the dish.
y
x
The x axis is the axis of revolution 1
The generating
curve is a parabola.
2
( 0.2, 0)
(0, 0.3)
The general form for a parabola with vertex on the x
axis is
x = ay
2
+ b
Evaluating this equation at the points ( 0.2, 0) and
(0, 0.3) gives the equations
0.2 = a(0)
2
+ b (1)
and
0 = a(0.3)
2
+ b
Solving for a and b and substituting back in Eq. 1
gives
x = 0.2222y
2
0.2 (2)
3
0.3 m
0.3 m

1910
9.3 Theorems of Pappus and Guldinus Example 17, page 2 of 3
x
To evaluate the integral y
el
dL in Eq. 4, we have to use
the equation of the parabola,
x = 0.2222y
2
0.2 (Eq. 2 repeated)
to express dL as a function of y. Thus
dL = (dx)
2
+ (dy)
2
=
2
+ 1 dy (5)
Differentiating Eq. 2 gives
= 0.4444y,
so Eq. 5 can be written as
dL = (0.4444y)
2
+ 1 dy (6)
5
C
( 0.2, 0)
(0, 0.3)
Apply the first theorem of Pappus-Guldinus
to calculate the surface area of the dish:
A = 2 rcL (3)
Thus we must calculate the product of the
length of the parabolic curve and its
centroidal coordinate. The product may be
found by considering the equation for the
centroidal coordinate:
rc =
L
Solving for the product rcL gives
rcL = y
el
dL
Thus Eq. 3 can be written as
A = 2 (rcL)

= 2 y
el
dL (4)
4
y
el
dL
dL
rc
y
dy
dx
dy
dx
dx
dy
dL
dL
y
el

1911
9.3 Theorems of Pappus and Guldinus Example 17, page 3 of 3
Noting that y = y
el
and also using Eq. 6 to replace dL in
Eq. 4 gives
y
A = 2 y
el
dL (Eq. 4 repeated)
= 2 y (0.4444y)
2
+ 1 dy
Using the integral function on a calculator gives
A = 0.284 m
2
Ans
6
x
C
( 0.2, 0)
(0, 0.3)
rc
y
0
0.3
(x, y)
y
el

1912
9.3 Theorems of Pappus and Guldinus Example 18, page 1 of 6
90 mm
80 mm
18. Determine the amount of coffee that
the coffee mug holds when full to the
brim. The radius of the rounded corners
and the rim is 15 mm.
Radius = 15 mm
x
y
Radius = 15 mm
The y axis is the axis of revolution, and the
generating area is the cross-hatched area shown.
1
2 The distance rc to the centroid of the area can
be calculated by dividing the crosshatched
area into composite parts and using the
formula
rc = (1)
where x
el
is the centroidal coordinate of the
part with area A
el
.
A
el
x
el
A
el

1913
9.3 Theorems of Pappus and Guldinus Example 18, page 2 of 6
However, we can save some work by noting that the second
theorem of Pappus-Guldinus involves the product of rc and the
generating area A:
V = 2 rcA (2)
where V is the volume of solid of revolution.
Solving Eq. 1 for the product gives
rc A
el
= x
el
A
el
and substituting this result in Eq. 2 gives
V = 2 x
el
A
el
(3)
Thus we do not need to calculate rc and A independently we
need only evaluate the first moment of the area, x
el
A
el
.
A
3

1914
9.3 Theorems of Pappus and Guldinus Example 18, page 3 of 6
4 To calculate x
el
A
el
, divide the area into the algebraic
sum of a rectangle, two squares, and two quarter circles,
and set up a table.
Radius = 15 mm
x
y
Radius = 15 mm
x
y
=
+
Rectangle
x
y
x
y
x
y
x
y
For clarity, the areas near the rim and
rounded corner have been drawn
disproportionately large.
+
90 mm
40 mm
Square 1
Square 2
Quarter circle 1
Quarter circle 2
Subtract quarter-
circular area
from square to
form area near
rim.
Subtract square
from quarter-
circular area to
form rounded
corner.
5
6

1915
9.3 Theorems of Pappus and Guldinus Example 18, page 4 of 6
15 mm
Square 1
Square 2
x
Radius = 15 mm
90 mm
40 mm
Radius = 15 mm
Original area
A
el
= (15 mm)
2
= 225 mm
2
A
el
= (40 mm)(90 mm)
= 3600 mm
2
15 mm
40 mm
xel = 40 mm (15 mm)/2
= 32.5 mm 15 mm
x
el
= 40 mm + (15 mm)/2
= 47.5 mm
y
7 Calculate the areas and centroidal coordinates of the
rectangle and the squares.
x
y
Rectangle
40 mm
90 mm
x
el
= 20 mm
x
y
x
y
40 mm 15 mm

1916
9.3 Theorems of Pappus and Guldinus Example 18, page 5 of 6
40 mm
8 Calculate the area and centroidal coordinates
of the quarter-circular regions..
x
y
x
y
40 mm
15 mm
Quarter circle 1
Quarter circle 2
9
A table of properties of planar regions
gives the information shown below.
4(15 mm)
y
x
r
C
4r
3

3
4r
3
r
2
4
A =
Quarter circular region
Centroid location
(15 mm)
2

In our particular problem, r = 15 mm, so the
distance to the centroid is
= = 6.3662 mm
4
Also, the area is
A
el
= = 176.7146 mm
2
4
10
x
el
= 40 mm + 15 mm 6.3662 mm
= 48.6338 mm
x
el
= 40 mm 15 mm
+ 6.3662 mm
= 31.3662 mm
15 mm

1917
9.3 Theorems of Pappus and Guldinus Example 18, page 6 of 6
11
Substituting the value of x
el
A
el
into Eq. 3 gives the volume of the solid
V = x
el
A
el
= 454 000 mm
3
Ans.
72 323.563 0 mm
3
Table
12
Region A
el
(mm
2
) x
el
(mm) x
el
A
el
(mm
3
)
Rectangle 3600 20 72 000
Square 1 225 47.5 10 687.5
Quarter circle 1 176.7146 48.6338 8 594.3025
Square 2 225 32.5 7 312.5
Quarter circle 2 176.7146 31.3662 5 542.8655
x
el
A
el
= 72 323.5630

1918
9.3 Theorems of Pappus and Guldinus Example 19, page 1 of 6
Radius = 20 mm
5 mm
17.5 mm
15 mm
19. Determine the capacity of the small bottle of lotion if the
bottle is filled half way up the neck.
Radius = 20 mm
x
y
The y axis is the axis of revolution. 1
15 mm
17.5 mm
2.5 mm
2 The generating area is the
crosshatched area shown.
Half of the
neck is filled:
(5 mm)/2

1919
9.3 Theorems of Pappus and Guldinus Example 19, page 2 of 6
3
A
The second theorem of Pappus-Guildinus gives the
volume as
V = 2 rcA (1)
where rc is the distance to the centroid of the generating
area, and A is the magnitude of the area. Thus we must
calculate the product rcA. This product may be found
by dividing the cross-hatched area into composite parts
and then using the formula
rc = (2)
where x
el
is the centroidal coordinate of the part with
area A
el
. Solving for the product rcA gives
rcA = x
el
A
el
Thus Eq. 1 can be written as
V = 2 (rcA) (Eq. 1 repeated)

= 2 x
el
A
el
(3)
and, to calculate V, we have to evaluate the sum x
el
A
el
.
A
el
x
el
A
el
Generating area divided into a rectangle,
two triangles and a circular sector
Radius = 20 mm
x
y
15 mm
17.5 mm
2.5 mm
O

1920
9.3 Theorems of Pappus and Guldinus Example 19, page 3 of 6
4
E
O
B
C D
O
Before the areas and centroidal coordinates can be
found, we first must find the distances and angles
shown below.
15 mm 20 mm
17.5 mm 20 mm
EB = (20 mm)
2
(15 mm)
2

= 13.2288 mm
DC = (20 mm)
2
(17.5 mm)
2

= 9.6825 mm
EOB = cos
-1
= 41.4096 (5)
15
20
DOC = cos
-1
= 28.9550 (4)
17.5
20
Radius = 20 mm
x
15 mm
17.5 mm
2.5 mm
E
O
C
D
B

1921
9.3 Theorems of Pappus and Guldinus Example 19, page 4 of 6
Triangle 1 Triangle 2
Rectangle
Circular sector
Radius = 20 mm
x
y
15 mm
17.5 mm
2.5 mm
E
O
C
D
B
=
D
C D
O
O
O
C
B E
x
el
= 9.6825 mm/2
= 4.8412 mm
13.2288 mm
9.6825 mm
A
el
= (2.5 mm)(9.6825 mm)
= 24.2062 mm
2
A
el
= (1/2)(15 mm)(13.2288 mm)
= 99.2160 mm
2
A
el
= (1/2)(17.5 mm)(9.6825 mm)
= 84.7219 mm
2
C
B
5 The areas and centroidal coordinates of
the rectangle and triangles can now be
calculated.
x
y
17.5 mm
x
y
15 mm
x
y
2.5 mm
x
y
+
+
+
x
el
= 9.6825 mm/3
= 3.2275 mm
x
el
= (13.2288 mm)/3
= 4.4096 mm
9.6825 mm

1922
9.3 Theorems of Pappus and Guldinus Example 19, page 5 of 6
x
y
= (180 DOC EOB)/2

by Eq. 4 by Eq. 5
= (180 28.9550 41.4096)/2
= 54.8177
= + EOB 90
= 54.8177 + 41.4096 90
= 6.2273
rc =
=

= 11.3903 mm
x
el
= rc cos
= (11.3903 mm) cos 6.2273
= 11.3231 mm
x
el
y
x
O
A
el
= r
2
= ( 54.8177/180) 20 mm)
2
= 382.6997 mm
2
r
C
2r sin
y
x
6
To calculate the area and centroidal
coordinate of the circular sector, we can use
the information shown below, which has been
taken from a table of properties of planar
regions. Note that in the formula equals
half the angle of the arc.
A = r
2
Circular sector region
C
B E
D
2r sin
3
2(20 mm) sin 54.8177
3(54.8177 /180)
rc
DOC = 28.9550
EOB = 41.4096
7
8
r = 20 mm
Centroid Location

1923
9.3 Theorems of Pappus and Guldinus Example 19, page 6 of 6
9
Substituting the value of x
el
A
el
into Eq. 3 gives the capacity of the bottle:
V = x
el
A
el
= 32 400 mm
3
Ans.
5161.4768 mm
3
Table
10
Region A
el
(mm
2
) x
el
(mm) x
el
A
el
(mm
3
)
Triangle 1 84.7219 3.2275 273.4399
Triangle 2 99.2160 4.4096 437.5029
Rectangle 24.2062 4.8412 117.1871
Circular sector 382.6997 11.3231 4333.3470
x
el
A
el
= 5161.4768

1924

9.4 Hydrostatic Pressure on Submerged Surfaces
1925
9.4 Hydrostatic Pressure on Submerged Surfaces Procedures and Strategies, page 1 of 1
(Symbol for a free surface)
Procedures and Strategies for Solving Problems
Involving Hydrostatics
1. On a sketch of the surface in contact with the liquid,
show the pressure acting perpendicular to the surface. If the
surface is horizontal, then the pressure is uniform;
otherwise the pressure should be shown increasing linearly
with depth.
2. Calculate numerical values of the pressure by using the
equation p = h, where h is the depth below the free surface,
and is the specific weight of the liquid.

3. For a flat surface of constant width, calculate the
resultant and line of action of the pressure force by applying
the same techniques used for finding these quantities for
distributed forces acting on a beam.
4. For a curved surface of constant width, calculate the
resultant by considering a free-body diagram consisting of
the liquid bounded by the curved surface and by horizontal
and vertical planes. If the width is not constant, then you
must integrate to obtain the resultant.
5. Remember that pressure exerts an upward force on a
submerged surface if the liquid lies beneath the surface
(That is why ships float).

1926
9.4 Hydrostatic Pressure on Submerged Surfaces Problem Statement for Example 1
A B
5 m Hinge
3 m
1. To prevent water pressure from pushing gate AB open, a small
extension, or lip, is provided at A. If the gate is 4-m wide
(measured perpendicular to the plane of the figure), determine the
force acting on the lip. The density of water is 10
3
kg/m
3
.

1927
9.4 Hydrostatic Pressure on Submerged Surfaces Problem Statement for Example 2
300-mm dia
2. Hydraulically-operated equipment is designed to transform a
relatively small input force into a much larger output force. For the
system shown, determine the weight W that can be supported by the
piston at B when a 200-N force is applied to the piston at A.
Density of hydraulic fluid

h
900 kg/m
3
W
300-mm dia
A
200 N
B
200 mm
25-mm dia

1928
9.4 Hydrostatic Pressure on Submerged Surfaces Problem Statement for Example 3
3. During construction, gate AB is temporarily held
in place by the horizontal strut CD. Determine the
force in the strut, if the gate is 4-m wide.
Density of water 10
3
kg/m
3
A
B
C D
2 m
3 m

1929
9.4 Hydrostatic Pressure on Submerged Surfaces Problem Statement for Example 4
Vertex
Parabola
B
A
x
10 m
4. Determine the magnitude and line of action
of the resultant hydrostatic force acting on a 1-m
wide section of the seawall. Assume that the
density of sea water is = 1.02 10
3
kg/m
3
.
12 m

1930
9.4 Hydrostatic Pressure on Submerged Surfaces Problem Statement for Example 5
6 ft
2 ft
h
5. Determine the distance h for which the gate is just about to open.
Neglect the weight of the gate. The specific weight of the fluid is .
D
C
B
A

1931
9.4 Hydrostatic Pressure on Submerged Surfaces Problem Statement for Example 6
Open top
Open bottom
A D
m
B C
0.4 m
Mass of form 60 kg
Density of concrete

c 2.4 10
3
kg/m
3
6. Concrete is poured into the open top of a form to produce a
ramp. If the ramp is 0.8-m wide, determine the minimum mass m
needed to keep the form from lifting off the ground.
0.7 m 0.5 m

1932
9.4 Hydrostatic Pressure on Submerged Surfaces Problem Statement for Example 7
2.5 ft
3 ft
4 ft
A
B
C
7. Determine the minimum weight W of gate BC required to keep the gate closed. The
gate is 2-ft wide and of uniform density. The specific weight of water is 62.4 lb/ft
3
.

1933
9.4 Hydrostatic Pressure on Submerged Surfaces Problem Statement for Example 8
8. Determine the magnitude and line of action of the resultant hydrostatic force
acting on the semicircular end of the tank. The tank is filled to the top with water.
The specific weight of water is 62.4 lb/ft
3
.
B C
Radius 4 ft
Cables

1934
9.4 Hydrostatic Pressure on Submerged Surfaces Problem Statement for Example 9
B C
D
E
1.1 m
1 m
0.55 m
1.5 m
9. Determine the magnitude and line of action of the resultant
hydrostatic force acting on the end of the tank, which is filled to
the top with water. The density of water is 10
3
kg/m
3
.

1935
9.4 Resultants of Hydrostatic Pressure Forces Example 1, page 1 of 3
A B
5 m Hinge
3 m
1. To prevent water pressure from pushing gate AB open, a small
extension, or lip, is provided at A. If the gate is 4-m wide
(measured perpendicular to the plane of the figure), determine the
force acting on the lip. The density of water is 10
3
kg/m
3
.

1936
9.4 Resultants of Hydrostatic Pressure Forces Example 1, page 2 of 3
p

AB
g 5 m
(10
3
kg/m
3
)(9.81 m/s
2
)(5 m)
49.05 10
3
(kg m/s
2
)/m
2
49.05 10
3
N/m
2
49.05 kN/m
2
49.05 kPa
A B
5 m
p

AB
Since AB is horizontal, the pressure
p
AB
acting on it is uniform.
1
The pressure must be p

AB
at all other
points at the same elevation. Since
these points lie 5 m below the free
surface of the water,
2

1937
9.4 Resultants of Hydrostatic Pressure Forces Example 1, page 3 of 3
B

x
B

y
A B
Force acting on lip, F
A
3 Free-body diagram of gate AB
Forces from hinge
3 m
Moment arm of
pressure resultant
Convert the pressure to a force per length by
multiplying by the width:
w p

AB
width of gate
(49.05 kPa) 4 m
196.20 kN/m
4
Equilibrium equation for gate AB
M

B
F

A
(3 m) (196.20 kN/m)(3 m)(3 m/2) 0
Solving gives
F

A
294 kN Ans.
resultant of w
+
w

1938
9.4 Resultants of Hydrostatic Pressure Forces Example 2, page 1 of 3
300-mm dia
2. Hydraulically-operated equipment is designed to transform a
relatively small input force into a much larger output force. For the
system shown, determine the weight W that can be supported by the
piston at B when a 200-N force is applied to the piston at A.
Density of hydraulic fluid

h
900 kg/m
3
W
300-mm dia
A
200 N
B
200 mm
25-mm dia

1939
9.4 Resultants of Hydrostatic Pressure Forces Example 2, page 2 of 3
Pressure p
200 mm
This is a typical result for
hydraulic equipment. The
effect of the weight of the fluid
is small compared to the
pressure caused by the pistons.
3
The pressure at two points at
the same elevation in a fluid
at rest must be equal.
4
The pressure 200 mm below A equals the pressure at A
plus an additional amount caused by the weight of the fluid
p = 407.437 kPa + (900 kg/m
3
)(9.81 m/s
2
)(200 mm)
407.437 kPa + 1.766 kPa
409.203 kPa
2
0.2 m
The pressure on the underside of the piston at A is 1
407.437 kN/m
2
407.437 kPa
200 N
(0.025 m)
2
407 437 N/m
2
Area of piston
Force
4
W
A
200 N
B
25-mm dia

1940
9.4 Resultants of Hydrostatic Pressure Forces Example 2, page 3 of 3
Equilibrium equation
F

y
W + (409.203 kPa)[ (0.3 m)
2
] 0
Solving gives
W 28.9 kN
6
Thus the 200-N input force produces a 28.9 kN
output force, a multiplication factor of
7
4
Ans.
200 N
28.9 kN 28.9 10
3
N
200 N
144
+
W
300-mm dia
p 409.203 kPa
Free-body diagram of piston B 5

1941
9.4 Resultants of Hydrostatic Pressure Forces Example 3, page 1 of 4
3. During construction, gate AB is temporarily held
in place by the horizontal strut CD. Determine the
force in the strut, if the gate is 4-m wide.
Density of water 10
3
kg/m
3
A
B
C D
2 m
3 m

1942
9.4 Resultants of Hydrostatic Pressure Forces Example 3, page 2 of 4
2 m
3 m
2 m
1
Convert the pressure to a force per length by
multiplying by the width, 4 m:
w

B
p

B
4 m
(49.05 kPa) 4 m
196.20 kN/m (1)
2
The water pressure varies linearly from 0 to p

B
, where
p

B
g (2 m + 3 m)
(10
3
kg/m
3
)(9.81 m/s
2
)(5 m)
49.05 kPa
B
C D
A
p

B
w

B
B
C D
A
3 m

1943
9.4 Resultants of Hydrostatic Pressure Forces Example 3, page 3 of 4
B
C
A
w

B
196.20 kN/m
The resultant passes through the centroid of the
triangle, or one-third of the distance from the
base to the opposite vertex:
(1/3)(2 m + 3 m) 1.667 m above point B
4
3 Resultant of distributed load
area under load curve
area of triangle
(1/2)(196.2 kN/m)(2 m + 3 m)
490.5 kN
D
2 m
3 m

1944
9.4 Resultants of Hydrostatic Pressure Forces Example 3, page 4 of 4
+
A
B
B

y
B

x
F

CD
3 m
490.5 kN
1.667 m
Free-body diagram of gate AB 5
Equilibrium equation
M

B
(F

CD
)(3 m) (490.5 kN)(1.667 m) 0
solving gives
F

CD
273 kN Ans.
This is a large force. Most likely more than one strut would
be used.
6

1945
9.4 Resultants of Hydrostatic Pressure Forces Example 4, page 1 of 10
B
A
x
10 m
4. Determine the magnitude and line of action
of the resultant hydrostatic force acting on a 1-m
wide section of the seawall. Assume that the
density of sea water is = 1.02 10
3
kg/m
3
.
12 m
Vertex
Parabola

1946
9.4 Resultants of Hydrostatic Pressure Forces Example 4, page 2 of 10
B
A
Pressure
B
A C
Integration can be used to calculate the
resultant of the pressure forces acting on
the curved wall, but it is easier to consider
a free body consisting of a portion of the
water behind the sea wall.
1

1947
9.4 Resultants of Hydrostatic Pressure Forces Example 4, page 3 of 10
C
W weight of water 4
R

x
R

y
Components of force from
seawall acting on water
5
Force per length, w

B
(pressure at B) (1-m distance normal to plane of figure)
( g 12 m)(1 m)
(1.02 10
3
kg/m
3
)(9.81 m/s
2
)(12 m)(1 m)
120.1 kN/m
3
B
A
10 m
Free-body diagram of region of ABC 2
w

B
12 m

1948
9.4 Resultants of Hydrostatic Pressure Forces Example 4, page 4 of 10
10 m
12 m
Vertex
The magnitude of the weight, W, can be calculated by using information
from a table of properties of planar regions as shown below:
6
In our particular example, the length of the side
opposite the vertex 12 m a, and h 10 m, so
Area 2ah/3
2(12 m)(10 m)/3
80 m
2
(1)
7
h
2h/5
C
Vertex
Semiparabolic region
a
Centroid Location
2ah
3
Area =
x
y

1949
9.4 Resultants of Hydrostatic Pressure Forces Example 4, page 5 of 10
B
A
10 m
Free-body diagram of region ABC (repeated) 8
12 m
C
R

x
R

y
w

B
120.1 kN/m
800.5 kN
R
720.6 kN
P resultant of pressure forces
area of w-diagram
(1/2)(12 m)(120.1 kN/m)
720.6 kN
9
W weight
g area width
(1.02 10
3
kg/m
3
)(9.81 m/s
2
)(80 m
2
)(1 m)
800.5 kN
10
Resultant 12
Equilibrium equations
F

x 0: R

x + 720.6 kN 0 (2)
F

y
0: R

y
800.5 kN 0 (3)
Solving gives
R

x 720.6 kN
R

y
800.5 kN
11
(720.6 kN)
2
+ (800.5 kN)
2
1.077 MN
R
+
+

1950
9.4 Resultants of Hydrostatic Pressure Forces Example 4, page 6 of 10
B
A
R is the force of the seawall on
the water. The force of the
water on the seawall is equal
and opposite.
13
720.6 kN
800.5 kN
R = 1.077 MN Ans.

1951
9.4 Resultants of Hydrostatic Pressure Forces Example 4, page 7 of 10
B
A
To determine the line of action of the resultant force,
consider a free-body diagram of region ABC again.
14
C
Resultant of triangular
distribution of pressure
equals 720.6 kN and acts
8 m [= (2/3)(12 m)] from
point C.
15
Resultant, R
The 800.5 kN weight of the water acts
through the centroid of the semiparabolic
area. The centroidal distance x

c can be
calculated using a table giving geometric
properties of common areas.
16
12 m
8 m
10 m
Line of action
of resultant, R
Vertex
800.5 kN
800.5 kN
720.6 kN
x

C

1952
9.4 Resultants of Hydrostatic Pressure Forces Example 4, page 8 of 10
17
In our particular example, the length of the straight side
adjacent to the vertex 10 m h, and so the centroid is
located 4 m [ = (2/5)(10)] to the right of of C.
18
A C
B
10 m
x

c = 4 m
To locate the centroid, we again make use
of a table of properties of planar regions as
shown below:
2h/5
h
C
Vertex
Semiparabolic region
a
Centroid Location
2ah
3
Area =
x
y
12 m
Vertex

1953
9.4 Resultants of Hydrostatic Pressure Forces Example 4, page 9 of 10
B
A
The free-body diagram of region ABC can be re-drawn
with the known distances specified.
19
C
A force can be considered to act
anywhere along its line of action
(Principle of transmissibility).
We can simplify our calculations
by considering the resultant R to
act at point D where the line of
action intersects the left edge of
the free body.
20
8 m
x

C
= 4 m
Vertex
800.5 kN
D
720.6 kN
Line of action
of resultant, R
720.6 kN
800.5 kN
Resultant, R

1954
9.4 Resultants of Hydrostatic Pressure Forces Example 4, page 10 of 10
B
A C
M

C
= (720.6 kN)(8 m) (720.6 kN)(d) (800.5 kN)(4 m) = 0
Solving gives the vertical distance below point C to the line of action of
the resultant:
d = 3.56 m Ans.
21
x

C
= 4 m
Line of action
of resultant, R
Vertex
D
8 m
720.6 kN
800.5 kN
d
R
+

1955
9.4 Resultants of Hydrostatic Pressure Forces: Example 5, page 1 of 6
D
C
B
A
6 ft
2 ft
h
5. Determine the distance h for which the gate is just about to open.
Neglect the weight of the gate. The specific weight of the fluid is .
The value of h will not depend on the gate width (distance
measured perpendicular to the plane of the figure) because the
width would cancel out of the equation for the sum of moments
about the support C. Accordingly we will base our calculations
on a 1-ft width of gate.
1

1956
9.4 Resultants of Hydrostatic Pressure Forces: Example 5, page 2 of 6
2 ft
h
6 ft
Free-body diagram of gate (distributed forces) 2
Pressure:
p

B
(2 ft)
p

D
(2 ft + h)
3
Force per length:
w

B
p

B
(1 ft) (2 ) 1 2
w

D
p

D
(1 ft) (2 + h) 1 (2 + h)
4
1-ft width
C
D
B
A
C

y
C

x
w

D
F

D
(force from ledge in ground)
w

B

1957
9.4 Resultants of Hydrostatic Pressure Forces: Example 5, page 3 of 6
D
B
A
2 ft
C
C
A
B
D
w

B
2
2
Area (1/2)(2 ft)(2 )
2
Resultant acts through centroid of
triangle (at a distance equal to one-third
of the height of the triangle)
6
(2/3) ft
Resultant force on AB 5

1958
9.4 Resultants of Hydrostatic Pressure Forces: Example 5, page 4 of 6
D
B
A
C
C
A
B
D
6 ft
3 ft 3 ft
12
w

B
= 2
Resultant force on BC 7
Area 2 6 12

1959
9.4 Resultants of Hydrostatic Pressure Forces: Example 5, page 5 of 6
A
B
D
C
B
A
A
B
+
h
h
h/2
2h/3
2
2
h
w

B
2
w

D
(2+h)
2 h
(h
2
Resultant force on CD 8
Area (1/2)(h )(h)
h
2
/2
Trapezoid triangle + rectangle
Area 2 h

1960
9.4 Resultants of Hydrostatic Pressure Forces: Example 5, page 6 of 6
2/3 ft
C

y
C

x
F

D
0, since gate is just about to open 10
3 ft
h/2
A
B
D
C
2 h
12
(h
2
2
2h/3
Free-body diagram (resultant forces) 9
Equilibrium equation
M

C
0: ( 2 )(2/3 ft) (12 )(3 ft) + (2 h)(h/2) + (h
2
/2)(2h/3) 0
Solving gives
h 4.0 ft Ans.
11
+

1961
9.4 Resultants of Hydrostatic Pressure Forces: Example 6, page 1 of 3
Open top
Open bottom
A D
m
B C
0.4 m
Mass of form 60 kg
Density of concrete

c 2.4 10
3
kg/m
3
6. Concrete is poured into the open top of a form to produce a
ramp. If the ramp is 0.8-m wide, determine the minimum mass m
needed to keep the form from lifting off the ground.
0.7 m 0.5 m

1962
9.4 Resultants of Hydrostatic Pressure Forces: Example 6, page 2 of 3
m
A
B
D
C B
A
m
C
0.4 m
D
We can solve the problem by finding the
resultant of the pressure of the concrete acting
on part AB of the form. But we would have to
compute the length AB and also compute the
angle the resultant makes with the vertical.
Although the problem can be done in this
manner, it is easier to consider a free body
consisting of the form together with the
concrete. Thus let's first find the resultant
force of the pressure of the floor pushing up
on the concrete.
1
Pressure at D 2.4 10
3
kg/m
3
)(9.81 m/s
2
)
(0.4 m)
= 9.418 10
3
N/m
2

Distributed load (force per length) =
pressure width of gate
(9.418 10
3
N)(0.8 m)
7.534 10
3
N/m
Resultant force = (7.534 10
3
N/m)
(0.7 m + 0.5 m)
= 9.041 10
3
N
2
0.5 m 0.7 m

1963
9.4 Resultants of Hydrostatic Pressure Forces: Example 6, page 3 of 3
A
B
Weight of form (60 kg)(9.81 m/s
2
)
= 588.6 N
4
C
D
0.4 m
F

y
0: 9.041 10
3
N mg 588.6 N .404 10
3
N 0
Setting g equal to 9.81 m/s
2
and solving gives
m = 209 kg Ans.
Free-body diagram of form together with concrete 3
Equilibrium equation 6
Weight of concrete

cg volume
2.4 10
3
kg/m
3
)(9.81 m/s
2
)[(1/2)(0.7 m)
(0.4 m)(0.8 m) + (0.5 m)(0.4 m)(0.8 m)]
6.404 10
3
N
5
+
mg
0.7 m 0.5 m
Reaction from floor = 9.041 10
3
N

1964
9.4 Resultants of Hydrostatic Pressure Forces: Example 7, page 1 of 5
A
B
C
7. Determine the minimum weight W of gate BC required to keep the gate closed. The
gate is 2-ft wide and of uniform density. The specific weight of water is 62.4 lb/ft
3
.
2.5 ft
3 ft
4 ft

1965
9.4 Resultants of Hydrostatic Pressure Forces: Example 7, page 2 of 5
If we attempt to solve the problem by considering the pressure acting directly
on BC, then we have to calculate the length and angle of inclination of BC.
1
A
B
C

1966
9.4 Resultants of Hydrostatic Pressure Forces: Example 7, page 3 of 5
3 Pressures:
at B,
p

B
2.5 ft 2.5
at C,
p

C
(2.5 ft + 3 ft) 5.5
Distributed loads (force per length):
w

B
p

B
(width of gate) (2.5 )(2 ft) 5 (1)

w

C
p

C
(width of gate) (5.5 )(2 ft) 11
Although the problem can be solved in this
manner, it is easier to consider a free body
consisting of gate BC together with the water
under BC.
3 ft
2.5 ft
w

C
w

C
w

B
C
B
A
2

1967
9.4 Resultants of Hydrostatic Pressure Forces: Example 7, page 4 of 5
+
w

C
w

B
B

x
B

y
W
4 ft
2 ft
4/3 ft
3 ft
The force from the ground at C acting
on the gate is zero since the gate is
about to lose contact with ground.
7
The trapezoidal distributed load from the fluid
acting on the right side of the free body can be
considered to be the sum of a rectangular and
triangular distributed load.
8
Weight of gate 5
Free-body diagram of gate BC
together with the water under BC
4
Weight of water
Volume
(1/2)(4 ft)(3 ft)(2 ft)
12
(Acts through centroid of triangle)

6
B
C
w

B
w

C

1968
9.4 Resultants of Hydrostatic Pressure Forces: Example 7, page 5 of 5
Resultant force from
uniform load on bottom
Resultant force from
uniform load on right side
Resultant force from
triangular load on right side
Substituting w

B
5 (Eq. 1), w

C
11 (Eq. 2), and
62.4 lb/ft
3
into Eq. 3 and solving gives
W 3,510 lb Ans.
10
Equation of equilibrium
M

B
0: W(2 ft) + 12 (4/3 ft) 4w

C
(2 ft) 3w

B
(3/2 ft) (1/2)(w

C
w

B
)(3 ft)(2/3 3 ft) = 0 (3)
9
+

1969
9.4 Resultants of Hydrostatic Pressure Forces: Example 8, page 1 of 4
8. Determine the magnitude and line of action of the resultant hydrostatic force
acting on the semicircular end of the tank. The tank is filled to the top with water.
The specific weight of water is 62.4 lb/ft
3
.
Because the end of the tank is not of uniform
width, we have to use integration to compute the
magnitude of the resultant.
1
B C
Radius 4 ft
Cables

1970
9.4 Resultants of Hydrostatic Pressure Forces: Example 8, page 2 of 4
dy
(x, y)
Pressure force on elemental area dA 2
C B
x
y
x
Distance y (Insert a minus sign
to get a positive distance y is
negative in the region shown)
5
Use = 62.4 lb/ft
3
and evaluate
with the integral function on a
calculator.
8
Solve for x
and substitute
4
Differential force:
dR pressure area
[ ( y)] [2 dy] 4
2
y
2
6
dA 2x dy
4
2
y
2
2 dy
3
x
2
+ y
2
4
2
dy] 4
2
y
2
[ ( y)] [2
0
4
R dR
2,662 lb Ans.
Resultant force: 7

1971
9.4 Resultants of Hydrostatic Pressure Forces: Example 8, page 3 of 4
x
y
dR
Distance y
d
z z
Moment of R about x axis = Integral of moment of dR about x axis
Evaluating the integral by using the integral function on
a calculator and then solving for d gives
d 2.36 ft Ans.
To determine the line of action of the resultant, equate the moment of the
resultant to the integral of the moment of the differential force, dR.
9
By symmetry, the line of action
passes through the y axis.
10
11
R d ( y) dR (1)
or
4
0
2,662 lb d ( y) [ 2 y
62.4 lb
4
2
y
2
] dy
y
x
R 2,662 lb

1972
9.4 Resultants of Hydrostatic Pressure Forces: Example 8, page 4 of 4
We could have saved some work by using a table of moments of
inertia as follows: Eq. 1 is
R d ( y) dR (Eq.1 repeated)
pressure dA
[ ( y)] dA
I

x, the moment of inertia of the
area about the x axis.
12
y
2
dA

1973
9.4 Resultants of Hydrostatic Pressure Forces: Example 9, page 1 of 8
B C
D
E
1.1 m
1 m
0.55 m
1.5 m
Because the end of the tank is not of
uniform width, we have to use integration
to compute the magnitude of the resultant.
1
9. Determine the magnitude and line of action of the resultant
hydrostatic force acting on the end of the tank, which is filled to
the top with water. The density of water is 10
3
kg/m
3
.

1974
9.4 Resultants of Hydrostatic Pressure Forces: Example 9, page 2 of 8
B
Region 1 Region 2
Region 3
D
C
E E
1 m
1.1 m
1.5 m
0.55 m 0.55 m
D
C
B
Because of symmetry, Regions 1 and 3 have the same resultant
force. Thus we need to consider only one of the regions. Let's
choose Region 3.
3
Let's divide the end of the tank into three regions. 2
+ +

1975
9.4 Resultants of Hydrostatic Pressure Forces: Example 9, page 3 of 8
(x, y)
y
1.1 m
x
x
dy
0.55 m
y
C
z
Pressure force acting on dA: dR

3
Depth below free surface
1.1 m y
Calculation of resultant of pressure on Region 3 4
Differential force
dR

3
pressure area
[ g(1.1 y)] [(y/2) dy] (1)
6
Solve for x and
substitute in the
expression for dA.
Elemental area:
dA x dy
(y/2) dy
5
0.55
1.1
y
x
2x
Equation of line:

1976
9.4 Resultants of Hydrostatic Pressure Forces: Example 9, page 4 of 8
We also need to determine the line of action of
R

3
. We can do this by equating the moment
produced by R

3
acting along its line of action
to the integral of the moment produced by dR

3
.
8
Use the integral function
on a calculator to evaluate
Resultant force for Region 3
R

3
dR

3

g (1.1 y) (y/2) dy
1.1
0
7
0.1109 g (2)
Eq. 1

1977
9.4 Resultants of Hydrostatic Pressure Forces: Example 9, page 5 of 8
1.1 m
x
y
C
z
(1.1 m y)
F
F
z
C
y
x
dR

3
R

3
d

3
Moment of R

3
about FC 9 Moment of differential force about FC 11
Vertical distance to
line of action of R

3
10
Equating moments about the top edge FC gives
R

3
d

3
=
Evaluating the integral and solving for d

3
gives
d

3
0.5501 m (3)
0.1109 g by Eq. 2
Eq. 1
12
(1.1 y) dR

3
(1.1 y) [ g(1.1 y) (y/2) dy]
1.1
0
(x , y)
y

1978
9.4 Resultants of Hydrostatic Pressure Forces: Example 9, page 6 of 8
1.5 m
E
w

E
Distributed load acting on Region 2
E
Pressure at point E (2.1 m) g
Distributed force (force per length), w

E
pressure width
2.1 g 1.5 m
3.15 g

Resultant force
R

2
(1/2)(2.1 m)(3.15 g)
3.3075 g (4)
14
Resultant force acting on Region 2 13
1.1 m + 1.0 m = 2.1 m

1979
9.4 Resultants of Hydrostatic Pressure Forces: Example 9, page 7 of 8
w

E
2.1 m
d

2
(2/3)(2.1) 1.4 m (5)
Line of action of R

2
passes through centroid of triangle. 15
Total force acting on end of tank
R = R

1
+ R

2
+ R

3
R

3
, by symmetry
R

2
+ 2R

3

3.3075 g + 2(0.1109 g)
3.5293 g (6)
16
Ans.
by Eq. 4
by Eq. 2
Using 10
3
kg/m
3
and g 9.81 m/s
2
gives
R 34.6 kN

1980
9.4 Resultants of Hydrostatic Pressure Forces: Example 9, page 8 of 8
d

2
= 1.4 m
d
d

1
= d
3
= 0.5501 m
x
z
y
d

3
= 0.5501 m
R

3
= 0.1109 g
R

2
= 3.3075 g
R

1
= R
3
= 0.1109 g
R
17 Determine the line of action of the resultant R by
equating the moment of R to the sum of moments of R

1
,
R

2
, and R

3
:
M
BC
: Rd R
1
d

1
+ R

2
d

2
+ R

3
d

3

Substituting the R, R
1
, d

1
, R

2
, d

2
, R

3
, and d

3
values
from the figures above into this equation and solving
gives
d = 1.347 m Ans.
B
C
+ +

1981

10. Inertia Properties of Plane Areas
1982

10.1 Moments of Inertia by Integration
1983
10.1 Moments of Inertia by Integration Procedures and Strategies, page 1 of 1
x
y
y = f(x)
(x, y)
Procedures and Strategies for Solving Problems
Involving Moments of Inertia by Integration.
To calculate the moment of inertia Ix about the x axis, use
one of the following two approaches:
1. If the curve bounding the planar region is given as a
function of x, y = f(x), use the formula
Ix = dIx
where, dIx = (1/3)y
3
dx = (1/3) f(x)
3
dx is the moment of
inertia (about the x axis) of a vertical rectangular strip of
length y (= f(x)) and width dx.
2. If the curve bounding the planar region is given as a
function of y, x = g(y), use the formula
Ix = y
2
dA
where the differential dA is the area of a horizontal strip
of length x and width dy.
To calculate I
y
, follow the same procedure described
above but with all x's replaced by y's and all original y's
replaced by x's.
x
y
x = g(y)
(x, y)
dx
dy

1984
10.1 Moments of Inertia by Integration Problem Statement for Example 1
1. Determine the moment of inertia of the rectangle
about the x axis, which passes through the centroid C.
C
b
h/2
x
y
h/2

1985
10.1 Moments of Inertia by Integration Problem Statement for Example 2
h
b
y
x
2. Determine the moment of inertia of the rectangle
about its base, which coincides with the x axis.

1986
10.1 Moments of Inertia by Integration Problem Statement for Example 3
3. Determine the moment of inertia of the
right triangle about the x axis.
x
y
h
b

1987
10.1 Moments of Inertia by Integration Problem Statement for Example 4
x
y
x
2
y
2
a
2
b
2
= 1 +
4. Determine the moments of inertia of the area
bounded by an ellipse about the x and y axes.
b
b
a a


1988
10.1 Moments of Inertia by Integration Problem Statement for Example 5
5. Determine the moments of inertia of the
crosshatched area about the x and y axes.
y = 4 x
2
x
y
4 ft
2 ft

1989
10.1 Moments of Inertia by Integration Problem Statement for Example 6
6. Determine the moment of inertia of the
crosshatched area about the y axis.
x = 2y
6
50y
5
y
3
+ 100
x
y
1.156 m
100 m
Scales on
the x and y
axes are
not the
same.

1990
10.1 Moments of Inertia by Integration Problem Statement for Example 7
x
y
y = 3x x
2
y = 9 x
2
7. Determine the moment of inertia of the
crosshatched area about the x axis.
9 m
3 m

1991
10.1 Moments of Inertia by Integration Problem Statement for Example 8
8. Determine the moment of inertia of the
crosshatched area about the y axis.
xy = 1
y = 2x
4 m
1 m 1 m 1/2 m
4 m 1/4 m
y
x

1992
10.1 Moments of Inertia by Integration Problem Statement for Example 9
9. Determine the moment of inertia of the
crosshatched area about the x axis.
y
x
y = 10e
x
2
x
2
e y = 10
1 m
1.5 m

1993
10.1 Moments of Inertia by Integration Problem Statement for Example 10
3 in.
1 in.
2 in.
4 in.
x
y
y = x
2
y = 3x
10. Determine the moment of inertia of the
crosshatched area about the y axis.

1994
10.1 Moments of Inertia by Integration Problem Statement for Example 11
C
x
2
+ y
2
= a
2
x
xc
yc
y
11. Given that the centroid C of the area bounded by a
quarter-circle lies a distance 4a/(3 above the base of the
quarter-circle, determine the moment of inertia of the
crosshatched area about an axis xc through the centroid.
a
4a
3

1995
10.1 Moments of Inertia by Integration Example 1, page 1 of 2
1. Determine the moment of inertia of the rectangle
about the x axis, which passes through the centroid C.
C
b
h/2
x
y
h/2

1996
10.1 Moments of Inertia by Integration Example 1, page 2 of 2
x
y
We want to evaluate
Ix = y
2
dA
where the differential element dA is located a
distance y from the x axis (y must have the same
value throughout dA).
y
h/2
h/2
Ix = y
2
dA

= y
2
(b dy)

by
3
3
bh
3
Ans.
12
=
=
Limits of integration
dA = area of rectangle
= b dy
1
2
3
4
h 2
-h 2
dy
h 2
-h 2
h 2
-h 2
C
b

1997
10.1 Moments of Inertia by Integration Example 2, page 1 of 2
h
b
y
x
2. Determine the moment of inertia of the rectangle
about its base, which coincides with the x axis.

1998
10.1 Moments of Inertia by Integration Example 2, page 2 of 2
b
y
x
We want to evaluate
Ix = y
2
dA
where the differential element of area dA is located a distance y from the
x-axis (y must have the same value throughout the element dA).
h
y
2 dA = area of rectangle

= b dy
3
=
0
h
Ix = y
2
dA

= y
2
(b dy)

by
3
3
= Ans.

Limits of integration 4
1
h
0
dy
bh
3
3

1999
10.1 Moments of Inertia by Integration Example 3, page 1 of 3
b
3. Determine the moment of inertia of the
right triangle about the x axis.
x
y
h

2000
10.1 Moments of Inertia by Integration Example 3, page 2 of 3
x
y
b
x
y
(x,y)
We want to evaluate
Ix = y
2
dA
where y has the same value throughout
differential element dA.
Equation of line
y = slope x + intercept
( ) x + h (1)
=
1
dA = area of rectangle
= x dy
h
Since we will integrate with respect to y,
we must replace x by a function of y.
Solve for x to get
x = ( ) y + b
so,
dA = x dy
= ( y + b) dy
4
5
3
2
dy
b
h
b
h
h
b

2001
10.1 Moments of Inertia by Integration Example 3, page 3 of 3
Ix = y
2
dA
= y
2
( y + b) dy
= b ( + y
2
) dy
= b[ + ]
= bh
3
[ + ]
= Ans.


6
Limits of integration are from
bottom of triangle to top.
7
h
0
b
h
y
3
h
y
4
4h
y
3
3
h
0
3
1 1
4
bh
3
12
0
h

2002
10.1 Moments of Inertia by Integration Example 4, page 1 of 3
x
y
x
2
y
2
a
2
b
2
= 1 +
4. Determine the moments of inertia of the area
bounded by an ellipse about the x and y axes.
b
b
a a


2003
10.1 Moments of Inertia by Integration Example 4, page 2 of 3
x
y
dy
x
y
We want to evaluate
Ix = y
2
dA
where y has the same value throughout the
differential element dA.
x = half the length of the
differential element,
so 2x = entire length
1
(x, y) dA = area of rectangle
= 2x dy
Since we will integrate with
respect to y, we must replace x
by a function of y
Since x locates a point to the right of the y axis, choose
the plus sign:

x = +a 1 (y/b)
2
(1)

+ 1 =
a
2
b
2
x
2
y
2
Solve for x to get
x = a 1 (y/b)
2
2
4
3
6
5

2004
10.1 Moments of Inertia by Integration Example 4, page 3 of 3
Evaluate the integral either with a
calculator that does symbolic
integration or use a table of integrals.
To calculate I
y
, use symmetry in the following way: in all the
above equations, replace x's by y's, original y's by x's, a's by b's
and original b's by a's. Then the result would be
I
y
= Ans.
Limits of integration from
bottom to top of ellipse
8
9
11
10
b
-b
4
ab
3
a
3
b
4
7 Using Eq. 1 in the expression for dA gives:
dA = 2x dy
= 2a 1 (y/b)
2
dy
Ix = y
2
dA
= y
2
(2a 1 (y/b)
2
) dy
= Ans.


2005
10.1 Moments of Inertia by Integration Example 5, page 1 of 4
2 ft
5. Determine the moments of inertia of the
crosshatched area about the x and y axes.
y = 4 x
2
x
y
4 ft

2006
10.1 Moments of Inertia by Integration Example 5, page 2 of 4
2 ft
We want to evaluate
I

x = y
2
dA
where y has the same value throughout the differential element dA.
dA = area of rectangle
= x dy
Since we will integrate with respect to y,
we must replace x by a function of y.
Solving for x to get

x = 4 y
Since x locates a point to the right of the y axis, choose the plus sign:
x = + 4 y (1)
1
2
3
4
5
y = 4 x
2
(x,y)
y
dy
x
y
4 ft

2007
10.1 Moments of Inertia by Integration Example 5, page 3 of 4
Using Eq. 1 in the expression for dA gives
dA = x dy
= 4 y dy

6
Ix = y
2
dA
= y
2
4 y dy
= 19.50 ft
4
Ans.

Use the integral function
on your calculator
7
8
9
y = 4 at the top of
the crosshatched
area
4
0

2008
10.1 Moments of Inertia by Integration Example 5, page 4 of 4
2 ft
Next we want to evaluate
I
y
= x
2
dA
where x has the same value throughout the differential element dA. Thus, we
choose a vertical rectangular strip.
dA = area of rectangle
= y dx
= (4 x
2
) dx
I
y
= x
2
dA = x
2
(4 x
2
) dx = 4.27 ft
4
Ans.
dx
x
10
11
12
0
2
y
y = 4 x
2
x
y
4 ft

2009
10.1 Moments of Inertia by Integration Example 6, page 1 of 3
6. Determine the moment of inertia of the
crosshatched area about the y axis.
x = 2y
6
50y
5
y
3
+ 100
x
y
1.156 m
100 m
Scales on
the x and y
axes are
not the
same.

2010
10.1 Moments of Inertia by Integration Example 6, page 2 of 3
100 m
1 We want to evaluate
I
y
= x
2
dA
where x has the same value throughout the differential
element dA. Thus, we choose a vertical rectangular strip.
dA = area of rectangle
= y dx
But this approach won't work because we
can't express y as a function of x.
2
3
y
x = 2y
6
50y
5
y
3
+ 100
x
y

2011
10.1 Moments of Inertia by Integration Example 6, page 3 of 3
4 An alternative approach is to use a horizontal
rectangular strip and employ the equation for the
moment of inertia of a rectangle about its base (BB)
:
(1)
B
B
h
b
100 m
1.156 m
y
Applying Eq.1 to the differential element
gives the differential moment of inertia.
dI
y


(dy)x
3
=
3
5
Replacing x by the function of y gives
dI
y
= x
3
dy
= (2y
6
50y
5
y
3
+ 100)
3
dy
I
y
= dI
y
= (2y
6
50y
5
y
3
+ 100)
3
dy
= 2.72 10
5
m
4
Ans.
Use the integral function on a calculator to evaluate
this integral.
6
7
8
x = 2y
6
50y
5
y
3
+ 100
x
x
dy
bh
3
3
I

BB
=
=
3
bh
3
3
1
3
1
3
1
0
1.156

2012
10.1 Moments of Inertia by Integration Example 7, page 1 of 3
y = 3x x
2
y = 9 x
2
7. Determine the moment of inertia of the
crosshatched area about the x axis.
9 m
3 m
x
y

2013
10.1 Moments of Inertia by Integration Example 7, page 2 of 3
x
y
We want to evaluate
Ix = y
2
dA
where y has the same value throughout the differential
element dA. Thus it appears that we should use a
horizontal rectangular strip.
But using a horizontal strip is awkward we would
have to use three different expressions for dA,
depending on the position of the strip.
2
1
A better approach is to use a vertical strip and then
apply the parallel-axis theorem to the strips.
Parallel-axis theorem
for a general region
Ix = Ic + Ad
2
d
Centroid
Area
A
x
y
For a rectangle in particular,
Ix = bh
3
+ (b h)d
2
= ( h
3
+ hd
2
)b
y
x
y
x
C
d
C
h
b
h
b dx
d
The moment of inertia of a strip of width
b dx is then
dIx = ( h
3
+ hd
2
) dx (1)
3
12
1
1
12
12
1
C

2014
10.1 Moments of Inertia by Integration Example 7, page 3 of 3
x
y
(x
el
,y
el
)
(x,y
1
)
(x,y
2
)
The y-coordinate of the element
centroid is the average of y
1
and y
2
d = y
el
= ( y
1
+ y
2
)
Eq. 1 becomes
dIx = ( h
3
+ hd
2
) dx
= { ( y
2
y
1
)
3
+ ( y
2
y
1
) [ ( y
1
+ y
2
)]
2
} dx
Since the point (x, y
2
) lies on the curve y = 9 x
2
, we
can substitute

y
2
= 9 x
2
in Eq. 2. Similarly, since (x, y
1
) satisfies y = 3x x
2
, we
can substitute
y
1
= 3x x
2

in Eq. 2. Thus Eq. 2 becomes
5
6
h = y
2
y
1
4
y
el
centroid
y = 3x x
2
y = 9 x
2
dIx = { ( y
2
y
1
)
3
+ ( y
2
y
1
) [ ( y
2
+ y
1
)]
2
} dx
= { [( 9 x
2
) ( 3x x
2
)]
3
+ [( 9 x
2
)
( 3x x
2
)][ ( 9 x
2
) + ( 3x x
2
)]
2
} dx
= { ( 9 3x )
3
+ ( 9 3x )[ + x x
2
]
2
} dx
Ix = dIx
= { ( 9 3x )
3
+ (9 3x) ( + x x
2
)
2
} dx

= 328 m
4
Ans.
Enter this expression directly into the
integral function of a calculator.
7
8
x ranges
from 0 to 3.
2
1
12
1
1
12 2
1
12
1 1
2
1
12
2
9 3
2
9
2 2
3
0
3
12
1
1
12
1
2 2
1
(2)

2015
10.1 Moments of Inertia by Integration Example 8, page 1 of 4
8. Determine the moment of inertia of the
crosshatched area about the y axis.
xy = 1
y = 2x
4 m
1 m 1 m 1/2 m
4 m 1/4 m
y
x

2016
10.1 Moments of Inertia by Integration Example 8, page 2 of 4
We want to evaluate

I
y
= x
2
dA
where x has the same value throughout the differential
element dA. Thus it appears that we should use
vertical differential strips.
But using a vertical strip is awkward
we would have to use three different
expressions for dA, depending on the
location of the strip.
2
1
x
y

2017
10.1 Moments of Inertia by Integration Example 8, page 3 of 4
3 A better approach is to use a horizontal strip and then
apply the parallel-axis theorem to the strip.
x
y
x
y
y
x
area A
centroid
C
Parallel-axis theorem for a general region
I
y
= Ic + Ad
2
For a rectangle in particular,
I
y
= bh
3
+ (b h)d
2
= ( h
3
+ hd
2
)b
The moment of inertia of a strip of width b dy is then
dI
y
= ( h
3
+ hd
2
) dy (1)
C
C
d
d
d
Area = b h
b
h
b dy
h
12
1
12
1
12
1

2018
10.1 Moments of Inertia by Integration Example 8, page 4 of 4
| x
el
| = | average of x
1
and x
2
| = | ( x
1
+ x
2
)|
d =
h = x
2
x
1
y
x
xy = 1
y = 2x
2
y
2
y
1
12 4
y
2y
1
2
y
2
y
y
1
y
1
4
1
y
4
1
2y
1
12 2
y
1
y
y
2
1
y
1
y
2
1
y
2
y
1
12
1
y
1
1
2
1
12
1
y
2
1
12
1
12
1
2
1
y
2
8
7
6
5
Top of region
at y = 4
Bottom of
region at y = 1
= 2.81 m
4
Ans.
Enter this expression directly into the
integral function of a calculator
I
y
= dI
xy
= [ ( + )
3
+ ( + ) ( )
2
] dy
dI
y
= { ( x
2
x
1
)
3
+ ( x
2
x
1
)[ ( x
2
+ x
1
)]
2
} dy
= { ( ( ))
3
+ ( ( ))[ ( + ( ) )]
2
} dy
= { ( + )
3
+ ( + )[ ]
2
} dy


Since the point (x
1
,y) lies on the line y = 2x,
we can solve for x and substitute
x
1
=
in Eq. 2. Similarly, since (x
2
,y) lies on the
curve xy = 1, we can substitute
x
2
=
in Eq. 2. Thus Eq. 2 becomes
4
Centroid of
strip
From the figure, we see that Eq. 1 can be written as
dI
y
= { h
3
+ hd
2
} dy (Eq. 1 repeated)
= { ( x
2
x
1
)
3
+ ( x
2
x
1
)[ ( x
2
+ x
1
)]
2
} dy (2)
(x
2
,y)
(x
1
,y)
(x
el
,y
el
)
C

2019
10.1 Moments of Inertia by Integration Example 9, page 1 of 3
9. Determine the moment of inertia of the
crosshatched area about the x axis.
y
x
y = 10e
x
2
x
2
e y = 10
1 m
1.5 m

2020
10.1 Moments of Inertia by Integration Example 9, page 2 of 3
1
2
3 Ix for region D
No integration needed.
Use the formula for the moment of inertia of a
rectangle about a centroidal axis.
4
b
h/2
h/2
So
=
=
C
I
xD
12
bh
3
Ic = (1)

12

33.1915 m
4
(2)
x
y
3.6788 m
3.6788 m
1 m
(1 m)(2 3.6788 m)
3
y(1) = 10e = 3.6788 m
(1)
2
1 m
We want to evaluate
Ix = y
2
dA
Because of the shape of the region, the integral has to
be evaluated over two sub-regions, D and E.
Region E
Region D

2021
10.1 Moments of Inertia by Integration Example 9, page 3 of 3
5 Ix for region E. Because the region is
symmetrical about the x axis, we can save
work by using vertical rectangular strips and
by applying Eq. 1 to these strips:

Ic = bh
3
dI
xE
= (dx)(2y)
3

= (dx)[2( )]
3


= dx

Thus, with the aid of the integral function on a
calculator, we have
I
xE
= dIx

= e dx

= 4.7985 m
4
(3)
Adding the results for regions D and E gives
Ix = I
xD
+ I
xE

= 33.1915 + 4.7985
= 38.0 m
4
Ans.
by Eq. 2 by Eq. 3
6
12
1
1
12
1
12
2000
3
1
1.5
3
2000
or,
x
y
dx
1 m
x
1.5 m
10e
x
2
x
2
e y = 10
3x
2
e
3x
2

2022
10.1 Moments of Inertia by Integration Example 10, page 1 of 3
1 in.
2 in.
4 in.
x
y
y = x
2
y = 3x
10. Determine the moment of inertia of the
crosshatched area about the y axis.
3 in.

2023
10.1 Moments of Inertia by Integration Example 10, page 2 of 3
1
2
3 Integrate over region B.
y
x
x
1 in.
(x,y
2
)
(x,y
1
)
y = 3x
y = x
2
dA = area of rectangle
= ( y
2
y
1
) dx
= ( 3x x
2
) dx
4
5 I
yB
= x
2
dA
= x
2
(3x x
2
) dx
= 0.5500 in
4
(2)
1
0
dx
y
x
y
2
y
1
We want to evaluate

I
y
= x
2
dA (1)
where x has the same value throughout the differential
element dA.
Given the shape of the area, we have to evaluate the
integral in Eq.1 over two sub-regions, B and C.
Region C
Region B

2024
10.1 Moments of Inertia by Integration Example 10, page 3 of 3
x
y
6 Integrate over region C.
4 in.
x
y
2
y
1

dx
2 in.
(x,y
1
)
y = x
2
(x,y
2
)
Write the equation of the line:

=
Solving gives
y = x + 2
dA = Area of rectangle

= ( y
2
y
1
) dx
= [(x + 2) x
2
] dx


I
yC
= x
2
dA
= x
2
( x + 2 x
2
) dx
= 2.2167 in
4
(3)
Adding the results for regions B and C gives
I
y
= I
yB
+ I
yC

= 0.5500 + 2.2167 = 2.77 in
4
Ans.
by Eq. 2 by Eq. 3
7
8
9
10
2 1
x 1
y 3
4 3
1
2
1 in.
3 in.

2025
10.1 Moments of Inertia by Integration Example 11, page 1 of 2
Area of quarter circle
4
a
2
4a
3
4a
4
9
3
4a
a
11. Given that the centroid C of the area bounded by a
quarter-circle lies a distance 4a/(3 above the base of the
quarter-circle, determine the moment of inertia of the cross
hatched area about an axis xc through the centroid.
y
yc
xc
x
x
2
+ y
2
= a
2
C
We want to evaluate
Ixc = yc
2
dA
but because the equation of the circle is given in
terms of x and y instead of xc and yc, it is easier to
evaluate
Ix y
2
dA
and then use the parallel-axis theorem:
Ix = Ixc + Ad
2
= Ixc + ( )
2
Solving gives
Ixc = Ix
Thus now we need to calculate Ix.
1

2026
10.1 Moments of Inertia by Integration Example 11, page 2 of 2
x
2
+ y
2
= a
2
x
y
2
y
x
(x,y)
dA = Area of rectangle
= x dy
Solve x
2
+ y
2
= a
2
to get
x = a
2
y
2
Substitute into the equation for dA
dA = x dy
= a
2
y
2
dy
Integrate

Ix = y
2
dA
= y
2
a
2
y
2
dy
= (2)
a
0
Use the result given by Eq. 2 in Eq. 1:

I
xC
= Ix (Eq. 1 repeated)

( ) a
4
Ans.
Evaluate the integral with a
calculator that does symbolic
integration or use a table.
3
4
5
6
7
dy
16
4a
4
9
9
4a
4
4
9
a
4
16
a
a
4
16

2027

10.2 Method of Composite Areas
2028
10.2 Method of Composite Areas Procedures and Strategies, page 1 of 1
Procedures and Strategies for Solving Problems Involving
Moments of Inertia of Composite Bodies
1. Divide the given region into a collection of simple
sub-regions that are listed in a table of geometric properties of
planar regions. Sometimes it is convenient to subtract one
region from another, when dividing the given region.
2. If the moment of inertia is to be calculated with respect to
an axis through the centroid of the entire region, then follow
the procedure that you have already learned for locating the
centroid.
3. Calculate the moment of inertia of each sub-region with
respect to an axis through the centroid of the sub-region.
4. If the centroidal axis of the sub-region does not coincide
with the moment of inertia axis for the entire given region,
then use the parallel-axis theorem, I = Ic + Ad
2
, where Ic is the
moment of inertia with respect to the sub-region centroidal
axis, A is the area, and d is the distance between the
centroidal axis of the sub-region and the x axis (the axis about
which the moment of inertia is to be found).
5. The parallel-axis theorem requires that you know Ic but
sometimes tables of geometric properties do not give you that
information directly and you must use the the parallel-axis
theorem to compute Ic.
=
x
Centroidal axis of semicircular region
x'
C
x
The table gives Ix', not what
you want, Ic.
The parallel-axis theorem says
Ix' = Ic + A(d')
2
Solve for Ic:
Ic = Ix' A(d')
2
Now that Ic is known, you can
get the moment of inertia with
respect to x by using the
parallel-axis theorem again:
Ix = Ic Ad
2
y y y
d
d'
x x

2029
10.2 Method of Composite Areas Problem Statement for Example 1
1. Determine the moment of inertia of the crosshatched region
about the x axis.
y
x
50 mm
20 mm

2030
10.2 Method of Composite Areas Problem Statement for Example 2
2. The figure shows the cross section of a
beam made by gluing four planks together.
Determine the moment of inertia of the
cross section about the x axis.
150 mm
60 mm
60 mm
60 mm
200 mm
60 mm
200 mm
x
y
150 mm


2031
10.2 Method of Composite Areas Problem Statement for Example 3
3. Determine the moment of inertia of the
beam cross section about the x centroidal
axis.
80 mm 80 mm
120 mm
120 mm
y
x
20 mm
20 mm
20 mm

2032
10.2 Method of Composite Areas Problem Statement for Example 4
110 mm 110 mm 10 mm
140 mm
140 mm
y
x
C
10 mm
A = 877 mm
2
Ixc = 0.202 10
6
mm
4
16.2 mm
x

c
10 m
4. A composite beam is constructed
from three plates and four standard
rolled-steel angles. Determine the
moment of inertia of the cross
section about the x centroidal axis.

2033
10.2 Method of Composite Areas Problem Statement for Example 5
4 in.
6 in.
x
y
5. Determine the moment of inertia of the
trapezoidal region about the x and y axes.
3 in. 3 in.
4 in.

2034
10.2 Method of Composite Areas Problem Statement for Example 6
0.8 in.
2 in.
0.8 in.
2 in.
x
y
6. Determine the moment of inertia of the
crosshatched region about the y axis.
4 in. 4 in.

2035
10.2 Method of Composite Areas Problem Statement for Example 7
425 mm
50 mm
50 mm
y
x
75 mm
7. A precast concrete floor beam has the cross section shown.
Locate the centroid of the section and determine the moment
of inertia about a horizontal axis through the centroid.
300 mm 300 mm
250 mm 250 mm

2036
10.2 Method of Composite Areas Problem Statement for Example 8
254 mm
A = 3780 mm
2
Ixc = 32.6 10
6
mm
4

I
yc
= 1.14 10
6
mm
4
127 mm
15.3 mm
C
xc
Centroid of channel
yc
127 mm
8. A beam is built up from two standard rolled-steel channels and a
cover plate. Locate the centroid of the section and determine the
moments of inertia with respect to horizontal and vertical axes through
the centroid.
100 mm 100 mm
40 mm
y
x
2037
10.2 Method of Composite Areas Example 1, page 1 of 2
1. Determine the moment of inertia of the crosshatched region
about the x axis.
y
x
50 mm
20 mm
1 Consider the crosshatched region to
be composed of the difference of
two circular regions.
x
y
x
y
x
y
=

2038
10.2 Method of Composite Areas Example 1, page 2 of 2
2
3 For our particular problem,
Large circle I

x = (50 mm)
4
= 4.9087 10
6
mm
4
Small circle I

x = (20 mm)
4
= 0.1257 10
6
mm
4
For the composite region, subtracting gives
I

x = Large circle I

x Small circle I

x
= 4.9087 10
6
mm
4
0.1257 10
6
mm
4
= 4.78 10
6
mm
4
Ans.
1
4
1
4

A table of properties of planar regions gives the
information shown below.
Circle
I
1
=
1
4
=
4
Iy
x

Moment of Inertia
C
r
y
x
r
4
r
4

2039
10.2 Method of Composite Areas Example 2, page 1 of 2
2. The figure shows the cross section of a
beam made by gluing four planks together.
Determine the moment of inertia of the
cross section about the x axis.
Large rectangle
150 mm
420 mm
520 mm
60 mm
60 mm
60 mm
150 mm + 150 mm = 300 mm
200 mm + 200 mm = 400 mm
400 mm
x
y
Small rectangle
300 mm
y
x
200 mm
60 mm
200 mm
x
y
150 mm
60 mm + 200 mm + 200 mm + 60 mm = 520 mm
1 Consider the crosshatched region to be consist of a
small rectangle subtracted from a large rectangle.
60 mm + 150 mm + 150 mm + 60 mm = 420 mm
y
=
x


2040
10.2 Method of Composite Areas Example 2, page 2 of 2
2
3 For our particular problem,
Large rectangle I

x =
=
= 4.9213 10
9
mm
4
4
For the composite region, subtracting gives
I

x = Large rectangle I

x Small rectangle I

x
= 4.9213 10
9
mm
4
1.6000 10
9
mm
4
= 3.32 10
9
mm
4
Ans.
420 mm
520 mm
300 mm
400 mm
x
y
x
y
(420 mm)(520 mm)
3

12
For our particular problem,
Small rectangle I

x =
=
= 1.6000 10
9
mm
4
(300 mm)(400 mm)
3

12
5
bh
3

12
bh
3

12

A table of properties of planar regions
gives the information shown below.
I
h
h
2


12
hb
y =
3

I
12
bh
= x
3
y
C
x
Rectangle
Moment of Inertia
2
b
2
b
2

2041
10.2 Method of Composite Areas Example 3, page 1 of 3
3. Determine the moment of inertia of the
beam cross section about the x centroidal
axis.
80 mm 80 mm
120 mm
120 mm
y
x
20 mm
20 mm
20 mm

2042
10.2 Method of Composite Areas Example 3, page 2 of 3
x
1 Consider the cross section to be composed of a large rectangle minus two small rectangles.
y
x
y
x
y
x
20 mm + 120 mm + 120 mm + 20 mm = 280 mm
80 mm + 20 mm + 80 mm = 180 mm
120 mm + 120 mm = 240 mm
280 mm
180 mm 80 mm
240 mm
240 mm
80 mm
=
2 Two regions of the same size and same position
relative to the x axis can be combined into 2
times a single region.
x
2
y
x
=
20 mm
20 mm
20 mm
80 mm
120 mm
120 mm
80 mm
y
x
y

2043
10.2 Method of Composite Areas Example 3, page 3 of 3
3 Large rectangle I

x =
=
= 3.2928 10
8
mm
4
4
For the composite region, subtracting gives
I

x = Large rectangle I

x 2 Small rectangle I

x
= 3.2928 10
8
mm
4
2(0.9216 10
8
mm
4
)
= 1.450 10
8
mm
4
Ans.
bh
3

12
(180 mm)(280 mm)
3

12
Small rectangle I

x =
=
= 0.9216 10
8
mm
4
(80 mm)(240 mm)
3

12
5
x
x
y
280 mm
180 mm
240 mm
80 mm
bh
3

12
x
y

2044
10.2 Method of Composite Areas Example 4, page 1 of 3
110 mm 110 mm 10 mm
140 mm
140 mm
y
x
1 Consider the cross section to be composed of four angles and three rectangles.
y
x
y
x
y
x
y
x
=
+ 2 + 4
The top and bottom
plates are identical.
The four angles
are identical.
2
3
C
10 mm
16.2 mm
C
x

c
A = 877 mm
2
Ixc = 0.202 10
6
mm
4
4. A composite beam is constructed
from three plates and four standard
rolled-steel angles. Determine the
moment of inertia of the cross
section about the x centroidal axis.
10 mm

2045
10.2 Method of Composite Areas Example 4, page 2 of 3
Middle rectangle
Middle rectangle I

x =
=
= 1.8293 10
7
mm
4
(1)
(10 mm)(140 mm + 140 mm)
3

12
4
140 mm
140 mm
x
y
10 mm
Upper rectangle
Use parallel axis theorem:
I

x = I

xc' + d
2
A (2)
C'
5
110 mm
140 mm
110 mm
x
y
x

c'
10 mm
2
= 5 mm
Moment of inertia
about axis x

c' through
centroid of rectangle
Distance between x

c' and x
Area
Here,
I

xc' = =
= 18333 mm
4
d = 140 mm + 5 mm = 145 mm
A = (110 mm + 110 mm)(10 mm)
= 2200 mm
2
(110 mm + 110 mm)(10 mm)
3

12
6
bh
3

12
bh
3

12
C'
10 mm

2046
10.2 Method of Composite Areas Example 4, page 3 of 3
7
Angle
A = 877 mm
2
Ixc = 0.202 10
6
mm
4
8
x
y
16.2 mm
C
x

c
The parallel axis theorem gives
Angle I

x = Ixc + d
2
A
= 0.202 10
6
mm
4
+ (123.8 mm)
2
(877 mm
2
)
= 1.3643 10
7
mm
4
(3)
9
16.2 mm
C'
140 mm
d = 140 mm 16.2 mm = 123.8 mm
For the composite region, adding gives
Ix = Middle rectangle Ix + 2 Upper rectangle Ix + 4 Angle Ix
= 1.8293 10
7
mm
4
+ 2(4.6273 10
7
mm
4
) + 4(1.3643 10
7
mm
4
)
= 165.4 10
6
mm
4
Ans.
10
The parallel axis theorem, Eq. 2, now gives
Upper rectangle I

x = I

xc' + d
2
A
= 18333 mm
4
+ (145 mm)
2
(2200 mm
2
)
= 4.6273 10
7
mm
4

2047
10.2 Method of Composite Areas Example 5, page 1 of 4
4 in.
6 in.
x
y
4 in.
3 in.
6 in.
4 in.
x
3 in.
y
3 in. 3 in.
3 in. 1 in.
6 in.
x
6 in.
x
y
y
=
+ 2
5. Determine the moment of
inertia of the trapezoidal region
about the x and y axes.
1 Consider the trapezoid to be the sum of a rectangle and two triangles.
3 in. 3 in.
4 in.

2048
10.2 Method of Composite Areas Example 5, page 2 of 4
Since the centroid C' does not lie on the x axis, we have to use
the parallel axis theorem to calculate I

x.
Rectangle I

x = I

xc' + d
2
A
=
+ (3 in.)
2
[(3 in. + 3 in.)(6 in.)]
= 432 in
4
(1)
(6 in.)(3 in. + 3 in.)
3

12
3
y
x
2 I

x and I

y
for the rectangle
6 in.
3 in.
3 in.
x

c'
C'
Since the centroid C' lies on the y axis, we do not have to
use the parallel axis theorem for I

y
.
Rectangle I

y
= I

yc'
=
= 108 in
4
(2)
(3 in. + 3 in.)(6 in.)
3

12
4

2049
10.2 Method of Composite Areas Example 5, page 3 of 4
For our particular triangle
Parallel axis theorem applied to triangle
in.
3 in.
3
7
d

x = = 2 in.
Triangle
B
6 in.
1
d

y
= 3 in. + in. = 3.3333 in.
3
3
1
3
Area, A = (1 in.)(6 in.)
= 3 in
2
2
1
b
B
y
C
Moment of Inertia
A table of properties of planar regions gives the
information below.
for the triangle
bh
3
I
I
x

3
h
2h

3
hb
= BB
12
= xx
36
5 and I Ix
y

x
2
Area =
bh
6
y
Triangle I

x = I

xc' + dx
2
A
= 6 in
4
+ (2 in.)
2
(3 in
2
)
= 18 in
4
(3)
Triangle I

y
= I

yc'
+ d
2
A
= 0.1667 in
4

+ (3.3333 in.)
2
(3 in
2
)
= 33.4994 in
4
(4)
6 in.
C'
1 in.
x
x

c'
y

c'
36
(1 in.)(6 in.)
3

b'(h')
3

36
(6 in.)(1 in.)
3

I

xc' =
=
= 6 in
4

I

yc'
=
=
= 0.1667 in
4
6 in.
1 in.
C'
x
x

c' 36
y

c'
bh
3

36

2050
10.2 Method of Composite Areas Example 5, page 4 of 4
8 For the composite region, using Eqs. 1, 2, 3, and 4 gives
I

x = Rectangle I

x + 2 Triangle I

x
= 432 in
4
+ 2(18 in
4
)
= 468 in
4
Ans.
` I

y
= Rectangle I

y
+ 2 Triangle I

y
= 108 in
4
+ 2(33.4994 in
4
)
= 175 in
4
Ans.

2051
10.2 Method of Composite Areas Example 6, page 1 of 5
0.8 in.
2 in.
0.8 in.
2 in.
x
y
6. Determine the moment of inertia of the
crosshatched region about the y axis.
1 Consider the crosshatched region to be composed of a rectangle
minus two circular regions plus two semicircular regions.
x
y
x
y
x
y
x
=
+ 2 2
4 in. 4 in.
y

2052
10.2 Method of Composite Areas Example 6, page 2 of 5
I

y
for rectangle
4 in. 4 in.
2 in.
2 in.
3
C'
(2 in. + 2 in.)(4 in. + 4 in.)
3

Since the y axis passes through the centroid of the
rectangle, the parallel axis theorem is not needed.
Rectangle I

y
= I

yc'
=
= 170.6667 in
4
(1)
12

x
y
2

2053
10.2 Method of Composite Areas Example 6, page 3 of 5
x
y
4
5
y

c'
r = 0.8 in.
C'
d = 4 in.
For the circular region, a table of properties of planar
regions gives the information shown below.
Moment of Inertia
For our particular problem,
I
yc'
= (0.8 in.)
4
= 0.3217 in
4
Area, A = r
2

= (0.8 in.)
2

= 2.0106 in
2
Applying the parallel axis theorem gives
Circle I

y
= I

yc'
+ d
2
A
= (0.3217 in
4
) + (4 in.)
2
(2.0106 in
2
)

= 32.4913 in
4
(2)
4
1
6
Circle
I
1
=
1
4
=
4
Iy
x

C
r
y
x
r
4
r
4

2054
10.2 Method of Composite Areas Example 6, page 4 of 5
y
x
4 in.
y

c'
C'
r = 2 in.
We would like to apply the
parallel-axis theorem:
I

y
= I

yc'
+ d
2
A (3)
to compute I

y
for the semicircle.
Unfortunately, the table gives us the
moment of inertia with respect to the
base, BB, of the semicircle, not with
respect to the axis through the
centroid yc'.
8 7

B
B
For the semicircle, a table of properties of
planar regions gives the information shown
below.
9 But we can still make use of the result I

BB
from the table by
applying the parallel axis theorem between the BB axis and
the y

c ( y

c') axis:
I

BB
= I

yc'
+ d
2
A
or,
= I

yc'
+ ( )
2
( )
Solving gives
I

yc'
= ( )r
4
(4)
4r
3

8 9
8
r
4

8
r
2

2
I
BB
=
r
4

8
3
4 r


C
x
= I
r
4

3
y

c =
y

c
I =
r
4

y
8
4r
x
8
r
y
Semicircle
Moment of Inertia

2055
10.2 Method of Composite Areas Example 6, page 5 of 5
y

c'
C'
r = 2 in.
Eqs. 3 and 4 can now be applied to the semicircular region
Semicircle I

y
= I

yc'
+ d
2
A
= ( )r
4
+ (4 in. + )
2
( )
Substituting r = 2 in. and evaluating the resulting expression gives
Semicircle I

y
= 149.4808 in
4
(5)
For the composite region, using Eqs. 1, 2, and 5 gives
I

y
= Rectangle I

y
2 Circle I

y
+ 2 Semicircle I

y
= 170.6667 in
4
2(32.4913 in
4
) + 2(149.4808 in
4
)
= 405 in
4
Ans.
3
4r
8
8
9
10
4r
3

11
r
2
2
y
x
4 in.

2056
10.2 Method of Composite Areas Example 7, page 1 of 5
7. A precast concrete floor beam has the cross section shown.
Locate the centroid of the section and determine the moment of
inertia about a horizontal axis through the centroid.
300 mm 300 mm
250 mm 250 mm
425 mm
50 mm 50 mm
y
x
1 Definition of centroid
X

c = 0, by symmetry
Y

c = (1)
where y

c is the centroidal coordinate of the
region with area A.
A
y

cA
75 mm

2057
10.2 Method of Composite Areas Example 7, page 2 of 5
y
Consider the section to be composed of a horizontal
rectangle and and two identical vertical rectangles.
2
=
y
x
x
y
x
+ 2

2058
10.2 Method of Composite Areas Example 7, page 3 of 5
y
x
425 mm
y

c'
C' (centroid)
4
425 mm
2
y

c'
50 mm
Region
upper rectangle
lower rectangles
A ( mm
2
)
A = 13.250 10
4
y

c' ( mm ) y

c'A ( mm
3
)
y

c'A = 50.656 10
6
9.031 10
6
41.625 10
6
9.000 10
4
2(2.125 10
4
)
462.5
212.5
5
Set up table
2 lower rectangles
C' (centroid)
425 mm
Upper rectangle
A = (1200 mm)(75 mm)
= 9 10
4
mm
2
y

c' = 425 mm + 37.5 mm
= 462.5 mm
y

c'
= 37.5 mm
75 mm
y
75 mm
2
3
x
2 (250 mm + 50 mm + 300 mm) = 1200 mm
Lower rectangle
A = (425 mm)(50 mm)
= 2.125 10
4
mm
2
y

c' =
= 212.5 mm

2059
10.2 Method of Composite Areas Example 7, page 4 of 5
y
x
425 mm
75 mm
Y

c = 382.31 mm
7
(1200 mm)(75 mm)
3

I

x of upper rectangle
I

xc' =
=
= 4.2188 10
7
mm
4
d = 75 mm + 425 mm
(37.5 mm + 382.31 mm)
= 80.19 mm
12
bh
3

12
d
C' (centroid of rectangle)
C (centroid of entire section)
x

c'
x

c
Parallel axis theorem:
Upper rectangle Ixc = I

xc' + d
2
A
= 4.2188 10
7
mm
4
+ (80.19 mm)
2
(9 10
4
mm
2
)
= 6.2093 10
8
mm
4
(2)
Eq. 1 gives the distance to the centroid of the entire cross section.
Y

c =
=
= 382.31 mm Ans.
50.656 10
6

13.250 10
4

6
y

cA
A
Area A was calculated previously (See the table).
= 37.5 mm
75 mm
2

2060
10.2 Method of Composite Areas Example 7, page 5 of 5
y
x
425 mm
= 212.5 mm
425 mm
2
d
C' (centroid of rectangle)
C (centroid of entire section)
x

c'
x

c
12
Parallel axis theorem:
Lower rectangle Ixc = I

xc' + d
2
A
= 3.1986 10
8
mm
4
+ (169.81 mm)
2
(2.125 10
4
mm
2
)
= 9.3261 10
8
mm
4
(3)
I

x of lower rectangle
I

xc' =
=
= 3.1986 10
8
mm
4
d = 382.31 mm 212.5 mm
= 169.81 mm
(50 mm)(425 mm)
3

12
bh
3

8
y

c'
For the composite region, using Eqs. 1and 2 gives
Ixc = Upper rectangle Ixc + 2 Lower rectangle Ixc
= 6.2093 10
8
mm
4
+ 2(9.3261 10
8
mm
4
)
= 249 10
7
mm
4
Ans.
9
Area A was calculated previously.
50 mm
Y

c = 382.31 mm

2061
10.2 Method of Composite Areas Example 8, page 1 of 6
254 mm
A = 3780 mm
2
Ixc = 32.6 10
6
mm
4

I
yc
= 1.14 10
6
mm
4
127 mm
15.3 mm
C
xc
Centroid of channel
yc
127 mm
Definition of centroid
X

c = 0, by symmetry
Y

c = (1)
where y

c is the centroidal coordinate of the region with area A.
y

cA
A
1
8. A beam is built up from two standard rolled-steel channels and a
cover plate. Locate the centroid of the section and determine the
moments of inertia with respect to horizontal and vertical axes through
the centroid.
100 mm 100 mm
40 mm
y
x

2062
10.2 Method of Composite Areas Example 8, page 2 of 6
y
Consider the cross section to be composed of a rectangle and two channels. 2
x
y
x
y
+ 2
=
3
x
y
y

c' = 127 mm
C'
Centroid of channel

x
127 mm
127 mm
Note that yc' is known.

2063
10.2 Method of Composite Areas Example 8, page 3 of 6
x
y
Rectangle
Area, A = (40 mm)(100 mm + 100 mm)
= 8000 mm
2

Locate centroid of rectangle:
y

c' = 254 mm + 20 mm
= 274 mm
40 mm
2
4
y

c'
40 mm
100 mm 100 mm
= 20 mm
y

c'A = 3.1521 10
6
Rectangle
Set up table
6
Channel
Region
8000
2(3780)
A = 15560
A ( mm
2
)
274
127
y

c' ( mm )
2 channels (area was given)
0.9601 10
6
2.1920 10
6
y

c'A ( mm
3
)
Eq. 1 gives the distance to the centroid of the entire cross section:
Y

c = (Eq. 1 repeated)
=
= 202.58 mm Ans.
15560
3.1521 10
6

7
y

cA
A
C'
5
127 mm + 127 mm = 254 mm

2064
10.2 Method of Composite Areas Example 8, page 4 of 6
C'
8
127 mm
x

c'
x

c
y

c'
Centroid of entire beam section
C
I

xc and I

yc
of channels (I

xc', I

yc'
, and A are given.)
Use the parallel axis theorem.
Channel I

xc = I

xc' + (dx)
2
A
= 32.6 10
6
mm
4
+ (202.58 mm 127 mm)
2
(3780 mm
2
)
= 54.193 10
6
mm
4
(2)
Channel I

yc
= I

yc'
+ (d
y
)
2
A
= 1.140 10
6
mm
4
+ (15.3 mm)
2
(3780 mm
2
)
= 2.025 10
6
mm
4
(3)
d
y
= 15.3 mm
dx
Centroid of
channel
x
y
Y

c = 202.58 mm

2065
10.2 Method of Composite Areas Example 8, page 5 of 6
100 mm
254 mm
= 20 mm 100 mm
40 mm
2
40 mm
y
C'
x
Y

c = 202.58 mm
x

c
x

c'
C
Centroid of entire beam section
I

xc and I

yc
of upper rectangle
I

xc' =

=
= 1.067 10
6
mm
4
d = (40 mm + 254 mm) (20 mm + 202.58 mm)
= 71.42 mm
Use the parallel axis theorem.
Rectangle I

xc = I

xc' + d
2
A
= 1.067 10
6
mm
4
+ (71.42 mm)
2
(8000 mm
2
)
= 41.874 10
6
mm
4
(4)
Rectangle I

yc
=
=
= 26.667 10
6
mm
4
(5)
(100 mm + 100 mm)(40 mm)
3

bh
3

12
12
9
(40 mm)(100 mm + 100 mm)
3

bh
3

12
12
Area A was calculated previously.
d

2066
10.2 Method of Composite Areas Example 8, page 6 of 6
I

xc = Rectangle I

xc + 2 Channel I

xc
by Eq. 4 by Eq. 2
= 41.874 10
6
mm
4
+ 2(54.193 10
6
mm
4
)
= 150.3 10
6
mm
4
Ans.
I

yc
= Rectangle I

yc
+ 2 Channel I

yc
by Eq. 5 by Eq. 3
= 26.667 10
6
mm
4
+ 2(2.025 10
6
mm
4
)
= 30.7 10
6
mm
4
Ans.
10 For the composite region,

2067

10.3 Products of Inertia
2068
10.3 Products of Inertia Procedures and Strategies, page 1 of 1
Procedures and Strategies for Solving Problems
Involving Products of Inertia
y
x
y
x
y
x
1. If the given area can be expressed as a sum of
simple shapes for which tabulated products of
inertia are available, then you can calculate the
product of inertia by applying the parallel-axis
theorem to each shape:
I
xy
= I
c-xy
+ xcycA
In applying this equation, remember that
coordinates of the centroid, xc and yc, may be
negative. Also remember that you can subtract
as well as add areas.
2. If the area cannot be expressed as a sum of simple
shapes, then you must evaluate the integral
defining the product of inertia:
I
xy
= xy dA
Note: You can often save work by using the fact
that the product of inertia about an axis of symmetry
is zero.
=
y
x
Axis of symmetry
I
xy
= 0

2069
10.3 Products of Inertia Problem Statement for Example 1
900 mm
900 mm
x
y
1. Determine the product of inertia of the narrow
strip with respect to the x and y axes.
3 mm
30

2070
10.3 Products of Inertia Problem Statement for Example 2
100 mm
x
2
+ y
2
= 100
2
x
y
2. Determine the product of inertia of the crosshatched
area with respect to the x and y axes.

2071
10.3 Products of Inertia Problem Statement for Example 3
y
x
x = 2y
6
50y
5
y
3
+ 100
3. Determine the product of inertia of the crosshatched
area with respect to the x and y axes.
Scales on the x
and y axes are
not the same.
100 m
1.156 m

2072
10.3 Products of Inertia Problem Statement for Example 4
x
y
x
y = a sin( )
2b
a
b
4. Determine the product of inertia of the crosshatched area
with respect to the x and y axes, if a = 3 m and b = 1 m.

2073
10.3 Products of Inertia Problem Statement for Example 5
1 m
x
y
x = 4 y
2
x = 3y
3 m 1 m
5. Determine the product of intertia of the crosshatched
area with respect to the x and y axes.

2074
10.3 Products of Inertia Problem Statement for Example 6
2
y = 10e
-x
x
y
1 m
1.5 m
3.68 m
6. Determine the product of inertia of the crosshatched
area with respect to the x and y axes.

2075
10.3 Products of Inertia Problem Statement for Example 7
15 m
x
y
80 mm
15 mm
7.5 mm
15 mm
7. Determine the product of inertia of the beam's
cross-sectional area with respect to axes passing through the
centroid C.
C
52.5 mm
105 mm
80 mm

2076
10.3 Products of Inertia Problem Statement for Example 8
8. Determine the product of inertia of the beam's
cross-sectional area with respect to x and y axes passing
through the centroid C.
6 in.
1 in.
8 in.
1 in.
5 in.
5 in.
x
y
6 in.
1 in.
11 in.
11 in.
C

2077
10.3 Products of Inertia Problem Statement for Example 9
x
y
1.99 in.
0.5 in.
4 in.
C
9. The cross section of a standard rolled-steel angle is shown.
Determine the product of inertia of the section with respect to x
and y axes passing through the centroid C. Make the
simplifying assumption that all corners are square.
0.5 in.
0.987 in.
6 in.

2078
10.3 Products of Inertia Problem Statement for Example 10
150 mm
15 mm
30 mm
x
y
10. Determine the product of inertia of the crosshatched
area with respect to the x and y axes.

2079
10.3 Products of Inertia Problem Statement for Example 11
330 mm
50 mm
210 mm
280 mm
40 mm
210 mm
x
y
11. Determine the product of inertia of the crosshatched
area with respect to the x and y axes.

2080
10.3 Products of Inertia Example 1, page 1 of 2
900 mm
900 mm
x
y
x
y
1. Determine the product of inertia of the narrow
strip with respect to the x and y axes.
The product of inertia is defined as
I
xy
= xy dA (1)
Here dA can be taken as an infinitesimal rectangle
with sides ds and 3 mm.
1
3 mm
30
3 mm
ds

2081
10.3 Products of Inertia Example 1, page 2 of 2
x
y
(900 mm) cos 30
30
x
y
30
900 mm
dx
ds
dy
30
2 From geometry
ds = dx/cos 30
dA = (3 mm) ds
= (3/cos 30) dx
y = x tan 30 (equation of the line)
Thus Eq 1 becomes
I
xy
= xy dA (Eq. 1 repeated)
= x( x tan 30)[(3/cos 30) dx]
= 6.31 10
8
mm
4
Ans.
900 cos 30
900 cos 30
(900 mm) cos 30
ds

2082
10.3 Products of Inertia Example 2, page 1 of 2
100 mm
x
2
+ y
2
= 100
2
x
y
2. Determine the product of inertia of the crosshatched
area with respect to the x and y axes.
The product of inertia ,
I
xy
= xy dA
can be evaluated using double integration. Instead, we will use an
alternative approach based on the equation
I
xy
= dI
xy
(1)
where dI
xy
is the product of inertia of an infinitesimal strip.
1

2083
10.3 Products of Inertia Example 2, page 2 of 2
x
2
+ y
2
= 100
2
x
y
x
el
= x
y
y
el
= y/2
dA = y dx
2 Arbitrarily choosing vertical, rather than horizontal,
strips and applying the parallel-axis theorem to the
strip gives
dI
xy
= dI
x'y'
+ x
el
y
el
dA (2)
where dI
x'y'
is the product of inertia of the
differential strip about the x y axes (centroidal axes
for the strip).
Because the x axis is an axis of symmetry for the
strip, dI
x'y'
= 0 and Eq. 2 reduces to
dI
xy
= x
el
y
el
dA
= x(y/2)(y dx) (3)
Because the variable of integration is x, we have to
use the equation of the bounding curve, x
2
+ y
2
=
100
2
, to obtain y as a function of x :
y = 100
2
x
2
Using this expression in Eq. 3 gives, after
simplifying,
dI
xy
= x(100
2
x
2
)/2 dx
x'
y'
Thus,
I
xy
= dI
xy
= x(100
2
x
2
)/2 dx
= 1.25 10
7
mm
4
Ans.
0
100
3
100 mm
Centroid of the
infinitesimal
strip

2084
10.3 Products of Inertia Example 3, page 1 of 2
y
x
x = 2y
6
50y
5
y
3
+ 100
3. Determine the product of inertia of the crosshatched
area with respect to the x and y axes.
1 The product of inertia,
Ixy = xy dA
can be evaluated using double integration. An
alternative approach, that will be used here, is to
evaluate
Ixy = dIxy (1)
where dIxy is the product of inertia of an
infinitesimal strip.
1.156 m
100 m
Scales on the x
and y axes are
not the same.

2085
10.3 Products of Inertia Example 3, page 2 of 2
y
x
dy
x
el
= x/2
x
y
el
= y
x
y
dA = x dy
Centroid of the
infinitesimal
strip
Choosing horizontal strips and applying the
parallel-axis theorem to the strip gives
dI
xy
= dI
x'y'
+ x
el
y
el
dA (2)
where dI
x'y'
is the product of inertia for the
differential strip about the x y axes.
Because the x axis is an axis of symmetry for the
strip, dI
x'y'
= 0 and Eq.2 reduces to
dI
xy
= x
el
y
el
dA
= (x/2)(y)(x dy)
= x
2
y dy/2 (3)
2
Replacing x in Eq. 3 by the expression for the
bounding curve gives
dI
xy
= x
2
y dy/2 (Eq. 3 repeated)
(2y
6
50y
5
y
3
+ 100)
3
Thus the product of inertia is
I
xy
= dI
xy
= (2y
6
50y
5
y
3
+ 100)
2
y dy/2
= 1973 m
4
Ans.
4
0
1.156
1.156 m

2086
10.3 Products of Inertia Example 4, page 1 of 3
x
y
x
y = a sin( )
2b
a
b
4. Determine the product of inertia of the crosshatched area
with respect to the x and y axes, if a = 3 m and b = 1 m.

2087
10.3 Products of Inertia Example 4, page 2 of 3
x
y
dA = x dy
x
x
el
= x/2
dy
y
el
= y
Evaluate the integral
Ixy = dIxy (1)
where dIxy is the product of inertia of a horizontal
infinitesimal strip.
1
2 Applying the parallel axis theorem to the strip gives
dI
xy
= dI
x'y'
+ x
el
y
el
dA
0, because y is an axis of symmetry of the strip
x/2 x dy

= x
el
y
el
dA
y

= x
2
y dy/2 (2)
The x
2
in this expression can be replaced by a function of y by
solving for x from the equation of the bounding curve:
y = a sin( )
x'
y'
Centroid of the
infinitesimal
strip
x
2b

2088
10.3 Products of Inertia Example 4, page 3 of 3
x
y
x = 2b/ sin
-1
(y/a)
a
3 Thus
x = (2b/ ) sin
-1
(y/a)
Eq. 2 becomes
dI
xy
= x
2
y dy/2 (Eq. 2 repeated)
= (2b/ ) sin
-1
(y/a)
2
y/2 dy
Eq. 1 then gives, after substituting a = 3 m and b = 1 m,
I
xy
= dI
xy
(Eq. 1 repeated)

= (2/ ) sin
-1
(y/3)
2
y/2 dy
= 0.669 m
4
Ans.
3
0

2089
10.3 Products of Inertia Example 5, page 1 of 3
1 m
x
y
x = 4 y
2
x = 3y
3 m 1 m
5. Determine the product of inertia of the crosshatched
area with respect to the x and y axes.

2090
10.3 Products of Inertia Example 5, page 2 of 3
y
(x
2
, y)
dy
x
(x
1
, y)
y
el
= y
(x
2
x
1
)/2 (x
2
x
1
)/2
x
el
= x
1
+ (x
2
x
1
)/2

= (x
2
+ x
1
)/2

x
y
(x
el
, y
el
)
Centroid
of the
strip
1 Evaluate the integral
Ixy = dIxy
where dIxy is the product of inertia of a
horizontal infinitesimal strip.
Applying the parallel-axis theorem to the strip
gives
dI
xy
= dI
xy
+ x
el
y
el
dA

(x
1
+ x
2
)/2 (x
2
x
1
) dy

= x
el
y
el
dA
y
= y(x
2
2

x
1
2
) dy (2)
2
'
' '
0, because y' is an axis of
symmetry of the strip
dA = (x
2
x
1
) dy
x
1

2091
10.3 Products of Inertia Example 5, page 3 of 3
y
x
x
1
= 3y
x
2
= 4 y
2
1 m
Replacing x
1
and x
2
in Eq. 2 by functions of y gives
dI
xy
= y(x
2
2
x
1
2
) dy (Eq. 2 repeated)
= y[(4 y
2
)
2
(3y)
2
]dy
Thus Eq. 1 becomes
I
xy
= dI
xy
(3)
= y[(4 y
2
)
2
(3y)
2
] dy
= 3.917 m
4
Ans.
3
0
1

2092
10.3 Products of Inertia Example 6, page 1of 2
2
y = 10e
-x
x
y
1 m
1.5 m
3.68 m
6. Determine the product of inertia of the crosshatched
area with respect to the x and y axes.
2
y = 10e
-x
x
y
3.68 m
dx
y
y
el
= y/2
x = x
el
dA = y dx
1 Evaluate the integral
Ixy = dIxy (1)
where dIxy is the product of inertia of a
vertical infinitesimal strip.
Applying the parallel-axis theorem to the strip gives
dI
xy
= dI
xy
+ x
el
y
el
dA
= x
el
y
el
dA

= xy
2
dx/2 (2)
2
y/2
' '
0, because y' is an axis of
symmetry of the strip
x
y dx
x'
y'
Centroid
of the
strip

2093
10.3 Products of Inertia Example 6, page 2 of 2
2
y = 10e
-x
x
1 m
1.5 m
3.68 m
Replacing y in Eq. 2 by y = 10e
-x
gives
dI
xy
= xy
2
dx/2 (Eq. 2 repeated)
Thus the equation for I
xy
becomes,
I
xy
= dI
xy
(Eq. 1 repeated)
= x(10e
-x
)
2
dx/2 (Evaluate numerically with a calculator)
= 1.553 m
4
Ans.
3
1
1.5
10e
-x
2
2
2
y

2094
10.3 Products of Inertia Example 7, page 1 of 4
15 m
x
y
80 mm
15 mm
7.5 mm
15 mm
7. Determine the product of inertia of the beam's
cross-sectional area with respect to axes passing through the
centroid C.
C
52.5 mm
105 mm
80 mm

2095
10.3 Products of Inertia Example 7, page 2 of 4
x, x

y, y
15
105
15
C
80
x

y y
x

52.5 15/2 = 45
80/2 + 7.5 = 47.5
=
+
+
C
x
y
x

y
80
15
105/2 - 15/2 = 45
80/2 + 7.5 = 47.5
Rectangle 1
Rectangle 2
Rectangle 3
Express the area as the sum of three rectangles.
All lengths are in millimeters.
1
Centroids of
rectangles
15
105
15
7.5
C
x
80
80
52.5
y
15
C

2096
10.3 Products of Inertia Example 7, page 3 of 4
15 mm
45 mm
80 mm
2 Apply the parallel-axis theorem to rectangle 1:
I
xy1
= I
x'y'
+ dxd
y
A
= 0 + ( 45 mm)( 47.5 mm)[(80 mm)(15 mm)]
= 2.5650 10
6
mm
4
(1)
0, because the x' and y' axes are axes
of symmetry of the rectangle
3 Similarly for rectangle 2:
I
xy2
= I
x'y'
+ dxd
y
A
= 0 + (0)(0)A
= 0 (2)
Rectangle 3:
I
xy3
= I
x'y'
+ dxd
y
A
= 0 + (45 mm)(47.5 mm)[(80 mm)(15 mm)]
= 2.5650 10
6
mm
4
(3)
4
47.5 mm
47.5 mm
C
x, x

y, y
15 mm
105 mm
15 mm
C
80 mm
x

y y
x

45 mm
C
x
y
x

y

2097
10.3 Products of Inertia Example 7, page 4 of 4
5 Add the I
xy
terms from Eqs. 1, 2, and 3:
I
xy
= I
xy1
+ I
xy2
+ I
xy3
= 2.565 10
6
mm
4
+ 0 + 2.5650 10
6
mm
4
= 5.13 10
6
mm
4
Ans.

2098
10.3 Products of Inertia Example 8, page 1 of 4
8. Determine the product of inertia of the beam's
cross-sectional area with respect to axes passing through
the centroid C.
6 in.
1 in.
8 in.
1 in.
5 in.
5 in.
x
y
6 in.
1 in.
11 in.
11 in.
C

2099
10.3 Products of Inertia Example 8, page 2 of 4
y
5 in.
5 in.
=
1 in.
x
11 in.
C
y
x

+
+
5 1/2 = 4.5
5 1/2 = 4.5
6 11/2 = 0.5
y
Rectangle 1
Rectangle 2
Rectangle 3
6 11/2 = 0.5
1 Express the area as the sum of three rectangles.
All lengths are in inches.
1 in.
8 in.
1 in.
x
6 in.
1 in.
11 in.
11 in.
C
6 in.
1
x
11
C
1
C
8
x, x

y, y
y y
x


2100
10.3 Products of Inertia Example 8, page 3 of 4
1 in.
x
11 in.
C
C
8 in.
x, x

y, y
y y
x

4.5 in.
0.5 in.
2 Apply the parallel-axis theorem to rectangle 1:
I
xy1
= I
x'y'
+ dxd
y
A
= 0 + ( 0.5 in.)( 4.5 in)[(1 in.)(11 in.)]
= 24.75 in.
4
(1)
0, because the x' and y' axes are axes
of symmetry of the rectangle
3 Similarly for rectangle 2:
I
xy2
= I
x'y'
+ dxd
y
A
= 0 + (0)(0)A
= 0 (2)
1 in.
Rectangle 1
Rectangle 2

2101
10.3 Products of Inertia Example 8, page 4 of 4
1 in.
x
11 in.
C
y
x

4.5 in.
0.5 in.
y
Rectangle 3:
I
xy3
= I
x'y'
+ dxd
y
A
= 0 + (0.5 in.)(4.5 in.)[(11 in.)(1 in.)]
= 24.75 in.
4
(3)
4
5 Add the I
xy
terms from Eqs. 1, 2, and 3:
I
xy
= I
xy1
+ I
xy2
+ I
xy3
= 24.75 in.
4
+ 0 + 24.75 in.
4

= 49.50 in.
4
Ans.
Rectangle 3

2102
10.3 Products of Inertia Example 9, page 1 of 3
x
y
1.99 in.
0.5 in.
4 in.
C
9. The cross section of a standard rolled-steel angle is shown.
Determine the product of inertia of the section with respect to x
and y axes passing through the centroid C. Make the
simplifying assumption that all corners are square.
0.5 in.
0.987 in.
6 in.

2103
10.3 Products of Inertia Example 9, page 2 of 3
3.5/2 + 0.5 0.987
= 1.263
=
+
Centroids
of rectangles
1 Express the area as the sum of two rectangles.
All lengths are in inches.
Rectangle 1
Rectangle 2
x
y
1.99 in.
0.5 in.
4 in.
C
0.5 in.
0.987 in.
6 in.
x
y
6 in.
C
x
y
C
y
x

0.987 0.5/2 = 0.737
6/2 1.99 = 1.01
0.5
y
x

1.99 0.5/2 = 1.74
3.5

2104
10.3 Products of Inertia Example 9, page 3 of 3
x
y
6 in.
C
x
y
C
y
x

0.737 in.
1.01 in.
0.5 in.
y
1.74 in.
3.5 in.

1.263 in.
2 Apply the parallel-axis theorem to rectangle 1 .
Rectangle 1:
I
xy1
= I
x'y'
+ dxd
y
A
= 0 + ( 0.737 in.)(1.01 in.)[(0.5 in.)(6 in.)]
= 2.233 in.
4
(1)
0, because the x' and y' axes are axes
of symmetry of the rectangle
Rectangle 2 :
I
xy2
= I
x'y'
+ dxd
y
A
= 0 + (1.263 in.)( 1.74 in.)[(3.5 in.)(0.5 in.)] (2)
= 3.846 in.
4
3
4 Add the I
xy
terms from Eqs. 1 and 2:
I
xy
= I
xy1
+ I
xy2

= 2.233 in.
4
+ ( 3.846 in.
4
)
= 6.08 in.
4
Ans.
Rectangle 1
Rectangle 2 x


2105
10.3 Products of Inertia Example 10, page 1 of 4
150 mm
15 mm
30 mm
x
y
10. Determine the product of inertia of the crosshatched
area with respect to the x and y axes.

2106
10.3 Products of Inertia Example 10, page 2 of 4
2
Express the area as a rectangle minus a circle
plus a semicircle.
1
x
y
x
y
30 mm
Centroid
150 mm
150 mm
30 mm
x
y
Radius
= 15 mm
x
y
4(30 mm)
3
= 12.7324 mm
=
+
A table of properties of planar regions
gives the information shown below.
y
x
r
C 4r
3
r
2
2
A =
Semicircular area
Centroid location
30 mm
150 mm
15 mm
30 mm
x
y
150 mm
x
y
y
x
30 mm
60 mm
75 mm

2107
10.3 Products of Inertia Example 10, page 3 of 4
150 mm
y y
x

y
30 mm
150 mm
Radius = 15 mm
3
Apply the parallel-axis theorem to the rectangle.
I
xy-rectangle
= I
x'y'
+ dxd
y
A

= 0 + (75 mm)(30 mm) (150 mm)(60 mm)
= 2.0250 10
7
mm
4
(1)
0, because the x' and
y' axes are axes of
symmetry
4 Similarly for the circle,
I
xy-circle
= I
x'y'
+ dxd
y
A
= 0 + (150 mm)(30 mm) 15 mm)
2
0.3181 10
7
mm
4
(2)
x
y
30 mm
60 mm
75 mm
x

2108
10.3 Products of Inertia Example 10, page 4 of 4
Similarly for the semicircle,
I
xy-semicircle
= Ix'y' + dxdy A

= 0 + (150 mm + 12.7324 mm)(30 mm)
30 mm)
2
/2
= 0.6902 10
7
mm
4
(3)
5
Add the I
xy
values for the rectangle and semicircle and subtract the circle.
I
xy
= I
xy-rectangle
+ I
xy-semicircle
I
xy-circle
= 2.0250 10
7
mm
4
+ 0.6902 10
7
mm
4
0.3181 10
7
mm
4
= 2.40 10
7
mm
4
Ans.
6
150 mm
30 mm
x
y
x
y
12.7324 mm
30 mm

2109
10.3 Products of Inertia Example 11, page 1 of 6
330 mm
50 mm
210 mm
280 mm
40 mm
210 mm
x
y
11. Determine the product of inertia of the crosshatched
region with respect to the x and y axes.

2110
10.3 Products of Inertia Example 11, page 2 of 6
330 mm
50 mm
210 mm
280 mm
40 mm
210 mm
x
y
330 mm
280 mm
x
y
210 mm
x
y
210 mm
y
x
y
x

=
280 mm/2 = 140 mm
40 mm + 210 mm/3
= 110 mm
330 mm/2 = 165 mm
50 mm + 210 mm/3
= 120 mm
Express the area as a rectangle minus a triangle. 1

2111
10.3 Products of Inertia Example 11, page 3 of 6
330 mm
280 mm
x
y
y
x

140 mm
165 mm
2 Apply the parallel-axis theorem to the rectangle.
I
xy-rectangle
= I
x'y'
+ dxd
y
A
= 0 + (165 mm)(140 mm) (280 mm)(330 mm)
= 21.3444 10
8
mm
4
(1)
0, because the x' and y' axes are
axes of symmetry

2112
10.3 Products of Inertia Example 11, page 4 of 6
3
4 Compute I
x'y'
by integration. The integration will be
simpler if we first find I
x''y''
and then use the
parallel-axis theorem. That is, by the parallel axis
theorem,
I
x''y''
= I
x'y'
+ dx'd
y'
A
Thus
I
x'y'
= I
x''y''
dx'd
y'
A
= I
x''y''
(70mm)(70mm)(22050 mm
2
)
= I
x''y''
1.0804 10
8
mm
4
(3)
A = (1/2)(210 mm)(210 mm)
= 22050 mm
2
210 mm
x
y
210 mm
y
x

110 mm
120 mm
210 mm
y
x

y
x
210 mm
210 mm/3 = 70 mm
210 mm/3 = 70 mm
Apply the parallel-axis theorem to the triangle. Note that I
x'y'
0.
I
xy-triangle
= I
x'y'
+ d
x
d
y
A
= I
x'y'
+ (120 mm)(110 mm)(22050 mm
2
)
= I
x'y'
+ 2.9106 10
8
mm
4
(2)

2113
10.3 Products of Inertia Example 11, page 5 of 6
210 mm
y
x

210 mm
dx
y
x
el
= x
Centroid of
infinitesimal strip
yc
xc
y
el
= y /2
Integrate by using vertical strips. Applying the parallel-axis
theorem to the vertical strip gives
dI
x''y''
= dI
xcyc
+ x
el
'' y
el
" dA
= 0 + x''(y''/2)(y'' dx'')
= x''( x'' + 210)
2
dx''/2
Thus
I
x''y''
= dI
x''y''
= x'' ( x'' + 210)
2
dx''/2
= 0.8103 10
8
mm
4
(4)
0
210
Equation of line:
y'' = x'' + 210
5

2114
10.3 Products of Inertia Example 11, page 6 of 6
Substitute in Eq. 3:
I
x'y'
= I
x''y''
1.0804 10
8
mm
4
(Eq. 3 repeated)
= 0.2701 10
8
mm
4
(5)
Substitute this result in Eq. 2:
I
xy-triangle
= I
x'y'
+ 2.9106 10
8
mm
4
(Eq. 2 repeated)
= 2.6405 10
8
mm
4
(6)
The product of inertia for the entire area is the difference between
I
xy-rectangle
and I
xy-triangle
:
I
xy
= I
xy-rectangle
I
xy-triangle
= 1.870 10
9
mm
4
Ans.
0.8103 10
8
, by Eq. 4
0.2701 10
8
mm
4
, by Eq. 5
21.3444 10
8
mm
4
, by Eq.1 2.6405 10
8
, by Eq. 6
6

2115

10.4 Moments of Inertia About Inclined Axes; Principal Moments
2116
10.4 Moments of Inertia About Inclined Axes; Principal Moments Procedures and Strategies, page 1 of 2
Procedures and Strategies for Solving Problems Involving
Moments of Inertia About Inclined Axes; Principal Moments
Begin by calculating I
x,
I
y
, and I
xy
. Then to determine moments of inertia about
axes rotated with respect to the x and y axes, use the formulas
Iu = + cos 2 I
xy
sin 2
Iv = cos 2 I
xy
sin 2
Iuv = sin 2 I
xy
cos 2
Determine the principal (maximum and minimum) moments of inertia either by
using formulas or by using Mohr's circle.
1. Formulas:
I
max, min
= + I
2
xy
The axes corresponding to Imax and I
min
are given by the roots and
2
of
the equation
tan 2 =
To determine which root,
1
or , goes with Imax and which goes with I
min
,
substitute
1
into Eq. 1 and see whether the formula gives Imax or I
min
.
Ix + I
y
2
Ix + I
y
2
Ix I
y
2
( )
2
Ix I
y
2
Ix I
y
2I
xy
x
y
Determine Ix, I
y
, and I
xy
first.
u
v
Ix + I
y
2
Ix I
y
2
Ix I
y
2

2117
10.4 Moments of Inertia About Inclined Axes; Principal Moments Procedures and Strategies, page 2 of 2
I
X
2. Mohr's circle:
a) Construct a coordinate system in which
the horizontal axis is labeled I (for values of
the moment of inertia, Ix and I
y
), and the
vertical axis is labeled I
xy
(for values of the
product of inertia).
b) Plot the point (Ix, I
xy
) and label it "X."
c) Plot the center of the circle, (Iave, 0),
where Iave = (Ix + I
y
)/2 = average moment of
inertia.
d) Draw a straight line connecting X and the
center, use geometrical relations to calculate
the value, R, of the radius, and then draw the
circle.
e) Imax and I
min
are the far left and far right
points on the circle.
f) The principal axes (axes corresponding to
I
max
and I
min
) can be found by rotating the
x and y axes half the amount of the rotation
in Mohr's circle.
X
I
xy
Ix Iave = (Ix I
y
)/2
X
R R
Imax
I
min
Imax = Iave + R
I
min
= Iave R
Iave = (Ix + I
y
)/2
Iave
X
Imax
x
y
y'
x'
p
p
/2
Radius R = + I
2
xy
Ix I
y
2
( )
2
R
I
xy
I
xy
I
xy
I
xy
I
I
I

2118
10.4 Moments of Inertia About Inclined Axes; Principal Moments Problem Statement for Example 1
1. Determine the moments of inertia of the standard rolled-steel
angle section with respect to the u and v axes.
Ix = 17.40 in
4
I
y
= 6.27 in
4
I
xy
= 6.08 in.
4

x
y
1.99 in.
0.5 in.
4 in.
C
0.5 in.
0.987 in.
6 in.
45
v
u

2119
10.4 Moments of Inertia About Inclined Axes; Principal Moments Problem Statement for Example 2
x
y
u
v
40 mm
220 mm
100 mm
20 mm
100 mm
2. Determine the moments of inertia of the crosshatched area with
respect to the u and v axes for a) = 25 and b) = 90

2120
10.4 Moments of Inertia About Inclined Axes; Principal Moments Problem Statement for Example 3
x
y
u
v
x
2
+ y
2
= 100
2
3. Determine the value of for which the product of inertia of the
crosshatched area with respect to the u and v axes is zero. Calculate
Iu and Iv for this value of and compare Iu and Iv to Imax and I
min
Ix = 1.9635 x 10
7
mm
4
I
y
= 1.9635 x 10
7
mm
4
I
xy
= 1.2500 x 10
7
mm
4

2121
10.4 Moments of Inertia About Inclined Axes; Principal Moments Problem Statement for Example 4
y
x
60 mm 60 mm
20 mm
50 mm
50 mm
20 mm
C
10 mm
4. Determine the principal moments of inertia with respect to all
possible rectanglular coordinate systems with their origin at the
centroid C.
Ix = 2.2013 x 10
7
mm
4
I
y
= 0.9213 x 10
7
mm
4

2122
10.4 Moments of Inertia About Inclined Axes; Principal Moments Problem Statement for Example 5
150 mm
15 mm
30 mm
x
y
5. Determine the principal moments of inertia and
principal axes having their origin at point O.
Ix = 1.1714 10
7
mm
4

I
y
= 8.9083 10
7
mm
4

I
xy
= 2.3971 10
7
mm
4
O

2123
10.4 Moments of Inertia About Inclined Axes; Principal Moments Problem Statement for Example 6
x
y
O
4 in.
6 in.
6. Determine the principal moments of inertia
and principal axes having their origin at point O.

2124
10.4 Moments of Inertia About Inclined Axes; Principal Moments Problem Statement for Example 7
127 mm
127 mm
C
x
y
Ix = 32.6 10
6
mm
4
I
y
= 1.14 10
6
mm
4
7. Use Mohr's circle to determine the principal moments of
inertia and principal axes having their origin at the centroid C
of the standard rolled-steel channel section.
15.3 mm

2125
10.4 Moments of Inertia About Inclined Axes; Principal Moments Problem Statement for Example 8
8. Use Mohr's circle to determine the principal moments of
inertia and principal axes having their origin at the centroid C
of the standard rolled-steel angle section.
Ix = 17.40 in
4
I
y
= 6.27 in
4
I
xy
= 6.08 in.
4

x
y
1.99 in.
0.5 in.
4 in.
C
0.5 in.
0.987 in.
6 in.

2126
10.4 Moments of Inertia About Inclined Axes; Principal Moments Problem Statement for Example 9
9. Use Mohr's circle to determine the principal moments of inertia
and principal axes having their origin at the centroid C.
Ix = 6.7245 10
6
mm
4
I
y
= 6.3520 10
6
mm
4
I
xy
= 5.1300 10
6
mm
4
x
y
80 mm
15 mm
7.5 mm
15 mm
C
52.5 mm
105 mm
80 mm
15 m

2127
10.4 Moments of Inertia About Inclined Axes; Principal Moments Problem Statement for Example 10
2 in. 2 in.
2 in.
2 in.
x
y
O
10. Use Mohr's circle to determine the principal moments of
inertia and principal axes having their origin at point O
2 in.
2 in.

2128
10.4 Moments of Inertia About Inclined Axes; Principal Moments Example 1, page 1 of 3
1. Determine the moments of inertia of the standard rolled-steel
angle section with respect to the u and v axes.
Ix = 17.40 in
4
I
y
= 6.27 in
4
I
xy
= 6.08 in.
4

x
y
1.99 in.
0.5 in.
4 in.
C
0.5 in.
0.987 in.
6 in.
45
v
u

2129
10.4 Moments of Inertia About Inclined Axes; Principal Moments Example 1, page 2 of 3
1 The formula for Iu is
Iu =
Ix + I
y Ix I
y
cos 2
I
xy
sin 2
2 2
+
(1)
We can save some work later, if we calculate and save the expressions
Ix + I
y
2
=
17.40 in
4
+ 6.27 in
4
2
= 11.835 in
4
(2)
Ix I
y
2
=
17.40 in
4
6.27 in
4
2
= 5.565 in
4
(3)
Eq. 1 becomes,
and
Ix + I
y Ix I
y
cos 2
I
xy
sin 2
2
2
+
11.835 in
4
, by Eq. 2
5.565 in
4
, by Eq. 3
45 ( is negative because the x axis
must be rotated clockwise to make it
coincide with the u axis)
= 5.76 in
4
Ans.
Iu =
6.08 in.
4

2130
10.4 Moments of Inertia About Inclined Axes; Principal Moments Example 1, page 3 of 3
2 Similarly, for Iv and Iuv,
Ix + I
y Ix I
y
cos 2
I
xy
sin 2
2 2
11.835 in
4
, by Eq. 2
5.565 in
4
, by Eq. 3
45
= 17.92 in
4
Ans.
Iv =
Ix I
y
sin 2
I
xy
cos 2
2
5.565 in
4
, by Eq. 3
= 5.57 in
4
Ans.
Iuv =
6.08 in.
4
45
6.08 in.
4

2131
10.4 Moments of Inertia About Inclined Axes; Principal Moments Example 2, page 1 of 5
x
y
u
v
40 mm
220 mm
100 mm
20 mm
100 mm
2. Determine the moments of inertia of the crosshatched area with
respect to the u and v axes for a) = 25 and b) = 90
1 Before we can use the equations for Iu, Iv, and Iuv, we must
determine Ix, I
y
, and I
xy
. Determining I
xy
is easy: the y axis is
an axis of symmetry, so
I
xy
= 0 (1)

2132
10.4 Moments of Inertia About Inclined Axes; Principal Moments Example 2, page 2 of 5
220mm/2 = 110 mm
x
y
y
Centroid
120 mm 120 mm
20 mm
220 mm =
+
Rectangle 1 Rectangle 2
To find Ix and I
y
, consider the crosshatched area to be
the sum of two rectangles and calculate Ix and I
y
for
each rectangle
2
I
x-rectangle-1
= Ix' + d
2
A
I = bh
3
/12 for
rectangle about
centroidal axis
=
(40 mm)(220 mm)
3

12
+ (110 mm)
2
[(40 mm)(220 mm)
3
I
y-rectangle-1

=
(220 mm)(40 mm)
3

12
(3)
= 1.4197 10
8
mm
4
(2)
= 0.0117 10
8
mm
4
x
y
40 mm
220 mm
100 mm
20 mm
100 mm
x
y
x
y
Centroid
20 mm
220 mm

2133
10.4 Moments of Inertia About Inclined Axes; Principal Moments Example 2, page 3 of 5
I
x-rectangle-2
= Ix' + d
2
A
4
=
(240 mm)(40 mm)
3

12
+ (220 mm + 20 mm)
2
(240 mm)(40 mm)
= 5.5424 10
8
mm
4
(4)
I
y-rectangle-2
=
(40 mm)(240 mm)
3

12
=
5 Adding the results for rectangles 1 and 2 gives
Ix = I
x-rectangle-1
+ I
x-rectangle-2
(6)
I
y
= I
y-rectangle-1
+ I
y-rectangle-2

(7)
6 Substitute for Ix, Iy, and Ixy in the equation for Iu, Iv, and Iuv.
We can save work if we calculate and save the expressions
Ix + I
y
2
=
6.9621 10
8
mm
4
+ 0.4725 10
8
mm
4
= 3.7173 10
8
mm
4
and
Ix I
y
2
=
= 3.2448 10
8
mm
4
2
2
(8)
(9)
1.4197 10
8
mm
4
, by Eq. 2
5.5424 10
8
mm
4
, by Eq. 4
= 6.9621 10
8
mm
4
0.0117 10
8
mm
4
, by Eq. 3
= 0.4608 10
8
mm
4
(5)
0.4608 10
8
mm
4
, by Eq. 5
= 0.4725 10
8
mm
4
6.9621 10
8
mm
4
0.4725 10
8
mm
4
bh
3

12

2134
10.4 Moments of Inertia About Inclined Axes; Principal Moments Example 2, page 4 of 5
7 Part a): Calculate Iu, Iv, and Iuv for = 25
Ix + I
y Ix I
y
cos 2
I
xy
sin 2
2
2
+
=
Iu
= 5.80 10
8
mm
4
Ans.
3.7173 10
8
mm
4
, by Eq. 8 3.2448 10
8
mm
4
, by Eq. 9
25 0, by Eq. 1
Ix + I
y Ix I
y
cos 2
I
xy
sin 2
2
2
=
Iv
= 1.632 10
8
mm
4
Ans.
25
0, by Eq. 1
Ix I
y
sin 2
I
xy
cos 2
2
=
Iuv
25
0, by Eq. 1
3.7173 10
8
mm
4
, by Eq. 8 3.2448 10
8
mm
4
, by Eq. 9
3.2448 10
8
mm
4
, by Eq. 9
= 2.49 10
8
mm
4
Ans.

2135
10.4 Moments of Inertia About Inclined Axes; Principal Moments Example 2, page 5 of 5
90
x
y, u
v
Part b) :
When the u and y axes coincide, so no transformation
equations are needed. Instead, we have
Iu = I
y

= 0.47 10
8
mm
4
Ans.
Similarly,
Iv = Ix
= 6.96 10
8
mm
4
Ans.
and
Iuv = I
xy
= 0, Ans.

6.9621 10
8
mm
4
, by Eq. 6
8
0.4725 10
8
mm
4
, by Eq. 7
by Eq. 1

2136
10.4 Moments of Inertia About Inclined Axes; Principal Moments Example 3, page 1 of 4
x
y
u
v
x
2
+ y
2
= 100
2
3. Determine the value of for which the product of inertia of the
crosshatched area with respect to the u and v axes is zero. Calculate
Iu and Iv for this value of and compare Iu and Iv to Imax and I
min
Ix = 1.9635 x 10
7
mm
4
I
y
= 1.9635 x 10
7
mm
4
I
xy
= 1.2500 x 10
7
mm
4

2137
10.4 Moments of Inertia About Inclined Axes; Principal Moments Example 3, page 2 of 4
The formula for Iuv is
Iuv =
We can save some work later, if we calculate and save
Setting Iuv equal to zero in Eq. 1 gives,
Solving this equation for leads to two roots:
= 45, 45

Ix
I
y
sin 2 I
xy
cos 2
2
Ix
I
y
2
=
1.9635 x 10
7
1.9635 x 10
7
2
= 0 (2)
(1)
Ix+ I
y
sin 2
I
xy
cos 2
2
= 0
0, by Eq. 2
1.2500 x 10
7
mm
4
1
Ans.

2138
10.4 Moments of Inertia About Inclined Axes; Principal Moments Example 3, page 3 of 4
The formula for Iu is
We can save some work by calculating and saving
Substituting numerical values in Eq. 3 gives
Similarly for Iv,
Ix + I
y
Ix I
y
cos 2 I
xy
sin 2
2
2
+ Iu =
(3)
Ix + I
y
2
=
1.9635 10
7
1.9635 10
7
2
=
1.9635 10
7
(4)
Ix + I
y Ix I
y
cos 2
I
xy
sin 2
2
2
+ Iu =
1.9635 10
7
mm
4
0, by Eq. 2
1.2500 10
7
mm
4
45
Ix + I
y Ix I
y
cos 2
I
xy
sin 2
2
2
Iv =
1.9635 10
7
mm
4
0, by Eq. 2
1.2500 10
7
mm
4
45
= 0.7135 10
7
mm
4
Ans.
= 3.2135 10
7
mm
4
Ans.
2

2139
10.4 Moments of Inertia About Inclined Axes; Principal Moments Example 3, page 4 of 4
x
y
u v
x
2
+ y
2
= 100
2
3 Calculate Imax and I
min
I
max,min
Ix + I
y Ix I
y
I
xy
2
2
2
1.9635 10
7
mm
4
0, by Eq. 2
1.2500 10
7
mm
4
=
)
2
= 3.2135 10
7
mm
4
, 0.7135 10
7
mm
4
same as Iv Same as Iu
Thus Iv = Imax and Iu = I
min
. In general if the product of inertia, Iuv, is zero for a
given orientation, then the moments of inertia, Iu and Iv, are the maximum and
minimum moments of inertia possible for any orientation.
4 Show the u and v axes' orientation.
(Axis of minimum moment of inertia)
(Axis of maximum
moment of inertia)
45

2140
10.4 Moments of Inertia About Inclined Axes; Principal Moments Example 4, page 1 of 2
y
x
60 mm 60 mm
20 mm
50 mm
50 mm
20 mm
C
10 mm
4. Determine the principal moments of inertia with respect to all
possible rectangular coordinate systems with their origin at the
centroid C.
Ix = 2.2013 x 10
7
mm
4
I
y
= 0.9213 x 10
7
mm
4
Because the x and y axes are axes of symmetry for the crosshatched area
I
xy
= 0 (1)
1

2141
10.4 Moments of Inertia About Inclined Axes; Principal Moments Example 4, page 2 of 2
2 Apply the equation for the principal moments of inertia.
I
max,min
=
Ix + I
y
(Ix I
y
)/2
2
+ I
xy
2
2
2.2013 10
7
mm
4
0, by Eq. 1
0.9213 10
7
mm
4
= 2.2013 10
7
mm
4
, 0.9213 10
7
mm
4
Ans.
That is, Ix and I
y
are principal moments of inertia. In general, you can
immediately recognize if Ix and I
y
are principal moments of inertia by noting if
the product of inertia is zero: I
xy
= 0 implies that Ix and I
y
are principal moments
of inertia.

2142
10.4 Moments of Inertia About Inclined Axes; Principal Moments Example 5, page 1 of 3
150 mm
15 mm
30 mm
x
y
5. Determine the principal moments of inertia and
principal axes having their origin at point O.
Ix = 1.1714 10
7
mm
4

I
y
= 8.9083 10
7
mm
4

I
xy
= 2.3971 10
7
mm
4
O

2143
10.4 Moments of Inertia About Inclined Axes; Principal Moments Example 5, page 2 of 3
Ix + I
y Ix I
y
I
xy
2
2
2
1.1714 10
7
mm
4
2.3971 10
7
mm
4
)
2
Apply the formula for the principal moments of inertia,
I
max,min
=
8.9083 10
7
mm
4
= 9.5908 10
7
mm
4
, 4.8892 10
7
mm
4
(1) Ans.
1
Apply the formula for the principal directions 2
2I
xy
= (1/2) tan
-1
Ix I
y
2.3971 10
7
mm
4
1.1714 10
7
mm
4
8.9083 10
7
mm
4
p
= 15.8923 and 74.1077
To determine which
p
value corresponds to Imax and which to I
min
,
substitute 15.8923 into the transformation equation for Ix':
Ix + I
y Ix I
y
cos 2
I
xy
sin 2
2
2
+
Iu =
1.1714 10
7
mm
4
8.9083 10
7
mm
4
2.3971 10
7
mm
4
= 4.8892 10
7
mm
4
= I
min
, by Eq. 1

2144
10.4 Moments of Inertia About Inclined Axes; Principal Moments Example 5, page 3 of 3
x
y
15.9
Axis of maximum
moment of inertia
Axis of minimum
moment of inertia
O
Thus a 15.9 counterclockwise rotation of the
x axis would give the axis for which the
quantity
Iu = v
2
dA
is smaller than the same integral evaluated for
any other orientation of the axis.
3
74.1

2145
10.4 Moments of Inertia About Inclined Axes; Principal Moments Example 6, page 1 of 4
x
y
O
4 in.
6 in.
6. Determine the principal moments of inertia
and principal axes having their origin at point O.

2146
10.4 Moments of Inertia About Inclined Axes; Principal Moments Example 6, page 2 of 4
x
y
4 in.
O
y
x
6 in.
3 in.
2 in.
Area: A = (4 in.)(6 in.) = 24 in
2
Calculate the moment of inertia.
Ix = Ix' + d
2
A (parallel axis theorem)
1
(4 in.)(6 in.)
3
12
+ (3 in.)
2
(24 in.
2
) =
= 288 in.
4
(6 in.)(4 in.)
3
12
+ (2 in.)
2
(24 in.
2
) =
= 128 in.
4
I
y
(1)
(2)
I
xy
= I
x'y'
+ dxd
y
A
=
0 +
(2 in.)(3 in.)(24 in.
2
)
= 144 in.
4
(3)

2147
10.4 Moments of Inertia About Inclined Axes; Principal Moments Example 6, page 3 of 4
Apply the formulas for the principal moments of inertia. 2
Ix + I
y
(Ix I
y
)/2
2
I
xy
2
2
288 in.
4
, by Eq. 1 144 in.
4
, By Eq. 3
128 in.
4
, by Eq. 2
I
max, min
=
372.73 in.
4
, 43.27 in.
4
(4) Ans.

2I
xy
=
1
2
tan
-1
Ix I
y
p
= 30.47 and 59.53
Apply the formulas for the principal directions 3
288 in.
4
, by Eq. 1 128 in.
4
, by Eq. 2
144 in.
4
, By Eq. 3
To determine which
p
value corresponds to max and which to
min
,
substitute 30.47 into the transformation equation for Iu:
Ix + I
y Ix I
y
cos 2
I
xy
sin 2
2
2
+
Iu =
= 372.73 in.
4
= Imax, by Eq. 4
288 in.
4
, by Eq. 1
128 in.
4
, by Eq. 2 144 in.
4
, By Eq. 3

2148
10.4 Moments of Inertia About Inclined Axes; Principal Moments Example 6, page 4 of 4
x
y
O
Axis of minimum
moment of inertia
Axis of maximum
moment of inertia
Thus a 30.47 clockwise rotation of the x axis
would give the axis for which the quantity
Iu = v
2
dA
is larger than the same integral evaluated for any
other orientation of the axis.
4
59.53
30.47

2149
10.4 Moments of Inertia About Inclined Axes; Principal Moments Example 7, page 1 of 3
127 mm
127 mm
C
x
y
Ix = 32.6 10
6
mm
4
I
y
= 1.14 10
6
mm
4
Because the x axis is an
axis of symmetry, I
xy
= 0
1
7. Use Mohr's circle to determine the principal moments of
inertia and principal axes having their origin at the centroid C
of the standard rolled-steel channel section.
15.3 mm

2150
10.4 Moments of Inertia About Inclined Axes; Principal Moments Example 7, page 2 of 3
C
I
xy
I
Draw the I and I
xy
axes. 2
3 Plot the point corresponding to the x axis:

(Ix, I
xy
) = (32.6 10
6
, 0).
4
Plot the point C at the center of Mohr's circle:
(I
average
, 0) = ((Ix + I
y
)/2, 0)
= ((32.6 10
6
+ 1.14 x 10
6
)/2, 0)
= (16.87 10
6
, 0)
5 Calculate the radius:
R = 32.6 x 10
6
16.87 x 10
6
= 15.73 10
6
R
X

2151
10.4 Moments of Inertia About Inclined Axes; Principal Moments Example 7, page 3 of 3
C
16.87 10
6
Imax
I
min
R = 15.73 10
6
Draw Mohr's circle. 6
7 Calculate Imax.
Imax = 16.87 10
6
+ 15.73 10
6
= 32.6 10
6
mm
4
Ans.
8 Calculate I
min
.
I
min
= 16.87 10
6
15.73 10
6
= 1.14 10
6
mm
4
Principal axes: Because Ix is the same
as Imax and I
y
is the same as I
min
, the x
and y axes are the principal axes.
9
I
xy
I

2152
10.4 Moments of Inertia About Inclined Axes; Principal Moments Example 8, page 1 of 4
8. Use Mohr's circle to determine the principal moments of
inertia and principal axes having their origin at the centroid C
of the standard rolled-steel angle section.
Ix = 17.40 in
4
I
y
= 6.27 in
4
I
xy
= 6.08 in.
4

x
y
1.99 in.
0.5 in.
4 in.
C
0.5 in.
0.987 in.
6 in.

2153
10.4 Moments of Inertia About Inclined Axes; Principal Moments Example 8, page 2 of 4
17.40 11.835 = 5.565
1
2 Plot the point corresponding to the x axis:
(Ix, I
xy
) = (17.40, 6.08).
3 Plot the point C at the center of Mohr's circle:
(I
average
, 0) = ((17.40 + 6.27)/2 , 0)
= (11.835, 0)
4 Calculate the radius.
R = ( 6.08)
2
+ (5.565)
2
= 8.242
C
R
11.835
I
xy
I
Draw the I and I
xy
axes.
6.08
17.40
X

2154
10.4 Moments of Inertia About Inclined Axes; Principal Moments Example 8, page 3 of 4
C
6.08
R
R = 8.242

R R
2
Draw Mohr's circle. 5
6 Calculate Imax.
Imax = 11.835 + 8.242
= 20.1 in.
4
Ans.
7 Calculate I
min
.
I
min
= 11.835 8.242
= 3.59 in.
4
Ans.
8 Determine the orientation of the Imax axis.
= (1/2) tan
-1
( )
6.08
5.665
= 23.8 (Counterclockwise rotation of the x
axis gives the axis of Imax.)
5.565
11.835
I
xy
I
X

2155
10.4 Moments of Inertia About Inclined Axes; Principal Moments Example 8, page 4 of 4
Sketch the principal axes. 9
x
y
0.987 in.
C
Axis of minimum
moment of inertia
Axis of maximum
moment of inertia
1.99 in.
23.8

2156
10.4 Moments of Inertia About Inclined Axes; Principal Moments Example 9, page 1 of 4
9. Use Mohr's circle to determine the principal moments of inertia
and principal axes having their origin at the centroid C.
Ix = 6.7245 10
6
mm
4
I
y
= 6.3520 10
6
mm
4
I
xy
= 5.1300 10
6
mm
4
x
y
80 mm
15 mm
7.5 mm
15 mm
C
52.5 mm
105 mm
80 mm
15 m

2157
10.4 Moments of Inertia About Inclined Axes; Principal Moments Example 9, page 2 of 4
C
6.5382 10
6
6.7245 10
6
6.5382 10
6

= 0.1863 10
6
5.1300 10
6
6.7245 10
6
R
1
2 Plot the point corresponding to the x axis:
(Ix, I
xy
) = (6.7245 10
6
, 5.1300 10
6
).
3 Plot the point C at the center of Mohr's circle:
(I
average
, 0)
= ((Ix + I
y
)/2, 0)
=
(6.7245 10
6
+ 6.3520 10
6
, 0)
2
= (6.5382 10
6
, 0)
4 Calculate the radius.
R = (5.1300 10
6
)
2
+ (0.1863 10
6
)
2
= 5.1334 10
6
I
xy
I
Draw the I and I
xy
axes.
X

2158
10.4 Moments of Inertia About Inclined Axes; Principal Moments Example 9, page 3 of 4
C
6.5382 10
6
0.1863 10
6
5.1300 10
6
R
R = 5.1334 10
6

R R
2
Draw Mohr's circle. 5
6 Calculate Imax.
Imax = 6.5382 10
6
+ 5.1334 10
6
= 11.67 10
6
mm
4
Ans.
7 Calculate I
min
.
I
min
= 6.5382 10
6
5.1334 10
6
= 1.40 10
6
mm
4
Ans.
8 Determine the orientation of the Imax axis.
= (1/2) tan
-1

5.1300 10
6
0.1863 10
6
= 44.0 (Clockwise rotation of the
x axis gives the axis of Imax.)
I
xy
I
X

2159
10.4 Moments of Inertia About Inclined Axes; Principal Moments Example 9, page 4 of 4
x
y
C
44.0
Axis of minimum moment of inertia
Axis of maximum moment of inertia
Sketch the principal axes. 9

2160
10.4 Moments of Inertia About Inclined Axes; Principal Moments Example 10, page 1 of 5
10. Use Mohr's circle to determine the principal moments of
inertia and principal axes having their origin at point O
2 in. 2 in.
2 in.
2 in.
x
y
O
2 in.
2 in.

2161
10.4 Moments of Inertia About Inclined Axes; Principal Moments Example 10, page 2 of 5
x
y
x
O
Square 1 Square 2
=
6 in.
3 in.
3 in.
x
y
y
x
3 in.
2 in.
2 in. 3 in.
Consider the crosshatched area to be the difference
between a large square and a small square.
1
2
Calculate the moment of inertia of square 1 about the x
axis (the moment about the y axis will be the same, by
symmetry):
I
x-square-1
= I
x'-square-1
+ d
2
A
= + (3 in.)
2
[(6 in.)(6 in.)]
= 432 in
4
(1)
12
(6 in.)(6 in.)
3
y
I
xy-square-1
= I
x'y'
+ dxd
y
A
= 0 + (3 in.)(3 in.) (6 in.)(6 in.)
= 324 in
4
(2)
3
2 in. 2 in.
2 in.
2 in.
x
y
O
2 in.
2 in.

2162
10.4 Moments of Inertia About Inclined Axes; Principal Moments Example 10, page 3 of 5
x
y
O
Square 2
y
x
3 in.
2 in.
2 in. 3 in.
4
Calculate the moment of inertia of square 2.
I
x-square-2
= I
x'-square-2
+ d
2
A
= + (3 in.)
2
(2 in.)(2 in.)
= 37.333 in.
4
(3)
I
xy-square-2
= I
x'y'
+ dxd
y
A
= 0 + (3 in.)(3 in.)[(2 in.)(2 in.)]
= 36 in
4
(4)
Calculate the moment of inertia for the composite square.
Ix = I
x-square-1
I
x-square-2
= 394.667 in.
4
(5)
I
y
= Ix, by symmetry
= 394.667 in.
4
(6)

6
I
xy
= I
xy-square-1
I
xy-square-2
= 288 in.
4
(7)
12
(2 in.)(2 in.)
3
324 in.
4
, by Eq. 2 36 in.
4
, by Eq. 4
5
432 in.
4
, by Eq. 1
37.333 in.
4
, by Eq. 3

2163
10.4 Moments of Inertia About Inclined Axes; Principal Moments Example 10, page 4 of 5
C
I
xy
I
7 Draw the I and I
xy
axes
10 Calculate the radius
R = 288
Plot the point corresponding to the x axis:
(Ix, I
xy
) = (394.667, 288)
8
9 Plot the point C at the center of Mohr's circle:
(I
average
, 0) = ((Ix + I
y
)/2, 0)
= ((394.667 + 394.667)/2, 0)
= (394.667, 0)
R
394.667
288
X

2164
10.4 Moments of Inertia About Inclined Axes; Principal Moments Example 10, page 5 of 5
X
X
C
I
xy
I
C
I
xy
I
R = 288
I
min
Imax
394.667
90
Imax I
min
x
y
O
Axis of
minimum
moment of
inertia
Axis of maximum moment of inertia
45
11
Calculate Imax and I
min
Imax = 394.667 + 288
= 683 in.
4
Ans.
I
min
= 394.667 288
= 106.7 in.
4
Ans.
12 Clockwise rotation of the x axis 90/2 = 45 gives the
axis of Imax.

2165

11. Energy Methods
2166

11.1 Virtual Work
2167
11.1 Virtual Work Procedures and Strategies, page 1 of 2
Procedures and Strategies for Solving Problems Involving Virtual
Work
1. Identify a single coordinate, q, that will completely define the
configuration (position of all parts) of the machine being analyzed
(The method of virtual work is often the best method to use when
you are analyzing a machine or mechanism, rather than a rigid
structure).
2. Draw a free-body diagram of the machine.
3. On the free-body diagram, superimpose a dashed-line sketch of the
position of the machine when the coordinate q is increased a small
amount, q.
4. Identify the active forces, that is, the forces that do work when q is
increased by an amount, q.
5. Introduce coordinates, x
i
, measured from a fixed point to the point
of application of each active force.
6. Write the virtual work equation, W = 0. W is the total work done
by each active force when its coordinate x
i
is increased a small
amount. Assign a positive sign to the work if the force and
incremental displacement are in the same direction, a minus sign if
in opposite directions.
7. Relate each x
i
to q by using relations derived from the geometry
of the mechanism. Often these relations between differentials can be
derived by differentiating equations relating x
i
to q.
q =
k
A
B
mg
F
spring
Ax
A
y
A
B
A
B
x
1
x
1
x
2
x
2
L/2
L/2
A
B
x
2
= L cos
Ax and A
y
are not
active forces
because they do not
move and thus do
no work.
x
2
= L sin
x
1

2168
11.1 Virtual Work Procedures and Strategies, page 2 of 2
q
1
=
1
q
2
=
2
1
2
8. Substitute, in the virtual work equation, W = 0, the expression for each
x
i
in terms of q, and then factor out q from each term in the equation.
The coefficient of q must equal zero if W is to be zero for arbitrary
values of q. This condition gives an equation from which you can find
the unknown force (or unknown value of q, if that is what is requested in
the problem statement).
Notes:
a) A good way to account for the effect of the force F in a spring is to
remove the spring and replace it by a pair of forces of magnitude F.
The virtual work of each of these forces is then F x
i
, where x
i
is the
virtual displacement of the points where the ends of the spring were
attached. After the virtual work equation has been derived, then you
can replace F by ks, where s is the extension of the spring. Note that
you do not apply a virtual displacement to s.
b) If you find that it is very difficult to derive the relations between the
virtual displacements x
i
and q, then probably you should analyze
the machine by drawing free-body diagrams and writing equilibrium
equations for the separate parts of the machine.
c) The above procedure applies to one degree-of-freedom machines,
that is, to machines the position of which can be described by a single
coordinate, q. For systems with more than a single degree of freedom,
multiple coordinates q
i
must be defined. The steps described above
can then be followed.

2169
11.1 Virtual Work Problem Statement for Example 1
A
B
L
Smooth surface
P
C
L
W
1. Determine the force P required to keep the two rods in
equilibrium when the angle = 30 and weight W is 50 lb.
The rods are each of length L and of negligible weight.
They are prevented from moving out of the plane of the
figure by supports not shown.

2170
11.1 Virtual Work Problem Statement for Example 2
C
B
A
W
D
2 ft
2 ft
M
2. Determine the value of moment M required to
maintain the mechanism in the position shown,
if = 35 and W = 200 lb.

2171
11.1 Virtual Work Problem Statement for Example 3
D
A
B
C
W
E
P
F
3 m 3 m
2 m
3 m
3. Determine the value of the weight W
required to maintain the mechanism in the
position shown, if P = 50 N.

2172
11.1 Virtual Work Problem Statement for Example 4
D
E
300 mm
250 mm
400 mm
150 mm
A
B
C
G
Q
F
P
250 mm
300 mm
4. Determine the force Q necessary to maintain
equilibrium when force P = 400 N.

2173
11.1 Virtual Work Problem Statement for Example 5
D
A
B C
F
P
E
Q
L
L/2 L/2
5. Link AB is connected to collar A, which can slide
with negligible friction on horizontal rod EF. Determine
the value of force Q necessary to maintain equilibrium
when = 50, L = 300 mm, and P = 100 N.

2174
11.1 Virtual Work Problem Statement for Example 6
C
W
B
150 mm
A
D
150 mm
6. Rotating the threaded rod AC of the automobile jack causes
joints A and C to move closer together, thus raising the weight W.
Determine the axial force in the rod, if = 30 and W = 2 kN.
150 mm
150 mm

2175
11.1 Virtual Work Problem Statement for Example 7
A
B
L
C
P
k
F
E D
L
L L
L L
7. The original length of the spring is L. Determine the
angle for equilibrium if L = 3 m and P = 300 N.
Spring constant,
k = 200 N/m

2176
11.1 Virtual Work Problem Statement for Example 8
E
B
A
x
y
Q
k 9 kN/m
8. Collars A and B can slide freely on rods CD and CE. Determine the
values of x and y, given that forces P = 900 N and Q = 800 N. The
unstretched length of the spring is 0.2 m, and the weight of the collars is
negligible.
P
D
C

2177
11.1 Virtual Work Problem Statement for Example 9
p
M
C
B
A
9 in.
4 in.
9. Determine the moment M applied to the crankshaft that
will keep the piston motionless when a pressure p = 400 psi
acts on the top of the piston and = 25. The diameter of
the piston is 3 in., and the piston slides with negligible
friction in the cylinder.

2178
11.1 Virtual Work Problem Statement for Example 10
L/2
C
E
B
F
D
a
L/2
P
Q
A
10. Pin B is rigidly attached to member AC and moves in the smooth
quarter-circle slot EF. Determine the value of force Q necessary to
keep the system in equilibrium, if = 30, L = 400 mm, a = 120 mm,
and P = 200 N.

2179
11.1 Virtual Work Problem Statement for Example 11
C
B
J
I
K
A
D
E
H
F
G
W
11. A scissors lift is used to raise a weight W = 800 lb.
Determine the force exerted on pin F by the hydraulic
cylinder AF when = 35. Each linkage member is 2-ft
long and pin connected at its midpoint and endpoints. The
lift consists of two identical linkages and cylinders the one
shown and one directly behind it.

2180
11.1 Virtual Work Problem Statement for Example 12
B
C
P
D
k 1.5 kN/m
3 m
A
1.5 m
2 m
4 m
12. The unstretched length of the spring is 1 m. Determine
the value of for equilibrium when force P = 2 kN.

2181
11.1 Virtual Work Problem Statement for Example 13
A
C
B
P
D
E
F
Hinges
10 ft 5 ft 5 ft 5 ft 5 ft
13. a) Determine the moment reaction at the wall F.
b) Determine the force reaction at the roller D.
In both cases P = 60 lb.

2182
11.1 Virtual Work Problem Statement for Example 14
A
C
B
4 m
P
3 m
3 m
14. Determine the vertical reaction at
support C, if P = 2 kN.

2183
11.1 Virtual Work Problem Statement for Example 15
P
Q
A
B
C
D E F G H
I
15. Determine the vertical reaction at
support I of the truss, if P = 10 kip = Q.
5 ft 5 ft 5 ft 5 ft 5 ft 5 ft 5 ft 5 ft

2184
11.1 Virtual Work Problem Statement for Example 16
D
C
m
B
A
16. Determine the tension in the cord. The pulleys are
frictionless and m = 90 kg.

2185
11.1 Virtual Work Problem Statement for Example 17
D
17. Determine the equilibrium values of and for the
two-bar linkage. The couple moment M = 5 N m; each bar is
uniform and has a mass m of 5 kg; the length L = 400 mm; and
the unstretched length of the spring is 250 mm.
k = 0.2 kN/m
500 mm
2
1
A
B
L
L
C
M

2186
11.1 Virtual Work Example 1, page 1 of 5
A
B
L
Smooth surface
P
C
L
W
1. Determine the force P required to keep the two rods in
equilibrium when the angle = 30 and weight W is 50 lb.
The rods are each of length L and of negligible weight.
They are prevented from moving out of the plane of the
figure by supports not shown.

2187
11.1 Virtual Work Example 1, page 2 of 5
W C P
B
A
C

x
C

y
N
The system has one degree of freedom,
because specifying the value of a single
coordinate, , completely determines the
configuration (shape) of the system.
Consider a free-body diagram and identify
the active forces those forces that would
do work if were increased slightly.
1
2
3
4
5
The reactions Cx and C
y
do no
work because point C does not
move. Thus Cx and C
y
are not
active forces.
The force W does work because point B
moves up, so W is an active force.
The normal force N does no work
because it is perpendicular to the
displacement of point A. Thus N
is not an active force.
The force P does work as
point A moves to the right,
so P is an active force.
Free-body diagram (The dashed line shows the position
of the system after has been increased a small amount.)
W
L
C
P
L
B
A

2188
11.1 Virtual Work Example 1, page 3 of 5
W C P
B
A
C

x
C

y
N
y

B
x

A
y

B
x

A
Introduce coordinates measured from a fixed point,
point C in the figure, to the point of application of the
active forces.
Compute the work done when the coordinates are
increased positive infinitesimal amounts, x
A
and x
B

(The custom followed by textbook writers is to use the
Greek letter rather than simply writing dx
A
and dy
B

because the infinitesimals represent hypothetical
motions motions that are possible but are not
necessarily motions that actually occur). The principle
of virtual work says that the total work must add to zero
for all possible motions, real or hypothetical that is,
"virtual."
U = 0: P x
A
W y
B
= 0 (1)
A negative sign is present because the force P and
displacement x
A
are in opposite directions. That is, P
does negative work (absorbs work from the system
rather than adding work to the system). Similarly, the
force W does negative work because W points down
and y
B
is directed up.
6
7

2189
11.1 Virtual Work Example 1, page 4 of 5
x

A
y

B
L
C
L
B
A
Relate the differentials x
A
and y
B
through the change
in the angle, : From the figure, it follows that
y
B
= L sin (2)
To relate y
B
to , use the ordinary formula from
calculus for calculating a differential: if y = f( ), then the
differential is
dy = d
Applying this formula to Eq. 2 and using rather than d
gives
y
B
= L cos (3)
Similarly
x
A
= 2L cos
x
A
= 2L sin (4)
Substitute Eqs. 3 and 4 for y
B
and x
A
into the
virtual-work equation:
2L sin
P x
A
W y
B
= 0 (Eq. 1 repeated)
L cos
8

df
d
or,
(2P sin W cos )(L ) = 0
Because L 0, it follows that
2P sin W cos = 0
Substituting the given values = 30 and W = 50 lb
and solving gives
P = 43.3 lb Ans.

9

2190
11.1 Virtual Work Example 1, page 5 of 5
C P
B
A
C

x
C

y
N
B

y
B

x
B

y
B

x
50 lb
Free body of AB
Free body of BC plus pin at B
B
Observation: the forces acting between the rods at pin B never
occurred in the virtual-work equation because the work done
by the equal-and-opposite force pairs acting between the parts
of the body cancel out for example the work done by Bx
acting on rod AB has the opposite sign of the work done by Bx
acting on rod BC. Forces such as Bx and B
y
would have had to
be considered if equilibrium equations rather than virtual work
had been used.
Often virtual work is easier to use than equilibrium
equations for problems involving connected rigid
bodies (typically machines and mechanisms), but
this advantage exists only if the relation between
displacements can be found easily. If the geometry
is difficult, then using equilibrium equations is
probably the better approach.
10 11

2191
11.1 Virtual Work Example 2, page 1 of 3
C
B
A
W
D
2 ft
2 ft
M
2. Determine the value of moment M required to
maintain the mechanism in the position shown,
if = 35 and W = 200 lb.

2192
11.1 Virtual Work Example 2, page 2 of 3
C
B
A
W
D
2 ft
2 ft
D
W
A
B
C
A

x
A

y
C

x
C

y
M
M
The reactions at A and
C do no work because
points A and C do not
move. Thus the
reactions are not active
forces.
2
The weight W of the
block does work because
the center of gravity of
the block moves
vertically; thus the
weight is an active force.
4
Couple-moment M does
work because member CD
rotates, so M is an active
"force" (better said, "an
active moment" or "active
generalized force")
3
The system has one degree of freedom: specifying the
value of the single coordinate, , completely
determines the configuration of the system. Consider a
free-body diagram and identify the active forces,
1
Free-body diagram (The dashed line shows the position of
the system after has been increased a small amount.)

2193
11.1 Virtual Work Example 2, page 3 of 3
2 ft
D
W
A
B
C
A

x
A

y
C

x
C

y
M
y
D
C
y
y
Introduce a coordinate y measured from the fixed
point A to the point of application of the force W.
Compute the work done when y and are increased a
positive infinitesimal amount.
U = M + W y = 0 (1)
Note that the work done by a moment equals moment
times angle of rotation. Here the work is negative
because M and have opposite senses.
Next use geometry to relate the y and :
y = (2 ft) sin
Differentiating gives
y = 2 cos (2)
Substitute Eq. 2 for y into Eq. 1.
M + W y = 0 (Eq. 1 repeated)
2 cos
Thus
( M + 2W cos ) = 0 (3)

5
Substituting the given values W = 200 lb and
= 35 into Eq. 3 and noting 0 gives
M 2(200 lb) cos 35 = 0
Solving gives
M = 328 lb ft Ans.
6

2194
11.1 Virtual Work Example 3, page 1 of 4
D
A
B
C
W
E
P
F
3 m 3 m
2 m
3 m
3. Determine the value of the weight W
required to maintain the mechanism in the
position shown, if P = 50 N.

2195
11.1 Virtual Work Example 3, page 2 of 4
D
A
B
C
W
E
P
F
F
P
E
W
C
B
A
D
B

x
B

y
E

x
E

y
The system has one degree of freedom because if the
displacement of one end of a bar is known, the
displacement of the other bars can be found by similar
triangles (as will be shown below). Consider a free-body
diagram and identify the active forces corresponding to a
small change in configuration of the system.
1
The force W does work
if A moves vertically, so
W is an active force.
2
The force P does work as point F moves
vertically, so P is an active force.
3
4
Free-body diagram (The dashed line shows the
position of the system after the bars have been
displaced a small amount.)
The reaction forces at B and E
do no work because B and E do
not move; thus the reactions are
not active forces.

2196
11.1 Virtual Work Example 3, page 3 of 4
D
A
B
C
W
E
P
F
y

F
y

F
y

A
y

A
Introduce coordinates measured from a fixed point to
the point of application of the active forces.
Compute the work done when the coordinates
are increased a positive infinitesimal amount.
U = W y
A
+ P y
F
= 0 (1)
The force W does negative work because it is
directed down, while the displacement is up.
5
6
D
y

A
y

D
y

C
3 m
3 m
2 m
3 m
y

F
F E
C
B
A
Now relate the differentials y
A
and y
F
. By
similar triangles
y
D
/2 = y
F
/3 (2)
and
y
A
/3 = y
C
/3 (3)
Member DC does not change length so ends C
and D move down the same amount, that is,
y
C
= y
D
(4)
Eqs. 2, 3, and 4 imply
y
A
= (2/3) y
F
(5)
7

2197
11.1 Virtual Work Example 3, page 4 of 4
Substitute Eq. 5 into the virtual-work equation, Eq. 1:
W y
A
+ P y
F
= 0 (Eq. 1 repeated)
(2/3) y
F
Thus
W(2/3) + P y
F
= 0
or, since y
F
0 and P is given as 50 N,
W(2/3) + 50 N = 0
Solving gives
W = 75 N Ans.
8

2198
11.1 Virtual Work Example 4, page 1 of 7
D
E
300 mm
250 mm
400 mm
150 mm
A
B
C
G
Q
F
P
250 mm
300 mm
4. Determine the force Q necessary to maintain
equilibrium when force P = 400 N.

2199
11.1 Virtual Work Example 4, page 2 of 7
P
F
Q
G
C
B
A
E
D
E

x
E

y
D
E
300 mm
250 mm
400 mm
150 mm
A
B
C
G
Q
F
P
250 mm
300 mm
B

x
B

y
The system has one degree of freedom because if member ABC is
rotated about point B a small amount, then the position of CD and
DEFG can be determined. Consider a free-body diagram and identify
the active forces corresponding to the displacements shown.
P does work so
is an active
force.
Points B and E do not
move, so the reactions at
these points do no work.
1
2
3
4
Free-body diagram (The dashed line shows the position of the
system after the bars have been displaced a small amount.)
Q does
work so is an
active force.

2200
11.1 Virtual Work Example 4, page 3 of 7
B

y
B

x
E

y
E

x
D
E
A
B
C
G
Q
F
P
x

A
x

A
y

G
y

G
Introduce coordinates measured from a fixed point
to the point of application of the forces.
5
Compute the work done when the coordinates are
increased a positive infinitesimal amount.
U = 0: P x
A
+ Q y
G
= 0 (1)
6

2201
11.1 Virtual Work Example 4, page 4 of 7
300
x

A
y

D
F
y

C
300
150
250 D
E
y

G
400
250
A
B
C
G

D

C
All dimensions are in mm.

A
Relate the differentials x
A
and y
G
. Begin by
noting that because
A
is a small angle, the tangent
of
A
can be replaced by the angle itself:
A
= (2)
7
8
x
A
300 mm




300 mm
x
A
x
A
300 mm


Member ABC is a rigid body, and all parts
must rotate the same amount. Thus
C
=
A

Substituting for
A
from Eq. 2 then gives
C
=
Again using the small angle approximation for
the tangent gives
y
C
= (250 mm)
C
= (250 mm)
= (5/6) x
A

Member CD is a rigid body and thus doesn't
shorten or lengthen. It follows that
y
D
= y
C
Thus
y
D
= (5/6) x
A
(3)
9
10

2202
11.1 Virtual Work Example 4, page 5 of 7
300
x

A
y

D
F
y

C
300
150
250 D
E
y

G
400
250
A
B
C
G
All dimensions are in mm.
11


300 mm
x
A

Using the small angle approximation for the
tangent gives
D
=
[(5/6) x
A
], by Eq. 3
=
Member DEFG is a rigid body and so all parts
must rotate the same amount. Thus
F
=
D
Thus
F
= (4)
y
D
250 mm

x
A
300 mm
12
F

D

2203
11.1 Virtual Work Example 4, page 6 of 7
G
400 mm
E
F
300 mm
(500 mm)

F

F
The remaining step is to relate y
G
to
F
. We can do
this in two ways, by geometry or by calculus. Let's begin
with the geometric approach. First consider a rotation of
line EG by an amount
F
.
13
15
16 y
G
is the vertical distance that point G moves up.
So, considering the small triangle gives
| y
G
| = (500
F
) sin
or,
y
G
= (5)
(Insert a minus sign, because y
G
was originally
defined as positive down)
x
A
300
400 mm
500 mm
=
4
5
, by Eq. 4
4
3
x
A
14 Length = (300 mm)
2
+ (400 mm)
2
= 500 mm

2204
11.1 Virtual Work Example 4, page 7 of 7
G 400 mm
E
F
300 mm
500 mm y

G
Consider an alternative solution for y
G
,
based on calculus:
17
y
G
= (500 mm) cos
y
G
= 500 sin
= 500
( )

F

=
= (Same as Eq. 5)
=
F
because both angles
measure the rotation of line EG)
500 mm
400 mm
x
A
300
, by Eq. 4
400
300
x
A
3
18
Note that we can't write
y
G
= 300 mm
and then differentiate to get y
G
(which would give y
G
= 0).
The equation for y
G
must define a continuous and differentiable
function, not a relationship that is only valid at a single value
of .
Substitute for y
G
from Eq. 5 into the virtual work equation:
P x
A
+ Q y
G
= 0 (Eq. 1 repeated)
, by Eq. 5
or,
[P + ( 4/3)Q] x
A
= 0
Dividing through by x
A
and using the given value P = 400 N
yields
Q = 300 N Ans.
x
A
4
3
19
20
4 x
A

2205
11.1 Virtual Work Example 5, page 1 of 4
D
A
B C
F
P
E
Q
L
L/2 L/2
5. Link AB is connected to collar A, which can slide
with negligible friction on horizontal rod EF. Determine
the value of force Q necessary to maintain equilibrium
when = 50, L = 300 mm, and P = 100 N.

2206
11.1 Virtual Work Example 5, page 2 of 4
D
A
B C
F
P
E
Q
Active force
P
C B
A
D
D

x
D

y
A

y
Forces from pin D are
not active forces.
L
The force from the rod acting on
the collar is not an active force.
Q
L/2 L/2
Free-body diagram showing active forces
corresponding to a small increase in
1
Active force

2207
11.1 Virtual Work Example 5, page 3 of 4
Q
C B
A
D
D

x
D

y
A

y
P
x

A
x

A
y

C
y

C
Introduce coordinates measured from the fixed point D
to the point of application of the active forces.
Compute the work done when the coordinates are
increased a positive infinitesimal amount.
U = 0: Q x
A
+ P y
C
= 0 (1)
2
3
x

A
L
L/2
L/2
D
A
B
C
Relate the differential x
A
to
the change in angle,
4
x
A
= L cos + +
x
A
= L sin + + (2)
2
L
2
L
2
L
2
L
(Length L does not change)
0
0
5

2208
11.1 Virtual Work Example 5, page 4 of 4
L
L/2
L/2
D
A
B
C
y

B
y

B
y

C
Relate the differential y
C
to the change in
angle From the figure, we have
y
B
= L sin
y
B
= L cos
By similar triangles,
L cos
Thus
y
C
= (3)
y
B
L/2 + L/2
=
L/2
y
C
6
Substitute Eqs. 2 and 3 for x
A
and y
C
into the virtual
work equation, Eq. 1:
L sin , by Eq. 2
Q x
A
+ P y
C
= 0 (Eq. 1 repeated)
, by Eq. 3
Thus
( Q sin + P )(L ) = 0
Because L 0, it follow that
Q sin + P = 0
Substituting = 50 and P = 100 N and solving for Q
gives
Q = 42.0 N Ans.
2
L cos
L cos
2
cos
2
7
2
cos

2209
11.1 Virtual Work Example 6, page 1 of 5
C
W
B
150 mm
A
D
150 mm
6. Rotating the threaded rod AC of the automobile jack causes
joints A and C to move closer together, thus raising the weight W.
Determine the axial force in the rod, if = 30 and W = 2 kN.
150 mm
150 mm

2210
11.1 Virtual Work Example 6, page 2 of 5
C
W
B
A
D
F

r
F

r
1
2
The system has one degree of freedom because once
is specified, the location of all parts of the jack can be
determined. Consider a free-body diagram of the jack
and identify the active forces corresponding to a small
change in .
To get a virtual-work equation that contains the
axial force in the rod, it is necessary to exclude the
rod from the free-body diagram. The effect of the
rod is then represented by the two forces Fr. W and
the two Fr forces are the active forces.
Free-body diagram

2211
11.1 Virtual Work Example 6, page 3 of 5
F

r
F

r
D
B
W
C
A
y

B
x

A
x

C
Introduce coordinates measured from fixed points to
the points of application of the active forces.
Calculate the work done.
U = 0: W y
B
Fr x
A
Fr
x
C
= 0 (1)
3
4
x
A
y
B
x
C

2212
11.1 Virtual Work Example 6, page 4 of 5
x
C
150 mm
150 mm
D
A
B
C
x
A
= (150 mm) cos
x
A
= 150 sin (2)
x
C
= (150 mm) cos
x
C
= 150 sin (3)
y
B
= 2(150 mm) sin + a
y
B
= 300 cos + a (4)
0
y
B
a
6
E
150 mm
150 mm
a
Distance "a," from the intersection of the two sloping
members, point E, to point B, does not change as
changes. Thus when "a" is differentiated, the result is
zero: a = 0.
D
A
B W
C
5
7
Relate y
B
, x
A
, and x
C
through the angle
change, d .
x
A

2213
11.1 Virtual Work Example 6, page 5 of 5
Substituting the expressions for y
B
, x
A
, and x
C
and into the
virtual work equation, Eq. 1, gives
300 cos by Eq. 4 150 sin by Eq. 3
W y
B
Fr
x
A
Fr
x
C
= 0 (Eq. 1 repeated)
150 sin by Eq. 2

or,
[ 300W cos + 2(150)Fr sin ] = 0
Dividing through by , substituting the given values W = 2 kN
and = 30, and solving gives
Fr = 3.46 kN Ans.
8

2214
11.1 Virtual Work Example 7, page 1 of 5
A
B
L
C
P
k
F
E D
L
L L
L L
7. The original length of the spring is L. Determine the
angle for equilibrium if L = 3 m and P = 300 N.
Spring constant,
k = 200 N/m

2215
11.1 Virtual Work Example 7, page 2 of 5
1
The spring is not part of the
free-body; the effect of the
spring is represented by the
forces Fs.
2
3
4 Forces P and Fs are
active forces because
their points of
application move in
the direction of the
forces.
Reactions A
y
and B
y
do
no work because they
are perpendicular to the
displacement of points
A and B.
Free-body diagram
A
B
L
C
P
k
F
E D
L
L L
L L
D
E
F
P
C
B
A
F

s F

s
A

y
B

y
The system can be described by a single coordinate,
Consider a free-body diagram and identify the active
forces corresponding to a small change in

2216
11.1 Virtual Work Example 7, page 3 of 5
D E
F
P
C
B
A
F

s F

s
A

y
B

y
y

F
y

F
x

D
x

E
x

E
x

D
O
Introduce coordinates measured from the point O directly
above pin C to the point of application of the active
forces.
Compute the work done when the coordinates are
increased a positive infinitesimal amount.
U = P y
F
+ Fs x
D
+ Fs x
E
= 0 (1)
5
6

2217
11.1 Virtual Work Example 7, page 4 of 5
D E
F
C
B A
y

F
O
L L
L L
L L
x

D
x
E
Relate the differentials y
F
, x
D
, and x
E
to the angle
change From the figure, we see that
y
F
= 3L sin
x
E
= L cos
x
D
= L cos
Differentiating gives
y
F
= 3L cos (2)
x
E
= L sin (3)
x
D
= L sin (4)
We can use the same figure to calculate the length of the
spring, L , say:
L = distance DE
= 2L cos (5)
7

2218
11.1 Virtual Work Example 7, page 5 of 5
8
Substituting the given values P = 300 N,
k = 200 N/m, and L = 3 m into Eq. 7 and
solving numerically gives
= 69.1 Ans.
The force in the spring is, then,
Fs = k compression of spring
= k (original length final length)
L (given) L = 2L cos , by Eq. 5
= kL(1 2 cos ) (6)
Substitute from Eqs. 2, 3, 4, and 6 into the virtual
work equation, Eq. 1:
kL(1 2 cos ), by Eq. 6
P y
F
+ Fs x
D
+ Fs x
E
= 0 (Eq. 1 repeated)
3L cos by Eq. 2 L sin by Eq. 3 L sin by Eq. 4
Thus
3P cos 2kL(1 2 cos ) sin (L ) = 0
or, since L 0,
3P cos 2kL(1 2 cos ) sin = 0 (7)
9

2219
11.1 Virtual Work Example 8, page 1 of 5
E
B
A
x
y
Q
k 9 kN/m
8. Collars A and B can slide freely on rods CD and CE. Determine the
values of x and y, given that forces P = 900 N and Q = 800 N. The
unstretched length of the spring is 0.2 m, and the weight of the collars is
negligible.
P
D
C

2220
11.1 Virtual Work Example 8, page 2 of 5
E
B
A
x
y
Q
k 9 kN/m
B
800 N
900 N
A
N

B
(Force from rod CE)
N

A
(Force from rod CD)
F

s
F

s
The system has two degree of freedom since both x and
y coordinates must be known if the configuration of the
system is to be determined. Consider a free-body
diagram and identify the active forces corresponding to
a small change in x, while y is held fixed.
Because y is fixed,
collar B does not
move, and so none
of the forces acting
on B is an active
force.
1
2
3
The forces acting on
collar A, Fs (the spring
force) and the 900-N
force, are the only active
forces.
Free-body diagram
P
D
C

2221
11.1 Virtual Work Example 8, page 3 of 5
A
x
y
900 N
800 N
y 0
(y held fixed)
F

s
F

s
N

B
N

A
L
x
L
The coordinate x locates the position of the
700-N force. Introduce an additional
coordinate, L, that locates the point of
application of the spring force, Fs.

Compute the work done:
U = 0: (900 N) x Fs L = 0 (1)
Relate x and L:
L
2
= x
2
+ y
2
(2)
Differentiating gives
2L L = 2x x + 2y y
0 (Because y is fixed)
Thus
L = x/L x
Introduce the latter equation into Eq. 1:
(900 N) x Fs L = 0 (Eq. 1 repeated)
x/L x
Thus
(900 Fsx/L) x = 0
Dividing through by x and re-arranging gives
Fsx = 900L (3)
4 5
C
B

2222
11.1 Virtual Work Example 8, page 4 of 5
C
B
A
x(fixed)
y
900 N
800 N
y
F

s
F

s
N

B
N

A
L
L
Next, hold x fixed and compute the work done when
collar B moves an amount y. Following the same
steps as were used for x leads to
Fsy = 800L (4)
The spring force, Fs, is related to L:
Fs = k extension of the spring
= (9000 N/m) (L original length)
0.2 m
Thus
Fs = 9000L 1800 (5)
We now have four simultaneous nonlinear equations
to solve:
L
2
= x
2
+ y
2
(2)
Fsx = 900L (3)
Fsy = 800L (4)
Fs = 9000L 1800 (5)
6

2223
11.1 Virtual Work Example 8, page 5 of 5
These equations can be solved directly if a calculator that
is able to handle such systems is available.
Alternatively, proceed as follows: square both sides of
Eqs. 3 and 4 and add the results to get
(Fsx)
2
+ (Fsy)
2
= (900L)
2
+ (800L)
2
.
or
Fs
2
(x
2
+ y
2
) = L
2
(900
2
+ 800
2
)
L
2
, by Eq. 2
Solving gives
Fs = 1204 N
Using this result in Eq. 5 gives
Fs = 9000L 1800 (Eq. 5 repeated)
1204 N
Solving gives
L = 0.3338 m
.
7 Distance x can now be found from Eq. 3:
Fsx = 900L (Eq. 3 repeated)
1204 N 0.3338 m
Solving gives
x = 0.250 m Ans.
Distance y can be found from Eq. 4:
Fsy = 800L (Eq. 4 repeated)
1204 N 0.3338 m
Solving gives
y = 0.222 m Ans.
8

2224
11.1 Virtual Work Example 9, page 1 of 4
p
M
C
B
A
M
A
B
C
A
y
Ax
N
9 in.
4 in.
9. Determine the moment M applied to the crankshaft that
will keep the piston motionless when a pressure p = 400 psi
acts on the top of the piston and = 25. The diameter of
the piston is 3 in., and the piston slides with negligible
friction in the cylinder.
The system can be described by a single coordinate, .
Consider a free-body diagram and identify the active
forces corresponding a small change in .
Since friction is negligible, only
the normal force N acts on the
side of the piston. The normal
force does no work since it acts
perpendicular to the motion of
the piston.
1
3
4
2 The resultant of the pressure is an active force:
(400 psi)( )(3 in./2)
2
= 2827 lb
The reaction forces Ax and A
y

do no work, because point A
does not move. The moment M
does work as link AB rotates.
Free-body diagram

2225
11.1 Virtual Work Example 9, page 2 of 4
M
A
B
C
A
y
Ax
N
2827 lb
y

C
y

C
d
Introduce coordinates measured from
a fixed reference at point A.
Compute the work done when the coordinates are increased
a positive infinitesimal amount.
U = (2827 lb) y
C
M = 0 (1)
5
6

2226
11.1 Virtual Work Example 9, page 3 of 4
A
C
A
y
Ax
N
2827 lb
A
C
9 in.
4 in.
y

C
Distance "a" does not
change as is
changed so a = 0.
B
a
Relate y
C
to
y
C
= (4 in.) cos + (9 in.) cos + a
y
C
= 4 sin 9 sin + a (2)
0
Relate to by the law of sines,
(3)
Differentiating gives
Thus
(4)
7
sin
4 in.
=
sin
9 in.
cos
4
=
cos
9
=
4 cos
9 cos

8

2227
11.1 Virtual Work Example 9, page 4 of 4
4 cos
9 cos
sin
4 in.
=
sin
9 in.
Using Eq. 4 in Eq. 2 gives
yC = 4 sin 9 sin (Eq. 2 repeated)
, by Eq. 4

= ( 4 sin 4 tan cos ) (5)
Substituting Eq. 5 in the virtual work equation gives
(2827) yC M = 0 (Eq. 1 repeated)
( 4 sin 4 tan cos ) by Eq. 5
or
[4(2827)(sin + tan cos ) M = 0
Dividing through by and solving gives
M = 4(2827)(sin tan cos ) (6)
Substituting the given value = 25 into Eq. 3 yields
(Eq. 3 repeated)
which can be solved to give = 10.83.
Using = 25 and = 10.83 in Eq. 6 produces
M = 0 lb in Ans.
Observation: this problem may be more easily
solved by using equations of equilibrium.
Virtual work is superior to using equations of
equilibrium provided that the relation between
displacements can be easily obtained. In the
present problem, deriving the relation between
and y
C
, Eq. 5, was more complicated than
simply writing equations of equilibrium.
9
11
10

2228
11.1 Virtual Work Example 10, page 1 of 5
L/2
C
E
B
F
D
a
L/2
P
Q
A
10. Pin B is rigidly attached to member AC and moves in the smooth
quarter-circle slot EF. Determine the value of force Q necessary to
keep the system in equilibrium, if = 30, L = 400 mm, a = 120 mm,
and P = 200 N.

2229
11.1 Virtual Work Example 10, page 2 of 5
C
B
D
Q (active)
A

y
A
P (active)
R
Pin B must move in the slot, that is, in a direction tangent to
the quarter circle and thus perpendicular to radius DB.
Thus the force R from the slot does no work because R is
perpendicular to the motion of the pin.
The system configuration can be
defined by the single coordinate, .
Consider a free-body diagram
showing the active forces
corresponding to a small increase in
1
Free-body diagram
L/2
C
E
B
F
D
a
L/2
P
Q
A
3 Point A must move
horizontally. It is difficult
to tell if A moves to the
right or left, but fortunately
it makes no difference. The
important thing is to note
that force Ay from the
rollers does no work so is
not an active force.
2
Motion of pin B
is perpendicular
to BD.
Point C moves both
horizontallly and vertically.
It is difficult to tell if C
moves vertically up or
vertically down, but it
makes no difference. All
we need to note is that
force P is an active force.
4

2230
11.1 Virtual Work Example 10, page 3 of 5
C
B
D
Q
A

y
A
P
R
x

A
x

A
y

C
y

C
Introduce coordinates measured from
the fixed point D to the point of
application of the active forces P and Q.
5
Compute the work done when the coordinate are
increased a positive infinitesimal amount.
U = 0: Q x
A
+ P y
C
= 0 (1)
6

2231
11.1 Virtual Work Example 10, page 4 of 5
B
D
A
L/2
L/2
C
a
x

A
y

C
Relate the differentials x
A
and y
C
to the angles and
Begin with y
C
.
y
C
= L sin
y
C
= L cos (2)
Note that this equation shows y
C
is positive if is
positive, that is, point C moves up as increases.
x
A
= (L/2) cos a cos
x
A
= (L/2) sin + a sin (3)
8
Relate and through the law of sines:

Because sin (180 ) = sin , the last equation can
be written as
a sin = (L/2) sin (4)
Differentiating gives
a cos = (L/2) cos
so
= (5)
L cos
2a cos
9
sin (180 )
L/2
sin
a
=
2a cos
L cos
Using Eq. 5 in Eq. 3 gives
x
A
= (L/2) sin + a sin (Eq. 3 repeated)
, by Eq. 5
= ( sin + cos tan )(L/2) (6)
10
7

2232
11.1 Virtual Work Example 10, page 5 of 5
11 The angle in Eq. 6 can be calculated by substituting the given
values = 30, L = 400 mm, and a = 120 mm into Eq. 4:
a sin = L/2 sin (Eq. 4 repeated)
and solving to get = 56.44.
Although it is not necessary for solving the problem, we can
now determine whether point A moves to the left or to the
right. From Eq. 6 we have
x
A
= ( sin + cos tan )(L/2) Eq. 6 repeated)
Substituting = 30 and = 56.44 into this equation gives
x
A
= (0.8054)(L/2)
That is, x
A
is positive when is positive, so x
A
increases,
that is, point A moves to the left for the particular values of
a, and L of our problem.
Substituting Eqs. 2 and 7 for y
C
and x
A
into the
virtual-work equation, Eq. 1, gives
(0.8054)(L/2) , by Eq. 7
Q x
A
+ P y
C
= 0 (Eq. 1 repeated)
L cos , by Eq. 2
so
Q(0.8054)/2 + P cos (L ) = 0
Because L 0, it follows that
Q(0.8054)/2 + P cos = 0
Substituting = 30 and P = 200 N and solving gives
Q = 430 N Ans.
12

2233
11.1 Virtual Work Example 11, page 1 of 5
D
E
H
F
G
W
11. A scissors lift is used to raise a weight W = 800 lb.
Determine the force exerted on pin F by the hydraulic
cylinder AF when = 35. Each linkage member is 2-ft
long and pin connected at its midpoint and endpoints. The
lift consists of two identical linkages and cylinders the one
shown and one directly behind it.
C
B
J
I
K
A

2234
11.1 Virtual Work Example 11, page 2 of 5
B
A
F
W/2
A

x
A

y
B

y
F
FA
Consider a free-body diagram and identify the active
forces associated with a small change in
1
3
Each side of the lift carries half of
the load. The W/2 load does work,
so it is an active force.
2
The force F
FA
of the hydraulic
cylinder acting on pin F does
work as pin F moves.
The force F
FA
of the hydraulic
cylinder acting on pin A does no
work because pin A does not
move. For the same reason, the
reaction forces Ax and A
y
from the
support do no work.
4
5 The reaction force at B does no
work because it is vertical while
the motion of point B is
horizontal.
Free-body diagram

2235
11.1 Virtual Work Example 11, page 3 of 5
J
A
F
y

J
y

J
s

F
F

FA
s

F
Introduce coordinates y
J
and s
F
measured from the fixed point A
to the point of application of the active forces.
Compute the work done when the coordinates are increased a
positive infinitesimal amount:
U = (W/2) y
J
+ F
FA
s
F
= 0 (1)
6
W/2

2236
11.1 Virtual Work Example 11, page 4 of 5
C
B
J K
A
D
E
H
F
G
O
A
F
O
C
s

F
(3 ft) sin
(1 ft) cos
(1 ft) sin
(1 ft) sin
(1 ft) sin
(1 ft) sin
(1 ft) sin
(1 ft) sin
y

J
(1 ft) cos
1 ft
1 ft
1 ft
1 ft
I
1 ft
Relate the coordinate y
J
to the angle
y
J
= (6 ft) sin
y
J
= 6 cos (2)
To relate s
F
to ,
consider triangle AFCO.
Use the Pythagorean Theorem and then differentiate to get s
F
.
s
F
= (3 sin )
2
+ (1 cos )
2
s
F
=
=
2(3 sin )(3 cos ) + 2(cos )( sin )
1
2
(3 sin )
2
+ (1 cos )
2
8 sin cos
9 sin
2
+ cos
2
(3)
7
8
9
1 ft

2237
11.1 Virtual Work Example 11, page 5 of 5
Substituting for s
F
and y
J
from Eqs. 2 and 3 in the
virtual-work equation, Eq. 1, gives
(W/2) y
J
+ F
FA
s
F
= 0 (Eq. 1 repeated)
Thus

3W + ] cos = 0
This implies, since cos 0, that
3W +
Substituting the given values = 35 and W = 800 lb and
solving gives
F
FA
= 997 lb Ans.
6 cos , by Eq. 2
8 sin cos
9 sin
2
+ cos
2
, by Eq. 3
9 sin
2
+ cos
2
8F
FA
sin
8F
FA
sin
9 sin
2
+ cos
2
= 0
Observation: Solving this problem by using
the equations of equilibrium would have
required drawing several free-body diagrams
and writing equations for each diagram.
Solving the problem by virtual work is much
easier because we don't have to calculate the
forces acting between the various links. In
general, problems involving connected rigid
bodies can be solved more easily by virtual
work than by equilibrium equations provided
that the relations between displacements can
be easily obtained.
10 11

2238
11.1 Virtual Work Example 12, page 1 of 5
k 1.5 kN/m
3 m
A
1.5 m
C
B

y
B

x
F

s
D
P
2 m
4 m
B
12. The unstretched length of the spring is 1 m. Determine
the value of for equilibrium when force P = 2 kN.
The system can be described by a single
coordinate, . Consider a free-body
diagram and identify the active forces
corresponding to a small change in .
The spring is not part of
the free-body; the effect of
the spring is represented
by the force Fs, which is an
active force.
Force P is an active force.
1
2
3
Free-body diagram
B
C
P
D

2239
11.1 Virtual Work Example 12, page 2 of 5
Procedures and Strategies for Solving Problems Involving
Potential Energy
1. Make a sketch that shows the system in an arbitrary position.
2. Express the gravitational potential energy of each weight W
i
in the
system as W
i
y
i
, where y
i
is the vertical distance measured from a
fixed reference point to the weight.
3. Express the elastic potential energy of each spring in the system
with spring constant k
i
as (1/2)k
i
s
i
2
, where s
i
is the change in length
of the spring.
4. Add the gravitational and elastic potential energies to obtain the
total system potential energy, V.
4. Express V as a function of a single variable, q, by using geometry to
express all distances s
i
and y
i
in terms of q.
5. Obtain the equilibrium equation of the system by setting the
derivative of V with respect to q to zero. Solve this equation for
the value of q or, if q is given, for the value of one of the other
parameters of the problem that you were asked to determine, such
as a weight or spring constant.
6. Determine the stability of the equilibrium position by calculating
the second derivative of V. If the second derivative is positive,
then the equilibrium is stable; if the second derivative is negative,
the equilibrium is unstable; and if derivatives of all orders equal
zero, the equilibrium is neutral.
k
L/2
L/2
s
y
W
B
C
s = (L/2) sin

A
A
L/2
L/2
B
C
y = L sin

2240
11.1 Virtual Work Example 12, page 3 of 5
C
D
A
B
y

D
4 m
3 m
1.5 m
2 m
s

C
s

B
Relate the differentials s
C
and y
D
to the angle
change
y
D
= (2 m + 4 m) sin
y
D
= 6 cos (2)
Law of cosines applied to triangle ABC:
s
C
2
= (2 m)
2
+ (s
B
)
2
2(2 m)(s
B
) cos ( + ) (3)
Here
s
B
= (3 m)
2
+ (1.5 m)
2
= 3.354 m (4)
and
= tan
-1
= 26.565 (5)
To avoid having to write equations containing several four and
five-digit numbers, introduce intermediate variables a and b:
s
C
2
= (2
2
+ s
B
2
) 2(2)(s
B
) cos ( + ) (Eq. 3 repeated)
Thus
s
C
2
= a b cos ( + ) (6)
where
3.354 m by Eq. 4
a = 2
2
+ (s
B
)
2
= 4 + 3.354
2
= 15.249 m (7)

1.5 m
3 m
6
7
8
a b

2241
11.1 Virtual Work Example 12, page 4 of 5


b sin ( + )
2s
C
The parameter b can also be evaluated, for later
use:
3.354 m, by Eq. 4
b = 2(2)(s
B
)
= 4(3.354)
= 13.416 m (8)
s
C
can be related to by differentiating Eq. 6:
s
C
2
= a b cos ( + ) (Eq. 6 repeated)
2s
C
s
C
= b sin ( + )
so
s
C
= (9)
Taking the square root of both sides of Eq. 6
gives an equation for s
C
.
s
C
= a b cos ( + ) (10)
b sin ( + )
2s
C
The spring force Fs can be expressed in terms of s
C
:
Fs = k (extension of the spring)
= k (stretched length unstretched length)
= k (s
C
1 m) (11)
Substituting for y
D
, s
C
, and Fs from Eqs 2, 9, and 11 into
the virtual-work equation, Eq. 1, gives
k(s
C
1), by Eq. 11
P y
D
Fs s
C
= 0 (Eq. 1 repeated)
6 cos , by Eq. 2 , by Eq. 9
or
[(6P) cos k(s
C
1) = 0
Since 0, the expression in brackets must equal to
zero.
(6P) cos k(s
C
1) = 0 (12)
10
9
2s
C
b sin ( + )
2s
C
b sin ( + )

2242
11.1 Virtual Work Example 12, page 5 of 5
11 Eq. 12 contains the distance s
C
, which can be calculated by using Eq. 10:
(6P) cos k(s
C
1) = 0 (Eq. 12 repeated)
, by Eq. 10
or
(6P) cos k( a b cos ( + ) 1) = 0
Substituting in the latter equation the values
P = 2 kN (Given)
k = 1.5 kN/m (Given)
a = 15.249 m (Eq. 7 repeated)
b = 13.416 m (Eq. 8 repeated)
= 26.565 (Eq. 5 repeated)
and solving numerically gives
= 53.4 Ans.
a b cos ( + )
2
b sin ( + )
a b cos ( + )
b sin ( + )
2s
C

2243
11.1 Virtual Work Example 13, page 1 of 6
A
C
B
P
D
E
F
Hinges
10 ft 5 ft 5 ft 5 ft 5 ft
13. a) Determine the moment reaction at the wall F.
b) Determine the force reaction at the roller D.
In both cases P = 60 lb.
Part a. Replace the wall at F by a moment
couple M
F
and a pin support.
1
A
C
B
P
D
E
F
M
F

2244
11.1 Virtual Work Example 13, page 2 of 6
A
C
B
P
D
E
F
F

x
F

y
D

y
B

y
M
F
A C
B
P
D
E
F
F

x
F

y
D

y
B

y
y

A
y

A
Consider a free-body diagram and identify the active forces
associated with a small rotation of the segments of the beam
Introduce coordinates y
A
and for calculating the work. U = 0: P y
A
M
F
= 0 (1)
2
4
The active "forces" are the force
P and the couple moment M
F
.
3
Free-body diagram
M
F

2245
11.1 Virtual Work Example 13, page 3 of 6
A
C B
D
E F 5 ft 5 ft
5 ft 5 ft
y

A
y

C
y

E
10 ft
Relate the differentials y
A
and 5
By similar triangles,
y
E
= y
C
and y
C
= y
A

so
y
E
= y
A
For small angles, y
E
is given by
y
E
= (10 ft)
But this becomes, after using the relation y
E
= y
A
,
y
A
= 10 (2)
Substitute this result in the virtual-work equation to get
P y
A
M
F
= 0 (Eq. 1 repeated)
10
Dividing through by , substituting the known value P = 60 lb,
and then solving for M
F
gives,
M
F
= 600 lb ft Ans.

6
7

2246
11.1 Virtual Work Example 13, page 4 of 6
A
C
B
60 lb
D E F
D

y
A
C
B
60 lb
D E F
F

y
D

y
B

y
M
F
A
C
B
60 lb
D E F
D

y
y

D
y

D
y

A
y

A
Part b. Replace the roller at D by a vertical force, D
y
. 8
Draw a free-body diagram and show a
small rotation of segments AC and CE.
9
Segment EF of the beam does
not move because the wall
support prevents both rotation
and vertical displacement.
Thus M
F
and F
y
do no work.
Introduce coordinates y
A
and y
D

for calculating the work. U = 0: (60 lb) y
A
D
y
y
D
= 0 (3)
10
12
The active forces are
the 60-lb force and D
y
.
11

2247
11.1 Virtual Work Example 13, page 5 of 6
A
C
B D
y

D
y

A
5 ft 5 ft
5 ft
5 ft
y

C
E F
Relate the differentials y
A
and y
D
.
By similar triangles,
y
A
= y
C
and
=
Eliminating y
C
gives
y
D
=
Use this equation to replace y
D
in the virtual-work equation:
(60) y
A
D
y
y
D
= 0 (Eq. 3 repeated)
Dividing through by y
A
and solving gives
D
y
= 120 lb Ans.
y
C
5 + 5
y
D
5
y
A
2
y
A
2
13
14

2248
11.1 Virtual Work Example 13, page 6 of 6
A
C
B
60 lb
D
E
F
F

y
A
C
B
60 lb
D
E
F
F

y
Comment: Let's extend the discussion. If we were asked to
calculate the vertical reaction force at the wall, we would
replace the wall by a support that prevents rotation but permits
vertical displacement.
15
Corresponding displacements 16
Segment EF translates but does not rotate. Thus the reaction
moment at the support does no work. The reaction force, F
y
,
however, does work and is thus an active force.
Observation: The method applied in this
beam example can be generalized.
Virtual work can be used to calculate a
force of constraint (a reaction) by
considering displacements which violate
the constraints and then accounting for
the work done by the force of constraint.
This procedure is equivalent to
converting a rigid structure into a
mechanism, as was done at the
beginning of the present example.
18
17

2249
11.1 Virtual Work Example 14, page 1 of 5
A
C
B
4 m
A
C
B
P
P
C

y
3 m
3 m
14. Determine the vertical reaction at
support C, if P = 2 kN.
Convert the structure
into a mechanism with
one degree of freedom
by replacing the pin
support at C by a roller
support and a vertical
force, C
y
.
1

2250
11.1 Virtual Work Example 14, page 2 of 5
P
C

x
C

y
A

x
A

y
A
C
B
P
C

x
C

y
A

x
A

y
y

C
y

C
y

B
y

B
C
y
and P are active
forces for the
displacements
shown.
Define coordinates y
B
and y
C
locating the point of
application of the active forces, and compute the work.
U = 0: P y
B
+ C
y
y
C
= 0 (1)
2
3
A
B
C

2251
11.1 Virtual Work Example 14, page 3 of 5
A
C
B
3 m
4 m
3 m
A
C
B
y

B
y

C
5 m
BC = (3 m)
2
+ (3 m)
2
2 3 m
2 3 m
AB = (3 m)
2
+ (4 m)
2
5 m
Relate the differentials y
B
and y
C
through the angles and
Begin by computing the lengths of bars BC and BA (Note that
these lengths do not change, as the angles and change).
From the above figure,
y
B
= (5 m) cos
y
B
= 5 sin (2)
y
C
= (5 m) cos + (3 2 m) cos
y
C
= 5 sin (3 2 ) sin (3)
Use the law of sines to relate and
(4)
4
5
6
7
=
3 2 m
5 m
sin sin

2252
11.1 Virtual Work Example 14, page 4 of 5
Differentiating Eq. 4 gives
Thus
= (5)
The equation relating and , Eq. 5, can be used in Eq. 3
to express y
C
in terms of alone:
y
C
= 5 sin 3 2 sin (Eq. 3 repeated)
= ( 5 sin 5 tan cos ) (6)
Substituting Eqs. 2 and 6 for y
B
and y
C
into the virtual
work equation gives
5 sin , by Eq. 2
P y
B
+ C
y
y
C
= 0 (Eq. 1 repeated)
( 5 sin 5 tan cos ) , by Eq. 6
=
3 2
5
cos cos
5 cos
2 cos

3
3 2 cos

5 cos
8
Thus
P sin C
y
(sin + tan cos ) (5 ) = 0
Dividing by 5 gives
P sin C
y
(sin + tan cos ) = 0 (7)
9

2253
11.1 Virtual Work Example 14, page 5 of 5
3 m
4 m
3 m
A
C
B
5 m
tan = = 1
sin =
cos =
3
3
3
5
4
5
Evaluating the functions of and in Eq. 7, substituting the given
value P = 2 kN, and then solving gives
P sin C
y
(sin + tan cos ) = 0 (Eq. 7 repeated)
C
y
= 0.857 kN Ans.
3
5
3
5
4
5
1
10
Observation: This example demonstrates that virtual
work can be used to calculate the reaction forces
from the supports acting on a structure. The example
also demonstrates that just because virtual work can
be used doesn't necessarily mean that it should be
used the reaction at support C could have been
found much more easily by employing equilibrium
equations. The usefulness of virtual work depends
on how easy it is to express relations between
coordinates.
11

2254
11.1 Virtual Work Example 15, page 1 of 3
5 ft 5 ft 5 ft
P
Q
A
B
C
D E F G H
I
1 Convert the structure to a mechanism with one degree of
freedom by replacing the pin support at I by a vertical force I
y

and a roller.
P
Q
A
B C D E F G H I
15. Determine the vertical reaction at
support I of the truss, if P = 10 kip = Q.
I
y
5 ft 5 ft 5 ft 5 ft 5 ft

2255
11.1 Virtual Work Example 15, page 2 of 3
P Q
A
B
C D
E
F
G H I
Identify the active forces corresponding to a set of
displacements compatible with the constraints.
I
y
B
y
D
y
F
y
H
y
2
P
Q
A
B
C
D E F
G H I
B
y
D
y
F
y
H
y
y
I
y
I
y
E
y
E
y
A
y
A
3 Introduce coordinates measured from fixed points to the
points of application of the applied forces.
Calculate the work done.
U = 0: P yA + Q yE Iy yI = 0 (1)
I
y
Ix
Ix

2256
11.1 Virtual Work Example 15, page 3 of 3
A
B
C D
E
F G H
I
Relate y
A
, y
E
, and y
I

by geometry (similar
triangles).
4
5 ft 5 ft 5 ft
5 ft 5 ft 5 ft 5 ft
5 ft
y
A
= y
C
(2)
Similarly,
y
C
= y
E
, y
E
= y
G
, and y
G
= y
I
These equations imply
y
A
= y
I
and y
E
= y
I
Substituting the latter pair of equations into the
virtual-work equation, Eq. 1, gives

P y
A
+ Q y
E
I

y
I
= 0 (Eq. 1 repeated)
y
I
y
I
or
(P + Q I
y
) y
I
= 0
Dividing through by y
I
, substituting the given values P
= 10 kip = Q, and solving gives
I
y
= 20 kip Ans.
y
A
y
C
y
E
y
G
y
I

2257
11.1 Virtual Work Example 16, page 1 of 4
D
C
m
B
A
D
C
B
A
T
16. Determine the tension in the cord. The pulleys are
frictionless and m = 90 kg.
Convert the pulley-cord
system into a mechanism
with one degree of
freedom by replacing the
support B by a tensile
force T acting on the end
of the cord.
1
Weight = mg

2258
11.1 Virtual Work Example 16, page 2 of 4
D
C
B
A
T
mg
y

D
y

B
y

B
y

D
The tension T and the weight do work (are active
forces) if end B of the cord moves up a small
amount.
Introduce the coordinates y
B
and y
D
.
2
3
Calculate the work done:
U = 0: T y
B
+ mg y
D
= 0 (1)
4

2259
11.1 Virtual Work Example 16, page 3 of 4
D
C
B
A
y

D
y

B
t
s
Diameter d

C
Diameter d

D
5 Relate y
B
to y
D
by first expressing the length, say L, of the cord
in terms of y
B
and y
D
:
L = (y
D
t) y
B

+ (y
D
t) u s
+ (y
D
t) s
+ d
D
/2
Half circumference of pulleys
+ d
C
/2
Thus
L = 3y
D
3t y
B
u 2s + d
D
/2 + d
C
/2
Now differentiate, taking into account that because L, t, u, s, d
D
,
and d
C
do not vary as y
D
and y
B
vary, we have L = 0 = t = u = s
= d
D
= d
C
; the result of the differentiation is, then,
0 = 3 y
D
y
B
Thus
y
B
= 3 y
D
(2)
u

2260
11.1 Virtual Work Example 16, page 4 of 4
Substituting this result in the virtual work equation, Eq. 1, gives
T y
B
+ mg y
D
= 0 (Eq. 1 repeated)
3 y
D
, by Eq. 2
Thus
( 3T + mg) y
D
= 0
Dividing through by y
D
, substituting m = 90 kg, g = 9.81 m/s
2

and solving gives
T = 294 N Ans.

6

2261
11.1 Virtual Work Example 17, page 1 of 9
D
17. Determine the equilibrium values of and for the
two-bar linkage. The couple moment M = 5 N m; each bar is
uniform and has a mass m of 5 kg; the length L = 400 mm; and
the unstretched length of the spring is 250 mm.
k = 0.2 kN/m
500 mm
2
1
A
B
L
L
C
M

2262
11.1 Virtual Work Example 17, page 2 of 9
mg
The system has two degrees of freedom because two
coordinates and must be specified to define the position
of the linkage. Consider a free-body diagram, and identify the
active forces corresponding to a small change in while
is held fixed.
1
2 Because point A does not move and is
fixed, the reactions Ax and A
y
, the weight
mg, and the spring force Fs do no work
when
1
is varied a small amount. Thus
they are not active forces.
The couple moment M and the weight
mg of the lower bar BC do work when
1

is varied, so M and mg are active forces.
3
1
2
C
M
A
B
A

y
A

x
(spring force)
F

s
mg

2263
11.1 Virtual Work Example 17, page 3 of 9
C
M
A
B
A

y
A

x
F

s
mg
mg
A
B
L
L/2
E
L/2
L/2
L/2
y

E
4
5
6
In addition to the coordinate
1
, introduce a vertical
coordinate y
E
measured downward from point A.
Compute the work done when the coordinates
are increased a positive infinitesimal amount.
dU = 0: M
1
+ mg y
E
= 0 (1)
E
Relate the differential y
E
to the angle change,
1
, by
writing
y
E
= L cos + (L/2) cos
1
and then differentiating with respect to
1
, while holding
fixed. That is, take the partial derivative with respect to
1

to obtain
y
E
= (L/2) sin
1

1
(2)
L/2
y
E
2
2
y
E
1

2264
11.1 Virtual Work Example 17, page 4 of 9
Substitute Eq. 2 for y
E
into the virtual-work equation:
M
1
+ mg y
E
= 0 (Eq. 1 repeated)
(L/2) sin
1

1
, by Eq. 2
Thus
[M (mgL/2) sin
1
]
1
= 0
Since
1
0, it follows that
M (mgL/2) sin
1
= 0 (4)
Substituting the following values into Eq. 4
M = 5 N m = 5 000 N mm
L = 400 mm
m = 5 kg
g = 9.81 kg m/s
2
and solving gives
1
= 30.6 Ans.
7

2265
11.1 Virtual Work Example 17, page 5 of 9
Next identify the active forces corresponding to
a small change in
2
while
1
is held fixed.
8
10
9 Because point A does not
move, Ax and A
y
do no work
and thus are not active forces.
Because
1
is fixed, link BC
does not rotate. That is, the
dashed line representing the new
position of link BC is parallel
to BC. Hence couple moment
M does no work, and M is not
an active force.
11 Active forces
1
2
C
M
A
B
A

y
A

x
F

s
mg
mg

2266
11.1 Virtual Work Example 17, page 6 of 9
C
A
B
A

y
A

x
F

s
mg
mg
x

B
x

B
L/2
L/2
y

F
E
F
y

E
y

F
y

E
12 Introduce coordinates measured from a fixed point to
the point of application of the active forces.
13 Compute the work done.
U = 0: mg y
E
+ mg y
F
+ Fs
x
B
= 0 (5)
1
2

2267
11.1 Virtual Work Example 17, page 7 of 9
C
A
B
E
F
L/2
L/2
L/2
L/2
y

E
x

B
Relate the differentials y
E
, y
F
, and y
F
to the change in angle
x
B
= L sin
y
F
= (L/2) cos
y
E
= L cos (L/2) cos
Differentiating each equation with respect to with held fixed,
gives
x
B
= L cos (6)
y
F
= (L/2) sin (7)
y
E
= L sin (8)
14
y
F
1
2

2268
11.1 Virtual Work Example 17, page 8 of 9
The force Fs in the spring can be related to
2
:
Fs = k extension of spring
= k (current length unstretched length)
= k [(500 mm L sin
2
) 250 mm]
= k(250 L sin
2
) (10)
Substituting Eqs. 6-8 for the differentials into the virtual-work
equation, Eq. 5, gives
L sin by Eq. 8
mg y
F
+ mg y
E
+ Fs
x
B
= 0 (Eq. 5 repeated)
L/2) sin by Eq. 7 L cos by Eq. 6
Thus
[ 3mg/2) sin + Fs cos ] L = 0
Since L 0, it follows that
3mg/2) sin + Fs cos = 0 (9)
15
16
A
B
L
C
D
500 mm
500 mm L sin
2
1
2

2269
11.1 Virtual Work Example 17, page 9 of 9
Substituting for Fs from Eq. 10 into the virtual-work equation, Eq. 9, gives
3mg/2) sin + Fs cos = 0 (Eq. 9 repeated)
k(250 L sin
2
), by Eq. 10
or,
(3mg/2) sin + k(250 L sin ) cos = 0 (11)
Substituting the following values into Eq. 11
L = 400 mm
m = 5 kg
k = 0.2 kN/m = 0.2 N/mm
g = 9.81 kg m/s
2
and solving numerically gives
= 18.50 Ans.
17

2270


11.2 Potential Energy
2271
11.2 Potential Energy Procedures and Strategies, page 1 of 2
k
L/2
L/2
s
y
W
B
C
s = (L/2) sin

A
A
L/2
L/2
B
C
y = L sin
Procedures and Strategies for Solving Problems Involving
Potential Energy
1. Make a sketch that shows the system in an arbitrary position.
2. Express the gravitational potential energy of each weight W
i
in the
system as W
i
y
i
, where y
i
is the vertical distance measured from a
fixed reference point to the weight.
3. Express the elastic potential energy of each spring in the system
with spring constant k
i
as (1/2)k
i
s
i
2
, where s
i
is the change in length
of the spring.
4. Add the gravitational and elastic potential energies to obtain the
total system potential energy, V.
4. Express V as a function of a single variable, q, by using geometry to
express all distances s
i
and y
i
in terms of q.
5. Obtain the equilibrium equation of the system by setting the
derivative of V with respect to q to zero. Solve this equation for
the value of q or, if q is given, for the value of one of the other
parameters of the problem that you were asked to determine, such
as a weight or spring constant.
6. Determine the stability of the equilibrium position by calculating
the second derivative of V. If the second derivative is positive,
then the equilibrium is stable; if the second derivative is negative,
the equilibrium is unstable; and if derivatives of all orders equal
zero, the equilibrium is neutral.

2272
11.2 Potential Energy Procedures and Strategies, page 2 of 2
q
1
=
1
q
2
=
2
1
2
q q
2
V
2
V
q
2
q
2
2
V
q
2
q
2
2
V
2
V
2
Notes:
a) The method of potential energy cannot be applied to solve
problems involving friction because friction forces are
non-conservative, that is, the work done by a friction force
depends on the path that the force follows when the system
configuration is changed.
b) The above procedure applies to one degree-of-freedom
systems, that is, to systems the position of which can be
described by a single coordinate, q. For systems with more
than a single degree of freedom, multiple coordinates q
i
must
be defined. Equilibrium equations can then be derived by
setting to zero the first derivative of V with respect to each q
i
.
Determining the stability of systems with multiple degrees of
freedom is more complicated than in the single
degree-of-freedom case. For example, an equilibrium
configuration for a system with two degrees of freedom, q
1

and q
2
, will be stable provided that
< 0

and
> 0 or > 0

2273
11.2 Potential Energy Problem Statement for Example 1
W
A
B
2W
C
L
1. The rigid rod ABC may rotate
freely about the support at B.
Weights of magnitude W and 2W
are attached at ends A and C.
Determine the possible equilibrium
positions of the rod and investigate
their stability.
L

2274
11.2 Potential Energy Problem Statement for Example 2
R
O
Vertex
Parabola
L
Semicircle
d
2. Determine if the composite solid is
in a stable equilibrium position. The
solid is of uniform density .

2275
11.2 Potential Energy Problem Statement for Example 3
L
B
m
A
3. Determine the mass m of the disk required for
equilibrium when = 50, and investigate the stability of
this equilibrium. The spring is unstretched when = 0,
and the masses of the rod AB and the roller C are
negligible.
L = 1.5 m
k = 200 N/m
C

2276
11.2 Potential Energy Problem Statement for Example 4
k
B A
C L L
m
4. The disk of mass m is supported by the two rods AC and
BC. Determine the angle for equilibrium and determine if
the equilibrium is stable. The weights of the rods are
negligible, and the unstretched length of the spring is L.
L = 0.7 m
k = 200 N/m
m = 10 kg

2277
11.2 Potential Energy Problem Statement for Example 5
k
A
Mass, m
R
5. Determine the spring constant k required for
equilibrium when = 75, and investigate the
stability of this equilibrium. The rod is uniform and
has mass m. The spring wraps around the cylinder
and is unstretched when = 0.
m = 10 kg
L = 0.5 m
R = 0.3 m
L

2278
11.2 Potential Energy Problem Statement for Example 6
k
A
R
B
O
m
C
6. Determine the value of for equilibrium, and investigate the
stability of this equilibrium. The block of mass m is connected
by a cord to a small collar at B, which slides freely on the
circular ring. The unstretched length of the spring is R.
R = 0.8 m
k = 60 N/m
m = 5 kg

2279
11.2 Potential Energy Problem Statement for Example 7
A
B
L
Mass m
k
k
C
7. The uniform bar of mass m is connected to two small
rollers at ends B and C. Spring AB, which is in tension,
has an unstretched length of L/4, and spring AC, which is
in compression, has an uncompressed length of L/4.
Determine the value of for equilibrium and investigate
the stability of the equilibrium.
L = 0.4 m
k = 800 N/m
m = 40 kg

2280
11.2 Potential Energy Problem Statement for Example 8
C
k
A
D
L
B
8. Light rods AB and BC support a block of weight W at
C. End D of the spring is attached to a light collar that
moves freely on the vertical rod. Assume that the spring is
always horizontal. The spring is unstretched when = 0.
Determine the angle for equilibrium and also determine
if the equilibrium is stable.
L = 9 in.
k = 100 lb/in.
W = 200 lb
L

2281
11.2 Potential Energy Problem Statement for Example 9
A
B
C
k
Weight, W
L
L
9. Determine the spring constant k required to keep the
uniform rod AB in equilibrium at an angle of = 20. Also
determine the stability of the equilibrium. The spring is
unstretched when = 0. Neglect the size of the pulley at C.
L = 30 in.
W = 200 lb

2282
11.2 Potential Energy Problem Statement for Example 10
T
H
A
N
K Y
O
U
A
B
C
10. The waste bin has a center of gravity at C and weight W.
Determine the steepest slope upon which the bin can be placed
without tipping. Also determine the stability of this
equilibrium position.
a = 25 in.
b = 10 in.
W = 60 lb
b
b
a

2283
11.2 Potential Energy Problem Statement for Example 11
A
B
D
4R
C
R
2R
m
k
11. The outer rim of stepped pulley A is connected by a
belt to pulley B. A spring is wrapped around the inner
rim and the cord supporting mass m is attached at point
C on the large pulley. Determine the equilibrium
position of the system and investigate its stability. The
spring is unstretched when the pulleys are in the position
shown.
R = 0.2 m
m = 2 kg
k = 900 N/m

2284
11.2 Potential Energy Problem Statement for Example 12
D
k
E
B
A
C
L
L/2
L/2
12. Determine if rods AB and BC are in stable equilibrium in the
position shown. The rods are uniform and have mass m. Rod BD is
pinned to a light slider at C that can move freely up and down between
the vertical walls. The spring is unstretched when rod AB is vertical.
One end of the spring is attached to a light collar at E that moves freely
on the rod. Assume that the spring remains horizontal for all positions
of the rods.

2285
11.2 Potential Energy Problem Statement for Example 13
P
C
B
A
E
k
k
D
L
L
13. Determine the range of values of the force P for which the
vertical equilibrium position = 0 =
2
is stable. The springs
are connected to collars D and E, which can slide freely on the
vertical rod. The springs are unstretched when = 0 =
2
.
Assume that the springs remain horizontal for all positions of
the rods and the weights of the rods and the frictionless collars
are negligible.
1

2286
11.2 Potential Energy Example 1, page 1 of 3
W
A
B
2W
C
L
2W
B
W
L
L
1. The rigid rod ABC may rotate
freely about the support at B.
Weights of magnitude W and 2W
are attached at ends A and C.
Determine the possible equilibrium
positions of the rod and investigate
their stability.
Let point B be the datum for gravitational
potential energy, V. Thus
V = WL cos 2WL cos
= WL cos (1)
The equilibrium position can be found from
the equation
= 0
Substituting for V from Eq. 1 and
differentiating gives
= WL sin = 0 (2)
The equilibrium positions of the rod are the
roots of Eq. 2:
1
= 0 and
2
= 180 Ans.
1
L cos
L cos
L
dV
d
d
dV

2287
11.2 Potential Energy Example 1, page 2 of 3
To investigate stability, evaluate d
2
V/d
2
for each root. Using the
expression for dV/d in Eq. 2, we have
=
= (WL sin
WL cos (3)
Substituting =
1
= 0 in Eq. 3 gives
= WL cos 0 = WL > 0
Since d
2
V/d
2
is positive, the equilibrium position
1
= 0 is stable.
Substituting =
2
= 180 in Eq. 3 gives
= WL cos 180 = WL < 0
Since d
2
V/d
2
is negative, the equilibrium position
2
= 180 is
unstable.
2
d
d
2
V dV
d d
d
d
d
d
d
2
V
d
2
V
d

2288
11.2 Potential Energy Example 1, page 3 of 3
A plot of of the potential energy shows the
stable and unstable equilibrium points.
3
V wL cos
-wL
wL
180
90
0
> 0 (stable)
= 0 (equilibrium)
= 0 (equilibrium)
< 0 (unstable)
Comment: The equations and the plot of V
confirm what you would expect; the system is
stable when the light weight is on top and
unstable when the heavy weight is on top.
4
W
2W
180
0
Unstable Stable
2W
W
d
dV
d
dV
d
2
V
d
d
2
V
d

2289
11.2 Potential Energy Example 2, page 1 of 6
R
O
Vertex
Parabola
L
Semicircle
d
Center of gravity
of semicircular
cylinder
Center of gravity of
parabolic cylinder
O
1
2. Determine if the composite solid is
in a stable equilibrium position. The
solid is of uniform density .
Let the point O be the datum
for gravitational potential
energy, and rotate the body
by a small angle from the
(vertical) equilibrium
position.

2290
11.2 Potential Energy Example 2, page 2 of 6
R
O
L
3L
5
L
5
3L 2L
5
4R
3
5
4
3
2 A table of properties of planar regions gives the
information shown below.
The potential energy can be expressed in terms of :
V = m
p
g( cos ) msg( cos ) (1)
6
Mass of parabolic
part of solid
Mass of
semicircular
part of solid
Negative because mass
center is below datum
(point O)
3
4R
5
2L
Semicircular region
Area =
a
2
2
Parabolic region
Area
C is the centroid of
the region.
3
4ab
4a
3
x
y
C
a
y
a
C
5
3a
x
b

2291
11.2 Potential Energy Example 2, page 3 of 6
d
L
R
7
8
9
Mass of parabolic part of solid:
m
p
= ( )d (2)
Mass of semicircular part of solid:
ms = ( )d (3)
The potential energy can now be written as
V = m
p
g( cos )
msg( cos ) (Eq. 1 repeated)
Thus V becomes, after collecting terms and
simplifying,
V= (4L
2
5R
2
) cos (4)
3
4LR
R
2
2
4R
3
5
2L
4LR
3
( )d, by Eq. 2
( )d, by Eq. 3
2
R
2
15
2 gRd
b
x
3a
5
C
a
y
a
C
y
x
3
4a
4ab
3
C is the centroid of
the region.
Parabolic region
Area
2
a
2
Semicircular region
Area =

2292
11.2 Potential Energy Example 2, page 4 of 6
10 The equilibrium position can be found from
= 0
or,
= [ (4L
2
5R
2
) cos ] = 0
= (4L
2
5R
2
)( sin = 0 (5)
The root of this equation is = 0. To investigate stability, evaluate
d
2
V/d
2
for = 0. Using the expression for dV/d in Eq. 5 gives
= = (4L
2
5R
2
)( cos (6)
If = 0, Eq. 6 becomes
= (4L
2
5R
2
)( 1 (7)
dV
d
d
dV
d
d
15
2 gRd
15
2 gRd
15
2 gRd
15
2 gRd
d
2
V
d
d
d d
dV
d
d
2
V

2293
11.2 Potential Energy Example 2, page 5 of 6
Eq. 7 shows that d
2
V/d
2
will be positive (and thus the = 0
equilibrium position will be stable) if 4L
2
5R
2
< 0, which implies
L < 5R/2
What if 4L
2
5R
2
= 0? Then
V = (4L
2
5R
2
)( cos
0

= 0
for all values of . Furthermore, all derivatives of V equal zero.
Thus if 4L
2
5R
2
= 0, the composite solid is in neutral equilibrium
for all values of .
11
2 gRd
15

2294
11.2 Potential Energy Example 2, page 6 of 6
R
R
L
12 These results can be summarized as follows:
14
R
2. If L > 5R/2, then the only
equilibrium position in the range
90 < < 90 is = 0 and this
position is unstable.
13
3. If L = 5R/2, then equilibrium is
possible for all values of in the
range 90 < < 90 and all of
these equilbrium positions are
neutral. Ans.
L
L
1. If L < 5R/2, then the only
equilibrium position in the range
90 < < 90 is = 0 and this
position is stable.
Ans.
Ans.

2295
11.2 Potential Energy Example 3, page 1 of 3
L
B
m
A
3. Determine the mass m of the disk required for
equilibrium when = 50, and investigate the stability of
this equilibrium. The spring is unstretched when = 0,
and the masses of the rod AB and the roller C are
negligible.
L = 1.5 m
k = 200 N/m
Let point A be the datum for the gravitational
potential energy, V
g
. Thus
V
g
= mg(L cos ) (1)
A
m
B
L
1
L cos
C

2296
11.2 Potential Energy Example 3, page 2 of 3
A
m
B
L
L sin
The change in length of the spring is L sin , so the elastic potential energy is
Ve = (1/2)k(L sin )
2
(2)
The potential energy of the system is, then,
mgL cos by Eq. 1
V = Ve + V
g
(1/2)k(L sin )
2
by Eq. 2
2
d
d
dV
d
The equilibrium position can be found from
= 0
or,
= [(1/2)k(L sin )
2
+ mgL cos ] = 0
or,
= kL
2
sin cos mgL sin = 0 (3)
Substituting = 50, g = 9.81 m/s
2
, k = 200 N/m, and L = 1.5 m and solving for m gives
m = 19.66 kg Ans.
d
dV
dV
d

2297
11.2 Potential Energy Example 3, page 3 of 3
3 To investigate stability of the equilibrium, evaluate d
2
V/d
2
for
= 50. Using the expression for dV/d in Eq. 3 gives
=
= [kL
2
sin cos mgL sin
= kL
2
(cos
2
sin
2
) mgL cos
Substituting m = 19.66 kg, = 50, g = 9.81 m/s
2
, k = 200 N/m,
and L = 1.5 m gives
= 264 N m
Since d
2
V/d
2
is negative, the equilibrium position is unstable:
the bar could not remain inclined at an angle of = 50 in a real
(that is, not mathematically idealized) system.
d
2
V
d
d
d
d
dV
d
d
d
d
2
V

2298
11.2 Potential Energy Example 4, page 1 of 3
k
B A
C L L
m
4. The disk of mass m is supported by the two rods AC and
BC. Determine the angle for equilibrium and determine if
the equilibrium is stable. The weights of the rods are
negligible, and the unstretched length of the spring is L.
L = 0.7 m
k = 200 N/m
m = 10 kg
1 Let point A be the datum for the gravitational potential
energy, V
g
. Thus
V
g
= mgL sin (1)
The elastic potential energy is
Ve = (1/2)k(current length unstretched length)
2
= (1/2)k(2L cos L)
2

= (1/2)kL
2
(2 cos 1)
2
(2)
Mass, m
L L C
A
B
k
Angle equals
because triangle
ABC is isosceles.
L sin
2L cos

2299
11.2 Potential Energy Example 4, page 2 of 3
The potential energy of the system is, then,
mgL sin by Eq. 1
V = Ve + V
g
(3)
(1/2)kL
2
(2 cos )
2
, by Eq. 2
The equilibrium position can be found from
= 0
or,
= [(1/2)kL
2
(2 cos )
2
+ mgL sin ] = 0
or,
= kL
2
(2 cos )( 2 sin )+ mgL cos = 0 (4)
Substituting m = 10 kg, g = 9.81 m/s
2
, k = 200 N/m and L = 0.7 m, and then solving
Eq. 4 numerically gives two roots:
= 22.43 and = 49.48 Ans.
2
d
dV
dV
d
dV
d
d
d

2300
11.2 Potential Energy Example 4, page 3 of 3
To investigate stability, evaluate d
2
V/d
2
. Using the
expression for dV/d in Eq. 4 gives
=
= [kL
2
(2 cos )( 2 sin )+ mgL cos ]
= kL
2
[( 2 sin )( 2 sin ) + (2 cos )
( 2 cos )] mgL sin
Substituting = 22.43, m = 10 kg, g = 9.81 m/s
2
, k = 200
N/m and L = 0.7 m into the above equation gives
= 123 N m
Because d
2
V/d
2
is negative, the equilibrium is unstable.
Substituting = 49.48into Eq. 5 gives
= 136 N m
Because d
2
V/d
2
is positive, the equilibrium is stable.
3
d
d
2
V
d
d
2
V
dV
d d
d
d
d
d
d
2
V
> 0 (stable)
0 22.43
90
V( )
4 A plot of of the potential energy shows the stable and
unstable equilibrium points. From Eq. 3, the potential
energy is
V( ) = (1/2)kL
2
(2 cos )
2
+ mgL sin
d
2
V
d
120 m N
40 m N
80 m N
49.48
= 0 (equilibrium)
d
dV
< 0 (unstable)
d
2
V
d
Observation: If you have difficulty finding
multiple roots of the equilibrium equation
with your calculator, plotting V( ) will give
you the approximate location of the roots,
which you can then use as initial estimates
in your calculator's solver.
5

2301
11.2 Potential Energy Example 5, page 1 of 3
Let point A be the datum for the gravitational
potential energy Ve. Thus
Ve = mg(L/2) cos (1)
k
A
Mass, m
R
5. Determine the spring constant k required for
equilibrium when = 75, and investigate the
stability of this equilibrium. The rod is uniform and
has mass m. The spring wraps around the cylinder
and is unstretched when = 0.
m = 10 kg
L = 0.5 m
R = 0.3 m
L
Center of mass
A
L
2
2
L
cos
1

2302
11.2 Potential Energy Example 5, page 2 of 3
dV
d
dV
d
dV
d
d
d
k
A
R
2 The change in length of the spring equals the
arc length, R expressed in radians), so
the elastic potential energy Ve is
Ve = (1/2)k(R )
2
(2)
Arc length = R
The potential energy of the system is
mg(L/2) cos by Eq. 1
V = Ve + V
g
(1/2)k(R )
2
by Eq. 2
The equilibrium position can be found from
= 0
or,
= [(1/2)k(R )
2
+ mg(L/2) cos ] = 0
or,
= kR
2
mg(L/2) sin = 0 (3)
Substituting = (75/180) rad, R = 0.3 m, g = 9.81 m/s
2
,
m = 10 kg, and L = 0.5 m, and then solving gives
k = 201.1 N/m Ans.
3

2303
11.2 Potential Energy Example 5, page 3 of 3
To investigate stability, evaluate dV
2
/d
2
. Using the
expression for dV/d in Eq. 3 gives
=
= [kR
2
mg(L/2) sin ]
= kR
2
mg(L/2) cos
Substituting k = 201.1 N/m, = (75/180) rad, m = 10 kg, g
= 9.81 m/s
2
, and L = 0.5 m, R = 0.3 m gives
= 11.8 N m
Because d
2
V/d
2
is positive, the equilibrium position = 75
is stable.
4
d
2
V
d
d
d d
dV
d
d
2
V
d
d

2304
11.2 Potential Energy Example 6, page 1 of 5
k
A
R
B
O
m
C
6. Determine the value of for equilibrium, and investigate the
stability of this equilibrium. The block of mass m is connected
by a cord to a small collar at B, which slides freely on the
circular ring. The unstretched length of the spring is R.
R = 0.8 m
k = 60 N/m
m = 5 kg
Let point O be the datum for the
gravitational potential energy Ve.
Thus
V
g
= mg(L y) (1)
where y is the vertical distance
from O to the collar at B, and L is
the length of the cord BC plus half
the height of the block. The value
of L is unknown, but this won't
matter because we will
differentiate V
g
later, and the
derivative of L will be zero.
Another way of saying the same
thing is to say that the magnitude
of V
g
doesn't matter; only the
change in V
g
matters.
1
C
m
O
B
A
L
L y
y

2305
11.2 Potential Energy Example 6, page 2 of 5
R
A
R
B
2 The coordinates and y are not independent
but can be related through geometry. First
note that the angles at A and B are equal
because triangle AOB is isosceles.
O
y
The sum of the angles of triangle AOB must
equal 180, that is, + 2 = 180, so =
180 2 and thus
y = R sin (180 )
= R sin [180 (180 2 )]
= R sin 2 (2)
5
3 The vertical distance
between O and B is
y = R sin
4 The angle can expressed as
180 so
y = R sin (180 )
The gravitational potential energy becomes, from Eq. 1,
V
g
= mg(L y) (Eq. 1 repeated)
R sin 2 by Eq. 2
or,
V
g
= mgR sin 2 mgL (3)
6

2306
11.2 Potential Energy Example 6, page 3 of 5
R
O
A
R
B
s
The elastic potential energy of the spring is
Ve = (1/2)k(current length unstretched length)
= (1/2)k(s R)
2
(4)
where s is the length of the spring (distance AB) and R is
known to be equal to the unstreteched length.
From geometry,
s = 2R cos
Thus Eq. 4 becomes
Ve = (1/2)k(s R)
2
2R cos
or,
Ve = (1/2)kR
2
(2 cos 1)
2
(5)
The potential energy of the system is, then,
mgR sin 2 mgL, by Eq. 3
V = Ve + V
g
(1/2)kR
2
(2 cos 1)
2
, by Eq. 5
7

2307
11.2 Potential Energy Example 6, page 4 of 5
8 The equilibrium position can be found from
= 0
or,
= [(1/2)kR
2
(2 cos 1)
2
+ mgR sin 2 mgL] = 0
or,
= kR
2
(2 cos 1)( 2 sin ) + 2mgR cos 2 = 0 (6)
Substituting k = 60 N/m, R = 0.8 m, g = 9.81 m/s
2
, and m = 5 kg and solving numerically gives
= 34.47 Ans.
To investigate stability, evaluate dV
2
/d
2
. Using the expression for dV/d in Eq. 6 gives
=
= [kR
2
(2 cos 1)( 2 sin ) + 2mgR cos 2
= kR
2
[( 2 sin )( 2 sin ) + (2 cos 1)( 2 cos )] 4mgR sin 2 (7)
dV
d
d
d
dV
d
d
dV
d
d
2
V dV
d d
d
d
d

2308
11.2 Potential Energy Example 6, page 5 of 5
9 Substituting k = 60 N/m, R = 0.8 m, =
34.47, m = 5 kg, and g = 9.81 m/s
2
in Eq. 7
gives
= 138 N m
Because d
2
V/d
2
is negative, the equilibrium
position = 34.47 is unstable.
d
2
V
d

2309
11.2 Potential Energy Example 7, page 1 of 3
B
A
L/2
C
C
7. The uniform bar of mass m is connected to two small
rollers at ends B and C. Spring AB, which is in tension,
has an unstretched length of L/4, and spring AC, which is
in compression, has an uncompressed length of L/4.
Determine the value of for equilibrium and investigate
the stability of the equilibrium.
L = 0.4 m
k = 800 N/m
m = 40 kg
Mass center
1 Let point A be the datum for the gravitational
potential energy V
g
. Thus
V
g
= mg(L/2) cos (1)
A
B
L
Mass m
k
(L/2) cos
k

2310
11.2 Potential Energy Example 7, page 2 of 3
L
B
A
L cos
L sin
C
2 The elastic potential energy is
Ve = (1/2)k(L cos L/4)
2
+ (1/2)k(L/4 L sin )
2
= (1/2)kL
2
[(cos 1/4)
2
+ (1/4 sin )
2
] (2)
The potential energy of the system is
(1/2)mgL cos by Eq. 1
V = Ve + V
g
(1/2)kL
2
[(cos 1/4)
2
+ (1/4 sin )
2
] by Eq. 2
The equilibrium position can be found from
= 0
or,
= {(1/2)kL
2
[(cos 1/4)
2
+ (1/4 sin )
2
] (1/2)mgL cos = 0
or, after differentiating and simplifying,
= (L/4)[(kL + 2mg) sin kL cos ] = 0 (3)
Substituting k = 800 N/m, L = 0.4 m, g = 9.81 m/s
2
, and m = 40 kg and solving gives
= 16.15 Ans.
dV
d
d
dV
d
d
d
dV

2311
11.2 Potential Energy Example 7, page 3 of 3
3
d
d
dV
d d
d
2
V
To investigate stability, evaluate dV
2
/d
2
. Using the
expression for dV/d in Eq. 3 gives
=
= (L/4)[(kL + 2mg) sin kL cos ]
= (L/4)[(kL + 2mg) cos + kL sin ]
Because all quantities on the right-hand side are positive
when = 16.15, d
2
V/d
2
is positive, and the equilibrium is
stable.
d
d

2312
11.2 Potential Energy Example 8, page 1 of 4
C
k
A
D
L
B
8. Light rods AB and BC support a block of weight W at
C. End D of the spring is attached to a light collar that
moves freely on the vertical rod. Assume that the spring is
always horizontal. The spring is unstretched when = 0.
Determine the angle for equilibrium and also determine
if the equilibrium is stable.
L = 9 in.
k = 100 lb/in.
W = 200 lb
L

2313
11.2 Potential Energy Example 8, page 2 of 4
A
D
L
B
L
d
d
d
dV
d
dV
d
dV
The potential energy of the system is
2WL cos by Eq. 2
V = Ve + V
g
(3)
(1/2)k(L sin )
2
, by Eq. 1
The equilibrium position can be found from
= 0
or,
= [(1/2)k(L sin )
2
+ 2WL cos ] = 0
or,
= (kL
2
cos 2WL) sin = 0 (4)
4
3 Let point A be the datum for the gravitational
potential energy V
g
. Thus
V
g
= W(2L cos ) (2)
The spring is unstretched when = 0.
Thus the extension of the spring is L sin
and the elastic potential energy is
Ve = (1/2)k(L sin )
2
(1)
2
1 Angle equals
because triangle
ABC is isosceles.
L sin
L cos
L cos
Weight, W
C
k

2314
11.2 Potential Energy Example 8, page 3 of 4
5 The equilibrium equation, Eq. 4, has two solutions. One
solution,
1
, can be found by requiring sin = 0, which
implies
1
= 0 Ans.
The other solution, , follows from requiring the term in
parentheses in Eq. 4 to vanish.
(kL
2
cos 2WL) sin = 0 (Eq. 4 repeated)
Thus
(kL
2
cos 2WL) = 0
Substituting L = 9 in., k = 100 lb/in., and W = 200 lb and
solving gives
= 63.62 Ans.
To investigate stability, evaluate dV
2
/d
2
. Using the
expression for dV/d in Eq. 4 gives
=
= [(kL
2
cos 2WL) sin ]
= ( kL
2
sin ) sin + (kL
2
cos
2WL) cos (6)
6
d
d
dV
d d
d
d
d
2
V
7 The stability of the equilibrium position
1
= 0 can
be determined by substituting
1
= 0 (and L = 9 in.,
k = 100 lb/in., and W = 200 lb) into Eq. 6. The
result is
= 4500 lb in
Because d
2
V/d
2
is positive, the position
1
= 0 is
stable.
d
2
V
d

2315
11.2 Potential Energy Example 8, page 4 of 4
3600
4050
90 63.62
d
d
2
V
8 Similarly evaluating d
2
V/d
2
at = 63.62 gives
= 6501 lb in.
Because d
2
V/d
2
is negative, the position
2
= 63.62 is
unstable.
A plot of V = V( ) shows the equilibrium points. From Eq. 3,
we have
V( ) = (1/2)kL
2
sin
2
+ 2WL cos (Eq. 3 repeated)
= (1/2)(100 lb/in.)(9 in.)
2
sin
2

+ 2(200 lb)(9 in.) cos
= 4050 sin
2
+ 3600 cos
0
V( )
(in. lb)
0
5000
Stable equilibrium
Unstable equilibrium

2316
11.2 Potential Energy Example 9, page 1 of 5
9. Determine the spring constant k required to keep the
uniform rod AB in equilibrium at an angle of = 20. Also
determine the stability of the equilibrium. The spring is
unstretched when = 0. Neglect the size of the pulley at C.
L = 30 in.
W = 200 lb
Let point A be the datum for the
gravitational potential energy, V
g
. Thus
V
g
= (WL/2) sin (1)
1
(L/2) sin
L/2
A
B
C
k
Weight, W
B
A
Weight, W
Center of gravity
L
L

2317
11.2 Potential Energy Example 9, page 2 of 5
2
C
B
A
k
L
L
Spring stretched
To determine the elastic potential energy, we have to determine the
change in length of the spring. But the change in length of the spring is
just the change in distance between points B and C. First let's calculate
the length of BC, say, BC
stretched
, when rod AB is rotated by an angle .
By the law of cosines,
BC
stretched
= L
2
+ L
2
2L
2
cos(90 + )
= L 2[1 cos(90 + )] (2)
The formula for the cosine of a sum gives
cos (90 + ) = cos 90 cos sin 90 sin
= sin
Thus Eq. 2 becomes
BC
stretched
= L 2(1 + sin ) (3)
90

2318
11.2 Potential Energy Example 9, page 3 of 5
3
L
k
90
L
C
B
A
4
Spring unstretched
BC
unstretched
= L
2
+ L
2
L
Next calculate the length BC when the spring is unstretched.
Now the change in length of the spring can be computed:
BC
stretched
BC
unstretched
= L 2(1 + sin ) L 2
= L 2 ( 1 + sin 1) (4)
The elastic potential energy is then
Ve = (1/2)k[L 2 ( 1 + sin 1)]
2
= kL
2
( 1 + sin 1)
2
(5)

2319
11.2 Potential Energy Example 9, page 4 of 5
5
d
dV
dV
d
d
dV
d
d
The potential energy of the system is, then,
W(L/2) sin by Eq. 1
V = Ve + V
g
kL
2
( 1 + sin 1)
2
by Eq. 5
The equilibrium position can be found from
= 0
or,
= [kL
2
( 1 + sin 1)
2
W(L/2) sin ] = 0
or, after differentiating and simplifying,
= (L cos )(kL ) = 0 (6)
Substituting = 20, W = 200 lb, and L = 30 in. and solving for k gives
k = 24.37 lb/in. Ans.
kL
1 + sin
2
W

2320
11.2 Potential Energy Example 9, page 5 of 5
6
d
d
d
2
V
d
d
d
d
d
2
V
To investigate stability, evaluate dV
2
/d
2
. Using the expression for dV/d
in Eq. 6 gives
=
= [(L cos kL )]
Differentiating gives
= ( L sin )(kL ) + (L cos )[
Substituting k = 24.37 lb/in. (and = 20, L = 30. in., and W = 200 lb)
gives
= 6229 in. lb
Because d
2
V/d
2
is positive, the equilibrium position = 20 is stable.
dV
d
2(1 + sin )
3

2

kL cos
/
d
d
2
V
W
2
1 + sin
kL
kL
1 + sin
2
W

2321
11.2 Potential Energy Example 10, page 1 of 4
T
H
A
N
K Y
O
U
A
B
C
10. The waste bin has a center of gravity at C and weight W.
Determine the steepest slope upon which the bin can be placed
without tipping. Also determine the stability of this
equilibrium position.
a = 25 in.
b = 10 in.
W = 60 lb
b
b
a

2322
11.2 Potential Energy Example 10, page 2 of 4
C
A
B
W
F
B
A
a
b
C
1 Because we are looking for the steepest
slope upon which the bin can be placed
without tipping, we know that the force F
from the sloping surface acting on the
bottom of the bin is concentrated at the
corner B the bin is just about to tip.
The potential energy V of the system is entirely
gravitational, since there is no elastic potential energy.
We want to write an expression for the gravitational
potential energy of the forces shown, as a function of .
Because point B does not move as changes, B can be
used as the datum. Thus
V W (vertical distance between B and C)
W(a cos + b sin ) (1)
2
a
b
a cos
b sin

2323
11.2 Potential Energy Example 10, page 3 of 4
3
d
d
dV
d
The equilibrium position can be found from
= 0
or,
= [W(b sin + a cos ] = 0
or,
= W(b cos a sin ) = 0 (2)
Substituting b = 10 in., and a = 25 in. into this
equation and solving gives
= 21.8 Ans.
d
dV
dV
d
d
d
dV
d
d
d
d
d
2
V
To investigate stability of the equilibrium, evaluate d
2
V/d
2

for = 21.8. Using the expression for dV/d in Eq. 2 gives
=
= [W(b cos a sin )
= W( b sin a cos )]
Because both terms are negative when = 21.8, d
2
V/d
2
is
negative, and the equilibrium position is unstable.
Comment No. 1: We have implicitly assumed that the corner
of the bin at B can develop enough friction to prevent the bin
from sliding down the slope. If sliding rather than tipping
does occur, then the friction force does work, the
potential-energy function does not exist, and the
potential-energy criterion cannot be used to find the
equilibrium state.
4

2324
11.2 Potential Energy Example 10, page 4 of 4
V( )
W
(in.)
0
100
25
Equilibrium state (unstable)
C
W
W
C
F F
C
W
F
Comment No. 2: Consider a plot of V/W = b sin + a cos = 10 sin + 25 cos
0 < < 21.8
The bin is not tipped far
enough to be balanced. The
bin is not in equilibrium.
= 21.8
The bin is tipped to a position where the line of action
of the weight passes through the bottom right corner of
the bin. The bin is in (unstable) equilibrium.
21.8 90
The bin is tipped too far to
be balanced. The bin is not
in equilibrium.
= 21.8
5 6
7
21.8

2325
11.2 Potential Energy Example 11, page 1 of 4
A
B
D
4R
C
R
2R
m
k
11. The outer rim of stepped pulley A is connected by a
belt to pulley B. A spring is wrapped around the inner
rim and the cord supporting mass m is attached at point
C on the large pulley. Determine the equilibrium
position of the system and investigate its stability. The
spring is unstretched when the pulleys are in the position
shown.
R = 0.2 m
m = 2 kg
k = 900 N/m

2326
11.2 Potential Energy Example 11, page 2 of 4
k
m
C
4R
D
B
A
R
2
The elastic potential energy is
Ve = (1/2)k(extension of spring)
2
(2)
Arc length = R
( measured in radians)
A
B
D
4R
C
m
k
4R sin
1 Let point B be the datum for gravitational potential
energy, V
g
. Thus
V
g
= mg(4R sin + L) (1)
The length L is unknown but will drop out of the
equations later when the expression for V
g
is
differentiated.
L
The potential energy of the system is
mg(4R sin + L) by Eq. 1
V = Ve + V
g
(3)
(1/2)kR
2 2
by Eq. 2
3

2327
11.2 Potential Energy Example 11, page 3 of 4
4 Angles and are not independent but can be related by noting that the
belt is inextensible, so that the arc lengths must be equal:
4R = 2R
Thus
= /2
Using this result in Eq. 3 gives
V = (1/2)kR
2 2
mg(4R sin + L) (Eq. 3 repeated)
/2
The equilibrium position can be found from
= 0
or,
= [(1/2)kR
2 2
mg(4R sin ( /2) + L)] = 0
or,
= kR
2
mgR cos ( /2) = 0 (4)
(We have used the fact that dL/d = 0 because L does not vary with .
2R 4R
B
A
d
d
d
dV
d
dV
dV
d

2328
11.2 Potential Energy Example 11, page 4 of 4
5
d
2
V
d
d
dV
Substituting R = 0.2 m, m = 2 kg, k = 900 N/m and g = 9.81 m/s
2
into Eq. 4
and solving numerically gives
= 0.2167 rad = 12.416 Ans.
To investigate stability, evaluate dV
2
/d
2
. Using the expression for dV/d
in Eq. 4 gives
=

or,
= (kR
2
2mgR cos ( /2)
or,
= kR
2
+ mgR sin ( /2)
Because both terms in this equation are positive for = 12.4, d
2
V/d
2
is
positive, and the equilibrium position = 12.4 is stable.
d
2
V
d
d
d
d
d
2
V
d
d

2329
11.2 Potential Energy Example 12, page 1 of 4
D
k
E
B
A
C
L
L/2
L/2
12. Determine if rods AB and BC are in stable equilibrium in the
position shown. The rods are uniform and have mass m. Rod BD is
pinned to a light slider at C that can move freely up and down between
the vertical walls. The spring is unstretched when rod AB is vertical.
One end of the spring is attached to a light collar at E that moves freely
on the rod. Assume that the spring remains horizontal for all positions
of the rods.

2330
11.2 Potential Energy Example 12, page 2 of 4
Center of
mass
C
A
B
E
k
D
(L/2) cos
(L/2) cos
1 Let point A be the datum for gravitational potential energy, V
g
. Thus
V
g
= mg(L/2) cos + mg[L cos + (L/2) cos ]

The elastic potential energy is
Ve = (1/2)k(extension of spring)
2
= (1/2)k(L sin )
2
(2)
The potential energy of the system is
mg(L/2) cos + mg[L cos + (L/2) cos ] by Eq. 1
V = V
g
+ Ve (3)
(1/2)k(L sin )
2
by Eq. 2
2
3
(L/2) cos
L sin

2331
11.2 Potential Energy Example 12, page 3 of 4
C
A
B
E
k
D
(L/2) sin
4 The angles and are not independent but can be related by noting that
L sin = (L/2) sin 4
If we now consider only the case where rods AB and BD are nearly vertical, then
both and are small and sin and sin Substituting these
approximations into Eq. 4 gives, after cancelling L and multiplying by 2,
= 2
Using this result in Eq. 3 gives
2
V = (1/2)kL
2
sin
2
+ (1/2)mgL(3 cos + cos (Eq. 3 repeated)
The equilibrium position can be found from
= 0
or,
= [(1/2)kL
2
sin
2
+ (1/2)mgL(3 cos + cos 2 ] = 0
or,
= kL
2
sin cos (1/2)mgL(3 sin + 2 sin 2 = 0 (5)
Note that = 0 (both rods are vertical) satisfies the equilibrium equation, Eq. 5.
L sin
d
dV
dV
d
d
dV
d
d

2332
11.2 Potential Energy Example 12, page 4 of 4
d
d
dV
d
d
d
d
d
2
V
To investigate stability of the equilibrium, evaluate d
2
V/d
2
for = 0. Using the
expression for dV/d in Eq. 5 gives
=
= [ kL
2
sin cos (1/2)mgL(3 sin + 2 sin 2


= kL
2
(cos
2
sin
2
) (1/2)mgL(3 cos + 4 cos 2
Substituting = 0 gives, after simplifying,
= (L/2)(2kL 7mg)
The equilibrium will be stable if d
2
V/d
2
is positive, that is, if
2kL 7mg > 0
or,
> (6)
Comment: The result expressed in Eq. 6 makes intuitive sense: The rods will be
stable in a vertical position if the spring is stiff (large k in the numerator of Eq.6)
and the mass is small (small m in the denominator of Eq. 6).
5
d
2
V
d
kL 7
mg
2

2333
11.2 Potential Energy Example 13, page 1 of 5
P
C
B
A
E
k
k
D
L
L
13. Determine the range of values of the force P for which the
vertical equilibrium position = 0 =
2
is stable. The springs
are connected to collars D and E, which can slide freely on the
vertical rod. The springs are unstretched when = 0 =
2
.
Assume that the springs remain horizontal for all positions of
the rods and the weights of the rods and the frictionless collars
are negligible.
1

2334
11.2 Potential Energy Example 13, page 2 of 5
1
L
P
C
B
A
L
Let point A be the datum for the potential
energy of the force P, V
P
. Thus
V
P
= P(L cos + L cos
2
) (1)
1 2 The elastic potential energy is
Ve = (1/2)k(L sin
1
)
2

+ (1/2)k(L sin
1
+ L sin
2
)
2
(2)
L cos
1
+ L cos
2
1
L
L
D
k
k
E
A
B
C
P
L sin
2
L sin
1
The potential energy of the system is
P(L cos
1
+ L cos
2
) by Eq. 1
V = V
g
+ V
P
(3)
(1/2)k(L sin
1
)
2
+ (1/2)k(L sin
1
+ L sin
2
)
2
by Eq. 2
3

2335
11.2 Potential Energy Example 13, page 3 of 5
The equilibrium equations are
= 0
or,
= [(1/2)k(L sin
1
)
2
+ (1/2)k(L sin
1
+ L sin
2
)
2
+ P(L cos
1
+ L cos
2
)] = 0
or,
= kL
2
[sin
1
cos
1
+ (sin
1
+ sin
2
)( cos
1
)] PL sin
1
= 0 (4)
and,
= 0
or,
= [(1/2)k(L sin
1
)
2
+ (1/2)k(L sin
1
+ L sin
2
)
2
+ P(L cos
1
+ L cos
2
)] = 0
or,
= kL
2
(sin
1
+ sin
2
) cos
2
PL sin
2
= 0 (5)
Note that the vertical position of the rods,
1
= 0 =
2
, is a solution of Eqs. 4 and 5.
4
V
V
V
V
V
V

2336
11.2 Potential Energy Example 13, page 4 of 5
The equilibrium configuration
1
= 0 =
2
will be
stable provided that
< 0 (6)
and
> 0 or > 0 (7)
Differentiating the expression for V/
1
(Eq. 4)
with respect to
1
and substituting
1
= 0 =
2
in
the result gives
= L(2kL P) (8)
Differentiating the expression for V/
1
(Eq. 4)
with respect to
2
and substituting
1
= 0 =
2
in
the result gives
= kL
2
(9)
5
2
V
2
2
V
2
2
V
2
2
V
2 2
2
V
2
V
6 Differentiating the expression for V/
2
(Eq. 5) with respect
to
2
and substituting
1
= 0 =
2
in the result gives
= L(kL P) (10)
The stability condition Eq. 6 becomes
kL
2
by Eq. 9
< 0 (Eq. 6 repeated)
L(2kL P) by Eq. 8 L(kL P) by Eq. 10
That is,
(kL
2
)
2
L(2kL P)L(kL P) < 0
or, after dividing by L
2
and re-arranging terms,
P
2
3kLP + k
2
L
2
> 0
Using the quadratic formula to factor this expression gives
(P kL)(P kL) > 0 (11)
2
3 + 5
2
3 5
2
2
V
2
V
2 2
2
V
2 2
2
V
2
V

2337
11.2 Potential Energy Example 13, page 5 of 5
The inequality given in Eq. 11 will be satisfied if both factors are positive:
(P kL) > 0, which implies P >
(P kL) > 0, which implies P >
Alternatively, the inequality given in Eq. 11 will be satisfied if both factors are negative:
(P kL) < 0, which implies P < (12)
(P kL) < 0, which implies P < (13)
The inequalities in Eq. 7 become
L(2kL P), by Eq. 8 L(kL P), by Eq. 10
> 0 or > 0 (Eq. 7 repeated)
That is,
P < 2kL or P < kL (14)
Noting that (3 5 )/2 = 0.382 < 1, we see that we can satisfy the inequalities of Eq. 14 and
Eqs. 12 and 13 simultaneously, and thus have a stable equilibrium, provided that we restrict
P/(kL) to the range
< Ans.
7
3 + 5
2
3 5
2
kL
3 + 5
2
kL
3 5
2
2
2
V
2
V
2
2
kL
3 5
2
3 5
kL
2
3 + 5
kL
3 + 5
2
3 5
2
P

2338

Appendix: Geometric Properties of Lines, Areas, and Solid Shapes
2339
Geometric Properties of Lines, Areas, and Solid Shapes page 1 of 4
h
3
2h
3
Circular arc segment
r sin
C
Length = 2 r r
x
y
C
2r
Length = r
2
r
Quarter circular segment
Centroid Location Centroid Location
x
y
y
x
r
C
4r
3
r
2
4
Area =
Quarter circular region
Centroid location
bh
2
Area =
Triangular region
Centroid location
b
C

2340
Geometric Properties of Lines, Areas, and Solid Shapes page 2 of 4
r
1
y
4(2+ sin )
3r(1+ sin )
2
Spherical cap
z
C
Volume = r
3
(2 3 sin + 3 sin
3

3
x
Centroid Location
Centroid Location
r
C
2r sin
y
x
Area = r
2
Circular sector region
y
a
C
5
3a
x
4ab
3
Centroid Location
Area =
b
Parabolic region
a
2a/5
C
Vertex
Semiparabolic region
b
Centroid Location
2ab
3
Area =
x
y

2341
Geometric Properties of Lines, Areas, and Solid Shapes page 3 of 4
I
h
h
2


12
hb
y =
3

I
12
bh
= x
3
y
C
x
Rectangle
Moment of Inertia
2
b
2
b
2
Volume = r
3

2
3
r
3
8
Hemisphere
z
y
x
C
r
Centroid Location
Cone
h
4
h
z
y
C
r
Volume =
r
2
h
3
x
Centroid Location
bh
2
Area =
Triangular
Moment of Inertia
b
C
h
3
2h
3
x
I


12
hb
BB =
3


I
36
bh
= xx
3
x
B B
Area = bh

2342
Geometric Properties of Lines, Areas, and Solid Shapes page 4 of 4


C
x
= I
r
4

3
y

c =
y

c
I =
r
4

y
8
4r
x
8
r
y
Semicircle
Moment of Inertia
Circle
I
1
=
1
4
=
4
Iy
x

Moment of Inertia
C
r
y
x
r
4
r
4
Area = r
2
r
2

2
Area =

2343
Index

A

Adding forces
parallelogram law, 24 59
rectangular components in plane, 61, 70 75, 97 112
rectangular components in space, 114, 119 122, 133 145,
148, 156 158, 182 199
Arch, Gateway, 1686, 1731 1733

B

Beams
distributed loads, 557 604
shear and bending-moment diagrams, 1536 1596,
1598 1669
Bridge, suspension, 1086, 1095 1098
Buoy, center of mass of, 1769, 1823 1831

C
Cables: concentrated loads, 1034 1081
Cables: uniform loads, 1083 1132
formula, 1085
Catenary, 1134 1190
formulas, 1134
Centroids
areas, 1672, 1674 1682, 1691 1721, 1750, 1758 1765,
1786 1811
concentrated masses, 1751 1754, 1772 1776
by integration, 1672 1748
by composite parts, 1748 1844
theorems of Pappus and Guldinus, 1846 1923
volumes, 1673, 1687 1690, 1734 1748, 1767 1770,
1810 1840
wires, 1673, 1683 1685, 1722 1730, 1755 1757,
1777 1785
Clamp
internal forces, 1490, 1515 1516
square-threaded screw, 1292, 1310 1313
wedges, 1253, 1263 1265
Clothesline, 1088, 1103 1104
Components
from parallelogram law, 32 34
rectangular components in plane, 61 112
rectangular components in space, 114 145,
148, 152 158, 167 199
Constraints, 607 614, 626 670
Couple, 402 443
Cross product
multiplication of base vectors, 354, 358, 367, 387, 394, 399
determinant, 363, 373, 376, 383, 388

D

Direction cosines, angles, 114, 119 121, 129 145, 163 171
Distributed loads on beams, 557 603
Dot products, use
angles, 201 206, 209 228
force projections, 201, 207 208, 229 238


2344
E

Equilibrium
particle in plane, 241 260
particle in space, 262 305
rigid bodies in plane, 689 730
rigid bodies in space, 732 789
Equivalent force-couple systems, 473 555

F

Flat belts, friction, 1348 1379
formula, 1348
Formulas
catenary cables, 1134
cables with uniform loads, 1087
flat belts, 1348
law of cosines, 24
law of sines, 24
magnitude of two-dimensional force vector, 61
magnitude of three-dimensional force vector, 114
Mohrs circle, 2117
moments of inertia about inclined axes, 2116
principal moments, 2116
square-threaded screws, 1287 1288
stability conditions for a two-degree-of-freedom
system, 2272
theorems of Pappus and Guldinus, 1846
thrust bearings and disks, 1381
Frames and machines 791 902
Friction
basic applications, 1193 1249
flat belts, 1348 1379
journal bearings, 1403 1449
rolling resistance, 1451 1480
square-threaded screws, 1287 1346
thrust bearings and disks, 1381 1401
wedges, 1251 1285

G

Geometric properties
centroids, 2339 2341
moments of inertia, 2341 2342

H

Hydrostatic pressure, 1925 1980

I

Ice cream cone, density of, 1768, 1817 1822
Improperly constrained, 607
Inclined axes, moments of inertia, 2116, 2118 2120, 2128 2139
formulas, 2116
Internal forces in structural members, 1482 1535

J

Jack, automobile,
square-threaded screw, 1294, 1318 1323
virtual work, 2174, 2209 2213
Journal bearings, 1403 1449
2345

L

Law of cosines, 24
Law of sines, 24

M

Magnitude of a force vector,
two-dimensional, 61
three-dimensional, 114
Mohrs circle, 2116, 2124 2127, 2149 2164
formulas, 2117
Moment diagram
equation form, 1540 1599
constructed by areas, 1601 1672
Moments
two-dimensional, 308 339
three-dimensional, 341 400
couple, 402 443
force about a line, 445 471
Moments of inertia of areas
about inclined axes, 2115, 2117 2120, 2128 2139
by composite parts, 2028 2066
by integration, 1983 2026
Mohrs circle, 2116, 2124 2127, 2149 2164
principal, 2116, 2120 2127, 2136 2164




N

Nutcracker, 796, 823 825

P

Parallelogram law, 24 59
Partially constrained, 607
Piano moving, 1453, 1462 1465
Position vectors
defining, 147 199
use in finding force, 148, 152 158, 167 199
Potential energy, 2271 2337
two degrees of freedom, 2285, 2332 2346
Principal moments of inertia, 2116, 2120 2127, 2136 2164
formulas, 2116
Pulleys
belt friction, 1354, 1356, 1371 1374, 1377 1379
free-body analysis, 798 802, 826 839
virtual work, 2184, 2256 2259
Products of inertia, 2068 2114
Properly constrained, 607

R

Rolling resistance, 1451 1480
Rounding off, 2, 4, 7 11, 16 19




2346
S

Scalar triple product, 445, 452 471
Self-locking
square-threaded screws, 1288
wedges, 1258, 1280 1285
Shear diagram
equation form, 1540 1599
constructed by areas, 1601 1672
Square-threaded screws, 1287 1346
formulas, 1287 1288
Stability, 2271 2337
conditions for a two-degree-of-freedom system, 2272
Static determinacy, 607, 615 617, 671 687
Stonehenge monument, 1457, 1476 1480

T

Theorems of Pappus and Guldinus, 1846 1923
formulas, 1846
Thrust bearings and disks, friction, 1381 1401
formula, 1382
Tripod cooking-stand, 156, 182 186
Trusses
method of joints, 904 959, 1002 1005, 1006 1029
zero-force members, 905, 911 912, 944 959
method of sections, 961 999, 1006, 1030 1034
space, 1002 1034
Turnbuckles, 1295 1296, 1324 1339


U

Units, 3 6, 12 21
Unit vectors, 148, 151, 163 166, 170, 174 175, 178, 181, 186,
191, 197 198
V

Virtual work, 2167 2269
active forces, 2167
used to calculate reactions, 2181 2183, 2243 2256
two degrees of freedom, 2176, 2185, 2219 2223,
2261 2279

W

Wedges, friction, 1251 1285
Wrench (equivalent force-couple system), 489 490, 536 555

Z

Zero-force truss members, 905, 911 912, 944 959

You might also like